You are on page 1of 236

 

CORPORATION  LAW  CASE  DIGESTS  –  ATTY.  DANTE  DELA  CRUZ  

1.  UNION  GLASS  &  CONTAINER  CORPORATION  vs.THE  SECURITIES  AND  EXCHANGE  COMMISSION  
L-­‐‑64013  November  28,  1983  
ESCOLIN,  J.  
 
DOCTRINE:    Section  3  of  PD  No.  902-­‐‑A  confers  upon  the  SEC  "absolute  jurisdiction,  supervision,  and  control  
over  all  corporations,  partnerships  or  associations,  who  are  grantees  of  primary  franchise  and/or  license  or  
permit  issued  by  the  government  to  operate  in  the  Philippines  ...  "    
 
Thus  the  law  explicitly  specified  and  delimited  its  jurisdiction  to  matters  intrinsically  connected  with  the  
regulation  of  corporations,  partnerships  and  associations  and  those  dealing  with  the  internal  affairs  of  such  
corporations,  partnerships  or  associations.  
 
The  fact  that  the  controversy  at  bar  involves  the  rights  of  petitioner  Union  Glass  who  has  no  intra-­‐‑corporate  
relation  either  with  complainant  or  the  DBP,  places  the  suit  beyond  the  jurisdiction  of  the  respondent  SEC.      
 
FACTS:-­‐‑  Private  respondent  Carolina  Hofileña,  complainant  in  SEC  Case  No.  2035,  is  a  stockholder  of  Pioneer  
Glass  Manufacturing  Corporation,  a  domestic  corporation  engaged  in  the  operation  of  silica  mines  and  the  
manufacture  of  glass  and  glassware.    
   
Pioneer   Glass   had   obtained   various   loan   accommodations   from   the   [DBP],   and   also   from   other   local   and  
foreign  sources  which  DBP  guaranteed.;    As  security,  Pioneer  Glass  mortgaged  and/or  assigned  its  assets,  
real  and  personal,  to  the  DBP.    
   
 DBP   was   able   to   gain   control   of   the   outstanding   shares   of   common   stocks   of   Pioneer   Glass,   and   to   get   two,  
later  three,  regular  seats  in  the  corporation's  board  of  directors.  
   
When   Pioneer   Glass   suffered   serious   liquidity   problems   such   that   it   could   no   longer   meet   its   financial  
obligations   with   DBP,   it   entered   into   a   dacion   en   pago   agreement   with   the   latter,   whereby   all   its   assets  
mortgaged  to  DBP  were  ceded  to  the  latter  in  full  satisfaction  of  the  corporation's  obligations  in  the  total  
amount  of  P59M.  ;  Part  of  the  assets  transferred  to  the  DBP  was  the  glass  plant  in  Rosario,  Cavite,  which  DBP  
leased  and  subsequently  sold  to  herein  petitioner  Union  Glass.  
   
Carolina   Hofileña   filed   a   complaint   before   the   respondent   SEC   against   the   DBP,   Union   Glass   and   Pioneer  
Glass,   based   on   the   alleged   illegality   of   the   aforesaid   dacion   en   pago   resulting   from:   [1]   the   self-­‐‑dealing  
indulged  in  by  DBP,  having  acted  both  as  stockholder/director  and  secured  creditor  of  Pioneer  Glass;  and  [2]  
the  wrongful  inclusion  by  DBP  in  its  statement  of  account  of  P26M  as  due  from  Pioneer  Glass  when  the  same  
had  already  been  converted  into  equity.  
   
Hofileña   asked   that   DBP   be   sentenced   to   pay   Pioneer   Glass   actual,   consequential,   moral   and   exemplary  
damages,  for  its  alleged  illegal  acts  and  gross  bad  faith;    
   
Petitioners   moved   for   dismissal   of   the   case   on   the   ground   that   the   SEC   had   no   jurisdiction   over   the   subject  
matter  or  nature  of  the  suit.    
   
SEC:  granted  the  motion  to  dismiss  for  lack  of  jurisdiction.  ;  MR  filed  by  respondent.    
Hearing  Officer:  reversed  and  upheld  the  SEC's  jurisdiction.  ;  The  present  action  is  in  the  form  of  a  derivative  
suit  instituted  by  a  stockholder  for  the  benefit  of  the  corporation,  respondent  Pioneer  Glass  against  another  
stockholder,   respondent   DBP,   for   alleged   illegal   acts   and   gross   bad   faith   which   resulted   in   the   dacion   en  
pago  arrangement  now  being  questioned  by  complainant.    
   

   
3H  A.Y.  2017-­‐2018   1  
 
 

CORPORATION  LAW  CASE  DIGESTS  –  ATTY.  DANTE  DELA  CRUZ  

   
petitioners  filed  the  instant  petition  for  certiorari  and  to  prevent  respondent  SEC  from  taking  cognizance  of  
SEC  Case  No.  2035.  
   
ISSUE:  Is  it  the  regular  court  or  the  SEC  that  has  jurisdiction  over  the  case?  
 
HELD:   -­‐‑   petitioner   Union   Glass,   as   transferee   and   possessor   of   the   glass   plant   covered   by   the   dacion   en  
pago  agreement,  should  be  joined  as  party-­‐‑defendant  under  the  general  rule  which  requires  the  joinder  of  
every  party  who  has  an  interest  in  or  lien  on  the  property  subject  matter  of  the  dispute.  .  
-­‐‑  But  since  petitioner  Union  Glass  has  no  intra-­‐‑corporate  relation  with  either  the  complainant  or  the  DBP,  its  
joinder  as  party-­‐‑defendant  in  SEC  Case  No.  2035  brings  the  cause  of  action  asserted  against  it  outside  the  
jurisdiction  of  the  respondent  SEC.  
   
The  jurisdiction  of  the  SEC  is  delineated  by  PD  No.  902-­‐‑A  ,  sec.5  
 
-­‐‑   This   grant   of   jurisdiction   must   be   viewed   in   the   light   of   the   nature   and   function   of   the   SEC   under   the   law.  
Section  3  of  PD  No.  902-­‐‑A  confers  upon  the  latter  "absolute  jurisdiction,  supervision,  and  control  over  all  
corporations,  partnerships  or  associations,  who  are  grantees  of  primary  franchise  and/or  license  or  permit  
issued  by  the  government  to  operate  in  the  Philippines  ...  "    
 
-­‐‑   The   principal   function   of   the   SEC   is   the   supervision   and   control   over   corporations,   partnerships   and  
associations  with  the  end  in  view  that  investment  in  these  entities  may  be  encouraged  and  protected,  and  
their  activities  pursued  for  the  promotion  of  economic  development.  5  
 
-­‐‑   Thus  the  law  explicitly  specified  and  delimited  its  jurisdiction  to  matters  intrinsically  connected  with  the  
regulation  of  corporations,  partnerships  and  associations  and  those  dealing  with  the  internal  affairs  of  such  
corporations,  partnerships  or  associations.  
 
Otherwise  stated,  in  order  that  the  SEC  can  take  cognizance  of  a  case,  the  controversy  must  pertain  to  any  of  
the   following   relationships:   [a]   between   the   corporation,   partnership   or   association   and   the   public;   [b]  
between  the  corporation,  partnership  or  association  and  its  stockholders,  partners,  members,  or  officers;  [c]  
between  the  corporation,  partnership  or  association  and  the  state  in  so  far  as  its  franchise,  permit  or  license  
to  operate  is  concerned;  and  [d]  among  the  stockholders,  partners  or  associates  themselves.  
 
The  fact  that  the  controversy  at  bar  involves  the  rights  of  petitioner  Union  Glass  who  has  no  intra-­‐‑corporate  
relation  either  with  complainant  or  the  DBP,  places  the  suit  beyond  the  jurisdiction  of  the  respondent  SEC.    
 
-­‐‑  The  case  should  be  tried  and  decided  by  the  court  of  general  jurisdiction,  the  Regional  Trial  Court.  This  view  
is  in  accord  with  the  rudimentary  principle  that  administrative  agencies,  like  the  SEC,  are  tribunals  of  limited  
jurisdiction  6  and,  as  such,  could  wield  only  such  powers  as  are  specifically  granted  to  them  by  their  enabling  
statutes.      
 
-­‐‑   Since   petitioner   has   no   intra-­‐‑corporate   relationship   with   the   complainant,   it   cannot   be   joined   as   party-­‐‑
defendant  in  said  case  as  to  do  so  would  violate  the  rule  or  jurisdiction.  
-­‐‑   Hofileñas   complaint   against   petitioner   for   cancellation   of   the   sale   of   the   glass   plant   should   therefore   be  
brought  separately  before  the  regular  court  ;  But  such  action,  if  instituted,  shall  be  suspended  to  await  the  
final   outcome   of   SEC   Case   No.   2035,   for   the   issue   of   the   validity   of   the   dacion   en   pago   posed   in   the   last  
mentioned  case  is  a  prejudicial  question,  the  resolution  of  which  is  a  logical  antecedent  of  the  issue  involved  
in  the  action  against  petitioner  Union  Glass.    
 

   
3H  A.Y.  2017-­‐2018   2  
 
 

CORPORATION  LAW  CASE  DIGESTS  –  ATTY.  DANTE  DELA  CRUZ  

 
2.  SPOUSES  JOSE  AND  AURORA  ABEJO,  ET.AL  VS.  HON.  RAFAEL  DELA  CRUZ  
GR  NO.  L-­‐‑63558  19  MAY  1987  
CJ  TEEHANKEE  
 
DOCTRINE:  
 
  An  intracorporate  controversy  is  one  which  arises  between  the  stockholders  and  the  corporation;  it  
is   broad   enough   to   cover   all   kinds   of   controversies   between   the   said   parties.   Thus,   jurisdiction   over  
intracorporate  controversies  fall  within  the  exlusive  and  original  jurisdiction  of  the  SEC  as  provided  in  PD  
902-­‐‑A  Section  5.    
 
FACTS:  
 
  Spouses  Jose  Abejo  and  Aurora  Abejo  are  the  principal  stockholders  of  Pocket  Bell  Philippines,  Inc.,  a  
tone   and   voice   paging   corporation.   Telectronic   Systems,   Inc.   purchased   Abejo’   133,000   minority  
shareholdings  and  63,000  shares  of  Virginia  Braga.  Due  to  the  said  purchases,  Telectronic  would  become  the  
majority  stockholder,  holding  56%  of  the  outstanding  stock  and  voting  power  of  Pocket  Bell.    
 
  Telectronics  requested  the  corporate  secretary  of  Pocket  Bell  to  register  and  transfer  to  its  name  in  
the  corporation’s  transfer  book,  cancel  the  surrendered  certificates  of  stock  and  issue  the  corresponding  new  
certificates  of  stock  in  its  name.  However,  Norberto  Braga,  the  corporate  secretary  and  son  of  the  Virginia  
Braga   refused   to   register   the   transfer   of   shares   asserting   that   Braga   claim   pre-­‐‑emptive   rights   over   the  
133,000  Abejo  shares  and  that  the  latter  never  transferred  her  63,000  shares  to  Telectronics  but  had  lost  the  
five  stock  certificates  representing  those  shares.    
 
  This  started  the  series  of  actions  between  the  parties,  all  centerd  on  the  question  of  jurisdiction  over  
the  dispute.  The  Bragas  assert  that  the  regular  civil  courts  have  original  and  exclusive  jurisdiction  as  against  
the  SEC,  while  the  Abejos  claim  to  the  contrary.    
 
ISSUE:  
 
  Whether  the  SEC  or  the  regular  trial  courts  have  exclusive  jurisdiction  over  disputes  between  the  
stockholders  of  a  corporation  
 
RULING:  
 
  The   Supreme   Court   ruled   that   it   is   the   SEC   which   has   original   and   exclusive   jurisdiction   over   the  
dispute   between   the   principal   stockholders   of   the   corporation.   The   SEC’s   jurisdiction   is   premised   on   and  
fully   supported   by   the   applicable   provisions   of   PD   No.   902-­‐‑A   which   reorganized   the   SEC   with   additional  
powers,  specifically:  
 
“SEC.   5.   In   addition   to   the   regulatory   and   adjudicative   functions   of   the   SEC   over   coproations,  
partnerships   and   other   forms   of   associations   registered   with   it   as   expressly   granted   under  
existing   laws   and   decrees,   it   shall   have   original   and   exclusive   jurisdiction   to   hear   and   decide  
cases  involving:  
 
(b)   Controversies   arising   out   of   intracorporate   or   partnership   relations,   between   and   among  
stockholders,   members   or   associates;   between   any   and/or   all   of   them   and   the   corporation,  
partnership  or  association  of  which  they  are  stockholders,  members  or  associates,  respectively;  

   
3H  A.Y.  2017-­‐2018   3  
 
 

CORPORATION  LAW  CASE  DIGESTS  –  ATTY.  DANTE  DELA  CRUZ  

and  between  such  corporation,  partnership  or  association  and  the  state  insofar  as  it  concerns  
their  individual  franchise  or  right  to  exist  as  such  entity;  xxx”  
 
  The   dispute   at   bar   is   an   intracorporate   dispute   that   has   arisen   between   and   among   the   principal  
stockholders  of  the  corporation  due  to  the  refusal  of  the  corporate  secretary  to  perform  his  “ministerial  duty”  
to  record  the  transfers  of  the  corporation’s  controlling  shares  of  stock,  covered  by  duly  endorsed  certificates  
of  stock,  in  favor  of  Telectronics  as  the  purchaser  thereof.  Mandamus  in  the  SEC  to  compel  the  corporate  
secretary   to   register   the   transfers   and   issue   the   new   certificates   in   favor   of   Telectronics   was   properly  
resorted  to.    
 
  An  intracorporate  controversy  is  one  which  arises  between  a  stockholder  and  the  corporation  There  
is  no  distinction,  qualification,  nor  any  exemption  whatsoever.  The  provision  is  broad  and  covers  all  kinds  of  
controversies  between  stockholders  and  corporations.    
 
  Thus,   the   dispute   between   the   contending   parties   for   control   of   the   corporation   manifestly   fans  
within   the   primary   and   exclusive   jurisdiction   of   the   SEC   in   whom   the   law   has   reserved   such   jurisdiction   as  
an  administrative  agency  of  special  competence  to  deal  promptly  and  expeditiously.    
 
 
3.  ALMA  MAGALAD  VS  PRIEMIER  FINANCING  CORP.  
G.R.  NO.  87135  MAY  22,  1992  
PARAS  J.  
   
DOCTRINE:  The  fact  that  Premiere's  authority  to  engage  in  financing  already  expired  will  not  have  the  effect  
of   divesting   the   SEC   of   its   original   and   exclusive   jurisdiction.   The   expanded   jurisdiction   of   the   SEC   was  
conceived  primarily  to  protect  the  interest  of  the  investing  public.  
 
FACTS:    
Premiere  is  a  financing  company  engaged  in  soliciting  and  accepting  money  market  placements  or  deposits.  
On  September  12,  1983  with  expired  permit  to  issue  commercial  papers,  Premiere  induced  and  misled  Alma  
Magalad  into  making  a  money  market  placement  of  P50,000.00  at  22%  interest  per  annum  for  which  it  issued  
a  receipt  and  issued  two  (2)  post-­‐‑dated  checks  in  the  total  sum  of  P51,079.00  and  assigned  to  Magalad  its  
receivable   from   a   certain   David   Saman   for   the   same   amount.   When   the   said   checks   were   presented   for  
payment  on  their  due  dates,  the  drawee  bank  dishonored  the  checks  for  lack  of  sufficient  funds  to  cover  the    
 
On  January  10,  1984,  Magalad  filed  a  complaint  for  damages  with  prayer  for  writ  of  preliminary  attachment  
with  the  RTC,  Quezon  City  Premiere.  RTC  rendered  judgment  in  favor  of  Magalad.  
Premiere  filed  a  motion  for  reconsideration  of  the  foregoing  decision,  based  principally  on  a  question  of  law  
alleging  that  the  Securities  and  Exchange  Commission  (SEC)  has  exclusive  and  original  jurisdiction  over  a  
corporation  under  a  state  of  suspension  of  payments.  
Magalad  submits  that  the  legal  suit  which  she  has  brought  against  Premiere  is  an  ordinary  action  for  damages  
with  the  preliminary  attachment  cognizable  solely  by  the  RTC.  Premiere,  on  the  other  hand,  espouses  the  
original  and  exclusive  jurisdiction  of  the  Securities  and  Exchange  Commission.  
 
ISSUE:  Whether  or  not  the  SEC  has  exclusive  and  original  jurisdiction  over  the  instant  case.  
HELD:  YES.  
Considering   that   Magalad's   complaint   sufficiently   alleges   acts   amounting   to   fraud   and   misrepresentation  
committed   by   Premiere,   the   SEC   must   be   held   to   retain   its   original   and   exclusive   jurisdiction   over   the   case,  
despite  the  fact  that  the  suit  involves  collection  of  sums  of  money  paid  to  said  corporation,  the  recovery  of  

   
3H  A.Y.  2017-­‐2018   4  
 
 

CORPORATION  LAW  CASE  DIGESTS  –  ATTY.  DANTE  DELA  CRUZ  

which   would   ordinarily   fall   within   the   jurisdiction   of   regular   courts.   The   fraud   committed   is   detrimental   to  
the  interest  of  the  public  and,  therefore,  encompasses  a  category  of  relationship  within  the  SEC  jurisdiction.  
In  this  case,  the  recitals  of  the  complaint  sufficiently  allege  that  devices  or  schemes  amounting  to  fraud  and  
misrepresentation  detrimental  to  the  interest  of  the  public  have  been  resorted  to  by  Premiere  Corporation.  
It  can  not  but  be  conceded,  therefore,  that  the  SEC  may  exercise  its  adjudicative  powers  pursuant  to  Sec.  5(a)  
of  Pres.  Decree  No.  902-­‐‑A  (Supra).  
The  fact  that  Premiere's  authority  to  engage  in  financing  already  expired  will  not  have  the  effect  of  divesting  
the   SEC   of   its   original   and   exclusive   jurisdiction.   The   expanded   jurisdiction   of   the   SEC   was   conceived  
primarily  to  protect  the  interest  of  the  investing  public.  That  Magalad's  money  placements  were  in  the  nature  
of  investments  in  Premiere  can  not  be  gainsaid.  Magalad  had  reasonably  expected  to  receive  returns  from  
moneys   she   had   paid   to   Premiere.   Unfortunately,   however,   she   was   the   victim   of   alleged   fraud   and  
misrepresentation.  
 
 
 
4.   SECURITIES   AND   EXCHANGE   COMMISSION   v.   SUBIC   BAY   GOLF   AND   COUNTRY   CLUB,   INC.   AND  
UNIVERSAL  INTERNATIONAL  GROUP  DEVELOPMENT  CORPORATION  
G.R.  No.  179047,  March  11,  2015  
LEONEN,  J.:  
 
DOCTRINE:   Intra-­‐‑corporate   controversies,   previously   under   the   SEC’s   jurisdiction,   are   now   under   the  
jurisdiction  of  the  Regional  Trial  Courts.  However  it  does  not  necessarily  oust  the  SEC’s  of  its  regulatory  and  
administrative  jurisdiction  to  determine  and  act  if  there  were  administrative  violations  committed.  
 
FACTS:    
Subic   Bay   Golf   and   Country   Club   (SBGCCI)   and   Universal   International   Group   Development   Corporation  
(UIGDC)  entered  into  a  Lease  and  Development  Agreement.  Under  the  agreement,  SBMA  agreed  to  lease  the  
golf  course  to  UIGDC  who  agreed  to  "develop,  manage  and  maintain  the  golf  course  and  other  related  facilities  
within  the  complex.  Complainants  Filart  and  Villareal  informed  the  SEC  that  they  had  been  asking  UIGDC  for  
the   refund   of   their   payment   for   their   SBGCCI   shares.   UIGDC   did   not   act   on   their   requests.   They   alleged   that  
they   purchased   the   shares   based   on   the   promise   of   SBGCCI   and   UIGDC   to   deliver   an   18   hole   golf   course,  
swimming   pool,   among   others.   However,   these   promises   were   not   delivered   and   despite   SBGCCI's   and  
UIGDC's   failure   to   deliver   the   promised   amenities,   they   started   to   charge   them   monthly   dues   and   were  
threatened  that  their  shares  will  be  paid  off  and  would  be  auctioned  off  if  their  alleged  back  dues  would  not  
be  paid.  SEC  conducted  an  inspection  and  found  that  SBGCCI  and  UIGDC  failed  to  substantially  comply  with  
their  commitment  to  complete  the  project  hence  SEC  ordered  the  return  of  the  purchase  price  of  the  shares.  
SBGCC  and  UIGDC,  in  a  petition  for  review  questioned  the  order  and  jurisdiction  of  the  SEC  since  the  same  
involved  intra-­‐‑corporate  dispute.  
The  Court  of  Appeals  found  that  the  case  involved  an  intra-­‐‑corporate  controversy.  The  SEC  acted  in  excess  of  
its  jurisdiction  when  it  ordered  UIGDC  and  SBGCCI  to  refund  Villareal  and  Filart  the  amount  they  paid  for  
SBGCCI  shares  of  stock.  Such  power  is  transferred  to  the  RTC.  
 
ISSUE:  
Whether  or  not  the  SEC  has  jurisdiction  over  the  case  involving  intra-­‐‑corporate  dispute.  
HELD:  
SEC   has   no   jurisdiction.   It   is   the   RTC   who   has   jurisdiction   over   the   case   at   bar.   Jurisdiction   over   intra-­‐‑
corporate  disputes  and  all  other  cases  enumerated  in  Section  5  of  Presidential  Decree  No.  902-­‐‑A  had  already  
been   transferred   to   designated   Regional   Trial   Courts.   Hence,   actions   pertaining   to   intra-­‐‑corporate   disputes  
should   be   filed   directly   before   designated   RTCs.   Intra-­‐‑corporate   disputes   brought   before   other   courts   or  

   
3H  A.Y.  2017-­‐2018   5  
 
 

CORPORATION  LAW  CASE  DIGESTS  –  ATTY.  DANTE  DELA  CRUZ  

tribunals   are   dismissible   for   lack   of   jurisdiction.   For   a   dispute   to   be   "intra-­‐‑corporate,"   it   must   satisfy   the  
relationship  and  nature  of  controversy  tests.  
This   case   is   an   intra-­‐‑corporate   dispute.   It   involves   a   dispute   between   the   corporation,   SBGCCI,   and   its  
shareholders,  Villareal  and  Filart.    It  also  involves  corporate  rights  and  obligations.  Villareal  and  Filart's  right  
to  a  refund  of  the  value  of  their  shares  was  based  on  SBGCCI  and  UIGDC's  alleged  failure  to  abide  by  their  
representations   when   they   purchased   shares.   This   involves   the   determination   of   a   shareholder's   rights  
under  the  Corporation  Code  or  other  intra-­‐‑corporate  rules  when  the  corporation  or  association  fails  to  fulfill  
its  obligations.  However,  even  though  the  Complaint  filed  before  the  SEC  contains  allegations  that  are  intra-­‐‑
corporate   in   nature,   it   does   not   necessarily   oust   the   SEC   of   its   regulatory   and   administrative   jurisdiction   to  
determine   and   act   if   there   were   administrative   violations   committed.   SEC’s   regulatory   power   does   not  
include  the  authority  to  order  the  refund  of  the  purchase  price  of  Villareal's  and  Filart's  shares  in  the  golf  
club.  The  issue  of  refund  is  intra-­‐‑corporate  or  civil  in  nature.  
 
 
 
5.    PHILCOMSAT  V.  SANDIGANBAYAN  
G.R.  NO.  203023,  JUNE  17,  2015  
CARPIO,  J.  
 
DOCTRINE:  A  combined  application  of  the  relationship  test  and  the  nature  of  the  controversy  test  has  become  
the   norm   in   determining   whether   a   case   is   an   intra-­‐‑corporate   controversy   to   be   “heard   and   decided   by   the  
[b]ranches  of  the  RTC  specifically  designed  by  the  Court  to  try  and  decide  such  cases.”  
 
FACTS:    
PHILCOMSAT  Holdings  Corp.  (PHC),  previously  known  as  Liberty  Mines,  Inc.  (LMI),  is  a  domestic  corporation  
listed   in   the   Philippine   Stock   Exchange   (PSE).   In   Sept.   1995,   LMI’s   then   Chairman   and   President   Oliverio  
Laperal   (Laperal)   and   PHILCOMSAT’s   then   President   Honorio   Poblador   III   (Poblador),   signed   a  
Memorandum  of  Agreement  (MOA)  for  the  latter  to  gain  controlling  interest  in  LMI  through  an  increase  in  
its   authorized   capital   stock.   In   June   1996,   Laperal   and   PHILCOMSAT   executed   a   Supplemental   MOA  
reiterating  the  increase  in  capital  stock  of  LMI,  to  which  PHILCOMSAT  subscribed  to  P79,050,000,000  shares.  
Sometime  in  1997,  LMI  changed  its  name  to  PHC.  
 
PHC  then  filed  its  application  with  the  PSE  for  listing  the  shares  representing  the  increase  in  its  capital  stock.  
Included  in  this  application  were  the  PHC  shares  owned  by  PHILCOMSAT.  
 
Pending   the   PSE’s   final   approval   of   PHC’s   application   for   listing   of   the   shares,   the   PCGG,   through   its   then  
Chairman  Camilio  Sabio  (Chairman  Sabio),  made  a  written  request  to  suspend  the  listing  of  the  increase  in  
PHC’s   capital   stock   citing   as   reason   the   need   to   settle   the   conflicting   claims   of   the   two   sets   of   board   of  
directors  of  the  Philippine  Overseas  Telecommunication  Corp.  (POTC)  and  PHILCOMSAT.  
 
In   November   2007,   then   Pres.   Gloria   Arroyo   appointed   new   government   nominees   to   the   POTC   and  
PHILCOMSAT  boards.  In  May  2008,  the  PCGG  issued  an  En  Banc  Resolution  recognizing  the  validity  of  the  
POTC’s  and  PHILCOMSAT’s  respective  stockholders’  meetings  and  elections.  
 
In  a  letter  dated  July  2011,  POTC’s  then  President  Katrina  Ponce-­‐‑Enrile  (Ponce-­‐‑Enrile)  wrote  to  the  PCGG  
demanding   that   the   PCGG   rescind   its   objection   to   the   listing   of   the   increase   in   PHC’s   capital   stock,   to   which  
the   PCGG   did   not   respond.   On   Feb.   2012,   PHILCOMSAT   filed   a   complaint   before   the   Sandiganbayan   against  
PCGG   to   compel   the   latter   to   withdraw   its   opposition   to   the   listing   of   the   increase   in   PHC’s   capital   stock.  
PHILCOMSAT  argued  that  the  PCGG  had  already  recognized  the  validity  of  the  stockholders’  meetings  in  the  

   
3H  A.Y.  2017-­‐2018   6  
 
 

CORPORATION  LAW  CASE  DIGESTS  –  ATTY.  DANTE  DELA  CRUZ  

two  corporations,  which  “practically  erased”  the  alleged  conflict  between  the  two  sets  of  directors.  The  PCGG  
filed  a  motion  to  dismiss  the  complaint.  
 
In  its  assailed  ruling  which  dismissed  the  complaint,  the  Sandiganbayan  held  that,  based  on  the  allegations,  
the  action  was  one  for  specific  performance,  since  it  sought  to  have  PCGG  withdraw  its  objection  to  the  listing  
of  the  increase  in  PHC’s  capital  stock  at  the  PSE.  Following  Sec.  19  of  BP  129,  as  amended  by  RA  7691,  the  
RTC   has   exclusive   jurisdiction   over   the   case.   The   Sandiganbayan   also   ruled   that   the   case   was   a   “dispute  
among  its  directors,”  and  thus,  was  an  intra-­‐‑corporate  dispute.  
 
ISSUE:  WON  the  matter  involves  an  intra-­‐‑corporate  controversy.  
 
HELD:  YES,  the  complaint  involves  an  intra-­‐‑corporate  controversy.  
 
To   determine   if   a   case   involves   an   intra-­‐‑corporate   controversy,   the   courts   have   applied   two   tests:   the  
relationship  test  and  the  nature  of  the  controversy  test.  
 
Under   the   relationship   test,   the   existence   of   any   of   the   following   relationships   makes   the   conflict   intra-­‐‑
corporate:   (1)   between   the   corporation,   partnership   or   association   and   the   public;   (2)   between   the  
corporation,  partnership  or  association  and  the  State  insofar  as  its  franchise,  permit  or  license  to  operate  is  
concerned;  (3)    between  the  corporation,  partnership  or  association  and  its  stockholders,  partners,  members  
or  officers;  and  (4)    among  the  stockholders,  partners  or  associates  themselves.  
 
On  the  other  hand,  the  nature  of  the  controversy  test  dictates  that  “the  controversy  must  not  only  be  rooted  
in  the  existence  of  an  intra-­‐‑corporate  relationship,  but  must  as  well  pertain  to  the  enforcement  of  the  parties’  
correlative   rights   and   obligations   under   the   Corporation   Code   and   the   internal   and   intra-­‐‑corporate  
regulatory  rules  of  the  corporation.  
 
A  combined  application  of  the  relationship  test  and  the  nature  of  the  controversy  test  has  become  the  norm  
in  determining  whether  a  case  is  an  intra-­‐‑corporate  controversy  to  be  “heard  and  decided  by  the  [b]ranches  
of  the  RTC  specifically  designed  by  the  Court  to  try  and  decide  such  cases.”  
 
Relationship  test  
 
Petitioners  insist  that  the  PCGG  is  not  a  stockholder,  partner,  member  or  officer  of  the  corporation.  This  is  
misleading  and  inaccurate.  
 
As  it  stands  today,  the  Republic  of  the  Philippines  owns  34.0%  of  POTC,  which  wholly  owns  PHILCOMSAT,  
which  in  turn  owns  81%  of  PHC.  The  Republic,  then,  has  an  interest  in  the  proper  operations  of  the  PHC,  
however  indirect  this  interest  may  seem  to  be.  
 
Nature  of  the  controversy  test  
 
The  controversy  in  the  present  case  stems  from  the  act  of  Chairman  Sabio  in  requesting  the  PSE  to  suspend  
the   listing   of   PHC’s   increase   in   capital   stock   because   of   still   unresolved   issues   on   the   election   of   the   POTC’s  
and  PHILCOMSAT’s  respective  board  of  directors.  
 
The  act  of  Chairman  Sabio  in  asking  the  SEC  to  suspend  the  listing  of  PHC’s  shares  was  done  in  pursuit  of  
protecting  the  interest  of  the  Republic  of  the  Philippines,  a  legitimate  stockholder  in  PHC’s  controlling  parent  
company,   POTC.   Xxx   It   was   an   act   that   had   no   relation   to   any   proceeding   or   question   of   ill-­‐‑gotten   wealth   or  
sequestration.  The  PCGG  was  merely  protecting  the  rights  and  interest  of  the  Republic  of  the  Philippines.  

   
3H  A.Y.  2017-­‐2018   7  
 
 

CORPORATION  LAW  CASE  DIGESTS  –  ATTY.  DANTE  DELA  CRUZ  

 
From  the  foregoing,  it  is  clear  that  the  dispute  in  the  present  case  is  an  intra-­‐‑corporate  controversy.  
As  such,  it  is  clear  that  the  jurisdiction  lies  with  the  regular  courts  and  not  with  the  Sandiganbayan.  
 
 
 
6.  THE  COLLECTOR  OF  INTERNAL  REVENUE  vs.  THE  CLUB  FILIPINO,  INC.  DE  CEBU  
G.R.  No.  L-­‐‑12719  -­‐‑  May  31,  1962  
PAREDES,  J.:  
 
DOCTRINE:  What  is  determinative  of  whether  or  not  the  Club  is  engaged  in  such  business  is  its  object  or  
purpose,   as   stated   in   its   articles   and   by-­‐‑laws.   For   a   stock   corporation   to   exist,   two   requisites   must   be  
complied  with,  to  wit:  (1)  a  capital  stock  divided  into  shares  and  (2)  an  authority  to  distribute  to  the  holders  
of  such  shares,  dividends  or  allotments  of  the  surplus  profits  on  the  basis  of  the  shares  held.  
 
FACTS:    
As  found  by  the  Court  of  Tax  Appeals,  the  "Club  Filipino,  Inc.  de  Cebu,"  (Club,  for  short),  is  a  civic  corporation  
organized  under  the  laws  of  the  Philippines  with  an  original  authorized  capital  stock  of  P22,000.00,  which  
was  subsequently  increased  to  P200,000.00,  among  others,  to  it  "proporcionar,  operar,  y  mantener  un  campo  
de  golf,  tenis,  gimnesio  (gymnasiums),  juego  de  bolos  (bowling  alleys),  mesas  de  billar  y  pool,  y  toda  clase  de  
juegos  no  prohibidos  por  leyes  generales  y  ordenanzas  generales;  y  desarollar  y  cultivar  deportes  de  toda  
clase   y   denominacion   cualquiera   para   el   recreo   y   entrenamiento   saludable   de   sus   miembros   y   accionistas".  
Neither  in  the  articles  or  by-­‐‑laws  is  there  a  provision  relative  to  dividends  and  their  distribution,  although  it  
is  covenanted  that  upon  its  dissolution,  the  Club's  remaining  assets,  after  paying  debts,  shall  be  donated  to  a  
charitable  Philippine  Institution  in  Cebu.  
 
The  Club  owns  and  operates  a  club  house,  a  bowling  alley,  a  golf  course  (on  a  lot  leased  from  the  government),  
and  a  bar-­‐‑restaurant  where  it  sells  wines  and  liquors,  soft  drinks,  meals  and  short  orders  to  its  members  and  
their  guests.  The  bar-­‐‑restaurant  was  a  necessary  incident  to  the  operation  of  the  club  and  its  golf-­‐‑course.  The  
club  is  operated  mainly  with  funds  derived  from  membership  fees  and  dues.  Whatever  profits  it  had,  were  
used  to  defray  its  overhead  expenses  and  to  improve  its  golf-­‐‑course.  In  1951.  as  a  result  of  a  capital  surplus,  
arising   from   the   re-­‐‑valuation   of   its   real   properties,   the   value   or   price   of   which   increased,   the   Club   declared  
stock   dividends;   but   no   actual   cash   dividends   were   distributed   to   the   stockholders.   In   1952,   a   BIR   agent  
discovered   that   the   Club   has   never   paid   percentage   tax   on   the   gross   receipts   of   its   bar   and   restaurant,  
although  it  secured  B-­‐‑4,  B-­‐‑9(a)  and  B-­‐‑7  licenses.  In  a  letter  dated  December  22,  1852,  the  Collector  of  Internal  
Revenue   assessed   against   and   demanded   from   the   Club   the   sum   of   12,068.84   as   fixed   percentage   tax,  
surcharges  and  compromise  penalty.  
 
The   Club   wrote   the   Collector,   requesting   for   the   cancellation   of   the   assessment.   The   request   having   been  
denied,  the  Club  filed  the  instant  petition  for  review.  
 
ISSUE:  
Whether  or  not  the  club  is  a  stock  corporation.  
 
HELD:  
 
The  fact  that  the  capital  stock  of  the  respondent  Club  is  divided  into  shares,  does  not  detract  from  the  finding  
of  the  trial  court  that  it  is  not  engaged  in  the  business  of  operator  of  bar  and  restaurant.  What  is  determinative  
of  whether  or  not  the  Club  is  engaged  in  such  business  is  its  object  or  purpose,  as  stated  in  its  articles  and  
by-­‐‑laws.   It   is   a   familiar   rule   that   the   actual   purpose   is   not   controlled   by   the   corporate   form   or   by   the  

   
3H  A.Y.  2017-­‐2018   8  
 
 

CORPORATION  LAW  CASE  DIGESTS  –  ATTY.  DANTE  DELA  CRUZ  

commercial  aspect  of  the  business  prosecuted,  but  may  be  shown  by  extrinsic  evidence,  including  the  by-­‐‑
laws  and  the  method  of  operation.  From  the  extrinsic  evidence  adduced,  the  Tax  Court  concluded  that  the  
Club  is  not  engaged  in  the  business  as  a  barkeeper  and  restaurateur.  
 
Moreover,   for   a   stock   corporation   to   exist,   two   requisites   must   be   complied   with,   to   wit:   (1)   a   capital   stock  
divided  into  shares  and  (2)  an  authority  to  distribute  to  the  holders  of  such  shares,  dividends  or  allotments  
of  the  surplus  profits  on  the  basis  of  the  shares  held  (sec.  3,  Act  No.  1459).  In  the  case  at  bar,  nowhere  in  its  
articles   of   incorporation   or   by-­‐‑laws   could   be   found   an   authority   for   the   distribution   of   its   dividends   or  
surplus   profits.   Strictly   speaking,   it   cannot,   therefore,   be   considered   a   stock   corporation,   within   the  
contemplation  of  the  corporation  law.  
 
A   tax   is   a   burden,   and,   as   such,   it   should   not   be   deemed   imposed   upon   fraternal,   civic,   non-­‐‑profit,   nonstock  
organizations,  unless  the  intent  to  the  contrary  is  manifest  and  patent"  (Collector  v.  BPOE  Elks  Club,  et  al.,  
supra),  which  is  not  the  case  in  the  present  appeal.  
 
Having  arrived  at  the  conclusion  that  respondent  Club  is  not  engaged  in  the  business  as  an  operator  of  a  bar  
and  restaurant,  and  therefore,  not  liable  for  fixed  and  percentage  taxes,  it  follows  that  it  is  not  liable  for  any  
penalty,  much  less  of  a  compromise  penalty.  
 
WHEREFORE,  the  decision  appealed  from  is  affirmed  without  costs.  
 
 
 
7.  DULAY  V.  COURT  OF  APPEALS  
G.R.  NO.  91889.  AUGUST  27,  1993  
NOCON,  J.;  
 
Doctrine:  A  corporate  action  taken  at  a  board  meeting  without  proper  call  or  notice  in  a  close  corporation  is  
deemed   ratified   by   the   absent   director   unless   the   latter   promptly   files   his   written   objection   with   the  
secretary  of  the  corporation  after  having  knowledge  of  the  meeting.  
 
Facts:  
Dulay  Enterprises,  through  its  president,  Manuel  Dulay,  obtained  various  loans  for  the  construction  of  its  
hotel   project,   Dulay   Continental   Hotel   (now   Frederick   Hotel).   It   even   had   to   borrow   money   from   Virgilio  
Dulay  to  be  able  to  continue  the  hotel  project.  As  a  result  of  said  loan,  Virgilio  Dulay  occupied  one  of  the  unit  
apartments  of  the  subject  property  since  1973  while  at  the  same  time  managing  the  Dulay  Apartment  as  his  
shareholdings  in  the  corporation  was  subsequently  increased  by  his  father.  
 
On  December  23,  1976,  Manuel  Dulay  by  virtue  of  Board  Resolution  No.  18  of  the  corporation  sold  the  subject  
property  to  private  respondents,  spouses  Maria  Theresa  and  Castrense  Veloso,  in  the  amount  of  P300,000.00.  
Manuel  Dulay  and  spouses  Veloso  executed  a  Memorandum  to  the  Deed  of  Absolute  Sale  giving  Manuel  Dulay  
within  two  (2)  years  or  until  December  9,  1979  to  repurchase  the  subject  property  for  P200,000.00  which  
was,  however,  not  annotated.  
 
On   December   24,   1976,   private   respondent   Maria   Veloso,   without   the   knowledge   of   Manuel   Dulay,  
mortgaged  the  subject  property  to  private  respondent  Manuel  A.  Torres  for  a  loan  of  P250,000.00  which  was  
duly  annotated  as  Entry  No.  68139  in  TCT  No.  23225.  
 
Upon  the  failure  of  Maria  Veloso  to  pay  private  respondent  Torres,  the  subject  property  was  sold  to  private  
respondent  Torres  as  the  highest  bidder  in  an  extrajudicial  foreclosure  sale.  Maria  Veloso  executed  a  Deed  

   
3H  A.Y.  2017-­‐2018   9  
 
 

CORPORATION  LAW  CASE  DIGESTS  –  ATTY.  DANTE  DELA  CRUZ  

of  Absolute  Assignment  of  the  Right  to  Redeem  in  favor  of  Manuel  Dulay  assigning  her  right  to  repurchase  
the  subject  property  from  private  respondent  Torres  as  a  result  of  the  extrajudicial  sale.  
 
As  neither  Maria  Veloso  nor  her  assignee  Manuel  Dulay  was  able  to  redeem  the  subject  property  within  the  
one  year  statutory  period  for  redemption,  private  respondent  Torres  filed  an  Affidavit  of  Consolidation  of  
Ownership  with  the  Registry  of  Deeds  of  Pasay  City  and  TCT  No.  24799was  subsequently  issued  to  private  
respondent  Manuel  Torres.  
 
Petitioners  contend  that  the  respondent  court  had  acted  with  grave  abuse  of  discretion  when  it  applied  the  
doctrine  of  piercing  the  veil  of  corporate  entity  in  the  instant  case  considering  that  the  sale  of  the  subject  
property   between   Veloso   and   Manuel   Dulay   has   no   binding   effect   on   petitioner   corporation   as   Board  
Resolution  No.  18  which  authorized  the  sale  of  the  subject  property  was  resolved  without  the  approval  of  all  
the  members  of  the  board  of  directors  and  said  Board  Resolution  was  prepared  by  a  person  not  designated  
by  the  corporation  to  be  its  secretary.  
 
Issue:  Whether  the  sale  by  the  corporation  was  valid.  (YES)  
 
Ratio:  
In   the   instant   case,   petitioner   corporation   is   classified   as   a   close   corporation   and   consequently   a   board  
resolution  authorizing  the  sale  or  mortgage  of  the  subject  property  is  not  necessary  to  bind  the  corporation  
for  the  action  of  its  president.  At  any  rate,  a  corporate  action  taken  at  a  board  meeting  without  proper  call  or  
notice  in  a  close  corporation  is  deemed  ratified  by  the  absent  director  unless  the  latter  promptly  files  his  
written  objection  with  the  secretary  of  the  corporation  after  having  knowledge  of  the  meeting  which,  in  this  
case,  petitioner  Virgilio  Dulay  failed  to  do.  
 
It  is  relevant  to  note  that  although  a  corporation  is  an  entity  which  has  a  personality  distinct  and  separate  
from  its  individual  stockholders  or  members,  the  veil  of  corporate  fiction  may  be  pierced  when  it  is  used  to  
defeat  public  convenience,  justify  wrong,  protect  fraud  or  defend  crime.  The  privilege  of  being  treated  as  an  
entity  distinct  and  separate  from  its  stockholders  or  members  is  therefore  confined  to  its  legitimate  uses  and  
is  subject  to  certain  limitations  to  prevent  the  commission  of  fraud  or  other  illegal  or  unfair  act.  When  the  
corporation  is  used  merely  as  an  alter  ego  or  business  conduit  of  a  person,  the  law  will  regard  the  corporation  
as   the   act   of   that   person.   The   Supreme   Court   had   repeatedly   disregarded   the   separate   personality   of   the  
corporation  where  the  corporate  entity  was  used  to  annul  a  valid  contract  executed  by  one  of  its  members.  
 
Petitioners'  claim  that  the  sale  of  the  subject  property  by  its  president,  Manuel  Dulay,  to  private  respondents  
spouses  Veloso  is  null  and  void  as  the  alleged  Board  Resolution  No.  18  was  passed  without  the  knowledge  
and  consent  of  the  other  members  of  the  board  of  directors  cannot  be  sustained.  
 
 
 
8.  NATIONAL  DEVELOPMENT  COMPANY  AND  NEW  AGRIX,  INC.  V.  PHILIPPINE  VETERANS  BANK  
G.R.  NOS.  84132-­‐‑33  -­‐‑  DECEMBER  10,  1990  
CRUZ,  J.  
 
DOCTRINE:  
A  private  corporation  should  be  organized  under  the  Corporation  Law  in  accordance  with  Article  XIV,  Section  
4  of  the  1973  Constitution  (now  Sec.  16,  Art.  XII  of  the  1987  Constitution).  
 
FACTS:  

   
3H  A.Y.  2017-­‐2018   10  
 
 

CORPORATION  LAW  CASE  DIGESTS  –  ATTY.  DANTE  DELA  CRUZ  

Pres.  Decree  No.  1717  ordered  the  rehabilitation  of  the  Agrix  Group  of  Companies  to  be  administered  mainly  
by  the  National  Development  Company  (NDC).  Sec.  4(1)  thereof  providing  that  "all  mortgages  and  other  liens  
presently   attaching   to   any   of   the   assets   of   the   dissolved   corporations   are   hereby   extinguished."   The   law  
outlined   the   procedure   for   filing   claims   against   the   Agrix   companies   and   created   a   Claims   Committee   to  
process  these  claims  and  also  created  New  Agrix,  Inc.  
 
Earlier,  the  Agrix  Marketing,  Inc.  (AGRIX)  had  executed  in  favor  of  private  respondent  Philippine  Veterans  
Bank  (PVB)  a  real  estate  mortgage  over  three  (3)  parcels  of  land.  During  the  existence  of  the  mortgage,  AGRIX  
went  bankrupt.  Pursuant  to  PD  1717,  the  PVB  filed  a  claim  with  the  AGRIX  Claims  Committee  for  the  payment  
of  its  loan  credit.  New  Agrix,  Inc.  and  NDC,  invoking  Sec.  4(1)  of  the  decree,  filed  a  petition  with  the  RTC  for  
the  cancellation  of  the  mortgage  lien  in  favor  of  the  PVB.  
 
ISSUE:  
Can  a  private  corporation  be  created  and  organized  pursuant  to  a  special  law?  
 
HELD:  
NO.  New  Agrix,  Inc.  was  created  by  special  decree  notwithstanding  the  provision  of  Article  XIV,  Section  4  of  
the  1973  Constitution,  then  in  force,  that:  
 
SEC.  4.  The  Batasang  Pambansa  shall  not,  except  by  general  law,  provide  for  the  formation,  organization,  or  
regulation  of  private  corporations,  unless  such  corporations  are  owned  or  controlled  by  the  Government  or  
any  subdivision  or  instrumentality  thereof.  
 
The   new   corporation   is   neither   owned   nor   controlled   by   the   government.   The   NDC   was   merely   required   to  
extend   a   loan   of   not   more   than   P10,000,000.00   to   New   Agrix,   Inc.   Pending   payment   thereof,   NDC   would  
undertake   the   management   of   the   corporation,   but   with   the   obligation   of   making   periodic   reports   to   the  
Agrix  board  of  directors.  After  payment  of  the  loan,  the  said  board  can  then  appoint  its  own  management.  
The  stocks  of  the  new  corporation  are  to  be  issued  to  the  old  investors  and  stockholders  of  AGRIX  upon  proof  
of  their  claims  against  the  abolished  corporation.  They  shall  then  be  the  owners  of  the  new  corporation.  New  
Agrix,  Inc.  is  entirely  private  and  so  should  have  been  organized  under  the  Corporation  Law  in  accordance  
with  the  above-­‐‑cited  constitutional  provision.  
 
Note:    Sec.  16,  Art.  XII  (National  Economy  and  Patrimony)  of  the  1987  Constitution  provides:  
 
The   Congress   shall   not,   except   by   general   law,   provide   for   the   formation,   organization,   or   regulation   of  
private   corporations.   Government-­‐‑owned   or   controlled   corporations   may   be   created   or   established   by  
special  charters  in  the  interest  of  the  common  good  and  subject  to  the  test  of  economic  viability.  
 
 
 
9.   PIONEER   INSURANCE   &   SURETY   CORPORATION   vs.   THE   HON.   COURT   OF   APPEALS,   BORDER  
MACHINERY  &  HEAVY  EQUIPMENT,  INC.,  (BORMAHECO),  CONSTANCIO  M.  MAGLANA  and  JACOB  S.  LIM  
G.R.  No.  84197  July  28,  1989  
GUTIERREZ,  JR.,  J  
 
DOCTRINE:  Where  persons  associate  themselves  together  under  articles  to  purchase  property  to  carry  on  a  
business,  and  their  organization  is  so  defective  as  to  come  short  of  creating  a  corporation  within  the  statute,  
they  become  in  legal  effect  partners  inter  se,  and  their  rights  as  members  of  the  company  to  the  property  
acquired   by   the   company   will   be   recognized.   One   who   takes   no   part   except   to   subscribe   for   stock   in   a  
proposed  corporation,  which  is  never  legally  formed,  does  not  become  a  partner  with  other  subscribers  who  

   
3H  A.Y.  2017-­‐2018   11  
 
 

CORPORATION  LAW  CASE  DIGESTS  –  ATTY.  DANTE  DELA  CRUZ  

engage  in  business  under  the  name  of  the  pretended  corporation,  so  as  to  be  liable  as  such  in  an  action  for  
settlement  of  the  alleged  partnership  and  contribution.    
FACTS:  Jacob  S.  Lim  (petitioner  in  G.R.  No.  84157)  was  engaged  in  the  airline  business  as  owner-­‐‑operator  of  
Southern  Air  Lines  (SAL)  a  single  proprietorship.  
Japan  Domestic  Airlines  (JDA)  and  Lim  entered  into  and  executed  a  sales  contract  (Exhibit  A)  for  the  sale  and  
purchase  of  two  (2)  DC-­‐‑3A  Type  aircrafts  and  one  (1)  set  of  necessary  spare  parts  for  the  total  agreed  price  
of  US  $109,000.00  to  be  paid  in  installments  
 
It  appears  that  Border  Machinery  and  Heavy  Equipment  Company,  Inc.  (Bormaheco),  Francisco  and  Modesto  
Cervantes  (Cervanteses)  and  Constancio  Maglana  (respondents  in  both  petitions)  contributed  some  funds  
used   in   the   purchase   of   the   above   aircrafts   and   spare   parts.   The   funds   were   supposed   to   be   their  
contributions  to  a  new  corporation  proposed  by  Lim  to  expand  his  airline  business.    
 
Lim  doing  business  under  the  name  and  style  of  SAL  executed  in  favor  of  Pioneer  as  deed  of  chattel  mortgage  
as   security   for   the   latter's   suretyship   in   favor   of   the   former.   It   was   stipulated   therein   that   Lim   transfer   and  
convey  to  the  surety  the  two  aircrafts.  Lim  defaulted  on  his  subsequent  installment  payments  prompting  JDA  
to  request  payments  from  the  surety.  Pioneer  paid  a  total  sum  of  P298,626.12.  
 
Pioneer   filed   an   action   for   judicial   foreclosure   with   an   application   for   a   writ   of   preliminary   attachment  
against   Lim   and   respondents,   the   Cervanteses,   Bormaheco   and   Maglana.   Maglana,   Bormaheco   and   the  
Cervanteses  filed  cross-­‐‑claims  against  Lim  alleging  that  they  were  not  privies  to  the  contracts  signed  by  Lim.  
 
It   was   established   that   no   corporation   was   formally   formed   between   Lim   and   Maglana,   Bormaheco,  
Cervanteses  
 
ISSUE:  Whether  or  not  Maglana  et  al  must  share  in  the  loss  as  general  partners.  
HELD:   No.   No   de   facto   partnership   was   created   among   the   parties,   which   would   entitle   Lim   to   a  
reimbursement   of   the   supposed   losses   of   the   proposed   corporation.   The   record   shows   that   the   Lim   was  
acting  on  his  own  and  not  in  behalf  of  his  other  would-­‐‑be  incorporators  in  transacting  the  sale  of  the  airplanes  
and  spare  parts.  This  can  be  inferred  from  acts  of  unilaterally  taking  out  a  surety  from  Pioneer  Insurance  and  
not  using  the  funds  he  got  from  Maglana  et  al.  Lim  denied  having  received  any  amount  from  respondents  
Bormaheco,  the  Cervanteses  and  Maglana.    
The  trial  court  and  the  appellate  court  found  that  Lim  received  the  amount  of  P151,000.00  representing  the  
participation  of  Bormaheco  and  Atty.  Constancio  B.  Maglana  in  the  ownership  of  the  subject  airplanes  and  
spare  parts.  The  record  shows  that  defendant  Maglana  gave  P75,000.00  to  Lim  thru  the  Cervanteses.  It  is  
therefore  clear  that  the  Lim  never  had  the  intention  to  form  a  corporation  with  the  respondents  despite  his  
representations  to  them.  This  gives  credence  to  the  cross-­‐‑claims  of  the  respondents  to  the  effect  that  they  
were   induced   and   lured   by   Lim   to   make   contributions   to   a   proposed   corporation,   which   was   never   formed  
because  Lim  reneged  on  their  agreement.  
 
10.   IRON   AND   STEEL   AUTHORITY   VS.   THE   COURT   OF   APPEALS   AND   MARIA   CRISTINA   FERTILIZER  
CORPORATION  
G.R.  NO.  102976/OCTOBER  25,  1995  
FELICIANO,  J.  
 
CASE   DOCTRINE:   When   the   statutory   term   of   a   non-­‐‑incorporated   agency   expires,   the   powers,   duties   and  
functions   as   well   as   the   assets   and   liabilities   of   that   agency   revert   back   to,   and   are   re-­‐‑assumed   by,   the  
Republic   of   the   Philippines,   in   the   absence   of   special   provisions   of   law   specifying   some   other   disposition  
thereof   such   as,   e.g.,   devolution   or   transmission   of   such   powers,   duties,   functions,   etc.   to   some   other  
identified  successor  agency  or  instrumentality  of  the  Republic  of  the  Philippines.  When  the  expiring  agency  

   
3H  A.Y.  2017-­‐2018   12  
 
 

CORPORATION  LAW  CASE  DIGESTS  –  ATTY.  DANTE  DELA  CRUZ  

is  an  incorporated  one,  the  consequences  of  such  expiry  must  be  looked  for,  in  the  first  instance,  in  the  charter  
of  that  agency  and,  by  way  of  supplementation,  in  the  provisions  of  the  Corporation  Code.  Since,  in  the  instant  
case,  ISA  is  a  non-­‐‑incorporated  agency  or  instrumentality  of  the  Republic,  its  powers,  duties,  functions,  assets  
and  liabilities  are  properly  regarded  as  folded  back  into  the  Government  of  the  Republic  of  the  Philippines  
and  hence  assumed  once  again  by  the  Republic,  no  special  statutory  provision  having  been  shown  to  have  
mandated  succession  thereto  by  some  other  entity  or  agency  of  the  Republic.  
FACTS:  
1.   Petitioner   Iron   and   Steel   Authority   ("ISA")   was   created   by   Presidential   Decree   (P.D.)   No.   272   dated   9  
August  1973  in  order,  generally,  to  develop  and  promote  the  iron  and  steel  industry  in  the  Philippines.  
2.   Among   the   powers   and   functions   of   the   ISA   includes   the   authority   to   to   initiate   expropriation   of   land  
required  for  basic  iron  and  steel  facilities  for  subsequent  resale  and/or  lease  to  the  companies  involved  if  it  
is  shown  that  such  use  of  the  State's  power  is  necessary  to  implement  the  construction  of  capacity  which  is  
needed  for  the  attainment  of  the  objectives  of  the  Authority.  
3.  P.D.  No.  272  initially  created  petitioner  ISA  for  a  term  of  five  (5)  years  counting  from  9  August  1973.  When  
ISA's   original   term   expired   on   10   October   1978,   its   term   was   extended   for   another   ten   (10)   years   by  
Executive  Order  No.  555  dated  31  August  1979.  
4.   The   National   Steel   Corporation   ("NSC")   then   a   wholly   owned   subsidiary   of   the   National   Development  
Corporation  which  is  itself  an  entity  wholly  owned  by  the  National  Government,  embarked  on  an  expansion  
program  embracing,  among  other  things,  the  construction  of  an  integrated  steel  mill  in  Iligan  City.  
  a.  The  construction  of  such  a  steel  mill  was  considered  a  priority  and  major  industrial  project  of    
  the   Government.   Pursuant   to   the   expansion   program   of   the   NSC,   Proclamation   No.   2239   was  
  issued  by  the  President  of  the  Philippines  on  16  November  1982  withdrawing  from  sale  or    
  settlement   a   large   tract   of   public   land   (totalling   about   30.25   hectares   in   area)   located   in   Iligan  
  City,  and  reserving  that  land  for  the  use  and  immediate  occupancy  of  NSC.  
5.  Since  certain  portions  of  the  public  land  subject  matter  Proclamation  No.  2239  were  occupied  by  a  non-­‐‑
operational   chemical   fertilizer   plant   and   related   facilities   owned   by   private   respondent   Maria   Cristina  
Fertilizer  Corporation  ("MCFC"),  Letter  of  Instruction  (LOI),  No.  1277,  also  dated  16  November  1982,  was  
issued  directing  the  NSC  to  "negotiate  with  the  owners  of  MCFC,  for  and  on  behalf  of  the  Government,  for  the  
compensation  of  MCFC's  present  occupancy  rights  on  the  subject  land."  
  a.   LOI   No.   1277   also   directed   that   should   NSC   and   private   respondent   MCFC   fail   to   reach   an  
  agreement  within  a  period  of  sixty  (60)  days  from  the  date  of  LOI  No.  1277,  petitioner  ISA  was  to  
  exercise   its   power   of   eminent   domain   under   P.D.   No.   272   and   to   initiate   expropriation  
  proceedings   in   respect   of   occupancy   rights   of   private   respondent   MCFC   relating   to   the   subject  
  public  land  as  well  as  the  plant  itself  and  related  facilities  and  to  cede  the  same  to  the  NSC.  
6.  Negotiations  between  NSC  and  private  respondent  MCFC  did  fail.  
  a.   Accordingly,   on   18   August   1983,   petitioner   ISA   commenced   eminent   domain   proceedings   against  
private  respondent  MCFC  in  the  Regional  Trial  Court,  Branch  1,  of  Iligan  City,  praying  that  it  (ISA)  be  places  
in   possession   of   the   property   involved   upon   depositing   in   court   the   amount   of   P1,760,789.69   representing  
ten  percent  (10%)  of  the  declared  market  values  of  that  property.  
7.  A  writ  of  possession  was  issued  by  the  trial  court  in  favor  of  ISA.  
  a.  ISA  in  turn  placed  NSC  in  possession  and  control  of  the  land  occupied  by  MCFC's  fertilizer    plant  
installation.  
8.  The  case  proceeded  to  trial.  While  the  trial  was  ongoing,  however,  the  statutory  existence  of  petitioner  ISA  
expired  on  11  August  1988.  
  a.   MCFC   then   filed   a   motion   to   dismiss,   contending   that   no   valid   judgment   could   be   rendered  
  against  ISA  which  had  ceased  to  be  a  juridical  person.  
9.  In  an  Order  dated  9  November  1988,  the  trial  court  granted  MCFC's  motion  to  dismiss  and  did  dismiss  the  
case.  
  a.  The  dismissal  was  anchored  on  the  provision  of  the  Rules  of  Court  stating  that  "only  natural  or  
  juridical  persons  or  entities  authorized  by  law  may  be  parties  in  a  civil  case."  

   
3H  A.Y.  2017-­‐2018   13  
 
 

CORPORATION  LAW  CASE  DIGESTS  –  ATTY.  DANTE  DELA  CRUZ  

  b.   Petitioner   ISA   moved   for   reconsideration   of   the   trial   court's   Order,   contending   that   despite  
  the  expiration  of  its  term,  its  juridical  existence  continued  until  the  winding  up  of  its  affairs     could  
be  completed.  
  c.  In  the  alternative,  petitioner  ISA  urged  that  the  Republic  of  the  Philippines,  being  the  real     party-­‐‑
in-­‐‑interest,  should  be  allowed  to  be  substituted  for  petitioner  ISA.  
10.  The  trial  court  denied  the  motion  for  reconsideration.  
11.  The  Court  of  Appeals  affirmed  the  order  of  the  dismissal  of  the  trial  court.  
  a.   The   Court   of   Appeals   held   that   petitioner   ISA,   "a   government   regulatory   agency   exercising  
  sovereign   functions,"   did   not   have   the   same   rights   as   an   ordinary   corporation   and   that   the   ISA,  
  unlike   corporations   organized   under   the   Corporation   Code,   was   not   entitled   to   a   period   for  
  winding   up   its   affairs   after   expiration   of   its   legally   mandated   term,   with   the   result   that   upon  
  expiration   of   its   term   on   11   August   1987,   ISA   was   "abolished   and   [had]   no   more   legal   authority  
  to  perform  governmental  functions."  
 
ISSUE:  The  principal  issue  which  the  Court  must  address  in  this  case  is  whether  or  not  the  Republic  of  the  
Philippines  is  entitled  to  be  substituted  for  ISA  in  view  of  the  expiration  of  ISA's  term.  
RULING:  
1.  Rule  3,  Section  1  of  the  Rules  of  Court  specifies  who  may  be  parties  to  a  civil  action:  
    Sec.  1.   Who  May  Be  Parties.  —  Only  natural  or  juridical  persons  or  entities  authorized    
    by  law  may  be  parties  in  a  civil  action.  
 
Under   the   above   quoted   provision,   it   will   be   seen   that   those   who   can   be   parties   to   a   civil   action   may   be  
broadly  categorized  into  two  (2)  groups:  
    (a)   those  who  are  recognized  as  persons  under  the  law  whether  natural,  i.e.,      
  biological  persons,  on  the  one  hand,  or  juridical  person  such  as  corporations,  on  the      
  other  hand;  and  
    (b)   entities  authorized  by  law  to  institute  actions.  
    Examination  of  the  statute  which  created  petitioner  ISA  shows  that  ISA  falls  under      
  category  (b)  above.  
2.   Clearly,   ISA   was   vested   with   some   of   the   powers   or   attributes   normally   associated   with   juridical  
personality.  
  a.   There   is,   however,   no   provision   in   P.D.   No.   272   recognizing   ISA   as   possessing   general   or  
  comprehensive  juridical  personality  separate  and  distinct  from  that  of  the  Government.  
  b.   The   ISA   in   fact   appears   to   the   Court   to   be   a   non-­‐‑incorporated   agency   or   instrumentality   of   the  
  Republic   of   the   Philippines,   or   more   precisely   of   the   Government   of   the   Republic   of   the  
  Philippines.  
  c.   It   is   common   knowledge   that   other   agencies   or   instrumentalities   of   the   Government   of   the  
  Republic   are   cast   in   corporate   form,   that   is   to   say,   are   incorporated   agencies   or  
  instrumentalities,  sometimes  with  and  at  other  times  without  capital  stock,  and  accordingly    vested  
with  a  juridical  personality  distinct  from  the  personality  of  the  Republic.  Among  such     incorporated  
agencies  or  instrumentalities  are:  National  Power  Corporation;  Philippine  Ports     Authority;   National  
Housing  Authority;  Philippine  National  Oil  Company;  Philippine  National     Railways;     Public   Estates  
Authority;    Philippine  Virginia  Tobacco  Administration,  and  so     forth.  It     is   worth   noting   that  
the  term  "Authority"  has  been  used  to  designate  both  incorporated     and     non-­‐‑incorporated   agencies   or  
instrumentalities  of  the  Government.  
3.   The   Court   consider   that   the   ISA   is   properly   regarded   as   an   agent   or   delegate   of   the   Republic   of   the  
Philippines.  
  a.   The   Republic   itself   is   a   body   corporate   and   juridical   person   vested   with   the   full   panoply   of  
  powers  and  attributes  which  are  compendiously  described  as  "legal  personality."  

   
3H  A.Y.  2017-­‐2018   14  
 
 

CORPORATION  LAW  CASE  DIGESTS  –  ATTY.  DANTE  DELA  CRUZ  

4.  When  the  statutory  term  of  a  non-­‐‑incorporated  agency  expires,  the  powers,  duties  and  functions  as  well  
as   the   assets   and   liabilities   of   that   agency   revert   back   to,   and   are   re-­‐‑assumed   by,   the   Republic   of   the  
Philippines,  in  the  absence  of  special  provisions  of  law  specifying  some  other  disposition  thereof  such  as,  e.g.,  
devolution  or  transmission  of  such  powers,  duties,  functions,  etc.  to  some  other  identified  successor  agency  
or  instrumentality  of  the  Republic  of  the  Philippines.  
  a.   When   the   expiring   agency   is   an   incorporated   one,   the   consequences   of   such   expiry   must   be  
  looked   for,   in   the   first   instance,   in   the   charter   of   that   agency   and,   by   way   of   supplementation,   in  
  the  provisions  of  the  Corporation  Code.  Since,  in  the  instant  case,  ISA  is  a  non-­‐‑incorporated     agency  
or  instrumentality  of  the  Republic,  its  powers,  duties,  functions,  assets  and  liabilities  are     properly  
regarded  as  folded  back  into  the  Government  of  the  Republic  of  the  Philippines  and     hence   assumed   once  
again  by  the  Republic,  no  special  statutory  provision  having  been  shown  to     have   mandated   succession  
thereto  by  some  other  entity  or  agency  of  the  Republic.  
5.  In  the  instant  case,  ISA  instituted  the  expropriation  proceedings  in  its  capacity  as  an  agent  or  delegate  or  
representative  of  the  Republic  of  the  Philippines  pursuant  to  its  authority  under  P.D.  No.  272.  
  a.   From   the   foregoing   premises,   it   follows   that   the   Republic   of   the   Philippines   is   entitled   to   be  
  substituted   in   the   expropriation   proceedings   as   party-­‐‑plaintiff   in   lieu   of   ISA,   the   statutory   term  
  of  ISA  having  expired.  
  b.   Put   a   little   differently,   the   expiration   of   ISA's   statutory   term   did   not   by   itself   require   or   justify  
  the  dismissal  of  the  eminent  domain  proceedings.  
 
 
 
11.  JACINTO  vs  CA  
G.R.  No.  80043/  198  SCRA  211  -­‐‑  June  6,  1991  
DAVIDE,  JR.,  J.  
 
DOCTRINE:   Corporate   veil   was   pierced   because   it   was   used   as   a   shield   to   perpetuate   fraud   and/or   confuse  
legitimate  issues.  There  was  no  clear  cut  delimitation  between  the  personality  of  Jacinto  and  the  corporation.  
FACTS:   Roberto   A.   Jacinto   is   the   President   and   General   Manager   of   the   Inland   Industries,   Inc.,   and   a  
substantial  stockholder  (he  and  his  wife  owns  52%)  thereof.  He  dealt  entirely  with  Metropolitan  Bank  and  
Trust  Co.  in  its  transactions.  The  stipulation  of  facts  show  that  Jacinto  acted  in  his  capacity  as  President/GM  
of  defendant  corporation  and  that  "all  the  goods  covered  by  the  3  Letters  of  Credit  and  paid  for  under  the  
Bills   of   Exchange   were   delivered   to   and   received   by   defendant   Inland   Industries,   Inc.   through   its   co-­‐‑
defendant  Roberto  A.  Jacinto,  its  President  and  General  Manager,  who  signed  for  and  in  behalf  of  defendant  
Inland  and  agreed  to  the  terms  and  conditions  of  3  Trust  Receipts  covering  the  same.    
Jacinto   absconds   and   tried   to   escape   liability   and   shift   the   entire   blame   under   the   trust   receipts   exclusively  
on  the  corporation.  He  was  ordered  by  the  RTC  in  its  decision  to  jointly  and  severally  pay  Metrobank.  He  
asserted  that  he  cannot  be  held  solidarily  liable  with  the  corporation  because  he  just  signed  said  instruments  
in  his  official  capacity  as  president  and  the  corporation  has  a  juridical  personality  distinct  and  separate  from  
its   officers   and   stockholders.   He   also   asserted,   citing   an   American   case,   that   the   principle   of   piercing   the  
fiction   of   corporate   entity   should   be   applied   with   great   caution   and   not   precipitately,   because   a   dual  
personality  by  a  corporation  and  its  stockholders  would  defeat  the  principal  purpose  for  which  a  corporation  
is   formed.   Metrobank,   on   the   other   hand,   alleged   that   defendant   corporation   is   just   a   mere   alter   ego   of  
Roberto  Jacinto  who  is  its  President  and  General  Manager,  while  the  wife  of  the  latter  owns  a  majority  of  its  
shares  of  stock.  
ISSUES:   Whether   or   not   the   respondent   Court   of   Appeals   can   validly   pierce   the   fiction   of   corporate   identity  
of  the  defendant  corporation  Inland  Industries,  Inc.  even  if  there  is  no  allegation  in  the  complaint  and  no  
proof  was  presented  in  court  to  serve  as  legal  justification  for  the  same.  
HELD:   YES.   The   circumstances   aforestated   lead   Us   to   conclude   that   the   corporate   veil   that   en-­‐‑shrouds  
defendant  Inland  Industries,  Inc.  could  be  validly  pierced,  and  a  host  of  cases  decided  by  our  High  Court  is  

   
3H  A.Y.  2017-­‐2018   15  
 
 

CORPORATION  LAW  CASE  DIGESTS  –  ATTY.  DANTE  DELA  CRUZ  

supportive  of  this  view.  Thus  it  held  that  "when  the  veil  of  corporate  fiction  is  made  as  a  shield  to  perpetuate  
fraud  and/or  confuse  legitimate  issues,  the  same  should  be  pierced."    
While   on   the   face   of   the   complaint   there   is   no   specific   allegation   that   the   corporation   is   a   mere   alter   ego   of  
petitioner,  subsequent  developments,  from  the  stipulation  of  facts  up  to  the  presentation  of  evidence  and  the  
examination   of   witnesses,   unequivocally   show   that   respondent   Metropolitan   Bank   and   Trust   Company  
sought  to  prove  that  petitioner  and  the  corporation  are  one  or  that  he  is  the  corporation.  No  serious  objection  
was   heard   from   petitioner.   Section   5   of   Rule   10   of   the   Rules   of   Court   provides   that   "when   evidence   is  
presented  by  one  party,  with  the  express  or  implied  consent  of  the  adverse  party,  as  to  issues  not  alleged  in  
the  pleadings,  judgment  may  be  rendered  validly  as  regards  those  issues,  which  shall  be  considered  as  if  they  
have  been  raised  in  the  pleadings.  There  is  implied  consent  to  the  evidence  thus  presented  when  the  adverse  
party  fails  to  object  thereto.    
 
 
 
12.  LYCEUM  V.  CA  (SAME  WITH  37)  
 
 
 
13.  PHILIPPINE  FIRST  INSURANCE  COMPANY  vs.  MARIA  CARMEN  HARTIGAN,  CGH,  and  O.  ENGKEE  
G.R.  No.  L-­‐‑26370    July  31,  1970  
BARREDO,  J.:  
 
DOCTRINE:   The   general   rule   as   to   corporations   is   that   each   corporation   shall   have   a   name   by   which   it   is   to  
sue  and  be  sued  and  do  all  legal  acts.  The  name  of  a  corporation  in  this  respect  designates  the  corporation  in  
the  same  manner  as  the  name  of  an  individual  designates  the  person."  Since  an  individual  has  the  right  to  
change  his  name  under  certain  conditions,  there  is  no  compelling  reason  why  a  corporation  may  not  enjoy  
the  same  right.  Of  course,  as  in  the  case  of  an  individual,  such  change  may  not  be  made  exclusively.  by  the  
corporation's   own   act.   It   has   to   follow   the   procedure   prescribed   by   law   for   the   purpose;   and   this   is   what   is  
important  and  indispensably  prescribed  —  strict  adherence  to  such  procedure.  
 
FACTS:  Plaintiff  was  originally  organized  as  an  insurance  corporation  under  the  name  of  'The  Yek  Tong  Lin  
Fire  and  Marine  Insurance  Co.,  Ltd.'  The  articles  of  incorporation  originally  presented  before  SEC  on  June  1,  
1953   state   that   the   name   of   the   corporation   was   'The   Yek   Tong   Lin   Fire   and   Marine   Insurance   Co.,   Ltd.'   On  
May  26,  1961  the  articles  of  incorporation  were  amended  pursuant  to  a  certificate  of  the  Board  of  Directors  
dated  March  8,  1961  changing  the  name  of  the  corporation  to  'Philippine  First  Insurance  Co.,  Inc.'.  
 
The  complaint  alleges  that  the  plaintiff  Philippine  First  Insurance  Co.,  Inc.,  doing  business  under  the  
name  of  'The  Yek  Tong  Lin  Fire  and  Marine  Insurance  Co.,  Lt.'  signed  as  co-­‐‑maker  together  with  defendant  
Maria  Carmen  Hartigan,  CGH,  a  promissory  note  for  P5,000.00  in  favor  of  the  China  Banking  Corporation,  
with  indemnity  agreement  stipulating  that  defendant  Maria  Carmen  Hartigan  and  CGH  shall  be  jointly  and  
severally   liable   to   pay   the   plaintiff   damages,   losses   or   expenses   of   whatever   kind   or   nature,   including  
attorney's  fees  and  legal  costs,  which  the  plaintiff  may  sustain  as  a  result  of  the  execution  by  the  plaintiff  and  
co-­‐‑maker  of  Maria  Carmen  Hartigan,  CGH,  of  the  said  promissory  note.  As  a  result  of  the  execution  of  the  
promissory  note  by  the  plaintiff  and  Maria  Carmen  Hartigan,  CGH,  the  China  Banking  Corporation  delivered  
to  the  defendant  Maria  Carmen  Hartigan,  CGH,  the  sum  of  P5,000.00  which  said  defendant  failed  to  pay  in  
full.  The  complaint  ends  with  a  prayer  for  judgment  against  the  defendants,  jointly  and  severally,  for  the  sum  
of  P4,559.50  with  interest  at  the  rate  of  12%  per  annum  from  November  23,  1961  plus  P911.90  by  way  of  
attorney's  fees  and  costs.    
 

   
3H  A.Y.  2017-­‐2018   16  
 
 

CORPORATION  LAW  CASE  DIGESTS  –  ATTY.  DANTE  DELA  CRUZ  

The  defendants  deny  the  allegation  that  the  plaintiff  formerly  conducted  business  under  the  name  
and  style  of  'The  Yek  Tong  Lin  Fire  and  Marine  Insurance  Co.,  Ltd.'  They  admit  the  execution  of  the  indemnity  
agreement   but   they   claim   that   they   signed   said   agreement   in   favor   of   the   Yek   Tong   Lin   Fire   and   Marine  
Insurance   Co.,   Ltd.'   and   not   in   favor   of   the   plaintiff.   Defendants   claim   that   there   is   no   privity   of   contract  
between  the  plaintiff  and  the  defendants  and  consequently,  the  plaintiff  has  no  cause  of  action  against  them,  
considering   that   the   complaint   does   not   allege   that   the   plaintiff   and   the   'Yek   Tong   Lin   Fire   and   Marine  
Insurance  Co.,  Ltd.'  are  one  and  the  same  or  that  the  plaintiff  has  acquired  the  rights  of  the  latter.    
 
ISSUE:  May  a  Philippine  corporation  change  its  name  and  still  retain  its  original  personality  and  individuality  
as  such?  
 
HELD:  The  answer  is  not  difficult  to  find.  It  can  be  gleaned  at  once  that  Section  18  of  the  Corporation  Code  
does   not   only   authorize   corporations   to   amend   their   charter;   it   also   lays   down   the   procedure   for   such  
amendment;  and,  what  is  more  relevant  to  the  present  discussion,  it  contains  provisos  restricting  the  power  
to  amend  when  it  comes  to  the  term  of  their  existence  and  the  increase  or  decrease  of  the  capital  stock.  There  
is  no  prohibition  therein  against  the  change  of  name.  The  inference  is  clear  that  such  a  change  is  allowed,  for  
if   the  legislature   had  intended   to  enjoin   corporations  from   changing  names,   it  would   have  expressly  stated  
so  in  this  section  or  in  any  other  provision  of  the  law.  
 
The  general  rule  as  to  corporations  is  that  each  corporation  shall  have  a  name  by  which  it  is  to  sue  and  be  
sued   and   do   all   legal   acts.   The   name   of   a   corporation   in   this   respect   designates   the   corporation   in   the   same  
manner   as   the   name   of   an   individual   designates   the   person."   Since   an   individual   has   the   right   to   change   his  
name  under  certain  conditions,  there  is  no  compelling  reason  why  a  corporation  may  not  enjoy  the  same  
right.   There   is   nothing   sacrosanct   in   a   name   when   it   comes   to   artificial   beings.   The   sentimental  
considerations  which  individuals  attach  to  their  names  are  not  present  in  corporations  and  partnerships.  Of  
course,  as  in  the  case  of  an  individual,  such  change  may  not  be  made  exclusively  by  the  corporation's  own  
act.   It   has   to   follow   the   procedure   prescribed   by   law   for   the   purpose;   and   this   is   what   is   important   and  
indispensably  prescribed  —  strict  adherence  to  such  procedure.  
 
As   correctly   pointed   out   by   appellant,   the   approval   by   the   stockholders   of   the   amendment   of   its  
articles   of   incorporation   changing   the   name   "The   Yek   Tong   Lin   Fire   &   Marine   Insurance   Co.,   Ltd."   to  
"Philippine   First   Insurance   Co.,   Inc."   on   March   8,   1961,   did   not   automatically   change   the   name   of   said  
corporation   on   that   date.   To   be   effective,   Section   18   of   the   Corporation   Law   requires   that   "a   copy   of   the  
articles  of  incorporation  as  amended,  duly  certified  to  be  correct  by  the  president  and  the  secretary  of  the  
corporation  and  a  majority  of  the  board  of  directors  or  trustees,  shall  be  filed  with  the  Securities  &  Exchange  
Commissioner",  and  it  is  only  from  the  time  of  such  filing,  that  "the  corporation  shall  have  the  same  powers  
and  it  and  the  members  and  stockholders  thereof  shall  thereafter  be  subject  to  the  same  liabilities  as  if  such  
amendment  had  been  embraced  in  the  original  articles  of  incorporation."  It  goes  without  saying  then  that  
appellant  rightly  acted  in  its  old  name  when  on  May  15,  1961,  it  entered  into  the  indemnity  agreement  with  
the  defendant-­‐‑appellees;  for  only  after  the  filing  of  the  amended  articles  of  incorporation  with  the  Securities  
&  Exchange  Commission  on  May  26,  1961,  did  appellant  legally  acquire  its  new  name;  and  it  was  perfectly  
right  for  it  to  file  the  present  case  in  that  new  name  on  December  6,  1961.  Such  is,  but  the  logical  effect  of  the  
change  of  name  of  the  corporation  upon  its  actions.  Actions  brought  by  a  corporation  after  it  has  changed  its  
name  should  be  brought  under  the  new  name  although  for  the  enforcement  of  rights  existing  at  the  time  the  
change  was  made.    
 
 
 
14.  ZUELLIG  FREIGHT  V.  NLRC  
 

   
3H  A.Y.  2017-­‐2018   17  
 
 

CORPORATION  LAW  CASE  DIGESTS  –  ATTY.  DANTE  DELA  CRUZ  

 
 
15.  MUNICIPALITY  OF  MALABANG  V.  BENITO  
G.R.  NO.  L-­‐‑28113;  MARCH  28,  1969  
CASTRO,  J;  
 
Doctrine:  :    It  is  indeed  true  that,  generally,  an  inquiry  into  the  legal  existence  of  a  municipality  is  reserved  to  
the  State  in  a  proceeding  for  quo  warranto  or  other  direct  proceeding,  and  that  only  in  a  few  exceptions  may  
a  private  person  exercise  this  function  of  government.  4   But  the  rule  disallowing  collateral  attacks  applies  
only   where   the   municipal   corporation   is   at   least   a   de   facto   corporations.   5   For   where   it   is   neither   a  
corporation  de  jure  nor  de  facto,  but  a  nullity,  the  rule  is  that  its  existence  may  be,  questioned  collaterally  or  
directly   in   any   action   or   proceeding   by   anyone   whose   rights   or   interests   ate   affected   thereby,   including   the  
citizens   of   the   territory   incorporated   unless   they   are   estopped   by   their   conduct   from   doing   so.  In   the   cases  
where  a  de  facto  municipal  corporation  was  recognized  as  such  despite  the  fact  that  the  statute  creating  it  
was  later  invalidated,  the  decisions  could  fairly  be  made  to  rest  on  the  consideration  that  there  was  some  
other  valid  law  giving  corporate  vitality  to  the  organization.  
Facts:   Petitioner   Amer   Macaorao   Balindong   is   the   mayor   of   Malabang,   Lanao   del   Sur,   while   the   respondent  
Pangandapun  Bonito  is  the  mayor,  and  the  rest  of  the  respondents  are  the  councilors,  of  the  municipality  of  
Balabagan  of  the  same  province.    
The   petitioners   brought   this   action   for   prohibition   to   nullify   Executive   Order   386   and   to   restrain   the  
respondent  municipal  officials  from  performing  the  functions  of  their  respective  office  relying  on  the  ruling  
of  this  Court  in  Pelaez  v.  Auditor  General  and  Municipality  of  San  Joaquin  v.  Siva  to  wit;  As  this  Court  summed  
up  its  discussion:  "In  short,  even  if  it  did  not  entail  an  undue  delegation  of  legislative  powers,  as  it  certainly  
does,   said   section   68,   as   part   of   the   Revised   Administrative   Code,   approved   on   March   10,   1917,   must   be  
deemed  repealed  by  the  subsequent  adoption  of  the  Constitution,  in  1935,  which  is  utterly  incompatible  and  
inconsistent  with  said  statutory  enactment."  
Respondents  on  the  other  hand,  while  admitting  the  facts  alleged  in  the  petition,  nevertheless  argue  that  the  
rule   announced   in   Pelaez   can   have   no   application   in   this   case   because   unlike   the   municipalities   involved  
in  Pelaez,  the  municipality  of  Balabagan  is  at  least  a  de  facto  corporation,  having  been  organized  under  color  
of  a  statute  before  this  was  declared  unconstitutional,  its  officers  having  been  either  elected  or  appointed,  
and  the  municipality  itself  having  discharged  its  corporate  functions  for  the  past  five  years  preceding  the  
institution  of  this  action.  It  is  contended  that  as  a  de  facto  corporation,  its  existence  cannot  be  collaterally  
attacked,  although  it  may  be  inquired  into  directly  in  an  action  for  quo  warranto  at  the  instance  of  the  State  
and  not  of  an  individual  like  the  petitioner  Balindong.  
Issues:  (a.)Whether  or  not  the  municipality  of  Balabagan  is  a  de  facto  corporation;    
                   (b.)  Whether  a  statute  can  lend  color  of  validity  to  an  attempted  organization  of  a  municipality  despite  
the  fact  that  such  statute  is  subsequently  declared  unconstitutional  
Ruling:    It  is  indeed  true  that,  generally,  an  inquiry  into  the  legal  existence  of  a  municipality  is  reserved  to  the  
State   in   a   proceeding   for   quo   warranto   or   other   direct   proceeding,   and   that   only   in   a   few   exceptions   may   a  
private  person  exercise  this  function  of  government.  4  But  the  rule  disallowing  collateral  attacks  applies  only  
where  the  municipal  corporation  is  at  least  a  de  facto  corporations.  5  For  where  it  is  neither  a  corporation  de  
jure  nor  de  facto,  but  a  nullity,  the  rule  is  that  its  existence  may  be,  questioned  collaterally  or  directly  in  any  
action  or  proceeding  by  anyone  whose  rights  or  interests  ate  affected  thereby,  including  the  citizens  of  the  
territory  incorporated  unless  they  are  estopped  by  their  conduct  from  doing  so.    
As  a  result  of  this  analysis  of  the  cases  the  following  principles  may  be  deduced  which  seem  to  reconcile  the  
apparently  conflicting  decisions:  
I.  The  color  of  authority  requisite  to  the  organization  of  a  de  facto  municipal  corporation  may  be:  
1.  A  valid  law  enacted  by  the  legislature.  

   
3H  A.Y.  2017-­‐2018   18  
 
 

CORPORATION  LAW  CASE  DIGESTS  –  ATTY.  DANTE  DELA  CRUZ  

2.  An  unconstitutional  law,  valid  on  its  face,  which  has  either  (a)  been  upheld  for  a  time  by  the  courts  or  (b)  
not  yet  been  declared  void;  provided  that  a  warrant  for  its  creation  can  be  found  in  some  other  valid  law  or  
in  the  recognition  of  its  potential  existence  by  the  general  laws  or  constitution  of  the  state.  
II.   There   can   be   no   de   facto   municipal   corporation   unless   either   directly   or   potentially,   such   a   de  
jurecorporation  is  authorized  by  some  legislative  fiat.  
III.  There  can  be  no  color  of  authority  in  an  unconstitutional  statute  alone,  the  invalidity  of  which  is  apparent  
on  its  face.  
IV.  There  can  be  no  de  facto  corporation  created  to  take  the  place  of  an  existing  de  jure  corporation,  as  such  
organization  would  clearly  be  a  usurper.10  
In  the  cases  where  a  de  facto  municipal  corporation  was  recognized  as  such  despite  the  fact  that  the  statute  
creating  it  was  later  invalidated,  the  decisions  could  fairly  be  made  to  rest  on  the  consideration  that  there  
was  some  other  valid  law  giving  corporate  vitality  to  the  organization.  Hence,  in  the  case  at  bar,  the  mere  fact  
that  Balabagan  was  organized  at  a  time  when  the  statute  had  not  been  invalidated  cannot  conceivably  make  
it  a  de  factocorporation,  as,  independently  of  the  Administrative  Code  provision  in  question,  there  is  no  other  
valid  statute  to  give  color  of  authority  to  its  creation.  Indeed,  in  Municipality  of  San  Joaquin  v.  Siva,  this  Court  
granted   a   similar   petition   for   prohibition   and   nullified   an   executive   order   creating   the   municipality   of  
Lawigan  in  Iloilo  on  the  basis  of  thePelaez  ruling,  despite  the  fact  that  the  municipality  was  created  in  1961,  
before   section   68   of   the   Administrative   Code,   under   which   the   President   had   acted,   was   invalidated.   'Of  
course  the  issue  of  de  facto  municipal  corporation  did  not  arise  in  that  case.  
 In  Norton  v.  Shelby  Count,  12  Mr.  Justice  Field  said:  "An  unconstitutional  act  is  not  a  law;  it  confers  no  rights;  
it  imposes  no  duties;  it  affords  no  protection;  it  creates  no  office;  it  is,  in  legal  contemplation,  as  inoperative  
as  though  it  had  never  been  passed."  Accordingly,  he  held  that  bonds  issued  by  a  board  of  commissioners  
created  under  an  invalid  statute  were  unenforceable.  
  Executive   Order   386   "created   no   office."   This   is   not   to   say,   however,   that   the   acts   done   by   the   municipality  
of  Balabagan  in  the  exercise  of  its  corporate  powers  are  a  nullity  because  the  executive  order  "is,  in  legal  
contemplation,  as  inoperative  as  though  it  had  never  been  passed."  For  the  existence  of  Executive,  Order  386  
is  "an  operative  fact  which  cannot  justly  be  ignored."    
There  is  then  no  basis  for  the  respondents'  apprehension  that  the  invalidation  of  the  executive  order  creating  
Balabagan  would  have  the  effect  of  unsettling  many  an  act  done  in  reliance  upon  the  validity  of  the  creation  
of  that  municipality.    
 
 
 
16.  JOSE  YULO  AGRICULTURAL  CORPORATION  V.  SPS.  DAVIS  
G.R.  NO.  197709,  AUGUST  03,  2015  
DEL  CASTILLO,  J.  
 
DOCTRINE:  
Where  two  certificates  of  title  purport  to  include  the  same  land,  the  earlier  in  date  prevails.  
 
Yulo,  and  petitioner  for  that  matter,  which  is  a  corporation  that  belonged  to  Yulo  himself  or  is  connected  to  
him   and   which   became   his   successor-­‐‑in-­‐‑interest,   knew   everything   as   far   as   his   land   is   concerned,   or   is  
charged   with   knowledge   at   least.   They   cannot   claim   to   be   ignorant   of   everything   that   went   on   with   the  
properties  they  owned.  
 
FACTS:  
•   Lot  62-­‐‑A  in  Binalbagan,  Negros  Occidental,  consisting  of  204,560  square  meters,  was  registered  in  the  
name  of  Jose  L.  Yulo  (Yulo).  It  was  subdivided  in  1963  into  lots  covered  by  TCT  Nos.  36824  to  36852.  
 

   
3H  A.Y.  2017-­‐2018   19  
 
 

CORPORATION  LAW  CASE  DIGESTS  –  ATTY.  DANTE  DELA  CRUZ  

•   TCT  36852,  covering  Lot  29  with  an  area  of  198,595  square  meters,  was  further  subdivided  in  1969  into  
several  lots  which  were  all  registered  in  Yulo's  name.  Among  these  lots  are  Lots  24,  25,  72  (TCT  T-­‐‑62499),  
91  (TCT  T-­‐‑64737),  92  (TCT  T-­‐‑64738),  and  96  (TCT  T-­‐‑64742).  The  titles  to  Lots  91,  92  and  96  were  issued  
in  1971.  
 
•   Yulo  sold  Lots  91,  92  and  96  to  spouses  Ignacio  Madrina,  Jr.  and  Teresa  Saldua  (the  Madrinas)  in  1975.  
 
•   Lots  24,  25,  91,  928  and  969  were  subsequently  mortgaged  to  Nation  Bank,  which  eventually  foreclosed  
and  became  owner  of  the  lots.  At  the  time  of  the  foreclosure  and  sale  to  Nation  Bank,  the  said  lots  already  
contained   improvements   in   the   form   of   a   house   and   fence   which   were   constructed   by   the   previous  
occupants,  spouses  Ernesto  and  Wendelina  Gabayeron  (the  Gabayerons).  In  1992,  Nation  Bank  sold  these  
five   lots   with   existing   improvements   to   the   herein   respondents,   spouses   Scott   and   Perla   Cabaylo   Davis.  
Consequently,  TCT  Nos.  T-­‐‑163622,  T-­‐‑163623  and  T-­‐‑163624  over  Lots  91,  92  and  96,  respectively,  were  
issued  in  respondents'  favor.  
 
•   On   the   other   hand,   TCT   T-­‐‑62499   covering   Lot   72   -­‐‑   consisting   of   183,920   square   meters   -­‐‑   was   cancelled  
and   TCT   T-­‐‑113437   was   issued   in   1979   in   the   name   of   herein   petitioner   Jose   L.   Yulo   Agricultural  
Corporation.  
 
•   In  1982,  Lot  72  was  further  subdivided  into  several  lots  and  registered  in  petitioner's  name.  Among  these  
lots  are  Lots  310  (TCT  T-­‐‑126644),  411  (TCT  T-­‐‑126645),  and  512  (TCT  T-­‐‑126646).  In  1994,  Lot  5  was  
sold  to  spouses  Jose  and  Petronila  Trajera  (the  Trajeras),  and  thus  TCT  T-­‐‑167841  over  said  lot  was  issued  
in  their  favor.  
 
•   In  1999,  respondents  received  separate  demand  letters  from  petitioner  and  the  Trajeras  requiring  them  
to  remove  a  portion  of  the  Gabayeron  home  and  fence  which  they  claim  encroached  upon  their  respective  
properties  (petitioner's  Lots  3  and  4,  and  the  Trajeras'  Lot  5).  Respondents  also  received  a  letter  from  
the  Local  Building  Official  of  Binalbagan,  Negros  Occidental  threatening  them  with  sanction  under  the  
National   Building   Code   unless   they   remove   the   encroaching   improvements   which   allegedly   extended  
beyond  the  boundary  lines  of  their  property.  
 
•   Respondents  filed  a  case  for  quieting  of  title  and  damages  against  the  Trajeras,  Yulo,  Nation  Bank  and  the  
Binalbagan  Local  Building  Official,  Engineer  Patrick  Mabag  before  the  RTC  of  Himamaylan  City,  Negros  
Occidental.  
 
•   RTC’s   Ruling:   decided   in   favour   of   the   plaintiffs.   Moral   damages   may   be   awarded   by   reason   of   the  
sufferings,  physical  or  mental,  sustained  by  the  claiming  party.  However,  the  grant  of  such  damages  is  
not  subject  to  the  whims  and  caprices  of  judges  or  courts.  The  court's  discretion  in  granting  or  refusing  
it  is  governed  by  reason  and  justice.  In  order  that  an  individual  may  be  made  liable,  the  law  requires  that  
his   act   be   wrongful.   The   adverse   result   of   an   action   does   not   per   se   make   it   wrongful   as   to   justify   an  
assessment  of  damages  against  the  actor  (Rubio  vs.  Court  of  Appeals,  141  SCRA  488).  
 
•   In  this  case,  there  is  no  basis  to  justify  the  award  for  moral  damages.  As  owner,  Jose  L.  Yulo  Agricultural  
Corporation   has   the   right   to   have   Lot   No.   72   subdivided   into   sublots.   The   overlapping   of   Lot   No.   3   over  
Lot  No.  92  was  caused  by  an  error  committed  by  the  Geodetic  Engineer  who  conducted  the  survey  of  said  
lot.  In  a  similar  situation,  the  overlapping  of  Lot  Nos.  4  and  5  over  Lot  Nos.  91  and  96  was  due  to  the  
subdivision  of  Lot  No.  72  into  sublots  made  by  a  Geodetic  Engineer.  In  other  words,  the  wrongful  act  was  
not  committed  by  the  management  of  the  corporation  in  order  for  it  to  be  held  liable  for  moral  damages.  
Settled  is  the  rule  that  moral  damages  cannot  be  awarded  in  the  absence  of  a  wrongful  act  or  omission  
or  of  fraud  or  bad  faith  (Siasat  vs.  Intermediate  Appellate  Court,  139  SCRA  238).  

   
3H  A.Y.  2017-­‐2018   20  
 
 

CORPORATION  LAW  CASE  DIGESTS  –  ATTY.  DANTE  DELA  CRUZ  

 
•   CA’s   Ruling:   In   an   action   for   quieting   of   title,   the   issue   to   be   resolved   is   who,   between   the   parties,   has   a  
better  right  to  the  challenged  property.  After  an  exhaustive  examination  of  the  evidence  and  the  records  
in  this  case,  We  rule  in  favor  of  the  Davis  spouses.  
 
•   Hence,  this  petition  for  review  on  certiorari.  
 
ISSUE:  W/N  respondents  have  a  better  right  than  petitioner.  -­‐‑  YES  
 
HELD:  
 
Yulo,  and  petitioner  for  that  matter,  which  is  a  corporation  that  belonged  to  Yulo  himself  or  is  connected  to  
him   and   which   became   his   successor-­‐‑in-­‐‑interest,   knew   everything   as   far   as   his   land   is   concerned,   or   is  
charged  with  knowledge  at  least.  Yulo  was  the  sole  owner  of  the  properties  involved,  and  he  and  his  outfit  
were  the  sellers  of  the  properties  which  eventually  were  acquired  by  the  respondents  and  the  Trajeras.  They  
cannot   claim   to   be   ignorant   of   everything   that   went   on   with   the   properties   they   owned.   They   cannot   be  
allowed  to  benefit  from  their  own  mistakes  at  the  expense  of  the  respondents.  Indeed,  if  there  is  anybody  
who  must  be  considered  in  bad  faith,  it  is  they;  they  should  have  known  that  there  was  an  overlapping  of  
titles  in  their  very  own  lands.  And  if  it  is  true  that  Lots  91,  92  and  96  are  non-­‐‑existent  lots,  Yulo  and  petitioner  
would   have   known   it;   yet   Yulo   sold   them   in   1975   to   the   Madrinas,   and   eventually   found   their   way   to  
respondents.  Indeed,  as  testified  to  by  the  Records  Officer  of  the  Register  of  Deeds  of  Negros  Occidental,  Lots  
91,   92   and   96   covered   by   T-­‐‑163622   to   163624   in   the   name   of   respondents   have   the   same   technical  
description  as  Lots  91,  92  and  96  covered  by  TCT  T-­‐‑64737,  T-­‐‑64738  and  T-­‐‑64742  and  registered  in  the  name  
of  Yulo.  In  other  words,  there  is  no  doubt  that  respondents'  titles  were  derived  from  Yulo's;  this  fact  is  not  
even  assailed  or  denied  by  petitioner  in  any  of  its  pleadings.  
 
As   for   damages,   we   can   only   reiterate   what   the   CA   has   said.   Since   the   award   of   damages   was   raised   for   the  
first  time  in  petitioner's  motion  for  reconsideration  of  the  assailed  CA  Decision  and  not  in  its  appellant's  brief,  
the  award  must  stand.  
 
WHEREFORE,  the  Petition  is  DENIED.  The  assailed  Decision  and  Resolution  of  the  CA  are  AFFIRMED.  
 
 
 
17.  ASIA  BANKING  CORPORATION  vs.  STANDARD  PRODUCTS,  CO.,  INC.  
G.R.  No.  22106                      September  11,  1924  
OSTRAND,  J.:  
 
DOCTRINE:  
  The  general  rule  is  that  in  the  absence  of  fraud  a  person  who  has  contracted  or  otherwise  dealt  with  
an  association  in  such  a  way  as  to  recognize  and  in  effect  admit  its  legal  existence  as  a  corporate  body  is  
thereby  estopped  to  deny  its  corporate  existence  in  any  action  leading  out  of  or  involving  such  contract  or  
dealing,   unless   its   existence   is   attacked   for   cause   which   have   arisen   since   making   the   contract   or   other  
dealing  relied  on  as  an  estoppel  and  this  applies  to  foreign  as  well  as  to  domestic  corporations.  
 
FACTS:  
  This  action  is  brought  to  recover  the  sum  of  P24,736.47,  the  balance  due  on  a  promissory  note.  At  the  
trial   of   the   case   the   plaintiff   failed   to   prove   affirmatively   the   corporate   existence   of   the   parties   and   the  
appellant  insists  that  under  these  circumstances  the  court  erred  in  finding  that  the  parties  were  corporations  
with  juridical  personality  and  assigns  same  as  reversible  error.  The  promissory  notes  states:  

   
3H  A.Y.  2017-­‐2018   21  
 
 

CORPORATION  LAW  CASE  DIGESTS  –  ATTY.  DANTE  DELA  CRUZ  

 
P37,757.22  
 
MANILA,  P.  I.,          Nov.  28,  1921.  
 
MANILA,  P.  I.,  Nov.  28,  1921.  
 
On  demand,  after  date  we  promise  to  pay  to  the  Asia  Banking  Corporation,  or  order,  the  sum  of  thirty-­‐‑seven  
thousand  seven  hundred  fifty-­‐‑seven  and  22/100  pesos  at  their  office  in  Manila,  for  value  received,  together  
with  interest  at  the  rate  of  ten  per  cent  per  annum.  
 
No.  ________  Due  __________  
 
THE  STANDARD  PRODUCTS  CO.,  INC.  
               By          (Sgd.)  GEORGE  H.  SEAVER  
   
By          President  
 
ISSUE:  
                         Whether  the  court  erred  in  finding  that  the  parties  were  corporations  with  juridical  personality.  
 
HELD:  
                          There   is   no   merit   whatever   in   the   appellant's   contention.   The   defendant   having   recognized   the  
corporate  existence  of  the  plaintiff  by  making  a  promissory  note  in  its  favor  and  making  partial  payments  on  
the  same  is  therefore  estopped  to  deny  said  plaintiff's  corporate  existence.  It  is,  of  course,  also  estopped  from  
denying   its   own   corporate   existence.   Under   these   circumstances   it   was   unnecessary   for   the   plaintiff   to  
present  other  evidence  of  the  corporate  existence  of  either  of  the  parties.  It  may  be  noted  that  there  is  no  
evidence  showing  circumstances  taking  the  case  out  of  the  rules  stated.  
 
 
 
18.  (NO  CASE)  
 
 
 
 
19.  CHIANG  KAI  SHEK  SCHOOL  VS.  COURT  OF  APPEALS  AND  FAUSTINA  FRANCO  OH  
G.R.  NO.  L-­‐‑58028  APRIL  18,  1989  
CRUZ,  J.:  
   
DOCTRINE:  
There  should  also  be  no  question  that  having  contracted  with  the  private  respondent  every  year  for  thirty  
two  years  and  thus  represented  itself  as  possessed  of  juridical  personality  to  do  so,  the  petitioner  is  now  
estopped  from  denying  such  personality  to  defeat  her  claim  against  it.  According  to  Article  1431  of  the  Civil  
Code,  "through  estoppel  an  admission  or  representation  is  rendered  conclusive  upon  the  person  making  it  
and  cannot  be  denied  or  disproved  as  against  the  person  relying  on  it."  
   
FACTS:    
 An   unpleasant   surprise   awaited   Fausta   F.   Oh   when   she   reported   for   work   at   the   Chiang   Kai   Shek   School   in  
Sorsogon  on  the  first  week  of  July,  1968.  She  was  told  she  had  no  assignment  for  the  next  semester.  Oh  was  

   
3H  A.Y.  2017-­‐2018   22  
 
 

CORPORATION  LAW  CASE  DIGESTS  –  ATTY.  DANTE  DELA  CRUZ  

shocked.  She  had  been  teaching  in  the  school  since  1932  for  a  continuous  period  of  almost  33  years.  And  
now,  out  of  the  blue,  and  for  no  apparent  or  given  reason,  this  abrupt  dismissal.  
Oh   sued.   She   demanded   separation   pay,   social   security   benefits,   salary   differentials,   maternity   benefits   and  
moral  and  exemplary  damages.  
The  Court  of  First  Instance  of  Sorsogon  dismissed  the  complaint.    
 
ISSUE/S:    
1.  Whether  or  not  a  school  that  has  not  been  incorporated  may  be  sued  by  reason  alone  of  its  long  continued  
existence  and  recognition  by  the  government,  
2.  Whether  or  not  a  complaint  filed  against  persons  associated  under  a  common  name  will  justify  a  judgment  
against  the  association  itself  and  not  its  individual  members.  
   
HELD:    
We  hold  against  the  petitioner  on  the  first  question.  It  is  true  that  Rule  3,  Section  1,  of  the  Rules  of  Court  
clearly  provides  that  "only  natural  or  juridical  persons  may  be  parties  in  a  civil  action."  It  is  also  not  denied  
that   the   school   has   not   been   incorporated.   However,   this   omission   should   not   prejudice   the   private  
respondent  in  the  assertion  of  her  claims  against  the  school.  
As  a  school,  the  petitioner  was  governed  by  Act  No.  2706  as  amended  by  C.A.  No.  180,  which  provided  as  
follows:  
Unless  exempted  for  special  reasons  by  the  Secretary  of  Public  Instruction,  any  private  school  
or  college  recognized  by  the  government  shall  be  incorporated  under  the  provisions  of  Act  
No.  1459  known  as  the  Corporation  Law,  within  90  days  after  the  date  of  recognition,  and  
shall  file  with  the  Secretary  of  Public  Instruction  a  copy  of  its  incorporation  papers  and  by-­‐‑
laws.  
Having  been  recognized  by  the  government,  it  was  under  obligation  to  incorporate  under  the  Corporation  
Law   within   90   days   from   such   recognition.   It   appears   that   it   had   not   done   so   at   the   time   the   complaint   was  
filed  notwithstanding  that  it  had  been  in  existence  even  earlier  than  1932.  The  petitioner  cannot  now  invoke  
its  own  non-­‐‑compliance  with  the  law  to  immunize  it  from  the  private  respondent's  complaint.  
There  should  also  be  no  question  that  having  contracted  with  the  private  respondent  every  year  for  thirty  
two  years  and  thus  represented  itself  as  possessed  of  juridical  personality  to  do  so,  the  petitioner  is  now  
estopped  from  denying  such  personality  to  defeat  her  claim  against  it.  According  to  Article  1431  of  the  Civil  
Code,  "through  estoppel  an  admission  or  representation  is  rendered  conclusive  upon  the  person  making  it  
and  cannot  be  denied  or  disproved  as  against  the  person  relying  on  it."  
As   the   school   itself   may   be   sued   in   its   own   name,   there   is   no   need   to   apply   Rule   3,   Section   15,   under   which  
the  persons  joined  in  an  association  without  any  juridical  personality  may  be  sued  with  such  association.  
Besides,   it   has   been   shown   that   the   individual   members   of   the   board   of   trustees   are   not   liable,   having   been  
appointed  only  after  the  private  respondent's  dismissal.  
 
 
 
20.  REYNALDO  M.  LOZANO,  LOZANO,  VS  .  HON.  ELIEZER  R.  DE  LOS  SANTOS,  PRESIDING  JUDGE,  RTC,  
BR.  58,  ANGELES  CITY;  AND  ANTONIO  ANDA,  RESPONDENTS.  
G.R.  NO.  125221.  JUNE  19,  1997  
PUNO,  J  
 
DOCTRINE:    
The  jurisdiction  of  the  Securities  and  Exchange  Commission  is  determined  by  a  concurrence  of  two  elements:  
(1)  the  status  or  relationship  of  the  parties;  and  (2)  the  nature  of  the  question  that  is  the  subject  of  their  
controversy.  
 

   
3H  A.Y.  2017-­‐2018   23  
 
 

CORPORATION  LAW  CASE  DIGESTS  –  ATTY.  DANTE  DELA  CRUZ  

The   principal   function   of   the   Securities   and   Exchange   Commission   is   the   supervision   and   control   of  
corporations,   partnerships   and   associations   with   the   end   in   view   that   investments   in   these   entities   may   be  
encouraged  and  protected,  and  their  activities  pursued  for  the  promotion  of  economic  development.  
 
There   is   no   intracorporate   nor   partnership   relation   between   two   jeepney   drivers’   and   operators’  
associations  whose  plan  to  consolidate  into  a  single  common  association  is  still  a  proposal—consolidation  
becomes   effective   not   upon   mere   agreement   of   the   members   but   only   upon   issuance   of   the   certificate   of  
consolidation  by  the  SEC.  
 
The  SEC  has  no  jurisdiction  over  a  dispute  between  members  of  separate  and  distinct  associations.    
 
The  doctrine  of  corporation  by  estoppel  cannot  override  jurisdictional  requirements—jurisdiction  is  fixed  
by  law  and  cannot  be  acquired  through  or  waived,  enlarged  or  diminished  by,  any  act  or  omission  of  the  
parties,  and  neither  can  it  be  conferred  by  the  acquiescence  of  the  court.  
 
Corporation   by   estoppel   is   founded   on   principles   of   equity   and   is   designed   to   prevent   injustice   and  
unfairness,   and   where   there   is   no   third   person   involved   and   the   conflict   arises   only   among   those   assuming  
the  form  of  a  corporation,  who  know  that  it  has  not  been  registered,  there  is  no  corporation  by  estoppel.  
 
FACTS:    
On  December  19,  1995,  Reynaldo  M.  Lozano  filed  civil  case  for  damages  against  Antonio  Anda  before  the  
Municipal  Circuit  Trial  Court  (MCTC),  Mabalacat  and  Magalang,  Pampanga.  Lozano  alleged  that  he  was  the  
president   of   the   Kapatirang   Mabalacat-­‐‑Angeles   Jeepney   Drivers'   Association,   Inc.   (KAMAJDA)   while   Anda  
was   the   president   of   the   Samahang   Angeles-­‐‑Mabalacat   Jeepney   Operators'   and   Drivers'   Association,   Inc.  
(SAMAJODA).  On  August  1995,  upon  the  request  of  the  Sangguniang  Bayan  of  Mabalacat,  Pampanga,  Lozano  
and   Anda   agreed   to   consolidate   their   respective   associations   and   form   the   Unified   Mabalacat-­‐‑Angeles  
Jeepney  Operators'  and  Drivers'  Association,  Inc.  (UMAJODA).  
 
In   the   elections   held   on   October   29,   1995,   both   Lozano   and   Anda   ran   for   president;   Lozano   won;   Anda  
protested  and,  alleging  fraud,  refused  to  recognize  the  results  of  the  election;  Anda  also  refused  to  abide  by  
their   agreement   and   continued   collecting   the   dues   from   the   members   of   his   association   despite   several  
demands  to  desist.    
Lozano  was  thus  constrained  to  file  the  complaint  to  restrain  Anda  from  collecting  the  dues  and  to  order  him  
to   pay   damages   in   the   amount   of   P25,000.00   and   attorney's   fees   of   P500.00.   Anda   moved   to   dismiss   the  
complaint   for   lack   of   jurisdiction,   claiming   that   jurisdiction   was   lodged   with   the   Securities   and   Exchange  
Commission  (SEC).  The  MCTC  denied  the  motion  on  February  9,  1996.  2  It  denied  reconsideration  on  March  
8,  1996.  Private  respondent  filed  a  petition  for  certiorari  before  the  Regional  Trial  Court,  Branch  58,  Angeles  
City.  The  trial  court  found  the  dispute  to  be  intracorporate,  hence,  subject  to  the  jurisdiction  of  the  SEC,  and  
ordered  the  MCTC  to  dismiss  Civil  Case  No.  1214  accordingly.  
ISSUE:  WON  the  dispute  SEC  has  jurisdiction  over  a  case  of  damages  between  heads/presidents  of  two  (2)  
associations  who  intended  to  consolidate/merge  their  associations  but  not  yet  approved  and  registered  with  
the  securities  and  exchange  commission.  
 
HELD:    
No.  The  jurisdiction  of  the  SEC  is  determined  by  a  concurrence  of  two  elements:  (1)  the  status  or  relationship  
of  the  parties;  and  (2)  the  nature  of  the  question  that  is  the  subject  of  their  controversy.  The  first  element  
requires  that  the  controversy  must  arise  out  of  intracorporate  or  partnership  relations  between  and  among  
stockholders,   members,   or   associates;   between   any   or   all   of   them   and   the   corporation,   partnership   or  
association   of   which   they   are   stockholders,   members   or   associates,   respectively;   and   between   such  
corporation,  partnership  or  association  and  the  State  in  so  far  as  it  concerns  their  individual  franchises.  The  

   
3H  A.Y.  2017-­‐2018   24  
 
 

CORPORATION  LAW  CASE  DIGESTS  –  ATTY.  DANTE  DELA  CRUZ  

second  element  requires  that  the  dispute  among  the  parties  be  intrinsically  connected  with  the  regulation  of  
the   corporation,   partnership   or   association   or   deal   with   the   internal   affairs   of   the   corporation,   partnership  
or   association.   After   all,   the   principal   function   of   the   SEC   is   the   supervision   and   control   of   corporations,  
partnerships  and  associations  with  the  end  in  view  that  investments  in  these  entities  may  be  encouraged  and  
protected,  and  their  activities  pursued  for  the  promotion  of  economic  development.    
 
There   is   no   intracorporate   nor   partnership   relation   between   petitioner   and   private   respondent.   The  
controversy   between   them   arose   out   of   their   plan   to   consolidate   their   respective   jeepney   drivers’   and  
operators’   associations   into   a   single   common   association.   This   unified   association   was,   however,   still   a  
proposal.  It  had  not  been  approved  by  the  SEC,  neither  had  its  officers  and  members  submitted  their  articles  
of   consolidation   in   accordance   with   Sections   78   and   79   of   the   Corporation   Code.   Consolidation   becomes  
effective  not  upon  mere  agreement  of  the  members  but  only  upon  issuance  of  the  certificate  of  consolidation  
by   the   SEC.   When   the   SEC,   q   processing   and   examining   the   articles   of   consolidation,   is   satisfied   that   the  
consolidation  of  the  corporations  is  not  inconsistent  with  the  provisions  of  the  Corporation  Code  and  existing  
laws,  it  issues  a  certificate  of  consolidation  which  makes  the  reorganization  official.  The  new  consolidated  
corporation  comes  into  existence  and  the  constituent  corporations  dissolve  and  cease  to  exist.  
 
The  KAMAJ-­‐‑DA  and  SAMAJODA  to  which  petitioner  and  private  respondent  belong  are  duly  registered  with  
the   SEC,   but   these   associations   are   two   separate   entities.   The   dispute   between   petitioner   and   private  
respondent   is   not   within   the   KAMAJDA   nor   the   SAMAJODA.   It   is   between   members   of   separate   and   distinct  
associations.  Petitioner  and  private  respondent  have  no  intracorporate  relation  much  less  do  they  have  an  
intracorporate  dispute.  The  SEC  therefore  has  no  jurisdiction  over  the  complaint.  
 
The   doctrine   of   corporation   by   estoppel   advanced   by   private   respondent   cannot   override   jurisdictional  
requirements.   Jurisdiction   is   fixed   by   law   and   is   not   subject   to   the   agreement   of   the   parties.   It   cannot   be  
acquired  through  or  waived,  enlarged  or  diminished  by,  any  act  or  omission  of  the  parties,  neither  can  it  be  
conferred  by  the  acquiescence  of  the  court.  
 
Corporation   by   estoppel   is   founded   on   principles   of   equity   and   is   designed   to   prevent   injustice   and  
unfairness.  It  applies  when  persons  assume  to  form  a  corporation  and  exercise  corporate  functions  and  enter  
into  business  relations  with  third  persons.  Where  there  is  no  third  person  involved  and  the  conflict  arises  
only  among  those  assuming  the  form  of  a  corporation,  who  therefore  know  that  it  has  not  been  registered,  
there  is  no  corporation  by  estoppel.  
 
 
 
21.  LIM  TONG  LIM  VS.  PHILIPPINE  FISHING  GEAR  INDUSTRIES,  INC.    
G.R.  NO.  136448.  NOVEMBER  3,  1999.  
PANGANIBAN,  J.  
 
Doctrine:   Under   the   law   on   estoppel,   those   acting   on   behalf   of   a   corporation   and   those   benefited   by   it,  
knowing   it   to   be   without   valid   existence,   are   held   liable   as   general   partners.  
 
Facts:  On  February  7,  1990,  Antonio  Chua  and  Peter  Yao  entered  into  a  contract  in  behalf  of  Ocean  Quest  
Fishing   Corporation   (OQFC)   for   the   purchase   of   fishing   nets   from   respondent   Philippine   Fishing   Gear  
Industries,   Inc.   (PFGII).   Chua   and   Yao   claimed   that   they   were   engaged   in   business   venture   with  
petitioner   Lim   Tong   Lim,   who,   however,   was   not   a   signatory   to   the   contract.    
 
The   buyers   failed   to   pay   the   fishing   nets.   Respondent   filed   a   collection   suit   with   a   prayer   of   writ   of  
preliminary  attachment  against  Chua,  Yao  and  petitioner  Lim  in  their  capacities  as  general  partners  because  

   
3H  A.Y.  2017-­‐2018   25  
 
 

CORPORATION  LAW  CASE  DIGESTS  –  ATTY.  DANTE  DELA  CRUZ  

it   turned   out   that   OQFC   is   a   non-­‐‑existent   corporation   as   shown   by   a   certification   from   the   SEC.  
 
RTC  and  CA  RULING:  The  trial  court  rendered  its  decision  ruling  that  respondent  was  entitled  to  the  Writ  of  
Attachment  and  that  Chua,  Yao  and  Lim,  as  general  partners,  were  jointly  liable  to  pay  respondent.  The  trial  
court  ruled  that  a  partnership  among  Lim,  Chua  and  Yao  existed  based  (1)  on  the  testimonies  of  the  witnesses  
presented  and  (2)  on  a  Compromise  Agreement  executed  by  the  three  in  Civil  Case  No.  1492-­‐‑MN  which  Chua  
and   Yao   had   brought   against   Lim.   Lim   appealed   to   CA   which   affirmed   the   decision   of   the   trial   court   that  
petitioner   Lim   is   a   partner   and   may   thus   be   held   liable   as   such.   Hence,   the   present   petition.  
 
ISSUE:   Whether   by   their   acts,   Lim,   Chua   and   Yao   could   be   deemed   to   have   entered   into   a   partnership?  
 
Held:   Yes.   Petitioner   claimed   that   since   his   name   did   not   appear   on   any   of   the   contracts   and   since   he   never  
directly   transacted   with   the   respondent   corporation,   ergo,   he   cannot   be   held   liable.  
In   arguing   that   he   should   not   be   held   liable   for   the   equipment   purchased   from   respondent,   petitioner  
controverts  the  CA  finding  that  a  partnership  existed  between  them.  He  asserts  that  the  CA  based  its  finding  
on  the  Compromise  Agreement  alone.  Furthermore,  he  disclaims  any  direct  participation  in  the  purchase  of  
the  nets,  alleging  that  the  negotiations  were  conducted  by  Chua  and  Yao  only,  and  that  he  has  not  even  met  
the  representatives  of  the  respondent  company.  Petitioner  further  argues  that  he  was  a  lessor,  not  a  partner,  
of  Chua  and  Yao,  for  the  "Contract  of  Lease”  which  showed  that  he  had  merely  leased  to  the  two  the  main  
asset  of  the  purported  partnership   —  the  fishing  boat  F/B  Lourdes.  The  lease  was  for  six  months,  with  a  
monthly   rental   of   P37,500   plus   25%   of   the   gross   catch   of   the   boat.

We  are  not  persuaded  by  the  arguments  of  petitioner.  The  facts  as  found  by  the  two  lower  courts  clearly  
showed  that  there  existed  a  partnership  among  Chua,  Yao  and  him,  pursuant  to  Article  1767  of  the  Civil  Code  
which  provides:  
"ARTICLE  1767.  By  the  contract  of  partnership,  two  or  more  persons  bind  themselves  to  contribute  
money,   property,   or   industry   to   a   common   fund,   with   the   intention   of   dividing   the   profits   among  
themselves."  
From  the  factual  findings  of  both  lower  courts,  it  is  clear  that  the  three  had  decided  to  engage  in  a  fishing  
business,  which  they  started  by  buying  boats  worth  P3.35m,  financed  by  a  loan  secured  from  Jesus  Lim  who  
was  petitioner's  brother.  In  their  Compromise  Agreement,  they  subsequently  revealed  their  intention  to  pay  
the  loan  with  the  proceeds  of  the  sale  of  the  boats,  and  to  divide  equally  among  them  the  excess  or  loss.  These  
boats,   the   purchase   and   the   repair   of   which   were   financed   with   borrowed   money,   fell   under   the   term  
"common  fund"  under  Article  1767.  The  contribution  to  such  fund  need  not  be  cash  or  fixed  assets;  it  could  
be   an   intangible   like   credit   or   industry.   That   the   parties   agreed   that   any   loss   or   profit   from   the   sale   and  
operation   of   the   boats   would   be   divided   equally   among   them   also   shows   that   they   had   indeed   formed   a  
partnership.  Moreover,  it  is  clear  that  the  partnership  extended  not  only  to  the  purchase  of  the  boat,  but  also  
to  that  of  the  nets  and  the  floats.  The  fishing  nets  and  the  floats,  both  essential  to  fishing,  were  obviously  
acquired   in   furtherance   of   their   business.   It   would   have   been   inconceivable   for  Lim  to  involve  himself  so  
much  in  buying  the  boat  but  not  in  the  acquisition  of  the  aforesaid  equipment,  without  which  the  business  
could  not  have  proceeded.  Given  the  preceding  facts,  it  is  clear  that  there  was,  among  petitioner,  Chua  and  
Yao,  a  partnership  engaged  in  the  fishing  business.  They  purchased  the  boats,  which  constituted  the  main  
assets  of  the  partnership,  and  they  agreed  that  the  proceeds  from  the  sales  and  operations  thereof  would  be  
divided  among  them.||  
 
In  implying  that  the  lower  courts  have  decided  on  the  basis  of  one  piece  of  document  alone  (Compromise  
agreement),  petitioner  fails  to  appreciate  that  the  CA  and  the  RTC  delved  into  the  history  of  the  document  
and  explored  all  the  possible  consequential  combinations  in  harmony  with  law,  logic  and  fairness.  Verily,  the  
two  lower  courts'  factual  findings  mentioned  above  nullified  petitioner's  argument  that  the  existence  of  a  
partnership  was  based  only  on  the  Compromise  Agreement.||  

   
3H  A.Y.  2017-­‐2018   26  
 
 

CORPORATION  LAW  CASE  DIGESTS  –  ATTY.  DANTE  DELA  CRUZ  

 
Petitioner   was   a   partner   and   not   a   lessor.   Petitioner   entered   into   a   business   agreement   with   Chua   and   Yao,  
in   which   debts   were   undertaken   in   order   to   finance   the   acquisition   and   the   upgrading   of   the   vessels   which  
would  be  used  in  their  fishing  business.  The  sale  of  the  boats,  as  well  as  the  division  among  the  three  of  the  
balance  remaining  after  the  payment  of  their  loans,  proves  beyond  cavil  that  F/B  Lourdes,  though  registered  
in  his  name,  was  not  his  own  property  but  an  asset  of  the  partnership.  It  is  not  uncommon  to  register  the  
properties  acquired  from  a  loan  in  the  name  of  the  person  the  lender  trusts,  who  in  this  case  is  the  petitioner  
himself.  After  all,  he  is  the  brother  of  the  creditor,  Jesus  Lim.  We  stress  that  it  is  unreasonable  —  indeed,  it  is  
absurd  —  for  petitioner  to  sell  his  property  to  pay  a  debt  he  did  not  incur,  if  the  relationship  among  the  three  
of  them  was  merely  that  of  lessor-­‐‑lessee,  instead  of  partners.  
 
Lastly  and  most  importantly,  there  is  no  dispute  that  the  respondent,  PFGII,  is  entitled  to  be  paid  for  the  nets.  
The   only   question   here   is   whether   petitioner   should   be   held   jointly   liable   with   Chua   and   Yao.   Petitioner  
contests   such   liability,   insisting   that   only   those   who   dealt   in   the   name   of   the   ostensible   corporation   should  
be  held  liable.  Since  his  name  does  not  appear  on  any  of  the  contracts  and  since  he  never  directly  transacted  
with  the  respondent  corporation,  ergo,  he  cannot  be  held  liable.  Unquestionably,  petitioner  benefited  from  
the  use  of  the  nets  found  inside  F/B  Lourdes,  the  boat  which  has  earlier  been  proven  to  be  an  asset  of  the  
partnership.  He  in  fact  questions  the  attachment  of  the  nets,  because  the  Writ  has  effectively  stopped  his  use  
of  the  fishing  vessel.  It  is  difficult  to  disagree  with  the  RTC  and  the  CA  that  Lim,  Chua  and  Yao  decided  to  form  
a  corporation.  Although  it  was  never  legally  formed  for  unknown  reasons,  this  fact  alone  does  not  preclude  
the  liabilities  of  the  three  as  contracting  parties  in  representation  of  it.  Clearly,  under  the  law  on  estoppel,  
those  acting  on  behalf  of  a  corporation  and  those  benefited  by  it,  knowing  it  to  be  without  valid  existence,  
are   held   liable   as   general   partners.   Technically,   it   is   true   that   petitioner   did   not   directly   act   on   behalf   of   the  
corporation.   However,   having   reaped   the   benefits   of   the   contract   entered   into   by   persons   with   whom   he  
previously  had  an  existing  relationship,  he  is  deemed  to  be  part  of  said  association  and  is  covered  by  the  
scope  of  the  doctrine  of  corporation  by  estoppel.  
 
 
 
22.  INTERNATIONAL  EXPRESS  TRAVEL  &  TOUR  SERVICES,  INC  VS.  HON.  COURT  OF  APPEALS,  HENRI  
KAHN,  PHILIPPINES  FOOTBALL  FEDERATION  
G.R.  NO.  119020  ;  G.R.  NO.  119020  
KAPUNAN,  J.  
 
DOCTRINE:  Any  person  acting  or  purporting  to  act  on  behalf  of  a  corporation  which  has  no  valid  existence  
assumes  such  privileges  and  becomes  personally  liable  for  contract  entered  into  or  for  other  acts  performed  
as  such  agent  
 
FACTS:       On   June   30,   1989,   petitioner   International   Express   Travel   and   Tours   Services   Inc.,   through   its  
managing  director,  wrote  a  letter  to  the  Philippine  Football  Federation  (Federation)  through  its  President  
Henri   Kahn,   wherein   the   former  offered   its   services   as   a   travel   agency   to   the   latter.   The   offer   was   accepted.  
Petitioner  secured  the  airline  tickets  for  the  trips  of  the  athletes  and  officials  of  the  Federation  to  the  South  
East   Asian   Games   in   Kuala   Lumpur   as   well   as   various   other   trips   to   the   People’s   Republic   of   China   and  
Brisbane.   The   total   cost   of   the   tickets   amounted   to   Php449,654.83.   For   the   tickets   received,   the   Federation  
made  two  partial  payments,  both  in  September  of  1989  in  the  total  amount  of  Php176,467.50.  On  October  4,  
1989,  petitioner  wrote  the  Federation,  through  the  private  respondent  a  demand  letter  requesting  for  the  
amount  of  Php265,844.33.  On  October  30,  1989,  the  Federation,  through  the  project  gintong  alay,  paid  the  
amount   of   Php31,603.   On   December   27,   1989,   Henri   Kahn   issued   a   personal   check   in   the   amount   of  
Php50,000  as  partial  payment  for  the  outstanding  balance  of  the  Federation.  Thereafter,  no  further  payments  
were  made  despite  repeated  demands.  Hence,  this  petition.  

   
3H  A.Y.  2017-­‐2018   27  
 
 

CORPORATION  LAW  CASE  DIGESTS  –  ATTY.  DANTE  DELA  CRUZ  

 
The  trial  court  rendered  judgment  and  ruled  in  favor  of  the  petitioner  and  declared  Henri  Kahn  personally  
liable  for  the  unpaid  obligation  of  the  Federation,  rationalizing  that  there  was  no  evidence  adduced  to  prove  
the  corporate  existence  of  the  Philippine  Football  Federation.  
In   finding   for   Henri   Kahn,   the   Court   of   Appeals   recognized   the   juridical   existence   of   the   Federation.   It  
rationalized  that  since  petitioner  failed  to  prove  that  Henri  Kahn  guaranteed  the  obligation  of  the  Federation,  
he  should  not  be  held  liable  for  the  same  as  said  entity  has  a  separate  and  distinct  personality  from  its  officers.  
ISSUES:      
1.   Whether  or  not  the  Federation  is  a  corporation  
2.   Whether  or  not  Henri  Kahn  may  be  held  personaly  liable  for  the  liablilities  of  the  Federation  
 
HELD:  (1)  NO,  while  we  agree  with  the  appellate  court  that  national  sports  associations  may  be  accorded  
corporate   status,   such   does   not   automatically   take   place   by   the   mere   passage   of   these   laws.     It   is   a   basic  
postulate   that   before   a   corporation   may   acquire   juridical   personality,   the   State   must   give   its   consent   either  
in  the  form  of  a  special  law  or  a  general  enabling  act.  We  cannot  agree  that  the  Philippine  Football  Federation  
came  into  existence  upon  the  passage  of  these  laws.  Nowhere  can  it  be  found  in  R.A.  3135  or  P.D.  604  any  
provision  creating  the  Philippine  Football  Federation.  These  laws  merely  recognized  the  existence  of  national  
sports  associations  and  provided  the  manner  by  which  these  entities  may  acquire  juridical  personality.    
This   fact   of   recognition,   however,   Henri   Kahn   failed   to   substantiate.   In   attempting   to   prove   the   juridical  
existence  of  the  Federation,  Henri  Kahn  attached  to  his  motion  for  reconsideration  before  the  trial  court  a  
copy  of  the  constitution  and  by-­‐‑laws  of  the  Philippine  Football  Federation.  Unfortunately,  the  same  does  not  
prove   that   said   Federation   has   indeed   been   recognized   and   accredited   by   either   the   Philippine   Amateur  
Athletic   Federation   or   the   Department   of   Youth   and   Sports   Development.   Accordingly,   we   rule   that   the  
Philippine  Football  Federation  is  not  a  national  sports  association  within  the  purview  of  the  aforementioned  
laws  and  does  not  have  corporate  existence  of  its  own    
(2)  YES,  as  the  Federation  is  not  a  corporation,    it  follows  that  private  respondent  Henry  Kahn  should  be  held  
liable  for  the  unpaid  obligations  of  the  unincorporated  Philippine  Football  Federation.  It  is  a  settled  principle  
in  corporation  law  that  any  person  acting  or  purporting  to  act  on  behalf  of  a  corporation  which  has  no  valid  
existence   assumes   such   privileges   and   becomes   personally   liable   for   contract   entered   into   or   for   other   acts  
performed   as   such   agent.  As   president   of   the   Federation,   Henri   Kahn   is   presumed   to   have   known   about   the  
corporate  existence  or  non-­‐‑existence  of  the  Federation.    
 
 
 
23.  LOYOLA  GRAND  VILLAS  HOMEOWNERS  (SOUTH)  ASSOCIATION,  INC  VS.  CA  
G.R.  NO.  117188.  AUGUST  7,  1997.  
ROMERO,  J  
 
DOCTRINE:  
Failure  to  file  the  by-­‐‑laws  within  any  period  does  not  imply  the  "demise"  of  the  corporation.  The  failure  to  
exercise  the  power  will  be  ascribed  to  mere  non-­‐‑action  which  will  not  render  void  any  acts  of  the  corporation  
which  would  otherwise  be  valid.  By-­‐‑laws  are  indispensable  to  corporations  in  this  jurisdiction  Nonetheless,  
failure   to   file   them   within   the   period   required   by   law   by   no   means   tolls   the   automatic   dissolution   of   a  
corporation.  Incorporators  must  be  given  the  chance  to  explain  their  neglect  or  omission  and  remedy  the  
same.  
 
FACTS:  
LGVHAI  was  organized  on  February  8,  1983  as  the  association  of  homeowners  and  residents  of  the  Loyola  
Grand   Villas.   It   was   registered   with   the   Home   Financing   Corporation,   (predecessor   of   HIGC),   as   the   sole  
homeowners'  organization  in  the  said  subdivision.  It  was  organized  by  the  developer  of  the  subdivision  and  

   
3H  A.Y.  2017-­‐2018   28  
 
 

CORPORATION  LAW  CASE  DIGESTS  –  ATTY.  DANTE  DELA  CRUZ  

its   first   president   was   Victorio   V.   Soliven,   himself   the   owner   of   the   developer.   For   unknown   reasons,  
however,  LGVHAI  did  not  file  its  corporate  by-­‐‑laws.  Sometime  in  1988,  the  officers  of  the  LGVHAI  tried  to  
register  its  by-­‐‑laws.  They  failed  to  do  so.  'To  the  officers'  consternation,  they  discovered  that  there  were  two  
other  organizations  within  the  subdivision  —  the  North  Association  and  the  South  Association.    In  July,  1989,  
when  Soliven  inquired  about  the  status  of  LGVHAI,  the  head  of  the  legal  department  of  the  HIGC,  informed  
him  that  LGVHAI  had  been  automatically  dissolved  because:  1.  It  did  not  submit  its  by-­‐‑laws  within  the  period  
required  by  the  Corporation  Code;  and  2.  There  was  non-­‐‑user  of  corporate  charter  because  HIGC  had  not  
received  any  report  on  the  association's  activities.  This  information  resulted  in  the  registration  of  the  South  
Association  with  the  HIGC.  These  developments  prompted  the  officers  of  the  LGVHAI  to  lodge  a  complaint  
with  the  HIGC.  They  questioned  the  revocation  of  LGVHAI  's  certificate  of  registration  without  due  notice  and  
hearing   and   concomitantly   prayed   for   the   cancellation   of   the   certificates   of   registration   of   the   North   and  
South  Associations  by  reason  of  the  earlier  issuance  of  a  certificate  of  registration  in  favor  of  LGVHAI.  After  
due  notice  and  hearing,  private  respondents  obtained  a  favorable  ruling  from  HIGC  Hearing  Officer  Danilo  C.  
Javier  .  The  South  Association  appealed  to  the  Appeals  Board  of  the  HIGC,  which  dismissed  the  appeal  for  lack  
of   merit.   Rebuffed,   the   South   Association   in   turn   appealed   to   the   Court   of   Appeals,   which   affirmed   the  
Resolution  of  the  HIGC  Appeals  Board.  Undaunted,  the  South  Association  filed  the  instant  petition  for  review  
on  certiorari.  
 
ISSUE:  
Whether  or  not  the  LGVHAI's  failure  to  file  its  by-­‐‑   laws  within  the  period  prescribed  by  Section  46  of  the  
Corporation  Code  had  the  effect  of  automatically  dissolving  the  said  corporation.  
 
HELD:  
NO.  Automatic  corporate  dissolution  for  failure  to  file  the  by-­‐‑laws  on  time  was  never  the  intention  of  the  
legislature.  Section  46  aforequoted  reveals  the  legislative  intent  to  attach  a  directory,  and  not  mandatory,  
meaning   for   the   word   ''must"   in   the   first   sentence   thereof.   By-­‐‑laws   may   be   necessary   for   the   "government"  
of  the  corporation  but  these  are  subordinate  to  the  articles  of  incorporation  as  well  as  to  the  Corporation  
Code   and   related   statutes.  There   are   in   fact   cases   where   by-­‐‑laws   are   unnecessary   to   corporate   existence   or  
to  the  valid  exercise  of  corporate  powers.  There  is  no  outright  "demise"  of  corporate  existence.  Proper  notice  
and  hearing  are  cardinal  components  of  due  process  in  any  democratic  institution,  agency  or  society.  In  other  
words,  the  incorporators  must  be  given  the  chance  to  explain  their  neglect  or  omission  and  remedy  the  same.  
 
 
 
24.  HENRY  FLEISCHER  V.  BOTICA  NOLASCO  CO.  INC.  
G.R.  NO.  L-­‐‑23241                          MARCH  14,  1925  
JOHNSON,  J.  
 
 
FACTS:  
•   Manuel   Gonzales   delivered   and   assigned   his   5   shares   of   stock   of   the   respondent   corporation   to  
petitioner  Henry  Fleischer.  
•   Dr.  Eduardo  Miciano,  who  was  the  secretary-­‐‑treasurer  of  said  corporation,  offered  to  buy  from  Henry  
Fleischer,  on  behalf  of  the  corporation,  said  shares  of  stock,  at  their  par  value  of  P100  a  share,  for  
P500.  
•   According  to  Dr.  Miciano,  by  virtue  of  article  12  of  the  by-­‐‑laws  of  Botica  Nolasco,  Inc.,  said  corporation  
had  the  preferential  right  to  buy  from  Manuel  Gonzalez  said  share.  
•   Plaintiff  refused  to  sell  them  to  the  defendant  so  he  requested  Doctor  Miciano  to  register  said  shares  
in  his  name  which  the  secretary  refused  to  do  so,  saying  that  it  would  be  in  contravention  of  the  by-­‐‑
laws  of  the  corporation.  

   
3H  A.Y.  2017-­‐2018   29  
 
 

CORPORATION  LAW  CASE  DIGESTS  –  ATTY.  DANTE  DELA  CRUZ  

•   The  latter  part  of  the  said  Article  of  the  By  laws  creates  in  favor  of  the  Botica,  a  preferential  right  to  
buy  the  share/s  of  stock/s  of  a  retiring  shareholder.  
 
ISSUE:  Whether  the  corporation  has  any  power  to  adopt  such  provision  on  its  by  laws.  
 
HELD:  
•   Section  13,  par.  7,  empowers  a  corporation  to  make  by-­‐‑laws,  not  inconsistent  with  any  existing  law,  
for  the  transferring  of  its  stock.  
•   It   follows   that   a   by-­‐‑law   adopted   by   a   corporation   relating   to   transfer   of   stock   should   be   in   harmony  
with  the  law  on  the  subject  of  transfer  of  stock.    
•   Section   35   specifically   provides   that   the   shares   of   stock   "are   personal   property   and   may   be  
transferred  by  delivery  of  the  certificate  indorsed  by  the  owner,  etc."    
•   Said   section   35   defines   the   nature,   character   and   transferability   of   shares   of   stock,   stating   that   they  
are  personal  property  and  may  be  transferred  as  therein  provided.    
•   It  contemplates  no  restriction  as  to  whom  they  may  be  transferred  or  sold.    
•   It  does  not  suggest  that  any  discrimination  may  be  created  by  the  corporation  in  favor  or  against  a  
certain  purchaser.    
•   The  holder  of  shares,  as  owner  of  personal  property,  is  at  liberty,  under  said  section,  to  dispose  of  
them  in  favor  of  whomsoever  he  pleases,  without  any  other  limitation  in  this  respect,  than  the  general  
provisions  of  law.    
•   Therefore,  a  stock  corporation  in  adopting  a  by-­‐‑law  governing  transfer  of  shares  of  stock  should  take  
into  consideration  the  specific  provisions  of  section  35  of  Act  No.  1459,  and  said  by-­‐‑law  should  be  
made  to  harmonize  with  said  provisions.    
•   It  should  not  be  inconsistent  therewith.  
 
 
 
25.  THE  GOVERNMENT  OF  THE  PHILIPPINE  ISLANDS  VS.  EL  HOGAR  FILIPINO  
G.R.  NO.  L-­‐‑26649;  JULY  13,  1927  
STREET,  J.:  
 
DOCTRINES:    
•   While   the   Constitution   and   the   statutes   provide   that   no   corporation   shall   engage   in   any   business  
other   than   that   expressly   authorized   by   its   charter,   we   are   of   opinion   that,   in   renting   out   the  
unoccupied   and   unused   portions   of   the   building   so   erected,   the   association   could   not   be   said   to  
engaged  in  any  other  business  than  that  authorized  by  its  charter.  The  renting  of  the  unused  portions  
of  the  building  is  a  mere  incident  in  the  conduct  of  its  real  business.  (2nd  COA)  
•   Unless  the  law  or  the  charter  of  a  corporation  expressly  provides  that  an  office  shall  become  vacant  
at  the  expiration  of  the  term  of  office  for  which  the  officer  was  elected,  the  general  rule  is  to  allow  the  
officer  to  holdover  until  his  successor  is  duly  qualified.  (5th  COA)  
•   It  is  true  that  the  corporation  law  does  not  expressly  grant  this  power  to  maintain  these  reserves,  but  
we  think  it  is  to  be  implied.  The  government  insists,  we  thing,  upon  an  interpretation  of  section  188  
of  the  Corporation  Law  that  is  altogether  too  strict  and  literal.  But  it  will  be  noted  that  it  is  provided  
in  the  same  section  that  the  profits  and  losses  shall  be  determined  by  the  board  of  directors  and  this  
means  that  they  shall  exercise  the  usual  discretion  of  good  businessmen  in  allocating  a  portion  of  the  
annual  profits  to  purposes  needful  to  the  welfare  of  the  association.  (11th  and  12th  COA)  
•   There  is  no  statute  here  expressly  declaring  that  loans  may  be  made  by  these  associations  solely  for  
the  purpose  of  building  homes.  On  the  contrary,  the  building  of  homes  is  mentioned  in  section  171  of  
the   Corporation   Law   as   only   one   among   several   ends   which   building   and   loan   associations   are  

   
3H  A.Y.  2017-­‐2018   30  
 
 

CORPORATION  LAW  CASE  DIGESTS  –  ATTY.  DANTE  DELA  CRUZ  

designed   to   promote.   Furthermore,   section   181   of   the   Corporation   Law   expressly   authorities   the  
Board  of  directors  of  the  association  from  time  to  time  to  fix  the  premium  to  be  charged.  (13th  COA)  
FACTS:  
This   is   a   quo   warranto   proceeding   instituted   originally   in   this   court   by   the   Government   of   the   Philippine  
Islands  on  the  relation  of  the  Attorney-­‐‑General  against  the  building  and  loan  association  known  as  El  Hogar  
Filipino.   On   March   1,   1906,   the   Philippine   Commission   enacted   what   is   known   as   the   Corporation   Law   (Act  
No.  1459)  effective  upon  April  1  1906.  Section  171  to  190,  inclusive,  of  this  Act  are  devoted  to  the  subject  of  
building  and  loan  associations,  defining  their  objects  making  various  provisions  governing  their  organization  
and   administration,   and   providing   for   the   supervision   to   be   exercised   over   them.   The   respondent,   El   Hogar  
Filipino,  was  apparently  the  first  corporation  organized  in  the  Philippine  Islands  under  the  provisions  cited.    
   
HELD:  
First  cause  of  action.  —  The  first  cause  of  action  is  based  upon  the  alleged  illegal  holding  by  the  respondent  
of   the   title   to   real   property   for   a   period   in   excess   of   five   years   after   the   property   had   been   bought   in   by   the  
respondent  at  one  of  its  own  foreclosure  sales.  The  provision  of  law  relevant  to  the  matter  is  found  in  section  
75  of  Act  of  Congress  of  July  1,  1902  (repeated  in  subsection  5  of  section  13  of  the  Corporation  Law.)    
The  Attorney-­‐‑General  points  out  that  the  respondent  acquired  title  on  December  22,  1920,  when  the  
deed   was   executed   and   delivered,   by   which   the   property   was   conveyed   to   it   as   purchaser   at   its   foreclosure  
sale,  and  this  title  remained  in  it  until  July  30,  1926,  when  the  property  was  finally  sold  to  Felipa  Alberto.  The  
interval   between   these   two   conveyances   is   thus   more   than   five   years;   and   it   is   contended   that   the   five   year  
period  did  not  begin  to  run  against  the  respondent  until  May  7,  1921,  when  the  register  of  deeds  of  Tarlac  
delivered   the   new   certificate   of   title   to   the   respondent   pursuant   to   the   deed   by   which   the   property   was  
acquired.   It   results   that   prior   to   May   7,   1921,   El   Hogar   Filipino   was   not   really   in   a   position   to   pass   an  
indefeasible   title   to   any   purchaser.   In   this   connection   it   will   be   noted   that   section   75   of   the   Act   of   Congress  
of  July  1,  1902,  and  the  similar  provision  in  section  13  of  the  Corporation  Law,  allow  the  corporation  "five  
years   after   receiving   the   title,"   within   which   to   dispose   of   the   property.   The   failure   of   the   respondent   to  
receive  the  certificate  sooner  was  not  due  in  any  wise  to  its  fault,  but  to  unexplained  delay  on  the  part  of  the  
register  of  deeds.  Again,  it  is  urged  for  the  respondent  that  the  period  between  March  25,  1926,  and  April  30,  
1926,  should  not  be  counted  as  part  of  the  five-­‐‑year  period.  This  was  the  period  during  which  the  respondent  
was  under  obligation  to  sell  the  property  to  Alcantara,  prior  to  the  rescission  of  the  contract  by  reason  of  
Alcantara's  failure  to  make  the  stipulated  first  payment.  Upon  this  point  the  contention  of  the  respondent  is,  
in  our  opinion,  well  founded.  Nonetheless,  even  supposing  the  five-­‐‑year  period  to  be  properly  counted  from  
that  date,  it  is  in  in  our  opinion  that  the  corporation  has  not  been  shown  to  have  offended  against  the  law  in  
a  manner  that  should  entail  a  forfeiture  of  its  charter.  section  212  of  the  Code  of  Civil  Procedure  clearly  shows  
that  the  court  has  a  discretion  with  respect  to  the  infliction  of  capital  punishment  upon  corporation  and  that  
there  are  certain  misdemeanors  and  misuses  of  franchises  which  should  not  be  recognized  as  requiring  their  
dissolution.    
But  the  case  for  the  plaintiff  supposes  that  the  discretion  of  this  court  in  matters  like  that  now  before  
us   has   been   expressly   taken   away   by   the   third   section   of   Act   No.   2792,   and   that   the   dissolution   of   the  
corporation  is  obligatory  upon  the  court  a  mere  finding  that  the  respondent  has  violated  the  provision  of  the  
Corporation  Law  in  any  respect.  The  third  section  contains  anew  enactment  to  be  inserted  as  section  190  (A)  
in  the  corporation  Law  immediately  following  section  190.  The  contention  for  the  plaintiff  is  to  the  effect  that  
the  second  sentence  in  this  enactment  has  entirely  abrogated  the  discretion  of  this  court  with  respect  to  the  
application  of  the  remedy  of  qou  warranto,  as  expressed  in  section  212  of  the  Code  of   Civil  Procedure,  and  
that  it  is  now  mandatory  upon  us  to  dissolved  any  corporation  whenever  we  find  that  it  has  committed  any  
violation  of  the  Corporation  Law,  however  trivial.  In  our  opinion  in  this  radical  view  of  the  meaning  of  the  
enactment  is  untenable.  When  the  statute  says,  "If  the  violation  is  committed  by  a  corporation,  the  same  shall,  
upon   such   violation   being   proved,   be   dissolved   by   quo   warranto   proceedings   .   .   .,"   the   intention   was   to  
indicate  that  the  remedy  against  the  corporation  shall  be  by  action  of  quo  warranto.  There  was  no  intention  
to  define  the  principles  governing  said  remedy,  and  it  must  be  understood  that  in  applying  the  remedy  the  

   
3H  A.Y.  2017-­‐2018   31  
 
 

CORPORATION  LAW  CASE  DIGESTS  –  ATTY.  DANTE  DELA  CRUZ  

court  is  still  controlled  by  the  principles  established  in  immemorial  jurisprudence.  Another  way  to  put  the  
same  conclusion  is  to  say  that  the  expression  "shall  be  dissolved  by  quo  warranto  proceedings"  means  in  
effect,   "may   be   dissolved   by   quo   warranto   proceedings   in   the   discretion   of   the   court."   The   proposition   that  
the   word   "shall"   may   be   construed   as   "may",   when   addressed   by   the   Legislature   to   the   courts,   is   well  
supported  in  jurisprudence.    
   
Second  cause  of  action.  —  The  second  cause  of  action  is  based  upon  a  charge  that  the  respondent  is  owning  
and  holding  a  business  lot,  with  the  structure  thereon,  in  the  financial  district  of  the  City  of  Manila  is  excess  
of  its  reasonable  requirements  and  in  contravention  of  subsection  5  of  section  13  of  the  corporation  Law.  
The  facts  on  which  this  charge  is  based  appear  to  be  these:  
  Under   subsection   5   of   section   13   of   the   Corporation   Law,   every   corporation   has   the   power   to  
purchase,  hold  and  lease  such  real  property  as  the  transaction  of  the  lawful  business  of  the  corporation  may  
reasonably   and   necessarily   require.   When   this   property   was   acquired   in   1916,   the   business   of   El   Hogar  
Filipino  had  developed  to  such  an  extent,  and  its  prospects  for  the  future  were  such  as  to  justify  its  directors  
in  acquiring  a  lot  in  the  financial  district  of  the  City  of  Manila  and  in  constructing  thereon  a  suitable  building  
as  the  site  of  its  offices;  and  it  cannot  be  fairly  said  that  the  area  of  the  lot  —  1,413  square  meters  —  was  in  
excess   of   its   reasonable   requirements.   A   different   ruling   on   this   point   would   compel   important   enterprises  
to  conduct  their  business  exclusively  in  leased  offices  —  a  result  which  could  serve  no  useful  end  but  would  
retard   industrial   growth   and   be   inimical   to   the   best   interests   of   society.   We   are   furthermore   of   the   opinion  
that,   inasmuch   as   the   lot   referred   to   was   lawfully   acquired   by   the   respondent,   it   is   entitled   to   the   full  
beneficial   use   thereof.   as   was   said   in   People   vs.   Pullman's   Palace   Car   Co.,   supra,   the   corporation   should   not  
necessarily   be   restricted   to   a   building   containing   the   precise   number   of   rooms   its   then   business   might  
require,  and  no  more,  but  that  the  future  probable  growth  and  volume  of  its  business  might  be  considered  
and  anticipated,  and  a  larger  building,  and  one  containing  more  rooms  than  the  present  volume  of  business  
required   be   erected,   and   the   rooms   not   needed   might   be   rented   by   the   corporation,   —   provided,   of   course,  
such   course   should   be   taken   in   good   faith,   and   not   as   a   mere   evasion   of   the   public   law   and   the   policy   of   the  
state  relative  to  the  ownership  of  real  estate  by  corporations.  The  limitation  which  the  statute  imposes  is  
that  proper  conduct  of  its  business,  but  it  does  not  attempt  to  place  any  restriction  or  limitation  upon  the  
right  of  the  corporation  or  association  as  to  the  character  of  building  it  shall  erect  on  said  real  estate;  and,  
while  the  Constitution  and  the  statutes  provide  that  no  corporation  shall  engage  in  any  business  other  than  
that  expressly  authorized  by  its  charter,  we  are  of  opinion  that,  in  renting  out  the  unoccupied  and  unused  
portions   of   the   building   so   erected,   the   association   could   not   be   said   to   engaged   in   any   other   business   than  
that  authorized  by  its  charter.  The  renting  of  the  unused  portions  of  the  building  is  a  mere  incident  in  the  
conduct  of  its  real  business.    
   
Third  cause  of  action.  —  Under  the  third  cause  of  action  the  respondent  is  charged  with  engaging  in  activities  
foreign  to  the  purposes  for  which  the  corporation  was  created  and  not  reasonable  necessary  to  its  legitimate  
ends.  The  specifications  under  this  cause  of  action  relate  to  three  different  sorts  of  activities.  The  first  consist  
of  the  administration  of  the  offices  in  the  El  Hogar  building  not  used  by  the  respondent  itself  and  the  renting  
of   such   offices   to   the   public.   The   activities   here   criticized   clearly   fall   within   the   legitimate   powers   of   the  
respondent,  as  shown  in  what  we  have  said  above  relative  to  the  second  cause  of  action.    
The   second   specification   under   the   third   cause   of   action   has   reference   to   the   administration   and  
management  of  properties  belonging  to  delinquent  shareholders  of  the  association  pursuant  to  clause  8  of  
its  standard  mortgage.  For  these  services  the  respondent  charges  a  commission.  The  case  for  the  government  
supposes  that  the  only  remedy  which  the  respondent  has  in  case  of  default  on  the  part  of  its  shareholders  is  
to   proceed   to   enforce   collection   of   the   whole   loan   in   the   manner   contemplated   in   section   185   of   the  
Corporation   Law.   It   will   be   noted,   however,   that,   according   to   said   section,   the   association   may   treat   the  
whole  indebtedness  as  due,  "at  the  option  of  the  board  of  directors,"  and  this  remedy  is  not  made  exclusive.  
We  see  no  reason  to  doubt  the  validity  of  the  clause  giving  the  association  the  right  to  take  over  the  property  
which  constitutes  the  security  for  the  delinquent  debt  and  to  manage  it  with  a  view  to  the  satisfaction  of  the  

   
3H  A.Y.  2017-­‐2018   32  
 
 

CORPORATION  LAW  CASE  DIGESTS  –  ATTY.  DANTE  DELA  CRUZ  

obligations  due  to  the  debtor  than  the  immediate  enforcement  of  the  entire  obligation,  and  the  validity  of  the  
clause  allowing  this  course  to  be  taken  appears  to  us  to  be  not  open  to  doubt.    
The   third   specification   under   this   cause   of   action   relates   to   certain   activities   which   are   described   in   the  
following   paragraphs   contained   in   the   agreed   statements   of   facts:.   El   Hogar   Filipino   has   undertaken   the  
management  of  some  parcels  of  improved  real  estate  not  under  mortgage  to  it,  but  owned  by  shareholders,  
and   has   held   itself   out   by   advertisement   as   prepared   to   do   so.   For   the   services   so   rendered   in   the  
management  of  such  properties  El  Hogar  Filipino  receives  compensation  in  the  form  of  commissions  upon  
the  gross  receipts  from  such  properties.  The  administration  of  property  in  the  manner  described  is  more  
befitting   to   the   business   of   a   real   estate   agent   or   trust   company   than   to   the   business   of   a   building   and   loan  
association.  It  is  a  general  rule  of  law  that  corporations  possess  only  such  express  powers.  The  management  
and  administration  of  the  property  of  the  shareholders  of  the  corporation  is  not  expressly  authorized  by  law,  
and  we  are  unable  to  see  that,  upon  any  fair  construction  of  the  law,  these  activities  are  necessary  to  the  
exercise  of  any  of  the  granted  powers.  The  corporation,  upon  the  point  now  under  the  criticism,  has  clearly  
extended  itself  beyond  the  legitimate  range  of  its  powers.  But  it  does  not  result  that  the  dissolution  of  the  
corporation  is  in  order,  and  it  will  merely  be  enjoined  from  further  activities  of  this  sort.  
   
Fourth  cause  of  action.  —  It  appears  that  among  the  by  laws  of  the  association  there  is  an  article  (No.  10)  
which  reads  as  follows:The  board  of  directors  of  the  association,  by  the  vote  of  an  absolute  majority  of  its  
members,  is  empowered  to  cancel  shares  and  to  return  to  the  owner  thereof  the  balance  resulting  from  the  
liquidation   thereof   whenever,   by   reason   of   their   conduct,   or   for   any   other   motive,   the   continuation   as  
members  of  the  owners  of  such  shares  is  not  desirable.  
This  by-­‐‑law  is  of  course  a  patent  nullity,  since  it  is  in  direct  conflict  with  the  latter  part  of  section  187  
of  the  Corporation  Law,  which  expressly  declares  that  the  board  of  directors  shall  not  have  the  power  to  force  
the   surrender   and   withdrawal   of   unmatured   stock   except   in   case   of   liquidation   of   the   corporation   or   of  
forfeiture  of  the  stock  for  delinquency.  It  is  agreed  that  this  provision  of  the  by-­‐‑laws  has  never  been  enforced,  
and   in   fact   no   attempt   has   ever   been   made   by   the   board   of   directors   to   make   use   of   the   power   therein  
conferred.  It  is  supposed,  in  the  fourth  cause  of  action,  that  the  existence  of  this  article  among  the  by-­‐‑laws  of  
the  association  is  a  misdemeanor  on  the  part  of  the  respondent  which  justifies  its  dissolution.  In  this  view  
we  are  unable  to  concur.  The  obnoxious  by-­‐‑law,  as  it  stands,  is  a  mere  nullity,  and  could  not  be  enforced  even  
if  the  directors  were  to  attempt  to  do  so.    
   
Fifth  cause  of  action.  —  In  section  31  of  the  Corporation  Law  it  is  declared  that,  "at  all  elections  of  directors  
there  must  be  present,  either  in  person  or  by  representative  authorized  to  act  by  written  proxy,  the  owners  
of   the   majority   of   the   subscribed   capital   stock   entitled   to   vote.   .   .   ."   Conformably   with   this   requirement   it   is  
declared   in   article   61   of   the   by-­‐‑laws   of   El   Hogar   Filipino   that,   "the   attendance   in   person   or   by   proxy   of  
shareholders  owning  one-­‐‑half  plus  one  of  the  shareholders  shall  be  necessary  to  constitute  a  quorum  for  the  
election  of  directors.  Owing  to  the  failure  of  a  quorum  at  most  of  the  general  meetings  since  the  respondent  
has  been  in  existence,  it  has  been  the  practice  of  the  directors  to  fill  vacancies  in  the  directorate  by  choosing  
suitable  persons  from  among  the  stockholders.  This  custom  finds  its  sanction  in  article  71  of  the  by-­‐‑laws,  
which  reads  as  follows:  
ART.  71.  The  directors  shall  elect  from  among  the  shareholders  members  to  fill  the  vacancies  that  may  occur  
in  the  board  of  directors  until  the  election  at  the  general  meeting.  
It  is  supposed  in  the  statement  of  the  fifth  cause  of  action  in  the  complaint  that  the  failure  of  the  corporation  
to  hold  annual  meetings  and  the  filling  of  vacancies  in  the  directorate  in  the  manner  described  constitute  
misdemeanors   on   the   part   of   the   respondent   which   justify   the   resumption   of   the   franchise   by   the  
Government  and  dissolution  of  the  corporation;  and  in  this  connection  it  is  charge  that  the  board  of  directors  
of  the  respondent  has  become  a  permanent  and  self  perpetuating  body  composed  of  wealthy  men  instead  of  
wage  earners  and  persons  of  moderate  means.  No  fault  can  be  imputed  to  the  corporation  on  account  of  the  
failure   of   the   shareholders   to   attend   the   annual   meetings;   and   their   non-­‐‑attendance   at   such   meetings   is  
doubtless   to   be   interpreted   in   part   as   expressing   their   satisfaction   of   the   way   in   which   things   have   been  

   
3H  A.Y.  2017-­‐2018   33  
 
 

CORPORATION  LAW  CASE  DIGESTS  –  ATTY.  DANTE  DELA  CRUZ  

conducted.   Unless   the   law   or   the   charter   of   a   corporation   expressly   provides   that   an   office   shall   become  
vacant  at  the  expiration  of  the  term  of  office  for  which  the  officer  was  elected,  the  general  rule  is  to  allow  the  
officer  to  holdover  until  his  successor  is  duly  qualified.  Mere  failure  of  a  corporation  to  elect  officers  does  not  
terminate  the  terms  of  existing  officers  nor  dissolve  the  corporation.    
   
Sixth  cause  of  action.  —  Under  the  sixth  cause  of  action  it  is  alleged  that  the  directors  of  El  Hogar  Filipino,  
instead  of  serving  without  pay,  or  receiving  nominal  pay  or  a  fixed  salary,  —  as  the  complaint  supposes  would  
be  proper,  —  have  been  receiving  large  compensation,  varying  in  amount  from  time  to  time,  out  of  the  profits  
of  the  respondent.  Under  section  92  of  the  by-­‐‑laws  of  El  Hogar  Filipino  5  per  centum  of  the  net  profit  shown  
by  the  annual  balance  sheet  is  distributed  to  the  directors  in  proportion  to  their  attendance  at  meetings  of  
the  board.    
It   will   be   note   that   the   compensation   above   indicated   as   accruing   to   the   directorate   as   a   whole   has   been  
divided  among  the  members  actually  present  at  the  different  meetings.  As  a  result  of  this  practice,  and  the  
liberal   measure   of   compensation   adopted,   we   find   that   the   attendance   of   the   membership   at   the   board  
meetings  has  been  extraordinarily  good.    The  Corporation  Law  does  not  undertake  to  prescribe  the  rate  of  
compensation  for  the  directors  of  corporations.  The  power  to  fixed  the  compensation  they  shall  receive,  if  
any,   is   left   to   the   corporation,   to   be   determined   in   its   by-­‐‑laws(Act   No.   1459,   sec.   21).   Pursuant   to   this  
authority  the  compensation  for  the  directors  of  El  Hogar  Filipino  has  been  fixed  in  section  92  of  its  by-­‐‑laws,  
as  already  stated.  If  a  mistake  has  been  made,  or  the  rule  adopted  in  the  by-­‐‑laws  meeting  to  change  the  rule.  
The  remedy,  if  any,  seems  to  lie  rather  in  publicity  and  competition,  rather  than  in  a  court  proceeding.    
   
Seventh  cause  of  action.  —  It  appears  that  the  promoter  and  organizer  of  El  Hogar  Filipino  was  Mr.  Antonio  
Melian,  and  in  the  early  stages  of  the  organization  of  the  association  the  board  of  directors  authorized  the  
association  to  make  a  contract  with  him  with  regard  to  the  services  him  therefor.  In  conformity  with  this  
agreement   there   was   inserted   in   section   92   of   the   by-­‐‑laws   of   the   association   a   provision   recognizing   the  
rights  of  Melian,  as  founder,  to  5  per  centum  of  the  net  profits  shown  by  the  annual  balance  sheet,  payment  
of  the  same  to  be  made  to  him  or  his  heirs  during  the  life  of  the  association.  As  a  seventh  cause  of  action  it  is  
alleged   in   the   complaint   that   this   royalty   of   the   founder   is   "unconscionable,   excessive   and   out   of   all  
proportion  to  the  services  rendered,  It  is  our  opinion  that  this  contention  is  entirely  without  merit.  The  mere  
fact  that  the  compensation  paid  under  this  contract  is  in  excess  of  what,  in  the  full  light  of  history,  may  be  
considered  appropriate  is  not  a  proper  consideration  for  this  court,  and  supplies  no  ground  for  interfering  
with  its  performance.  In  the  case  of  El  Hogar  Filipino  vs.  Rafferty  (37  Phil.,  995),  which  was  before  this  court  
nearly   ten   years   ago,   this   court   held   that   the   El   Hogar   Filipino   is   contract   with   Mr.   Melian   did   not   affect   the  
association's   legal   character.   The   inference   is   that   the   contract   under   consideration   was   then   considered  
binding,  and  it  occurred  to  no  one  that  it  was  invalid.    
   
Eight  cause  of  action.  —Under  the  eight  cause  of  action  the  alleged  ground  for  putting  an  end  to  the  corporate  
life  of  the  respondent  is  found  in  the  presence  of  other  articles  in  the  by-­‐‑laws,  namely,  articles  70  and  76,  
which  are  alleged  to  be  unlawful  but  which,  as  will  presently  be  seen,  are  entirely  valid.  Article  70  of  the  by-­‐‑
laws  in  effect  requires  that  persons  elected  to  the  board  of  directors  must  be  holders  of  shares  of  the  paid  up  
value  of  P5,000  which  shall  be  held  as  security  may  be  put  up  in  the  behalf  of  any  director  by  some  other  
holder  of  shares  in  the  amount  stated.  Article  76  of  the  by-­‐‑laws  declares  that  the  directors  waive  their  right  
as  shareholders  to  receive  loans  from  the  association.  Section  21  of  the  Corporation  Law  expressly  gives  the  
power  to  the  corporation  to  provide  in  its  by-­‐‑laws  for  the  qualifications  of  directors;  and  the  requirement  of  
security  from  them  for  the  proper  discharge  of  the  duties  of  their  office,  in  the  manner  prescribed  in  article  
70,  is  highly  prudent  and  in  conformity  with  good  practice.  Article  76,  prohibiting  directors  from  making  
loans   to   themselves,   is   of   course   designed   to   prevent   the   possibility   of   the   looting   of   the   corporation   by  
unscrupulous  directors.  Clearly,  the  eighth  cause  of  action  cannot  be  sustained.  
   

   
3H  A.Y.  2017-­‐2018   34  
 
 

CORPORATION  LAW  CASE  DIGESTS  –  ATTY.  DANTE  DELA  CRUZ  

Ninth  cause  of  action.  —  The  specification  under  this  head  is  in  effect  that  the  respondent  has  abused  its  
franchise  in  issuing  "special"  shares.  The  issuance  of  these  shares  is  allege  to  be  illegal  and  inconsistent  with  
the  plan  and  purposes  of  building  and  loan  associations;  and  in  particular,  it  is  alleged  and  inconsistent  with  
the  plan  and  purposes  of  building  and  loan  associations;  and  in  particular,  it  is  alleged  that  they  are,  in  the  
main,  held  by  well-­‐‑to-­‐‑wage-­‐‑earners  for  accumulating  their  modest  savings  for  the  building  of  homes.  The  
ground   for   supposing   the   issuance   of   the   "special"   shares   to   be   unlawful   is   that   special   shares   are   not  
mentioned  in  the  Corporation  Law  as  one  of  the  forms  of  security  which  may  be  issued  by  the  association.  
The  matter  of  the  propriety  of  the  issuance  of  special  shares  by  El  Hogar  Filipino  has  been  before  this  court  
in  two  earlier  cases,  in  both  of  which  the  question  has  received  the  fullest  consideration  from  this  court.  Upon  
an  elaborate  review  of  the  authorities,  the  court,  though  divided,  adhered  to  the  principle  announced  in  the  
earlier  case  and  held  that  the  issuance  of  the  special  shares  did  not  affect  the  respondent's  character  as  a  
building  and  loan  association  nor  make  its  loans  usurious.  it  is  satisfactorily  demonstrated  in  Severino  vs.  El  
Hogar  Filipino,  supra,  that  even  assuming  that  the  statute  has  not  expressly  authorized  such  shares,  yet  the  
association  has  implied  authority  to  issue  them.  The  complaint  consequently  fails  also  as  regards  the  stated  
in  the  ninth  cause  of  action.  
   
Tenth  cause  of  action.  —  Under  this  head  of  the  complaint  it  is  alleged  that  the  defendant  is  pursuing  a  policy  
of  depreciating,  at  the  rate  of  10  per  centum  per  annum,  the  value  of  the  real  properties  acquired  by  it  at  its  
sales;  There  is  no  positive  provision  of  law  prohibiting  the  association  from  writing  off  a  reasonable  amount  
for   depreciation   on   its   assets   for   the   purpose   of   determining   its   real   profits;   and   article   74   of   its   by-­‐‑laws  
expressly  authorizes  the  board  of  directors  to  determine  each  year  the  amount  to  be  written  down  upon  the  
expenses   of   installation   and   the   property   of   the   corporation.   There   can   be   no   question   that   the   power   to  
adopt  such  a  by-­‐‑law  is  embraced  within  the  power  to  make  by-­‐‑laws  for  the  administration  of  the  corporate  
affairs  of  the  association  and  for  the  management  of  its  business,  as  well  as  the  care,  control  and  disposition  
of   its   property   (Act   No.   1459,   sec.   13   [7]).   Certainly   this   court   cannot   undertake   to   control   the   discretion   of  
the  board  of  directors  of  the  association  about  an  administrative  matter  as  to  which  they  have  legitimate  
power  of  action.  The  tenth  cause  of  action  is  therefore  not  well  founded.  
   
Eleventh   and   twelfth   causes   of   action.   —It   is   insisted   in   the   brief   of   the   Attorney-­‐‑General   that   the  
maintenance  of  reserve  funds  is  unnecessary  in  the  case  of  building  and  loan  associations,  and  at  any  rate  
the  keeping  of  reserves  is  inconsistent  with  section  188  of  the  Corporation  Law.  Moreover,  it  is  said  that  the  
practice   of   the   association   in   declaring   regularly   a   10   per   cent   dividend   is   in   effect   a   guaranty   by   the  
association  of  a  fixed  dividend  which  is  contrary  to  the  intention  of  the  statute.  
It  is  true  that  the  corporation  law  does  not  expressly  grant  this  power  to  maintain  these  reserves,  but  
we  think  it  is  to  be  implied.  It  is  a  fact  of  common  observation  that  all  commercial  enterprises  encounter  
periods   when   earnings   fall   below   the   average,   and   the   prudent   manager   makes   provision   for   such  
contingencies.   Building   and   loan   associations,   though   among   the   most   solid   of   financial   institutions,   are  
nevertheless   subject   to   vicissitudes.   Fluctuations   in   the   dividend   rate   are   highly   detrimental   to   any   fiscal  
institutions,  while  uniformity  in  the  payments  of  dividends,  continued  over  long  periods,  supplies  the  surest  
foundations  of  public  confidence.  
The   government   insists,   we   thing,   upon   an   interpretation   of   section   188   of   the   Corporation   Law   that   is  
altogether  too  strict  and  literal.  But  it  will  be  noted  that  it  is  provided  in  the  same  section  that  the  profits  and  
losses   shall   be   determined   by   the   board   of   directors:   and   this   means   that   they   shall   exercise   the   usual  
discretion   of   good   businessmen   in   allocating   a   portion   of   the   annual   profits   to   purposes   needful   to   the  
welfare  of  the  association.  The  law  contemplates  the  distribution  of  earnings  and  losses  after  other  legitimate  
obligations  have  been  met.  Our  conclusion  is  that  the  respondent  has  the  power  to  maintain  the  reserves  
criticized   in   the   eleventh   and   twelfth   counts   of   the   complaint;   and   at   any   rate,   if   it   be   supposed   that   the  
reserves  referred  to  have  become  excessive,  the  remedy  is  in  the  hands  of  the  Legislature.  It  is  no  proper  
function  of  the  court  to  arrogate  to  itself  the  control  of  administrative  matters  which  have  been  confided  to  
the  discretion  of  the  board  of  directors.    

   
3H  A.Y.  2017-­‐2018   35  
 
 

CORPORATION  LAW  CASE  DIGESTS  –  ATTY.  DANTE  DELA  CRUZ  

   
Thirteenth  cause  of  action.  —  The  specification  under  this  head  is,  in  effect,  that  the  respondent  association  
has   made   loans   which,   to   the   knowledge   of   the   associations   officers   were   intended   to   be   used   by   the  
borrowers  for  other  purposes  than  the  building  of  homes  which  has  illegally  departed  from  its  character  and  
made  itself  amenable  to  the  penalty  of  dissolution.  Aside  from  being  directly  opposed  to  the  decision  of  this  
court  in  Lopez  and  Javelona  vs.  El  Hogar  Filipino  and  Registrar  of  Deeds  of  Occidental  Negros  (47  Phil.,  249),  
this  contention  finds  no  substantial  support  in  the  prevailing  decisions  made  in  American  courts;  and  our  
attention  has  not  been  directed  to  a  single  case  wherein  the  dissolution  of  a  building  and  loan  association  has  
been   decreed   in   a   quo   warranto   proceeding   because   the   association   allowed   its   borrowers   to   use   the   loans  
for  other  purposes  than  the  acquisition  of  homes.  There  is  no  statute  here  expressly  declaring  that  loans  may  
be   made   by   these   associations   solely   for   the   purpose   of   building   homes.   On   the   contrary,   the   building   of  
homes   is   mentioned   in   section   171   of   the   Corporation   Law   as   only   one   among   several   ends   which   building  
and  loan  associations  are  designed  to  promote.  Furthermore,  section  181  of  the  Corporation  Law  expressly  
authorities  the  Board  of  directors  of  the  association  from  time  to  time  to  fix  the  premium  to  be  charged.    
   
Fourteenth  cause  of  action.  —  The  specification  under  this  head  is  that  the  loans  made  by  the  defendant  for  
purposes  other  than  building  or  acquiring  homes  have  been  extended  in  extremely  large  amounts  and  to  
wealthy  persons  and  large  companies.  The  law  states  no  limit  with  respect  to  the  size  of  the  loans  to  be  made  
by   the   association.   That   matter   is   confided   to   the   discretion   of   the   board   of   directors;   and   this   court   cannot  
arrogate  to  itself  a  control  over  the  discretion  of  the  chosen  officials  of  the  company.  If  it  should  be  thought  
wise  in  the  future  to  put  a  limit  upon  the  amount  of  loans  to  be  made  to  a  single  person  or  entity,  resort  
should  be  had  to  the  Legislature;  it  is  not  a  matter  amenable  to  judicial  control.    
   
Fifteenth  cause  of  action.  —Under  the  fifteenth  cause  of  action  it  is  claimed  that  upon  the  expiration  of  the  
franchise   of   the   association   through   the   effluxion   of   time,   or   earlier   liquidation   of   its   business,   the  
accumulated  reserves  and  other  properties  will  accrue  to  the  founder,  or  his  heirs,  and  the  then  directors  of  
the  corporation  and  to  those  persons  who  may  at  that  time  to  be  holders  of  the  ordinary  and  special  shares  
of  the  corporation.  In  this  connection  we  note  that  article  95  of  the  by-­‐‑laws.  It  seems  to  us  that  this  is  matter  
that   may   be   left   to   the   prevision   of   the   directors   or   to   legislative   action   if   it   should   be   deemed   expedient   to  
require   the   gradual   suppression   of   the   reserve   funds   as   the   time   for   dissolution   approaches.   It   is   no   matter  
for  judicial  interference,  and  much  less  could  the  resumption  of  the  franchise  on  this  ground  be  justified.    
   
Sixteenth  cause  of  action.   —  This  part  of  the  complaint  assigns  as  cause  of  action  that  various  loans  now  
outstanding  have  been  made  by  the  respondent  to  corporations  and  partnerships,  and  that  these  entities  
have  in  some  instances  subscribed  to  shares  in  the  respondent  for  the  sole  purpose  of  obtaining  such  loans.  
Nothing  is  said  in  the  agreed  statement  of  facts  on  the  point  whether  the  corporations  and  partnerships  that  
have  taken  loans  from  the  respondent  are  qualified  by  law  governing  their  own  organization  to  enter  into  
these   contracts   with   the   respondent.   In   section   173   of   the   Corporation   Law   it   is   declared   that   "any   person"  
may  become  a  stockholder  in  building  and  loan  associations.  The  word  "person"  appears  to  be  here  used  in  
its  general  sense,  and  there  is  nothing  in  the  context  to  indicate  that  the  expression  is  used  in  the  restricted  
sense  of  both  natural  and  artificial  persons,  as  indicated  in  section  2  of  the  Administrative  Code.  At  any  rate  
the   question   whether   these   loans   and   the   attendant   subscriptions   were   properly   made   involves   a  
consideration   of   the   power   of   the   subscribing   corporations   and   partnerships   to   own   the   stock   and   take   the  
loans;  and  it  is  not  alleged  in  the  complaint  that  they  were  without  power  in  the  premises.    
   
Seventeenth  cause  of  action.  —  Under  the  seventeenth  cause  of  action,  it  is  charged  that  in  disposing  of  real  
estates  purchased  by  it  in  the  collection  of  its  loans,  the  defendant  has  no  various  occasions  sold  some  of  the  
said  real  estate  on  credit,  transferring  the  title  thereto  to  the  purchaser;  that  the  properties  sold  are  then  
mortgaged  to  the  defendant  to  secure  the  payment  of  the  purchase  price,  said  amount  being  considered  as  a  
loan,  and  carried  as  such  in  the  books  of  the  defendant,  and  that  several  such  obligations  are  still  outstanding.  

   
3H  A.Y.  2017-­‐2018   36  
 
 

CORPORATION  LAW  CASE  DIGESTS  –  ATTY.  DANTE  DELA  CRUZ  

It   is   further   charged   that   the   persons   and   entities   to   which   said   properties   are   sold   under   the   condition  
charged  are  not  members  or  shareholders  nor  are  they  made  members  or  shareholders  of  the  defendant.  
This   part   of   the   complaint   is   based   upon   a   mere   technicality   of   bookkeeping.   The   central   idea  
involved  in  the  discussion  is  the  provision  of  the  Corporation  Law  requiring  loans  to  be  stockholders  only  
and  on  the  security  of  real  estate  and  shares  in  the  corporation,  or  of  shares  alone.  It  seems  to  be  supposed  
that,  when  the  respondent  sells  property  acquired  at  its  own  foreclosure  sales  and  takes  a  mortgage  to  secure  
the  deferred  payments,  the  obligation  of  the  purchaser  is  a  true  loan,  and  hence  prohibited.  But  in  requiring  
the   respondent   to   sell   real   estate   which   it   acquires   in   connection   with   the   collection   of   its   loans   within   five  
years  after  receiving  title  to  the  same,  the  law  does  not  prescribe  that  the  property  must  be  sold  for  cash  or  
that  the  purchaser  shall  be  a  shareholder  in  the  corporation.  Such  sales  can  of  course  be  made  upon  terms  
and  conditions  approved  by  the  parties;  and  when  the  association  takes  a  mortgage  to  secure  the  deferred  
payments,  the  obligation  of  the  purchaser  cannot  be  fairly  described  as  arising  out  of  a  loan.  Nor  does  the  
fact  that  it  is  carried  as  a  loan  on  the  books  of  the  respondent  make  it  a  loan  on  the  books  of  the  respondent  
make  it  a  loan  in  law.  The  contention  of  the  Government  under  this  head  is  untenable.  
   
In  conclusion,  the  respondent  is  enjoined  in  the  future  from  administering  real  property  not  owned  by  itself,  
except  as  may  be  permitted  to  it  by  contract  when  a  borrowing  shareholder  defaults  in  his  obligation.  In  all  
other  respects  the  complaint  is  dismissed.  
 
 
 
26.  STOCKHOLDERS  OF  F.  GUANZON  AND  SONS,  INC.  VS.  REGISTER  OF  DEEDS  OF  MANILA  
G.R.  NO.  L-­‐‑18216.  OCTOBER  30,  1962  
J.  BAUTISTA  ANGELO    
 
DOCTRINE:   While   shares   of   stock   constitute   personal   property   they   do   not   represent   property   of   the  
corporation.  The  corporation  has  property  of  its  own  which  consists  chiefly  of  real  estate.  A  share  of  stock  
only  typifies  an  aliquot  part  of  the  corporation's  property,  or  the  right  to  share  in  its  proceeds  to  that  extent  
when  distributed  according  to  law  and  equity,  but  its  holder  is  not  the  owner  of  any  part  of  the  capital  of  the  
corporation.  Nor  is  he  entitled  to  the  possession  of  any  definite  portion  of  its  property  or  assets.  
 
FACTS:  On  September  19,  1960,  the  five  stockholders  of  the  F.  Guanzon  and  Sons,  Inc.  executed  a  certificate  
of  liquidation  of  the  assets  of  the  corporation  reciting,  among  other  things,  that  by  virtue  of  a  resolution  of  
the   stockholders   adopted   on   September   17,   1960,   dissolving   the   corporation,   they  have  distributed  among  
themselves   in   proportion   to   their   shareholdings,   as   liquidating   dividends,   the   assets   of   said   corporation,  
including  real  properties  located  in  Manila.  
 
The  certificate  of  liquidation,  when  presented  to  the  Register  of  Deeds  of  Manila,  was  denied  registration  on  
seven  grounds,  of  which  the  following  were  disputed  by  the  stockholders:  
 
3.   The  number  of  parcels  not  certified  to  in  the  acknowledgment;  
 
5.   P430.50  Reg.  fees  need  be  paid;  
 
6.   P940.45  documentary  stamps  need  be  attached  to  the  document;  
 
7.   The  judgment  of  the  Court  approving  the  dissolution  and  directing  the  disposition  of  the  assets  of  the  
corporation  need  be  presented  
 
 

   
3H  A.Y.  2017-­‐2018   37  
 
 

CORPORATION  LAW  CASE  DIGESTS  –  ATTY.  DANTE  DELA  CRUZ  

Deciding  the  consulta  elevated  by  the  stockholders,  the  Commissioner  of  Land  Registration  overruled  ground  
No.  7  and  sustained  requirements  Nos.  3,  5  and  6.    
 
ISSUE:  Should  the  liquidation  be  granted?    
 
 
 
HELD:   As   correctly   stated   by   the   Commissioner   of   Land   Registration,   the   propriety   or   impropriety   of   the  
three   grounds   on   which   the   denial   of   the   registration   of   the   certificate   of   liquidation   was   predicated   hinges  
on   whether   or   not   that   certificate   merely   involves   a   distribution   of   the   corporation's   assets   or   should   be  
considered  a  transfer  or  conveyance.    
 
The  Commissioner  of  Land  Registration,  however,  entertained  a  different  opinion.  He  concurred  in  the  view  
expressed  by  the  register  of  deed  to  the  effect  that  the  certificate  of  liquidation  in  question,  though  it  involves  
a  distribution  of  the  corporation's  assets,  in  the  last  analysis  represents  a  transfer  of  said  assets  from  the  
corporation  to  the  stockholders.  Hence,  in  substance  it  is  a  transfer  or  conveyance.  
 
The  Court  agrees  with  the  opinion  of  these  two  officials  as  a  corporation  is  a  juridical  person  distinct  from  
the  members  composing  it.  Properties  registered  in  the  name  of  the  corporation  are  owned  by  it  as  an  entity  
separate   and   distinct   from   its   members.   While   shares   of   stock   constitute   personal   property   they   do   not  
represent  property  of  the  corporation.  The  corporation  has  property  of  its  own  which  consists  chiefly  of  real  
estate.  A  share  of  stock  only  typifies  an  aliquot  part  of  the  corporation's  property,  or  the  right  to  share  in  its  
proceeds  to  that  extent  when  distributed  according  to  law  and  equity,  but  its  holder  is  not  the  owner  of  any  
part  of  the  capital  of  the  corporation.  Nor  is  he  entitled  to  the  possession  of  any  definite  portion  of  its  property  
or   assets.   On   the   basis   of   the   foregoing   authorities,   it   is   clear   that   the   act   of   liquidation   made   by   the  
stockholders  of  the  F.  Guanzon  and  Sons,  Inc.  of  the  latter's  assets  is  not  and  cannot  be  considered  a  partition  
of   community   property,   but   rather   a   transfer   or   conveyance   of   the   title   of   its   assets   to   the   individual  
stockholders.   Indeed,   since   the   purpose   of   the   liquidation,   as   well   as   the   distribution   of   the   assets   of   the  
corporation,   is   to   transfer   their   title   from   the   corporation   to   the   stockholders   in   proportion   to   their  
shareholdings,  —  and  this  is  in  effect  the  purpose  which  they  seek  to  obtain  from  the  Register  of  Deeds  of  
Manila,   —   that   transfer   cannot   be   effected   without   the   corresponding   deed   of   conveyance   from   the  
corporation  to  the  stockholders.  
 
 
 
27.  CARAM  VS  COURT  OF  APPEALS  
G.R.  NO.  L-­‐‑48627    JUNE  30,  1987  
CRUZ,  J.  
 
DOCTRINE:  
-­‐‑   The  petitioners  were  merely  among  the  financiers  whose  interest  was  to  be  invited  and  who  were  in  fact  
persuaded,  on  the  strength  of  the  project  study,  to  invest  in  the  proposed  airline.  As  a  bona  fide  corporation,  
the  Filipinas  Orient  Airways  should  alone  be  liable  for  its  corporate  acts  as  duly  authorized  by  its  officers  and  
directors.  
 
-­‐‑  The  most  that  can  be  said  is  that  they  benefited  from  such  services,  but  that  surely  is  no  justification  to  hold  
them  personally  liable  therefor.    
Otherwise,  all  the  other  stockholders  of  the  corporation,  including  those  who  came  in  later,  and  regardless  
of  the  amount  of  their  share  holdings,  would  be  equally  and  personally  liable  also  with  the  petitioners  for  the  
claims  of  the  private  respondent.  

   
3H  A.Y.  2017-­‐2018   38  
 
 

CORPORATION  LAW  CASE  DIGESTS  –  ATTY.  DANTE  DELA  CRUZ  

 
FACTS:    
 
Petitioners  herein  questioned  their  solidary  liability  with  their  co-­‐‑defendants.    
 
Petitioners  challenged  the  decision  of  the  court  ordering  Defendants  to  jointly  and  severally  pay  the  plaintiff  
the   amount   of   P50K   for   the   preparation   of   the   project   study   and   his   technical   services   that   led   to   the  
organization  of  the  defendant  corporation,  plus  P10K  attorney's  fees;    
 
PETTIONER   contends:   that   this   order   has   no   support   in   fact   and   law   because   they   had   no   contract  
whatsoever  with  the  private  respondent  regarding  the  above-­‐‑mentioned  services.    
-­‐‑  Their  position  is  that  as  mere  subsequent  investors  in  the  corporation  that  was  later  created,  they  
should  not  be  held  solidarily  liable  with  the  Filipinas  Orient  Airways,  a  separate  juridical  entity,  and  with  
Barretto   and   Garcia,   their   co-­‐‑defendants   in   the   lower   court,   who   were   the   ones   who   requested   the   said  
services  from  the  private  respondent.    
 
“The  reasons  for  the  said  order  are  given  by  the  respondent  court  in  its  decision  in  this  wise:  
 
As   to   the   remuneration   due   the   plaintiff   for   the   preparation   of   the   project   study   and   the   pre-­‐‑organizational  
services  in  the  amount  of  P50,000.00,  not  only  the  defendant  corporation  but  the  other  defendants  including  
defendants  Caram  should  be  jointly  and  severally  liable  for  this  amount.  
 
As  we  above  related  it  was  upon  the  request  of  defendants  Barretto  and  Garcia  that  plaintiff  handled  the  
preparation  of  the  project  study  which  project  study  was  presented  to  defendant  Caram  so  the  latter  was  
convinced  to  invest  in  the  proposed  airlines.    
 
The   project   study   was   revised   for   purposes   of   presentation   to   financiers   and   the   banks.   It   was   on   the   basis  
of  this  study  that  defendant  corporation  was  actually  organized  and  rendered  operational.    
 
Defendants   Garcia   and   Caram,   and   Barretto   became   members   of   the   Board   and/or   officers   of   defendant  
corporation.  Thus,  not  only  the  defendant  corporation  but  all  the  other  defendants  who  were  involved  in  the  
preparatory  stages  of  the  incorporation,  who  caused  the  preparation  and/or  benefited  from  the  project  study  
and  the  technical  services  of  plaintiff  must  be  liable.  “  
 
ISSUE:  whether  or  not  the  petitioners  themselves  are  also  and  personally  liable  for  such  expenses  and,  if  so,  
to  what  extent.  
 
HELD  :  
It   was   held   the   petitioners   cannot   be   held   personally   liable   for   the   compensation   claimed   by   the   private  
respondent  for  the  services  performed  by  him  in  the  organization  of  the  corporation.    
 
It  would  appear  from  the  above  justification  that  the  petitioners  were  not  really  involved  in  the  initial  steps  
that  finally  led  to  the  incorporation  of  the  Filipinas  Orient  Airways.    
 
Elsewhere  in  the  decision,  Barretto  was  described  as  "the  moving  spirit."  The  finding  of  the  respondent  court  
is  that  the  project  study  was  undertaken  by  the  private  respondent  at  the  request  of  Barretto  and  Garcia  who,  
upon   its   completion,   presented   it   to   the   petitioners   to   induce   them   to   invest   in   the   proposed   airline.   The  
study  could  have  been  presented  to  other  prospective  investors.    
At  any  rate,  the  airline  was  eventually  organized  on  the  basis  of  the  project  study  with  the  petitioners  
as  major  stockholders  and,  together  with  Barretto  and  Garcia,  as  principal  officers.  

   
3H  A.Y.  2017-­‐2018   39  
 
 

CORPORATION  LAW  CASE  DIGESTS  –  ATTY.  DANTE  DELA  CRUZ  

 
The  petitioners  were  not  involved  in  the  initial  stages  of  the  organization  of  the  airline,  which  were  being  
directed  by  Barretto  as  the  main  promoter.  
It   was   he   who   was   putting   all   the   pieces   together,   so   to   speak.   The   petitioners   were   merely   among   the  
financiers  whose  interest  was  to  be  invited  and  who  were  in  fact  persuaded,  on  the  strength  of  the  project  
study,  to  invest  in  the  proposed  airline.  
 
As  a  bona  fide  corporation,  the  Filipinas  Orient  Airways  should  alone  be  liable  for  its  corporate  acts  as  duly  
authorized  by  its  officers  and  directors.  
 
To  repeat,  the  petitioners  did  not  contract  such  services.  It  was  only  the  results  of  such  services  that  Barretto  
and  Garcia  presented  to  them  and  which  persuaded  them  to  invest  in  the  proposed  airline.  The  most  that  can  
be  said  is  that  they  benefited  from  such  services,  but  that  surely  is  no  justification  to  hold  them  personally  
liable  therefor.    
 
Otherwise,  all  the  other  stockholders  of  the  corporation,  including  those  who  came  in  later,  and  regardless  
of  the  amount  of  their  share  holdings,  would  be  equally  and  personally  liable  also  with  the  petitioners  for  the  
claims  of  the  private  respondent.  
 
Categorically,   the   Court   holds   that   the   petitioners   are   not   liable   at   all,   jointly   or   jointly   and   severally,   under  
the  first  paragraph  of  the  dispositive  portion  of  the  challenged  decision.    
 
 
 
28.  PALAY,  INC.  VS  JACOBO  CLAVE  
GR  NO.  L-­‐‑56076  21  SEPTEMBER  1983  
JUSTICE  MELENCIO-­‐‑HERRERA  
 
DOCTRINE:  
The  veil  of  corporate  fiction  cannot  be  pierced  when  no  sufficient  proof  exists  that  the  corporation  was  used  
to   commit   acts   of   fraud.   Thus,   the   President   cannot   be   held   personally   liable,   jointly   and   severally,   with   the  
corporation  for  the  latter’s  liabilities.    
 
FACTS:  
  Petitioner  Palay,  Inc.  through  its  President,  Albert  Onstott  executed  in  favor  of  private  respondent,  
Nazario  Dimpit,  a  contract  to  sell  a  parcel  of  land  owned  by  the  corporation  in  Antipolo,  Rizal.  The  sale  price  
was  P23,300.00  with  interest  and  payable  with  a  downpayment  and  monthly  installments  until  fully  paid.    
 
  Section   6   of   the   CTS   provided   for   automatic   rescission   upon   default   in   payment   of   any   monthly  
installment  after  the  lapse  of  90  days  from  expiration  of  the  grace  period  of  one  month,  without  need  of  notice  
and   with   forfeiture   of   installments   paid.   Respondent   Dumpit   paid   the   corresponding   downpayment   and  
several  installments  until  December  1967  for  installments  up  to  September  1967.  
 
  Six  years  later,  Dumpit  wrote  a  letter  to  Palay,  Inc.  offering  to  update  all  his  overdue  account  with  
interest  and  seeking  its  written  consent  to  the  assignment  of  his  rights  to  a  certain  Lourdes  Dizon.  However,  
on  reply,  petitioner  informed  Dumpit  that  his  CTS  had  long  been  rescinded  due  to  paragraph  6  and  that  the  
lot  had  already  been  resold.    
 
  Thus,  Dumpit  filed  a  complaint  with  the  National  Housing  Authority  questioning  the  validity  of  the  
rescission   of   the   contract.   The   NHA   found   the   rescission   void   in   the   absence   of   either   judicial   or   notarial  

   
3H  A.Y.  2017-­‐2018   40  
 
 

CORPORATION  LAW  CASE  DIGESTS  –  ATTY.  DANTE  DELA  CRUZ  

demand  and  ordered  Palay,  Inc.  and  its  President,  jointly  and  severally,  to  refund  Dumpit  of  his  payment.  On  
appeal  to  the  Office  of  the  President,  the  latter  affirmed  the  NHA’s  Resolution.    
 
ISSUE:  
Whether  the  doctrine  of  piercing  the  veil  of  corporate  fiction  has  application  to  the  case  
 
RULING:  
  The  Supreme  Court  affirmed  the  former  resolutions  in  holding  petitioner  corporation  liable  for  the  
refund  of  the  payments  made  by  Dumpit  due  to  the  lack  of  notice  to  the  latter  of  the  rescission  of  the  contract.  
However,   the   Court   further   ruled   that   the   corporation’s   president,   Albert   Onstott   should   not   be   held  
personally  liable  with  the  corporation  for  refund  to  Dumpit.    
 
A   corporation   is   invested   by   law   with   a   personality   separate   and   distinct   from   those   of   the   persons  
composing   it.   As   a   general   rule,   a   corporation   may   not   be   made   to   answer   the   acts   or   liabilities   of   its  
stockholders  and  vice  versa.  However,  the  veil  of  corporate  fiction  may  be  pierced  when  it  is  used  to  justify  
a  wrong,  protect  fraud,  or  defend  crime;  or  to  perpetuate  fraud  or  confuse  legitimate  issues;  or  to  circumvent  
the  law  or  perpetuate  deception.    
 
The  Court  found  no  badges  of  fraud  on  petitioner’s  part.  They  had  literally  relied  on  the  provisions  of  
the  contract  to  sell  when  it  was  extrajudicially  rescinded  and  had  sold  the  lot  a  third  person.  No  sufficient  
proof  exists  on  record  that  petitioner  used  the  corporation  to  defraud  private  respondent,  Dumpit.    
 
 
 
29.  JG  SUMMIT  HOLDINGS  VS.  CA  
G.R.  NO.  124293.  SEPTEMBER  24,  2003  
PUNO,  J.  
 
DOCTRINE:  Public  bidding  is  the  accepted  method  in  arriving  at  a  fair  and  reasonable  price  and  ensures  that  
overpricing,  favoritism  and  other  anomalous  practices  are  eliminated  or  minimized.  But  the  requirement  for  
public  bidding  does  not  negate  the  exercise  of  the  right  of  first  refusal.  
 
FACTS:    
On  January  27,  1977,  the  National  Investment  and  Development  Corporation  (NIDC),  entered  into  a  Joint  
Venture   Agreement   (JVA)   with   Kawasaki   Heavy   Industries,   Ltd.   of   Kobe,   Japan   (KAWASAKI)   for   the  
construction,   operation   and   management   of   the   Subic   National   Shipyard,   Inc.   (SNS)   which   subsequently  
became   the   Philippine   Shipyard   and   Engineering   Corporation   (PHILSECO).   One   of   its   salient   features   is   the  
grant   to   the   parties   of   the   right   of   first   refusal   should   either   of   them   decide   to   sell,   assign   or   transfer   its  
interest  in  the  joint  venture.  
NIDC  transferred  all  its  rights,  title  and  interest  in  PHILSECO  to  the  National  Government  pursuant  to  
Administrative   Order   No.   14.   There   after   a   trust   agreement   was   entered   into   between   the   National  
Government  and  the  Asset  Privatization  Trust  (APT)  wherein  the  latter  was  named  the  trustee  of  the  National  
Governments  share  in  PHILSECO.    
In  the  interest  of  the  national  economy  and  the  government,  the  Committee  on  Privatization  (COP)  and  
APT  deemed  it  best  to  sell  the  National  Governments  share  in  PHILSECO  to  private  entities.  After  a  series  of  
negotiations  between  the  APT  and  KAWASAKI,  they  agreed  that  the  latter’s  right  of  first  refusal  under  the  
JVA  be  exchanged  for  the  right  to  top  by  five  percent  (5%)  the  highest  bid  for  the  said  shares.  
At  the  public  bidding  on  the  said  date,  petitioner  J.G.  Summit  Holdings,  Inc.  (JGSMI)  was  declared  the  
highest   bidder,   the   COP   approved   the   sale   on   December   3,   1993   subject   to   the   right   of   Kawasaki   Heavy  
Industries,  Inc./[PHILYARDS]  Holdings,  Inc.  right  to  top  JGSMI's  bid  by  5%  as  specified  in  the  bidding  rules."  

   
3H  A.Y.  2017-­‐2018   41  
 
 

CORPORATION  LAW  CASE  DIGESTS  –  ATTY.  DANTE  DELA  CRUZ  

On  December  29,  1993,  petitioner  informed  APT  that  it  was  protesting  the  offer  of  PHI  to  top  its  bid  on  
the  grounds  that:  among  others,  xxx  (b)  only  KAWASAKI  could  exercise  the  right  to  top;  (c)  giving  the  same  
option   to   top   to   PHI   constituted   unwarranted   benefit   to   a   third   party;   (d)   no   right   of   first   refusal   can   be  
exercised   in   a   public   bidding   or   auction   sale;   xxx  
 
ISSUE:  Whether  or  not  the  existence  of  KAWASAKIs  right  to  top  destroys  the  essence  of  competitive  bidding  
so  as  to  say  that  the  bidders  did  not  have  an  opportunity  for  competition  
HELD:  NO.  
In   the   instant   case,   the   sale   of   the   Government   shares   in   PHILSECO   was   publicly   known.   All   interested  
bidders  were  welcomed.  The  basis  for  comparing  the  bids  were  laid  down.  All  bids  were  accepted  sealed  and  
were  opened  and  read  in  the  presence  of  the  COAs  official  representative  and  before  all  interested  bidders.  
The  essence  of  competition  in  public  bidding  is  that  the  bidders  are  placed  on  equal  footing.  This  means  
that  all  qualified  bidders  have  an  equal  chance  of  winning  the  auction  through  their  bids.  In  the  case  at  bar,  
all  of  the  bidders  were  exposed  to  the  same  risk  and  were  subjected  to  the  same  condition,  i.e.,  the  existence  
of  KAWASAKIs  right  to  top.  Under  the  ASBR,  the  Government  expressly  reserved  the  right  to  reject  any  or  all  
bids,  and  manifested  its  intention  not  to  accept  the  highest  bid  should  KAWASAKI  decide  to  exercise  its  right  
to  top  under  the  ABSR.  This  reservation  or  qualification  was  made  known  to  the  bidders  in  a  pre-­‐‑bidding  
conference  held  on  September  28,  1993.  They  all  expressly  accepted  this  condition  in  writing  without  any  
qualification.  
 
 
 
30.  YOUNG  AUTO  SUPPLY  CO.  AND  NEMESIO  GARCIA  VS.  THE  HONORABLE  COURT  OF  APPEALS  
G.R.  NO.  104175  JUNE  25,  1993  
QUIASON,  J.:  
 
Doctrine:  A  corporation  has  no  residence  in  the  same  sense  in  which  this  term  is  applied  to  a  natural  person.  
But  for  practical  purposes,  a  corporation  is  in  a  metaphysical  sense  a  resident  of  the  place  where  its  principal  
office  is  located  as  stated  in  the  articles  of  incorporation.  
 
FACTS:  
Young  Auto  Supply  Co.  Inc.  (YASCO)  represented  by  Nemesio  Garcia,  its  president,  Nelson  Garcia  and  Vicente  
Sy,  sold  all  of  their  shares  of  stock  in  Consolidated  Marketing  &  Development  Corporation  (CMDC)  to  Roxas.  
The  purchase  price  was  P8M  payable  with  down  payment  of  P4M  and  the  balance  of  P4M  in  four  post  dated  
checks  of  P1M  each.  The  first  check  of  P4M  representing  the  down-­‐‑payment,  was  honored  by  the  drawee  
bank  but  the  four  other  checks  representing  the  balance  were  dishonored.  In  the  meantime,  Roxas  sold  one  
of  the  markets  to  a  third  party.  Out  of  the  proceeds  of  the  sale,  YASCO  received  P600,000,  leaving  a  balance  
of  P3.4M.  
Petitioners  filed  a  complaint  against  Roxas  in  the  RTC,  Branch  11,  Cebu  City,  praying  that  Roxas  be  ordered  
to  pay  petitioners  the  sum  of  P3.4M  or  that  full  control  of  the  three  markets  be  turned  over  to  YASCO  and  
Garcia.  Roxas  filed  a  motion  to  dismiss  on  the  ground  that  the  venue  was  improperly  laid.  The  RTC  denied  
Roxas’   motion   to   dismiss.   The   Court   of   Appeals   ordered   the   dismissal   of   the   complaint   on   the   ground   of  
improper  venue.  
 
ISSUE:  
Whether  or  not  the  Court  of  Appeals  erred  in  holding  that  the  venue  was  improperly  laid  in  Cebu  City.  
 
RULING:  
The  Court  of  Appeals  erred  in  holding  that  venue  was  improperly  laid.  In  the  RTC,  all  personal  actions  are  
commenced  and  tried  in  the  province  or  city  where  the  defendant  or  any  of  the  defendants  resides  or  may  

   
3H  A.Y.  2017-­‐2018   42  
 
 

CORPORATION  LAW  CASE  DIGESTS  –  ATTY.  DANTE  DELA  CRUZ  

be   found,   or   where   the   plaintiff   or   any   of   the   plaintiffs   resides,   at   the   election   of   the   plaintiff   [Sec.   2(b)   Rule  
4,  Revised  Rules  of  Court].  
There  are  two  plaintiffs  in  the  case  at  bench:  a  natural  person  and  a  domestic  corporation.  Both  plaintiffs  
aver  in  their  complaint  that  they  are  residents  of  Cebu  City.  
A   corporation   has   no   residence   in   the   same   sense   in   which   this   term   is   applied   to   a   natural   person.   But   for  
practical  purposes,  a  corporation  is  in  a  metaphysical  sense  a  resident  of  the  place  where  its  principal  office  
is  located  as  stated  in  the  articles  of  incorporation.  The  Corporation  Code  precisely  requires  each  corporation  
to   specify   in   its   articles   of   incorporation   the   "place   where   the   principal   office   of   the   corporation   is   to   be  
located  which  must  be  within  the  Philippines"  (Sec.  14  [3]).  A  corporation  cannot  be  allowed  to  file  personal  
actions   in   a   place   other   than   its   principal   place   of   business   unless   such   a   place   is   also   the   residence   of   a   co-­‐‑
plaintiff   or   a   defendant.   With   the   finding   that   the   residence   of   YASCO   for   purposes   of   venue   is   in   Cebu   City,  
where  its  principal  place  of  business  is  located,  it  becomes  unnecessary  to  decide  whether  Garcia  is  also  a  
resident  of  Cebu  City.  
 
 
 
31.  MARVEL  BUILDING  V.  SATURNINO  DAVID  
G.R.  NO.  L-­‐‑5081,  FEBRUARY  24,  1954  
LABRADOR,  J.  
 
DOCTRINE:   Piercing   the   veil   of   corporate   fiction;   Castro   would   not   have   asked   them   to   endorse   their   stock  
certificates,  or  be  keeping  these  in  her  possession,  if  they  were  really  the  owners.  
 
FACTS:    
The  Secretary  of  Finance,  upon  consideration  of  the  report  of  a  special  committee  assigned  to  study  the  war  
profits  tax  case  of  Mrs.  Maria  B.  Castro  (Castro),  recommended  the  collection  of  P3.59M  as  war  profits  taxes  
for  the  latter.  In  Sept  1953,  the  President  instructed  the  Collector  that  steps  be  taken  to  collect  the  same.  
Pursuant   thereto,   the   Collector   seized   and   distrained   various   properties   of   Marvel   Building   Corporation  
(Marvel),   including   three   parcels   of   land   with   the   buildings   situated   thereon,   known   as   the   Aguinaldo  
Building,  the  Wise  Building,  and  the  Dewey  Boulevard-­‐‑Padre  Faura  Mansion.  
 
Plaintiffs  allege  that  the  said  three  properties  (lands  and  buildings)  belong  to  Marvel,  and  not  to  Castro,  while  
the  defendant  claims  that  Castro  is  the  true  and  sole  owner  of  all  the  subscribed  stock  of  Marvel,  including  
those  appearing  to  have  been  subscribed  and  paid  for  by  the  other  members,  and  consequently,  said  Castro  
is  also  the  true  and  exclusive  owner  of  the  properties  seized.  The  most  important  evidence  presented  by  the  
Collector  to  prove  his  claim  is  the  supposed  endorsement  in  blank  of  the  shares  of  stock  issued  in  the  name  
of  the  other  incorporators,  and  the  possession  thereof  by  Castro.    
 
ISSUE:   WON   Maria   B.   Castro   is   the   owner   of   all   the   shares   of   stocks   of   Marvel   Building   and   the   other  
stockholders  were  mere  dummies  of  hers?  
 
HELD:  Yes.  The  existence  of  endorsed  certificates,  discovered  by  the  internal  revenue  agents  between  1948  
and   1949   in   the   possession   of   the   Secretary-­‐‑Treasurer,   the   fact   that   25   certificates   were   signed   by   the  
president   of   the   corporation,   for   no   justifiable   reason,   the   fact   that   2   sets   of   certificates   were   issued,   the  
undisputed   fact   that   Castro   had   made   enormous   profits   and,   therefore,   had   a   motive   to   hide   them   to   evade  
the  payment  of  taxes,  the  fact  that  the  other  subscribers  had  no  incomes  of  sufficient  magnitude  to  justify  
their  big  subscriptions,  the  fact  that  the  subscriptions  were  not  receipted  for  and  deposited  by  the  treasurer  
in  the  name  of  the  corporation  but  were  kept  by  Castro  herself,  the  fact  that  the  stockholders  or  the  directors  
never  appeared  to  have  ever  met  to  discuss  the  business  of  the  corporation,  the  fact  that  Castro  advanced  big  
sums  of  money  to  the  corporation  without  any  previous  arrangement  or  accounting,  and  the  fact  that  the  

   
3H  A.Y.  2017-­‐2018   43  
 
 

CORPORATION  LAW  CASE  DIGESTS  –  ATTY.  DANTE  DELA  CRUZ  

books   of   accounts   were   kept   as   if   they   belonged   to   Castro   alone   –   these   facts   are   patent   and   potent  
significance.  What  are  their  necessary  implications?  Castro  would  not  have  asked  them  to  endorse  their  stock  
certificates,  or  be  keeping  these  in  her  possession,  if  they  were  really  the  owners.  
 
Each  and  every  one  of  the  facts  all  set  forth  above,  in  the  same  manner,  is  inconsistent  with  the  claim  that  the  
stockholders,  other  than  Castro,  own  their  shares  in  their  own  right.  On  the  other  hand,  each  and  every  one  
of  them,  and  all  of  them,  can  point  to  no  other  conclusion  than  that  Castro  was  the  sole  and  exclusive  owner  
of  the  shares  and  that  they  were  only  her  dummies.  
 
 
 
32.  GREGORIO  PALACIO  VS.  FELY  TRANSPORTATION  COMPANY  
G.R.  NO.  L-­‐‑15121-­‐‑  AUGUST  31,  1962  
REGALA,  J.  
 
DOCTRINE:  Corporation  should  not  be  heard  to  say  that  it  has  a  personality  separate  and  distinct  from  its  
members  when  to  allow  it  to  do  so  would  be  to  sanction  the  use  of  the  fiction  of  corporate  entity  as  a  shield  
to  further  an  end  subversive  of  justice.  
 
FACTS:    
About  December  1952,  the  defendant  company  hired  Alfredo  Carillo  as  driver  of  AC-­‐‑787  (687)  owned  and  
operated   by   the   said   defendant   company.   On   December   24,   1952,   at   about   11:30   a.m.,   while   the   driver  
Alfonso   (Alfredo)   Carillo   was   driving   at   Halcon   Street,   Quezon   City,   wilfully,   unlawfully   and   feloniously   and  
in  a  negligent,  reckless  and  imprudent  manner,  run  over  a  child  Mario  Palacio  of  the  herein  plaintiff  Gregorio  
Palacio.   On   account   of   the   aforesaid   injuries,   Mario   Palacio   suffered   a   simple   fracture   of   the   right   tenor,  
complete  third,  thereby  hospitalizing  him  at  the  Philippine  Orthopedic  Hospital  from  December  24,  1952,  up  
to  January  8,  1953,  and  continued  to  be  treated  for  a  period  of  five  months  thereafter.  The  plaintiff  Gregorio  
Palacio  is  a  welder  by  occupation  and  owner  of  a  small  welding  shop  and  because  of  the  injuries  of  his  child  
he  has  abandoned  his  shop  where  he  derives  income  of  P10.00  a  day  for  the  support  of  his  big  family  and  
that  during  the  period  that  the  plaintiff's  child  was  in  the  hospital  and  was  under  treatment  for  five  months,  
in   order   to   meet   the   needs   of   his   big   family,   he   was   forced   to   sell   one   air   compressor   (heavy   duty)   and   one  
heavy   duty   electric   drill,   for   a   sacrifice   sale   of   P150.00   which   could   easily   sell   at   P350.00.   And   as   a  
consequence  of  the  negligent  and  reckless  act  of  the  driver  Alfredo  Carillo  of  the  herein  defendant  company,  
the  plaintiffs  were  forced  to  litigate  the  case  in  Court  for  an  agreed  amount  of  P300.00  for  attorney's  fee.  The  
plaintiffs  have  now  incurred  the  amount  of  P500.00  actual  expenses  for  transportation,  representation  and  
similar  expenses  for  gathering  evidence  and  witnesses  and  because  of  the  nature  of  the  injuries  of  plaintiff  
Mario  Palacio  and  the  fear  that  the  child  might  become  a  useless  invalid,  the  herein  plaintiff  Gregorio  Palacio  
has  suffered  moral  damages  which  could  be  conservatively  estimated  at  P1,200.00.  
 
On  May  23,  1956,  defendant  Fely  Transportation  Co.,  filed  a  Motion  to  Dismiss  but  theCourt  deferred  the  
determination  of  the  grounds  alleged  in  the  Motion  to  Dismiss  until  the  trial  of  this  case.  
 
On  June  20,  1956,  defendant  filed  its  answer  and  alleges  (1)  that  complaint  states  no  cause  of  action  against  
defendant,   and   (2)   that   the   sale   and   transfer   of   the   jeep   AC-­‐‑687   by   Isabelo   Calingasan   to   the   Fely  
Transportation  was  made  on  December  24,  1955,  long  after  the  driver  Alfredo  Carillo  of  said  jeep  had  been  
convicted  and  had  served  his  sentence,  in  which  both  the  civil  and  criminal  cases  were  simultaneously  tried  
by  agreement  of  the  parties  in  said  case.  
 

   
3H  A.Y.  2017-­‐2018   44  
 
 

CORPORATION  LAW  CASE  DIGESTS  –  ATTY.  DANTE  DELA  CRUZ  

The  Court  of  First  Instance  of  Quezon  City  in  its  decision  in  Criminal  Case  No.  1084  (Exhibit  "2")  determined  
and   thoroughly   discussed   the   civil   liability   of   the   accused   in   that   case.   The   dispositive   part   thereof   reads   as  
follows:  
 
IN  VIEW  OF  THE  FOREGOING,  the  Court  finds  the  accused  Alfredo  Carillo  y  Damaso  guilty  beyond  reasonable  
doubt  of  the  crime  charged  in  the  information  and  he  is  hereby  sentenced  to  suffer  imprisonment  for  a  period  
of   Two   Months   &   One   Day   of   Arresto   Mayor;   to   indemnify   the   offended   party,   by   way   of   consequential  
damages,  in  the  sum  of  P500.00  which  the  Court  deems  reasonable;  with  subsidiary  imprisonment  in  case  of  
insolvency  but  not  to  exceed  ¹/3  of  the  principal  penalty  imposed;  and  to  pay  the  costs.  
On  the  basis  of  these  facts,  the  lower  court  held  action  is  barred  by  the  judgment  in  the  criminal  case  and,  
that   under   Article   103   of   the   Revised   Penal   Code,   the   person   subsidiarily   liable   to   pay   damages   is   Isabel  
Calingasan,  the  employer,  and  not  the  defendant  corporation.  
 
ISSUE:    
W/N  defendant  corporation  should  be  made  subsidiarily  liable  for  damages  in  the  criminal  case  because  the  
sale  to  it  of  the  jeep  in  question  after  the  conviction  of  Alfred  Carillo  was  merely  an  attempt  on  the  part  of  
Isabelo  Calingasan  its  president  and  general  manager  to  evade  his  subsidiary  civil  liability.  
 
HELD:  
Isabelo  Calingasan  and  defendant  Fely  Transportation  may  be  regarded  as  one  and  the  same  person.  It  is  
evident  that  Isabelo  Calingasan's  main  purpose  in  forming  the  corporation  was  to  evade  his  subsidiary  civil  
liability  resulting  from  the  conviction  of  his  driver,  Alfredo  Carillo.  This  conclusion  is  borne  out  by  the  fact  
that  the  incorporators  of  the  Fely  Transportation  are  Isabelo  Calingasan,  his  wife,  his  son,  Dr.  Calingasan,  and  
his  two  daughters.  We  believe  that  this  is  one  case  where  the  defendant  corporation  should  not  be  heard  to  
say  that  it  has  a  personality  separate  and  distinct  from  its  members  when  to  allow  it  to  do  so  would  be  to  
sanction   the   use   of   the   fiction   of   corporate   entity   as   a   shield   to   further   an   end   subversive   of   justice.  
Furthermore,  the  failure  of  the  defendant  corporation  to  prove  that  it  has  other  property  than  the  jeep  (AC-­‐‑
687)  strengthens  the  conviction  that  its  formation  was  for  the  purpose  above  indicated.  
 
And  while  it  is  true  that  Isabelo  Calingasan  is  not  a  party  in  this  case,  yet,  is  held  in  the  case  of  Alonso  v.  
Villamor,  16  Phil.  315,  this  Court  can  substitute  him  in  place  of  the  defendant  corporation  as  to  the  real  party  
in  interest.  This  is  so  in  order  to  avoid  multiplicity  of  suits  and  thereby  save  the  parties  unnecessary  expenses  
and  delay.    
Accordingly,  defendants  Fely  Transportation  and  Isabelo  Calingasan  should  be  held  subsidiarily  liable  for  
P500.00   which   Alfredo   Carillo   was   ordered   to   pay   in   the   criminal   case   and   which   amount   he   could   not   pay  
on  account  of  insolvency.  
 
WHEREFORE,  the  decision  of  the  lower  court  is  hereby  reversed  and  defendants  Fely  Transportation  and  
Isabelo   Calingasan   are   ordered   to   pay,   jointly   and   severally,   the   plaintiffs   the   amount   of   P500.00   and   the  
costs.  
 
 
 
33.  NATIONAL  MARKETING  CORP.  V.  ASSOCIATED  FINANCE  CO.,  INC.  
G.R.  NO.  L-­‐‑20886.  APRIL  27,  1967  
DIZON,  J.;  
 
 

   
3H  A.Y.  2017-­‐2018   45  
 
 

CORPORATION  LAW  CASE  DIGESTS  –  ATTY.  DANTE  DELA  CRUZ  

Doctrine:  It  is  settled  law  in  this  and  other  jurisdictions  that  when  the  corporation  is  the  mere  alter  ego  of  a  
person,  the  corporate  fiction  may  be  disregarded;  the  same  being  true  when  the  corporation  is  controlled,  
and  its  affairs  are  so  conducted  as  to  make  it  merely  an  instrumentality,  agency  or  conduit  of  another.  
 
Facts:  
ASSOCIATED,  a  domestic  corporation,  through  its  President,  Francisco  Sycip,  entered  into  an  agreement  to  
exchange  sugar  with  NAMARCO,  represented  by  its  then  General  Manager,  Benjamin  Estrella,  whereby  the  
former  would  deliver  to  the  latter  22,516  bags  (each  weighing  100  pounds)  of  "Victorias"  and/or  "National"  
refined   sugar   in   exchange   for   7,732.71   bags   of   "Busilak"   and   17,285.08   piculs   of   "Pasumil"   raw   sugar  
belonging  to  NAMARCO,  both  agreeing  to  pay  liquidated  damages  equivalent  to  20%  of  the  contractual  value  
of   the   sugar   should   either   party   fail   to   comply   with   the   terms   and   conditions.   Pursuant   thereto,   on   May   19,  
1958,   NAMARCO   delivered   to   ASSOCIATED   7,732.71   bags   of   "Busilak"   and   17,285.08   piculs   of   "Pasumil"  
domestic  raw  sugar.  As  ASSOCIATED  failed  to  deliver  to  NAMARCO  the  22,516  bags  of  "Victorias"  and/or  
"National"   refined   sugar   agreed   upon,   latter,   an   January   12,   1959,   demanded   in   writing   from   the  
ASSOCIATED  either  (a)  immediate  delivery  thereof  before  January  20,  or  (b)  payment  of  its  equivalent  cash  
value  amounting  to  P372,639.80.  
 
On   January   19,   1959,   ASSOCIATED,   through   Sycip,   offered   to   pay   NAMARCO   the   value   of   22,516   bags   of  
refined  sugar  at  the  rate  of  P15.30  per  bag,  but  the  latter  rejected  the  offer.  Instead,  on  January  21  of  the  same  
year,   it   demanded   payment   of   the   7,732.71   bags   of   "Busilak"   raw   sugar   at   P15.30   per   bag,   amounting   to  
P118,310.40,   and   of   the   17,285.08   piculs   of   "Pasumil"   raw   sugar   at   P16.50   per   picul,   amounting   to  
P285,203.82,   or   a   total   price   of   P403,514.28   for   both  kinds   of   sugar,   based   on   the   sugar   quotations   as   of  
March  20,  1958  —  the  date  when  the  exchange  agreement  was  entered  into.  
 
As   ASSOCIATED   refused   to   deliver   the   raw   sugar   or   pay   for   the   refined   sugar   delivered   to   it,   in   spite   of  
repeated  demands  therefore,  NAMARCO  instituted  the  present  action  in  the  lower  court  to  recover  the  sum  
of   P403,514.28   in   payment   of   the   raw   sugar   received   by   defendants   from   it;   P80,702.86;   as   liquidated  
damages;  P10,000.00  as  attorney's  fees,  expenses  of  litigation  and  exemplary  damages,  with  legal  interest  
thereon  from  the  filing  of  the  complaint  until  fully  paid.  
 
Issue:  Whether  rancisco  Sycip  may  be  held  liable,  jointly  and  severally  with  his  co-­‐‑defendant,  for  the  sums  of  
money  adjudged  in  favor  of  NAMARCO.  (YES)  
 
Ratio:  
The  foregoing  facts,  fully  established  by  the  evidence,  can  lead  to  no  other  conclusion  than  that  Sycip  was  
guilty   of   fraud   because   through   false   representations   he   succeeded   in   inducing   NAMARCO   to   enter   into   the  
aforesaid   exchange   agreement,   with   full   knowledge,   on   his   part   of   the   fact   that   ASSOCIATED   whom   he  
represented  and  over  whose  business  and  affairs  he  had  absolute  control,  was  in  no  position  to  comply  with  
the  obligation  it  had  assumed.  Consequently,  he  can  not  now  seek  refuge  behind  the  general  principle  that  a  
corporation   has   a   personality   distinct   and   separate   from   that   of   its   stockholders   and   that   the   latter   are   not  
personally   liable   for   the   corporate   obligations.   To   the   contrary,   upon   the   proven   facts,   We   feel   perfectly  
justified  in  "piercing  the  veil  of  corporate  fiction"  and  in  holding  Sycip  personally  liable,  jointly  and  severally  
with  his  co-­‐‑defendant,  for  the  sums  of  money  adjudged  in  favor  of  appellant.  It  is  settled  law  in  this  and  other  
jurisdictions   that   when   the   corporation   is   the   mere   alter   ego   of   a   person,   the   corporate   fiction   may   be  
disregarded;  the  same  being  true  when  the  corporation  is  controlled,  and  its  affairs  are  so  conducted  as  to  
make  it  merely  an  instrumentality,  agency  or  conduit  of  another.  
 
 
 
 

   
3H  A.Y.  2017-­‐2018   46  
 
 

CORPORATION  LAW  CASE  DIGESTS  –  ATTY.  DANTE  DELA  CRUZ  

34.  TAN  BOON  BEE  &  CO.,  INC.,  V.  THE  HONORABLE  HILARION  U.  JARENCIO  
G.R.  NO.  L-­‐‑41337  -­‐‑  JUNE  30,  1988  
PARAS,  J.  
 
DOCTRINE:  
It   is   true   that   a   corporation,   upon   coming   into   being,   is   invested   by   law   with   a   personality   separate   and  
distinct  from  that  of  the  persons  composing  it  as  well  as  from  any  other  legal  entity  to  which  it  may  be  related.  
However,  this  separate  personality  of  the  corporation  may  be  disregarded,  or  the  veil  of  corporate  fiction  
pierced,  in  cases  where  it  is  used  as  a  cloak  or  cover  for  fraud  or  illegality,  or  to  work  an  injustice,  or  where  
necessary  to  achieve  equity  or  when  necessary  for  the  protection  of  creditors.  
 
FACTS:  
Petitioner  sold  on  credit  to  herein  private  respondent  Graphic  Publishing,  Inc.  (GRAPHIC)  paper  products.  
 
GRAPHIC   made   partial   payment   by   check   to   petitioner   and   a   promissory   note   was   executed   to   cover   the  
balance.  In  the  said  promissory  note,  it  was  stipulated  that  the  amount  will  be  paid  on  monthly  installments  
and  that  failure  to  pay  any  installment  would  make  the  amount  immediately  demandable  with  an  interest  of  
12%  per  annum.  
 
For   failure   of   GRAPHIC   to   pay   any   installment,   petitioner   filed   with   the   then   CFI   a   civil   case   for   a   sum   of  
money.  The  trial  court  ordered  GRAPHIC  to  pay  the  petitioner.  On  motion  of  petitioner,  a  writ  of  execution  
was  issued  but  the  writ  expired  without  the  sheriff  finding  any  property  of  GRAPHIC.  Hence,  an  alias  writ  of  
execution  was  issued.  
 
Pursuant  to  the  said  issued  alias  writ  of  execution,  the  executing  sheriff  levied  upon  one  (1)  unit  printing  
machine  identified  as  "Original  Heidelberg  Cylinder  Press"  found  in  the  premises  of  GRAPHIC.  In  a  Notice  of  
Sale  of  Execution  of  Personal,  said  printing  machine  was  scheduled  for  auction  sale  but  prior  to  such  sale,  in  
a   letter,   Philippine   American   Drug   Company   (PADCO)   had   informed   the   sheriff   that   the   printing   machine  is  
its  property  and  not  that  of  GRAPHIC,  and  accordingly,  advised  the  sheriff  to  cease  and  desist  from  carrying  
out   the   scheduled   auction   sale.   Notwithstanding   the   said   letter,   the   sheriff   proceeded   with   the   scheduled  
auction  sale,  sold  the  property  to  the  petitioner,  it  being  the  highest  bidder,  and  issued  a  Certificate  of  Sale  in  
favor  of  petitioner.  
 
PADCO  filed  an  "Affidavit  of  Third  Party  Claim"  with  the  Office  of  the  City  Sheriff.  Thereafter,  PADCO  filed  
with  the  CFI  a  Motion  to  Nullify  Sale  on  Execution  (With  Injunction)  which  was  opposed  by  the  petitioner.  
The  CFI  ruled  in  favor  of  PADCO.    
 
Petitioner   contends   that   the   controlling   stockholders   of   the   PADCO   are   also   the   same   controlling  
stockholders   of   the   GRAPHIC   and,   therefore,   the   levy   upon   the   said   machinery   which   was   found   in   the  
premises  occupied  by  the  GRAPHIC  should  be  upheld.  
 
ISSUE:  
Should  PADCO's  veil  of  corporate  identity  be  pierced?  
 
HELD:  
YES.  It  is  true  that  a  corporation,  upon  coming  into  being,  is  invested  by  law  with  a  personality  separate  and  
distinct  from  that  of  the  persons  composing  it  as  well  as  from  any  other  legal  entity  to  which  it  may  be  related.  
However,  this  separate  personality  of  the  corporation  may  be  disregarded,  or  the  veil  of  corporate  fiction  
pierced,  in  cases  where  it  is  used  as  a  cloak  or  cover  for  fraud  or  illegality,  or  to  work  an  injustice,  or  where  
necessary  to  achieve  equity  or  when  necessary  for  the  protection  of  creditors.  Corporations  are  composed  of  

   
3H  A.Y.  2017-­‐2018   47  
 
 

CORPORATION  LAW  CASE  DIGESTS  –  ATTY.  DANTE  DELA  CRUZ  

natural  persons  and  the  legal  fiction  of  a  separate  corporate  personality  is  not  a  shield  for  the  commission  of  
injustice  and  inequity.  Likewise,  this  is  true  when  the  corporation  is  merely  an  adjunct,  business  conduit  or  
alter  ego  of  another  corporation.  In  such  case,  the  fiction  of  separate  and  distinct  corporation  entities  should  
be  disregarded.  
 
In  the  instant  case,  petitioner's  evidence  established  that  PADCO  was  never  engaged  in  the  printing  business;  
that  the  board  of  directors  and  the  officers  of  GRAPHIC  and  PADCO  were  the  same;  and  that  PADCO  holds  
50%   share   of   stock   of   GRAPHIC.   Petitioner   likewise   stressed   that   PADCO's   own   evidence   shows   that   the  
printing  machine  in  question  had  been  in  the  premises  of  GRAPHIC  since  May,  1965,  long  before  PADCO  even  
acquired  its  alleged  title  on  July  11,  1966  from  Capitol  Publishing.  That  the  said  machine  was  allegedly  leased  
by  PADCO  to  GRAPHIC  on  January  24,  1966,  even  before  PADCO  purchased  it  from  Capital  Publishing  on  July  
11,   1966,   only   serves   to   show   that   PADCO's   claim   of   ownership   over   the   printing   machine   is   not   only   farce  
and  sham  but  also  unbelievable.  
 
Considering  the  aforestated  principles  and  the  circumstances  established  in  this  case,  the  CFI  should  have  
pierced  PADCO's  veil  of  corporate  identity.  
 
 
 
35.   CONCEPCION   MAGSAYSAY-­‐‑LABRADOR,   SOLEDAD   MAGSAYSAY-­‐‑CABRERA,   LUISA   MAGSAYSAY-­‐‑
CORPUZ,  ASSISTED  BE  HER  HUSBAND,  DR.  JOSE  CORPUZ,  FELICIDAD  P.  MAGSAYSAY,  AND  MERCEDES  
MAGSAYSAY-­‐‑DIAZ,    VS.  THE  COURT  OF  APPEALS  AND  ADELAIDA  RODRIGUEZ-­‐‑MAGSAYSAY  
G.R.  NO.  58168  DECEMBER  19,  1989  
FERNAN,  C.J.  
 
DOCTRINE:   While   a   share   of   stock   represents   a   proportionate   or   aliquot   interest   in   the   property   of   the  
corporation,  it  does  not  vest  the  owner  thereof  with  any  legal  right  or  title  to  any  of  the  property,  his  interest  
in   the   corporate   property   being   equitable   or   beneficial   in   nature.   Shareholders   are   in   no   legal   sense   the  
owners  of  corporate  property,  which  is  owned  by  the  corporation  as  a  distinct  legal  person.  
 
FACTS:  On  February  9,  1979,  Adelaida  Rodriguez-­‐‑Magsaysay,  widow  and  special  administratix  of  the  estate  
of  the  late  Senator  Genaro  Magsaysay,  brought  before  the  then  Court  of  First  Instance  of  Olongapo  an  action  
against   Artemio   Panganiban,   Subic   Land   Corporation   (SUBIC),   Filipinas   Manufacturer's   Bank  
(FILMANBANK)   and   the   Register   of   Deeds   of   Zambales.   for   the   annulment   of   the   Deed   of   Assignment  
executed  by  the  late  Senator  in  favor  of  SUBIC  (as  a  result  of  which  TCT  3258  was  cancelled  and  TCT  22431  
issued   in   the   name   of   SUBIC),   for   the   annulment   of   the   Deed   of   Mortgage   executed   by   SUBIC   in   favor   of  
FILMANBANK  (dated  28  April  1977  in  the  amount  of  P  2,700,000.00),  and  cancellation  of  TCT  22431  by  the  
Register  of  Deeds,  and  for  the  latter  to  issue  a  new  title  in  her  favor.  
 
On  March  7,  1979,  herein  petitioners,  sisters  of  the  late  senator,  filed  a  motion  for  intervention  on  the  ground  
that  on  June  20,  1978,  their  brother  conveyed  to  them  one-­‐‑half  (1/2  )  of  his  shareholdings  in  SUBIC  or  a  total  
of  416,566.6  shares  and  as  assignees  of  around  41  %  of  the  total  outstanding  shares  of  such  stocks  of  SUBIC,  
they  have  a  substantial  and  legal  interest  in  the  subject  matter  of  litigation  and  that  they  have  a  legal  interest  
in  the  success  of  the  suit  with  respect  to  SUBIC.  
 
On   July   26,   1979,   the   court   denied   the   motion   for   intervention,   and   ruled   that   petitioners   have   no   legal  
interest   whatsoever   in   the   matter   in   litigation   and   their   being   alleged   assignees   or   transferees   of   certain  
shares   in   SUBIC   cannot   legally   entitle   them   to   intervene   because   SUBIC   has   a   personality   separate   and  
distinct  from  its  stockholders.  
 

   
3H  A.Y.  2017-­‐2018   48  
 
 

CORPORATION  LAW  CASE  DIGESTS  –  ATTY.  DANTE  DELA  CRUZ  

On  appeal,  the  Court  of  Appeals  found  no  factual  or  legal  justification  to  disturb  the  findings  of  the  lower  
court.  The  appellate  court  further  stated  that  whatever  claims  the  Magsaysay  sisters  have  against  the  late  
Senator   or   against   SUBIC   for   that   matter   can   be   ventilated   in   a   separate   proceeding.   The   motion   for  
reconsideration  of  the  Magsaysay  sisters  was  denied.  Hence,  the  petition  for  review  on  certiorari.    
 
ISSUE:  Whether  the  Magsaysay  sister,  allegedly  stockholders  of  SUBIC,  are  interested  parties  in  a  case  where  
corporate  properties  are  in  dispute.    
 
 
HELD:   NO.   Viewed   in   the   light   of   Section   2,   Rule   12   of   the   Revised   Rules   of   Court,   this   Court   affirms   the  
respondent  court's  holding  that  petitioners  herein  have  no  legal  interest  in  the  subject  matter  in  litigation  so  
as  to  entitle  them  to  intervene  in  the  proceedings.  
 
In  the  case  of  Batama  Farmers'  Cooperative  Marketing  Association,  Inc.  v.  Rosal,  4  we  held:  "As  clearly  stated  
in  Section  2  of  Rule  12  of  the  Rules  of  Court,  to  be  permitted  to  intervene  in  a  pending  action,  the  party  must  
have  a  legal  interest  in  the  matter  in  litigation,  or  in  the  success  of  either  of  the  parties  or  an  interest  against  
both,   or   he   must   be   so   situated   as   to   be   adversely   affected   by   a   distribution   or   other   disposition   of   the  
property  in  the  custody  of  the  court  or  an  officer  thereof  ."  
 
Here,   the   interest,   if   it   exists   at   all,   of   petitioners-­‐‑movants   is   indirect,   contingent,   remote,   conjectural,  
consequential  and  collateral.  At  the  very  least,  their  interest  is  purely  inchoate,  or  in  sheer  expectancy  of  a  
right   in   the   management   of   the   corporation   and   to   share   in   the   profits   thereof   and   in   the   properties   and  
assets  thereof  on  dissolution,  after  payment  of  the  corporate  debts  and  obligations.  
 
While  a  share  of  stock  represents  a  proportionate  or  aliquot  interest  in  the  property  of  the  corporation,  it  
does  not  vest  the  owner  thereof  with  any  legal  right  or  title  to  any  of  the  property,  his  interest  in  the  corporate  
property  being  equitable  or  beneficial  in  nature.  Shareholders  are  in  no  legal  sense  the  owners  of  corporate  
property,  which  is  owned  by  the  corporation  as  a  distinct  legal  person.  
 
The  petitioners  cannot  claim  the  right  to  intervene  on  the  strength  of  the  transfer  of  shares  allegedly  executed  
by  the  late  Senator.  Perforce,  no  transfer  was  ever  recorded,  much  less  effected  as  to  prejudice  third  parties.  
The   transfer   must   be   registered   in   the   books   of   the   corporation   to   affect   third   persons.   The   law   on  
corporations  is  explicit.  Section  63  of  the  Corporation  Code  provides,  thus:  "No  transfer,  however,  shall  be  
valid,  except  as  between  the  parties,  until  the  transfer  is  recorded  in  the  books  of  the  corporation  showing  
the   names   of   the   parties   to   the   transaction,   the   date   of   the   transfer,   the   number   of   the   certificate   or  
certificates  and  the  number  of  shares  transferred."  
 
 
 
 
36.  INDOPHIL  TEXTILE  MILL  WORKERS  UNION-­‐‑PTGWO  VS.  VOLUNTARY  ARBITRATOR  TEODORICO  P.  
CALICA  AND  INDOPHIL  TEXTILE  MILLS,  INC  
G.R.  NO.  96490   FEBRUARY  3,  1992  
MEDIALDEA,  J.  
 
CASE  DOCTRINE:  Under  the  doctrine  of  piercing  the  veil  of  corporate  entity,  when  valid  grounds  therefore  
exist,  the  legal  fiction  that  a  corporation  is  an  entity  with  a  juridical  personality  separate  and  distinct  from  
its  members  or  stockholders  may  be  disregarded.  In  such  cases,  the  corporation  will  be  considered  as  a  mere  
association   of   persons.   The   members   or   stockholders   of   the   corporation   will   be   considered   as   the  
corporation,  that  is  liability  will  attach  directly  to  the  officers  and  stockholders.  The  doctrine  applies  when  

   
3H  A.Y.  2017-­‐2018   49  
 
 

CORPORATION  LAW  CASE  DIGESTS  –  ATTY.  DANTE  DELA  CRUZ  

the  corporate  fiction  is  used  to  defeat  public  convenience,  justify  wrong,  protect  fraud,  or  defend  crime,  or  
when  it  is  made  as  a  shield  to  confuse  the  legitimate  issues,  or  where  a  corporation  is  the  mere  alter  ego  or  
business   conduit   of   a   person,   or   where   the   corporation   is   so   organized   and   controlled   and   its   affairs   are   so  
conducted  as  to  make  it  merely  an  instrumentality,  agency,  conduit  or  adjunct  of  another  corporation.  (Umali  
et  al.  v.  Court  of  Appeals,  G.R.  No.  89561,  September  13,  1990,  189  SCRA  529,  542)  
FACTS:  
1.   Petitioner   Indophil   Textile  Mill   Workers   Union-­‐‑PTGWO  is  a  legitimate  labor  organization  duly  registered  
with  the  Department  of  Labor  and  Employment  and  the  exclusive  bargaining  agent  of  all  the  rank-­‐‑and-­‐‑file  
employees  of  Indophil  Textile  Mills,  Incorporated.  
2.   Respondent   Teodorico   P.   Calica   is   impleaded   in   his   official   capacity   as   the   Voluntary   Arbitrator   of   the  
National   Conciliation   and   Mediation   Board   of   the   Department   of   Labor   and   Employment,   while   private  
respondent  Indophil  Textile  Mills,  Inc.  is  a  corporation  engaged  in  the  manufacture,  sale  and  export  of  yarns  
of   various   counts   and   kinds   and   of   materials   of   kindred   character   and   has   its   plants   at   Barrio   Lambakin.  
Marilao,  Bulacan.  
3.  Petitioner  Indophil  Textile  Mill  Workers  Union-­‐‑PTGWO  and  private  respondent  Indophil  Textile  Mills,  Inc.  
executed  a  collective  bargaining  agreement  effective  from  April  1,  1987  to  March  31,  1990.  
4.  Indophil  Acrylic  Manufacturing  Corporation  was  formed  and  registered  with  the  Securities  and  Exchange  
Commission.  
  a.   Subsequently,   Acrylic   applied   for   registration   with   the   Board   of   Investments   for   incentives  
  under  the  1987  Omnibus  Investments  Code.  
  b.  The  application  was  approved  on  a  preferred  non-­‐‑pioneer  status.  
5.   Sometime   in   July,   1989,   the   workers   of   Acrylic   unionized   and   a   duly   certified   collective   bargaining  
agreement  was  executed.  
  a.   In   1990   or   a   year   after   the   workers   of   Acrylic   have   been   unionized   and   a   CBA   executed,   the  
  petitioner   union   claimed   that   the   plant   facilities   built   and   set   up   by   Acrylic   should   be   considered  
  as  an  extension  or  expansion  of  the  facilities  of  private  respondent  Company  pursuant  to     Section  
1(c),  Article  I  of  the  CBA.  
6.   The  petitioner's  contention  was  opposed   by   private   respondent   which   submits   that   it   is   a   juridical  entity  
separate  and  distinct  from  Acrylic.  
7.  The  public  respondent  Voluntary  Arbitrator  rendered  its  award,  the  dispositive  portion  of  which  provides  
as  follows:  
      PREMISES  CONSIDERED,  it  would  be  a  strained  interpretation  and  application  of    
      the  questioned  CBA  provision  if  we  would  extend  to  the  employees  of  Acrylic    
      the  coverage  clause  of  Indophil  Textile  Mills  CBA.  Wherefore,  an  award  is  made    
      to  the  effect  that  the  proper  interpretation  and  application  of  Sec.  l,  (c),  Art.  I,  of    
    the  1987  CBA  do  (sic)  not  extend  to  the  employees  of  Acrylic  as  an  extension  or      
    expansion  of  Indophil  Textile  Mills,  Inc.  
ISSUE:  
1.  The  central  issue  submitted  for  arbitration  is  whether  or  not  the  operations  in  Indophil  Acrylic  Corporation  
are  an  extension  or  expansion  of  private  respondent  Company.  
  a.   Corollary   to   the   aforementioned   issue   is   the   question   of   whether   or   not   the   rank-­‐‑and-­‐‑file  
  employees   working   at   Indophil   Acrylic   should   be   recognized   as   part   of,   and/or   within   the   scope  
  of  the  bargaining  unit.  
RULING:  
1.  The  petition  is  devoid  of  merit.  
2.  Under  the  doctrine  of  piercing  the  veil  of  corporate  entity,  when  valid  grounds  therefore  exist,  the  legal  
fiction  that  a  corporation  is  an  entity  with  a  juridical  personality  separate  and  distinct  from  its  members  or  
stockholders  may  be  disregarded.  
  a.  In  such  cases,  the  corporation  will  be  considered  as  a  mere  association  of  persons.  

   
3H  A.Y.  2017-­‐2018   50  
 
 

CORPORATION  LAW  CASE  DIGESTS  –  ATTY.  DANTE  DELA  CRUZ  

  b.  The     members   or   stockholders   of   the   corporation   will   be   considered   as   the   corporation,   that  
  is  liability  will  attach  directly  to  the  officers  and  stockholders.  
  c.   The   doctrine   applies   when   the   corporate   fiction   is   used   to   defeat   public   convenience,   justify  
  wrong,   protect   fraud,   or   defend   crime,   or   when   it   is   made   as   a   shield   to   confuse   the   legitimate  
  issues,   or   where   a   corporation   is   the   mere   alter   ego   or   business   conduit   of   a   person,   or   where  
  the  corporation  is  so  organized  and  controlled  and  its  affairs  are  so  conducted  as  to  make  it     merely  
an  instrumentality,  agency,  conduit  or  adjunct  of  another  corporation.  
3.  In  the  case  at  bar,  petitioner  seeks  to  pierce  the  veil  of  corporate  entity  of  Acrylic,  alleging  that  the  creation  
of   the   corporation   is   a   devise   to   evade   the   application   of   the   CBA   between   petitioner   Union   and   private  
respondent  Company.  
  a.   While   the   Court   do   not   discount   the   possibility   of   the   similarities   of   the   businesses   of   private  
  respondent   and   Acrylic,   neither   are   the   Court   inclined   to   apply   the   doctrine   invoked   by  
  petitioner   in   granting   the   relief   sought.   The   fact   that   the   businesses   of   private   respondent   and  
  Acrylic   are   related,   that   some   of   the   employees   of   the   private   respondent   are   the   same   persons  
  manning   and   providing   for   auxilliary   services   to   the   units   of   Acrylic,   and   that   the   physical   plants,  
  offices   and   facilities   are   situated   in   the   same   compound,   it   is   the   Court’s   considered   opinion   that  
  these     facts  are  not  sufficient  to  justify  the  piercing  of  the  corporate  veil  of  Acrylic.  
4.   In   the   same   case   of   Umali,   et   al.   v.   Court   of   Appeals   (supra),   the   Court   already   emphasized   that   "the   legal  
corporate  entity  is  disregarded  only  if  it  is  sought  to  hold  the  officers  and  stockholders  directly  liable  for  a  
corporate   debt   or   obligation."   In   the   instant   case,   petitioner   does   not   seek   to   impose   a   claim   against   the  
members  of  the  Acrylic.  
 
 
 
37.  LYCEUM  VS  CA  
G.R.  NO.  101897/  219  SCRA  610  -­‐‑  MARCH  5,  1993  
FELICIANO,  J  
 
DOCTRINE:  The  policy  underlying  the  prohibition  in  Section  18  against  the  registration  of  a  corporate  name  
which   is   "identical   or   deceptively   or   confusingly   similar"   to   that   of   any   existing   corporation   or   which   is  
"patently  deceptive"  or  "patently  confusing"  or  "contrary  to  existing  laws,"  is  the  avoidance  of  fraud  upon  the  
public   which   would   have   occasion   to   deal   with   the   entity   concerned,   the   evasion   of   legal   obligations   and  
duties,   and   the   reduction   of   difficulties   of   administration   and   supervision   over   corporations.   We   do   not  
consider  that  the  corporate  names  of  private  respondent  institutions  are  "identical  with,  or  deceptively  or  
confusingly   similar"   to   that   of   the   petitioner   institution.   True   enough,   the   corporate   names   of   private  
respondent  entities  all  carry  the  word  "Lyceum"  but  confusion  and  deception  are  effectively  precluded  by  
the   appending   of   geographic   names   to   the   word   "Lyceum."   Thus,   we   do   not   believe   that   the   "Lyceum   of  
Aparri"   can   be   mistaken   by   the   general   public   for   the   Lyceum   of   the   Philippines,   or   that   the   "Lyceum   of  
Camalaniugan"  would  be  confused  with  the  Lyceum  of  the  Philippines.  
FACTS:  Petitioner  is  an  educational  institution  duly  registered  with  the  SEC.  When  it  first  registered  with  the  
SEC  on  21  September  1950,  it  used  the  corporate  name  Lyceum  of  the  Philippines,  Inc.  and  has  used  that  
name  ever  since.  
In  1984,  petitioner  instituted  proceedings  before  the  SEC  to  compel  the  private  respondents,  which  are  also  
educational  institutions,  to  delete  the  word  "Lyceum"  from  their  corporate  names  and  permanently  to  enjoin  
them  from  using  "Lyceum"  as  part  of  their  names.  
These   are   the   following,   the   dates   of   their   original   SEC   registration   being   set   out   below   opposite   their  
respective  names:  
Western  Pangasinan  Lyceum  —  27  October  1950  
Lyceum  of  Cabagan  —  31  October  1962  
Lyceum  of  Lallo,  Inc.  —  26  March  1972  

   
3H  A.Y.  2017-­‐2018   51  
 
 

CORPORATION  LAW  CASE  DIGESTS  –  ATTY.  DANTE  DELA  CRUZ  

Lyceum  of  Aparri  —  28  March  1972  


Lyceum  of  Tuao,  Inc.  —  28  March  1972  
Lyceum  of  Camalaniugan  —  28  March  1972  
Petitioner  had  sometime  before  commenced  in  the  SEC  a  proceeding  (SEC-­‐‑Case  No.  1241)  against  the  Lyceum  
of   Baguio,   Inc.   to   require   it   to   change   its   corporate   name   and   to   adopt   another   name   not   "similar   [to]   or  
identical"   with   that   of   petitioner.   Associate   Commissioner   Julio   Sulit   held   that   the   corporate   name   of  
petitioner  and  that  of  the  Lyceum  of  Baguio,  Inc.  were  substantially  identical  because  of  the  presence  of  a  
"dominant"  word,  i.e.,  "Lyceum,"  the  name  of  the  geographical  location  of  the  campus  being  the  only  word  
which   distinguished   one   from   the   other   corporate   name.   The   SEC   also   noted   that   petitioner   had   registered  
as   a   corporation   ahead   of   the   Lyceum   of   Baguio.   The   latter   assailed   the   SEC   Order   before   the   SC   which   was  
denied.  
Armed  with  the  Resolution  of  this  Court  in  the  above  cited  case,  petitioner  then  wrote  all  the  educational  
institutions   it   could   find   using   the   word   "Lyceum"   as   part   of   their   corporate   name,   and   advised   them   to  
discontinue  such  use.  When  it  became  clear  that  this  recourse  had  failed,  petitioner  instituted  a  case  before  
the   SEC   to   enforce   what   it   claims   as   its   proprietary   right   to   the   word   "Lyceum."   The   SEC   hearing   officer  
rendered  a  decision  sustaining  petitioner's  claim  to  an  exclusive  right  to  use  the  word  "Lyceum."    
On  appeal  to  the  SEC  En  Banc,  the  decision  was  reversed  and  it  held  that  the  word  "Lyceum"  is  not  identified  
with  petitioner  as  to  render  use  thereof  by  other  institutions  as  productive  of  confusion  about  the  identity  of  
the  schools  concerned  in  the  mind  of  the  general  public.  SEC  En  Banc  held  that  the  attaching  of  geographical  
names  to  the  word  "Lyceum"  served  sufficiently  to  distinguish  the  schools  from  one  another,  especially  that  
the  campuses  were  physically  remote  from  each  other.    
Petitioner  then  went  to  the  CA  which  affirmed  the  questioned  Orders  of  the  SEC  En  Banc.  Hence,  this  petition.  
ISSUE:   Whether   or   not   the   corporate   names   of   private   respondent   institutions   are   "identical   with,   or  
deceptively  or  confusingly  similar"  to  that  of  the  petitioner  institution  
HELD:   NO.   Section   18   of   the   Corporation   Code   establishes   a   restrictive   rule   insofar   as   corporate   names   are  
concerned:  
"SECTION   18.   Corporate   name.   —   No   corporate   name   may   be   allowed   by   the   Securities   an   Exchange  
Commission  if  the  proposed  name  is  identical  or  deceptively  or  confusingly  similar  to  that  of  any  existing  
corporation  or  to  any  other  name  already  protected  by  law  or  is  patently  deceptive,  confusing  or  contrary  to  
existing  laws.  When  a  change  in  the  corporate  name  is  approved,  the  Commission  shall  issue  an  amended  
certificate  of  incorporation  under  the  amended  name."  (Emphasis  supplied)  
The  policy  underlying  the  prohibition  in  Section  18  against  the  registration  of  a  corporate  name  which  is  
"identical   or   deceptively   or   confusingly   similar"   to   that   of   any   existing   corporation   or   which   is   "patently  
deceptive"  or  "patently  confusing"  or  "contrary  to  existing  laws,"  is  the  avoidance  of  fraud  upon  the  public  
which  would  have  occasion  to  deal  with  the  entity  concerned,  the  evasion  of  legal  obligations  and  duties,  and  
the  reduction  of  difficulties  of  administration  and  supervision  over  corporations.  We  do  not  consider  that  the  
corporate  names  of  private  respondent  institutions  are  "identical  with,  or  deceptively  or  confusingly  similar"  
to  that  of  the  petitioner  institution.  True  enough,  the  corporate  names  of  private  respondent  entities  all  carry  
the  word  "Lyceum"  but  confusion  and  deception  are  effectively  precluded  by  the  appending  of  geographic  
names  to  the  word  "Lyceum."    
Etymologically,  the  word  "Lyceum"  is  the  Latin  word  for  the  Greek  lykeion  which  in  turn  referred  to  a  locality  
on  the  river  Ilissius  in  ancient  Athens  "comprising  an  enclosure  dedicated  to  Apollo  frequented  by  the  youth  
for  exercise  and  by  Aristotle  and  his  followers  for  teaching."  In  time,  the  word  "Lyceum"  became  associated  
with   schools   and   other   institutions   providing   public   lectures   and   concerts   and   public   discussions.   Thus  
today,  the  word  "Lyceum"  generally  refers  to  a  school  or  an  institution  of  learning.  "Lyceum"  is  in  fact  as  
generic  in  character  as  the  word  "university."  Since  "Lyceum"  or  "Liceo"  denotes  a  school  or  institution  of  
learning,  it  is  not  unnatural  to  use  this  word  to  designate  an  entity  which  is  organized  and  operating  as  an  
educational  institution.  
We  conclude  that  petitioner  institution  is  not  entitled  to  a  legally  enforceable  exclusive  right  to  use  the  word  
"Lyceum"  in  its  corporate  name.  To  determine  whether  a  given  corporate  name  is  "identical"  or  "confusingly  

   
3H  A.Y.  2017-­‐2018   52  
 
 

CORPORATION  LAW  CASE  DIGESTS  –  ATTY.  DANTE  DELA  CRUZ  

or  deceptively  similar"  with  another  entity's  corporate  name,  it  is  not  enough  to  ascertain  the  presence  of  
"Lyceum"  or  "Liceo"  in  both  names.  One  must  evaluate  corporate  names  in  their  entirety  and  when  the  name  
of   petitioner   is   juxtaposed   with   the   names   of   private   respondents,   they   are   not   reasonably   regarded   as  
"identical"   or   "confusingly   or   deceptively   similar"   with   each   other.   WHEREFORE,   Petition   for   Review   is  
DENIED  for  lack  of  merit,  and  the  Decision  of  the  CA  is  hereby  AFFIRMED.  
 
 
 
38.    CONCEPT  BUILDERS  INC.,  VS.  NLRC  
 
 
 
39.  EDUARDO  CLAPAROLS,  ROMULO  AGSAM  and/or  CLAPAROLS  STEEL  AND  NAIL  PLANT  vs.  COURT  
OF   INDUSTRIAL   RELATIONS,   ALLIED   WORKERS'   ASSOCIATION   and/or   DEMETRIO   GARLITOS,  
ALFREDO   ONGSUCO,   JORGE   SEMILLANO,   SALVADOR   DOROTEO,   ROSENDO   ESPINOSA,   LUDOVICO  
BALOPENOS,  ASER  AMANCIO,  MAXIMO  QUIOYO,  GAUDENCIO  QUIOYO,  and  IGNACIO  QUIOYO  
G.R.  No.  L-­‐‑30822  July  31,  1975  
MAKASIAR,  J.:  
 
DOCRTINE:    When  the  notion  of  legal  entity  is  used  to  defeat  public  convenience,  justify  wrong,  protect  fraud,  
or   defend   crime,   the   law   will   regard   the   corporation   as   an   association   or   persons,   or,   in   the   case   of   two  
corporations,  will  merge  them  into  one.    
 
FACTS:  On  August  6,  1957,  a  complaint  for  unfair  labor  practice  was  filed  by  herein  private  respondent  Allied  
Workers'   Association,   respondent   Demetrio   Garlitos   and   ten   (10)   respondent   workers   against   herein  
petitioners  on  account  of  the  dismissal  of  respondent  workers  from  petitioner  Claparols  Steel  and  Nail  Plant.  
Respondent  Court  rendered  its  decision  finding  Mr.  Claparols  guilty  of  union  busting  and  of  having  "ismissed  
said   complainants   because   of   their   union   activities,   and   ordering   respondents   to   cease   and   desist   from  
committing  unfair  labor  practices  against  their  employees  and  laborers  and  to  reinstate  said  complainants  
to   their   former   or   equivalent   jobs,   as   soon   as   possible,   with   back   wages   from   the   date   of   their   dismissal   up  
to  their  actual  reinstatement.  Then,  respondent  workers  were  accompanied  by  the  Chief  of  Police  of  Talisay,  
Negros  Occidental  to  the  compound  of  herein  petitioner  company  to  report  for  reinstatement  per  order  of  
the  court.  Respondent  workers  were,  however,  refused  reinstatement  by  company  accountant  Francisco  Cusi  
for   he   had   no   order   from   plant   owner   Eduardo   Claparols   nor   from   his   lawyer   Atty.   Plaridel   Katalbas,   to  
reinstate  respondent  workers.  
 
On  January  15,   1965,  the  CIR  Chief  Examiner  Submitted  his  report  containing  three  computations.  The  first  
computation   covers   the   period   February   1,   1957   to   October   31,   1964.   The   second   is   up   to   and   including  
December  7,  1962,  when  the  corporation  stopped  operations,  while  the  third  is  only  up  to  June  30,  1957  
when  the  Claparols  Steel  and  Nail  Plant  ceased  to  operate.    
with  the  explanation  that:  
 
On   January   23,   1965,   petitioners   filed   an   opposition   alleging   that   under   the   circumstances   presently  
engulfing  the  company,  petitioner  Claparols  could  not  personally  reinstate  respondent  workers.  Petitioner  
further   contends   that   assuming   the   workers   are   entitled   to   back   wages,   the   same   should   only   be   limited   to  
three   months   and   that   since   Claparols   Steel   Corporation   ceased   to   operate   on   December   7,   1962,   re-­‐‑
employment  of  respondent  workers  cannot  go  beyond  December  7,  1962.  
 
A  reply  to  petitioner's  opposition  was  filed  by  respondent  workers,  alleging  among  others,  that  Claparols  
Steel  and  Nail  Plant  and  Claparols  Steel  and  Nail  Corporation  are  one  and  the  same  corporation  controlled  

   
3H  A.Y.  2017-­‐2018   53  
 
 

CORPORATION  LAW  CASE  DIGESTS  –  ATTY.  DANTE  DELA  CRUZ  

by  petitioner  Claparols,  with  the  latter  corporation  succeeding  the  former.  


 
ISSUE:  Whether  or  not  petitioner  is  liable  to  pay  the  respective  back  wages  and  bonuses  of  the  complainants.    
 
HELD:   Yes.   Respondent   Court's   findings   that   indeed   the   Claparols   Steel   and   Nail   Plant,   which   ceased  
operation  of  June  30,  1957,  was  SUCCEEDED  by  the  Claparols  Steel  Corporation  effective  the  next  day,  July  1,  
1957  up  to  December  7,  1962,  when  the  latter  finally  ceased  to  operate,  were  not  disputed  by  petitioners.  It  
is  very  clear  that  the  latter  corporation  was  a  continuation  and  successor  of  the  first  entity,  and  its  emergence  
was  skillfully  timed  to  avoid  the  financial  liability  that  already  attached  to  its  predecessor,  the  Claparols  Steel  
and   Nail   Plant.   Both   predecessors   and   successor   were   owned   and   controlled   by   the   petitioner   Eduardo  
Claparols  and  there  was  no  break  in  the  succession  and  continuity  of  the  same  business.  This  "avoiding-­‐‑the-­‐‑
liability"  scheme  is  very  patent,  considering  that  90%  of  the  subscribed  shares  of  stocks  of  the  Claparols  Steel  
Corporation   was   owned   by   respondent   Claparols   himself,   and   all   the   assets   of   the   dissolved   Claparols   Steel  
and  Nail  Plant  were  turned  over  to  the  emerging  Claparols  Steel  Corporation.  
 
It  is  very  obvious  that  the  second  corporation  seeks  the  protective  shield  of  a  corporate  fiction  whose  veil  in  
the  present  case  could,  and  should,  be  pierced  as  it  was  deliberately  and  maliciously  designed  to  evade  its  
financial  obligation  to  its  employees.  
 
 
 
40.    VILLA  REY  TRANSIT  V.  FERRER  
 
 
 
41.  SECOSA  V.  HEIRS  OF  FRANCISCO  
G.R.  NO.  160039;  JUNE  29,  2004  
YNARES-­‐‑SANTIAGO,  J.:  
 
Doctrine:  The  so-­‐‑called  veil  of  corporation  fiction  treats  as  separate  and  distinct  the  affairs  of  a  corporation  
and  its  officers  and  stockholders.  As  a  general  rule,  a  corporation  will  be  looked  upon  as  a  legal  entity,  unless  
and  until  sufficient  reason  to  the  contrary  appears.  When  the  notion  of  legal  entity  is  used  to  defeat  public  
convenience,   justify   wrong,   protect   fraud,   or   defend   crime,   the   law   will   regard   the   corporation   as   an  
association  of  persons.[19]  Also,  the  corporate  entity  may  be  disregarded  in  the  interest  of  justice  in  such  cases  
as  fraud  that  may  work  inequities  among  members  of  the  corporation  internally,  involving  no  rights  of  the  
public   or   third   persons.   In   both   instances,   there   must   have   been   fraud   and   proof   of   it.   For   the   separate  
juridical  personality  of  a  corporation  to  be  disregarded,  the  wrongdoing  must  be  clearly  and  convincingly  
established.  It  cannot  be  presumed.    
 
Facts:    
On   June   27,   1996,   at   around   4:00   p.m.,   Erwin   Suarez   Francisco,   an   eighteen   year   old   third   year   physical  
therapy  student  of  the  Manila  Central  University,  was  riding  a  motorcycle  along  Radial  10  Avenue,  near  the  
Veteran   Shipyard   Gate   in   the   City   of   Manila.   At   the   same   time,   petitioner,   Raymundo   Odani   Secosa,   was  
driving   an   Isuzu   cargo   truck   with   plate   number   PCU-­‐‑253   on   the   same   road.   The   truck   was   owned   by  
petitioner,  Dassad  Warehousing  and  Port  Services,  Inc.  
 
Traveling   behind   the   motorcycle   driven   by   Francisco   was   a   sand   and   gravel   truck,   which   in   turn   was   being  
tailed  by  the  Isuzu  truck  driven  by  Secosa.  The  three  vehicles  were  traversing  the  southbound  lane  at  a  fairly  
high   speed.   When   Secosa   overtook   the   sand   and   gravel   truck,   he   bumped   the   motorcycle   causing   Francisco  

   
3H  A.Y.  2017-­‐2018   54  
 
 

CORPORATION  LAW  CASE  DIGESTS  –  ATTY.  DANTE  DELA  CRUZ  

to   fall.   The   rear   wheels   of   the   Isuzu   truck   then   ran   over   Francisco,   which   resulted   in   his   instantaneous  
death.  Fearing  for  his  life,  petitioner  Secosa  left  his  truck  and  fled  the  scene  of  the  collision.[3]  
Respondents,   the   parents   of   Erwin   Francisco,   thus   filed   an   action   for   damages   against   Raymond   Odani  
Secosa,  Dassad  Warehousing  and  Port  Services,  Inc.  and  Dassads  president,  El  Buenasucenso  Sy.  
 
Issue:  Whether  or  not  Court  of  Appeals  erred  in  affirming  the  trial  court’s  decision  in  holding  petitioner  El  
Buenasesnso  solidarily  liable  with  petitioner  Dassad  and  Secosa  in  violation  of  Corporation  Law  
 
Ruling:    
The  Court  find  that  petitioner  El  Buenasenso  Sy  cannot  be  held  solidarily  liable  with  his  co-­‐‑petitioners.  While  
it  may  be  true  that  Sy  is  the  president  of  petitioner  Dassad  Warehousing  and  Port  Services,  Inc.,  such  fact  is  
not  by  itself  sufficient  to  hold  him  solidarily  liable  for  the  liabilities  adjudged  against  his  co-­‐‑petitioners.  
 
It  is  a  settled  precept  in  this  jurisdiction  that  a  corporation  is  invested  by  law  with  a  personality  separate  
from   that   of   its   stockholders   or   members.[16]   It   has   a   personality   separate   and   distinct   from   those   of   the  
persons  composing  it  as  well  as  from  that  of  any  other  entity  to  which  it  may  be  related.  Mere  ownership  by  
a  single  stockholder  or  by  another  corporation  of  all  or  nearly  all  of  the  capital  stock  of  a  corporation  is  not  
in  itself  sufficient  ground  for  disregarding  the  separate  corporate  personality.[17]  A  corporations  authority  to  
act  and  its  liability  for  its  actions  are  separate  and  apart  from  the  individuals  who  own  it.[18]  
 
The  so-­‐‑called  veil  of  corporation  fiction  treats  as  separate  and  distinct  the  affairs  of  a  corporation  and  its  
officers  and  stockholders.  As  a  general  rule,  a  corporation  will  be  looked  upon  as  a  legal  entity,  unless  and  
until   sufficient   reason   to   the   contrary   appears.   When   the   notion   of   legal   entity   is   used   to   defeat   public  
convenience,   justify   wrong,   protect   fraud,   or   defend   crime,   the   law   will   regard   the   corporation   as   an  
association  of  persons.[19]  Also,  the  corporate  entity  may  be  disregarded  in  the  interest  of  justice  in  such  cases  
as  fraud  that  may  work  inequities  among  members  of  the  corporation  internally,  involving  no  rights  of  the  
public   or   third   persons.   In   both   instances,   there   must   have   been   fraud   and   proof   of   it.   For   the   separate  
juridical  personality  of  a  corporation  to  be  disregarded,  the  wrongdoing  must  be  clearly  and  convincingly  
established.[20]  It  cannot  be  presumed.    
 
The  records  of  this  case  are  bereft  of  any  evidence  tending  to  show  the  presence  of  any  grounds  enumerated  
above  that  will  justify  the  piercing  of  the  veil  of  corporate  fiction  such  as  to  hold  the  president  of  Dassad  
Warehousing  and  Port  Services,  Inc.  solidarily  liable  with  it.  
 
The  Isuzu  cargo  truck  which  ran  over  Erwin  Francisco  was  registered  in  the  name  of  Dassad  Warehousing  
and  Port  Services,  Inc.,  and  not  in  the  name  of  El  Buenasenso  Sy.  Raymundo  Secosa  is  an  employee  of  Dassad  
Warehousing  and  Port  Services,  Inc.  and  not  of  El  Buenasenso  Sy.  All  these  things,  when  taken  collectively,  
point   toward   El   Buenasenso   Sys   exclusion   from   liability   for   damages   arising   from   the   death   of   Erwin  
Francisco.  
 
 
 
42.  BENJAMIN  YU  VS.  NLRC  AND  JADE  MOUNTAIN  PRODUCTS  COMPANY  LIMITED,  ET.  AL.    
G.R.  NO.  97212,  JUNE  30,  1993  
FELICIANO,  J.  
 
DOCTRINE:  
The   legal   effect   of   the   changes   in   the   membership   of   the   partnership   was   the   dissolution   of   the   old  
partnership   and   the   emergence   of   a   new   firm.   The   occurrence   of   events   which   precipitate   the   legal  
consequence   of   dissolution   of   a   partnership   do   not,   however,   automatically   result   in   the   termination   of   the  

   
3H  A.Y.  2017-­‐2018   55  
 
 

CORPORATION  LAW  CASE  DIGESTS  –  ATTY.  DANTE  DELA  CRUZ  

legal   personality   of   the   old   partnership.   Article   1829   of   the   Civil   Code   states   that   “on   dissolution   the  
partnership  is  not  terminated,  but  continues  until  the  winding  up  of  partnership  affairs  is  completed.”  In  the  
ordinary   course   of   events,   the   legal   personality   of   the   expiring   partnership   persists   for   the   limited   purpose  
of  winding  up  and  closing  of  the  affairs  of  the  partnership.  
 
FACTS:  
•   Petitioner  Yu  was  formerly  the  Assistant  General  Manager  of  the  marble  quarrying  and  export  business  
operated   by   a   registered   partnership   with   the   firm   name   of   Jade   Mountain   Products   Company   Limited  
("Jade  Mountain").  The  partnership  was  originally  organized  on  28  June  1984  with  Lea  Bendal  and  Rhodora  
Bendal  as  general  partners  and  Chin  Shian  Jeng,  Chen  Ho-­‐‑Fu  and  Yu  Chang,  all  citizens  of  the  Republic  of  
China   (Taiwan),   as   limited   partners.   The   partnership   business   consisted   of   exploiting   a   marble   deposit  
found   on   land   owned   by   the   Sps.   Ricardo   and   Guillerma   Cruz,   situated   in   Bulacan   Province,   under   a  
Memorandum   Agreement   dated   26   June   1984   with   the   Cruz   spouses.   The   partnership   had   its   main   office  
in  Makati.  
 
•   Yu  was  hired  by  virtue  of  a  Partnership  Resolution  dated  14  March  1985,  as  Assistant  General  Manager  
with   a   monthly   salary   of   P4,000.00.   According   to   Yu,   however,   he   actually   received   only   half   of   his  
stipulated  monthly  salary,  since  he  had  accepted  the  promise  of  the  partners  that  the  balance  would  be  
paid  when  the  firm  shall  have  secured  additional  operating  funds  from  abroad.  Yu  actually  managed  the  
operations  and  finances  of  the  business;  he  had  overall  supervision  of  the  workers  at  the  marble  quarry  in  
Bulacan  and  took  charge  of  the  preparation  of  papers  relating  to  the  exportation  of  the  firm's  products.  
 
•   Sometime   in   1988,   without   the   knowledge   of   Benjamin   Yu,   the   general   partners   Lea   Bendal   and   Rhodora  
Bendal  sold  and  transferred  their  interests  in  the  partnership  to  private  respondent  Willy  Co  and  to  one  
Emmanuel   Zapanta.   Mr.   Yu   Chang,   a   limited   partner,   also   sold   and   transferred   his   interest   in   the  
partnership   to   Willy   Co.   Between   Mr.   Emmanuel   Zapanta   and   himself,   private   respondent   Willy   Co  
acquired  the  great  bulk  of  the  partnership  interest.  The  partnership  now  constituted  solely  by  Willy  Co  and  
Emmanuel  Zapanta  continued  to  use  the  old  firm  name  of  Jade  Mountain,  though  they  moved  the  firm's  
main  office  from  Makati  to  Mandaluyong.  A  Supplement  to  the  Memorandum  Agreement  relating  to  the  
operation  of  the  marble  quarry  was  entered  into  with  the  Cruz  spouses  in  February  of  1988.  The  actual  
operations  of  the  business  enterprise  continued  as  before.  All  the  employees  of  the  partnership  continued  
working  in  the  business,  all,  save  petitioner  Benjamin  Yu  as  it  turned  out.  
 
•   On  16  November  1987,  having  learned  of  the  transfer  of  the  firm's  main  office  from  Makati  to  Mandaluyong,  
petitioner  Yu  reported  to  the  Mandaluyong  office  for  work  and  there  met  private  respondent  Willy  Co  for  
the  first  time.  Petitioner  was  informed  by  Willy  Co  that  the  latter  had  bought  the  business  from  the  original  
partners  and  that  it  was  for  him  to  decide  whether  or  not  he  was  responsible  for  the  obligations  of  the  old  
partnership,  including  petitioner's  unpaid  salaries.  Petitioner  was  in  fact  not  allowed  to  work  anymore  in  
the  Jade  Mountain  business  enterprise.  His  unpaid  salaries  remained  unpaid.  
 
•   On   21   December   1988.   Yu   filed   a   complaint   for   illegal   dismissal   and   recovery   of   unpaid   salaries   accruing  
from  November  1984  to  October  1988,  moral  and  exemplary  damages  and  attorney's  fees,  against  Jade  
Mountain,  Mr.  Willy  Co  and  the  other  private  respondents.  
 
•   Petitioner’s  Contention:  a  partnership  has  a  juridical  personality  separate  and  distinct  from  that  of  each  of  
its  members.  Such  independent  legal  personality  subsists  notwithstanding  changes  in  the  identities  of  the  
partners.   Consequently,   the   employment   contract   between   Benjamin   Yu   and   the   partnership   Jade  
Mountain  could  not  have  been  affected  by  changes  in  the  latter's  membership.  
 

   
3H  A.Y.  2017-­‐2018   56  
 
 

CORPORATION  LAW  CASE  DIGESTS  –  ATTY.  DANTE  DELA  CRUZ  

•   Respondent’s   Contention:   The   partnership   and   Willy   Co   denied   petitioner's   charges,   contending   in   the  
main  that  Benjamin  Yu  was  never  hired  as  an  employee  by  the  present  or  new  partnership.  
 
•   Labor  Arbiter’s  Ruling:  rendered  a  decision  holding  that  petitioner  had  been  illegally  dismissed.  The  Labor  
Arbiter   decreed   his   reinstatement   and   awarded   him   his   claim   for   unpaid   salaries,   backwages   and  
attorney's  fees.  
 
•   NLRC’s  Ruling:  reversed  the  decision  of  the  Labor  Arbiter  and  dismissed  petitioner's  complaint.  The  NLRC  
held  that  a  new  partnership  consisting  of  Mr.  Willy  Co  and  Mr.  Emmanuel  Zapanta  had  bought  the  Jade  
Mountain   business,   that   the   new   partnership   had   not   retained   petitioner   Yu   in   his   original   position   as  
Assistant   General   Manager,   and   that   there   was   no   law   requiring   the   new   partnership   to   absorb   the  
employees   of   the   old   partnership.   Benjamin   Yu,   therefore,   had   not   been   illegally   dismissed   by   the   new  
partnership   which   had   simply   declined   to   retain   him   in   his   former   managerial   position   or   any   other  
position.  Finally,  the  NLRC  held  that  Benjamin  Yu's  claim  for  unpaid  wages  should  be  asserted  against  the  
original   members   of   the   preceding   partnership,   but   these   though   impleaded   had,   apparently,   not   been  
served  with  summons  in  the  proceedings  before  the  Labor  Arbiter.  
 
•   Hence,  this  Petition  for  Certiorari.  
 
ISSUES:  
(1)  W/N  partnership  which  had  hired  petitioner  Yu  as  Assistant  General  Manager  had  been  extinguished  and  
replaced  by  a  new  partnership  composed  of  Willy  Co  and  Emmanuel  Zapanta.  -­‐‑  YES  
(2)  If  indeed  a  new  partnership  had  come  into  existence,  whether  petitioner  Yu  could  nonetheless  assert  his  
rights  under  his  employment  contract  as  against  the  new  partnership.  -­‐‑  YES  
 
HELD:  
1.   The   legal   effect   of   the   changes   in   the   membership   of   the   partnership   was   the   dissolution   of   the   old  
partnership  which  had  hired  petitioner  in  1984  and  the  emergence  of  a  new  firm  composed  of  Willy  Co  and  
Emmanuel  Zapanta  in  1987.  
 
Art.  1828  -­‐‑  The  dissolution  of  a  partnership  is  the  change  in  the  relation  of  the  partners  caused  by  any  partner  
ceasing  to  be  associated  in  the  carrying  on  as  distinguished  from  the  winding  up  of  the  business.  
 
In  the  case  at  bar,  just  about  all  of  the  partners  had  sold  their  partnership  interests  (amounting  to  82%  of  the  
total  partnership  interest)  to  Mr.  Willy  Co  and  Emmanuel  Zapanta.  The  record  does  not  show  what  happened  
to  the  remaining  18%  of  the  original  partnership  interest.  The  acquisition  of  82%  of  the  partnership  interest  
by  new  partners,  coupled  with  the  retirement  or  withdrawal  of  the  partners  who  had  originally  owned  such  
82%  interest,  was  enough  to  constitute  a  new  partnership.  
 
The   occurrence   of   events   which   precipitate   the   legal   consequence   of   dissolution   of   a   partnership   do   not,  
however,  automatically  result  in  the  termination  of  the  legal  personality  of  the  old  partnership.  Article  1829  
of  the  Civil  Code  states  that  “on  dissolution  the  partnership  is  not  terminated,  but  continues  until  the  winding  
up  of  partnership  affairs  is  completed.”  
 
In  the  ordinary  course  of  events,  the  legal  personality  of  the  expiring  partnership  persists  for  the  limited  
purpose   of   winding   up   and   closing   of   the   affairs   of   the   partnership.   In   the   case   at   bar,   it   is   important   to  
underscore   the   fact   that   the   business   of   the   old   partnership   was   simply   continued   by   the   new   partners,  
without  the  old  partnership  undergoing  the  procedures  relating  to  dissolution  and  winding  up  of  its  business  
affairs.   In   other   words,   the   new   partnership   simply   took   over   the   business   enterprise   owned   by   the  
preceeding   partnership,   and   continued   using   the   old   name   of   Jade   Mountain   Products   Company   Limited,  

   
3H  A.Y.  2017-­‐2018   57  
 
 

CORPORATION  LAW  CASE  DIGESTS  –  ATTY.  DANTE  DELA  CRUZ  

without   winding   up   the   business   affairs   of   the   old   partnership,   paying   off   its   debts,   liquidating   and  
distributing  its  net  assets,  and  then  re-­‐‑assembling  the  said  assets  or  most  of  them  and  opening  a  new  business  
enterprise.  
 
What  is  important  for  present  purposes  is  that,  under  the  above  described  situation,  not  only  the  retiring  
partners  (Rhodora  Bendal,  et  al.)  but  also  the  new  partnership  itself  which  continued  the  business  of  the  old  
are  liable  for  the  debts  of  the  preceding  partnership.    
 
In  Singson,  et  al.  v.  Isabela  Saw  Mill,  the  Court  held  that  under  facts  very  similar  to  those  in  the  case  at  bar,  a  
withdrawing   partner   remains   liable   to   a   third   party   creditor   of   the   old   partnership.   The   liability   of   the   new  
partnership,  upon  the  other  hand,  in  the  set  of  circumstances  obtaining  in  the  case  at  bar,  is  established  in  
Article  1840  of  the  Civil  Code.  
 
2.   Creditors   of   the   old   Jade   Mountain   are   also   creditors   of   the   new   Jade   Mountain   which   continued   the  
business   of   the   old   one   without   liquidation   of   the   partnership   affairs.   Indeed,   a   creditor   of   the   old   Jade  
Mountain,   like   petitioner   Benjamin   Yu   in   respect   of   his   claim   for   unpaid   wages,   is   entitled   to   priority   vis-­‐‑a-­‐‑
vis  any  claim  of  any  retired  or  previous  partner  insofar  as  such  retired  partner's  interest  in  the  dissolved  
partnership  is  concerned.  It  is,  however,  clear  to  the  Court  that  under  Article  1840  above,  Benjamin  Yu  is  
entitled  to  enforce  his  claim  for  unpaid  salaries,  as  well  as  other  claims  relating  to  his  employment  with  the  
previous  partnership,  against  the  new  Jade  Mountain.  
 
It  is  at  the  same  time  also  evident  to  the  Court  that  the  new  partnership  was  entitled  to  appoint  and  hire  a  
new  general  or  assistant  general  manager  to  run  the  affairs  of  the  business  enterprise  take  over.  An  assistant  
general   manager   belongs   to   the   most   senior   ranks   of   management   and   a   new   partnership   is   entitled   to  
appoint   a   top   manager   of   its   own   choice   and   confidence.   The   non-­‐‑retention   of   Benjamin   Yu   as   Assistant  
General   Manager   did   not   therefore   constitute   unlawful   termination,   or   termination   without   just   or  
authorized   cause.   We   think   that   the   precise   authorized   cause   for   termination   in   the   case   at   bar   was  
redundancy.  The  new  partnership  had  its  own  new  General  Manager,  apparently  Mr.  Willy  Co,  the  principal  
new   owner   himself,   who   personally   ran   the   business   of   Jade   Mountain.   Benjamin   Yu's   old   position   as  
Assistant  General  Manager  thus  became  superfluous  or  redundant.  It  follows  that  petitioner  Benjamin  Yu  is  
entitled  to  separation  pay  at  the  rate  of  one  month's  pay  for  each  year  of  service  that  he  had  rendered  to  the  
old  partnership,  a  fraction  of  at  least  6  months  being  considered  as  a  whole  year.  
 
While   the   new   Jade   Mountain   was   entitled   to   decline   to   retain   petitioner   Benjamin   Yu   in   its   employ,   we  
consider  that  Benjamin  Yu  was  very  shabbily  treated  by  the  new  partnership.  The  new  Jade  Mountain  did  
not  notify  him  of  the  change  in  ownership  of  the  business,  the  relocation  of  the  main  office  of  Jade  Mountain  
from  Makati  to  Mandaluyong  and  the  assumption  by  Mr.  Willy  Co  of  control  of  operations.  The  treatment  
(including  the  refusal  to  honor  his  claim  for  unpaid  wages)  accorded  to  Assistant  General  Manager  Benjamin  
Yu   was   so   summary   and   cavalier   as   to   amount   to   arbitrary,   bad   faith   treatment,   for   which   the   new   Jade  
Mountain   may   legitimately   be   required   to   respond   by   paying   moral   damages.   This   Court,   exercising   its  
discretion  and  in  view  of  all  the  circumstances  of  this  case,  believes  that  an  indemnity  for  moral  damages  in  
the  amount  of  P20,000.00  is  proper  and  reasonable.  
 
WHEREFORE,  petition  is  granted.  
 
 
 
43.  CEASE  VS  COURT  OF  APPEALS  
G.R.  NO.  L-­‐‑33172   OCTOBER  18,  1979  
GUERRERO,  J  

   
3H  A.Y.  2017-­‐2018   58  
 
 

CORPORATION  LAW  CASE  DIGESTS  –  ATTY.  DANTE  DELA  CRUZ  

 
FACTS:  
  Forrest  L.  Cease  common  predecessor  in  interest  of  the  parties  together  with  five  (5)  other  American  
citizens  organized  the  Tiaong  Milling  and  Plantation  Company  and  in  the  course  of  its  corporate  existence  
the   company   acquired   various   properties   but   at   the   same   time   all   the   other   original   incorporators   were  
bought  out  by  Forrest  L.  Cease  together  with  his  children  namely  Ernest,  Cecilia,  Teresita,  Benjamin,  Florence  
and  one  Bonifacia  Tirante;  the  charter  of  the  company  lapsed  in  June  1958;  but  whether  there  were  steps  to  
liquidate  it,  the  record  is  silent;  on  13  August  1959,  Forrest  L.  Cease  died  and  by  extrajudicial  partition  of  his  
shares,  among  the  children,  this  was  disposed  of  on  19  October  1959;  it  was  here  where  the  trouble  among  
them  came  to  arise  because  it  would  appear  that  Benjamin  and  Florence  wanted  an  actual  division  while  the  
other  children  wanted  reincorporation;  and  proceeding  on  that,  these  other  children  Ernesto,  Teresita  and  
Cecilia  and  aforementioned  other  stockholder  Bonifacia  Tirante  proceeded  to  incorporate  themselves  into  
the   F.L.   Cease   Plantation   Company   and   registered   it   with   the   Securities   and   Exchange   Commission;  
apparently  in  view  of  that,  Benjamin  and  Florence  for  their  part  initiated  a  Special  Proceeding  No.  3893  of  
the  Court  of  First  Instance  of  Tayabas  for  the  settlement  of  the  estate  of  Forest  L.  Cease  on  21  April,  1960  and  
one  month  afterwards  on  19  May  1960  they  filed  Civil  Case  No.  6326  against  Ernesto,  Teresita  and  Cecilia  
Cease  together  with  Bonifacia  Tirante  asking  that  the  Tiaong  Milling  and  Plantation  Corporation  be  declared  
Identical   to   F.L.   Cease   and   that   its   properties   be   divided   among   his   children   as   his   intestate   heirs;   this   Civil  
Case   was   resisted   by   aforestated   defendants   and   notwithstanding   efforts   of   the   plaintiffs   to   have   the  
properties   placed   under   receivership,   they   were   not   able   to   succeed   because   defendants   filed   a   bond   to  
remain  as  they  have  remained  in  possession;  after  that  and  already,  during  the  pendency  of  Civil  Case  No.  
6326  specifically  on  21  May,  1961  apparently  on  the  eve  of  the  expiry  of  the  three  (3)  year  period  provided  
by   the   law   for   the   liquidation   of   corporations,   the   board   of   liquidators   of   Tiaong   Milling   executed   an  
assignment  and  conveyance  of  properties  and  trust  agreement  in  favor  of  F.L.  Cease  Plantation  Co.  Inc.  as  
trustee  of  the  Tiaong  Milling  and  Plantation  Co.  so  Chat  upon  motion  of  the  plaintiffs  trial  Judge  ordered  that  
this  alleged  trustee  be  also  included  as  party  defendant;  now  this  being  the  situation,  it  will  be  remembered  
that  there  were  thus  two  (2)  proceedings  pending  in  the  Court  of  First  Instance  of  Quezon  namely  Civil  Case  
No.  6326  and  Special  Proceeding  No.  3893  but  both  of  these  were  assigned  to  the  Honorable  Respondent  
Judge  Manolo  L.  Maddela  p.  43  and  the  case  was  finally  heard  and  submitted  upon  stipulation  of  facts  pp,  34-­‐‑
110,  rollo;  and  trial  Judge  by  decision  dated  27  December  1969  held  for  the  plaintiffs  Benjamin  and  Florence.  
 
ISSUE:  
  Whether  the  registered  properties  of  Tiaong  Milling  are  also  properties  of  the  estate  of  Forrest  L.  
Cease.  
 
HELD:  
  Yes.  An  indubitable  deduction  from  the  findings  of  the  trial  court  cannot  but  lead  to  the  conclusion  
that  the  business  of  the  corporation  is  largely,  if  not  wholly,  the  personal  venture  of  Forrest  L.  Cease.  There  
is  not   even   a  shadow  of   a  showing   that  his  children   were   subscribers   or   purchasers   of   the   stocks   they  own.  
Their   participation   as   nominal   shareholders   emanated   solely   from   Forrest   L.   Cease's   gratuitous   dole   out   of  
his  own  shares  to  the  benefit  of  his  children  and  ultimately  his  family.    
  Were  we  sustain  the  theory  of  petitioners  that  the  trial  court  acted  in  excess  of  jurisdiction  or  abuse  
of  discretion  amounting  to  lack  of  jurisdiction  in  deciding  Civil  Case  No.  6326  as  a  case  for  partition  when  the  
defendant   therein,   Tiaong   Milling   and   Plantation   Company,   Inc.   as   registered   owner   asserted   ownership   of  
the   assets   and   properties   involved   in   the   litigation,   which   theory   must   necessarily   be   based   on   the  
assumption  that  said  assets  and  properties  of  Tiaong  Milling  and  Plantation  Company,  Inc.  now  appearing  
under  the  name  of  F.  L.  Cease  Plantation  Company  as  Trustee  are  distinct  and  separate  from  the  estate  of  
Forrest   L.   Cease   to   which   petitioners   and   respondents   as   legal   heirs   of   said   Forrest   L.   Cease   are   equally  
entitled  share  and  share  alike,  then  that  legal  fiction  of  separate  corporate  personality  shall  have  been  used  
to  delay  and  ultimately  deprive  and  defraud  the  respondents  of  their  successional  rights  to  the  estate  of  their  

   
3H  A.Y.  2017-­‐2018   59  
 
 

CORPORATION  LAW  CASE  DIGESTS  –  ATTY.  DANTE  DELA  CRUZ  

deceased  father.   For   Tiaong   Milling  and  Plantation   Company  shall  have  been  able  to  extend  its  corporate  
existence  beyond  the  period  of  its  charter  which  lapsed  in  June,  1958  under  the  guise  and  cover  of  F.  L,  Cease  
Plantation  Company,  Inc.  as  Trustee  which  would  be  against  the  law,  and  as  Trustee  shall  have  been  able  to  
use  the  assets  and  properties  for  the  benefit  of  the  petitioners,  to  the  great  prejudice  and  defraudation.  of  
private  respondents.  Hence,  it  becomes  necessary  and  imperative  to  pierce  that  corporate  veil.  
 
 
 
44.  NO  CASE  
 
 
 
45.  JARDINE  DAVIES,  INC.  VS.  JRB  REALTY,  INC.,  
G.R.  NO.  151438  
CALLEJO,  SR.,  J.:  
 
 
FACTS:    
In  1979-­‐‑1980,  respondent  JRB  Realty,  Inc.  built  a  nine-­‐‑storey  building,  named  Blanco  Center,  on  its  
parcel  of  land  located  in  Makati  City.  An  air  conditioning  system  was  needed  for  the  Blanco  Law  Firm  housed  
at   the   second   floor   of   the   building.   The   respondents’   Executive   Vice-­‐‑President,   accepted   the   contract  
quotation  of  Mr.  A.G.  Morrison,  President  of  Aircon  and  Refrigeration  Industries,  Inc.  (Aircon),  for  two  (2)  
sets  of  air  conditioning  equipment  with  a  net  total  selling  price  of  P99,586.00.  Thereafter,  two  (2)  brand  new  
packaged  air  conditioners  were  installed  by  Aircon.  When  the  units  with  rotary  compressors  were  installed,  
they   could   not   deliver   the   desired   cooling   temperature.   Despite   several   adjustments   and   corrective  
measures,  the  respondent  conceded  that  Fedders  Air  Conditioning  USAs  technology  for  rotary  compressors  
for  big  capacity  conditioners  like  those  installed  at  the  Blanco  Center  had  not  yet  been  perfected.  The  parties  
thereby  agreed  to  replace  the  units  with  reciprocating/semi-­‐‑hermetic  compressors  instead.  In  a  Letter  dated  
March  26,  1981,   Aircon  stated  that  it  would  be  replacing  the  units  currently  installed  with  new  ones  using  
rotary   compressors,   at   the   earliest   possible   time.   Regrettably,   however,   it   could   not   specify   a   date   when  
delivery  could  be  effected.  
TempControl   Systems,   Inc.   (a   subsidiary   of   Aircon   until   1987)   undertook   the   maintenance   of   the  
units,  inclusive  of  parts  and  services.  In  October  1987,  the  respondent  learned,  through  newspaper  ads,  that  
Maxim   Industrial   and   Merchandising   Corporation   (Maxim,   for   short)   was   the   new   and   exclusive   licensee   of  
Fedders   Air   Conditioning   USA   in   the   Philippines   for   the   manufacture,   distribution,   sale,   installation   and  
maintenance   of   Fedders   air   conditioners.   The   respondent   requested   that   Maxim   honor   the   obligation   of  
Aircon,  but  the  latter  refused.  Considering  that  the  ten-­‐‑year  period  of  prescription  was  fast  approaching,  an  
action   for   specific   performance   with   damages   against   Aircon   &   Refrigeration   Industries,   Inc.,   Fedders   Air  
Conditioning   USA,   Inc.,   Maxim   Industrial   &   Merchandising   Corporation   and   petitioner   Jardine   Davies,  
Inc.  The  latter  was  impleaded  as  defendant,  considering  that  Aircon  was  a  subsidiary  of  the  petitioner.    
The  RTC  rendered  its  Decision,  finding  Jardine,  Fedders  and  Maxim  liable  to    JBR.  The  petitioner  filed  
its  notice  of  appeal  with  the  CA,  alleging  that  the  trial  court  erred  in  holding  it  liable  because  it  was  not  a  
party  to  the  contract  between  JRB  Realty,  Inc.  and  Aircon,  and  that  it  had  a  personality  separate  and  distinct  
from  that  of  Aircon.  On  March  23,  2000,  the  CA  affirmed  the  trial  courts  ruling  in  toto;  hence,  this  petition.  
 
ISSUE:    
Whether  or  not  Jardine  Davies,  Inc.  has  a  personality  separate  and  distinct  from  that  of  Aircon.  
 
HELD:    

   
3H  A.Y.  2017-­‐2018   60  
 
 

CORPORATION  LAW  CASE  DIGESTS  –  ATTY.  DANTE  DELA  CRUZ  

 The  petitioner  is  now  before  us,  reiterating  its  defense  of  separateness,  and  the  fact  that  it  
is  not  a  party  to  the  contract.  We  find  merit  in  the  petition.  
It  is  an  elementary  and  fundamental  principle  of  corporation  law  that  a  corporation  is  an  artificial  
being   invested   by   law   with   a   personality   separate   and   distinct   from   its   stockholders   and   from   other  
corporations  to  which  it  may  be  connected.  While  a  corporation  is  allowed  to  exist  solely  for  a  lawful  purpose,  
the  law  will  regard  it  as  an  association  of  persons  or  in  case  of  two  corporations,  merge  them  into  one,  when  
this  corporate  legal  entity  is  used  as  a  cloak  for  fraud  or  illegality.  This  is  the  doctrine  of  piercing  the  veil  of  
corporate  which  applies  only  when  such  corporate  fiction  is  used  to  defeat  public  convenience,  justify  wrong,  
protect  fraud  or  defend  crime.    
While  it  is  true  that  Aircon  is  a  subsidiary  of  the  petitioner,  it  does  not  necessarily  follow  that  Aircons’  
corporate  legal  existence  can  just  be  disregarded.  In  applying  the  doctrine,  the  following  requisites  must  be  
established:  (1)  control,  not  merely  majority  or  complete  stock  control;  (2)  such  control  must  have  been  used  
by  the  defendant  to  commit  fraud  or  wrong,  to  perpetuate  the  violation  of  a  statutory  or  other  positive  legal  
duty,  or  dishonest  acts  in  contravention  of  plaintiffs  legal  rights;  and  (3)  the  aforesaid  control  and  breach  of  
duty  must  proximately  cause  the  injury  or  unjust  loss  complained  of.    
The  records  bear  out  that  Aircon  is  a  subsidiary  of  the  petitioner  only  because  the  latter  acquired  
Aircons  majority  of  capital  stock.  It,  however,  does  not  exercise  complete  control  over  Aircon;  nowhere  can  
it  be  gathered  that  the  petitioner  manages  the  business  affairs  of  Aircon.  Indeed,  no  management  agreement  
exists  between  the  petitioner  and  Aircon,  and  the  latter  is  an  entirely  different  entity  from  the  petitioner.    
   The  existence  of  interlocking  directors,  corporate  officers  and  shareholders,  which  the  respondent  
court  considered,  is  not  enough  justification  to  pierce  the  veil  of  corporate  fiction,  in  the  absence  of  fraud  or  
other   public   policy   considerations.   Any   piercing   of   the   corporate   veil   has   to   be   done   with   caution.   The  
wrongdoing  must  be  clearly  and  convincingly  established.  It  cannot  just  be  presumed.    
In   the   instant   case,   there   is   no   evidence   that   Aircon   was   formed   or   utilized   with   the   intention   of  
defrauding  its  creditors  or  evading  its  contracts  and  obligations.  There  was  nothing  fraudulent  in  the  acts  of  
Aircon   in   this   case.   Aircon,   as   a   manufacturing   firm   of   air   conditioners,   complied   with   its   obligation   of  
providing   air    
conditioning  units  for  the  second  floor  of  the  Blanco  Center  in  good  faith,  pursuant  to  its  contract  with  the  
respondent.  
We  sustain  the  petitioners’  separateness  from  that  of  Aircon  in  this  case.  It  bears  stressing  that  the  
petitioner   was   never   a   party   to   the   contract.   Privity   of   contracts   take   effect   only   between   parties,   their  
successors-­‐‑in-­‐‑interest,   heirs   and   assigns.   The   petitioner   which   has   a   separate   and   distinct   legal   personality  
from  that  of  Aircon,  cannot,  therefore,  be  held  liable.  
   
 
46.  KOPPEL  (PH),  INC.,  KOPPEL  (PH),  INC.-­‐‑KOPPEL  (PH),  INC,  VS  .  ALFREDO  L.  
YATCO,  COLLECTOR  OF  INTERNAL  REVENUE,  CIR-­‐‑APPELLEE  
G.R.  NO.  L-­‐‑47673.  OCTOBER  10,  1946  
HILADO,  J  
 
DOCTRINE:    
A  corporation  will  be  looked  upon  as  a  legal  entity  as  a  general  rule,  and  until  sufficient  reason  to  the  contrary  
appears;  but,  when  the  notion  of  legal  entity  is  used  to  defeat  public  convenience,  justify  wrong,  protect  fraud,  
or  defend  crime,  the  law  will  regard  the  corporation  as  an  association  of  persons.  
 
Control  by  another  corporation  -­‐‑    The  corporate  entity  is  disregard  where  it  is  so  organized  and  controlled,  
and  its  affairs  are  so  conducted,  as  to  make  it  merelan  instrumentality,  agency,  conduit  or  adjunct  of  another  
corporation.  
 
FACTS:    

   
3H  A.Y.  2017-­‐2018   61  
 
 

CORPORATION  LAW  CASE  DIGESTS  –  ATTY.  DANTE  DELA  CRUZ  

On  February,  1929,  Miguel  J.  Ossorio,  of  Manila,  Philippines,  placed  an  option  with  Koppel  Industrial  Car  and  
Equipment  Company,  through  Koppel  (Philippines),  Inc.,  to  purchase  within  three  months  a  pair  of  Atlas-­‐‑
Diesel   Marine   Engines.   Koppel   Industrial   Car   and   Equipment   Company   purchased   said   Diesel   Engines   in  
Stockholm,   Sweden,   for   $16,508.32.   Later,   Miguel   J.   Ossorio   definitely   called   the   deal   off,   and   as   Koppel  
Industrial  Car  and  Equipment  Company  could  not  ship  to  or  draw  on  said  Mr.  Miguel  J.  Ossorio,  it  in  turn  
drew   another   draft   on   Koppel   (Philippines),   Inc.   for   the   same   amount   at   six   months   sight,   with   the  
understanding   that   Koppel   Industrial   Car   and   Equipment   Company   would   reimburse   Koppel   (Philippines),  
Inc.  when  said  engines  were  disposed  of.  Koppel  (Philippines),  Inc.  honored  the  draft  and  debited  the  said  
sum  of  $16,508.32  to  merchandise  account.  The  engines  were  left  stored  at  Stockholm,  Sweden.    
 
On  April  1,  1930,  a  new  local  buyer,  Mr.  Cesar  Barrios,  of  Iloilo,  Philippines,  was  found  and  the  same  engines  
were  sold  to  him  for  $21,000  (P42,000)  C.  I.  F.  Hongkong.  The  engines  were  shipped  to  Hongkong  and  a  draft  
for  $21,000  was  drawn  by  Koppel  Industrial  Car  and  Equipment  Company  on  Mr.  Cesar  Barrios.  After  the  
draft   was   fully   paid   by   Mr.   Barrios,   Koppel   Industrial   Car   and   Equipment   Company   reimbursed   Koppel  
(Philippines),  Inc.  with  cost  price  of  $16,508.32  and  credited  it  with  $1,152.95  as  its  share  of  the  profit  on  the  
transaction.    
 
Koppel   (Philippines),   Inc.'s   share   in   the   profits   realized   out   of   these   transactions   were   totaling  
P3,772,403.82,  amounts  to  P132,201.30;  and  that  Koppel  (Philippines),  Inc.  within  the  time  provided  by  law  
returned  the  aforesaid  amount  P132,201.30  for  the  purpose  of  the  commercial  broker's  4  per  cent  tax  and  
paid  thereon  the  sum  P5,288.05  as  such  tax.  
 
CIR  demanded  of  the  Koppel  (Philippines),  Inc.  the  sum  of  P64,122.51  as  the  merchants'  sales  tax  of  1%  per  
cent  on  the  amount  of  P3,772,403.82,  representing  the  total  gross  value  of  the  sales  mentioned  in  the  above  
transaction,  including  the  25  per  cent  surcharge  for  the  late  payment  of  the  said  tax,  which  tax  and  surcharge  
were  determined  after  the  amount  of  P5,288.05  (broker's  4  per  cent  tax)  was  deducted.  
 
Koppel  (Philippines),  Inc.,  on  October  30,  1936,  paid  under  protest  said  sum  of  P64,122.51  in  order  to  avoid  
further  penalties,  levy  and  distraint  proceedings.  
 
The   lower   court   found   and   held   that   Koppel   (Philippines),   Inc.   is   a   mere   dummy   or   brach   ("hechura")   of  
Koppel  industrial  Car  and  Equipment  Company.  The  lower  court  did  not  deny  legal  personality  to  Koppel  
(Philippines),  Inc.  for  any  and  all  purposes,  but  in  effect  its  conclusion  was  that,  in  the  transactions  involved  
herein,   the   public   interest   and   convenience   would   be   defeated   and   what   would   amount   to   a   tax   evasion  
perpetrated,  unless  resort  is  had  to  the  doctrine  of  "disregard  of  the  corporate  fiction."  
 
ISSUE:  
(a)   WON   the   lower   court   disregarded   the   principle   that   a   corporation’s   legal   personality   cannot   be  
collaterally  attacked.  
(b)  WON  the  lower  court  is  justified  in  disregarding  the  corporate  fiction.  
 
HELD:    
(a)   No.  The  lower  court  did  not  deny  legal  personality  to  Koppel  (Philippines),  Inc.  but  held  in  effect  that  
in  the  transaction  involved  in  this  case  the  public  interest  and  convenience  would  be  defeated  and  
what  would  amount  to  a  tax  evasion  perpetrated,  unless  resort  is  had  to  the  doctrine  of  "disregard  of  
the  corporate  fiction."  In  other  words,  in  looking  through  the  corporate  form  to  the  ultimate  person  
or   corporation   behind   that   form,   in   the   particular   transactions   which   were   involved   in   the   case  
submitted  to  its  determination  and  judgment,  the  court  did  so  in  order  to  prevent  the  contravention  
of   the   local   internal   revenue   laws,   and   the   perpetration   of   what   would   amount   to   a   tax   evasion,  

   
3H  A.Y.  2017-­‐2018   62  
 
 

CORPORATION  LAW  CASE  DIGESTS  –  ATTY.  DANTE  DELA  CRUZ  

inasmuch  as  it  considered  that  Koppel  (Philippines),  Inc  Koppel  (Philippines),  Inc.  was  a  mere  branch  
or  agency  or  dummy  ("hechura")  of  Koppel  Industrial  Car  and  Equipment  Co.    
 
The   court   did   not   hold   that   the   corporate   personality   of   Koppel   (Philippines),   Inc.,   would   also   be  
disregarded  in  other  cases  or  for  other  purposes.  It  would  have  had  no  power  to  so  hold.  The  courts'  
action  in  this  regard  must  be  confined  to  the  transactions  involved  in  the  case  at  bar  "for  the  purpose  
of  adjudging  the  rights  and  liabilities  of  the  parties  in  the  case.  They  have  no  jurisdiction  to  do  more."  
 
A  corporation  will  be  looked  upon  as  a  legal  entity  as  a  general  rule,  and  until  sufficient  reason  to  the  
contrary  appears;  but,  when  the  notion  of  legal  entity  is  used  to  defeat  public  convenience,  justify  
wrong,   protect   fraud,   or   defend   crime,   the   law   will   regard   the   corporation   as   an   association   of  
persons.  
 
(b)  Yes.   Generally,   the   entity   is   normally   regarded   but   is   disregarded   to   prevent   injustice,   or   the  
distortion  or  hiding  of  the  truth,  or  to  let  in  a  just  defense.  Another  rule  is  that,  when  the  corporation  
is  the  mere  alter  ego,  or  business  conduit  of  a  person,  it  may  de  disregarded."    
 
The  principle  is  the  same  whether  the  "person"  be  natural  or  artificial.  A  very  numerous  and  growing  
class  of  cases  wherein  the  corporate  entity  is  disregarded  is  that  (it  is  so  organized  and  controlled,  
and   its   affairs   are   so   conducted,   as   to   make   it   merely   an   instrumentality,   agency,   conduit   or   adjunct  
of  another  corporation)."  While  we  recognize  the  legal  principle  that  a  corporation  does  not  lose  its  
entity  by  the  ownership  of  the  bulk  or  even  the  whole  of  its  stock,  by  another  corporation.  Where  it  
appears  that  two  business  enterprises  are  owned,  conducted  and  controlled  by  the  same  parties,  both  
law  and  equity  will,  when  necessary  to  protect  the  rights  of  third  persons,  disregard  the  legal  fiction  
that  two  corporations  are  distinct  entities,  and  treat  them  as  identical.  The  legal  fiction  of  distinct  
corporate  existence  will  be  disregarded  in  a  case  where  a  corporation  is  so  organized  and  controlled  
and   its   affairs   are   so   conducted,   as   to   make   it   merely   an   instrumentality   or   adjunct   of   another  
corporation.    
 
In  so  far  as  the  sales  involved  herein  are  concerned,  Koppel  (Philippines),  Inc.,  and  Koppel  Industrial  
Car   and   Equipment   company   are   to   all   intents   and   purposes   one   and   the   same;   or,   as   regards   those  
transactions,   the   former   corporation   is   a   mere   branch,   subsidiary   or   agency   of   the   latter.   This   is  
conclusively  borne  out  by  the  fact,  among  others,  that  the  so-­‐‑called  "share  in  the  profits"  of  Koppel  
(Philippines),   Inc.,   was   ultimately   left   to   the   sole,   unbridled   control   of   Koppel   Industrial   Car   and  
Equipment  Company.  If,  in  their  relations  with  each  other,  Koppel  (Philippines),  Inc.,  was  considered  
and   intended   to   function   as   a   bona   fide   separate   corporation,   we   can   not   conceive   how   this  
arrangement  could  have  been  adopted,  for  if  there  was  any  factor  in  its  business  as  to  which  it  would  
in  that  case  naturally  have  been  opposed  to  being  thus  controlled,  it  must  have  been  precisely  the  
amount   of   profit   which   it   could   endeavor   and   hope   to   earn.   No   group   of   businessmen   could   be  
expected  to  organize  a  mercantile  corporation  —  the  ultimate  end  of  which  could  only  be  profit  —  if  
the  amount  of  that  profit  were  to  be  subjected  to  such  a  unilateral  control  of  another  corporation,  
unless   indeed   the   former   has   previously   been   designed   by   the   incorporators   to   serve   as   a   mere  
subsidiary,  branch  or  agency  of  the  latter.  Evidently,  Koppel  Industrial  Car  and  Equipment  Company  
made  us  of  its  ownership  of  the  overwhelming  majority  —  99.5%  —  of  the  capital  stock  of  the  local  
corporation   to   control   the   operations   of   the   latter   to   such   an   extent   that   it   had   the   final   say   even   as  
to  how  much  should  be  allotted  to  said  local  entity  in  the  so-­‐‑called  sharing  in  the  profits.  We  can  not  
overlook  the  fact  that  in  the  practical  working  of  corporate  organizations  of  the  class  to  which  these  
two   entities   belong,   the   holder   or   holders   of   the   controlling   part   of   the   capital   stock   of   the  
corporation,  particularly  where  the  control  is  determined  by  the  virtual  ownership  of  the  totality  of  
the  shares,  dominate  not  only  the  selection  of  the  Board  of  Directors  but,  more  often  than  not,  also  

   
3H  A.Y.  2017-­‐2018   63  
 
 

CORPORATION  LAW  CASE  DIGESTS  –  ATTY.  DANTE  DELA  CRUZ  

the  action  of  that  Board.  Applying  this  to  the  instant  case,  we  can  not  conceive  how  the  Philippine  
corporation   could   effectively   go   against   the   policies,   decisions,   and   desires   of   the   American  
corporation   with   regards   to   the   scheme   which   was   devised   through   the   instrumentality   of   the  
contract  Exhibit  H,  as  well  as  all  the  other  details  of  the  system  which  was  adopted  in  order  to  avoid  
paying  the  1½  per  cent  merchants  sales  tax.  Neither  can  we  conceive  how  the  Philippine  corporation  
could  avoid  following  the  directions  of  the  American  corporation  held  99.5  per  cent  of  the  capital  
stock  of  the  Philippine  corporation.  In  the  present  instance,   we   note   that   Koppel   (Philippines),   Inc.,  
was  represented  in  the  Philippines  by  its  "resident  Vice-­‐‑President."  This  fact  necessarily  leads  to  the  
inference  that  the  corporation  had  at  least  a  Vice-­‐‑President,  and  presumably  also  a  President,  who  
were  not  resident  in  the  Philippines  but  in  America,  where  the  parent  corporation  is  domiciled.  If  
Koppel   (Philippines),   Inc.,   had   been   intended   to   operate   as   a   regular   domestic   corporation   in   the  
Philippines,  where  it  was  formed,  the  record  and  the  evidence  do  not  disclose  any  reason  why  all  its  
officers  should  not  reside  and  perform  their  functions  in  the  Philippines.  
 
As   already   stated   above,   under   the   evidence   the   sales   in   the   Philippines   of   the   railway   materials,  
machinery  and  supplies  imported  here  by  Koppel  Industrial  Car  and  Equipment  Company  could  have  
been  as  conveniently  and  efficiently  transacted  and  handled  —  if  not  more  so  —  had  said  corporation  
merely  established  a  branch  or  agency  in  the  Philippines  and  obtained  license  to  do  business  locally;  
and  if  it  had  done  so  and  said  sales  had  been  effected  by  such  branch  or  agency,  there  seems  to  be  no  
dispute  that  the  1½  per  cent  merchants'  sales  tax  then  in  force  would  have  been  collectible.  So  far  as  
we   can   discover,   there   would   be   only   one,   but   very   important,   difference   between   the   two   schemes  
—  a  difference  in  tax  liability  on  the  ground  that  the  sales  were  made  through  another  and  distinct  
corporation,  as  alleged  broker,  when  we  have  seen  that  this  latter  corporation  is  virtually  owned  by  
the  former,  or  that  they  practically  one  and  the  same,  is  to  sanction  a  circumvention  of  our  tax  laws,  
and  permit  a  tax  evasion  of  no  mean  proportions  and  the  consequent  commission  of  a  grave  injustice  
to  the  Government.  Not  only  this;  it  would  allow  the  taxpayer  to  do  by  indirection  what  the  tax  laws  
prohibited  to  be  done  directly  (non-­‐‑payment  of  legitimate  taxes).  
 
 
 
47.  LIDDELL  &  CO.,  INC  VS.  THE  COLLECTOR  OF  INTERNAL  REVENUE  
G.R.  NO.  L-­‐‑9687.  JUNE  30,  1961  
BENGZON,  C.J.    
 
Doctrine:   Where   a   corporation   is   a   dummy   and   serves   no   business   purpose   and   is   intended   only   as   a   blind,  
the  corporate  form  may  be  ignored.  
 
Facts:  Petitioner,  Liddell  &  Co.  Inc.,  is  a  domestic  corporation  established  in  PH  on  February  1,  1946,  with  an  
authorized  capital  of  P100,000  divided  into  1000  shares  at  P100  each.  Of  this  authorized  capital,  196  shares  
valued  at  P19,600  were  subscribed  and  paid  by  Frank  Liddell  while  the  other  four  shares  were  in  the  name  
of  Charles  Kurz,  E.  J.  Darras,  Angel  Manzano  and  Julian  Serrano  at  one  share  each.  Its  purpose  was  to  engage  
in  the  business  of  importing  and  retailing  Oldsmobile  and  Chevrolet  passenger  cars  and  GMC  and  Chevrolet  
trucks.  
 
On  January  31,  1947,  with  the  limited  paid-­‐‑in  capital  of  P20,000,  Liddell  &  Co.  was  able  to  declare  a  90%  
stock  dividend  after  which  declaration,  Frank  Liddell's  holdings  in  the  company  increased  to  1,960  shares  
and   the   employees,   Charles   Kurz,   E.J.   Darras,   Angel   Manzano   and   Julian   Serrano   at   10   shares   each.   The  
declaration  of  stock  dividend  was  followed  by  a  resolution  increasing  the  authorized  capital  of  the  company  
to  P1,000,000  which  the  SEC  approved.  Upon  such  approval,  Frank  Liddell  subscribed  to  3,000  additional  
shares,  for  which  he  paid  into  the  corporation  P300,000  so  that  he  had  in  his  own  name  4,960  shares.  

   
3H  A.Y.  2017-­‐2018   64  
 
 

CORPORATION  LAW  CASE  DIGESTS  –  ATTY.  DANTE  DELA  CRUZ  

 
On   May   24,   1947,   Frank   Liddell,   on   one   hand   and   Messrs.   Kurz,   Darras,   Manzano   and   Serrano   on   the   other,  
executed   an   agreement   (The   Agreement)   which   was   further   supplemented   by   two   other   agreements,  
wherein  Frank  Liddell  transferred  to  various  employees  of  Liddell  &  Co.  shares  of  stock.  
 
At  the  annual  meeting  of  stockholders,  a  100%  stock  dividend  was  declared,  thereby  increasing  the  issued  
capital   stock   of   said   corporation   to   P1,000,000.   The   stockholders   also   approved   a   resolution   increasing  
the   authorized   capital   stock   from   P1,000,000   to   P3,000,000   which   increase   was   duly   approved   by   the   SEC.  
Frank  Liddell  subscribed  to  and  paid  20%  of  the  increase  of  P400,000.  He  paid  25%  thereof  in  the  amount  of  
P100,000   and   the   balance   of   P300,000   was   merely   debited   to   Frank   Liddell-­‐‑Drawing   Account   and   credited  
to   Subscribed   Capital   Stock   on   December   31,   1948.   Thereafter,   there   were   several   stock   dividends  
declaration  which  followed.  
 
The  supplemental  agreements  to  The  Agreement  provided  for  the  manner  by  which  they  sought  to  prevent  
their   shareholdings   from   being   transferred   to   others   who   may   be   complete   strangers   to   the   business   of  
Liddell  &  Co.  
 
From  1946-­‐‑1948  when  the  purpose  clause  of  the  Articles  of  Incorporation  of  Liddell  &  Co.  Inc.,  was  amended  
so  as  to  limit  its  business  activities  to  importations  of  automobiles  and  trucks,  Liddell  &  Co.  was  engaged  in  
business  as  an  importer  and  at  the  same  time  retailer  of  Oldsmobile  and  Chevrolet  passenger  cars  and  GMC  
and  Chevrolet  trucks.  
 
On  December  20,  1948,  the  Liddell  Motors,  Inc.  was  organized  and  registered  with  the  SEC  with  an  authorized  
capital  stock  of  P100,000  of  which  P20,000  was  subscribed  and  paid  for  as  follows:  Irene  Liddell,  wife  of  
Frank  Liddell,  19,996  shares  and  Messrs,  Marcial  P.  Lichauco,  E.  K.  Bromwell,  V.  E.  del  Rosario  and  Esmenia  
Silva,  1  share  each.  
 
At   about   the   end   of   the   year   1948,   Messrs.   Manzano,   Kurz   and   Kernot   resigned   from   their   respective  
positions  in  the  Retail  Dept.  of  Liddell  &  Co.  and  they  were  taken  in  and  employed  by  Liddell  Motors,  Inc.,  
Kurz  as  Manager-­‐‑Treasurer,  Manzano  as  General  Sales  Manager  for  cars  and  Kernot  as  General  Sales  Manager  
for  trucks.  
 
Beginning  January,  1949,  Liddell  &  Co.  stopped  retailing  cars  and  trucks;  it  conveyed  them  instead  to  Liddell  
Motors,  Inc.  which  in  turn  sold  the  vehicles  to  the  public  with  a  steep  mark-­‐‑up.  Since  then,  Liddell  &  Co.  paid  
sales  taxes  on  the  basis  of  its  sales  to  Liddell  Motors,  Inc.  considering  said  sales  as  its  original  sales.  
 
Upon  review  of  the  transactions  between  Liddell  &  Co.  and  Liddell  Motors  Inc.,  CIR  determined  that  the  latter  
was  but  an  alter  ego  of  Liddell  &  Co.  Wherefore,  he  concluded,  that  for  sales  tax  purposes,  those  sales  made  
by  Liddell  Motors,  Inc.  to  the  public  were  considered  as  the  original  sales  of  Liddell  &  Co.  Accordingly,  the  
CIR   assessed   against   Liddell   &   Co.   a   sales   tax   deficiency,   including   surcharges,   in   the   amount   of  
P1,317,629.61.    
 
Issue:   Whether   or   not   Liddell   &   Co.   Inc.,   and   the   Liddell   Motors   Inc.   are   (practically)   identical   corporations,  
the  latter  being  merely  the  alter  ego  of  the  former?  
 
Held:  Yes.  We  are  fully  convinced  that  Liddell  &  Co.  is  wholly  owned  by  Frank  Liddell.  As  of  the  time  of  its  
organization,  98%  of  the  capital  stock  belonged  to  Frank  Liddell.  The  20%  paid-­‐‑up  subscription  with  which  
the  company  began  its  business  was  paid  by  him.  The  subsequent  subscriptions  to  the  capital  stock  were  
made  by  him  and  paid  with  his  own  money.  
 

   
3H  A.Y.  2017-­‐2018   65  
 
 

CORPORATION  LAW  CASE  DIGESTS  –  ATTY.  DANTE  DELA  CRUZ  

The   stipulations   and   conditions   appearing   in   The   Agreement:   (1)   that   Frank   Liddell   had   the   authority   to  
designate  in  the  future  the  employee  who  could  receive  earnings  of  the  corporation;  to  apportion  among  the  
stockholders   the   share   in   the   profits;   (2)   that   all   certificates   of   stock   in   the   names   of   the   employees   should  
be  deposited  with  Frank  Liddell  duly  endorsed  in  blank  by  the  employees  concerned;  (3)  that  each  employee  
was   required   to   sign   an   agreement   with   the   corporation   to   the   effect   that,   upon   his   death   or   upon   his  
retirement  or  separation  for  any  cause  whatsoever  from  the  corporation,  the  said  corporation  should,  within  
a  period  of  sixty  days  therefor,  have  the  absolute  and  exclusive  option  to  purchase  and  acquire  the  whole  of  
the   stock   interest   of   the   employees   so   dying,   resigning,   retiring   or   separating.   These   stipulations   in   our  
opinion  attest  to  the  fact  that  Frank  Liddell  owned  Liddell  &  Co.  Inc.  They  guarantee  his  complete  control  
over  the  corporation.  
 
As  to  Liddell  Motors  Inc.  we  are  fully  persuaded  that  Frank  Liddell  also  owned  it.  He  supplied  the  original  
capital  funds.  It  is  not  proven  that  his  wife  Irene,  ostensibly  the  sole  incorporator  of  Liddell  Motors,  Inc.  had  
money  of  her  own  to  pay  for  her  P20,000  initial  subscription.  Her  income  in  the  United  States  in  the  years  
1943  and  1944  and  the  savings  therefrom  could  not  be  enough  to  cover  the  amount  of  subscription,  much  
less  to  operate  an  expensive  trade  like  the  retail  of  motor  vehicles.  The  alleged  sale  of  her  property  in  Oregon  
might  have  been  true,  but  the  money  received  therefrom  was  never  shown  to  have  been  saved  or  deposited  
so  as  to  be  still  available  at  the  time  of  the  organization  of  the  Liddell  Motors,  Inc.  
 
The  evidence  at  hand  also  shows  that  Irene  Liddell  had  scant  participation  in  the  affairs  of  Liddell  Motors,  
Inc.   She   could   hardly   be   said   to   possess   business   experience.   The   income   tax   forms   record   no   independent  
income  of  her  own.  As  a  matter  of  fact,  the  checks  that  represented  her  salary  and  bonus  from  Liddell  Motors,  
Inc.   found   their   way   into   the   personal   account   of   Frank   Liddell.   Her   frequent   absences   from   the   country  
negate  any  active  participation  in  the  affairs  of  the  Motors  company.  
 
There   are   quite   a   series   of   conspicuous   circumstances   that   militate   against   the   separate   and   distinct  
personality  of  Liddell  Motors,  Inc.  from  Liddell  &  Co.  We  notice  that  the  bulk  of  the  business  of  Liddell  &  Co.  
was   channeled   through   Liddell   Motors,   Inc.   On   the   other   hand,   Liddell   Motors,   Inc.   pursued   no   activities  
except  to  secure  cars,  trucks,  and  spare  parts  from  Liddell  &  Co.  Inc.  and  then  sell  them  to  the  general  public.  
These  sales  of  vehicles  by  Liddell  &  Co.  to  Liddell  Motors,  Inc.  for  the  most  part  were  shown  to  have  taken  
place  on  the  same  day  that  Liddell  Motors,  Inc.  sold  such  vehicles  to  the  public.  We  may  even  say  that  the  cars  
and  trucks  merely  touched  the  hands  of  Liddell  Motors,  Inc.  as  a  matter  of  formality.  
 
During  the  first  six  months  of  1949,  Liddell  &  Co.  issued  ten  (10)  checks  payable  to  Frank  Liddell  which  were  
deposited  by  Frank  Liddell  in  his  personal  account  with  the  PNB.  During  this  time  also,  he  issued  in  favor  of  
Liddell  Motors,  Inc.  six  (6)  checks  drawn  against  his  personal  account  with  the  same  bank.  The  checks  issued  
by  Frank  Liddell  to  the  Liddell  Motors,  Inc.  were  significantly  for  the  most  part  issued  on  the  same  day  when  
Liddell  &  Co.,  Inc.  issued  the  checks  for  Frank  Liddell  and  for  the  same  amounts.  
 
It  is  of  course  accepted  that  the  mere  fact  that  one  or  more  corporations  are  owned  and  controlled  by  a  single  
stockholder  is  not  of  itself  sufficient  ground  for  disregarding  separate  corporate  entities.  Authorities  support  
the  rule  that  it  is  lawful  to  obtain  a  corporation  charter,  even  with  a  single  substantial  stockholder,  to  engage  
in  a  specific  activity,  and  such  activity  may  co-­‐‑exist  with  other  private  activities  of  the  stockholder.  If  the  
corporation  is  a  substantial  one,  conducted  lawfully  and  without  fraud  on  another,  its  separate  identity  is  to  
be  respected.  
 
Accordingly,  the  mere  fact  that  Liddell  &  Co.  and  Liddell  Motors,  Inc.  are  corporations  owned  and  controlled  
by   Frank   Liddell   directly   or   indirectly   is   not   by   itself   sufficient   to   justify   the   disregard   of   the   separate  
corporate  identity  of  one  from  the  other.  There  is  however,  in  this  instant  case,  a  peculiar  consequence  of  the  
organization  and  activities  of  Liddell  Motors,  Inc.  

   
3H  A.Y.  2017-­‐2018   66  
 
 

CORPORATION  LAW  CASE  DIGESTS  –  ATTY.  DANTE  DELA  CRUZ  

 
48.  LA  CAMPANA  COFFEE  FACTORY,  INC.,  AND  TAN  TONG,  DOING  BUSINESS  UNDER  THE  TRADE  NAME  
"LA   CAMPANA   GAUGAU   PACKING”   VS.   KAISAHAN   NG   MGA   MANGGAGAWA   SA   LA   CAMPANA   (KKM)  
AND  THE  COURT  OF  INDUSTRIAL  RELATIONS  
G.R.  NO.  L-­‐‑5677.  MAY  25,  1953.  
REYES,  J.  
 
DOCTRINE:    The  doctrine  that  a  corporation  is  a  legal  entity  existing  separate  and  apart  from  the  persons  
composing  it  is  a  legal  theory  introduced  for  purposes  of  convenience  and  to  subserve  the  ends  of  justice.  
The  concept  cannot,  therefore,  be  extended  to  a  point  beyond  its  reason  and  policy,  and  when  invoked  in  
support  of  an  end  subversive  of  this  policy,  will  be  disregarded  by  the  courts.  
 
FACTS:    Tan  Tong  has  since  1932  been  engaged  in  the  business  of  buying  and  selling  gaugau  under  the  trade  
name  La  Campana  Gaugau  Packing.  On  July  6,  1950,  Tan  Tong,  with  himself  and  members  of  his  family  as  sole  
incorporators  and  stockholders,  organized  a  family  corporation  known  as  La  Campana  Coffee  Factory  Co.,  
Inc.,  with  its  principal  office  located  in  the  same  place  as  that  of  La  Campana  Gaugau  Packing  in  Quezon  City.  
A  year  prior  to  the  organization  of  La  Campana  Coffee,    Tan  Tong  entered  into  a  CBA  with  the  Philippine  
Legion  of  Organized  Workers,  known  as  PLOW  for  short,  to  which  the  union  of  Tan  Tong's  employees.  Later  
on,   his   employees   formed   Kaisahan   ng   mga   Manggagawa   ng   La   Campana   (Kaisihan)   and   applied   for  
registration  in  the  Department  of  Labor  as  an  independent  entity.  Pending  consideration  of  this  application,  
the  Department  gave  the  new  organization  legal  standing  by  issuing  it  a  permit  as  an  affliate  to  the  Kalipunan  
Ng  Mga  Kaisahang  Manggagawa.  Kaisahan  with  66  members  presented  a  demand  for  higher  wages  and  more  
privileges  to  La  Campana  Starch  and  Coffee  Factory.  The  demand  was  not  granted  and  the  DOLE  certified  the  
issue   to   the   CIR.   La   Campana   filed   a   motion   to   dismiss   alleging   that   the   action   was   directedagainst   two  
different  entities  with  distinct  personalities.  This  was  denied,  hence  this  petition.    
ISSUE:  Whether  or  not  CIR  has  jurisdiction  over  the  case  
 
HELD:  YES.  La  Campana  Gaugau  Packing  and  La  Campana  Coffee  Factory  Co.  Inc.,  are  operating  under  one  
single   management,   that   is,   as   one   business   though   with   two   trade   names.   True,   the   coffee   factory   is   a  
corporation  and,  by  legal  fiction,  an  entity  existing  separate  and  apart  from  the  persons  composing  it,  that  is,  
Tan  Tong  and  his  family.  But  it  is  settled  that  this  action  of  law,  which  has  been  introduced  as  a  matter  of  
convenience   and   to   subserve   the   ends   of   justice   cannot   be   invoked   to   further   an   end   subversive   of   that  
purpose.    
"Disregarding   Corporate   Entity.   —   The   doctrine   that   a   corporation   is   a   legal   entity  
existing   separate   and   apart   from   the   persons   composing   it   is   a   legal   theory   introduced   for  
purposes   of   convenience   and   to   subserve   the   ends   of   justice.   The   concept   cannot,   therefore,   be  
extended   to   a   point   beyond   its   reason   and   policy,   and   when   invoked   in   support   of   an   end  
subversive  of  this  policy,  will  be  disregarded  by  the  courts.  Thus,  in  an  appropriate  case  and  in  
furtherance  of  the  ends  of  justice,  a  corporation  and  the  individual  or  individuals  owning  all  its  
stocks  and  assets  will  be  treated  as  identical,  the  corporate  entity  being  disregarded  where  used  
as  a  cloak  or  cover  for  fraud  or  illegality.    
"  .  .  .  A  subsidiary  or  auxiliary  corporation  which  is  created  by  a  parent  corporation  merely  as  an  
agency  for  the  latter  may  sometimes  be  regarded  as  identical  with  the  parent  corporation,  especially  if  
the   stockholders   or   officers   of   the   two   corporations   are   substantially   the   same   or   their   system   of  
operation  unified.    
In  the  present  case  Tan  Tong  appears  to  be  the  owner  of  the  gaugau  factory.  And  the  coffee  factory,  
though  an  incorporated  business,  is  in  reality  owned  exclusively  by  Tan  Tong  and  his  family.  As  found  by  the  
CIR,  the  two  factories  have  but  one  office,  one  management  and  one  payroll,  except  after  July  17,  the  day  the  
case  was  certified  to  the  Court  of  Industrial  Relations,  when  the  person  who  was  discharging  the  office  of  
cashier  for  both  branches  of  the  business  began  preparing  separate  payrolls  for  the  two.  And  above  all,  it  

   
3H  A.Y.  2017-­‐2018   67  
 
 

CORPORATION  LAW  CASE  DIGESTS  –  ATTY.  DANTE  DELA  CRUZ  

should   not   be   overlooked   that,   as   also   found   by   the   industrial   court,   the   laborers   of   the   gaugau   factory   and  
the   coffee   factory   were   interchangeable,   that   is,   the   laborers   from   the   gaugau   factory   were   sometimes  
transferred  to  the  coffee  factory  and  vice-­‐‑versa.  In  view  of  all  these,  the  attempt  to  make  the  two  factories  
appear  as  two  separate  businesses,  when  in  reality  they  are  but  one,  is  but  a  device  to  defeat  the  ends  of  the  
law  (the  Act  governing  capital  and  labor  relations)  and  should  not  be  permitted  to  prevail.    
 
 
 
49.  WPM  INTERNATIONAL  TRADING,  INC  VS.  FE  CORAZON  LABAYEN  
G.R.  NO.  182770.  SEPTEMBER  17,  2014.  
BRION,  J  
 
DOCTRINE:    
The  mere  ownership  by  a  single  stockholder  of  even  all  or  nearly  all  of  the  capital  stocks  of  a  corporation  is  
not  by  itself  a  sufficient  ground  to  disregard  the  separate  corporate  personality.  The  control  necessary  to  
invoke   the   instrumentality   or   alter   ego   rule   is   not   majority   or   even   complete   stock   control   but   such  
domination  of  finances,  policies  and  practices  that  the  controlled  corporation  has,  so  to  speak,  no  separate  
mind,   will   or   existence   of   its   own,   and   is   but   a   conduit   for   its   principal.   The   control   must   be   shown   to   have  
been  exercised  at  the  time  the  acts  complained  of  took  place.  Moreover,  the  control  and  breach  of  duty  must  
proximately  cause  the  injury  or  unjust  loss  for  which  the  complaint  is  made.    
 
FACTS:    
The   respondent,   Fe   Corazon   Labayen,   is   the   owner   of   H.B.O.   Systems   Consultants,   a   management   and  
consultant  firm.  The  petitioner,  WPM  International  Trading,  Inc.  (WPM),  is  a  domestic  corporation  engaged  
in   the   restaurant   business,   while   Warlito   P.   Manlapaz   (Manlapaz)   is   its   president.   WPM   entered   into   a  
management  agreement  with  the  respondent,  by  virtue  of  which  the  respondent  was  authorized  to  operate,  
manage   and   rehabilitate   Quickbite,   a   restaurant   owned   and   operated   by   WPM.   As   part   of   her   tasks,   the  
respondent  looked  for  a  contractor  who  would  renovate  the  two  existing  Quickbite  outlets.  Pursuant  to  the  
agreement,   the   respondent   engaged   the   services   of   CLN   Engineering   Services   (CLN)   to   renovate   Quickbite-­‐‑
Divisoria   at   the   cost   of   P432,876.02.   Quickbite-­‐‑Divisoria's   renovation   was   finally   completed,   and   its  
possession   was   delivered   to   the   respondent.   However,   out   of   the   P432,876.02   renovation   cost,   only   the  
amount  of  P320,000.00  was  paid  to  CLN,  leaving  a  balance  of  P112,876.02.  CLN  filed  a  complaint  for  sum  of  
money  and  damages  before  the  RTC  against  the  respondent  and  Manlapaz.  CLN  later  amended  the  complaint  
to  exclude  Manlapaz  as  defendant.  The  respondent  was  declared  in  default  for  her  failure  to  file  a  responsive  
pleading.    
 
The   RTC   found   the   respondent   liable   to   pay   CLN.   Thereafter,   the   respondent   instituted   a   complaint   for  
damages  against  the  petitioners,  WPM  and  Manlapaz.  The  respondent  alleged  that  she  was  adjudged  liable  
for  a  contract  that  she  entered  into  for  and  in  behalf  of  the  petitioners,  to  which  she  should  be  entitled  to  
reimbursement.  The  RTC  declared  WPM  in  default  for  its  failure  to  file  a  responsive  pleading.  In  its  decision,  
the   RTC   held   that   the   respondent   is   entitled   to   indemnity   from   Manlapaz.   The   RTC   found   that   based   on   the  
records,  there  is  a  clear  indication  that  WPM  is  a  mere  instrumentality  or  business  conduit  of  Manlapaz  and  
as   such,   WPM   and   Manlapaz   are   considered   one   and   the   same.   The   RTC   also   found   that   Manlapaz   had  
complete  control  over  WPM  considering  that  he  is  its  chairman,  president  and  treasurer  at  the  same  time.  
The  RTC  thus  concluded  that  Manlapaz  is  liable  in  his  personal  capacity  to  reimburse  the  respondent  the  
amount  she  paid  to  CLN  in  connection  with  the  renovation  agreement.  On  appeal,  the  CA  affirmed  the  ruling  
of  the  RTC.    
 
ISSUES:  

   
3H  A.Y.  2017-­‐2018   68  
 
 

CORPORATION  LAW  CASE  DIGESTS  –  ATTY.  DANTE  DELA  CRUZ  

Whether  or  not  the  application  of  the  principle  of  piercing  the  veil  of  corporate  fiction  is  warranted  in  the  
present  case.    
 
HELD:  
NO.   In   the   present   case,   the   attendant   circumstances   do   not   establish   that   WPM   is   a   mere   alter   ego   of  
Manlapaz.   The   mere   ownership   by   a   single   stockholder   of   even   all   or   nearly   all   of   the   capital   stocks   of   a  
corporation  is  not  by  itself  a  sufficient  ground  to  disregard  the  separate  corporate  personality.  That  Manlapaz  
concurrently  held  the  positions  of  president,  chairman  and  treasurer,  or  that  the  Manlapaz's  residence  is  the  
registered  principal  office  of  WPM,  are  insufficient  considerations  to  prove  that  he  had  exercised  absolute  
control  over  WPM.  In  this  connection,  w  e  stress  that  the  control  necessary  to  invoke  the  instrumentality  or  
alter  ego  rule  is  not  majority  or  even  complete  stock  control  but  such  domination  of  finances,  policies  and  
practices  that  the  controlled  corporation  has,  so  to  speak,  no  separate  mind,  will  or  existence  of  its  own,  and  
is   but   a   conduit   for   its   principal.   The   control   must   be   shown   to   have   been   exercised   at   the   time   the   acts  
complained  of  took  place.  Moreover,  the  control  and  breach  of  duty  must  proximately  cause  the  injury  or  
unjust  loss  for  which  the  complaint  is  made.  Here,  the  respondent  failed  to  prove  that  Manlapaz,  acting  as  
president,  had  absolute  control  over  WPM.  Even  granting  that  he  exercised  a  certain  degree  of  control  over  
the   finances,   policies   and   practices   of   WPM,   in   view   of   his   position   as   president,   chairman   and   treasurer   of  
the   corporation,   such   control   does   not   necessarily   warrant   piercing   the   veil   of   corporate   fiction   since   there  
was  not  a  single  proof  that  WPM  was  formed  to  defraud  CLN  or  the  respondent,  or  that  Manlapaz  was  guilty  
of   bad   faith   or   fraud.   That   WPM   later   reneged   on   its   monetary   obligation   to   CLN,   resulting   to   the   filing   of   a  
civil  case  for  sum  of  money  against  the  respondent,  does  not  automatically  indicate  fraud,  in  the  absence  of  
any  proof  to  support  it.  Since  no  harm  could  be  said  to  have  been  proximately  caused  by  Manlapaz  for  which  
the   latter   could   be   held   solidarily   liable   with   WPM,   and   considering   that   there   was   no   proof   that   WPM   had  
insufficient  funds,  there  was  no  sufficient  justification  for  the  RTC  and  the  CA  to  have  ruled  that  Manlapaz  
should  be  held  jointly  and  severally  liable  to  the  respondent  for  the  amount  she  paid  to  CLN.  Hence,  only  
WPM  is  liable  to  indemnify  the  respondent.    
 
Nevertheless,  in  the  present  case,  when  payment  for  the  balance  of  the  renovation  cost  was  demanded,  W  
PM,   instead   of   complying   with   its   obligation,   denied   having   authorized   the   respondent   to   contract   in   its  
behalf  and  accordingly  refused  to  pay.  Such  cold  refusal  to  pay  a  just  debt  amounts  to  a  breach  of  contract  in  
bad  faith,  as  contemplated  by  Article  2220.  Hence,  the  order  to  pay  moral  damages  was  in  order.    
 
 
 
 
50.  LANUZA  V.  BF  CORPORATION  
G.R.  NO.  174938                              OCTOBER  1,  2014  
LEONEN,  J.;  
 
FACTS:  
-­‐‑   A  building  was  constructed  by  respondent  corporation  in  favor  of  their  co  respondents  Shangri  La  
corporation.  
-­‐‑   BF  Corporation  alleged  that  despite  repeated  demands,  Shangri-­‐‑La  refused  to  pay  the  balance  owed  
to  it.  
-­‐‑   It   also   alleged   that   the   Shangri-­‐‑La’s   directors,   petitioners   herein,   were   in   bad   faith   in   directing  
Shangri-­‐‑La’s  affairs.    
-­‐‑   Therefore,  they  should  be  held  jointly  and  severally  liable  with  Shangri-­‐‑La  for  its  obligations  as  well  
as  for  the  damages  that  BF  Corporation  incurred  as  a  result  of  Shangri-­‐‑La’s  default.  

   
3H  A.Y.  2017-­‐2018   69  
 
 

CORPORATION  LAW  CASE  DIGESTS  –  ATTY.  DANTE  DELA  CRUZ  

-­‐‑   After  the  decision  of  the  Court  of  Appeals  that  the  trial  should  first  be  suspended  due  to  failure  to  
take  arbitration  proceedings,    the  petitioners  now  alleged  that  they  should  not  be  impleaded  as  party  
to  the  arbitration.  
-­‐‑   They   alleged   that   the   corporation   has   a   personality   separate   and   distinct   from   its  
directors/stockholders.  
 
ISSUE:   whether   petitioners   should   be   made   parties   to   the   arbitration   proceedings,   pursuant   to   the  
arbitration  clause  provided  in  the  contract  between  BF  Corporation  and  Shangri-­‐‑La.  
 
HELD:  
•   The  personalities  of  petitioners  and  the  corporation  may  later  be  found  to  be  indistinct  that  we  rule  
that  petitioners  may  be  compelled  to  submit  to  arbitration.  
•   When  the  courts  disregard  the  corporation’s  distinct  and  separate  personality  from  its  directors  or  
officers,  the  courts  do  not  say  that  the  corporation  is  the  same  as  its  directors,  stockholders,  officers,  
and  agents.  
•   Courts  merely  discount  the  distinction  and  treat  them  as  one,  in  relation  to  a  specific  act,  in  order  to  
extend   the   terms   of   the   contract   and   the   liabilities   for   all   damages   to   erring   corporate   officials   who  
participated  in  the  corporation’s  illegal  acts.    
•   This  is  done  so  that  the  legal  fiction  cannot  be  used  to  perpetrate  illegalities  and  injustices.  
•   Thus,   in   cases   alleging   solidary   liability   with   the   corporation   or   praying   for   the   piercing   of   the  
corporate  veil,  parties  who  are  normally  treated  as  distinct  individuals  should  be  made  to  participate  
in  the  arbitration  proceedings  in  order  to  determine  if  such  distinction  should  indeed  be  disregarded  
and,  if  so,  to  determine  the  extent  of  their  liabilities.  
 
 
 
 
51.  JOSE  EMMANUEL  P.  GUILLERMO,  PETITIONER,  V.  CRISANTO  P.  USON,  RESPONDENT.  
G.R.  NO.  198967,  MARCH  07,  2016  
PERALTA,  J.  
 
DOCTRINE:  
The   veil   of   corporate   fiction   can   be   pierced,   and   responsible   corporate   directors   and   officers   or   even   a  
separate  but  related  corporation,  may  be  impleaded  and  held  answerable  solidarily  in  a  labor  case,  even  after  
final  judgment  and  on  execution,  so  long  as  it  is  established  that  such  persons  have  deliberately  used  the  
corporate  vehicle  to  unjustly  evade  the  judgment  obligation,  or  have  resorted  to  fraud,  bad  faith  or  malice  in  
doing  so.  
 
FACTS:  
Respondent   Crisanto   P.   Uson   (Uson)   filed   a   Complaint   for   Illegal   Dismissal,   with   prayers   for  
backwages,  reinstatement,  salaries  and  13thmonth  pay,  moral  and  exemplary  damages  and  attorney's  fees  
against  Royal  Class  Venture.5  The  Labor  Arbiter  rendered  a  Decision8  in  favor  of  the  complainant  Uson.  Royal  
Class  Venture,  as  the  losing  party,  did  not  file  an  appeal  of  the  decision.9  Consequently,  upon  Uson's  motion,  
a  Writ  of  Execution10  was  issued  to  implement  the  Labor  Arbiter's  decision.  For  failure  to  implement,  an  Alias  
writ  of  execution  was  issued.  Thereafter,  a  second  Alias  writ  was  issued  which  remained  "unsatisfied."  Thus,  
Uson   filed   a   Motion   for   Alias   Writ   of   Execution   and   to   Hold   Directors   and   Officers   of   Respondent   Liable   for  
Satisfaction   of   the   Decision.13   The   motion   quoted   a   portion   of   the   Sheriffs   Return,   which   states   that   the  
establishment  erected  in  the  present  business  office  address  of  Royal  Class  Venture  is  not  [in]  the  latter's  
name   but   JOEL   and   SONS   CORPORATION,   a   family   corporation   owned   by   the   Guillermos   of   which,   Jose  
Emmanuel   F.   Guillermo   the   General   Manager   of   the   Royal   Class   Venture,   is   one   of   the   stockholders   who  
   
3H  A.Y.  2017-­‐2018   70  
 
 

CORPORATION  LAW  CASE  DIGESTS  –  ATTY.  DANTE  DELA  CRUZ  

received  the  writ  using  his  nickname  "Joey,"  and  who  concealed  his  real  identity  and  pretended  that  he  was  
the  brother  of  Jose,  which  was  contrary  to  the  statement  of  the  guard-­‐‑on-­‐‑duty  that  Jose  and  Joey  were  one  
and   the   same   person.   The   former   also   informed   the   undersigned   that   the   Royal   class   Venture(sic)  
corporation  has  been  dissolved.  The  Labor  Arbiter  issued  an  Order  granting  the  motion  filed  by  Uson.  The  
Labor   Arbiter   pierced   the   veil   of   corporate   fiction   of   Royal   Class   Venture   and   held   herein   petitioner   Jose  
Emmanuel   Guillermo   (Guillermo),   in   his   personal   capacity,   jointly   and   severally   liable   with   the   corporation  
for  the  enforcement  of  the  claims  of  Uson.17  Guillermo  elevated  the  matter  to  the  NLRC  which  dismissed  the  
appeal.  Guillermo  filed  a  Petition  for  Certiorari27  before  the  Court  of  Appeals  which  denied  the  same.  Hence,  
the  instant  petition.  Guillermo  assails  the  so-­‐‑called  "piercing  the  veil"  of  corporate  fiction  which  allegedly  
discriminated  against  him  when  he  alone  was  belatedly  impleaded  despite  the  existence  of  other  directors  
and  officers  in  Royal  Class  Venture.40      
 
ISSUE:  Whether  the  veil  of  corporate  fiction  was  properly  pierced  thereby  holding  Guillermo  liable  even  after  
the  Decision  already  attained  finality.  
 
HELD:  
YES.  In  labor  case,  Pantranco  Employees  Association)  v.  NLRC,  et  al.,55  the  doctrine  of  piercing  the  
corporate  veil  is  held  to  apply  only  in  three  (3)  basic  areas,  namely:  (  1)  defeat  of  public  convenience  as  when  
the  corporate  fiction  is  used  as  a  vehicle  for  the  evasion  of  an  existing  obligation;  (2)  fraud  cases  or  when  the  
corporate  entity  is  used  to  justify  a  wrong,  protect  fraud,  or  defend  a  crime;  or  (3)   alter  ego   cases,  where  a  
corporation   is   merely   a   farce   since   it   is   a   mere   alter   ego   or   business   conduit   of   a   person,   or   where   the  
corporation   is   so   organized   and   controlled   and   its   affairs   are   so   conducted   as   to   make   it   merely   an  
instrumentality,   agency,   conduit   or   adjunct   of   another   corporation.   The   veil   of   corporate   fiction   can   be  
pierced,  and  responsible  corporate  directors  and  officers  or  even  a  separate  but  related  corporation,  may  be  
impleaded  and  held  answerable  solidarily  in  a  labor  case,  even  after  final  judgment  and  on  execution,  so  long  
as   it   is   established   that   such   persons   have   deliberately   used   the   corporate   vehicle   to   unjustly   evade   the  
judgment  obligation,  or  have  resorted  to  fraud,  bad  faith  or  malice  in  doing  so.    
The   records   of   the   present   case   bear   allegations   and   evidence   that   Guillermo,   the   officer   being   held  
liable,   is   the   person   responsible   in   the   actual   running   of   the   company   and   for   the   malicious   and   illegal  
dismissal   of   the   complainant;   he,   likewise,   was   shown   to   have   a   role   in   dissolving   the   original   obligor  
company   in   an   obvious   "scheme   to   avoid   liability"   which   jurisprudence   has   always   looked   upon   with   a  
suspicious  eye  in  order  to  protect  the  rights  of  labor.  It  is  also  clearly  reflected  in  the  records  that  it  was  
Guillermo  himself,  as  President  and  General  Manager  of  the  company,  who  received  the  summons  to  the  case,  
and  who  also  subsequently  and  without  justifiable  cause  refused  to  receive  all  notices  and  orders  of  the  Labor  
Arbiter  that  followed.66This  makes  Guillermo  responsible  for  his  and  his  company's  failure  to  participate  in  
the  entire  proceedings  before  the  said  office.  Guillermo's  knowledge  of  the  case's  filing  and  existence  and  his  
unexplained  refusal  to  participate  in  it  as  the  responsible  official  of  his  company,  again  is  an  indicia  of  his  bad  
faith  and  malicious  intent  to  evade  the  judgment  of  the  labor  tribunals.  Finally,  the  records  likewise  bear  that  
Guillermo  dissolved  Royal  Class  Venture  and  helped  incorporate  a  new  firm,  located  in  the  same  address  as  
the  former,  wherein  he  is  again  a  stockholder.  WHEREFORE,  the  petition  is  DENIED.    
 
 
 
52.    VIUDA  DE  TAN  TOCO  VS.  THE  MUNICIPAL  COUNCIL  OF  ILOILO  
G.R.  NO.  L-­‐‑24950  -­‐‑  MARCH  25,  1926  
JUSTICE  VILLAMOR    
 
DOCTRINE:   It   is   evident   that   the   movable   and   immovable   property   of   a   municipality,   necessary   for  
governmental  purpose,  may  not  be  attached  and  sold  for  the  payment  of  a  judgment  against  the  municipality.  
The  supreme  reason  for  this  rule  is  the  character  of  the  public  use  to  which  such  kind  of  property  is  devoted.  

   
3H  A.Y.  2017-­‐2018   71  
 
 

CORPORATION  LAW  CASE  DIGESTS  –  ATTY.  DANTE  DELA  CRUZ  

The  necessity  for  government  service  justifies  that  the  property  of  public  of  the  municipality  be  exempt  from  
execution  just  as  it  is  necessary  to  exempt  certain  property  of  private  individuals  in  accordance  with  section  
452  of  the  Code  of  Civil  Procedure.  
 
FACTS:  The  widow  of  Tan  Toco  had  sued  the  municipal  council  of  Iloilo  for  the  amount  of  P42,966.40,  being  
the  purchase  price  of  two  strips  of  land,  one  on  Calle  J.  M.  Basa  consisting  of  592  square  meters,  and  the  other  
on   Calle   Aldiguer   consisting   of   59   square   meters,   which   the   municipality   of   Iloilo   had   appropriated   for  
widening  said  street.  The  Court  of  First  Instance  of  Iloilo  sentenced  the  said  municipality  to  pay  the  plaintiff  
the  amount  so  claimed,  plus  the  interest,  and  the  said  judgment  was  on  appeal  affirmed  by  this  court.  On  
account  of  lack  of  funds  the  municipality  of  Iloilo  was  unable  to  pay  the  said  judgment,  wherefore  plaintiff  
had   a   writ   of   execution   issue   against   the   property   of   the   said   municipality,   by   virtue   of   which   the   sheriff  
attached   two   auto   trucks   used   for   street   sprinkling,   one   police   patrol   automobile,   the   police   stations   on  
Mabini  street,  and  in  Molo  and  Mandurriao  and  the  concrete  structures,  with  the  corresponding  lots,  used  as  
markets  by  Iloilo,  Molo,  and  Mandurriao.  
 
After   notice   of   the   sale   of   said   property   had   been   made,   and   a   few   days   before   the   sale,   the   provincial   fiscal  
of  Iloilo  filed  a  motion  which  the  Court  of  First  Instance  praying  that  the  attachment  on  the  said  property  be  
dissolved,  that  the  said  attachment  be  declared  null  and  void  as  being  illegal  and  violative  of  the  rights  of  the  
defendant  municipality.  The  municipal  law,  section  2165  of  the  Administrative  Code,  provides  that:  
 
Municipalities   are   political   bodies   corporate,   and   as   such   are   endowed   with   the   faculties   of   municipal  
corporations,  to  be  exercised  by  and  through  their  respective  municipal  government  in  conformity  with  law.  
It   shall   be   competent   for   them,   in   their   proper   corporate   name,   to   sue   and   be   sued,   to   contract   and   be  
contracted  with,  to  acquire  and  hold  real  and  personal  property  for  municipal  purposes,  and  generally  to  
exercise  the  powers  hereinafter  specified  or  otherwise  conferred  upon  them  by  law.  
 
 
ISSUE:  whether  or  not  the  property  levied  upon  is  exempt  from  execution  
 
 
HELD:  For  the  purposes  of  the  matter  here  in  question,  the  Administrative  Code  does  not  specify  the  kind  of  
property   that   a   municipality   may   acquire.   However,   article   343   of   the   Civil   Code   divides   the   property   of  
provinces  and  towns  (municipalities)  into  property  for  public  use  and  patrimonial  property.  According  to  
article  344  of  the  same  Code,  provincial  roads  and  foot-­‐‑path,  squares,  streets,  fountains  and  public  waters,  
drives  and  public  improvements  of  general  benefit  built  at  the  expense  of  the  said  towns  or  provinces,  are  
property  for  public  use.  
 
The   American   Law   is   more   explicit   about   this   matter   as   expounded   by   Mcquilin   in   Municipal   Corporations,  
volume  3,  paragraph  1160,  where  he  says  that:  
States  statutes  often  provide  the  court  houses,  jails  and  other  buildings  owned  by  municipalities  and  the  lots  
on   which   they   stand   shall   be   exempt   from   attachment   and   execution.   But   independent   of   express   statutory  
exemption,  as  a  general  proposition,  property,  real  and  personal,  held  by  municipal  corporations,  in  trust  for  
the  benefit  of  their  inhabitants,  and  used  for  public  purposes,  is  exempt.  
For  example,  public  buildings,  school  houses,  streets,  squares,  parks,  wharves,  engines  and  engine  houses,  
and   the   like,   are   not   subject   to   execution.   So   city   waterworks,   and   a   stock   of   liquors   carried   in   a   town  
dispensary,   are   exempt.   The   reason   for   the   exemption   is   obvious.   Municipal   corporations   are   created   for  
public  purposes  and  for  the  good  of  the  citizens  in  their  aggregate  or  public  capacity.  That  they  may  properly  
discharge   such   public   functions   corporate   property   and   revenues   are   essential,   and   to   deny   them   these  
means  the  very  purpose  of  their  creation  would  be  materially  impeded,  and  in  some  instances  practically  
destroy   it.   Respecting   this   subject   the   Supreme   Court   of   Louisiana   remarked:   "On   the   first   view   of   this  

   
3H  A.Y.  2017-­‐2018   72  
 
 

CORPORATION  LAW  CASE  DIGESTS  –  ATTY.  DANTE  DELA  CRUZ  

question   there   is   something   very   repugnant   to   the   moral   sense   in   the   idea   that   a   municipal   corporation  
should  contract  debts,  and  that,  having  no  resources  but  the  taxes  which  are  due  to  it,  these  should  not  be  
subjected  by  legal  process  to  the  satisfaction  of  its  creditors.  This  consideration,  deduced  from  the  principles  
of  moral  equity  has  only  given  way  to  the  more  enlarged  contemplation  of  the  great  and  paramount  interests  
of  public  order  and  the  principles  of  government."  
It  is  generally  held  that  property  owned  by  a  municipality,  where  not  used  for  a  public  purpose  but  for  quasi  
private  purposes,  is  subject  to  execution  on  a  judgment  against  the  municipality,  and  may  be  sold.  This  rule  
applies   to   shares   of   stock   owned   by   a   municipal   corporation,   and   the   like.   But   the   mere   fact   that   corporate  
property   held   for   public   uses   is   being   temporarily   used   for   private   purposes   does   not   make   it   subject  
execution.  
If  municipal  property  exempt  from  execution  is  destroyed,  the  insurance  money  stands  in  lieu  thereof  and  is  
also  exempt.  The  members  or  inhabitants  of  a  municipal  corporation  proper  are  not  personally  liable  for  the  
debts  of  the  municipality,  except  that  in  the  New  England  States  the  individual  liability  of  the  inhabitant  is  
generally  maintained.  
It   is   evident   that   the   movable   and   immovable   property   of   a   municipality,   necessary   for   governmental  
purpose,  may  not  be  attached  and  sold  for  the  payment  of  a  judgment  against  the  municipality.  The  supreme  
reason  for  this  rule  is  the  character  of  the  public  use  to  which  such  kind  of  property  is  devoted.  The  necessity  
for  government  service  justifies  that  the  property  of  public  of  the  municipality  be  exempt  from  execution  just  
as  it  is  necessary  to  exempt  certain  property  of  private  individuals  in  accordance  with  section  452  of  the  
Code  of  Civil  Procedure.  
 
 
 
53.  ADELIO  C.  CRUZ  VS.  QUITERIO  L.  DALISAY  
ADM.  MATTER  NO.  R-­‐‑181-­‐‑P  JULY  31,  1987  
FERNAN,  J.:  
 
 
DOCTRINE:     Respondent   (Deputy   Sheriff)   chose   to   "pierce   the   veil   of   corporate   entity"   usurping   a   power  
belonging   to   the   court   and   assumed   improvidently   that   since   the   complainant   is   the   owner/president   of  
Qualitrans  Limousine  Service,  Inc.,  they  are  one  and  the  same.  
 
It  is  a  well-­‐‑settled  doctrine  both  in  law  and  in  equity  that  as  a  legal  entity,  a  corporation  has  a  personality  
distinct   and   separate   from   its   individual   stockholders   or   members.   The   mere   fact   that   one   is   president   of   a  
corporation  does  not  render  the  property  he  owns  or  possesses  the  property  of  the  corporation,  since  the  
president,  as  individual,  and  the  corporation  are  separate  entities.  
 
FACTS:   Adelio   C.   Cruz   charged   Quiterio   L.   Dalisay,   Senior   Deputy   Sheriff   of   Manila,   with   "malfeasance   in  
office,  corrupt  practices  and  serious  irregularities".    Respondent  sheriff  attached  and/or  levied  the  money  
belonging  to  complainant  Cruz  when  he  was  not  himself  the  judgment  debtor  in  the  final  judgment  of  NLRC  
NCR  Case  sought  to  be  enforced  but  rather  the  company  known  as  "Qualitrans  Limousine  Service,  Inc.,"  a  
duly  registered  corporation;    
 
DALISAY   CONTENDS:   that   when   he   garnished   complainant's   cash   deposit   at   the   Philtrust   bank,   he   was  
merely  performing  a  ministerial  duty.  While  it  is  true  that  said  writ  was  addressed  to  Qualitrans  Limousine  
Service,   Inc.,   yet   it   is   also   a   fact   that   complainant   had   executed   an   affidavit   stating   that   he   is   the  
owner/president  of  said  corporation  and,  because  of  that  declaration,  the  counsel  for  the  plaintiff  in  the  labor  
case  advised  him  to  serve  notice  of  garnishment  on  the  Philtrust  bank.  
 

   
3H  A.Y.  2017-­‐2018   73  
 
 

CORPORATION  LAW  CASE  DIGESTS  –  ATTY.  DANTE  DELA  CRUZ  

Thereafter,   complainant   executed   an   affidavit   of   desistance   stating   that   he   is   no   longer   interested   in  


prosecuting  the  case  against  respondent  Dalisay  and  that  it  was  just  a  "misunderstanding"  between  them.    
 
ISSUE:    Whether  or  not  respondent  Deputy  Sheriff  may  pierce  the  veil  of  corporate  entity  in  enforcing  a  writ  
of  execution.    
 
HELD:  No.  We  hold  that  respondent's  actuation  in  enforcing  a  judgment  against  complainant  who  is  not  the  
judgment  debtor  in  the  case  calls  for  disciplinary  action.  Considering  the  ministerial  nature  of  his  duty  in  
enforcing  writs  of  execution,  what  is  incumbent  upon  him  is  to  ensure  that  only  that  portion  of  a  decision  
ordained  or  decreed  in  the  dispositive  part  should  be  the  subject  of  execution.  No  more,  no  less.    
 
It  has  been  held  that  the  desistance  of  complainant  does  not  preclude  the  taking  of  disciplinary  action  against  
respondent.    
 
That   the   title   of   the   case   specifically   names   complainant   as   one   of   the   respondents   is   of   no   moment   as  
execution  must  conform  to  that  directed  in  the  dispositive  portion  and  not  in  the  title  of  the  case.  
The  tenor  of  the  NLRC  judgment  and  the  implementing  writ  is  clear  enough.  It  directed  Qualitrans  Limousine  
Service,  Inc.  to  reinstate  the  discharged  employees  and  pay  them  full  backwages.    
 
Respondent,  however,  chose  to  "pierce  the  veil  of  corporate  entity"  usurping  a  power  belonging  to  the  court  
and   assumed   improvidently   that   since   the   complainant   is   the   owner/president   of   Qualitrans   Limousine  
Service,  Inc.,  they  are  one  and  the  same.    
It  is  a  well-­‐‑settled  doctrine  both  in  law  and  in  equity  that  as  a  legal  entity,  a  corporation  has  a  personality  
distinct   and   separate   from   its   individual   stockholders   or   members.   The   mere   fact   that   one   is   president   of   a  
corporation  does  not  render  the  property  he  owns  or  possesses  the  property  of  the  corporation,  since  the  
president,  as  individual,  and  the  corporation  are  separate  entities.  
ACCORDINGLY,  we  find  Respondent  Deputy  Sheriff  Quiterio  L.  Dalisay  NEGLIGENT  in  the  enforcement  of  the  
writ  of  execution  in  NLRC  Case.  
 
 
 
54.  NASECO  GUARDS  ASSOCIATION-­‐‑PEMA  V.  NATIONAL  SERVICE  CORPORATION  
GR  NO.  165442  25  AUGUST  2010  
JUSTICE  VILLARAMA,  JR.  
 
DOCTRINE:  
  Control,   by   itself,   does   not   mean   that   the   controlled   corporation   is   a   mere   instrumentality   or   a  
business  conduit  of  the  mother  company.  There  must  be  a  perpetuation  of  fraud  behind  the  control  or  at  least  
a  fraudulent  or  illegal  purpose  behind  the  control  in  order  to  justify  piercing  the  veil  of  corporate  fiction.    
 
FACTS:  
  Respondent  NASECO  is  a  wholly-­‐‑owned  subsidiary  of  Philippine  National  Bank,  organized  under  the  
Corporation  Code.  It  supplies  security  services  to  different  clients  which,  among  others,  includes  PNB.  On  the  
other   hand,   Petitioner   NEMU-­‐‑PEMA   is   the   collective   bargaining   representative   of   rank-­‐‑and-­‐‑file   security  
guards   of   NASECO   while   NEMU-­‐‑PEMA   is   the   collective   bargaining   representative   of   regular   rank-­‐‑and-­‐‑file  
non-­‐‑security  employees.  
 
  On   08   June   1995,   NAGA-­‐‑PEMA   and   NASECO   signed   a   Collective   Bargaining   Agreement   on   non-­‐‑
economic   terms.   In   1996,   NAGA-­‐‑PEMA   filed   a   notice   of   strike   due   to   NASECO’s   refusal   to   bargain   for  
economic   benefits   in   the   CBA.   Meanwhile,   NASECO   and   NEMU-­‐‑PEMA   also   entered   into   a   CBA   on   non-­‐‑

   
3H  A.Y.  2017-­‐2018   74  
 
 

CORPORATION  LAW  CASE  DIGESTS  –  ATTY.  DANTE  DELA  CRUZ  

economic  terms.  Unfortunately,  dispute  as  to  leadership  of  the  latter  arose  which  led  to  the  suspension  of  the  
negotiations  for  the  CBA.    
   
  After  a  year,  NEMU-­‐‑PEMA  filed  a  notice  of  strike  against  NASECO  and  PNB  on  the  ground  of  unfair  
labor   practice   due   to   a   bargaining   deadlock.   Due   to   failure   to   conciliate   the   parties,   the   DOLE   Secretary  
assumed  jurisdiction  and  issued  a  Resolution  directing  the  parties  to  execute  a  new  CBA  incorporating  the  
dispositions  regarding  benefits  of  the  employees.      
 
ISSUE:  
  Whether  or  not  PNB  should  be  liable  to  pay  the  CBA  benefits  to  NAGA-­‐‑PEMA,  being  the  owner  and  
exercising  control  over  NASECO  
 
RULING:  
  To  hold  PNB  liable  for  the  CBA  Benefits,  being  the  mother  company,  is  asking  the  Court  to  pierce  the  
veil  of  corporate  fiction  of  NASECO.    
 
Corporation  law  dictates  that  a  corporation  is  an  entity  separate  and  distinct  from  its  stockholders  
and  from  other  corporations  to  which  it  may  be  connected.  However,  when  the  separate  juridical  personality  
is  used  to  defeat  public  convenience,  justify  wrong,  protect  fraud  or  defend  crime,  or  is  used  as  a  device  to  
defeat  the  labor  laws,  this  separate  juridical  personality  of  the  corporation  may  be  disregarded  or  the  veil  of  
corporate  fiction  pierced.    
 
In   the   case   at   bar,   there   is   no   reason   to   pierce   the   corporate   veil   of   NASECO   and   go   beyond   its   legal  
personality.   Control,   by   itself,   does   not   mean   that   the   controlled   corporation   is   a   mere   instrumentality   or   a  
business  conduit  of  the  mother  company.  There  must  be  a  perpetuation  of  fraud  behind  the  control  or  at  least  
a  fraudulent  or  illegal  purpose  behind  the  control  in  order  to  justify  piercing  the  veil  of  corporate  fiction.  
There  is  no  fraudulent  intent  in  this  case.    
 
 
 
55.  PACIFIC  REHOUSE  VS.  CA  AND  EIB-­‐‑SECURITIES  
G.R.  NO.  199687.  MARCH  24,  2014  
REYES,  J.  
 
DOCTRINE:   Existence   of   interlocking   directors,   corporate   officers   and   shareholders   is   not   enough  
justification   to   pierce   the   veil   of   corporate   fiction   in   the   absence   of   fraud   or   other   public   policy  
considerations.  
 
FACTS:  A  complaint  instituted  with  RTC  Makati  against  EIB  Securities  Inc.  (E-­‐‑Securities)  for  unauthorized  
sale   of   32,180,000   DMCI   shares   of   Pacific   Rehouse   Corporation,   Pacific   Concorde   Corporation,   Mizpah  
Holdings,  Inc.,  Forum  Holdings  Corporation,  and  East  Asia  Oil  Company,  Inc.  RTC  ordered  [E-­‐‑Securities]  to  
return   the   petitioners’     32,180,000   DMCI   shares.   When   the   Writ   of   Execution   was   returned   unsatisfied,  
petitioners  moved  for  the  issuance  of  an  alias  writ  of  execution  to  hold  Export  and  Industry  Bank,  Inc.  (E-­‐‑
Bank)   liable   for   the   judgment   obligation   as   EIB   Securities   is   “a   wholly-­‐‑owned   controlled   and   dominated  
subsidiary  of  E-­‐‑Bank    and  is  thus,  a  mere  alter  ego  and  business  conduit  of  the  latter.  E-­‐‑Securities  opposed  
the  motion,  arguing  that  it  has  a  corporate  personality  that  is  separate  and  distinct  from  E-­‐‑Bank.  
 
RTC   decided   in   favour   of   petitioners,   ratiocinated   that   being   one   and   the   same   entity   in   the   eyes   of   the   law,  
the   service   of   summons   upon   EIB   Securities,   Inc.   (E-­‐‑Securities)   has   bestowed   jurisdiction   over   both   the  
parent  and  wholly-­‐‑owned  subsidiary.  E-­‐‑  Bank  filed  before  the  CA  a  petition  for  certiorari  with  prayer  for  the  

   
3H  A.Y.  2017-­‐2018   75  
 
 

CORPORATION  LAW  CASE  DIGESTS  –  ATTY.  DANTE  DELA  CRUZ  

issuance  of  a  TRO  seeking  the  nullification  of  the  RTC  Order.  In  its  petition,  E-­‐‑Bank  made  reference  to  several  
rulings  of  the  Court  upholding  the  separate  and  distinct  personality  of  a  corporation.    
 
CA  issued  a  60-­‐‑day  TRO  enjoining  the  execution  of  the  Orders  of  the  RTC.  CA  rendered  the  assailed  Decision  
on  the  merits  of  the  case,  granting  Export  Bank’s  petition  and  nullifying  the  RTC  Order.  
 
ISSUE:  
1.   Whether  the  RTC  was  correct  in  applying  the  Doctrine  of  Piercing  the  Veil  of  Corporate  Fiction.  
2.   (sub-­‐‑issue)  Assuming  that  E-­‐‑Securities  is  a  mere  alter  ego  of  E-­‐‑bank,  may  the  Writ  of  Execution  be  
enforced  against  the  alleged  parent  corporation?  
HELD:  
 
1.   No.  Furthermore,  ownership  by  Export  Bank  of  a  great  majority  or  all  of  stocks  of  E-­‐‑Securities  and  
the   existence   of   interlocking   directorates   may   serve   as   badges   of   control,   but   ownership   of   another  
corporation,  per  se,  without  proof  of  actuality  of  the  other  conditions  are  insufficient  to  establish  an  
alter  ego  relationship  or  connection  between  the  two  corporations,  which  will  justify  the  setting  aside  
of  the  cover  of  corporate  fiction.  The  Court  has  declared  that  “mere  ownership  by  a  single  stockholder  
or   by   another   corporation   of   all   or   nearly   all   of   the   capital   stock   of   a   corporation   is   not   of   itself  
sufficient  ground  for  disregarding  the  separate  corporate  personality.”  The  Court  has  likewise  ruled  
that   the   “existence   of   interlocking   directors,   corporate   officers   and   shareholders   is   not   enough  
justification   to   pierce   the   veil   of   corporate   fiction   in   the   absence   of   fraud   or   other   public   policy  
considerations.”  
 
While  the  courts  have  been  granted  the  colossal  authority  to  wield  the  sword  which  pierces  through  
the  veil  of  corporate  fiction,  concomitant  to  the  exercise  of  this  power,  is  the  responsibility  to  uphold  
the  doctrine  of  separate  entity,  when  rightly  so;  as  it  has  for  so  long  encouraged  businessmen  to  enter  
into  economic  endeavors  fraught  with  risks  and  where  only  a  few  dared  to  venture.  
 
2.   No.  As  E-­‐‑Bank  was  neither  served  with  summons,  nor  has  it  voluntarily  appeared  before  the  court,  
the  judgment  sought  to  be  enforced  against  E-­‐‑Securities  cannot  be  made  against  its  parent  company,  
E-­‐‑Bank.  E-­‐‑Bank  has  consistently  disputed  the  RTC  jurisdiction  and  pleaded  that  RTC  Makati  never  
acquired  jurisdiction  over  E-­‐‑Bank.  E-­‐‑Bank  was  not  pleaded  as  a  party  in  this  case.  It  was  never  served  
with   summons   by   nor   did   it   voluntarily   appear   before   RTC   of   Makati   so   as   to   be   subjected   to   the  
latter’s  jurisdiction.”  
 
 
 
56.  KUKAN  INTERNATIONAL  CORPORATION  VS.  HON.  AMOR  REYES  
G.R.  NO.  182729;  SEPTEMBER  29,  2010  
VELASCO,  JR.,  J.  
 
Doctrine:  The  principle  of  piercing  the  veil  of  corporate  fiction,  and  the  resulting  treatment  of  two  related  
corporations  as  one  and  the  same  juridical  person  with  respect  to  a  given  transaction,  is  basically  applied  
only   to   determine   established   liability;   it   is   not   available   to   confer   on   the   court   a   jurisdiction   it   has   not  
acquired,  in  the  first  place,  over  a  party  not  impleaded  in  a  case.  
 
FACTS:  
Kukan,  Inc.  conducted  a  bidding  for  the  supply  and  installation  of  signages  in  a  building  being  constructed  in  
Makati  City.  Morales  tendered  the  winning  bid.  Despite  his  compliance  with  his  contractual  undertakings,  

   
3H  A.Y.  2017-­‐2018   76  
 
 

CORPORATION  LAW  CASE  DIGESTS  –  ATTY.  DANTE  DELA  CRUZ  

Morales   was   not   paid   in   full,   leaving   a   balance   in   the   contract   price.   Kukan,   Inc.   refused   to   pay   despite  
demands.  Morales  filed  a  Complaint  with  the  RTC  against  Kukan,  Inc.  
Trial  ensued  but  thereafter  Kukan  Inc.  no  longer  appeared  and  participated  in  the  proceedings  before  the  
trial  court,  prompting  the  RTC  to  declare  Kukan,  Inc.  in  default  and  paving  the  way  for  Morales  to  present  his  
evidence  ex  parte.  The  sheriff  then   levied  upon  various  personal  properties  found  at  what  was  supposed  to  
be  Kukan,  Inc.’s  office.  Alleging  that  it  owned  the  properties  thus  levied  and  that  it  was  a  different  corporation  
from  Kukan,  Inc.,  Kukan  International  Corporation  (KIC)  filed  an  Affidavit  of  Third-­‐‑Party  Claim.  
In   reaction   to   the   third   party   claim,   Morales   prayed,   applying   the   principle   of   piercing   the   veil   of   corporate  
fiction,  that  an  order  be  issued  for  the  satisfaction  of  the  judgment  debt  of  Kukan,  Inc.  with  the  properties  
under  the  name  or  in  the  possession  of  KIC,  it  being  alleged  that  both  corporations  are  but  one  and  the  same  
entity.  
The  case  was  re-­‐‑raffled  before  respondent  Judge  Reyes,  Manila  RTC,  Branch  21,  who  by  Order  granted  the  
motion.  The  CA  affirmed  the  RTC.  
 
ISSUE:  
Whether  or  not  the  trial  and  appellate  courts  correctly  applied  the  principle  of  piercing  the  veil  of  corporate  
fiction  in  the  case  at  bar.  
 
RULING:  
No.  The  trial  and  appellate  courts  erred  in  applying  the  principle  of  piercing  the  veil  of  corporate  fiction.  The  
principle  of  piercing  the  veil  of  corporate  fiction,  and  the  resulting  treatment  of  two  related  corporations  as  
one  and  the  same  juridical  person  with  respect  to  a  given  transaction,  is  basically  applied  only  to  determine  
established  liability;  it  is  not  available  to  confer  on  the  court  a  jurisdiction  it  has  not  acquired,  in  the  first  
place,  over  a  party  not  impleaded  in  a  case.  Elsewise  put,  a  corporation  not  impleaded  in  a  suit  cannot  be  
subject  to  the  court’s  process  of  piercing  the  veil  of  its  corporate  fiction.  
In   those   instances   when   the   Court   pierced   the   veil   of   corporate   fiction   of   two   corporations,   there   was   a  
confluence  of  the  following  factors:  (1)  A  first  corporation  is  dissolved;  (2)  The  assets  of  the  first  corporation  
is   transferred   to   a   second   corporation   to   avoid   a   financial   liability   of   the   first   corporation;   and;   (3)   Both  
corporations   are   owned   and   controlled   by   the   same   persons   such   that   the   second   corporation   should   be  
considered  as  a  continuation  and  successor  of  the  first  corporation.  
In   the   instant   case,   however,   the   second   and   third   factors   are   conspicuously   absent.     There   is,   therefore,   no  
compelling  justification  for  disregarding  the  fiction  of  corporate  entity  separating  Kukan,  Inc.  from  KIC.    In  
applying   the   principle,   both   the   RTC   and   the   CA   miserably   failed   to   identify   the   presence   of   the  
abovementioned  factors.  
 
 
 
57.  PHILIPPINE  NATIONAL  BANK  V.  HYDRO  RESOURCES  
G.R.  NO.  167530,  MARCH  13,  2013  
LEONARDO-­‐‑DE  CASTRO,  J.  
 
DOCTRINE:   Piercing   the   corporate   veil   based   on   the   alter   ego   theory   requires   the   concurrence   of   three  
elements:   control   of   the   corporation   by   the   stockholder   or   parent   corporation,   fraud   or   fundamental  
unfairness   imposed   on   the   plaintiff,   and   harm   or   damage   caused   to   the   plaintiff   by   the   fraudulent   or   unfair  
act  of  the  corporation.  The  absence  of  any  of  these  elements  prevents  piercing  the  corporate  veil.  
 
FACTS:  Petitioners  DBP  and  PNB  foreclosed  on  certain  mortgages  made  on  the  properties  of  Marinduque  
Mining  and  Industrial  Corp  (MMIC).  As  a  result  of  the  foreclosure,  DBP  and  PNB  acquired  substantially  all  the  
assets  of  MMIC  and  resumed  the  business  operations  of  the  defunct  MMIC  by  organizing  Nonoc  Mining  and  
Industrial   Corp   (NMIC).   NMIC   engaged   the   services   of   Hercon,   Inc.,   for   NMIC’s   Mine   Stripping   and   Road  

   
3H  A.Y.  2017-­‐2018   77  
 
 

CORPORATION  LAW  CASE  DIGESTS  –  ATTY.  DANTE  DELA  CRUZ  

Construction   Program   and   after   computing   the   payments   already   made   by   NMIC   and   crediting   NMIC’s  
receivables  from  Hercon,  the  latter  found  that  NMIC  still  has  an  unpaid  balance  of  P8.37M.  Hercon’s  demands  
for   payment   went   unheeded,   which   prompted   it   to   initiate   a   complaint   for   sum   of   money   before   the   RTC   of  
Makati   seeking   to   hold   petitioners   NMIC,   DBP   and   PNB   solidarily   liable   for   the   amount   owing   Hercon.  
Subsequent  to  the  filing  of  the  complaint,  Hercon  was  acquired  by  HRCC  in  a  merger,  which  prompted  the  
amendment  of  the  complaint  to  substitute  HRCC  for  Hercon.    
 
HRCC   claims   that   NMIC   was   the   alter   ego   of   DBP   and   PNB   which   owned,   conducted   and   controlled   the  
business   of   NMIC   as   shown   by   the   following   circumstances:   NMIC   was   owned   by   DBP   and   PNB,   the   officers  
of  DBP  and  PNB  were  also  the  officers  of  NMIC,  and  DBP  and  PNB  financed  the  operations  of  NMIC.  
 
On  the  other  hand,  the  three  petitioners,  namely  DBP,  PNB  &  ATP,  assert  that  NMIC  is  a  corporate  entity  with  
a  juridical  personality  separate  and  distinct  from  both  PNB  and  DBP.  They  insist  that  majority  ownership  by  
DBP  and  PNB  of  NMIC  is  not  a  sufficient  ground  for  disregarding  the  separate  corporate  personality  of  NMIC  
because  NMIC  was  not  a  mere  adjunct,  business  conduit  or  alter  ego  of  DBP  and  PNB.  They  further  argue  that,  
assuming  they  may  be  held  solidarily  liable  with  NMIC  to  pay  NMIC’s  exclusive  and  separate  indebtedness  to  
HRCC,  such  liability  of  the  two  banks  was  transferred  to  and  assumed  by  the  National  Government  through  
the   APT   (now   PMO),   under   the   respective   deeds   of   transfer   executed   by   DBP   and   PNB   pursuant   to  
Proclamation  No.  50  and  AO  14.  
 
The   trial   and   appellate   courts,   who   both   relied   on   the   alter   ego   theory   when   they   disregarded   the   separate  
corporate  personality  of  NMIC,  ruled  in  favor  of  HRCC.  
 
ISSUE:  WON  NMIC  is  a  corporate  entity  with  a  juridical  personality  separate  and  distinct  from  both  PNB  and  
DBP.  
 
HELD:   Yes.   Piercing   the   corporate   veil   based   on   the   alter   ego   theory   requires   the   concurrence   of   three  
elements:   control   of   the   corporation   by   the   stockholder   or   parent   corporation,   fraud   or   fundamental  
unfairness   imposed   on   the   plaintiff,   and   harm   or   damage   caused   to   the   plaintiff   by   the   fraudulent   or   unfair  
act  of  the  corporation.  The  absence  of  any  of  these  elements  prevents  piercing  the  corporate  veil.  
 
This  Court  finds  that  none  of  the  tests  has  been  satisfactorily  met  in  this  case.    
 
Both  the  RTC  and  the  CA  applied  the  alter  ego  theory  and  penetrated  the  corporate  cover  of  NMIC  based  on  
two  factors:  (1)  ownership  by  DBP  and  PNB  of  effectively  all  stocks  of  NMIC,  and  (2)  the  alleged  interlocking  
directorates  of  DBP,  PNB  and  NMIC.  
 
This  Court  has  declared  that  “mere  ownership  by  a  single  stockholder  or  by  another  corporation  of  all  or  
nearly   all   of   the   capital   stock   of   a   corporation   is   not   of  itself   sufficient   ground   for   disregarding  the  separate  
corporate   personality.   This   Court   has   likewise   ruled   that   the   “existence   of   interlocking   directors,   corporate  
officers   and   shareholders   is   not   enough   justification   to   pierce   the   veil   of   corporate   fiction   in   the   absence   of  
fraud  or  other  public  policy  considerations.  
 
Also,  even  assuming  that  DBP  and  PNB  exercised  control  over  NMIC,  there  is  no  evidence  that  the  juridical  
personality  of  NMIC  was  used  by  DBP  and  PNB  to  commit  a  fraud  or  to  do  a  wrong  against  HRCC.    
 
As   regards   the   third   element,   in   the   absence   of   both   control   by   DBP   and   PNB   of   NMIC   and   fraud   or  
fundamental   unfairness   perpetuated   by   DBP   and   PNB   through   the   corporate   cover   of   NMIC,   no   harm   could  
be  said  to  have  been  proximately  caused  by  DBP  and  PNB  on  HRCC  for  which  HRCC  could  hold  DBP  and  PNB  
solidarily  liable  with  NMIC.  

   
3H  A.Y.  2017-­‐2018   78  
 
 

CORPORATION  LAW  CASE  DIGESTS  –  ATTY.  DANTE  DELA  CRUZ  

 
Considering  that,  under  the  deeds  of  transfer  executed  by  DBP  and  PNB,  the  liability  of  the  APT  as  transferee  
of  the  rights,  titles  and  interests  of  DBP  and  PNB  in  NMIC  will  attach  only  if  DBP  and  PNB  are  held  liable,  the  
APT  incurs  no  liability  for  the  judgment  indebtedness  of  NMIC.  
 
Only   NMIC   as   a   distinct   and   separate   legal   entity   is   liable   to   pay   its   corporate   obligation   to   HRCC   in   the  
amount  of  P8.37M  with  legal  interest  thereon  from  the  date  of  demand.  
 
   
 
58.  MACASAET  VS  CO  
G.R.  NO.  156759,  JUNE  5,  2013  
BERSAMIN,  J.  
                             
DOCTRINE:  To  warrant  the  substituted  service  of  the  summons  and  copy  of  the  complaint,  the  serving  officer  
must  first  attempt  to  effect  the  same  upon  the  defendant  in  person.  Only  after  the  attempt  at  personal  service  
has  become  futile  or  impossible  within  a  reasonable  time  may  the  officer  resort  to  substituted  service.  
 
 
FACTS:    
Respondent,   a   retired   police   officer,   sued   Abante   Tonite,   a   daily   tabloid   of   general   circulation;   its   Publisher  
Allen  A.  Macasaet;  its  Managing  Director  Nicolas  V.  Quijano;  its  Circulation  Manager  Isaias  Albano;  its  Editors  
Janet  Bay,  Jesus  R.  Galang  and  Randy  Hagos;  and  its  Columnist/Reporter  Lily  Reyes  (petitioners),  claiming  
damages  because  of  an  allegedly  libelous  article  petitioners  published  in  the  June  6,  2000  issue  of  Abante  
Tonite.   The   suit   was   raffled   to   Branch   51   of   the   RTC,   which   in   due   course   issued   summons   to   be   served   on  
each  defendant,  including  Abante  Tonite,  at  their  business  address  at  Intramuros,  Manila.  RTC  Sheriff  Raul  
Medina   proceeded   to   the   stated   business   address   to   effect   the   personal   service   of   the   summons   on   the  
defendants.   But   his   efforts   to   personally   serve   each   defendant   in   the   address   were   futile   because   the  
defendants  were  then  out  of  the  office  and  unavailable.  He  returned  in  the  afternoon  of  that  day  to  make  a  
second  attempt  at  serving  the  summons,  but  he  was  informed  that  petitioners  were  still  out  of  the  office.  He  
decided  to  resort  to  substituted  service  of  the  summons,  and  explained  why  in  his  sheriff’s  return.  Petitioners  
moved  for  the  dismissal  of  the  complaint  alleging  lack  of  jurisdiction  over  their  persons  because  of  the  invalid  
and  ineffectual  substituted  service  of  summons.  They  contended  that  the  sheriff  had  made  no  prior  attempt  
to  serve  the  summons  personally  on  each  of  them  in  accordance  with  Section  6  and  Section  7,  Rule  14  of  the  
Rules  of  Court.  They  further  moved  to  drop  Abante  Tonite  as  a  defendant  by  virtue  of  its  being  neither  a  
natural  nor  a  juridical  person  that  could  be  impleaded  as  a  party  in  a  civil  action.  At  the  hearing  of  petitioners’  
motion  to  dismiss,  Medina  testified  that  he  had  gone  to  the  office  address  of  petitioners  in  the  morning  of  
September  18,  2000  to  personally  serve  the  summons  on  each  defendant;  that  petitioners  were  out  of  the  
office  at  the  time;  that  he  had  returned  in  the  afternoon  of  the  same  day  to  again  attempt  to  serve  on  each  
defendant  personally  but  his  attempt  had  still  proved  futile  because  all  of  petitioners  were  still  out  of  the  
office;   that   some   competent   persons   working   in   petitioners’   office   had   informed   him   that   Macasaet   and  
Quijano  were  always  out  and  unavailable,  and  that  Albano,  Bay,  Galang,  Hagos  and  Reyes  were  always  out  
roving  to  gather  news;  and  that  he  had  then  resorted  to  substituted  service  upon  realizing  the  impossibility  
of  his  finding  petitioners  in  person  within  a  reasonable  time.  RTC  denied  the  motion  to  dismiss.  Petitioners  
filed  a  motion  for  reconsideration,  asserting  that  the  sheriff  had  immediately  resorted  to  substituted  service  
of  the  summons  upon  being  informed  that  they  were  not  around  to  personally  receive  the  summons,  and  that  
Abante  Tonite,  being  neither  a  natural  nor  a  juridical  person,  could  not  be  made  a  party  in  the  action.  RTC  
denied  petitioners’  motion  for  reconsideration.  Regarding  the  impleading  of  Abante  Tonite  as  defendant,  the  
RTC  held,  viz:  "Abante  Tonite"  is  a  daily  tabloid  of  general  circulation.  People  all  over  the  country  could  buy  
a  copy  of  "Abante  Tonite"  and  read  it,  hence,  it  is  for  public  consumption.  The  persons  who  organized  said  

   
3H  A.Y.  2017-­‐2018   79  
 
 

CORPORATION  LAW  CASE  DIGESTS  –  ATTY.  DANTE  DELA  CRUZ  

publication  obviously  derived  profit  from  it.  The  information  written  on  the  said  newspaper  will  affect  the  
person,  natural  as  well  as  juridical,  who  was  stated  or  implicated  in  the  news.  All  of  these  facts  imply  that  
"Abante  Tonite"  falls  within  the  provision  of  Art.  44  (2  or  3),  New  Civil  Code.  Assuming  arguendo  that  "Abante  
Tonite"   is   not   registered   with   the   Securities   and   Exchange   Commission,   it   is   deemed   a   corporation   by  
estoppels  considering  that  it  possesses  attributes  of  a  juridical  person,  otherwise  it  cannot  be  held  liable  for  
damages  and  injuries  it  may  inflict  to  other  persons.  
 
Undaunted,   petitioners   brought   a   petition   for   certiorari,   prohibition,   mandamus   in   the   CA   to   nullify   the  
orders  of  the  RTC.  CA  dismiss  the  petition  for  certiorari,  prohibition,  mandamus.  The  CA  ruled:  Anent  the  
issue  raised  by  petitioners  that  "Abante  Tonite  is  neither  a  natural  or  juridical  person  who  may  be  a  party  in  
a   civil   case,"   and   therefore   the   case   against   it   must   be   dismissed   and/or   dropped,   is   untenable.   The  
respondent  Judge,  in  denying  petitioners’  motion  for  reconsideration,  held  that:  Abante  Tonite’s  newspapers  
are   circulated   nationwide,   showing   ostensibly   its   being   a   corporate   entity,   thus   the   doctrine   of   corporation  
by   estoppel   may   appropriately   apply.   An   unincorporated   association,   which   represents   itself   to   be   a  
corporation,   will   be   estopped   from   denying   its   corporate   capacity   in   a   suit   against   it   by   a   third   person   who  
relies   in   good   faith   on   such   representation.   There   being   no   grave   abuse   of   discretion   committed   by   the  
respondent  Judge  in  the  exercise  of  his  jurisdiction,  the  relief  of  prohibition  is  also  unavailable.  
 
ISSUE:    
Whether  or  not  Abante  Tonite  should  be  included  as  party  in  the  instant  case.  
 
HELD:  
YES.  There  is  no  question  that  Sheriff  Medina  twice  attempted  to  serve  the  summons  upon  each  of  petitioners  
in   person   at   their   office   address,   the   first   in   the   morning   of   September   18,   2000   and   the   second   in   the  
afternoon  of  the  same  date.  Each  attempt  failed  because  Macasaet  and  Quijano  were  "always  out  and  not  
available"  and  the  other  petitioners  were  "always  roving  outside  and  gathering  news."  After  Medina  learned  
from   those   present   in   the   office   address   on   his   second   attempt   that   there   was   no   likelihood   of   any   of  
petitioners   going   to   the   office   during   the   business   hours   of   that   or   any   other   day,   he   concluded   that   further  
attempts   to   serve   them   in   person   within   a   reasonable   time   would   be   futile.   The   circumstances   fully  
warranted  his  conclusion.  He  was  not  expected  or  required  as  the  serving  officer  to  effect  personal  service  
by  all  means  and  at  all  times,  considering  that  he  was  expressly  authorized  to  resort  to  substituted  service  
should   he   be   unable   to   effect   the   personal   service   within   a   reasonable   time.   In   that   regard,   what   was   a  
reasonable  time  was  dependent  on  the  circumstances  obtaining.  While  we  are  strict  in  insisting  on  personal  
service   on   the   defendant,   we   do   not   cling   to   such   strictness   should   the   circumstances   already   justify  
substituted  service  instead.  It  is  the  spirit  of  the  procedural  rules,  not  their  letter,  that  governs.  
 
The   Supreme   Court   cannot   we   sustain   petitioners’   contention   that   Abante   Tonite   could   not   be   sued   as   a  
defendant   due   to   its   not   being   either   a   natural   or   a   juridical   person.   In   rejecting   their   contention,   the   CA  
categorized  Abante  Tonite  as  a  corporation  by  estoppel  as  the  result  of  its  having  represented  itself  to  the  
reading   public   as   a   corporation   despite   its   not   being   incorporated.   Thereby,   the   CA   concluded   that   the   RTC  
did  not  gravely  abuse  its  discretion  in  holding  that  the  non-­‐‑incorporation  of  Abante  Tonite  with  the  Securities  
and  Exchange  Commission  was  of  no  consequence,  for,  otherwise,  whoever  of  the  public  who  would  suffer  
any  damage  from  the  publication  of  articles  in  the  pages  of  its  tabloids  would  be  left  without  recourse.  We  
cannot   disagree   with   the   CA,   considering   that   the   editorial   box   of   the   daily   tabloid   disclosed   that   basis,  
nothing  in  the  box  indicated  that  Monica  Publishing  Corporation  had  owned  Abante  Tonite.  
 
 
 
59.  ABOITIZ  EQUITY  VENTURES,  INC.  V.  CHIONGBIAN  
G.R.  NO.  197530.  JULY  9,  2014  

   
3H  A.Y.  2017-­‐2018   80  
 
 

CORPORATION  LAW  CASE  DIGESTS  –  ATTY.  DANTE  DELA  CRUZ  

LEONEN,  J.;  
 
Doctrine:   It   is   basic   that   a   corporation   has   a   personality   separate   and   distinct   from   that   of   its   individual  
stockholders.  Thus,  a  stockholder  does  not  automatically  assume  the  liabilities  of  the  corporation  of  which  
he  is  a  stockholder.  
 
Facts:  
On  January  8,  1996,  Aboitiz  Shipping  Corporation  ("ASC"),  principally  owned  by  the  Aboitiz  family,  CAGLI,  
principally   owned   by   the   Gothong   family,   and   William   Lines,   Inc.   ("WLI"),   principally   owned   by   the  
Chiongbian  family,  entered  into  an  agreement  (the  "Agreement"),  4  whereby  ASC  and  CAGLI  would  transfer  
their  shipping  assets  to  WLI  in  exchange  for  WLI's  shares  of  stock.  5  WLI,  in  turn,  would  run  their  merged  
shipping  businesses  and,  henceforth,  be  known  as  WG&A,  Inc.  ("WG&A").    
 
Among  the  attachments  to  the  Agreement  was  Annex  SL-­‐‑V.  This  was  a  letter  dated  January  8,  1996,  from  WLI,  
through   its   President   (herein   respondent)   Victor   S.   Chiongbian   addressed   to   CAGLI,   through   its   Chief  
Executive   Officer   Bob   D.   Gothong   and   Executive   Vice   President   for   Engineering   (herein   respondent)  
Benjamin  D.  Gothong.  On  its  second  page,  Annex  SL-­‐‑V  bore  the  signatures  of  Bob  D.  Gothong  and  respondent  
Benjamin  D.  Gothong  by  way  of  a  conforme  on  behalf  of  CAGLI.  IEAaST  
 
Annex  SL-­‐‑V  confirmed  WLI's  commitment  to  acquire  certain  inventories  of  CAGLI.  These  inventories  would  
have  a  total  aggregate  value  of,  at  most,  PhP400  million,  "as  determined  after  a  special  examination  of  the  
[i]nventories".  Annex  SL-­‐‑V  also  specifically  stated  that  such  acquisition  was  "pursuant  to  the  Agreement".  
 
Pursuant  to  Annex  SL-­‐‑V,  inventories  were  transferred  from  CAGLI  to  WLI.  These  inventories  were  assessed  
to  have  a  value  of  PhP514  million,  which  was  later  adjusted  to  PhP558.89  million.  Of  the  total  amount  of  
PhP558.89  million,  "CAGLI  was  paid  the  amount  of  PhP400  Million."  In  addition  to  the  payment  of  PhP400  
million,  petitioner  Aboitiz  Equity  Ventures  ("AEV")  noted  that  WG&A  shares  with  a  book  value  of  PhP38.5  
million  were  transferred  to  CAGLI.  
 
Sometime  in  2002,  the  Chiongbian  and  Gothong  families  decided  to  leave  the  WG&A  enterprise  and  sell  their  
interest  in  WG&A  to  the  Aboitiz  family.  As  such,  a  share  purchase  agreement  ("SPA")  was  entered  into  by  
petitioner  AEV  and  the  respective  shareholders  groups  of  the  Chiongbians  and  Gothongs.  In  the  SPA,  AEV  
agreed  to  purchase  the  Chiongbian  group's  40.61%  share  and  the  Gothong  group's  20.66%  share  in  WG&A's  
issued  and  outstanding  stock.  
 
As   a   result   of   the   SPA,   AEV   became   a   stockholder   of   WG&A.   Subsequently,   WG&A   was   renamed   Aboitiz  
Transport  Shipping  Corporation  ("ATSC").  
 
Petitioner   AEV   alleged   that   in   2008,   CAGLI   resumed   making   demands   despite   having   already   received  
PhP120.04   million   worth   of   excess   inventories.   CAGLI   initially   made   its   demand   to   ATSC   (the   renamed  
WLI/WG&A)   through   a   letter   dated   February   14,   2008.   As   alleged   by   AEV,   however,   CAGLI   subsequently  
resorted   to   a   "shotgun   approach"   and   directed   its   subsequent   demand   letters   to   AEV   29   as   well   as   to   FCLC  
(a  company  related  to  respondent  Chiongbian).  
 
AEV  responded  to  CAGLI's  demands  through  several  letters.  In  these  letters,  AEV  rebuffed  CAGLI's  demands  
noting  that:  (1)  CAGLI  already  received  the  excess  inventories;  (2)  it  was  not  a  party  to  CAGLI's  claim  as  it  
had  a  personality  distinct  from  WLI/WG&A/ATSC;  and  (3)  CAGLI's  claim  was  already  barred  by  prescription.  
 
Issue:  Whether  there  is  an  agreement  binding  AEV  to  arbitrate  with  CAGLI  on  the  latter's  
claims  arising  from  Annex  SL-­‐‑V.  

   
3H  A.Y.  2017-­‐2018   81  
 
 

CORPORATION  LAW  CASE  DIGESTS  –  ATTY.  DANTE  DELA  CRUZ  

 
Ratio:  
Annex  SL-­‐‑V  is  only  between  WLI  and  CAGLI  —  it  necessarily  follows  that  none  but  WLI/WG&A/ATSC  and  
CAGLI   are   bound   by   the   terms   of   Annex   SL-­‐‑V.   It   is   elementary   that   contracts   are   characterized   by   relativity  
or  privity,  that  is,  that  "[c]ontracts  take  effect  only  between  the  parties,  their  assigns  and  heirs".  As  such,  one  
who  is  not  a  party  to  a  contract  may  not  seek  relief  for  such  contract's  breach.  Likewise,  one  who  is  not  a  
party  to  a  contract  may  not  be  held  liable  for  breach  of  any  its  terms.  HAEIac  
 
While   the   principle   of   privity   or   relativity   of   contracts   acknowledges   that   contractual   obligations   are  
transmissible  to  a  party's  assigns  and  heirs,  AEV  is  not  WLI's  successor-­‐‑in-­‐‑interest.  In  the  period  relevant  to  
this   petition,   the   transferee   of   the   inventories   transferred   by   CAGLI   pursuant   to   Annex   SL-­‐‑V   assumed   three  
(3)   names:   (1)   WLI,   the   original   name   of   the   entity   that   survived   the   merger   under   the   January   8,   1996  
Agreement;  (2)  WG&A,  the  name  taken  by  WLI  in  the  wake  of  the  Agreement;  and  (3)  ATSC,  the  name  taken  
by  WLI/WG&A  in  the  wake  of  the  SPA.  As  such,  it  is  now  ATSC  that  is  liable  under  Annex  SL-­‐‑V.  
 
Pursuant  to  the  January  8,  1996  Agreement,  the  Aboitiz  group  (via  ASC)  and  the  Gothong  group  (via  CAGLI)  
became  stockholders  of  WLI/WG&A,  along  with  the  Chiongbian  group  (which  initially  controlled  WLI).  This  
continued  until,  pursuant  to  the  SPA,  the  Gothong  group  and  the  Chiongbian  group  transferred  their  shares  
to  AEV.  With  the  SPA,  AEV  became  a  stockholder  of  WLI/WG&A,  which  was  subsequently  renamed  ATSC.  
Nonetheless,  AEV's  status  as  ATSC's  stockholder  does  not  subject  it  to  ATSC's  obligations.  
 
It  is  basic  that  a  corporation  has  a  personality  separate  and  distinct  from  that  of  its  individual  stockholders.  
Thus,   a   stockholder   does   not   automatically   assume   the   liabilities   of   the   corporation   of   which   he   is   a  
stockholder.  
 
 
 
60.  ERIC  GODFREY  STANLEY  LIVESEY  V  BINSWANGER  PHILIPPINES,  INC.  AND  KEITH  ELLIOT  
G.R.  NO.  177493  -­‐‑  MARCH  19,  2014  
BRION,  J.  
 
DOCTRINE:  
Evasion   of   unfulfilled   financial   obligation   can   only   be   attributed   to   the   President   as   it   was   apparent   that  
Binswanger's  stockholders  had  nothing  to  do  with  the  corporation’s  operations.  The  President  knew  that  the  
corporation  had  not  fully  complied  with  its  financial  obligation  under  the  compromise  agreement.  He  made  
sure  that  it  would  not  be  fulfilled  when  he  allowed  the  corporation's  closure,  despite  the  condition  in  the  
agreement   that   "unless   and   until   the   Compromise   Amount   has   been   fully   settled   and   paid   by,   the   Company  
shall  not  suspend,  discontinue,  or  cease  its  entire  or  a  substantial  portion  of  its  business  operations."  
 
FACTS:  
Eric  Godfrey  Stanley  Livesey  filed  a  complaint  for  illegal  dismissal  with  money  claims  against  CBB  Philippines  
Strategic   Property   Services,   Inc.   (CBB)   and   Paul   Dwyer.   CBB   was   a   domestic   corporation   engaged   in   real  
estate  brokerage  and  Dwyer  was  its  President.  
 
The  Labor  Arbiter  (LA)  found  that  Livesey  had  been  illegally  dismissed.  The  LA  ordered  CBB  to  reinstate  
Livesey  to  his  former  position  and  to  pay  him  US$23,000.00  in  accrued  salaries,  and  US$5,000.00  a  month  in  
back   salaries   (from   January   2002   until   reinstatement);   and   10%   of   the   total   award   as   attorney's   fees.  
Thereafter,  the  parties  entered  into  a  compromise  agreement  which  the  LA  approved.  Under  the  agreement,  
Livesey   was   to   receive   US$31,000.00   in   full   satisfaction   of   LA   Reyno's   decision.   CBB   paid   Livesey   the   initial  
amount  of  US$13,000.00,  but  not  the  remaining  amount  as  the  company  ceased  operations.  

   
3H  A.Y.  2017-­‐2018   82  
 
 

CORPORATION  LAW  CASE  DIGESTS  –  ATTY.  DANTE  DELA  CRUZ  

 
In  reaction,  Livesey  moved  for  the  issuance  of  a  writ  of  execution.  The  LA  granted  the  writ,  but  it  was  not  
enforced.  Livesey  then  filed  a  motion  for  the  issuance  of  an  alias  writ  of  execution,  alleging  that  in  the  process  
of  serving  respondents  the  writ,  he  learned  "that  respondents,  in  a  clear  and  willful  attempt  to  avoid  their  
liabilities  to  complainant  have  organized  another  corporation,  Binswanger  Philippines,  Inc."  He  claimed  that  
there  was  evidence  showing  that  CBB  and  Binswanger  Philippines,  Inc.  (Binswanger)  are  one  and  the  same  
corporation,  pointing  out  that  CBB  stands  for  Chesterton  Blumenauer  Binswanger.  Invoking  the  doctrine  of  
piercing   the   veil   of   corporate   fiction,   Livesey   prayed   that   an   alias   writ   of   execution   be   issued   against  
respondents  Binswanger  and  Keith  Elliot,  CBB's  former  President,  and  now  Binswanger's  President  and  Chief  
Executive  Officer  (CEO).  
 
Livesey  alleged,  among  others,  that  after  executing  the  compromise  agreement  with  him,  through  Elliot,  CBB  
ceased  operations  following  a  transaction  where  a  substantial  amount  of  CBB  shares  changed  hands.  Almost  
simultaneously  with  CBB's  closing,  Binswanger  was  established  with  its  headquarters  set  up  beside  CBB's  
office   ;   key   CBB   officers   and   employees   moved   to   Binswanger   led   by   Elliot,   former   CBB   President   who  
became   Binswanger's   President   and   CEO,   Ferdie   Catral,   former   CBB   Director   and   Head   of   Operations,  
Evangeline  Agcaoili  and  Janet  Pei;  summons  served  on  Binswanger  in  an  earlier  labor  case  was  received  by  
Binswanger  using  CBB's  receiving  stamp;  notwithstanding  CBB's  closure,  Binswanger's  Web  Editor  (Young),  
in   an   e-­‐‑mail   correspondence,   supplied   the   information   that   Binswanger   is   "now   known"   as   either   CBB  
(Chesterton  Blumenauer  Binswanger  or  as  Chesterton  Petty,  Ltd.,  in  the  Philippines.  
 
 
ISSUE:  
Should  the  doctrine  of  piercing  the  veil  of  corporate  fiction  be  applied  to  the  case  and  make  the  President  
liable?  
 
HELD:  
YES.  Piercing  the  veil  of  corporate  fiction  is  an  equitable  doctrine  developed  to  address  situations  where  the  
separate  corporate  personality  of  a  corporation  is  abused  or  used  for  wrongful  purposes.  Under  the  doctrine,  
the  corporate  existence  may  be  disregarded  where  the  entity  is  formed  or  used  for  non-­‐‑legitimate  purposes,  
such  as  to  evade  a  just  and  due  obligation,  or  to  justify  a  wrong,  to  shield  or  perpetrate  fraud  or  to  carry  out  
similar  or  inequitable  considerations,  other  unjustifiable  aims  or  intentions,  in  which  case,  the  fiction  will  be  
disregarded  and  the  individuals  composing  it  and  the  two  corporations  will  continue,  as  it  did  continue,  CBB's  
real  estate  brokerage  business.  
 
Livesey's   evidence,   whose   existence   the   respondents   never   denied,   converged   to   show   this   continuity   of  
business  operations  from  CBB  to  Binswanger.  
 
While  the  ostensible  reason  for  Binswanger's  establishment  is  to  continue  CBB's  business  operations  in  the  
Philippines,   which   by   itself   is   not   illegal,   the   close   proximity   between   CBB's   disestablishment   and  
Binswanger's  coming  into  existence  points  to  an  unstated  but  urgent  consideration  which  was  to  evade  CBB's  
unfulfilled  financial  obligation  to  Livesey  under  the  compromise  agreement.  
 
This   underhanded   objective,   it   must   be   stressed,   can   only   be   attributed   to   Elliot   as   it   was   apparent   that  
Binswanger's  stockholders  had  nothing  to  do  with  Binswanger's  operations  as  noted  by  the  NLRC  and  which  
the  respondents  did  not  deny.  Elliot  was  well  aware  of  the  compromise  agreement  between  Livesey  and  CBB,  
as  he  "agreed  and  accepted"  the  terms  of  the  agreement  for  CBB.  He  was  also  well  aware  that  the  last  two  
installments   of   CBB's   obligation   to   Livesey   were   due   on   June   30,   2003   and   September   30,   2003.   These  
installments  were  not  met  and  the  reason  is  that  after  the  alleged  sale  of  the  majority  of  CBB's  shares  of  stock,  
it  closed  down.  

   
3H  A.Y.  2017-­‐2018   83  
 
 

CORPORATION  LAW  CASE  DIGESTS  –  ATTY.  DANTE  DELA  CRUZ  

 
Elliot   knew   that   CBB   had   not   fully   complied   with   its   financial   obligation   under   the   compromise   agreement.  
He   made   sure   that   it   would   not   be   fulfilled   when   he   allowed   CBB's   closure,   despite   the   condition   in   the  
agreement  that  "unless  and  until  the  Compromise  Amount  has  been  fully  settled  and  paid  by  the  Company  in  
favor  of  Mr.  Livesey,  the  Company  shall  not  suspend,  discontinue,  or  cease  its  entire  or  a  substantial  portion  
of  its  business  operations."  
 
Therefore,  Elliot  is  as  liable  as  Binswanger  for  CBB's  unfulfilled  obligation  to  Livesey.  
 
 
61.   HEIRS   OF   FE   TAN   UY   (REPRESENTED   BY   HER   HEIR,   MAULING   UY   LIM)   VS.   INTERNATIONAL  
EXCHANGE  BANK,    
G.R.  NO.  166282      FEBRUARY  13,  2013  
MENDOZA,  J.:  
 
DOCTRINE:   A   corporation   is   a   juridical   entity   which   is   vested   with   a   legal   personality   separate   and   distinct  
from  those  acting  for  and  in  its  behalf  and,  in  general,  from  the  people  comprising  it.  Following  this  principle,  
obligations   incurred   by   the   corporation,   acting   through   its   directors,   officers   and   employees,   are   its   sole  
liabilities.   A   director,   officer   or   employee   of   a   corporation   is   generally   not   held   personally   liable   for  
obligations   incurred   by   the   corporation.   Nevertheless,   this   legal   fiction   may   be   disregarded   if   it   is   used   as   a  
means   to   perpetrate   fraud   or   an   illegal   act,   or   as   a   vehicle   for   the   evasion   of   an   existing   obligation,   the  
circumvention  of  statutes,  or  to  confuse  legitimate  issues.  Before  a  director  or  officer  of  a  corporation  can  be  
held   personally   liable   for   corporate   obligations,   however,   the   following   requisites   must   concur:   (1)   the  
complainant  must  allege  in  the  complaint  that  the  director  or  officer  assented  to  patently  unlawful  acts  of  the  
corporation,   or   that   the   officer   was   guilty   of   gross   negligence   or   bad   faith;   and   (2)   the   complainant   must  
clearly  and  convincingly  prove  such  unlawful  acts,  negligence  or  bad  faith  
 
FACTS:  On  several  occasions,  from  June  23,  1997  to  September  3,  1997,  respondent  International  Exchange  
Bank  (iBank),  granted  loans  to  Hammer  Garments  Corporation  (Hammer),  covered  by  promissory  notes  and  
deeds  of  assignment.    
 
As   of   October   28,   1997,   Hammer   had   an   outstanding   obligation   of   P25,420,177.62   to   iBank.   Hammer  
defaulted   in   the   payment   of   its   loans,   prompting   iBank   to   foreclose   on   Goldkey’s   third-­‐‑party   Real   Estate  
Mortgage.  The  mortgaged  properties  were  sold  for  P  12  million  during  the  foreclosure  sale,  leaving  an  unpaid  
balance  of  P  13,420,177.62.  For  failure  of  Hammer  to  pay  the  deficiency,  iBank  filed  Complaint  for  sum  of  
money  on  December  16,  1997  against  Hammer,  Chua,  Uy,  and  Goldkey  before  the  Regional  Trial  Court,  Makati  
City  (RTC).  
 
Despite  service  of  summons,  Chua  and  Hammer  did  not  file  their  respective  answers  and  were  declared  in  
default.  In  her  separate  answer,  Uy  claimed  that  she  was  not  liable  to  iBank  because  she  never  executed  a  
surety  agreement  in  favor  of  iBank.  Goldkey,  on  the  other  hand,  also  denies  liability,  averring  that  it  acted  
only  as  a  third-­‐‑party  mortgagor  and  that  it  was  a  corporation  separate  and  distinct  from  Hammer.  
 
Aggrieved,   the   heirs   of   Uy   and   Goldkey   (petitioners)   elevated   the   case   to   the   CA.   On   August   16,   2004,   it  
promulgated  its  decision  affirming  the  findings  of  the  RTC.  The  CA  found  that  iBank  was  not  negligent  in  
evaluating  the  financial  stability  of  Hammer.  According  to  the  appellate  court,  iBank  was  induced  to  grant  
the  loan  because  petitioners,  with  intent  to  defraud  the  bank,  submitted  a  falsified  Financial  Report  for  1996  
which  incorrectly  declared  the  assets  and  cashflow  of  Hammer.  Because  petitioners  acted  maliciously  and  in  
bad  faith  and  used  the  corporate  fiction  to  defraud  iBank,  they  should  be  treated  as  one  and  the  same  as  
Hammer.  

   
3H  A.Y.  2017-­‐2018   84  
 
 

CORPORATION  LAW  CASE  DIGESTS  –  ATTY.  DANTE  DELA  CRUZ  

 
Hence,   these   petitions   filed   separately   by   the   heirs   of   Uy   and   Goldkey.   On   February   9,   2005,   this   Court  
ordered  the  consolidation  of  the  two  cases.  
 
ISSUE:  Whether  or  not  Uy  can  be  held  liable  to  iBank  for  the  loan  obligation  of  Hammer  as  an  officer  and  
stockholder  of  the  said  corporation  
 
 
HELD:  NO.  Uy  is  not  liable  While  it  is  true  that  the  determination  of  the  existence  of  any  of  the  circumstances  
that   would   warrant   the   piercing   of   the   veil   of   corporate   fiction   is   a   question   of   fact   which   cannot   be   the  
subject  of  a  petition  for  review  on  certiorari  under  Rule  45,  this  Court  can  take  cognizance  of  factual  issues  if  
the   findings   of   the   lower   court   are   not   supported   by   the   evidence   on   record   or   are   based   on   a  
misapprehension  of  facts.    
 
In  this  case,  petitioners  are  correct  to  argue  that  it  was  not  alleged,  much  less  proven,  that  Uy  committed  an  
act  as  an  officer  of  Hammer  that  would  permit  the  piercing  of  the  corporate  veil.  A  reading  of  the  complaint  
reveals   that   with   regard   to   Uy,   iBank   did   not   demand   that   she   be   held   liable   for   the   obligations   of   Hammer  
because  she  was  a  corporate  officer  who  committed  bad  faith  or  gross  negligence  in  the  performance  of  her  
duties  such  that  the  lifting  of  the  corporate  mask  would  be  merited.  What  the  complaint  simply  stated  is  that  
she,  together  with  her  errant  husband  Chua,  acted  as  surety  of  Hammer,  as  evidenced  by  her  signature  on  
the  Surety  Agreement  which  was  later  found  by  the  RTC  to  have  been  forged.  
 
Considering  that  the  only  basis  for  holding  Uy  liable  for  the  payment  of  the  loan  was  proven  to  be  a  falsified  
document,   there   was   no   sufficient   justification   for   the   RTC   to   have   ruled   that   Uy   should   be   held   jointly   and  
severally   liable   to   iBank   for   the   unpaid   loan   of   Hammer.   Neither   did   the   CA   explain   its   affirmation   of   the  
RTC’s  ruling  against  Uy.  The  Court  cannot  give  credence  to  the  simplistic  declaration  of  the  RTC  that  liability  
would  attach  directly  to  Uy  for  the  sole  reason  that  she  was  an  officer  and  stockholder  of  Hammer.  At  most,  
Uy  could  have  been  charged  with  negligence  in  the  performance  of  her  duties  as  treasurer  of  Hammer  by  
allowing  the  company  to  contract  a  loan  despite  its  precarious  financial  position.  
 
 
 
62  NO  CASE  
 
 
 
63.  MEDICAL  PLAZA  MAKATI  CONDOMINIUM  CORPORATION  VS.  CULLEN  
GR  NO.  181416/  NOVEMBER  11,  2013  
PERALTA,  J.  
 
DOCTRINE:  The  nature  of  an  action  involving  any  dispute  as  to  the  validity  of  the  assessment  of  association  
dues  has  been  settled  by  the  Court  in  Chateau  de  Baie  Condominium  Corporation  v.  Moreno.  The  Court  held  
that  the  dispute  as  to  the  validity  of  the  assessments  is  purely  an  intra-­‐‑corporate  matter.  More  so  in  this  case  
as   respondent   repeatedly   questioned   his   characterization   as   a   delinquent   member   and,   consequently,  
petitioner’s   decision   to   bar   him   from   exercising   his   rights   to   vote   and   be   voted   for.   These   issues   are   clearly  
corporate  and  the  demand  for  damages  is  just  incidental.  Being  corporate  in  nature,  the  issues  should  be  
threshed  out  before  the  RTC  sitting  as  a  special  commercial  court.  Pursuant  to  Section  5.2  of  RA  No.  8799,  
otherwise  known  as  the  Securities  Regulation  Code,  the  jurisdiction  of  the  SEC  over  all  cases  enumerated  
under   Section   5   of   Presidential   Decree   No.   902-­‐‑A   has   been   transferred   to   RTCs   designated   by   this  Court   as  
Special  Commercial  Courts.  

   
3H  A.Y.  2017-­‐2018   85  
 
 

CORPORATION  LAW  CASE  DIGESTS  –  ATTY.  DANTE  DELA  CRUZ  

FACTS:  Respondent  Robert  H.  Cullen  purchased  from  Meridien  Land  Holding,  Inc.  (MLHI)  a  condominium  
unit   of   the   Medical   Plaza   Makati.   A   Condominium   Certificate   of   Title   was   issued   in   the   name   of   respondent.  
Petitioner,   through   its   corporate   secretary,   Dr.   Jose   Giovanni   E.   Dimayuga,   demanded   from   respondent  
payment  for  alleged  unpaid  association  dues  and  assessments  amounting.  Respondent  disputed  this  demand  
claiming   that   he   had   been   religiously   paying   his   dues   shown   by   the   fact   that   he   was   previously   elected  
president  and  director  of  petitioner.  Petitioner  claimed  that  respondent’s  obligation  was  a  carry-­‐‑over  of  that  
of  MLHI.  Consequently,  respondent  was  prevented  from  exercising  his  right  to  vote  and  be  voted  for  during  
the   2002   election   of   petitioner’s   Board   of   Directors.   Respondent   thus   clarified   from   MLHI   the   veracity   of  
petitioner’s  claim,  but  MLHI  allegedly  claimed  that  the  same  had  already  been  settled.  Respondent  demanded  
from   petitioner   an   explanation   why   he   was   considered   a   delinquent   payer   despite   the   settlement   of   the  
obligation.  Petitioner  failed  to  make  such  explanation.  Hence,  the  Complaint  for  Damages  filed  by  respondent  
against   petitioner   and   MLHI.   RTC   dismissed   respondent’s   complaint.   On   appeal,   the   CA   reversed   and  
remanded  the  case  to  the  RTC  for  further  proceedings.  Aggrieved,  petitioner  comes  before  the  Court.    
 
ISSUE:  Does  the  controversy  involve  intra-­‐‑corporate  issues  as  would  fall  within  the  jurisdiction  of  the  RTC  
sitting   as   a   special   commercial   court  or   an   ordinary   action   for   damages   within   the   jurisdiction   of   regular  
courts  
 
HELD:   RTC,   sitting   as   special   commercial   court.   In   determining   whether   a   dispute   constitutes   an   intra-­‐‑
corporate   controversy,   the   Court   uses   two   tests,   namely,   the   relationship   test   and   the   nature   of   the  
controversy  test.  
An  intra-­‐‑corporate  controversy  is  one  which  pertains  to  any  of  the  following  relationships:  (1)  between  the  
corporation,   partnership   or   association   and   the   public;   (2)   between   the   corporation,   partnership   or  
association  and  the  State  insofar  as  its  franchise,  permit  or  license  to  operate  is  concerned;  (3)  between  the  
corporation,   partnership   or   association   and   its   stockholders,   partners,   members   or   officers;   and   (4)   among  
the  stockholders,  partners  or  associates  themselves.  Thus,  under  the  relationship  test,  the  existence  of  any  
of   the   above   intra-­‐‑corporate   relations   makes   the   case   intra-­‐‑corporate.   Under   the   nature   of   the   controversy  
test,   jurisdiction   should   be   determined   by   considering   both   the   relationship   of   the   parties   as   well   as   the  
nature   of   the   question   involved.   Applying   the   two   tests,   we   find   and   so   hold   that   the   case   involves   intra-­‐‑
corporate  controversy.    
The   nature   of   the   action   is   determined   by   the   body   rather   than   the   title   of   the   complaint.   Though  
denominated   as   an   action   for   damages,   an   examination   of   the   allegations   made   by   respondent   in   his  
complaint   shows   that   the   case   principally   dwells   on   the   propriety   of   the   assessment   made   by   petitioner  
against   respondent   as   well   as   the   validity   of   petitioner’s   act   in   preventing   respondent   from   participating   in  
the  election  of  the  corporation’s  Board  of  Directors.    
The  issue  is  not  novel.  The  nature  of  an  action  involving  any  dispute  as  to  the  validity  of  the  assessment  of  
association  dues  has  been  settled  by  the  Court  in  Chateau  de  Baie  Condominium  Corporation  v.  Moreno.  The  
Court  held  that  the  dispute  as  to  the  validity  of  the  assessments  is  purely  an  intra-­‐‑corporate  matter.  More  so  
in   this   case   as   respondent   repeatedly   questioned   his   characterization   as   a   delinquent   member   and,  
consequently,  petitioner’s  decision  to  bar  him  from  exercising  his  rights  to  vote  and  be  voted  for.  These  issues  
are  clearly  corporate  and  the  demand  for  damages  is  just  incidental.  Being  corporate  in  nature,  the  issues  
should  be  threshed  out  before  the  RTC  sitting  as  a  special  commercial  court.    
Moreover,   Presidential   Decree   No.   902-­‐‑A   enumerates   the   cases   over   which   SEC   exercises   exclusive  
jurisdiction:  
a)   Controversies   arising   out   of   intra-­‐‑corporate   or   partnership   relations,   between   and   among  
stockholders,  members  or  associates;  between  any  or  all  of  them  and  the  corporation,  partnership  or  
association  of  which  they  are  stockholders,  members,  or  associates,  respectively;  and  between  such  
corporation,  partnership  or  association  and  the  State  insofar  as  it  concerns  their  individual  franchise  
or  right  to  exist  as  such  entity;  and  

   
3H  A.Y.  2017-­‐2018   86  
 
 

CORPORATION  LAW  CASE  DIGESTS  –  ATTY.  DANTE  DELA  CRUZ  

b)   Controversies   in   the   election   or   appointment   of   directors,   trustees,   officers,   or   managers   of   such  


corporations,  partnerships,  or  associations.29  
To  be  sure,  this  action  partakes  of  the  nature  of  an  intra-­‐‑corporate  controversy,  the  jurisdiction  over  which  
pertains  to  the  SEC.  Pursuant  to  Section  5.2  of  RA  No.  8799,  otherwise  known  as  the  Securities  Regulation  
Code,  the  jurisdiction  of  the  SEC  over  all  cases  enumerated  under  Section  5  of  Presidential  Decree  No.  902-­‐‑A  
has  been  transferred  to  RTCs  designated  by  this  Court  as  Special  Commercial  Courts.  CA  gravely  erred  in  
remanding  the  case  to  the  RTC  for  further  proceedings.  
Indeed,  RA  No.  9904,  or  the  Magna  Carta  for  Homeowners  and  Homeowners’  Associations  empowers  the  
HLURB  to  hear  and  decide  inter-­‐‑association  and/or  intra-­‐‑association  controversies  or  conflicts  concerning  
homeowners’   associations.   However,   we   cannot   apply   the   same   in   the   present   case   as   it   involves   a  
controversy   between   a   condominium   unit   owner   and   a   condominium   corporation.   While   the   term  
association   as   defined   in   the   law   covers   homeowners’   associations   of   other   residential   real   property   which  
is  broad  enough  to  cover  a  condominium  corporation,  it  does  not  seem  to  be  the  legislative  intent.  A  thorough  
review  of  the  deliberations  of  the  bicameral  conference  committee  would  show  that  the  lawmakers  did  not  
intend  to  extend  the  coverage  of  the  law  to  such  kind  of  association.    
 
 
 
64.  GAMBOA  V.  TEVEZ  
 
 
 
65.  NARRA  NICKEL  MINING  AND  DEVELOPMENT  CORP.,  TESORO  MINING  AND  DEVELOPMENT,  INC.,  
AND  MCARTHUR  MINING,  INC.  VS.  REDMONT  CONSOLIDATED  MINES  CORP.  
G.R.  NO.  195580  APRIL  21,  2014  
VELASCO,  JR.,  J.:  
 
DOCTRINE:  The  Grandfather  Rule  applies  only  when  the  60-­‐‑40  Filipino-­‐‑foreign  equity  ownership  is  in  doubt.    
Stated  differently,  where  the  60-­‐‑40  Filipino-­‐‑  foreign  equity  ownership  is  not  in  doubt,  the  Grandfather  Rule  
will  not  apply.  
 
FACTS:  Sometime  in  December  2006,  respondent  Redmont  Consolidated  Mines  Corp.  (Redmont),  a  domestic  
corporation  organized  and  existing  under  Philippine  laws,  took  interest  in  mining  and  exploring  certain  areas  
of   the   province   of   Palawan.   After   inquiring   with   the   Department   of   Environment   and   Natural   Resources  
(DENR),   it   learned   that   the   areas   where   it   wanted   to   undertake   exploration   and   mining   activities   where  
already  covered  by  Mineral  Production  Sharing  Agreement  (MPSA)  applications  of  petitioners  Narra,  Tesoro  
and  McArthur.  
 
Petitioner   McArthur,   through   its   predecessor-­‐‑in-­‐‑interest   Sara   Marie   Mining,   Inc.   (SMMI),   filed   an  
application  for  an  MPSA  and  Exploration  Permit  (EP)  with  the  Mines  and  Geo-­‐‑Sciences  Bureau  (MGB),  Region  
IV-­‐‑B,  Office  of  the  Department  of  Environment  and  Natural  Resources  (DENR).  Petitioner  Narra  acquired  its  
MPSA   from   Alpha   Resources   and   Development   Corporation   and   Patricia   Louise   Mining   &   Development  
Corporation  (PLMDC).    
 
On   January   2,   2007,   Redmont   filed   before   the   Panel   of   Arbitrators   (POA)   of   the   DENR   three   (3)   separate  
petitions  for  the  denial  of  petitioners’  applications  for  MPSA.    
 
In  the  petitions,  Redmont  alleged  that  at  least  60%  of  the  capital  stock  of  McArthur,  Tesoro  and  Narra  
are   owned   and   controlled   by   MBMI   Resources,   Inc.   (MBMI),   a   100%   Canadian   corporation.   Redmont  
reasoned   that   since   MBMI   is   a   considerable   stockholder   of   petitioners,   it   was   the   driving   force   behind  

   
3H  A.Y.  2017-­‐2018   87  
 
 

CORPORATION  LAW  CASE  DIGESTS  –  ATTY.  DANTE  DELA  CRUZ  

petitioners’   filing   of   the   MPSAs   over   the   areas   covered   by   applications   since   it   knows   that   it   can   only  
participate  in  mining  activities  through  corporations  which  are  deemed  Filipino  citizens.  Redmont  argued  
that  given  that  petitioners’  capital  stocks  were  mostly  owned  by  MBMI,  they  were  likewise  disqualified  from  
engaging  in  mining  activities  through  MPSAs,  which  are  reserved  only  for  Filipino  citizens.  
 
In   their   Answers,   petitioners   averred   that   they   were   qualified   persons   under   Section   3(aq)   of  
Republic  Act  No.  (RA)  7942  or  the  Philippine  Mining  Act  of  1995.  They  claimed  that  the  issue  on  nationality  
should  not  be  raised  since  McArthur,  Tesoro  and  Narra  are  in  fact  Philippine  Nationals  as  60%  of  their  capital  
is  owned  by  citizens  of  the  Philippines.  They  asserted  that  though  MBMI  owns  40%  of  the  shares  of  PLMC,    
40%  of  the  shares  of  MMC  and  40%  of  the  shares  of  SLMC,  the  shares  of  MBMI  will  not  make  it  the  owner  of  
at  least  60%  of  the  capital  stock  of  each  of  petitioners.  They  added  that  the  best  tool  used  in  determining  the  
nationality   of   a   corporation   is   the   "control   test,"   embodied   in   Sec.   3   of   RA   7042   or   the   Foreign   Investments  
Act  of  1991.    
 
The   POA   issued   a   Resolution   disqualifying   petitioners   from   gaining   MPSAs.   The   POA   considered  
petitioners   as   foreign   corporations   being   "effectively   controlled"   by   MBMI,   a   100%   Canadian   company   and  
declared  their  MPSAs  null  and  void.  
 
ISSUE:  Whether  or  not  Narra,  Tesoro  and  McArthur  are  foreign  corporations.  
 
HELD:  Yes.  There  are  two  acknowledged  tests  in  determining  the  nationality  of  a  corporation:  the  control  
test   and   the   grandfather   rule.     Shares   belonging   to   corporations   or   partnerships   at   least   60%   of   the   capital  
of   which   is   owned   by   Filipino   citizens   shall   be   considered   as   of   Philippine   nationality,   but   if   the   percentage  
of   Filipino   ownership   in   the   corporation   or   partnership   is   less   than   60%,   only   the   number   of   shares  
corresponding  to  such  percentage  shall  be  counted  as  of  Philippine  nationality.    
 
 
The   Grandfather   Rule   applies   only   when   the   60-­‐‑40   Filipino-­‐‑foreign   equity   ownership   is   in   doubt.    
Stated  differently,  where  the  60-­‐‑40  Filipino-­‐‑  foreign  equity  ownership  is  not  in  doubt,  the  Grandfather  Rule  
will  not  apply.  After  a  scrutiny  of  the  evidence  extant  on  record,  the  Court  finds  that  this  case  calls  for  the  
application   of   the   grandfather   rule   since   doubt   prevails   and   persists   in   the   corporate   ownership   of  
petitioners.  Also,doubt  is  present  in  the  60-­‐‑40  Filipino  equity  ownership  of  petitioners  Narra,  McArthur  and  
Tesoro,  since  their  common  investor,  the  100%  Canadian  corporation––MBMI,  funded  them.    
 
(To  establish  the  actual  ownership,  interest  or  participation  of  MBMI  in  each  of  petitioners’  corporate  
structure,  they  have  to  be  "grandfathered."  Under  corporate  structure  of  MMC,  it  has  a  similar  structure  and  
composition   as   McArthur.   It   would   seem   that   MBMI   is   also   a   major   investor   and   "controls"   MBMI  and   also,  
similar   nominal   shareholders   were   present.   Olympic   Mines   &   Development   Corporation   (the   majority  
stockholder  in  MMC)  did  not  pay  any  amount  with  respect  to  the  number  of  shares  they  subscribed  to  in  the  
corporation,  which  is  quite  absurd  since  Olympic  is  the  major  stockholder  in  MMC.    
Thus,  McArthur,  when  it  is  "grandfathered,"  company  layering  was  utilized  by  MBMI  to  gain  control  over  
McArthur.  It  is  apparent  that  MBMI  has  more  than  60%  or  more  equity  interest  in  McArthur,  making  the  
latter  a  foreign  corporation.  Accordingly,  after  "grandfathering"  petitioner  Tesoro  and  factoring  in  Olympic’s  
participation  in  SMMI’s  corporate  structure,  it  is  clear  that  MBMI  is  in  control  of  Tesoro  and  owns  60%  or  
more   equity   interest   in   Tesoro.   This   makes   petitioner   Tesoro   a   non-­‐‑Filipino   corporation   and,   thus,  
disqualifies  it  to  participate  in  the  exploitation,  utilization  and  development  of  our  natural  resources.)  
 
Concluding   from   the   stated   facts,   it   is   quite   safe   to   say   that   petitioners   McArthur,   Tesoro   and   Narra  
are  not  Filipino  since  MBMI,  a  100%  Canadian  corporation,  owns  60%  or  more  of  their  equity  interests.  Such  
conclusion  is  derived  from  grandfathering  petitioners’  corporate  owners,  namely:  MMI,  SMMI  and  PLMDC.  

   
3H  A.Y.  2017-­‐2018   88  
 
 

CORPORATION  LAW  CASE  DIGESTS  –  ATTY.  DANTE  DELA  CRUZ  

Going  further  and  adding  to  the  picture,  MBMI’s  Summary  of  Significant  Accounting  Policies  statement–  –
regarding   the   "joint   venture"   agreements   that   it   entered   into   with   the   "Olympic"   and   "Alpha"   groups––
involves   SMMI,   Tesoro,   PLMDC   and   Narra.   Noticeably,   the   ownership   of   the   "layered"   corporations   boils  
down   to   MBMI,   Olympic   or   corporations   under   the   "Alpha"   group   wherein   MBMI   has   joint   venture  
agreements  with,  practically  exercising  majority  control  over  the  corporations  mentioned.  In  effect,  whether  
looking   at   the   capital   structure   or   the   underlying   relationships   between   and   among   the   corporations,  
petitioners  are  NOT  Filipino  nationals  and  must  be  considered  foreign  since  60%  or  more  of  their  capital  
stocks  or  equity  interests  are  owned  by  MBMI.  
 
 
 
66.  NARRA  V.  CONSOLIDATED  MINING  
 
 
 
67.  CAGAYAN  FISHING  DEVELOPMENT  CO.  INC.  V.  SANDIKO  
G.R.  NO.  L-­‐‑43350  DECEMBER  23,  1937  
LAUREL  J;  
 
Doctrine:  A  corporation,  until  organized,  has  no  life  and  therefore  no  faculties.  It  is,  as  it  were,  a  child  in  ventre  
sa  mere.  This  is  not  saying  that  under  no  circumstances  may  the  acts  of  promoters  of  a  corporation  be  ratified  
by   the   corporation   if   and   when   subsequently   organized.   There   are,   of   course,   exceptions   but   under   the  
peculiar   facts   and   circumstances   of   the   present   case   we   decline   to   extend   the   doctrine   of   ratification   which  
would  result  in  the  commission  of  injustice  or  fraud  to  the  candid  and  unwary.  
 
Facts:  Manuel  Tabora  is  the  registered  owner  of  four  parcels  of  land  situated  in  the  barrio  of  Linao,  town  of  
Aparri,   Province   of   Cagayan,   as   evidenced   by   transfer   certificate   of   title   No.   217   of   the   land   records   of  
Cagayan.   To   guarantee   the   payment   of   a   loan   in   the   sum   of   P8,000,   Manuel   Tabora,   on   August   14,   1929,  
executed  in  favor  of  the  PNB  three  mortgages  on  such  lands  mentioned.  These  mortgages  were  registered  
and  annotations  thereof  appear  at  the  back  of  transfer  certificate  of  title  No.  217.  
 
Subsequently,  Tabora  executed  a  public  document  entitled  "Escritura  de  Transpaso  de  Propiedad  Inmueble"  
by  virtue  of  which  the  four  parcels  of  land  owned  by  him  was  sold  to  the  plaintiff  company,  said  to  under  
process  of  incorporation,  in  consideration  of  one  peso  (P1)  subject  to  the  mortgages  in  favor  of  the  Philippine  
National  Bank  and  Severina  Buzon  and,  to  the  condition  that  the  certificate  of  title  to  said  lands  shall  not  be  
transferred   to   the   name   of   the   plaintiff   company   until   the   latter   has   fully   and   completely   paid   Tabora's  
indebtedness  to  the  Philippine  National  Bank.  
 
The   plaintiff   company   filed   its   article   incorporation   with   the   Bureau   of   Commerce   and   Industry   on   October  
22,  1930.  A  year  later,  on  October  28,  1931,  the  board  of  directors  of  said  company  adopted  a  resolution  
authorizing  its  president,  Jose  Ventura,  to  sell  the  four  parcels  of  lands  in  question  to  Teodoro  Sandiko  for  
P42,000.  Exhibits  were  thereafter  made  and  executed.  Exhibit  B  is  a  deed  of  sale  executed  before  a  notary  
public  by  the  terms  of  which  the  plaintiff  sold  ceded  and  transferred  to  the  defendant  all  its  right,  titles,  and  
interest   in   and   to   the   four   parcels   of   land   described   in   transfer   certificate   in   turn   obligated   himself   to  
shoulder  the  three  mortgages  hereinbefore  referred  to.  Exhibit  C  is  a  promisory  note  for  P25,300.  drawn  by  
the  defendant  in  favor  of  the  plaintiff,  payable  after  one  year  from  the  date  thereof.  Exhibit  D  is  a  deed  of  
mortgage  executed  before  a  notary  public  in  accordance  with  which  the  four  parcels  of  land  were  given  a  
security  for  the  payment  of  the  promissory  note,  Exhibit  C.  All  these  three  instrument  were  dated  February  
15,  1932.  

   
3H  A.Y.  2017-­‐2018   89  
 
 

CORPORATION  LAW  CASE  DIGESTS  –  ATTY.  DANTE  DELA  CRUZ  

The  defendant  having  failed  to  pay  the  sum  stated  in  the  promissory  note,  plaintiff  brought  this  action  in  the  
Court  of  First  Instance  of  Manila  praying  that  judgment  be  rendered  against  the  defendant  for  the  sum  of  
P25,300,  with  interest  at  legal  rate  from  the  date  of  the  filing  of  the  complaint,  and  the  costs  of  the  suits.    
 
Issues:  (a.)Whether  or  not  such  corporation  has  the  juridical  personality  to  enter  into  the  contract  
                         (b.)Can  promoters  of  a  corporation  act  as  agents  of  a  corporation?  
 
Ruling:   The   transfer   made   by   Tabora   to   the   Cagayan   fishing   Development   Co.,   Inc.,   plaintiff   herein,   was  
affected  on  May  31,  1930  and  the  actual  incorporation  of  said  company  was  affected  later  on  October  22,  
1930.  In  other  words,  the  transfer  was  made  almost  five  months  before  the  incorporation  of  the  company.  
Unquestionably,  a  duly  organized  corporation  has  the  power  to  purchase  and  hold  such  real  property  as  the  
purposes   for   which   such   corporation   was   formed   may   permit   and   for   this   purpose   may   enter   into   such  
contracts   as   may   be   necessary.   But   before   a   corporation   may   be   said   to   be   lawfully   organized,   many   things  
have  to  be  done.  Among  other  things,  the  law  requires  the  filing  of  articles  of  incorporation  .  Although  there  
is  a  presumption  that  all  the  requirements  of  law  have  been  complied  with,  in  the  case  before  us  it  cannot  be  
denied  that  the  plaintiff  was  not  yet  incorporated  when  it  entered  into  a  contract  of  sale.  The  contract  itself  
referred  to  the  plaintiff  as  "una  sociedad  en  vias  de  incorporacion."  It  was  not  even  a  de  facto  corporation  at  
the  time.  Not  being  in  legal  existence  then,  it  did  not  possess  juridical  capacity  to  enter  into  the  contract.  
 
Boiled  down  to  its  naked  reality,  the  contract  here  was  entered  into  not  between  Manuel  Tabora  and  a  non-­‐‑
existent   corporation   but   between   the   Manuel   Tabora   as   owner   of   the   four   parcels   of   lands   on   the   one   hand  
and  the  same  Manuel  Tabora,  his  wife  and  others,  as  mere  promoters  of  a  corporations  on  the  other  hand.  
For  reasons  that  are  self-­‐‑evident,  these  promoters  could  not  have  acted  as  agent  for  a  projected  corporation  
since   that   which   no   legal   existence   could   have   no   agent.   A   corporation,   until   organized,   has   no   life   and  
therefore  no  faculties.  It  is,  as  it  were,  a  child  in  ventre  sa  mere.  This  is  not  saying  that  under  no  circumstances  
may   the   acts   of   promoters   of   a   corporation   be   ratified   by   the   corporation   if   and   when   subsequently  
organized.  There  are,  of  course,  exceptions  but  under  the  peculiar  facts  and  circumstances  of  the  present  case  
we   decline   to   extend   the   doctrine   of   ratification   which   would   result   in   the   commission   of   injustice   or   fraud  
to  the  candid  and  unwary.  It  should  be  observed  that  Manuel  Tabora  was  the  registered  owner  of  the  four  
parcels  of  land,  which  he  succeeded  in  mortgaging  to  the  Philippine  National  Bank  so  that  he  might  have  the  
necessary  funds  with  which  to  convert  and  develop  them  into  fishery.  He  appeared  to  have  met  with  financial  
reverses.   He   formed   a   corporation   composed   of   himself,   his   wife,   and   a   few   others.   From   the   articles   of  
incorporation,  it  appears  that  out  of  the  P48,700,  amount  of  capital  stock  subscribed,  P45,000  was  subscribed  
by  Manuel  Tabora  himself  and  P500  by  his  wife,  Rufina  Q.  de  Tabora;  and  out  of  the  P43,300,  amount  paid  
on  subscription,  P42,100  is  made  to  appear  as  paid  by  Tabora  and  P200  by  his  wife.  Both  Tabora  and  His  
wife  were  directors  and  the  latter  was  treasurer  as  well.  In  fact,  to  this  day,  the  lands  remain  inscribed  in  
Tabora's   name.   The   defendant   always   regarded   Tabora   as   the   owner   of   the   lands.   He   dealt   with   Tabora  
directly.  Jose  Ventura,  president  of  the  plaintiff  corporation,  intervened  only  to  sign  the  contract,  in  behalf  of  
the  plaintiff.  Even  the  Philippine  National  Bank,  mortgagee  of  the  four  parcels  of  land,  always  treated  Tabora  
as  the  owner  of  the  same.  Two  civil  suits    were  brought  against  Tabora  in  the  Court  of  First  Instance  of  Manila  
and  in  both  cases  a  writ  of  attachment  against  the  four  parcels  of  land  was  issued.  The  Philippine  National  
Bank  threatened  to  foreclose  its  mortgages.  Tabora  approached  the  defendant  Sandiko  and  succeeded  in  the  
making  him  sign  Exhibits  B,  C,  and  D  and  in  making  him,  among  other  things,  assume  the  payment  of  Tabora's  
indebtedness  to  the  Philippine  National  Bank.  The  promisory  note  was  made  payable  to  the  plaintiff  company  
so  that  it  may  not  attached  by  Tabora's  creditors,  two  of  whom  had  obtained  writs  of  attachment  against  the  
four  parcels  of  land.  
 
If  the  plaintiff  corporation  could  not  and  did  not  acquire  the  four  parcels  of  land  here  involved,  it  follows  that  
it  did  not  possess  any  resultant  right  to  dispose  of  them  by  sale  to  the  defendant,  Teodoro  Sandiko.  

   
3H  A.Y.  2017-­‐2018   90  
 
 

CORPORATION  LAW  CASE  DIGESTS  –  ATTY.  DANTE  DELA  CRUZ  

Some   of   the   members   of   this   court   are   also   of   the   opinion   that   the   transfer   from   Manuel   Tabora   to   the  
Cagayan  Fishing  Development  Company,  Inc.,  which  transfer,  was  subject  to  a  condition  precedent    namely,  
the  payment  of  the  mortgage  debt  of  said  Tabora  to  the  Philippine  National  Bank,  and  that  this  condition  not  
having  been  complied  with  by  the  Cagayan  Fishing  Development  Company,  Inc.,  the  transfer  was  ineffective.  
However,   having   arrived   at   the   conclusion   that   the   transfer   by   Manuel   Tabora   to   the   Cagayan   Fishing  
Development  Company,  Inc.  was  null  because  at  the  time  it  was  affected  the  corporation  was  non-­‐‑existent,  
we  deem  it  unnecessary  to  discuss  this  point.  
 
 
 
68.   RIZAL   LIGHT   &   ICE   CO.,   INC.   VS.   THE   PUBLIC   SERVICE   COMMISSION   AND   MORONG   ELECTRIC   CO.,  
INC.  
G.R.  NO.  L-­‐‑21221,  SEPTEMBER  28,  1968  
ZALDIVAR,  J.  
 
DOCTRINE:  
Before  any  certificate  may  be  granted,  authorizing  the  operation  of  a  public  service,  three  requisites  must  be  
complied  with,  namely:  (1)  the  applicant  must  be  a  citizen  of  the  Philippines  or  of  the  United  States,  or  a  
corporation  or  co-­‐‑partnership,  association  or  joint-­‐‑stock  company  constituted  and  organized  under  the  laws  
of  the  Philippines,  sixty  per  centum  at  least  of  the  stock  or  paid-­‐‑up  capital  of  which  belongs  entirely  to  citizens  
of  the  Philippines  or  of  the  United  States;  (2)  the  applicant  must  be  financially  capable  of  undertaking  the  
proposed  service  and  meeting  the  responsibilities  incident  to  its  operation;  and  (3)  the  applicant  must  prove  
that  the  operation  of  the  public  service  proposed  and  the  authorization  to  do  business  will  promote  the  public  
interest  in  a  proper  and  suitable  manner.    
 
FACTS:  
Consolidated  case:  
1.   G.R.  No.  L-­‐‑20993  -­‐‑  petition  to  review  and  set  aside  the  orders  of  respondent  Public  Service  Commission  
cancelling  and  revoking  the  certificate  of  public  convenience  and  necessity  and  forfeiting  the  franchise  of  
said  petitioner.  
2.   G.  R.  No.  L-­‐‑21221  -­‐‑  petition  to  review  and  set  aside  the  decision  of  the  Commission  granting  a  certificate  
of   public   convenience   and   necessity   to   respondent   Morong   Electric   Co.,   Inc.   to   operate   an   electric   light,  
heat  and  power  service  in  the  municipality  of  Morong,  Rizal.    
 
•   Petitioner   Rizal   Light   &   Ice   Co.,   Inc.   is   a   domestic   corporation   with   business   address   at   Morong,   Rizal.   On  
August  15,  1949,  it  was  granted  by  the  Commission  a  certificate  of  public  convenience  and  necessity  for  
the  installation,  operation  and  maintenance  of  an  electric  light,  heat  and  power  service  in  the  municipality  
of  Morong,  Rizal.  
 
•   December  19,  1956  -­‐‑   PSC  required  the  petitioner  to  appear  before  it  to  show  cause  why  it  should  not  be  
penalized   for   violation   of   the   conditions   of   its   certificate   of   public   convenience   and   the   regulations   of   the  
Commission,  and  for  failure  to  comply  with  the  directives  to  raise  its  service  voltage  and  maintain  them  
within   the   limits   prescribed   in   the   Revised   Order   No.   1   of   the   Commission,   and   to   acquire   and   install   a  
kilowattmeter  to  indicate  the  load  in  kilowatts  at  any  particular  time  of  the  generating  unit.  
 
•   For  failure  of  the  petitioner  to  appear  at  the  hearing  on  February  18,  1957,  the  PSC  ordered  the  cancellation  
and   revocation   of   petitioner's   certificate   of   public   convenience   and   necessity   and   the   forfeiture   of   its  
franchise.   Petitioner   moved   for   reconsideration   of   said   order   on   the   ground   that   its   manager,   Juan   D.  
Francisco,   was   not   aware   of   said   hearing.   Respondent   municipality   opposed   the   motion   alleging   that  

   
3H  A.Y.  2017-­‐2018   91  
 
 

CORPORATION  LAW  CASE  DIGESTS  –  ATTY.  DANTE  DELA  CRUZ  

petitioner  has  not  rendered  efficient  and  satisfactory  service  and  has  not  complied  with  the  requirements  
of  the  Commission  for  the  improvement  of  its  service.  The  motion  was  set  for  hearing.  
 
•   Finding  that  the  failure  of  the  petitioner  to  appear  at  the  hearing,  which  was  the  sole  basis  of  the  revocation  
of  petitioner's  certificate,  was  really  due  to  the  illness  of  its  manager,  Juan  D.  Francisco,  the  Commission  
set  aside  its  order  of  revocation.  Respondent  municipality  moved  for  reconsideration,  but  this  was  denied.  
 
•   Meanwhile,  inspections  had  been  made  of  petitioner's  electric  plant  and  installations  by  the  engineers  of  
the  Commission.  The  inspection  on  June  21-­‐‑24,  1961  was  made  upon  the  request  of  the  petitioner  who  
manifested  during  the  hearing  on  December  15,  1960  that  improvements  have  been  made  on  its  service  
since  the  inspection  on  July  12-­‐‑13,  1960,  and  that,  on  the  basis  of  the  inspection  report  to  be  submitted,  it  
would  agree  to  the  submission  of  the  case  for  decision  without  further  hearing.  
 
•   When  the  case  was  called  for  hearing  on  July  5,  1961,  petitioner  failed  to  appear.  Respondent  municipality  
was  then  allowed  to  present  its  documentary  evidence,  and  thereafter  the  case  was  submitted  for  decision.  
 
•   On  July  7,  1961,  petitioner  filed  a  motion  to  reopen  the  case  upon  the  ground  that  it  had  not  been  furnished  
with  a  copy  of  the  report  of  the  June  21-­‐‑24,  1961  inspection  for  it  to  reply  as  previously  agreed.  Petitioner  
was  granted  a  period  of  ten  (10)  days  within  which  to  submit  its  written  reply  to  said  inspection  report,  on  
condition  that  should  it  fail  to  do  so  within  the  said  period  the  case  would  be  considered  submitted  for  
decision.  Petitioner  failed  to  file  the  reply.  Hence,  the  Commission  proceeded  to  decide  the  case.  On  July  29,  
1962  petitioner's  electric  plant  was  burned.  
 
•   The   Commission   concluded   that   the   petitioner   "cannot   render   the   efficient,   adequate   and   satisfactory  
electric  service  required  by  its  certificate  and  that  it  is  against  public  interest  to  allow  it  to  continue  its  
operation."   Accordingly,   it   ordered   the   cancellation   and   revocation   of   petitioner's   certificate   of   public  
convenience  and  the  forfeiture  of  its  franchise.  
 
•   Petitioner  filed  its  MR,  alleging  that  before  its  electric  plant  was  burned  on  July  29,  1962,  its  service  was  
greatly  improved  and  that  it  had  still  existing  investment  which  the  Commission  should  protect.  But  eight  
days  before  said  MR  was  filed,  or  on  September  10,  1962,  Morong  Electric,  having  been  granted  a  municipal  
franchise  on  May  6,  1962  by  respondent  municipality  to  install,  operate  and  maintain  an  electric  heat,  light  
and  power  service  in  said  municipality  —  approved  by  the  Provincial  Board  of  Rizal  on  August  31,  1962  —  
filed   with   the   Commission   an   application   for   a   certificate   of   public   convenience   and   necessity   for   said  
service.  Said  application  was  entitled  "Morong  Electric  Co.,  Inc.,  Applicant",  and  docketed  as  Case  No.  62-­‐‑
5143.  
 
•   Petitioner  opposed  in  writing  the  application  of  Morong  Electric,  alleging  among  other  things,  that  it  is  a  
holder  of  a  certificate  of  public  convenience  to  operate  an  electric  light,  heat  and  power  service  in  the  same  
municipality   of   Morong,   Rizal,   and   that   the   approval   of   said   application   would   not   promote   public  
convenience,   but   would   only   cause   ruinous   and   wasteful   competition.   It   asked   for   the   dismissal   of   the  
application   upon   the   ground   that   applicant   Morong   Electric   had   no   legal   personality   when   it   filed   its  
application  on  September  10,  1962,  because  its  certificate  of  incorporation  was  issued  by  the  Securities  
and   Exchange   Commission   only   on   October   17,   1962.   This   motion   to   dismiss   was   denied   by   the  
Commission  on  the  premise  that  applicant  Morong  Electric  was  a  de  facto  corporation.    
 
•   PSC  found  that  there  was  an  absence  of  electric  service  in  the  municipality  of  Morong  and  that  applicant  
Morong   Electric,   a   Filipino-­‐‑owned   corporation   duly   organized   and   existing   under   the   laws   of   the  
Philippines,  has  the  financial  capacity  to  maintain  said  service.  PSC  approved  the  application  of  Morong  

   
3H  A.Y.  2017-­‐2018   92  
 
 

CORPORATION  LAW  CASE  DIGESTS  –  ATTY.  DANTE  DELA  CRUZ  

Electric  and  ordered  the  issuance  in  its  favor  of  the  corresponding  certificate  of  public  convenience  and  
necessity.  
 
•   In   Case   No.   39715,   petitioner   contends:   (1)   that   the   Commission   acted   without   or   in   excess   of   its  
jurisdiction  when  it  delegated  the  hearing  of  the  case  and  the  reception  of  evidence  to  Mr.  Pedro  S.  Talavera  
who  is  not  allowed  by  law  to  hear  the  same;  (2)  that  the  cancellation  of  petitioner's  certificate  of  public  
convenience  was  unwarranted  because  no  sufficient  evidence  was  adduced  against  the  petitioner  and  that  
petitioner  was  not  able  to  present  evidence  in  its  defense;  (3)  that  the  Commission  failed  to  give  protection  
to   petitioner's   investment;   and   (4)   that   the   Commission   erred   in   imposing   the   extreme   penalty   of  
revocation  of  the  certificate.  
 
•   In  Case  No.  62-­‐‑5143,  petitioner  contends:  (1)  that  the  Commission  erred  in  denying  petitioner's  motion  to  
dismiss  and  proceeding  with  the  hearing  of  the  application  of  the  Morong  Electric;  (2)  that  the  Commission  
erred  in  granting  Morong  Electric  a  certificate  of  public  convenience  and  necessity  since  it  is  not  financially  
capable  to  render  the  service;  (3)  that  the  Commission  erred  when  it  made  findings  of  facts  that  are  not  
supported   by   the   evidence   adduced   by   the   parties   at   the   trial;   and   (4)   that   the   Commission   erred   when   it  
did  not  give  to  petitioner  protection  to  its  investment  —  a  reiteration  of  the  third  assignment  of  error  in  
the  other  case.  
 
ISSUE:  
In  G.R.  No.  L-­‐‑20993:  
1.  W/N  the  protection  of  investment  rule  applies.  -­‐‑  NO    
2.  W/N  the  Commission  erred  in  imposing  the  extreme  penalty  of  revocation  of  the  certificate.  -­‐‑  NO  
 
In  G.  R.  No.  L-­‐‑21221:  
W/N  the  franchise  granted  to  Morong  Electric  is  valid,  and  whether  it  has  corporate  personality    to  accept  
the  same.  -­‐‑  YES  
 
HELD:  
 
In  G.R.  No.  L-­‐‑20993:  
1.   The   "protection-­‐‑of-­‐‑investment   rule"   enunciated   by   this   Court   in   Batangas   Transportation   Co.   vs.   Orlanes  
in  this  wise:  The  Government  having  taken  over  the  control  and  supervision  of  all  public  utilities,  so  long  as  
an  operator  under  a  prior  license  complies  with  the  terms  and  conditions  of  his  license  and  reasonable  rules  
and   regulations   for   its   operation   and   meets   the   reasonable   demands   of   the   public,   it   is   the   duty   of   the  
Commission  to  protect  rather  than  to  destroy  his  investment  by  the  granting  of  the  second  license  to  another  
person  for  the  same  thing  over  the  same  route  of  travel.  The  granting  of  such  a  license  does  not  serve  its  
convenience  or  promote  the  interests  of  the  public.  
 
This  rule,  however,  is  not  absolute,  for  nobody  has  exclusive  right  to  secure  a  franchise  or  a  certificate  of  
public  convenience.  Where,  as  in  the  present  case,  it  has  been  shown  by  ample  evidence  that  the  petitioner,  
despite  ample  time  and  opportunity  given  to  it  by  the  Commission,  had  failed  to  render  adequate,  sufficient  
and   satisfactory   service   and   had   violated   the   important   conditions   of   its   certificate   as   well   as   the   directives  
and  the  rules  and  regulations  of  the  Commission,  the  rule  cannot  apply.  
 
2.  A  grant  of  a  certificate  of  public  convenience  confers  no  property  rights  but  is  a  mere  license  or  privilege,  
and  such  privilege  is  forfeited  when  the  grantee  fails  to  comply  with  his  commitments  behind  which  lies  the  
paramount   interest   of   the   public,   for   public   necessity   cannot   be   made   to   wait,   nor   sacrificed   for   private  
convenience.  
 

   
3H  A.Y.  2017-­‐2018   93  
 
 

CORPORATION  LAW  CASE  DIGESTS  –  ATTY.  DANTE  DELA  CRUZ  

The  Public  Service  Commission,...  has  the  power  to  specify  and  define  the  terms  and  conditions  upon  which  
the  public  utility  shall  be  operated,  and  to  make  reasonable  rules  and  regulations  for  its  operation  and  the  
compensation  which  the  utility  shall  receive  for  its  services  to  the  public,  and  for  any  failure  to  comply  with  
such   rules   and   regulations   or   the   violation   of   any   of   the   terms   and   conditions   for   which   the   license   was  
granted,  the  Commission  has  ample  power  to  enforce  the  provisions  of  the  license  or  even  to  revoke  it,  for  
any  failure  or  neglect  to  comply  with  any  of  its  terms  and  provisions.  
 
In  the  instant  case,  as  shown  by  the  evidence,  the  contumacious  refusal  of  the  petitioner  since  1954  to  comply  
with   the   directives,   rules   and   regulations   of   the   Commission,   its   violation   of   the   conditions   of   its   certificate  
and   its   incapability   to   comply   with   its   commitment   as   shown   by   its   inadequate   service,   were   the  
circumstances  that  warranted  the  action  of  the  Commission  in  not  merely  imposing  a  fine  but  in  revoking  
altogether   petitioner's   certificate.   To   allow   petitioner   to   continue   its   operation   would   be   to   sacrifice   public  
interest  and  convenience  in  favor  of  private  interest.  
 
In  G.  R.  No.  L-­‐‑21221:  
 
The   Commission   found   that   Morong   Electric   is   a   corporation   duly   organized   and   existing   under   the   laws   of  
the  Philippines,  the  stockholders  of  which  are  Filipino  citizens,  that  it  is  financially  capable  of  operating  an  
electric  light,  heat  and  power  service,  and  that  at  the  time  the  decision  was  rendered  there  was  absence  of  
electric   service   in   Morong,   Rizal.   While   the   petitioner   does   not   dispute   the   need   of   an   electric   service   in  
Morong,  Rizal,  it  claims,  in  effect,  that  Morong  Electric  should  not  have  been  granted  the  certificate  of  public  
convenience  and  necessity  because  (1)  it  did  not  have  a  corporate  personality  at  the  time  it  was  granted  a  
franchise  and  when  it  applied  for  said  certificate;  (2)  it  is  not  financially  capable  of  undertaking  an  electric  
service,  and  (3)  petitioner  was  rendering  efficient  service  before  its  electric  plant  was  burned,  and  therefore,  
being  a  prior  operator  its  investment  should  be  protected  and  no  new  party  should  be  granted  a  franchise  
and  certificate  of  public  convenience  and  necessity  to  operate  an  electric  service  in  the  same  locality.  
 
The  juridical  personality  and  legal  existence  of  Morong  Electric  began  only  on  October  17,  1962  when  its  
certificate  of  incorporation  was  issued  by  the  SEC.  Before  that  date,  or  pending  the  issuance  of  said  certificate  
of  incorporation,  the  incorporators  cannot  be  considered  as  de  facto  corporation.  25  But  the  fact  that  Morong  
Electric  had  no  corporate  existence  on  the  day  the  franchise  was  granted  in  its  name  does  not  render  the  
franchise  invalid,  because  later  Morong  Electric  obtained  its  certificate  of  incorporation  and  then  accepted  
the  franchise  in  accordance  with  the  terms  and  conditions  thereof.  
 
The  incorporation  of  Morong  Electric  on  October  17,  1962  and  its  acceptance  of  the  franchise  as  shown  by  
its  action  in  prosecuting  the  application  filed  with  the  Commission  for  the  approval  of  said  franchise,  not  only  
perfected  a  contract  between  the  respondent  municipality  and  Morong  Electric  but  also  cured  the  deficiency  
pointed   out   by   the   petitioner   in   the   application   of   Morong   EIectric.   Thus,   the   Commission   did   not   err   in  
denying  petitioner's  motion  to  dismiss  said  application  and  in  proceeding  to  hear  the  same.  The  efficacy  of  
the  franchise,  however,  arose  only  upon  its  approval  by  the  Commission  on  March  13,  1963.  The  reason  is  
that  —  
 
Under   Act   No.   667,   as   amended   by   Act   No.   1022,   a   municipal   council   has   the   power   to   grant   electric  
franchises,  subject  to  the  approval  of  the  provincial  board  and  the  President.  However,  under  Section  16(b)  
of   Commonwealth   Act   No.   146,   as   amended,   the   Public   Service   Commission   is   empowered   "to   approve,  
subject   to   constitutional   limitations   any   franchise   or   privilege   granted   under   the   provisions   of   Act   No.   667,  
as  amended  by  Act  No.  1022,  by  any  political  subdivision  of  the  Philippines  when,  in  the  judgment  of  the  
Commission,  such  franchise  or  privilege  will  properly  conserve  the  public  interests  and  the  Commission  shall  
in  so  approving  impose  such  conditions  as  to  construction,  equipment,  maintenance,  service,  or  operation  as  
the  public  interests  and  convenience  may  reasonably  require,  and  to  issue  certificates  of  public  convenience  

   
3H  A.Y.  2017-­‐2018   94  
 
 

CORPORATION  LAW  CASE  DIGESTS  –  ATTY.  DANTE  DELA  CRUZ  

and   necessity   when   such   is   required   or   provided   by   any   law   or   franchise."   Thus,   the   efficacy   of   a   municipal  
electric  franchise  arises,  therefore,  only  after  the  approval  of  the  Public  Service  Commission.  (Almendras  vs.  
Ramos,  90  Phil.  231)  
 
The   conclusion   herein   reached   regarding   the   validity   of   the   franchise   granted   to   Morong   Electric   is   not  
incompatible  with  the  holding  of  this  Court  in  Cagayan  Fishing  Development  Co.,  Inc.  vs.  Teodoro  Sandiko  
upon   which   the   petitioner   leans   heavily   in   support   of   its   position.   In   said   case   this   Court   held   that   a  
corporation  should  have  a  full  and  complete  organization  and  existence  as  an  entity  before  it  can  enter  into  
any  kind  of  a  contract  or  transact  any  business.  It  should  be  pointed  out,  however,  that  this  Court  did  not  say  
in   that   case   that   the   rule   is   absolute   or   that   under   no   circumstances   may   the   acts   of   promoters   of   a  
corporation  be  ratified  or  accepted  by  the  corporation  if  and  when  subsequently  organized.  Of  course,  there  
are   exceptions.   It   will   be   noted   that   American   courts   generally   hold   that   a   contract   made   by   the   promoters  
of  a  corporation  on  its  behalf  may  be  adopted,  accepted  or  ratified  by  the  corporation  when  organized.  
 
WHEREFORE,   the   two   decisions   of   the   Public   Service   Commission,   appealed   from,   should   be,   as   they   are  
hereby  affirmed,  with  costs  in  the  two  cases  against  petitioner  Rizal  Light  &  Ice  Co.,  Inc.  
 
 
 
69.  REPUBLIC  OF  THE  PHILIPPINES  VS.  ACOJE  MINING  COMPANY,  INC.  
G.R.  NO.  L-­‐‑18062                          FEBRUARY  28,  1963  
BAUTISTA  ANGELO,  J.:  
 
DOCTRINE:  
                          While   as   a   rule   an   ultra   vires   act   is   one   committed   outside   the   object   for   which   a   corporation   is  
created  as  defined  by  the  law  of  its  organization  and  therefore  beyond  the  powers  conferred  upon  it  by  law  
(19  C.J.S.,  Section  965,  p.  419),  there  are  however  certain  corporate  acts  that  may  be  performed  outside  of  
the   scope   of   the   powers   expressly   conferred   if   they   are   necessary   to   promote   the   interest   or   welfare   of   the  
corporation.  
 
FACTS:  
  Acoje  Mining  Company,  Inc.  wrote  the  Director  of  Posts  requesting  the  opening  of  a  post,  telegraph  
and  money  order  offices  at  its  mining  camp  at  Sta.  Cruz,  Zambales,  to  service  its  employees  and  their  families  
that  were  living  in  said  camp.  The  Director  of  Posts  wrote  a  letter  to  the  company  stating  among  other  things  
that  "In  cases  where  a  post  office  will  be  opened  under  circumstances  similar  to  the  present,  it  is  the  policy  
of  this  office  to  have  the  company  assume  direct  responsibility  for  whatever  pecuniary  loss  may  be  suffered  
by   the   Bureau   of   Posts   by   reason   of   any   act   of   dishonesty,   carelessness   or   negligence   on   the   part   of   the  
employee   of   the   company   who   is   assigned   to   take   charge   of   the   post   office.   The   company   informed   the  
Director  of  Posts  of  the  passage  by  its  board  of  directors  of  a  resolution  of  the  following  tenor:  "That  the  
requirement   of   the   Bureau   of   Posts   that   the   Company   should   accept   full   responsibility   for   all   cash   received  
by  the  Postmaster  be  complied  with,  and  that  a  copy  of  this  resolution  be  forwarded  to  the  Bureau  of  Posts."  
  The  post  office  branch   was   opened   at   the   camp   on   October   13,   1949   with   one   Hilario   M.   Sanchez   as  
postmaster.  He  is  an  employee  of  the  company.  On  May  11,  1954,  the  postmaster  went  on  a  three-­‐‑day  leave  
but   never   returned.   The   company   immediately   informed   the   officials   of   the   Manila   Post   Office   and   the  
provincial  auditor  of  Zambales  of  Sanchez'  disappearance  with  the  result  that  the  accounts  of  the  postmaster  
were  checked  and  a  shortage  was  found  in  the  amount  of  P13,867.24.  
  The   government   commenced   the   present   action   on   September   10,   1954   before   the   Court   of   First  
Instance   of   Manila   seeking   to   recover   the   amount   of   Pl3,867.24.   The   company   in   its   answer   denied   liability  
for  said  amount  contending  that  the  resolution  of  the  board  of  directors  wherein  it  assumed  responsibility  
for  the  act  of  the  postmaster  is  ultra  vires,  and  in  any  event  its  liability  under  said  resolution  is  only  that  of  a  

   
3H  A.Y.  2017-­‐2018   95  
 
 

CORPORATION  LAW  CASE  DIGESTS  –  ATTY.  DANTE  DELA  CRUZ  

guarantor   who   answers   only   after   the   exhaustion   of   the   properties   of   the   principal,   aside   from   the   fact   that  
the  loss  claimed  by  the  plaintiff  is  not  supported  by  the  office  record.  
 
ISSUE:  
  Whether  the  resolution  adopted  by  the  company  is  ultra  vires  in  the  sense  that  it  has  no  authority  to  
act  on  a  matter  which  may  render  the  company  liable  as  a  guarantor.  
 
HELD:  
  No.   In   the   first   place,   it   should   be   noted   that   the   opening   of   a   post   office   branch   at   the   mining   camp  
of  appellant  corporation  was  undertaken  because  of  a  request  submitted  by  it  to  promote  the  convenience  
and   benefit   of   its   employees.   The   idea   did   not   come   from   the   government,   and   the   Director   of   Posts   was  
prevailed   upon   to   agree   to   the   request   only   after   studying   the   necessity   for   its   establishment   and   after  
imposing   upon   the   company   certain   requirements   intended   to   safeguard   and   protect   the   interest   of   the  
government.  
  The  claim  that  the  resolution  adopted  by  the  board  of  directors  of  appellant  company  is  an  ultra  vires  
act   cannot   also   be   entertained   it   appearing   that   the   same   covers   a   subject   which   concerns   the   benefit,  
convenience   and   welfare   of   its   employees   and   their   families.   While   as   a   rule   an   ultra   vires   act   is   one  
committed  outside  the  object  for  which  a  corporation  is  created  as  defined  by  the  law  of  its  organization  and  
therefore   beyond   the   powers   conferred   upon   it   by   law   (19   C.J.S.,   Section   965,   p.   419),   there   are   however  
certain  corporate  acts  that  may  be  performed  outside  of  the  scope  of  the  powers  expressly  conferred  if  they  
are  necessary  to  promote  the  interest  or  welfare  of  the  corporation.  Thus,  it  has  been  held  that  "although  not  
expressly   authorized   to   do   so   a   corporation   may   become   a   surety   where   the   particular   transaction   is  
reasonably   necessary   or   proper   to   the   conduct   of   its   business,"   and   here   it   is   undisputed   that   the  
establishment  of  the  local  post  office  is  a  reasonable  and  proper  adjunct  to  the  conduct  of  the  business  of  
appellant  company.  
 
 
 
70.  IRINEO  G.  CARLOS  V.  MINDORO  SUGAR  CO.,  ET  AL.  
G.R.  NO.  L-­‐‑36207  /  OCTOBER  26,  1932  
IMPERIAL,  J.  
 
DOCTRINE:  
 
It  is  not,  however,  ultra  vires  for  a  corporation  to  enter  into  contracts  of  guaranty  or  suretyship  where  it  does  
so  in  the  legitimate  furtherance  of  its  purposes  and  business.  And  it  is  well  settled  that  where  a  corporation  
acquires   commercial   paper   or   bonds   in   the   legitimate   transaction   of   its   business   it   may   sell   them,   and   in  
furtherance  of  such  a  sale  it  may,  in  order  to  make  them  the  more  readily  marketable,  indorse  or  guarantee  
their  payment.    
 
FACTS:    
 
The   Philippine   Trust   Company   (PTC)   is   also   a   domestic   corporation   engaged   in   the   trust   business.   In   1917,  
the  board  of  directors  of  the  PTC  adopted  a  resolution  authorizing  its  president  to  purchase  at  par  and  in  the  
name  and  for  the  use  of  the  corporation  all  or  such  part  as  he  may  deem  expedient,  of  the  bonds  in  the  value  
of  P3,000,000  that  the  Mindoro  Sugar  Company  (MSC)  was  about  to  issue,  and  to  resell  them,  with  or  without  
the   guarantee   of   said   trust   corporation,   at   a   price   not   less   than   par,   and   to   guarantee   to   the   Philippine  
National  Bank  the  payment  of  the  indebtedness  to  said  bank  by  the  Mindoro  Sugar  Company  or  Charles  J.  
Welch  and  Horace  Havemeyer,  up  to  P2,000,000.    
 

   
3H  A.Y.  2017-­‐2018   96  
 
 

CORPORATION  LAW  CASE  DIGESTS  –  ATTY.  DANTE  DELA  CRUZ  

In   pursuance   of   this   resolution,   MSC   executed   in   favor   of   the   PTC   a   Deed   of   Trust   transferring   all   of   its  
property   to   it   in   consideration   of   the   bonds   it   had   issued   to   the   value   of   P3,000,000,   the   value   of   each   bond  
being  $1,000,  with  interest  at  8%  per  annum.  The  PTC  sold  thirteen  bonds,  Nos.  1219  to  1231,  to  Ramon  Diaz  
for  P27,300,  at  a  net  profit  of  P100  per  bond.  
 
The  PTC  paid  the  appellant  the  stipulated  interest  from  the  date  of  their  maturity  until  the  July  1,  1928,  when  
it   stopped   payments   claiming   that   it   did   not   deem   itself   bound   to   pay   such   interest   or   to   redeem   the  
obligation  because  the  guarantee  given  for  the  bonds  was  illegal  and  void.  The  plaintiff  brought  this  action  
to   recover   from   the   defendants   the   value   of   four   bonds,   Nos.   1219,   1220,   1221,   and   1222,   with   due   and  
unpaid  interest.  
 
ISSUE:  
 
WON  the  PTC  acquired  the  four  bonds  in  question.  
 
WON   PTC   bound   itself   legally   and   acted   within   its   corporate   powers   in   guaranteeing   them.  
 
HELD:  
 
•   The  SC  ruled  in  the  affirmative.  
 
In  adopting  this  conclusion  we  have  relied  principally  upon  the  following  facts  and  circumstances:    
 
Firstly,   that   the   Philippine   Trust   Company,   although   secondarily   engaged   in   banking,   was   primarily  
organized  as  a  trust  corporation  with  full  power  to  acquire  personal  property  such  as  the  bonds  in  
question   according   to   both   the   Corporation   Law   and   its   duly   registered   by-­‐‑laws   and   articles   of  
incorporation;    
 
Secondly,   that   being   thus   authorized   to   acquire   the   bonds,   it   was   given   implied   power   to   guarantee  
them   in   order   to   place   them   upon   the   market   under   better,   more   advantageous   conditions,   and  
thereby  secure  the  profit  derived  from  their  sale;    
 
Thirdly,  that  although  it  does  not  clearly  appear  in  the  deed  of  trust  that  the  Mindoro  Sugar  Company  
transferred   the   bonds   therein   referred   to,   to   the   Philippine   Trust   Company,   nevertheless,   in   the  
resolution  of  the  board  of  directors,  the  president  of  the  Philippine  Trust  Company  was  expressly  
authorized  to  purchase  all  or  some  of  the  bonds  and  to  guarantee  them;  whence  it  may  be  inferred  
that  subsequent  purchasers  of  the  bonds  in  the  market  relied  upon  the  belief  that  they  were  acquiring  
securities  of  the  Philippine  Trust  Company,  guaranteed  by  this  corporation;  
 
Fourthly,  that  as  soon  as  P3,000,000  worth  of  bonds  was  issued,  and  by  the  deed  of  trust  the  Mindoro,  
Sugar   Company   transferred   all   its   real   property   to   the   Philippine   Trust   Company,   the   cause   or  
consideration  of  the  transfer  being,  (1)  the  guarantee  given  by  the  purchaser  to  the  bonds,  and  (2)  its  
having   likewise   guaranteed   its   obligations   and   those   of   Welch   and   Havemeyer   in   favor   of   the  
Philippine  National  Bank  up  to  the  amount  of  P2,000,000;  
 
Fifthly,  that  in  transferring  its  real  property  as  aforesaid  the  Mindoro  Sugar  Company  was  reduced  
to   a   real   state   of   bankruptcy,   as   the   parties   specifically   agreed   during   the   hearing   of   the   case,   to   the  
point  of  having  become  a  nominal  corporation  without  any  assets  whatsoever;  
 

   
3H  A.Y.  2017-­‐2018   97  
 
 

CORPORATION  LAW  CASE  DIGESTS  –  ATTY.  DANTE  DELA  CRUZ  

Sixthly,  that  such  operation  or  transaction  cannot  mean  anything  other  than  that  the  real  intention  
of  the  parties  was  that  the  Philippine  Trust  Company  acquired  the  bonds  issued  and  at  the  same  time  
guaranteed  the  payment  of  their  par  value  with  interest,  because  otherwise  the  transaction  would  be  
fraudulent,   inasmuch   as   nobody   would   be   answerable   to   the   bond-­‐‑holders   for   their   value   and  
interest;  
 
Seventhly,   that   the   Philippine   Trust   Company   had   been   paying   the   appellant   the   interest   accrued  
upon  the  four  bonds  from  the  date  of  their  issuance  until  July  1,  1928,  such  payment  of  interest  being  
another  proof  that  said  corporation  had  really  become  the  owner  of  the  aforesaid  bonds;  and,  
 
Eightly,   that   the   Philippine   Trust   Company   has   not   adduced   any   evidence   to   show   any   other  
conclusions.  
 
•   There  are  other  considerations  leading  to  the  same  result  even  in  the  supposition  that  the  Philippine  
Trust   Company   did   not   acquire   the   bonds   in   question,   but   only   guaranteed   them.   In   such   a   case   the  
guarantee  of  these  bonds  would  at  any  rate,  be  valid  and  the  said  corporation  would  be  bound  to  pay  
the  appellant  their  value  with  the  accrued  interest  in  view  of  the  fact  that  they  become  due  on  account  
of   the   lapse   of   sixty   (60)   days,   without   the   accrued   interest   due   having   been   paid;   and   the   reason   is  
that  it  is  estopped  from  denying  the  validity  of  its  guarantee.  
 
•   It  has  been  intimated  according  to  Section  121  of  the  Corporation  Law,  the  Philippine  Trust  Company,  
as   a   banking   institution,   could   not   guarantee   the   bonds   to   the   value   of   P3,000,000   because   this  
amount  far  exceeds  its  capital  of  P1,000,000  of  which  only  one-­‐‑half  has  been  subscribed  and  paid.    
 
This  difficulty  is  easily  obviated  by  bearing  in  mind  that,  as  we  stated  above,  the  banking  operations  
are  not  the  primary  aim  of  said  corporation,  which  is  engaged  essentially  in  the  trust  business,  and  
that  the  prohibition  of  the  law  is  not  applicable  to  the  Philippine  Trust  Company,  for  the  evidence  
shows  that  Mindoro  Sugar  Company  transferred  all  its  real  property,  with  the  improvements,  to  it,  
and   the   value   of   both,   which   surely   could   not   be   less   than   the   value   of   the   obligation   guaranteed,  
became  a  part  of  its  capital  and  assets;  in  other  words,  with  the  value  of  the  real  property  transferred  
to   it,   the   Philippine   Trust   Company   had   enough   capital   and   assets   to   meet   the   amount   of   the   bonds  
guaranteed  with  interest  thereon.  
 
 
 
71.    NO  CASE  
 
 
 
72.  NATIONAL  POWER  CORPORATION  VS.  HONORABLE  ABRAHAM  P.  VERA  
G.R.  NO.  83558,  FEBRUARY  27,  1989  
CORTES,  J  
 
DOCTRINE:    
For  if  that  act  is  one  which  is  lawful  in  itself  and  not  otherwise  prohibited,  and  is  done  for  the  purpose  of  
serving  corporate  ends,  and  reasonably  contributes  to  the  promotion  of  those  ends  in  a  substantial  and  not  
in  a  remote  and  fanciful  sense,  it  may  be  fairly  considered  within  the  corporation's  charter  powers  
FACTS:    
 

   
3H  A.Y.  2017-­‐2018   98  
 
 

CORPORATION  LAW  CASE  DIGESTS  –  ATTY.  DANTE  DELA  CRUZ  

The  case  arose  from  a  complaint  for  prohibition  and  mandamus  with  damages  filed  by  Sea  Lion  International  
Port  Terminal  Services,  Inc  against  National  Power  Corporation  (NPC)  and  Philippine  Ports  Authority  (PPA),  
wherein  private  respondent  alleged  that  NPC  had  acted  in  bad  faith  and  with  grave  abuse  of  discretion  in  not  
renewing  its  Contract  for  Stevedoring  Services  for  Coal-­‐‑Handling  Operations  at  NPC's  plant,  and  in  taking  
over  its  stevedoring  services.  
 
Judge  Vera  issued  a  restraining  order  against  NPC  enjoining  the  latter  from  undertaking  stevedoring  services  
at  its  pier.  And  in  the  order  denying  NPC's  motion  and  issuing  a  writ  of  preliminary  injunction,  after  finding  
that  NPC  was  not  empowered  by  its  Charter,  Republic  Act  No.  6395,  as  amended,  to  engage  in  stevedoring  
and  arrastre  services.  
 
ISSUE:  
WON  NPC  empowered  to  undertake  stevedoring  services  in  its  pier.  
 
HELD:    
Yes.  Presidential  Decree  No.  1818  explicitly  provides:  
 
SECTION  1.   No   court   in   the   Philippines   shall   have   jurisdiction   to   issue   any   restraining   order,  
preliminary   injunction,   or   preliminary   mandatory   injunction   in   any   case,   dispute,   or   controversy  
involving  an  infrastructure  project,  or  a  mining,  fishery,  forest  or  other  natural  resource  development  
project  of  the  government,  or  any  public  utility  operated  by  the  government,  including  among  others  
public  utilities  for  the  transport  of  the  goods  or  commodities,  stevedoring  and  arrastre  contracts,  to  
prohibit  any  person  or  persons,  entity  or  government  official  from  proceeding  with,  or  continuing  the  
execution  or  implementation  of  any  such  project,  or  the  operation  of  such  public  utility,  or  pursuing  
any  lawful  activity  necessary  for  such  execution,  implementation  or  operation.  
 
Undeniably,   NPC   is   a   public   utility,   created   under   special   legislation   engaged   in   the   generation   and  
distribution  of  electric  power  and  energy.  It,  therefore,  enjoys  the  protective  mantle  of  the  above  decree.  
 
Moreover,  respondent  judge's  finding  that  NPC  is  not  empowered  by  its  Charter  to  undertake  stevedoring  
services  in  its  pier  is  erroneous.  
 
To   carry   out   the   national   policy   of   total   electrification   of   the   country,   specifically   the   development   of  
hydroelectric   generation   of   power   and   the   production   of   electricity   from   nuclear,   geothermal   and   other  
sources  to  meet  the  needs  of  industrial  development  and  dispersal  and  the  needs  of  rural  electrification  [Secs.  
1  and  2,  Rep.  Act  No.  6395,  as  amended],  the  NPC  was  created  and  empowered  not  only  to  construct,  operate  
and  maintain  power  plants,  reservoirs,  transmission  lines,  and  other  works,  but  also:  
 
xxx   xxx   xxx  
 
...   To   exercise   such   powers   and   do   such   things   as   may   be   reasonably   necessary   to   carry   out   the  
business  and  purposes  for  which  it  was  organized,  or  which,  from  time  to  time,  may  be  declared  by  
the  Board  to  be  necessary,  useful,  incidental  or  auxiliary  to  accomplish  said  purpose,  .  .  .  [Sec.  3  (1)  of  
Rep.  Act  No.  6395,  as  amended.]  
 
In  determining  whether  or  not  an  NPC  act  falls  within  the  purview  of  the  above  provision,  the  Court  must  
decide  whether  or  not  a  logical  and  necessary  relation  exists  between  the  act  questioned  and  the  corporate  
purpose   expressed   in   the   NPC   charter.   For   if   that   act   is   one   which   is   lawful   in   itself   and   not   otherwise  
prohibited,   and   is   done   for   the   purpose   of   serving   corporate   ends,   and   reasonably   contributes   to   the  
promotion   of   those   ends   in   a   substantial   and   not   in   a   remote   and   fanciful   sense,   it   may   be   fairly   considered  

   
3H  A.Y.  2017-­‐2018   99  
 
 

CORPORATION  LAW  CASE  DIGESTS  –  ATTY.  DANTE  DELA  CRUZ  

within   the   corporation's   charter   powers   [Montelibano   v.   Bacolod-­‐‑Murcia   Milling   Co.,   Inc.,   G.R.   No.   L-­‐‑15092,  
May  18,  1962,  5  SCRA  36.]  
 
This  Court  is,  guided  by  jurisprudence  in  the  application  of  the  above  standard.  In  the  1963  case  of  Republic  
of  the  Philippines  v.  Acoje  Mining  Company,  Inc.  [G.R.  No.  L-­‐‑18062,  February  28,  1963,  7  SCRA  3611  the  Court  
affirmed  the  rule  that  a  corporation  is  not  restricted  to  the  exercise  of  powers  expressly  conferred  upon  it  by  
its  charter,  but  has  the  power  to  do  what  is  reasonably  necessary  or  proper  to  promote  the  interest  or  welfare  
of   the   corporation.   Thus,   the   Court,   finding   that   a   "post   office   is   a   vital   improvement   in   the   living   condition  
of  its  employees  and  laborers  who  came  to  settle  in  its  mining  camp  which  is  far  removed  from  the  postal  
facilities  or  means  of  communication  accorded  to  people  living  in  a  city  or  municipality"  [Id.,  at  P.  365],  held  
that  respondent  mining  corporation  was  empowered  to  operate  and  maintain  postal  facilities  servicing  its  
employees  and  their  families  at  its  mining  camp  in  Sta.  Cruz,  Zambales  despite  absence  of  a  provision  in  the  
company's  charter  authorizing  the  former  to  do  so.  
 
The  Court  in  the  case  of  Teresa  Electric  &  Power  Co.,  Inc.  v.  Public  Service  Commission  and  Filipinos  Cement  
Corporation   [G.R.   No.   L-­‐‑21804,   September   25,   1967,   21   SCRA   198]   in   interpreting   a   provision   found   in  
respondent  corporations  articles  of  incorporation  authorizing  the  corporation  to  perform  any  and  all  acts  
connected  with  the  business  of  manufacturing  portland  cement  or  arising  therefrom  or  incidental  thereto,  
concluded  that  the  corporation  must  be  deemed  authorized  to  operate  and  maintain  an  electric  power  plant  
exclusively  for  its  own  use  in  connection  with  the  operation  of  its  cement  factory  in  a  remote  barrio.  The  
Court  found  that  the  operation  of  such  plant  was  necessarily  connected  with  the  business  of  manufacturing  
cement.  
 
In  the  instant  case,  it  is  an  undisputed  fact  that  the  pier  located  at  Calaca,  Batangas,  which  is  owned  by  NPC,  
receives   the   various   shipments   of   coal   which   is   used   exclusively   to   fuel   the   Batangas   Coal-­‐‑Fired   Thermal  
Power   Plant   of   the   NPC   for   the   generation   of   electric   power.   The   stevedoring   services   which   involve   the  
unloading   of   the   coal   shipments   into   the   NPC   pier   for   its   eventual   conveyance   to   the   power   plant   are  
incidental  and  indispensable  to  the  operation  of  the  plant  The  Court  holds  that  NPC  is  empowered  under  its  
Charter  to  undertake  such  services,  it  being  reasonably  necessary  to  the  operation  and  maintenance  of  the  
power  plant.  
 
 
 
73.   MADRIGAL   &   COMPANY,   INC.,   PETITIONER,   VS.   HON.   RONALDO   B.   ZAMORA,   PRESIDENTIAL  
ASSISTANT  FOR  LEGAL  AFFAIRS,  THE  HON.  SECRETARY  OF  LABOR,  AND  MADRIGAL  CENTRAL  OFFICE  
EMPLOYEES  UNION,  RESPONDENTS.  
G.R.  NO.  L-­‐‑48237.  JUNE  30,  1987.  
 
MADRIGAL  &  COMPANY,  INC.,  PETITIONER,  VS.  HON.  MINISTER  OF  LABOR  AND  MADRIGAL  CENTRAL  
OFFICE  EMPLOYEES  UNION,  RESPONDENTS.  
G.R.  NO.  L-­‐‑49023.  JUNE  30,  1987.  
SARMIENTO,  J.  
 
Doctrine:  It  is  incorrect  to  say  that  such  profits  —  in  the  form  of  dividends  —  are  beyond  the  reach  of  the  
petitioner's  creditors  since  the  petitioner  had  received  them  as  compensation  for  its  management  services  
in  favor  of  the  companies  it  managed  as  a  shareholder  thereof.  As  such  shareholder,  the  dividends  paid  to  it  
were   its   own   money,   which   may   then   be   available   for   wage   increments.   It   is   not   a   case   of   a   corporation  
distributing   dividends   in   favor   of   its   stockholders,   in   which   case,   such   dividends   would   be   the   absolute  
property  of  the  stockholders  and  hence,  out  of  reach  by  creditors  of  the  corporation.  
 

   
3H  A.Y.  2017-­‐2018   100  
 
 

CORPORATION  LAW  CASE  DIGESTS  –  ATTY.  DANTE  DELA  CRUZ  

Facts:  Petitioner  was  engaged,  among  several  other  corporate  objectives,  in  the  management  of  Rizal  Cement  
Co.,  Inc.  Admittedly,  the  petitioner  and  Rizal  Cement  Co.,  Inc.  are  sister  companies.  Both  are  owned  by  the  
same   or   practically   the   same   stockholders.   Respondent,   the   Madrigal   Central   Office   Employees   Union  
(MCOE),   sought   for   the   renewal   of   its   CBA   with   the   petitioner,   which   was   due   to   expire.   Specifically,   it  
proposed   a   wage   increase   of   P200.00   a   month,   an   allowance   of   P100.00   a   month,   and   other   economic  
benefits.  The  petitioner,  however,  requested  for  a  deferment  in  the  negotiations.  
 
On   1974,   by   an   alleged   resolution   of   its   stockholders,   the   petitioner   reduced   its   capital   stock   from   765,000  
shares  to  267,366  shares.  This  was  effected  through  the  distribution  of  the  marketable  securities  owned  by  
the  petitioner  to  its  stockholders  in  exchange  for  their  shares  in  an  equivalent  amount  in  the  corporation.  
 
On   1975,   by   yet   another   alleged   stockholders'   action,   the   petitioner   reduced   its   authorized   capitalization  
from  267,366  shares  to  110,085  shares,  again,  through  the  same  scheme.  
 
After  the  petitioner's  failure  to  sit  down  with  the  respondent  union,  the  latter,  commenced  Case  No.  LR-­‐‑5415  
(C-­‐‑5415)  with  the  NLRC  on  a  complaint  for  unfair  labor  practice.  In  due  time,  the  petitioner  filed  its  position  
paper,   alleging   operational   losses.   Pending   the   resolution   of   C-­‐‑5415,   the   petitioner,   informed   the   Secretary  
of   Labor   that   Rizal   Cement   Co.,   Inc.,   "from   which   it   derives   income"   "as   the   General   Manager   or   Agent"   had  
"ceased   operating   temporarily."   In   addition,   "because   of   the   desire   of   the   stockholders   to   phase   out   the  
operations  of  the  Madrigal  &  Co.,  Inc.  due  to  lack  of  business  incentives  and  prospects,  and  in  order  to  prevent  
further  losses,"  it  had  to  reduce  its  capital  stock  on  two  occasions  "As  the  situation,  therefore,  now  stands,  
the   Madrigal   &   Co.,   Inc.   is   without   substantial   income   to   speak   of,   necessitating   a   reorganization,   by   way   of  
retrenchment,  of  its  employees  and  operations."  The  petitioner  then  requested  that  it  "be  allowed  to  effect  
said  reorganization  gradually  considering  all  the  circumstances,  by  phasing  out  in  at  least  three  stages,  or  in  
a  manner  the  Company  deems  just,  equitable  and  convenient  to  all  concerned,  about  which  your  good  office  
will  be  apprised  accordingly."  
 
On   January   19,   1976,   the   labor   arbiter   rendered   a   decision   granting   the   request   of   MCOE.   The   arbiter  
specifically   found   that   the   petitioner   "had   been   making   substantial   profits   in   its   operation"   since   1972  
through  1975.  The  petitioner  appealed.  
 
On  January  29,  1976,  the  petitioner  applied  for  clearance  to  terminate  the  services  of  a  number  of  employees  
pursuant  supposedly  to  its  retrenchment  program.  On  February  3,  1976,  the  petitioner  applied  for  clearance  
to  terminate  18  employees  more.  On  the  same  date,  the  respondent  union  went  to  the  Regional  Office  of  the  
DOLE   to   complain   of   illegal   lockout   against   the   petitioner.   Acting   on   this   complaint,   the   Secretary   of   Labor,  
in  a  decision,  found  the  dismissals  "to  be  contrary  to  law"  and  ordered  the  petitioner  to  reinstate  some  40  
employees,  37  of  them  with  backwages.  The  petitioner  then  moved  for  reconsideration,  which  the  Acting  
Labor  Secretary,  Amado  Inciong,  denied.  
 
Thereafter,   the   petitioner   filed   an   appeal   to   the   Office   of   the   President.   The   respondent,   the   Presidential  
Assistant  on  Legal  Affairs,  affirmed  with  modification  the  Labor  Department's  decision,  excluding  some  from  
the  reinstatement  and  the  others  from  the  payment  of  back  wages.  
 
Meanwhile,  the  NLRC  rendered  a  decision  affirming  the  labor  arbiter's  judgment  in  C-­‐‑5415.  The  petitioner  
appealed  to  the  Secretary  of  Labor.  On  June  9,  1978,  the  Secretary  of  Labor  dismissed  the  appeal.  Following  
these  successive  reversals,  the  petitioner  came  anew  to  this  court.  
 
Issues:   Whether   or   not   the   claim   of   petitioner   of   continuous   loss   in   business   is   valid   so   as   to   justify   the  
retrenchment  of  their  employees?  
 

   
3H  A.Y.  2017-­‐2018   101  
 
 

CORPORATION  LAW  CASE  DIGESTS  –  ATTY.  DANTE  DELA  CRUZ  

Held:  No.  What  clearly  emerges  from  the  recorded  facts  is  that  the  petitioner,  awash  with  profits  from  its  
business  operations  but  confronted  with  the  demand  of  the  union  for  wage  increases,  decided  to  evade  its  
responsibility  towards  the  employees  by  a  devised  capital  reduction.  While  the  reduction  in  capital  stock  
created   an   apparent   need   for   retrenchment,   it   was,   by   all   indications,   just   a   mask   for   the   purge   of   union  
members,  who,  by  then,  had  agitated  for  wage  increases.  In  the  face  of  the  petitioner  company's  piling  profits,  
the  unionists  had  the  right  to  demand  for  such  salary  adjustments.  
 
The  petitioner  would,  however,  have  us  believe  that  it  in  fact  sustained  losses.  Whatever  profits  it  earned,  so  
it  claims  were  in  the  nature  of  dividends  "declared  on  its  share  holdings  in  other  companies  in  the  earning  of  
which   the   employees   had   no   participation   whatsoever."   "Cash   dividends,"   according   to   it,"   are   the   absolute  
property  of  the  stockholders  and  cannot  be  made  available  for  disposition  if  only  to  meet  the  employees'  
economic  demands."  
 
There  is  no  merit  in  this  contention.  We  agree  with  the  NLRC  that  "[t]he  dividends  received  by  the  company  
are   corporate   earnings   arising   from   corporate   investment."   Indeed,   as   found   by   the   Commission,   the  
petitioner  had  entered  such  earnings  in  its  financial  statements  as  profits,  which  it  would  not  have  done  if  
they  were  not  in  fact  profits.  
 
Moreover,  it  is  incorrect  to  say  that  such  profits  —  in  the  form  of  dividends  —  are  beyond  the  reach  of  the  
petitioner's  creditors  since  the  petitioner  had  received  them  as  compensation  for  its  management  services  
in  favor  of  the  companies  it  managed  as  a  shareholder  thereof.  As  such  shareholder,  the  dividends  paid  to  it  
were   its   own   money,   which   may   then   be   available   for   wage   increments.   It   is   not   a   case   of   a   corporation  
distributing   dividends   in   favor   of   its   stockholders,   in   which   case,   such   dividends   would   be   the   absolute  
property  of  the  stockholders  and  hence,  out  of  reach  by  creditors  of  the  corporation.  Here,  the  petitioner  was  
acting  as  stockholder  itself,  and  in  that  case,  the  right  to  a  share  in  such  dividends,  by  way  of  salary  increases,  
may  not  be  denied  its  employees.  
 
The   petitioner's   capital   reduction   efforts   were,   to   begin   with,   a   subterfuge,   a   deception   as   it   were,   to  
camouflage   the   fact   that   it   had   been   making   profits,   and   consequently,   to   justify   the   mass   layoff   in   its  
employee  ranks,  especially  of  union  members.  They  were  nothing  but  a  premature  and  plain  distribution  of  
corporate  assets  to  obviate  a  just  sharing  to  labor  of  the  vast  profits  obtained  by  its  joint  efforts  with  capital  
through  the  years.  Surely,  we  can  neither  countenance  nor  condone  this.  It  is  an  unfair  labor  practice.  
 
 
 
74.  HENRY  DELA  RAMA  CO,  PETITIONER,  VS.  ADMIRAL  UNITED  SAVINGS  BANK,  RESPONDENT.  
G.R.  NO.  154740.  APRIL  16,  2008  
NACHURA,  J  
 
DOCTRINE:    
 
FACTS  :  Admiral  United  Savings  Bank  (ADMIRAL)  extended  a  loan  of  P500,000  to  petitioner  Henry  Dela  Rama  
Co   (Co),   with   Leocadio   O.   Isip   (Isip)   as   co-­‐‑maker.   The   loan   was   evidenced   by   a   promissory   note   dated  
February  28,  1983  and  payable  on  or  before  February  23,  1984.  Co  and  Isip  failed  to  pay  the  loan  when  it  
became  due  and  demandable.  ADMIRAL  made  demands  for  payment,  but  these  were  not  heeded.    
 Co  answered  the  complaint  alleging  that  the  promissory  note  was  sham  and  frivolous;  hence,  void  ab  initio.  
He  denied  receiving  any  benefits  from  the  loan  transaction,  claiming  that  ADMIRAL  merely  induced  him  into  
executing  a  promissory  note.  He  also  claimed  that  the  obligations,  if  any,  had  been  paid,  waived  or  otherwise  
extinguished.  Co  allegedly  ceded  several  vehicles  to  ADMIRAL,  the  value  of  which  was  more  than  enough  to  
cover  the  alleged  obligation.  He  added  that  there  was  condonation  of  debt  and  novation  of  the  obligation.  

   
3H  A.Y.  2017-­‐2018   102  
 
 

CORPORATION  LAW  CASE  DIGESTS  –  ATTY.  DANTE  DELA  CRUZ  

Finally,  he  argued  that  the  case  was  prematurely  filed  and  was  not  prosecuted  against  the  real  parties-­‐‑in-­‐‑
interest.    
Pending   resolution   of   the   case,   Isip   died.   Co   then   filed   a   third   party   complaint   against   Metropolitan   Rentals  
&  Sales,  Inc.  (METRO  RENT).  He  averred  that  the  incorporators  and  officers  of  METRO  RENT  were  the  ones  
who  prodded  him  in  obtaining  a  loan  of  P500,000.00  from  ADMIRAL.  The  proceeds  of  the  loan  were  given  to  
the   directors   and   officers   of   METRO   RENT,   who   assured   him   of   prompt   payment   of   the   loan   obligation.  
METRO   RENT   denied   receiving   the  loan,   and   claimed   that   Co   took   the   subject   loan   on   his   personal   use   and  
benefit.  
 
ISSUE:  Whether  or  not  Co  is  liable    
 
HELD:   YES.   The   document,   bearing   Co's   signature,   speaks   for   itself.   To   repeat,   Co   has   not   questioned   the  
genuineness  and  due  execution  of  the  note.  By  signing  the  promissory  note,  Co  acknowledged  receipt  of  the  
loan   amounting   to   P500,000.00,   and   undertook   to   pay   the   same,   plus   interest,   to   ADMIRAL   on   or   before  
February  28,  1984.  Thus,  he  cannot  validly  set  up  the  defense  that  he  did  not  receive  the  value  of  the  note  or  
any   consideration   therefor.   At   any   rate,   Co's   assertion   that   he   merely   acted   as   an   accommodation   party   for  
METRO  RENT  cannot  release  him  from  liability  under  the  note.  An  accommodation  party  who  lends  his  name  
to  enable  the  accommodated  party  to  obtain  credit  or  raise  money  is  liable  on  the  instrument  to  a  holder  for  
value  even  if  he  receives  no  part  of  the  consideration.    He  assumes  the  obligation  to  the  other  party  and  binds  
himself  to  pay  the  note  on  its  due  date.  By  signing  the  note,  Co  thus  became  liable  for  the  debt  even  if  he  had  
no  direct  personal  interest  in  the  obligation  or  did  not  receive  any  benefit  therefrom.    
Co  also  offered  the  alternative  defense  that  the  loan  had  already  been  extinguished  by  payment.  He  testified  
that  METRO  RENT  paid  the  loan  a  week  before  April  11,  1983.      
In   Alonzo   v.   San   Juan,     we   held   that   the   receipts   of   payment,   although   not   exclusive,   were   deemed   to   be   the  
best   evidence   of   the   fact   of   payment.     In   this   case,   no   receipt   was   presented   to   substantiate   the   claim   of  
payment.  Instead,  Co  presented  a  Release  of  Real  Estate  Mortgage    dated  April  11,  1983  to  prove  his  assertion.  
But  a  cancellation  of  mortgage  is  not  conclusive  proof  of  payment  of  a  loan,  even  as  it  may  serve  as  basis  for  
an  inference  that  payment  of  the  principal  obligation  had  been  made.  Thus,  in  the  case  at  bench,  ADMIRAL  
can  still  run  after  Co  for  the  payment  of  the  loan  under  the  promissory  note,  even  after  the  release  of  the  
mortgage  on  the  properties,  especially  because  there   was  no  showing  that  the  mortgage  was  constituted  as  
a  security  for  the  loan  covered  by  the  promissory  note.    
 
 
 
75.  UNIVERSITY  OF  MINDANAO,  INC  VS.  BANGKO  SENTRAL  NG  PILIPINAS  
G.R.  NOS.  194964-­‐‑65.  JANUARY  11,  2016.  
LEONEN,  J.  
 
DOCTRINE:    
Acts   of   an   officer   that   are   not   authorized   by   the   board   of   directors/trustees   do   not   bind   the   corporation  
unless   the   corporation   ratifies   the   acts   or   holds   the   officer   out   as   a   person   with   authority   to   transact   on   its  
behalf.    
 
FACTS:    
University  of  Mindanao  is  an  educational  institution.  For  the  year  1982,  its  Board  of  Trustees  was  chaired  by  
Guillermo  B.  Torres.  His  wife,  Dolores  P.  Torres,  sat  as  University  of  Mindanao's  Assistant  Treasurer.  Before  
1982,  Guillermo  B.  Torres  and  Dolores  P.  Torres  incorporated  and  operated  two  (2)  thrift  banks:  (1)  First  
Iligan  Savings  &  Loan  Association,  Inc.  (FISLAI);  and  (2)  Davao  Savings  and  Loan  Association,  Inc.  (DSLAI).  
Guillermo   B.   Torres   chaired   both   thrift   banks.   He   acted   as   FISLAI's   President,   while   his   wife,   Dolores   P.  
Torres,  acted  as  DSLAI's  President  and  FISLAI's  Treasurer.  Upon  Guillermo  B.  Torres'  request,  Bangko  Sentral  

   
3H  A.Y.  2017-­‐2018   103  
 
 

CORPORATION  LAW  CASE  DIGESTS  –  ATTY.  DANTE  DELA  CRUZ  

ng   Pilipinas   issued   a   P1.9   million   standby   emergency   credit   to   FISLAI.   The   release   of   standby   emergency  
credit  was  evidenced  by  three  (3)  promissory  notes,  signed  by  Guillermo  B.  Torres,  and  were  co-­‐‑signed  by  
either   his   wife,   Dolores   P.   Torres,   or   FISLAI's   Special   Assistant   to   the   President,   Edmundo   G.   Ramos,   Jr.  
University   of   Mindanao's   Vice   President   for   Finance,   Saturnino   Petalcorin,   executed   a   deed   of   real   estate  
mortgage   over   University   of   Mindanao's   property   in   Cagayan   de   Oro   City   in   favor   of   Bangko   Sentral   ng  
Pilipinas.     "The  mortgage  served  as  security  for  FISLAI's  P1.9  Million  loan[.]"   It  was  allegedly  executed  on  
University  of  Mindanao's  behalf.  As  proof  of  his  authority  to  execute  a  real  estate  mortgage  for  University  of  
Mindanao,  Saturnino  Petalcorin  showed  a  Secretary's  Certificate  signed  on  April  13,  1982  by  University  of  
Mindanao's   Corporate   Secretary,   Aurora   de   Leon.   On   October   21,   1982,   Bangko   Sentral   ng   Pilipinas   g  
Saturnino   Petalcorin   executed   another   deed   of   real   estate   mortgage,   allegedly   on   behalf   of   University   of  
Mindanao,   over   its   two   properties   in   Iligan   City.   This   mortgage   served   as   additional   security   for   FISLAI's  
loans  granted  FISLAI  an  additional  loan.  Bangko  Sentral  ng  Pilipinas'  mortgage  lien  over  the  Cagayan  De  Oro  
and   Iligan   City   properties   were   annotated   on   the   certitifcate   of   titles.   Subsequently,   FISLAI   and   DSLAI  
merged,  with  DSLAI  as  the  surviving  corporation.  DSLAI  later  became  known  as  Mindanao  Savings  and  Loan  
Association,  Inc.  (MSLAI),  which  failed  to  recover  from  its  losses.  Bangko  Sentral  ng  Pilipinas  sent  a  letter  to  
University  of  Mindanao,  informing  it  that  the  bank  would  foreclose  its  properties  if  MSLAI's  total  outstanding  
obligation   of   P12,534,907.73   remained   unpaid.   University   of   Mindanao,   through   its   Vice   President   for  
Accounting,  Gloria  E.  Detoya,  denied  that  University  of  Mindanao's  properties  were  mortgaged.  It  also  denied  
having   received   any   loan   proceeds   from   Bangko   Sentral   ng   Pilipinas.   University   of   Mindanao   filed   two  
Complaints  for  nullification  and  cancellation  of  mortgage.  University  of  Mindanao  also  alleged  that  Aurora  
de   Leon's   certification   was   anomalous.   It   never   authorized   Saturnino   Petalcorin   to   execute   real   estate  
mortgage  contracts  involving  its  properties  to  secure  FISLAI's  debts.  It  never  ratified  the  execution  of  the  
mortgage   contracts.   Moreover,   as   an   educational   institution,   it   cannot   mortgage   its   properties   to   secure  
another  person's  debts.    
 
The   Regional   Trial   Court   of   Cagayan   de   Oro   City   rendered   a   Decision   in   favor   of   University   of   Mindanao.  
Similarly,   the   Regional   Trial   Court   of   Iligan   City   rendered   a   Decision   on   December   7,   2001   in   favor   of  
University   of   Mindanao.   Bangko   Sentral   ng   Pilipinas   separately   appealed   the   Decisions   of   both   the   Cagayan  
de   Oro   City   and   the   Iligan   City   trial   courts.     After   consolidating   both   cases,   the   Court   of   Appeals   issued   a  
Decision  on  December  17,  2009  in  favor  of  Bangko  Sentral  ng  Pilipinas.    
ISSUES:  
Whether  petitioner  University  of  Mindanao  is  bound  by  the  real  estate  mortgage  contracts.  
 
HELD:  
NO.  Petitioner  does  not  have  the  power  to  mortgage  its  properties  in  order  to  secure  loans  of  other  persons.  
As  an  educational  institution,  it  is  limited  to  developing  human  capital  through  formal  instruction.  It  is  not  a  
corporation   engaged   in   the   business   of   securing   loans   of   others.   Securing   FISLAI's   loans   by   mortgaging  
petitioner's  properties  does  not  appear  to  have  even  the  remotest  connection  to  the  operations  of  petitioner  
as  an  educational  institution.    
 
Regardless  of  the  number  of  shares  that  petitioner  had  with  FISLAI,  DSLAI,  or  MSLAI,  securing  loans  of  third  
persons  is  still  beyond  petitioner's  power  to  do.  It  is  still  inconsistent  with  its  purposes  under  the  law  and  its  
articles  of  incorporation.  In  attempting  to  show  petitioner's  interest  in  securing  FISLAI's  loans  by  adverting  
to   their   interlocking   directors   and   shareholders,   respondent   disregards   petitioner's   separate   personality  
from   its   officers,   shareholders,   and   other   juridical   persons.   Petitioner's   key   officers,   as   shareholders   of  
FISLAI,  may  have  an  interest  in  ensuring  the  viability  of  FISLAI  by  obtaining  a  loan  from  respondent  and  
securing  it  by  whatever  means.  However,  having  interlocking  officers  and  stockholders  with  FISLAI  does  not  
mean  that  petitioner,  as  an  educational  institution,  is  or  must  necessarily  be  interested  in  the  affairs  of  FISLAI.  
Since  petitioner  is  an  entity  distinct  and  separate  not  only  from  its  own  officers  and  shareholders  but  also  
from  FISLAI,  its  interests  as  an  educational  institution  may  not  be  consistent  with  FISLAI's.  Petitioner  has  no  

   
3H  A.Y.  2017-­‐2018   104  
 
 

CORPORATION  LAW  CASE  DIGESTS  –  ATTY.  DANTE  DELA  CRUZ  

business   in   securing   FISLAI,   DSLAI,   or   MSLAI's   loans.   This   activity   is   not   compatible   with   its   business   of  
providing  quality  instruction  to  its  constituents.    
 
Furthermore,  there  is  no  evidence  pointing  to  the  possibility  that  petitioner  used  its  separate  personality  to  
defraud  third  persons  or  commit  illegal  acts.  Neither  is  there  evidence  to  show  that  petitioner  was  merely  a  
farce   of   a   corporation.   What   has   been   shown   instead   was   that   petitioner,   too,   had   been   victimized   by  
fraudulent  and  unauthorized  acts  of  its  own  officers  and  directors.    
 
The  mortgage  contracts  executed  in  favor  of  respondent  do  not  bind  petitioner.  They  were  executed  without  
authority   from   petitioner.   BSP   failed   to   prove   that   the   UM   Board   of   Trustees   actually   passed   a   Board  
Resolution  authorizing  Petalcorin  to  mortgage  the  subject  real  properties  Hence,  not  having  the  proper  board  
resolution   to   authorize   Saturnino   Petalcorin   to   execute   the   mortgage   contracts   for   petitioner,   the   contracts  
he  executed  are  unenforceable  against  petitioner.  They  cannot  bind  petitioner.  However,  personal  liabilities  
may   be   incurred   by   directors   who   assented   to   such   unauthorized   act     and   by   the   person   who   contracted   in  
excess  of  the  limits  of  his  or  her  authority  without  the  corporation's  knowledge.    
 
In  addition,  even  though  the  Spouses  Guillermo  and  Dolores  Torres  were  officers  of  both  the  thrift  banks  and  
petitioner,  their  knowledge  of  the  mortgage  contracts  cannot  be  considered  as  knowledge  of  the  corporation.    
 
Lastly,  for  its  failure  to  exercise  the  degree  of  diligence  required  of  banks,  respondent  cannot  claim  good  faith  
in  the  execution  of  the  mortgage  contracts  with  Saturnino  Petalcorin.    
 
 
 
76.  HARDEN  V.  BENGUET  CONSOLIDATED  MINING  COMPANY  
G.R.  NO.  L-­‐‑37331                          MARCH  18,  1933  
STREET  J.;  
 
FACTS:  
-­‐‑   Benguet   Consolidated   Mining   Co.   was   organized   in   June,   1903,   as   a   sociedad   anonima   under   the  
Spanish  law,  while  the  Balatoc  Mining  Co.  was  organized  in  December  1925,  as  a  corporation,  under  
Corporation  Law  (Act  No.  1459).  
-­‐‑   In   1926,   while   all   the   works   of   the   Balatoc   Mining   remained   undeveloped,   the   Board   ordered   the  
suspension  of  its  work.  
-­‐‑   Come  1927,  Benguet  Consolidated,  having  interested  in  the  completion  of  the  purpose  of  the  Balatoc,  
agreed  to  erect  and  operate  the  mine.  
-­‐‑   Benguet  Consolidated  garnering  success  on  its  operations,  plaintiff  Fred  Harden  with  complacency  
now  questions  the  rights  of  Benguet  to  control  Balatoc,  he  being  the  owner  of  many  thousands  of  the  
shares  of  the  Balatoc  Company.  
-­‐‑   Plaintiff  claims  that  it  is  unlawful  for  any  member  of  a  corporation  engaged  in  agriculture  or  mining  
and   for   any   corporation   organized   for   any   purpose   except   irrigation   to   be   in   any   wise   interested   in  
any  other  corporation  engaged  in  agriculture  or  in  mining."  
 
ISSUE:  Whether  respondent  could  be  considered  as  a  corporation  and  subject  to  the  prohibition  as  stated.  
 
HELD:  
•   As   it   was   the   intention   of   our   lawmakers   of   the   introduction   of   the   American   Corporation   into  
Philippine  law  in  the  place  of  the  sociedad  anonima,  certain  adjustments  resulting  from  the  continued  
co-­‐‑existence,  for  a  time,  of  the  two  forms  of  commercial  entities.    

   
3H  A.Y.  2017-­‐2018   105  
 
 

CORPORATION  LAW  CASE  DIGESTS  –  ATTY.  DANTE  DELA  CRUZ  

•   Section  75  of  the  Corporation  Law,  a  provision  is  found  making  the  sociedad  anonima  subject  to  the  
provisions  of  the  Corporation  Law  "so  far  as  such  provisions  may  be  applicable",  and  giving  to  the  
sociedades  anonimas  previously  created  in  the  Islands  the  option  to  continue  business  as  such  or  to  
reform  and  organize  under  the  provisions  of  the  Corporation  Law.    
•   Again,  in  section  191  of  the  Corporation  Law,  the  Code  of  Commerce  is  repealed  in  so  far  as  it  relates  
to  sociedades  anonimas.    
•   The   purpose   of   the   commission   in   repealing   this   part   of   the   Code   of   Commerce   was   to   compel  
commercial   entities   thereafter   organized   to   incorporate   under   the   Corporation   Law,   unless   they  
should  prefer  to  adopt  some  form  or  other  of  the  partnership.    
•   To  this  provision  was  added  another  to  the  effect  that  existing  sociedades  anonimas,  which  elected  
to  continue  their  business,  instead  of  reforming  and  reorganizing  under  the  Corporation  Law,  should  
continue  to  be  governed  by  the  laws  that  were  in  force  prior  to  the  passage  of  this  Act.  
•   However,  the  defendant  Benguet  Company  has  committed  no  civil  wrong  against  the  plaintiffs.  
 
 
 
77.   HARRY   S.   STONEHILL,   ROBERT   P.   BROOKS,   JOHN   J.   BROOKS   AND   KARL   BECK,   PETITIONERS,    
VS.  HON.  JOSE  W.  DIOKNO,  IN  HIS  CAPACITY  AS  SECRETARY  OF  JUSTICE  
G.R.  NO.  L-­‐‑19550                          JUNE  19,  1967  
CONCEPCION,  C.J.:  
 
DOCTRINE:  
The   right   to   object   to   the   admission   of   said   papers   in   evidence   belongs   exclusively   to   the  
corporations,   to   whom   the   seized   effects   belong,   and   may   not   be   invoked   by   the   corporate   officers   in  
proceedings  against  them  in  their  individual  capacity.  
 
FACTS:  
Upon   application   of   the   officers   of   the   government   hereinafter   referred   to   as   Respondents-­‐‑
Prosecutors  Respondents-­‐‑Judges  issued,  on  different  dates,3  a  total  of  42  search  warrants  against  petitioners  
herein4  and/or  the  corporations  of  which  they  were  officers,5  directed  to  the  any  peace  officer,  to  search  the  
persons  above-­‐‑named  and/or  the  premises  of  their  offices,  warehouses  and/or  residences,  and  to  seize  and  
take  possession  of  Books  of  accounts,  financial  records,  vouchers,  correspondence,  receipts,  ledgers,  journals,  
portfolios,   credit   journals,   typewriters,   and   other   documents   and/or   papers   showing   all   business  
transactions  including  disbursements  receipts,  balance  sheets  and  profit  and  loss  statements  and  Bobbins  
(cigarette  wrappers).  as  "the  subject  of  the  offense;  stolen  or  embezzled  and  proceeds  or  fruits  of  the  offense,"  
or   "used   or   intended   to   be   used   as   the   means   of   committing   the   offense,"   which   is   described   in   the  
applications   adverted   to   above   as   "violation   of   Central   Bank   Laws,   Tariff   and   Customs   Laws,   Internal  
Revenue  (Code)  and  the  Revised  Penal  Code."  This  Court  issued  the  writ  of  preliminary  injunction  prayed  for  
in   the   petition.   However,   by   resolution,   the   writ   was   partially   lifted   or   dissolved,   insofar   as   the   papers,  
documents   and   things   seized   from   the   offices   of   the   corporations   above   mentioned   are   concerned;   but,   the  
injunction  was  maintained  as  regards  the  papers,  documents  and  things  found  and  seized  in  the  residences  
of  petitioners  herein.7  
 
ISSUE:  Whether  Petitioners  can  assail  the  validity  of  the  search  warrant  conducted  against  the  corporation.  
 
HELD:  
NO.  The  petitioners  herein  have  no  cause  of  action  to  assail  the  legality  of  the  contested  warrants  and  
of  the  seizures  made  in  pursuance  thereof,  for  the  simple  reason  that  said  corporations  have  their  respective  
personalities,  separate  and  distinct  from  the  personality  of  herein  petitioners,  regardless  of  the  amount  of  
shares  of  stock  or  of  the  interest  of  each  of  them  in  said  corporations,  and  whatever  the  offices  they  hold  
   
3H  A.Y.  2017-­‐2018   106  
 
 

CORPORATION  LAW  CASE  DIGESTS  –  ATTY.  DANTE  DELA  CRUZ  

therein  may  be.8  Indeed,  it  is  well  settled  that  the  legality  of  a  seizure  can  be  contested  only  by  the  party  
whose  rights  have  been  impaired  thereby,9  and  that  the  objection  to  an  unlawful  search  and  seizure  is  purely  
personal  and  cannot  be  availed  of  by  third  parties.  10  Consequently,  petitioners  herein  may  not  validly  object  
to  the  use  in  evidence  against  them  of  the  documents,  papers  and  things  seized  from  the  offices  and  premises  
of  the  corporations  adverted  to  above,  since  the  right  to  object  to  the  admission  of  said  papers  in  evidence  
belongs  exclusively  to  the  corporations,  to  whom  the  seized  effects  belong,  and  may  not  be  invoked  by  the  
corporate  officers  in  proceedings  against  them  in  their  individual  capacity.    
 
 
 
78.  BACHE  &  CO.  (PHIL.),  INC.  AND  FREDERICK  E.  SEGGERMAN,  VS.  HON.  JUDGE  VIVENCIO  M.  RUIZ  
G.R.  NO.  L-­‐‑32409.  FEBRUARY  27,  1971  
J.  VILLAMOR      
 
DOCTRINE:   A   search   warrant   may   be   said   to   particularly   describe   the   things   to   be   seized   when   the  
description   therein   is   as   specific   as   the   circumstances   will   ordinarily   allow;   or   when   the   description  
expresses  a  conclusion  of  fact  —  not  of  law  —  by  which  the  warrant  officer  may  be  guided  in  making  the  
search   and   seizure;   or   when     things   described   are   limited   to   those   which   bear   direct   relation   to   the   offense  
for  which  the  warrant  is  being  issued  (Sec.  2,  Rule  126,  Revised  Rules  of  Court).  
 
FACTS:  On  February  24,  1970,  respondent  Misael  P.  Vera,  Commissioner  of  Internal  Revenue,  wrote  a  letter  
addressed   to   respondent   Judge   Vivencio   M.   Ruiz   requesting   the   issuance   of   a   search   warrant   against  
petitioners   for   violation   of   Section   46(a)   of   the   National   Internal   Revenue   Code,   in   relation   to   all   other  
pertinent   provisions   thereof,   particularly   Sections   53,   72,   73,   208   and   209,   and   authorizing   Revenue  
Examiner  Rodolfo  de  Leon,  one  of  herein  respondents,  to  make  and  file  the  application  for  search  warrant  
which  was  attached  to  the  letter.  On  February  28,  1970,  which  was  a  Saturday,  the  BIR  agents  served  the  
search  warrant  petitioners  at  the  offices  of  petitioner  corporation  on  Ayala  Avenue,  Makati,  Rizal.  Petitioners'  
lawyers  protested  the  search  on  the  ground  that  no  formal  complaint  or  transcript  of  testimony  was  attached  
to  the  warrant.  The  agents  nevertheless  proceeded  with  their  search  which  yielded  six  boxes  of  documents.    
 
On  March  3,  1970,  petitioners  filed  a  petition  with  the  Court  of  First  Instance  of  Rizal  praying  that  the  search  
warrant  be  quashed,  dissolved  or  recalled,  that  preliminary  prohibitory  and  mandatory  writs  of  injunction  
be  issued,  that  the  search  warrant  be  declared  null  and  void,  and  that  the  respondents  be  ordered  to  pay  
petitioners,  jointly  and  severally,  damages  and  attorney's  fees.  On  March  18,  1970,  the  respondents,  thru  the  
Solicitor  General,  filed  an  answer  to  the  petition.  After  hearing,  the  court,  presided  over  by  respondent  Judge,  
issued   on   July   29,   1970,   an   order   dismissing   the   petition   for   dissolution   of   the   search   warrant.   In   the  
meantime,   or   on   April   16,   1970,   the   Bureau   of   Internal   Revenue   made   tax   assessments   on   petitioner  
corporation  in  the  total  sum  of  P2,594,729.97,  partly,  if  not  entirely,  based  on  the  documents  thus  seized.  
 
ISSUE:  Should  the  search  warrant  be  quashed?    
 
HELD:  Yes.  The  documents,  papers  and  effects  sought  to  be  seized  are  described  in  Search  Warrant  No.  2-­‐‑M-­‐‑
70  in  this  manner:  "Unregistered  and  private  books  of  accounts  (ledgers,  journals,  columnars,  receipts  and  
disbursements   books,   customers   ledgers);   receipts   for   payments   received;   certificates   of   stocks   and  
securities;   contracts,   promissory   notes   and   deeds   of   sale;   telex   and   coded   messages;   business  
communications,   accounting   and   business   records;   checks   and   check   stubs;   records   of   bank   deposits   and  
withdrawals;  and  records  of  foreign  remittances,  covering  the  years  1966  to  1970."  The  description  does  not  
meet  the  requirement  in  Art  III,  Sec.  1,  of  the  Constitution,  and  of  Sec.  3,  Rule  126  of  the  Revised  Rules  of  
Court,   that   the   warrant   should   particularly   describe   the   things   to   be   seized.   While   the   term   "all   business  
transactions"  does  not  appear  in  Search  Warrant  No.  2-­‐‑M-­‐‑70,  the  said  warrant  nevertheless  tends  to  defeat  

   
3H  A.Y.  2017-­‐2018   107  
 
 

CORPORATION  LAW  CASE  DIGESTS  –  ATTY.  DANTE  DELA  CRUZ  

the  major  objective  of  the  Bill  of  Rights,  i.e.,  the  elimination  of  general  warrants,  for  the  language  used  therein  
is   so   all-­‐‑embracing   as   to   include   all   conceivable   records   of   petitioner   corporation,   which,   if   seized,   could  
possibly  render  its  business  inoperative.  
 
A  search  warrant  may  be  said  to  particularly  describe  the  things  to  be  seized  when  the  description  therein  is  
as  specific  as  the  circumstances  will  ordinarily  allow;  or  when  the  description  expresses  a  conclusion  of  fact  
—  not  of  law  —  by  which  the  warrant  officer  may  be  guided  in  making  the  search  and  seizure;  or  when    
things  described  are  limited  to  those  which  bear  direct  relation  to  the  offense  for  which  the  warrant  is  being  
issued  (Sec.  2,  Rule  126,  Revised  Rules  of  Court).  The  herein  search  warrant  does  not  conform  to  any  of  the  
foregoing   tests.   If   the   articles   desired   to   be   seized   have   any   direct   relation   to   an   offense   committed,   the  
applicant  must  necessarily  have  some  evidence,  other  than  those  articles,  to  prove  the  said  offense;  and  the  
articles  subject  of  search  and  seizure  should  come  in  handy  merely  to  strengthen  such  evidence.  In  this  event,  
the  description  contained  in  the  herein  disputed  warrant  should  have  mentioned,  at  least,  the  dates,  amounts,  
persons,  and  other  pertinent  data  regarding  the  receipts  of  payments,  certificates  of  stocks  and  securities,  
contracts,   promissory   notes,   deeds   of   sale,   messages   and   communications,   checks,   bank   deposits   and  
withdrawals,  records  of  foreign  remittances,  among  others,  enumerated  in  the  warrant.  
 
 
79.  MAMBULAO  LUMBER  COMPANY  VS.  PHILIPPINE  NATIONAL  BANK  
G.R.  NO.  L-­‐‑22973                      JANUARY  30,  1968  
ANGELES,  J.:  
 
DOCTRINE:   Obviously,   an   artificial   person   like   herein   appellant   corporation   cannot   experience   physical  
sufferings,  mental  anguish,  fright,  serious  anxiety,  wounded  feelings,  moral  shock  or  social  humiliation  which  
are   basis   of   moral   damages.   A   corporation   may   have   a   good   reputation   which,   if   besmirched,   may   also   be   a  
ground  for  the  award  of  moral  damages.    
FACTS:   Plaintiff   applied   for   an   industrial   loan   of   P155K   with   the   Naga   Branch   of   defendant   PNB   and   the  
former  offered  real  estate,  machinery,  logging  and  transportation  equipments  as  collaterals.    
 
To   secure   the   payment   of   the   loan,   the   plaintiff   mortgaged   to   defendant   PNB   a   parcel   of   land,   situated  
province  of  Camarines  Norteas  well  as  various  sawmill  equipment,  rolling  unit  and  other  fixed  assets.  
 
August  2,  1956-­‐‑  the  PNB  released  from  the  approved  loan  the  sum  of  P27,500,  for  which  the  plaintiff  signed  
a  promissory  note  wherein  it  promised  to  pay  to  the  PNB  the  said  sum  in  five  equal  yearly  installments  every  
year  thereafter.  
October  19,  1956,  the  PNB  -­‐‑  made  another  release  of  P15,500  another  PN  executed.    
 
The  plaintiff  failed  to  pay  the  amortization  on  the  amounts  released  to  and  received  by  it.  Repeated  demands  
were  made  upon  the  plaintiff  to  pay  its  obligation  but  it  failed  or  otherwise  refused  to  do  so.  Thereafter,  it  
was  found  that  the  plaintiff  had  already  stopped  operation  about  the  end  of  1957  or  early  part  of  1958.  
 
Thereafter,  PNB  requested  prov.  Sheriff  to  take  possession  of  the  parcel  of  land  improvements  covered  and  
to   sell   it   at   public   auction   for   the   satisfaction   of   the   unpaid   obligation   of   the   plaintiff.   Consequently,   the  
Provincial  Sheriff  issued  the  notice  of  extra-­‐‑judicial  sale  and  sent  a  copy  thereof  to  the  plaintiff.    Then  PNB  
sent  notice  to  the  plaintiff  that  the  former  was  foreclosing  extrajudicially  the  chattels  mortgaged  by  the  latter.  
 
Consequently,  plaintiff  protested  against  the  foreclosure  of  the  real  estate  and  chattel  mortgages.  
 

   
3H  A.Y.  2017-­‐2018   108  
 
 

CORPORATION  LAW  CASE  DIGESTS  –  ATTY.  DANTE  DELA  CRUZ  

The   foreclosure   sale   of   the   parcel   of   land,   together   with   the   buildings   and   improvements   thereon,   was,  
however,  held  and  the  said  property  was  sold  to  PNB.  Also,  the  foreclosure  sale  of  the  mortgaged  chattels  
was  held  and  they  were  awarded  to  the  PNB  for  the  sum  of  P4,200.  
 
RTC:  rendered  the  decision  appealed  from  which,  as  stated  in  the  first  paragraph  of  this  opinion,  sentenced  
the  Mambulao  Lumber  Company  to  pay  to  the  defendant  PNB  the  sum  of  P3,582.52  with  interest  thereon  at  
the  rate  of  6%  per  annum  from  December  22,  1961  (day  following  the  date  of  the  questioned  foreclosure  of  
plaintiff's  chattels)  until  fully  paid,  and  the  costs.  Mambulao  Lumber  Company  interposed  the  instant  appeal.  
 
PLAINTIFF’S  CONTENTIONS:  
1)   its   obligation   under   the   terms   of   the   two   promissory   notes   it   had   executed   in   favor   of   the   PNB  
amounts  only  to  P56,485.87  when  the  sale  of  real  property  was  effected,  and  not  P58,213.51  as  found  
by  the  trial  court.  
2)   That  the  subsequent  foreclosure  sale  of  its  chattels  is  null  and  void.  
3)   That  for  the  acts  of  the  PNB  in  proceeding  with  the  sale  of  the  chattels,  in  utter  disregard  of  plaintiff's  
vigorous  opposition  thereto,  and  in  taking  possession  thereof  after  the  sale  thru   force,  intimidation,  
coercion,   and   by   detaining   its   "man-­‐‑in-­‐‑charge"   of   said   properties,   the   PNB   is   liable   to   plaintiff   for  
damages  and  attorney's  fees.  
 
ISSUE:  1)  Whether  on  not  interest  due  and  unpaid  should  earn  interest  in  this  case;  
2)  Whether  or  not  the  foreclosure  of  chattel  mortgage  is  valid;  
3)  whether  or  not  the  appellant  t  corporation  should  be  entitled  to  moral  damages  
 
HELD:    
1)   There  is  merit  to  this  claim.  Examining  the  terms  of  the  promissory  note  executed  by  the  appellant  in  
favor  of  the  PNB,  we  find  that  the  agreed  interest  on  the  loan  of  P43,000.00  was  six  per  cent  (6%)  per  
annum  from  date  of  said  notes  "until  paid".    
it   appears   that   in   arriving   at   the   total   indebtedness   of   P57,646.59   as   of   that   date,   the   PNB   had  
compounded   the   principal   of   the   loan   and   the   accrued   6%   interest   thereon   each   time   the   yearly  
amortizations  became  due;  and  to  this  erroneously  computed  total  of  P57,646.59,  RTC  added  6%  interest  
per   annum   In   effect,   the   PNB   has   claimed,   interest   on   accrued   interests   from   the   time   the   various  
amortizations   of   the   loan   became   due   until   the   real   estate   mortgage   executed   to   secure   the   loan   was  
extra-­‐‑judicially  foreclosed  on  November  21,  1961.  This  is  an  error.    
This  is  also  the  clear  mandate  of  Article  2212  of  the  new  Civil  Code  which  provides  that  interest  due  shall  
earn  legal  interest  only  from  the  time  it  is  judicially  demanded,  and  of  Article  1959  ,  that  interest  due  and  
unpaid  shall  not  earn  interest.  By  stipulation,  PNB  can  capitalize  the  interest  due  and  unpaid,  which  as  
added   principal   shall   earn   new   interest;   but   such   stipulation   is   nowhere   to   be   found   in   the   terms   of   the  
promissory  notes  involved  in  this  case.    
2)   The  sale  of  appellant's  chattels  on  the  said  date,  illegal  and  void.  While  the  law  grants  power  and  authority  
to   the   mortgagee   to   sell   the   mortgaged   property   at   a   public   place   in   the   municipality   where   the  
mortgagor  resides  or  where  the  property  is  situated,    this  Court  has  held  that  the  sale  of  a  mortgaged  
chattel   may   be   made   in   a   place   other   than   that   where   it   is   found,   provided   that   the   owner   thereof  
consents  thereto;  or  that  there  is  an  agreement  to  this  effect  between  the  mortgagor  and  the  mortgagee.  .    
So,  when  herein  mortgagor  and  mortgagee  agreed  in  the  mortgage  contract  that  in  cases  of  both  judicial  
and  extra-­‐‑judicial  foreclosure  under  Act  1508,  as  amended,  the  corresponding  complaint  for  foreclosure  
or  the  petition  for  sale  should  be  filed  with  the  courts  or  the  Sheriff  of  Manila,  as  the  case  may  be,  they  
waived  their  corresponding  rights  under  the  law.    
3)   Herein  appellant's  claim  for  moral  damages,  however,  seems  to  have  no  legal  or  factual  basis.    
Obviously,  an  artificial  person  like  herein  appellant  corporation  cannot  experience  physical  sufferings,  
mental  anguish,  fright,  serious  anxiety,  wounded  feelings,  moral  shock  or  social  humiliation  which  are  

   
3H  A.Y.  2017-­‐2018   109  
 
 

CORPORATION  LAW  CASE  DIGESTS  –  ATTY.  DANTE  DELA  CRUZ  

basis   of   moral   damages.   A   corporation   may   have   a   good   reputation   which,   if   besmirched,   may   also   be   a  
ground  for  the  award  of  moral  damages.    
The  same  cannot  be  considered  under  the  facts  of  this  case,  however,  not  only  because  it  is  admitted  that  
herein  appellant  had  already  ceased  in  its  business  operation  at  the  time  of  the  foreclosure  sale  of  the  
chattels,  but  also  for  the  reason  that  whatever  adverse  effects  of  the  foreclosure  sale  of  the  chattels  could  
have   upon   its   reputation   or   business   standing   would   undoubtedly   be   the   same   whether   the   sale   was  
conducted   at   Jose   Panganiban,   Camarines   Norte,   or   in   Manila   which   is   the   place   agreed   upon   by   the  
parties  in  the  mortgage  contract.  
But  for  the  wrongful  acts  of  herein  appellee  bank  and  the  deputy  sheriff  of  Camarines  Norte  in  proceeding  
with  the  sale  in  utter  disregard  of  the  agreement  to  have  the  chattels  sold  in  Manila  as  provided  for  in  the  
mortgage  contract,  to  which  their  attentions  were  timely  called  by  herein  appellant,  and  in  disposing  of  the  
chattels   in   gross   for   the   miserable   amount   of   P4,200.00,   herein   appellant   should   be   awarded   exemplary  
damages.  
 
 
 
80.  LBC  EXPRESS,  INC.  VS  COURT  OF  APPEALS  
GR  NO.  108670  21  SEPTEMBER  1994  
JUSTICE  PUNO  
 
DOCTRINE:  
  Moral  damages  cannot  be  awarded  to  a  corporation,  the  latter  being  an  artificial  person  which  has  
no  feelings,  no  emotions,  no  senses;  thus,  cannot  experience  physical  suffering  and  mental  anguish.      
 
FACTS:  
  Private  respondent  Adolfo  Carloto  is  the  President-­‐‑Manager  of  Rural  Bank  of  Labason.  In  November  
1984,  he  alleged  that  he  was  in  Cebu  transacting  business  with  the  Central  Bank  Regional  Office.  He  was  then  
asked  to  proceed  to  Manila  to  follow-­‐‑up  the  Rural  Bank’s  plan  of  payment  of  rediscounting  obligations  with  
Central  Bank’s  main  office.  He  purchased  a  round  trip  plane  ticket  to  Manila  and  called  his  sister  Eslie  Carloto-­‐‑
Concha   to   send   him   P1,000.00   and   other   documents   through   LBC.   Thus,   Ms.   Concha   through   her   clerk,  
Adelina  Antigo,  sent  the  said  items  to  Carloto’s  Cebu  address  from  LBC  Dipolog  Branch.    
 
  The  next  day,  Carloto  received  the  documents  without  the  cashpack.  Carloto  alleged  that  he  was  being  
compelled  to  claim  the  money  at  LBC’s  office.  When  he  returned  without  the  money,  he  was  advised  that  the  
money  has  been  returned  to  LBC  Dipolog  upon  shipper’s  request.  Carloto  was  only  able  to  receive  the  money  
a  month  thereafter.    
 
  Due   to   the   delay   of   the   transmittal   of   the   cashpack,   Carloto   claimed   that   he   failed   to   submit   the  
documents  to  the  Central  Bank  on  time  and  consequently,  the  Rural  Bank  was  made  to  pay  P32,000.00  as  
penalty.  Carloto  thus  instituted  an  action  against  LBC,  joining  Rural  Bank  of  Labason  as  one  of  the  plaintiffs,  
for  moral  damages  and  reimbursement  of  the  penalty  amount.  The  trial  court  ruled  in  his  favor,  as  well  as  in  
appeal.  
 
ISSUE:  
  Whether  or  not  Rural  Bank  of  Labason,  being  an  artificial  person  should  be  awarded  moral  damages  
 
RULING:  
  No,  moral  damages  cannot  be  awarded  to  private  respondent  Rural  Bank  of  Labason,  Inc.,  the  latter  
being  an  artificial  person.    
 

   
3H  A.Y.  2017-­‐2018   110  
 
 

CORPORATION  LAW  CASE  DIGESTS  –  ATTY.  DANTE  DELA  CRUZ  

Moral   damages   are   granted   in   recompense   for   physical   suffering,   mental   anguish,   fright,   serious  
anxiety,   besmirched   reputation,   wounded   feelings,   moral   shock,   social   humiliation   and   similar   injury.   A  
corporation,   being   an   artificial   person   and   having   existence   only   in   legal   contemplation,   has   no   feelings,   no  
emotions,  no  senses;  therefore,  it  cannot  experience  physical  suffering  and  mental  anguish.  Mental  suffering  
can   be   experienced   only   by   one   having   a   nervous   system   and   it   flows   from   real   ills,   sorrows,   and   grieves   of  
life  –  all  of  which  cannot  be  suffered  by  respondent  bank  as  an  artificial  person.  
 
 
   
81.  ABS-­‐‑CBN  VS.  CA,  REPUBLIC  BROADCASTING,  AND  VIVA  PRODUCTIONS  
G.R.  NO.  128690.  JANUARY  21,  1999  
DAVIDE,  J.  
 
 
DOCTRINE:   The   award   of   moral   damages   cannot   be   granted   in   favor   of   a   corporation   because,   being   an  
artificial  person  and  having  existence  only  in  legal  contemplation,  it  has  no  feelings,  no  emotions,  no  senses.  
 
FACTS:    
In  1990,  ABS-­‐‑CBN  and  VIVA  executed  a  Film  Exhibition  Agreement  whereby  Viva  gave  ABS-­‐‑CBN  an  exclusive  
right   to   exhibit   some   Viva   films.   Paragraph   2.4   of   said   agreement   states   that:     ABS-­‐‑CBN   shall   have   the   right  
of  first  refusal  to  the  next  twenty-­‐‑four  (24)  Viva  films  for  TV  telecast  under  such  terms  as  may  be  agreed  
upon  by  the  parties  hereto,  provided,  however,  that  such  right  shall  be  exercised  by  ABS-­‐‑CBN  from  the  actual  
offer  in  writing.  
 
In  a  letter  dated  in  January  6,  1992,  ABS-­‐‑CBN  expressed  to  defendant  Vicente  Del  Rosario,  Executive  Producer  
of  VIVA,  that  it  can  only  accept  10  films  out  of  the  35  titles  that  could  be  aired  in  television.  Subsequently,  
defendant  Del  Rosario  and  ABS-­‐‑CBNs  general  manager,  Eugenio  Lopez  III,  met  at  a  Restaurant  in  Quezon  City  
to  discuss  the  package  proposal  of  VIVA.  
However  no  concrete  agreement  between  the  two  has  been  sealed.  
On   a   later   date   Del   Rosario   and   Mr.   Graciano   Gozon,   Senior   vice-­‐‑president   for   Finance   of   Republic  
Broadcasting  Corporation  (RBS)  discussed  the  terms  and  conditions  of  Vivas  offer  to  sell  the  104  films,  after  
the   rejection   of   the   same   package   by   ABS-­‐‑CBN.   After   several   negotiations   and   meetings,   Del   Rosario   and  
VIVA’s  President  Teresita  Cruz,  in  consideration  of  P60  million,  signed  a  letter  of  agreement  dated  April  24,  
1992,  granting  RBS  the  exclusive  right  to  air  104  Viva-­‐‑produced  and/or  acquired  films.  
 
ABS-­‐‑CBN  filed  before  the  RTC  a  complaint  for  specific  performance  with  a  prayer  for  a  writ  of  preliminary  
injunction   and/or   temporary   restraining   order   against   private   respondents   RBS.   As   counterclaim,   RBS  
prayed   for   the   award   of   moral   and   exemplary   damages   contending   that   its   reputation   has   been   debased   by  
ABS-­‐‑CBNs  acts  in  this  case.  When  RBS  was  not  able  to  fulfill  its  commitment  to  the  viewing  public  to  show  
the  film  Maging  Sino  Ka  Man  on  the  scheduled  dates  and  times  ,  it  suffered  serious  embarrassment  and  social  
humiliation.”  
 
RTC  dismissed  the  complaint  and  granted  the  award  of  damages  ratiocinating  that  there  was  no  perfected  
contract.  The  alleged  agreement  between  Lopez  III  and  Del  Rosario  was  subject  to  the  approval  of  the  VIVA  
Board  of  Directors,  and  said  agreement  was  disapproved  during  the  meeting  of  the  Board  on  7  April  1992.  
An   rendered   an   award   of   damages   stating   that   there   being   adequate   proof   of   the   pecuniary   loss   which   RBS  
has  suffered  as  a  result  of  the  filing  of  the  complaint  by  ABS-­‐‑CBN  and  that  the  latter  debased  RBS’s  reputation.  
 
CA  affirmed  RTC  decision  but  reduced  the  award  of  damages.  Hence,  this  appeal.  
 

   
3H  A.Y.  2017-­‐2018   111  
 
 

CORPORATION  LAW  CASE  DIGESTS  –  ATTY.  DANTE  DELA  CRUZ  

 
ISSUE:    Whether  RBS  is  entitled  to  moral  and  exemplary  damages  and  attorneys  fees.  
 
HELD:    NO.  
It   may   be   reiterated   that   the   claim   of   RBS   against   ABS-­‐‑CBN   is   not   based   on   contract,   quasi-­‐‑contract,  
delict,  or  quasi-­‐‑delict.  Hence,  the  claims  for  moral  and  exemplary  damages  can  only  be  based  on  Articles  19,  
20,  and  21  of  the  Civil  Code.  
Verily  then,  malice  or  bad  faith  is  at  the  core  of  Articles  19,  20,  and  21.  Malice  or  bad  faith  implies  a  
conscious  and  intentional  design  to  do  a  wrongful  act  for  a  dishonest  purpose  or  moral  obliquity.  Such  must  
be  substantiated  by  evidence.  There  is  no  adequate  proof  that  ABS-­‐‑CBN  was  inspired  by  malice  or  bad  faith.  It  
was  honestly  convinced  of  the  merits  of  its  cause  after  it  had  undergone  serious  negotiations  culminating  in  
its  formal  submission  of  a  draft  contract.  
 
As   regards   attorneys   fees,   the   law   is   clear   that   in   the   absence   of   stipulation,   attorneys   fees   may   be  
recovered   as   actual   or   compensatory   damages   under   any   of   the   circumstances   provided   for   in   Article   2208  
of  the  Civil  Code.  
 
 
 
82.  FILIPINAS  BROADCASTING  NETWORK,  INC.  VS.  AGO  MEDICAL  AND  EDUCATIONAL  CENTER-­‐‑BICOL  
CHRISTIAN  COLLEGE  OF  MEDICINE,  (AMEC-­‐‑BCCM)  AND  ANGELITA  F.  AGO  
G.R.  NO.  141994;  JANUARY  17,  2005  
CARPIO,  J.  
 
Doctrine:  A  juridical  person  is  generally  not  entitled  to  moral  damages  because,  unlike  a  natural  person,  it  
cannot   experience   physical   suffering   or   such   sentiments   as   wounded   feelings,   serious   anxiety,   mental  
anguish  or  moral  shock,  however,  a  corporation  may  have  a  good  reputation  which,  if  besmirched,  may  also  
be  a  ground  for  the  award  of  moral  damages.  
 
FACTS:  
"Exposé"  a  radio  documentary  program  hosted  by  Rima  and  Alegre,  is  aired  every  morning  over  DZRC-­‐‑AM  
which   is   owned   by   Filipinas   Broadcasting   Network,   Inc.   ("FBNI").   Rima   and   Alegre   exposed   various   alleged  
complaints  from  students,  teachers  and  parents  against  Ago  Medical  and  Educational  Center-­‐‑Bicol  Christian  
College   of   Medicine   ("AMEC")   and   its   administrators.   Claiming   that   the   broadcasts   were   defamatory,   AMEC  
and   Angelita   Ago   ("Ago"),   as   Dean,   filed   a   complaint   for   damages   against   FBNI,   Rima   and   Alegre.   AMEC   and  
Ago  included  FBNI  as  defendant  for  allegedly  failing  to  exercise  due  diligence  in  the  selection  and  supervision  
of   its   employees,   particularly   Rima   and   Alegre.   Thereafter,   trial   ensued.   FBNI   filed   a   separate   Answer  
claiming  that  it  exercised  due  diligence  in  the  selection  and  supervision  of  Rima  and  Alegre.  FBNI  claimed  
that  before  hiring  a  broadcaster.  The  trial  court  rendered  a  Decision  finding  FBNI  and  Alegre  liable  for  libel  
except  Rima.  
The   Court   of   Appeals   affirmed   the   trial   court’s   judgment   with   modification   that   FBNI,   Rima   and   Alegre  
solidarily  liable  to  pay  AMEC  moral  damages,  attorney’s  fees  and  costs  of  suit.  
 
ISSUE:  
Whether  or  not  AMEC,  being  a  corporation,  is  entitled  to  moral  damages.  
 
RULING:  
A   juridical   person   is   generally   not   entitled   to   moral   damages   because,   unlike   a   natural   person,   it   cannot  
experience  physical  suffering  or  such  sentiments  as  wounded  feelings,  serious  anxiety,  mental  anguish  or  
moral  shock.  The  Court  of  Appeals  cites  Mambulao  Lumber  Co.  v.  PNB,  et  al.  to  justify  the  award  of  moral  

   
3H  A.Y.  2017-­‐2018   112  
 
 

CORPORATION  LAW  CASE  DIGESTS  –  ATTY.  DANTE  DELA  CRUZ  

damages.   However,   the   Court’s   statement   in   Mambulao   that   "a   corporation   may   have   a   good   reputation  
which,  if  besmirched,  may  also  be  a  ground  for  the  award  of  moral  damages"  is  an  obiter  dictum.    
Nevertheless,   AMEC’s   claim   for   moral   damages   falls   under   item   7   of   Article   2219   of   the   Civil   Code.   This  
provision  expressly  authorizes  the  recovery  of  moral  damages  in  cases  of  libel,  slander  or  any  other  form  of  
defamation.  Article  2219(7)  does  not  qualify  whether  the  plaintiff  is  a  natural  or  juridical  person.  Therefore,  
a  juridical  person  such  as  a  corporation  can  validly  complain  for  libel  or  any  other  form  of  defamation  and  
claim  for  moral  damages.    
Moreover,  where  the  broadcast  is  libelous  per  se,  the  law  implies  damages.  In  such  a  case,  evidence  of  an  
honest  mistake  or  the  want  of  character  or  reputation  of  the  party  libeled  goes  only  in  mitigation  of  damages.  
Neither   in   such   a   case   is   the   plaintiff   required   to   introduce   evidence   of   actual   damages   as   a   condition  
precedent  to  the  recovery  of  some  damages.  In  this  case,  the  broadcasts  are  libelous  per  se.  Thus,  AMEC  is  
entitled  to  moral  damages.  
 
 
 
83.  NO  CASE  
 
 
 
84.  J.  F.  RAMIREZ  VS.  THE  ORIENTALIST  CO.,  AND  RAMON  J.  FERNANDEZ  
G.R.  NO.  11897  -­‐‑  SEPTEMBER  24,  1918  
STREET,  J;  
 
DOCTRINE:  If  a  corporation  knowingly  permits  one  of  its  officer,  or  any  other  agent,  to  do  acts  within  the  
scope  of  an  apparent  authority,  and  thus  hold  him  out  to  the  public  as  possessing  power  to  do  those  acts,  the  
corporation   will   as   against   anyone   who   has   in   good   faith   dealt   with   the   corporation   through   such   agent,   be  
estopped  from  denying  his  authority.  
 
FACTS:   The   Orientalist   Company   (Orientalist)   is   a   corporation   duly   organized   under   the   laws   of   the  
Philippine  and  was  engaged  in  the  business  of  maintaining  and  conducting  a  theatre  in  the  city  of  Manila  for  
the  exhibition  of  cinematographic  films.  Under  the  articles  of  incorporation,  the  company  is  authorized  to  
manufacture,  buy,  or  otherwise  obtain  all  accessories  necessary  for  conducting  such  a  business.  The  plaintiff  
J.   F.   Ramirez   was   a   resident   of   the   city   of   Paris,   France,   and   was   engaged   in   the   business   of   marketing   films  
for  a  manufacturer  or  manufacturers,  there  engaged  in  the  production  or  distribution  of  cinematographic  
material  and  was  represented  in  the  city  of  Manila  by  his  son,  Jose  Ramirez.  
 
In  July,  1913,  directors  of  the  Orientalist  became  apprised  of  the  fact  that  the  plaintiff  in  Paris  had  control  of  
the  agencies  for  two  different  marks  of  films,  namely,  the  "Eclair  Films"  and  the  "Milano  Films.”  Negotiations  
were  begun  with  Orientalist  by  Jose  Ramirez  for  the  purpose  of  placing  the  exclusive  agency  of  these  films  in  
the  hands  of  Orientalist.  Ramon  J.  Fernandez,  one  of  the  directors  of  Orientalist  and  also  its  treasure,  was  
chiefly   active   in   this   matter.   Jose   Ramirez   placed   in   the   hands   of   Ramon   J.   Fernandez   an   offer   stating   the  
terms   upon   which   the   plaintiff   would   undertake   to   supply   from   Paris   the   aforesaid   films.   This   offer   was  
declared  to  be  good  until  the  end  of  July.  Accordingly,  Ramon  J.  Fernandez,  with  approval  of  the  directors  
with  whom  he  had  communicated,  addressed  a  letter  to  Jose  Ramirez  accepting  the  offer  for  the  exclusive  
agency  of  the  Eclair  films  and,  a  few  days  later,  of  the  exclusive  agency  for  the  Milano  Films.  
 
The   memorandum   offer   contained   a   clause   in   which   J.   F.   Ramirez   described   his   function   as   that   of   a  
commission   agent   and   stated   that   he   would   see   to   the   prompt   shipment   of   the   films,   would   pay   the  
manufacturer,  and  take  care  that  the  films  were  insured  —  his  commission  for  such  services  being  fixed  at  5  
per  cent.  

   
3H  A.Y.  2017-­‐2018   113  
 
 

CORPORATION  LAW  CASE  DIGESTS  –  ATTY.  DANTE  DELA  CRUZ  

 
The  communications  were  signed  in  the  following  form,  in  which  it  will  be  noted  the  separate  signature  of  R.  
J.  Fernandez,  as  an  individual,  is  placed  somewhat  below  and  to  the  left  of  the  signature  of  the  Orientalist  
Company  as  singed  by  R.  J.  Fernandez,  in  the  capacity  of  treasurer:  
 
THE  ORIENTALIST  COMPANY,  
By  R.  J.  FERNANDEZ,  
Treasurer,  
R.  J.  FERNANDEZ.  
 
The   films   began   to   arrive   in   Manila.   It   appears   that   Orientalist   was   without   funds   to   meet   these   obligations  
and  the  first  few  drafts  were  accepted  in  the  name  of  the  Orientalist  Company  by  its  president  B.  Hernandez,  
and  were  taken  up  by  the  latter  with  his  own  funds.  There  arrived  in  Manila  several  remittances  of  films  from  
Paris,   and   it   is   these   shipments   which   have   given   occasion   for   the   present   action.   All   of   the   drafts  
accompanying  these  films  were  drawn,  as  on  former  occasions.  None  of  the  drafts  thus  accepted  were  taken  
up  by  the  drawee  or  by  B.  Hernandez  when  they  fell  due;  and  it  was  finally  necessary  for  the  plaintiff  himself  
to   take   them   up   as   dishonored   by   non-­‐‑payment.   Plaintiff   instituted   action   against   Orientalist,   and   Ramon   J.  
Fernandez.  In  the  judgment  of  the  trial  court,  the  Orientalist  Company  was  declared  to  be  a  principal  debtor  
and  Ramon  J.  Fernandez  was  declared  to  be  liable  subsidiarily  as  guarantor.  From  this  judgment  both  of  the  
parties  defendant  appealed.  
 
ISSUE:    
W/N   the   corporation   is   liable   upon   the   contracts   entered   into   with   J.   F.   Ramirez   for   the   exclusive   agency   of  
Éclair  and  Milano  films.  
W/N  Fernandez  is  liable  jointly  with  the  Orientalists  Company  as  a  principal  obligor  or  as  a  mere  guarantor.  
 
HELD:  
In  dealing  with  corporations  the  public  at  large  is  bound  to  rely  to  a  large  extent  upon  outward  appearances.  
If  a  man  is  found  acting  for  a  corporation  with  the  external  indicia  of  authority,  any  person,  not  having  notice  
of   want   of   authority,   may   usually   rely   upon   those   appearances;   and   if   it   be   found   that   the   directors   had  
permitted   the   agent   to   exercise   that   authority   and   thereby   held   him   out   as   a   person   competent   to   bind   the  
corporation,  or  had  acquiesced  in  a  contract  and  retained  the  benefit  supposed  to  have  been  conferred  by  it,  
the  corporation  will  be  bound,  notwithstanding  the  actual  authority  may  never  have  been  granted.  The  public  
is  not  supposed  nor  required  to  know  the  transactions  which  happen  around  the  table  where  the  corporate  
board  of  directors  or  the  stockholders  are  from  time  to  time  convoked.  Whether  a  particular  officer  actually  
possesses  the  authority  which  he  assumes  to  exercise  is  frequently  known  to  very  few,  and  the  proof  of  it  
usually  is  not  readily  accessible  to  the  stranger  who  deals  with  the  corporation  on  the  faith  of  the  ostensible  
authority  exercised  by  some  of  the  corporate  officers.  It  is  therefore  reasonable,  in  a  case  where  an  officer  of  
a   corporation   has   made   a   contract   in   its   name,   that   the   corporation   should   be   required,   if   it   denies   his  
authority,  to  state  such  defense  in  its  answer.  By  this  means  the  plaintiff  is  apprised  of  the  fact  that  the  agent's  
authority  is  contested;  and  he  is  given  an  opportunity  to  adduce  evidence  showing  either  that  the  authority  
existed  or  that  the  contract  was  ratified  and  approved.  
 
We   are   of   the   opinion   that   the   failure   of   the   defendant   corporation   to   make   any   issue   in   its   answer   with  
regard  to  the  authority  of  Ramon  J.  Fernandez  to  bind  it,  and  particularly  its  failure  to  deny  specifically  under  
oath   the   genuineness   and   due   execution   of   the   contracts   sued   upon,   have   the   effect   of   elimination   the  
question  of  his  authority  from  the  case,  considered  as  a  matter  of  mere  pleading.  The  statute  plainly  says  that  
if  a  written  instrument,  the  foundation  of  the  suit,  is  not  denied  upon  oath,  it  shall  be  deemed  to  be  admitted.  
It  is  familiar  doctrine  that  an  admission  made  in  a  pleading  can  not  be  controverted  by  the  party  making  such  
admission;   and   all   proof   submitted   by   him   contrary   thereto   or   inconsistent   therewith   should   simply   be  

   
3H  A.Y.  2017-­‐2018   114  
 
 

CORPORATION  LAW  CASE  DIGESTS  –  ATTY.  DANTE  DELA  CRUZ  

ignored  by  the  court,  whether  objection  is  interposed  by  the  opposite  party  or  not.  We  can  see  no  reason  why  
a  constructive  admission,  created  by  the  express  words  of  the  statute,  should  be  considered  to  have  less  effect  
than  any  other  admission.  
 
This  Court,  however,  under  section  109  of  the  Code  of  Civil  Procedure,  has  authority  to  permit  the  answer  of  
the  defendant  to  be  amended;  and  if  we  believed  that  the  interests  of  justice  so  required,  we  would  either  
exercise   that   authority   or   remand   the   cause   for   a   new   trial   in   court   below.   As   will   appear   further   on   in   this  
opinion,   however,   we   think   that   the   interests   of   justice   will   best   be   promoted   by   deciding   the   case,   without  
more  ado,  upon  the  issues  presented  in  the  record  as  it  now  stands.  
 
It  must,  at  the  outset,  be  premised  that  Ramon  J.  Fernandez,  as  treasurer,  had  no  independent  authority  to  
bind  the  company  by  signing  its  name  to  the  letters  in  question.  It  is  declared  by  signing  its  name  to  the  letters  
in  question.  It  is  declared  in  section  28  of  the  Corporation  Law  that  corporate  power  shall  be  exercised,  and  
all  corporate  business  conducted  by  the  board  of  directors;  and  this  principle  is  recognized  in  the  by-­‐‑laws  of  
the  corporation  in  question  which  contain  a  provision  declaring  that  the  power  to  make  contracts  shall  be  
vested  in  the  board  of  directors.  It  is  true  that  it  is  also  declared  in  the  same  by-­‐‑laws  that  the  president  shall  
have  the  power,  and  it  shall  be  his  duty,  to  sign  contract;  but  this  has  reference  rather  to  the  formality  of  
reducing  to  proper  form  the  contract  which  are  authorized  by  the  board  and  is  not  intended  to  confer  an  
independent  power  to  make  contract  binding  on  the  corporation.  
 
Ramon  J.  Fernandez  was  the  particular  officer  and  member  of  the  board  of  directors  who  was  most  active  in  
the   effort   to   secure   the   films   for   the   corporation.   The   negotiations   were   conducted   by   him   with   the  
knowledge  and  consent  of  other  members  of  the  board;  and  the  contract  was  made  with  their  prior  approval.  
As   appears   from   the   papers   in   this   record,   Fernandez   was   the   person   to   who   keeping   was   confided   the  
printed  stationery  bearing  the  official  style  of  the  corporation,  as  well  as  rubber  stencil  with  which  the  name  
of  the  corporation  could  be  signed  to  documents  bearing  its  name.  
 
As   already   observed,   it   is   familiar   doctrine   that   if   a   corporation   knowingly   permits   one   of   its   officer,   or   any  
other   agent,   to   do   acts   within   the   scope   of   an   apparent   authority,   and   thus   hold   him   out   to   the   public   as  
possessing   power   to   do   those   acts,   the   corporation   will   as   against   any   one   who   has   in   good   faith   dealt   with  
the   corporation   through   such   agent,   be   estopped   from   denying   his   authority;   and   where   it   is   said   "if   the  
corporation   permits"   this   means   the   same   as   "if   the   thing   is   permitted   by   the   directing   power   of   the  
corporation."  
 
As  appears  upon  the  face  of  the  contracts,  the  signature  of  Fernandez,  in  his  individual  capacity,  is  not  in  line  
with   the   signature   of   the   Orientalist   Company,   but   is   set   off   to   the   left   of   the   company's   signature   and  
somewhat  who  sign  contracts  in  some  capacity  other  than  that  of  principal  obligor  to  place  their  signature  
alone  would  justify  a  court  in  holding  that  Fernandez  here  took  upon  himself  the  responsibility  of  a  guarantor  
rather   than   that   of   a   principal   obligor.   We   do,   however,   think,   that   the   form   in   which   the   contract   is   signed  
raises  a  doubt  as  to  what  the  real  intention  was;  and  we  feel  justified,  in  looking  to  the  evidence  to  discover  
that   intention.   In   this   connection   it   is   entirely   clear,   from   the   testimony   of   both   Ramirez   and   Ramon   J.  
Fernandez,  that  the  responsibility  of  the  latter  was  intended  to  be  that  of  guarantor.  There  is,  to  be  sure,  a  
certain  difference  between  these  witnesses  as  to  the  nature  of  this  guaranty,  inasmuch  as  Fernandez  would  
have   us   believe   that   his   name   was   signed   as   a   guaranty   that   the   contract   would   be   approved   by   the  
corporation,  while  Ramirez  says  that  the  name  was  put  on  the  contract  for  the  purpose  of  guaranteeing,  not  
the  approval  of  the  contract,  but  its  performance.  We  are  convinced  that  the  latter  was  the  real  intention  of  
the  contracting  parties.  
 
The  judgment  appealed  from  is  affirmed,  with  costs  equally  against  the  two  appellant.  So  ordered.  
 

   
3H  A.Y.  2017-­‐2018   115  
 
 

CORPORATION  LAW  CASE  DIGESTS  –  ATTY.  DANTE  DELA  CRUZ  

 
 
85.  LOPEZ  V.  ERICTA  
G.R.  NO.  L-­‐‑32991.  JUNE  29,  1972  
MAKALINTAL,  J.;  
 
 
Doctrine:   The   votes   of   abstention,   viewed   in   their   setting,   can   in   no   way   be   construed   as   votes   for  
confirmation  of  the  appointment.  
 
Facts:  
The  Board  of  Regents  met  on  May  26,  1970,  and  President  Lopez  submitted  to  it  the  ad  interim  appointment  
of  Dr.  Blanco  for  reconsideration.  The  minutes  of  that  meeting  disclose  that  "the  Board  voted  to  defer  action  
on  the  matter  in  view  of  the  objections  cited  by  Regent  Kalaw  based  on  the  petition  against  the  appointment,  
addressed  to  the  Board,  from  a  majority  of  the  faculty  and  from  a  number  of  alumni  .  .  ."  The  "deferment  for  
further   study"   having   been   approved,   the   matter   was   referred   to   the   Committee   on   Personnel.   The   opinion  
was  then  expressed  by  the  Chairman  of  the  Board  that  in  view  of  its  decision  to  defer  action  Dr.  Blanco's  
appointment  had  lapsed,  but  that  there  should  be  no  objection  to  another  ad  interim  appointment  in  favor  of  
Dr.  Blanco  pending  final  action  by  the  Board.  
 
Accordingly,  on  the  same  day,  President  Lopez  extended  another  ad  interim  appointment  to  her,  effective  
from   May   26,   1970   to   April   30,   1971,   with   the   same   conditions   as   the   first,   namely,   "unless   sooner  
terminated,  and  subject  to  the  approval  of  the  Board  of  Regents  and  to  pertinent  University  regulations."  
 
The  next  meeting  of  the  Board  of  Regents  was  held  on  July  9,  1970.  The  roll-­‐‑call  voting  on  which  the  Chairman  
of  the  Board  of  Regents  based  his  ruling  aforesaid  gave  the  following  results:  five  (5)  votes  in  favor  of  Dr.  
Blanco's   ad   interim   appointment,   three   (3)   votes   against,   and   four   (4)   abstentions   —   all   the   twelve  
constituting  the  total  membership  of  the  Board  of  the  time.  The  next  day,  Dr.  Blanco  addressed  a  letter  to  the  
Board  requesting  "a  reconsideration  of  the  interpretation  made  by  the  Board  as  to  the  legal  effect  of  the  vote  
of   five   in   favor,   three   against   and   four   abstentions   on   my   ad   interim   appointment."   On   August   18,   1970   Dr.  
Blanco  wrote  the  President  of  the  University,  protesting  the  appointment  of  Oseas  A.  del  Rosario  as  Officer-­‐‑
in-­‐‑Charge  of  the  College  of  Education.  Neither  communication  having  elicited  any  official  reply,  Dr.  Blanco  
went  to  the  Court  of  First  Instance  of  Quezon  City  on  a  petition  for  certiorari  and  prohibition  with  preliminary  
injunction,  the  decision  wherein  is  the  subject  of  the  present  appeal.  
 
Issue:  Whether  the  appointment  of  Del  Rosario  is  valid.  (Yes)  
 
Ratio:  
The   votes   of   abstention,   viewed   in   their   setting,   can   in   no   way   be   construed   as   votes   for  
confirmation   of   the   appointment.   There   can   be   no   doubt   whatsoever   as   to   the   decision   and  
recommendation   of   the   three   members   of   the   Personnel   Committee:   it   was   for   rejection   of   the  
appointment.  If  the  committee  opted  to  withdraw  the  recommendation  it  was  on  the  understanding  (also  
referred   to   in   the   record   as   gentlemen's   agreement)   that   the   President   would   balk   to   Dr.   Blanco   for   the  
purpose  of  having  her  appointment  withdrawn  in  order  to  save  them  from  embarrassment.  No  inference  
can   be   drawn   from   this   that   the   members   of   the   Personnel   Committee,   by   their   abstention,   intended   to  
acquiesce   in   the   action   taken   by   those   who   voted   affirmatively.   Neither,   for   that   matter,   can   such  
inference  be  drawn  from  the  abstention  that  he  was  abstaining  because  he  was  not  then  ready  to  make  a  
decision.  
All  arguments  on  the  legal  question  of  how  an  abstention  should  be  treated,  all  authorities  cited  
in   support   of   one   or   the   other   position,   become   academic   and   purposeless   in   the   face   of   the   fact   that  

   
3H  A.Y.  2017-­‐2018   116  
 
 

CORPORATION  LAW  CASE  DIGESTS  –  ATTY.  DANTE  DELA  CRUZ  

respondent   Dr.   Blanco   was   clearly   not   the   choice   of   a   majority   of   the   members   of   the   Board   of   Regents,  
as  unequivocally  demonstrated  by  the  transcript  of  the  proceedings.  This  fact  cannot  be  ignored  simply  
because  the  Chairman,  in  submitting  the  question  to  the  actual  vote,  did  not  frame  it  as  accurately  as  the  
preceding  discussion  called  for,  such  that  two  of  the  Regents  present  (Silva  and  Kalaw)  had  to  make  some  
kind  of  clarification.  
 
 
 
86.  EXPERTRAVEL  &  TOURS,  INC.,  V.  COURT  OF  APPEALS  AND  KOREAN  AIRLINES  
G.R.  NO.  152392  -­‐‑  MAY  26,  2005  
CALLEJO,  SR.,  J.  
 
DOCTRINE:  
The  authority  of  the  resident  agent  of  a  foreign  corporation  with  license  to  do  business  in  the  Philippines  is  
to   receive,   for   and   in   behalf   of   the   foreign   corporation,   services   and   other   legal   processes   in   all   actions   and  
other  legal  proceedings  against  such  corporation.  
 
FACTS:  
Korean  Airlines  (KAL)  is  a  corporation  established  and  registered  in  the  Republic  of  South  Korea  and  licensed  
to  do  business  in  the  Philippines.  Its  general  manager  in  the  Philippines  is  Suk  Kyoo  Kim,  while  its  appointed  
counsel  was  Atty.  Mario  Aguinaldo  and  his  law  firm.  
 
KAL,   through   Atty.   Aguinaldo,   filed   a   Complaint   against   Expertravel   and   Tours,   Inc.   (ETI)   with   the   (RTC)   of  
Manila,   for   the   collection   of   the   principal   amount   of   P260,150.00,   plus   attorney's   fees   and   exemplary  
damages.   The   verification   and   certification   against   forum   shopping   was   signed   by   Atty.   Aguinaldo,   who  
indicated  therein  that  he  was  the  resident  agent  and  legal  counsel  of  KAL  and  had  caused  the  preparation  of  
the  complaint.  
 
ETI  filed  a  motion  to  dismiss  the  complaint  on  the  ground  that  Atty.  Aguinaldo  was  not  authorized  to  execute  
the  verification  and  certificate  of  non-­‐‑forum  shopping  as  required  by  Section  5,  Rule  7  of  the  Rules  of  Court.  
KAL  opposed  the  motion,  contending  that  Atty.  Aguinaldo  was  its  resident  agent  and  was  registered  as  such  
with  the  Securities  and  Exchange  Commission  (SEC)  as  required  by  the  Corporation  Code  of  the  Philippines.  
It  was  further  alleged  that  Atty.  Aguinaldo  was  also  the  corporate  secretary  of  KAL.  Appended  to  the  said  
opposition  was  the  identification  card  of  Atty.  Aguinaldo,  showing  that  he  was  the  lawyer  of  KAL.  
 
ISSUE:  
Was  Atty.  Aguinaldo  authorized  to  execute  the  certificate  of  non-­‐‑forum  shopping  by  the  respondent's  Board  
of  Directors?  
 
HELD:  
NO.  There  was  no  allegation  that  Atty.  Aguinaldo  had  been  authorized  to  execute  the  certificate  of  non-­‐‑forum  
shopping  by  the  respondent's  Board  of  Directors;  moreover,  no  such  board  resolution  was  appended  thereto  
or  incorporated  therein.  
 
While   Atty.   Aguinaldo   is   the   resident   agent   of   the   respondent   in   the   Philippines,   this   does   not   mean   that   he  
is  authorized  to  execute  the  requisite  certification  against  forum  shopping.  Under  Section  127,  in  relation  to  
Section  128  of  the  Corporation  Code,  the  authority  of  the  resident  agent  of  a  foreign  corporation  with  license  
to  do  business  in  the  Philippines  is  to  receive,  for  and  in  behalf  of  the  foreign  corporation,  services  and  other  
legal  processes  in  all  actions  and  other  legal  proceedings  against  such  corporation,  thus:  
 

   
3H  A.Y.  2017-­‐2018   117  
 
 

CORPORATION  LAW  CASE  DIGESTS  –  ATTY.  DANTE  DELA  CRUZ  

SEC.   127.   Who   may   be   a   resident   agent.   —   A   resident   agent   may   either   be   an   individual   residing   in   the  
Philippines  or  a  domestic  corporation  lawfully  transacting  business  in  the  Philippines:  Provided,  That  in  the  
case  of  an  individual,  he  must  be  of  good  moral  character  and  of  sound  financial  standing.  
 
SEC.   128.   Resident   agent;   service   of   process.   —   The   Securities   and   Exchange   Commission   shall   require   as   a  
condition   precedent   to   the   issuance   of   the   license   to   transact   business   in   the   Philippines   by   any   foreign  
corporation   that   such   corporation   file   with   the   Securities   and   Exchange   Commission   a   written   power   of  
attorney   designating   some   persons   who   must   be   a   resident   of   the   Philippines,   on   whom   any   summons   and  
other   legal   processes   may   be   served   in   all   actions   or   other   legal   proceedings   against   such   corporation,   and  
consenting  that  service  upon  such  resident  agent  shall  be  admitted  and  held  as  valid  as  if  served  upon  the  
duly-­‐‑authorized  officers  of  the  foreign  corporation  as  its  home  office.  
 
Under  the  law,  Atty.  Aguinaldo  was  not  specifically  authorized  to  execute  a  certificate  of  non-­‐‑forum  shopping  
as  required  by  Section  5,  Rule  7  of  the  Rules  of  Court.  This  is  because  while  a  resident  agent  may  be  aware  of  
actions   filed   against   his   principal   (a   foreign   corporation   doing   business   in   the   Philippines),   such   resident  
may   not   be   aware   of   actions   initiated   by   its   principal,   whether   in   the   Philippines   against   a   domestic  
corporation  or  private  individual,  or  in  the  country  where  such  corporation  was  organized  and  registered,  
against  a  Philippine  registered  corporation  or  a  Filipino  citizen.  
 
The  respondent  knew  that  its  counsel,  Atty.  Aguinaldo,  as  its  resident  agent,  was  not  specifically  authorized  
to  execute  the  said  certification.  It  attempted  to  show  its  compliance  with  the  rule  subsequent  to  the  filing  of  
its   complaint   by   submitting,   on   March   6,   2000,   a   resolution   purporting   to   have   been   approved   by   its   Board  
of  Directors  during  a  teleconference  held  on  June  25,  1999,  allegedly  with  Atty.  Aguinaldo  and  Suk  Kyoo  Kim  
in  attendance.  However,  such  attempt  of  the  respondent  casts  veritable  doubt  not  only  on  its  claim  that  such  
a  teleconference  was  held,  but  also  on  the  approval  by  the  Board  of  Directors  of  the  resolution  authorizing  
Atty.  Aguinaldo  to  execute  the  certificate  of  nonforum  shopping.  
 
 
 
87.  CITIBANK  VS.  HON.  SEGUNDINO  G.  CHUA  
G.R.  NO.  102300.  MARCH  17,  1993.  
CAMPOS,  JR.,  J  
 
DOCTRINE:  In  the  corporate  hierarchy,  there  are  three  levels  of  control:  (1)  the  board  of  directors,  which  is  
responsible  for  corporate  policies  and  the  general  management  of  the  business  affairs  of  the  corporation;  (2)  
the   officers,   who   in   theory   execute   the   policies   laid   down   by   the   board,   but   in   practice   often   have   wide  
latitude  in  determining  the  course  of  business  operations;  and  (3)  the  stockholders  who  have  the  residual  
power  over  fundamental  corporate  changes,  like  amendments  of  the  articles  of  incorporation.  However,  just  
as  a  natural  person  may  authorize  another  to  do  certain  acts  in  his  behalf,  so  may  the  board  of  directors  of  a  
corporation  validly  delegate  some  of  its  functions  to  individual  officers  or  agents  appointed  by  it.  
 
FACTS:   Petitioner   is   a   foreign   commercial   banking   corporation   duly   licensed   to   do   business   in   the  
Philippines.   Private   respondents,   spouses   Cresencio   and   Zenaida   Velez,   were   good   clients   of   petitioner  
bank's   branch   in   Cebu   until   March   14,   1986   when   they   filed   a   complaint   for   specific   performance   and  
damages  against  it.  
 
On   March   30,   1990,   the   date   of   the   pre-­‐‑trial   conference,   counsel   for   petitioner   bank   appeared,   presenting   a  
special  power  of  attorney  executed  by  Citibank  officer  Florencia  Tarriela  in  favor  of  petitioner  bank's  counsel,  
the  J.P.  Garcia  &  Associates,  to  represent  and  bind  petitioner  bank  at  the  pre-­‐‑trial  conference.    
 

   
3H  A.Y.  2017-­‐2018   118  
 
 

CORPORATION  LAW  CASE  DIGESTS  –  ATTY.  DANTE  DELA  CRUZ  

Inspite  of  this  special  power  of  attorney,  counsel  for  private  respondents  orally  moved  to  declare  petitioner  
bank   as   in   default   on   the   ground   that   the   special   power   of   attorney   was   not   executed   by   the   Board   of  
Directors  of  Citibank.  Petitioner  bank  was  then  required  to  file  a  written  opposition  to  this  oral  motion  to  
declare  it  as  in  default.  In  said  opposition  petitioner  bank  attached  another  special  power  of  attorney  made  
by  William  W.  Ferguson,  Vice  President  and  highest  ranking  officer  of  Citibank,  Philippines,  constituting  and  
appointing  the  J.P.  Garcia  &  Associates  to  represent  and  bind  the  BANK  at  the  pre-­‐‑trial  conference  and/or  
trial  of  the  case.  Respondent  judge  denied  private  respondents'  oral  motion  to  declare  petitioner  bank  as  in  
default  and  set  the  continuation  of  the  pre-­‐‑trial  conference.  
 
On  the  scheduled  pre-­‐‑trial  conference,  private  respondents  reiterated,  by  way  of  asking  for  reconsideration,  
their  oral  motion  to  declare  petitioner  bank  as  in  default.  Petitioner  bank  again  filed  on  May  14,  1990  its  
opposition  thereto,  stating  as  follows:  
".  .  .  While  it  has  been  the  practice  of  Citibank  to  appoint  its  counsels  as  its  attorney-­‐‑in-­‐‑fact  in  civil  
cases   because   it   considers   said   counsels   equivalent   to   a   Citibank   employee,   yet,   in   order   to   avoid   further  
arguments  on  the  matter,  the  defendant  Citibank  will  secure  another  power  of  attorney  from  Mr.  William  W.  
Ferguson  in  favor  of  its  employee/s  who  will  represent  the  defendant  Citibank  in  the  pre-­‐‑trial  conferences  
of  this  case.”  
 
In  compliance  with  the  above  promise,  petitioner  bank  filed  a  manifestation,  dated  May  23,  1990,  attaching  
therewith  a  special  power  of  attorney  executed  by  William  W.  Ferguson  in  favor  of  Citibank  employees  to  
represent   and   bind   Citibank   on   the   pre-­‐‑trial   conference   of   the   case.   Respondent   judge   issued   an   order  
declaring  petitioner  bank  as  in  default  citing  the  following  reasons:      
"Defendant-­‐‑bank,   although   a   foreign   corporation,   is   bound   by   Philippine   laws   when   doing   and  
conducting   business   in   the   Philippines   (Sec.   129,   B.P.   Blg.   68),   and   its   corporate   powers   could   only   be  
exercised  by  its  Board    
of  Directors  (Sec.  23,  B.P.  Blg.  68)”  
 
The  alleged  authority  (Special  Power  of  Attorney,  Annex  "A")  executed  by  Mr.  William  W.  Ferguson  in  favor  
of  the  alleged  Citibank  employees,  assuming  the  same  to  be  a  delegable  authority,  to  represent  the  defendant  
in   the   pre-­‐‑trial   conference,   made   no   mention   of   J.P.   Garcia   &   Associates   as   one   of   the   employees   of   the  
defendant.  It  stands  to  reason  therefore,  that  the  defendant-­‐‑bank  has  no  proper  representation  during  the  
pre-­‐‑trial  conference.  
 
Petitioner  bank  then  filed  a  petition  for  certiorari,  prohibition  and  mandamus  with  preliminary  injunction  
and/or   temporary   restraining   order   with   the   Court   of   Appeals.   On   June   26,   1991,   the   Court   of   Appeals  
dismissed  the  petition  
 
ISSUE:  Whether  or  not  a  resolution  of  the  board  of  directors  of  a  corporation  is  always  necessary  for  granting  
authority  to  an  agent  to  represent  the  corporation  in  court  cases?  
 
HELD:  No.  As  a  general  rule,  all  corporate  powers  are  to  be  exercised  by  the  board  of  directors,  exceptions  
are   made   where   the   Code   provides   otherwise.   Section   25   of   said   Code   provides   that   the   directors   of   the  
corporation  shall  elect  its  corporate  officers,  and  further  provides  as  follows:  
"SEC.  25.  Corporate  officers;  quorum.  —  .  .  .  The  directors  or  trustees  and  officers  to  be  elected  shall  
perform  the  duties  enjoined  on  them  by  law  and  by  the  by-­‐‑laws  of  the  corporation  .  .  ."  
 
Furthermore,  Section  47  of  the  same  Code  enumerates  what  may  be  contained  in  the  by-­‐‑laws,  among  which  
is   a   provision   for   the   "qualifications,   duties   and   compensation   of   directors   or   trustees,   officers   and  
employees".  
 

   
3H  A.Y.  2017-­‐2018   119  
 
 

CORPORATION  LAW  CASE  DIGESTS  –  ATTY.  DANTE  DELA  CRUZ  

Taking   all   the   above   provisions   of   law   together,   it   is   clear   that   corporate   powers   may   be   directly   conferred  
upon  corporate  officers  or  agents  by  statute,  the  articles  of  incorporation,  the  by-­‐‑laws  or  by  resolution  or  
other  act  of  the  board  of  directors.  In  addition,  an  officer  who  is  not  a  director  may  also  appoint  other  agents  
when   so   authorized   by   the   by-­‐‑laws   or   by   the   board   of   directors.   Such   are   referred   to   as   express   powers.  
There  are  also  powers  incidental  to  express  powers  conferred.  It  is  a  fundamental  principle  in  the  law  of  
agency  that  every  delegation  of  authority,  whether  general  or  special,  carries  with  it,  unless  the  contrary  be  
expressed,  implied  authority  to  do  all  of  those  acts,  naturally  and  ordinarily  done  in  such  cases,  which  are  
reasonably  necessary  and  proper  to  be  done  in  order  to  carry  into  effect  the  main  authority  conferred.  
 
Since  the  by-­‐‑laws  are  a  source  of  authority  for  corporate  officers  and  agents  of  the  corporation,  a  resolution  
of   the   Board   of   Directors   of   Citibank   appointing   an   attorney   in   fact   to   represent   and   bind   it   during   the   pre-­‐‑
trial  conference  of  the  case  at  bar  is  not  necessary  because  its  by-­‐‑laws  allow  its  officers,  the  Executing  Officer  
and   the   Secretary   Pro-­‐‑Tem,   **   to   execute   a   power   of   attorney   to   a   designated   bank   officer,   William   W.  
Ferguson  in  this  case,  clothing  him  with  authority  to  direct  and  manage  corporate  affairs.  
 
Since  the  general  power  of  attorney  granted  to  Ferguson  allows  him  to  delegate  his  powers  in  whole  or  in  
part,  there  can  be  no  doubt  that  the  special  power  of  attorney  in  favor,  first,  of  J.P.  Garcia  &  Associates  and  
later,  of  the  bank's  employees,  constitutes  a  valid  delegation  of  Ferguson's  express  power  (under  paragraph  
XVII  above)  to  represent  petitioner  bank  in  the  pre-­‐‑trial  conference  in  the  lower  court.  
 
From  the  outset,  petitioner  bank  showed  a  willingness,  if  not  zeal,  in  pursuing  and  defending  this  case.  It  even  
acceded  to  private  respondent's  insistence  on  the  question  of  proper  representation  during  the  pre-­‐‑trial  by  
presenting  not  just  one,  but  three,  special  powers  of  attorney.  Initially,  the  special  power  of  attorney  was  
executed   by   Florencia   Tarriela   in   favor   of   J.P.   Garcia   &   Associates,   petitioner   bank's   counsel.   Private  
respondents  insisted  that  this  was  not  proper  authority  required  by  law.  To  avoid  further  argument,  a  second  
special   power   of   attorney   was   presented   by   petitioner   bank,   executed   by   William   W.   Fersugon,   the   highest  
ranking  officer  of  Citibank  in  the  Philippines,  in  favor  of  its  counsel  J.P.  Garcia  &  Associates.  But  since  the  
authority  to  delegate  of  William  A.  Fersugon  in  favor  of  an  agent  is  limited  to  bank  employees,  another  special  
power   of   attorney   from   Wiliam   W.   Fersugon   in   favor   of   the   Citibank   employees   was   presented.   But   the  
respondent  trial  court  judge  disregarded  all  these  and  issued  the  assailed  default  order.  There  is  nothing  to  
show   that   petitioner   bank   "miserably   failed   to   oblige";   on   the   contrary,   three   special   powers   of   attorney  
manifest  prudence  and  diligence  on  petitioner  bank's  part.  
 
In   fact,   there   was   no   need   for   the   third   power   of   attorney   because   we   believe   that   the   second   power   of  
attorney  was  sufficient  under  the  by-­‐‑law  provision  authorizing  Fersugon  to  delegate  any  of  his  functions  to  
any   one   or   more   employees   of   the   petitioner   bank.   A   reasonable   interpretation   of   this   provision   would  
include  an  appointment  of  a  legal  counsel  to  represent  the  bank  in  court,  for,  under  the  circumstances,  such  
legal  counsel  can  be  considered,  and  in  fact  was  considered  by  the  petitioner  bank,  an  employee  for  a  special  
purpose.   Furthermore,   Fersugon,   who   heads   the   Philippine   office   thousands   of   miles   away   from   its   main  
office  in  the  United  States,  must  be  understood  to  have  sufficient  powers  to  act  promptly  in  order  to  protect  
the  interests  of  his  principal.  
 
 
 
88.  NO  CASE  
 
 
89.  EPG  CONSTRUCTION  COMPANY,  INC  VS.  CA  
GR  NO.  103372/  JUNE  22,  1992  
CRUZ,  J.  

   
3H  A.Y.  2017-­‐2018   120  
 
 

CORPORATION  LAW  CASE  DIGESTS  –  ATTY.  DANTE  DELA  CRUZ  

 
DOCTRINE:  It  is  settled  that  a  corporation  is  invested  by  law  with  a  personality  separate  and  distinct  from  
those  of  the  persons  composing  it  as  well  as  from  that  of  any  other  entity  to  which  it  may  be  related.    
 
FACTS:  Petitioner  and  the  University  of  the  Philippines,  herein  private  respondent,  entered  into  a  contract  
for   the   construction   of   the   UP   Law   Library   Building   for   P7,545,000.00.   The   agreement   included,   among  
others,  that  the  CONTRACTOR  guarantees  that  the  work  completed  under  the  contract  shall  be  in  accordance  
with  the  plans  and  specification  prepared  by  ARCHITECT  and  the  CONTRACTOR  shall  repair  at  his  own  cost  
for  a  period  of  1  year  from  date  of  substantial  completion  and  acceptance  of  the  work  by  the  OWNER,  all  the  
work  covered  under  the  contract  that  may  prove  defective  except  maintenance  works.    
 
Upon   its   completion,   the   building   was   formally   turned   over   to   the   private   respondent.   UP   issued   a  
certification   of   acceptance   dated   January   13,   1983.   Sometime   in   July   1983,   the   private   respondent  
complained  that  6  air-­‐‑conditioning  units  on  the  third  floor  of  the  building  were  not  cooling  properly.  After  
inspection  of  the  equipment,  EPG  agreed  to  shoulder  the  expenses  for  their  repair.  For  whatever  reason,  the  
repair   was   never   undertaken.   UP   repeated   its   complaints   to   EPG,   which   again   sent   its   representatives   to  
assess   the   defects.   Finally,   it   made   UP   a   written   offer   to   repair   the   system   for   P194,000.00.   UP   insisted   that  
EPG   was   obligated   to   repair   the   defects   at   its   own   expense   under   the   guarantee   provision   in   their   contract.  
UP  contracted  with  another  company,  which  repaired  the  defects.  
 
UP  subsequently  demanded  from  EPG  reimbursement  of  the  said  amount  plus  liquidated  damages.  When  the  
demand  was  rejected,  UP  sued  EPG  and  its  president,  Emmanuel  P.  de  Guzman  before  the  RTC.  Judgment  was  
rendered   requiring   both   defendants   jointly   and   severally   to   pay   the   plaintiff   actual   damages,   liquidated  
damages,   attorney's   fees   and   costs.   EPG   appealed   to   the   CA,   which   sustained   the   trial   court.   Hence,   the  
recourse  to  the  SC  alleging  that  1)  UP  was  estopped  by  its  certificate  of  acceptance  from  imputing  liability  to  
EPG   for   the   defects;   2)   the   defects   were   due   to   force   majeure   or   fortuitous   event;   and   3)   Emmanuel   de  
Guzman  has  a  separate  personality  from  that  of  EPG  Construction  Co.,  Inc.  
 
ISSUE:  Whether  or  not  de  Guzman  has  a  separate  legal  personality  from  EPG  Construction  Co.,  Inc.  and  should  
not  be  held  solidarity  liable  with  it.    
 
HELD:  YES.  When  Emmanuel  de  Guzman  moved  to  dismiss  the  complaint  as  to  him,  UP  said  in  its  opposition  
to  the  motion  that  it  was  suing  him  "in  his  official  capacity  and  not  in  his  personal  capacity."  His  inclusion  as  
President  of  the  company  was  therefore  superfluous,  as  De  Guzman  correctly  contended,  because  his  acts  as  
such   were   corporate   acts   imputable   to   EPG   itself   as   his   principal.   It   is   settled   that   a   corporation   is   invested  
by   law   with   a   personality   separate   and   distinct   from   those   of   the   persons   composing   it   as   well   as   from   that  
of  any  other  entity  to  which  it  may  be  related.    
 
The  exception  noted  is  where  the  official  "had  acted  maliciously  or  in  bad  faith,"  in  which  event  he  may  be  
made  personally  liable  for  his  own  act.  That  exception  is  not  applicable  in  the  case  at  bar,  because  it  has  not  
been  proved  that  De  Guzman  acted  maliciously  or  in  bad  faith  when,  as  President  of  EPG,  he  sought  to  protect  
its  interests  and  resisted  UP's  claims.  Whatever  damage  was  caused  to  UP  as  a  result  of  his  acts  is  the  sole  
responsibility  of  EPG  even  though  De  Guzman  was  its  principal  officer  and  controlling  stockholder.  
 
In  sum,  we  hold  that  the  lower  court  did  not  err  in  holding  EPG  liable  for  the  repair  of  the  air-­‐‑conditioning  
system  at  its  expense  pursuant  to  the  guarantee  provision  in  the  construction  contract  with  UP.  However,  
Emmanuel   de   Guzman   is   not   solidarily   liable   with   it,   having   acted   on   its   behalf   within   the   scope   of   his  
authority  and  without  any  demonstrated  malice  or  bad  faith.  
 
 

   
3H  A.Y.  2017-­‐2018   121  
 
 

CORPORATION  LAW  CASE  DIGESTS  –  ATTY.  DANTE  DELA  CRUZ  

90.  BENGUET  COOPERATIVE  V  NLRC  


 
 
 
91.  WOODCHILD  HOLDINGS,  INC.,  VS.  CALLEJO,  SR.,  TINGA  AND  ROXAS  ELECTRIC  AND    
CONSTRUCTION  COMPANY,  INC.,  
G.R.  NO.  140667  AUGUST  12,  2004  
CALLEJO,  SR.,  J.:  
 
   
FACTS:  The  respondent  Roxas  Electric  and  Construction  Company,  Inc.  (RECCI  was  the  owner  of  two  parcels  
of  land,  identified  as  Lot  No.  491-­‐‑A-­‐‑3-­‐‑B-­‐‑1  and  Lot  No.  491-­‐‑A-­‐‑3-­‐‑B-­‐‑2.  At  a  special  meeting,  the  respondents  
Board   of   Directors   approved   a   resolution   authorizing   the   corporation,   through   its   president,   Roberto   B.  
Roxas,  to  sell  Lot  No.  491-­‐‑A-­‐‑3-­‐‑B-­‐‑2  at  a  price  and  under  such  terms  and  conditions  which  he  deemed  most  
reasonable  and  advantageous  to  the  corporation.    
   
Petitioner   Woodchild   Holdings,   Inc.   (WHI)   wanted   to   buy   the   said   lot   which   it   planned   to   construct   its  
warehouse  building,  and  a  portion  of  the  adjoining  lot,  Lot  No.  491-­‐‑A-­‐‑3-­‐‑B-­‐‑1,  so  that  its  45-­‐‑foot  container  van  
would   be   able   to   readily   enter   or   leave   the   property.   In   a   Letter   to   Roxas,   WHI   President   Jonathan   Y.   Dy  
offered  to  buy  Lot  No.  491-­‐‑A-­‐‑3-­‐‑B-­‐‑2  Roxas  indicated  his  acceptance  of  the  offer  on  the  deed.  Less  than  a  month  
later  Roxas,  as  President  of  RECCI  and  Dy,  as  President  of  WHI  executed  a  contract  to  sell.  A  Deed  of  Absolute  
Sale   in   favor   of   WHI   was   then   issued,   receipt   of   which   was   acknowledged   by   Roxas   under   the   terms   and  
conditions  that  the  Vendor  agrees  and  binds  itself  to  give  Vendee  the  beneficial  use  of  and  a  right  of  way  from  
Sumulong  Highway  to  the  property  herein  conveyed  consists  of  25  square  meters  wide  to  be  used  as  the  
latters  egress  from  and  ingress  to  and  an  additional  25  square  meters  in  the  corner  of  Lot  No.  491-­‐‑A-­‐‑3-­‐‑B-­‐‑1,  
as  turning  and/or  maneuvering  area  for  Vendees  vehicles.  
 
Thereafter,  WHI  complained  to  Roberto  Roxas  that  the  vehicles  of  RECCI  were  parked  on  a  portion  of  the  
property  over  which  WHI  had  been  granted  a  right  of  way.  Dy  and  Roxas  discussed  the  need  of  the  WHI  to  
buy  a  500-­‐‑square-­‐‑meter  portion  of  Lot  No.  491-­‐‑A-­‐‑3-­‐‑B-­‐‑1  as  provided  for  in  the  deed  of  absolute  sale.  However,  
Roxas   died   soon   thereafter.   Then,   the   WHI   wrote   the   RECCI,   reiterating   its   verbal   requests   to   purchase   a  
portion  of  the  said  lot  as  provided  for  in  the  deed  of  absolute  sale.  
   
The  WHI  demanded  that  the  RECCI  sell  a  portion  of  Lot  No.  491-­‐‑A-­‐‑3-­‐‑B-­‐‑1  for  its  beneficial  use  within  72  hours  
from   notice   thereof,   otherwise   the   appropriate   action   would   be   filed   against   it.   RECCI   rejected   the   demand  
of  WHI.  WHI  reiterated  its  demand  but  there  was  no  response  from  RECCI.  WHI  filed  a  complaint  against  the  
RECCI,  for  specific  performance  and  damages,  and  alleged  that  defendant  Roxas  Electric  is  in  patent  violation  
of   the   express   and   valid   terms   of   the   Deed   of   Absolute   Sale   unjustifiably   refused   to   deliver  to   Woodchild  
Holdings  the  stipulated  beneficial  use  and  right  of  way  consisting  of  25  square  meters  and  55  square  meters  
to  the  prejudice  of  the  plaintiff.  It  contends  that  the  respondent  never  objected  to  Roxas  acceptance  of  its  
offer  to  purchase  the  property  and  the  terms  and  conditions  therein;  the  respondent  even  allowed  Roxas  to  
execute  the  deed  of  absolute  sale  in  its  behalf.  The  petitioner  notes  that  the  respondents  Board  of  Directors  
never  approved  any  resolution  rejecting  the  deed  of  absolute  sale  executed  by  Roxas  for  and  in  its  behalf.  
 
In  its  answer  to  the  complaint,  the  RECCI  alleged  that  it  never  authorized  its  former  president,  Roberto  Roxas,  
to  grant  the  beneficial  use  of  any  portion  of  Lot  No.  491-­‐‑A-­‐‑3-­‐‑B-­‐‑1,  nor  agreed  to  sell  any  portion  thereof  or  
create  a  lien  or  burden  thereon.  It  alleged  that,  under  the  approved  Resolution  it  merely  authorized  Roxas  to  
sell  Lot  No.  491-­‐‑A-­‐‑3-­‐‑B-­‐‑2.  As  such,  the  grant  of  a  right  of  way  and  the  agreement  to  sell  a  portion  of  Lot  No.  
491-­‐‑A-­‐‑3-­‐‑B-­‐‑1  in  the  said  deed  are  ultra  vires.    
   

   
3H  A.Y.  2017-­‐2018   122  
 
 

CORPORATION  LAW  CASE  DIGESTS  –  ATTY.  DANTE  DELA  CRUZ  

 
ISSUES:   Whether   the   respondent   is   bound   by   the   provisions   in   the   deed   of   absolute   sale   granting   to   the  
petitioner  beneficial  use  and  a  right  of  way  over  a  portion  of  Lot  No.  491-­‐‑A-­‐‑3-­‐‑B-­‐‑1.      
 
HELD:  No.  Roxas  was  not  specifically  authorized  under  the  approved  resolution  of  the  Board  of  Directors  to  
grant  a  right  of  way  in  favor  of  the  petitioner  on  a  portion  of  Lot  No.  491-­‐‑A-­‐‑3-­‐‑B-­‐‑1  or  to  agree  to  sell  to  the  
petitioner  a  portion  thereof.  The  authority  of  Roxas,  under  the  resolution,  to  sell  Lot  No.  491-­‐‑A-­‐‑3-­‐‑B-­‐‑2  did  not  
include  the  authority  to  sell  a  portion  of  the  adjacent  lot,  Lot  No.  491-­‐‑A-­‐‑3-­‐‑B-­‐‑1,  or  to  create  or  convey  real  
rights  thereon.  Neither  may  such  authority  be  implied  from  the  authority  granted  to  Roxas  to  sell  Lot  No.  
491-­‐‑A-­‐‑3-­‐‑B-­‐‑2   to   the   petitioner   on   such   terms   and   conditions   which   he   deems   most   reasonable   and  
advantageous.   Under   paragraph   12,   Article   1878   of   the   New   Civil   Code,   a   special   power   of   attorney   is  
required  to  convey  real  rights  over  immovable  property.  Article  1358  of  the  New  Civil  Code  requires  that  
contracts   which   have   for   their   object   the   creation   of   real   rights   over   immovable   property   must   appear   in   a  
public   document.   The   petitioner   cannot   feign   ignorance   of   the   need   for   Roxas   to   have   been   specifically  
authorized   in   writing   by   the   Board   of   Directors   to   be   able   to   validly   grant   a   right   of   way   and   agree   to   sell   a  
portion   of   Lot   No.   491-­‐‑A-­‐‑3-­‐‑B-­‐‑1.   The   rule   is   that   if   the   act   of   the   agent   is   one   which   requires   authority   in  
writing,  those  dealing  with  him  are  charged  with  notice  of  that  fact.  
   
NOTE:   The   Court   rejected   the   petitioner’s   submission   that,   in   allowing   Roxas   to   execute   the   contract   to   sell  
and  the  deed  of  absolute  sale  and  failing  to  reject  or  disapprove  the  same,  the  respondent  thereby  gave  him  
apparent  authority  to  grant  a  right  of  way  over  Lot  No.  491-­‐‑A-­‐‑3-­‐‑B-­‐‑1  and  to  grant  an  option  for  the  respondent  
to  sell  a  portion  thereof  to  the  petitioner.  Absent  estoppel  or  ratification,  apparent  authority  cannot  remedy  
the  lack  of  the  written  power  required  under  the  statement  of  frauds.  In  addition,  the  petitioners  fallacy  is  its  
wrong   assumption   of   the   unproved   premise   that   the   respondent   had   full   knowledge   of   all   the   terms   and  
conditions  contained  in  the  deed  of  absolute  sale  when  Roxas  executed  it.  There  can  be  no  apparent  authority  
of  an  agent  without  acts  or  conduct  on  the  part  of  the  principal  and  such  acts  or  conduct  of  the  principal  must  
have  been  known  and  relied  upon  in  good  faith  and  as  a  result  of  the  exercise  of  reasonable  prudence  by  a  
third  person  as  claimant  and  such  must  have  produced  a  change  of  position  to  its  detriment.  The  apparent  
power  of  an  agent  is  to  be  determined  by  the  acts  of  the  principal  and  not  by  the  acts  of  the  agent.  
   
For  the  principle  of  apparent  authority  to  apply,  the  petitioner  was  burdened  to  prove  the  following:  (a)  the  
acts   of   the   respondent   justifying   belief   in   the   agency   by   the   petitioner;   (b)   knowledge   thereof   by   the  
respondent   which   is   sought   to   be   held;   and,   (c)   reliance   thereon   by   the   petitioner   consistent   with   ordinary  
care   and   prudence.   In   this   case,   there   is   no   evidence   on   record   of   specific   acts   made   by   the   respondent  
showing  or  indicating  that  it  had  full  knowledge  of  any  representations  made  by  Roxas  to  the  petitioner  that  
the   respondent   had   authorized   him   to   grant   to   the   respondent   an   option   to   buy   a   portion   of   Lot   No.   491-­‐‑A-­‐‑
3-­‐‑B-­‐‑1  covered  by  TCT  No.  78085,  or  to  create  a  burden  or  lien  thereon,  or  that  the  respondent  allowed  him  
to  do  so.  
   
The   petitioners   contention   that   by   receiving   and   retaining   the   P5,000,000   purchase   price   of   Lot   No.   491-­‐‑A-­‐‑
3-­‐‑B-­‐‑2,  the  respondent  effectively  and  impliedly  ratified  the  grant  of  a  right  of  way  on  the  adjacent  lot,  Lot  No.  
491-­‐‑A-­‐‑3-­‐‑B-­‐‑1,  and  to  grant  to  the  petitioner  an  option  to  sell  a  portion  thereof,  is  barren  of  merit.  It  bears  
stressing  that  the  respondent  sold  Lot  No.  491-­‐‑A-­‐‑3-­‐‑B-­‐‑2  to  the  petitioner,  and  the  latter  had  taken  possession  
of  the  property.  As  such,  the  respondent  had  the  right  to  retain  the  P5,000,000,  the  purchase  price  of  the  
property   it   had   sold   to   the   petitioner.   For   an   act   of   the   principal   to   be   considered   as   an   implied   ratification  
of  an  unauthorized  act  of  an  agent,  such  act  must  be  inconsistent  with  any  other  hypothesis  than  that  he  
approved   and   intended   to   adopt   what   had   been   done   in   his   name.   Ratification   is   based   on   waiver   the  
intentional  relinquishment  of  a  known  right.  Ratification  cannot  be  inferred  from  acts  that  a  principal  has  a  
right  to  do  independently  of  the  unauthorized  act  of  the  agent.  Moreover,  if  a  writing  is  required  to  grant  an  
authority  to  do  a  particular  act,  ratification  of  that  act  must  also  be  in  writing.  Since  the  respondent  had  not  

   
3H  A.Y.  2017-­‐2018   123  
 
 

CORPORATION  LAW  CASE  DIGESTS  –  ATTY.  DANTE  DELA  CRUZ  

ratified  the  unauthorized  acts  of  Roxas,  the  same  are  unenforceable.  Hence,  by  the  respondents  retention  of  
the  amount,  it  cannot  thereby  be  implied  that  it  had  ratified  the  unauthorized  acts  of  its  agent,  Roberto  Roxas.  
 
 
 
92.  YU  CHUCK  V.  KONG  LI  PO  
 
 
 
93.  LAPULAPU  FOUNDATION  V.  TAN  
G.R.  NO.  126006.  JANUARY  29,  2004  
CALLEJO,  SR.,  J.:  
 
Doctrine:   It   is   a   familiar   doctrine   that   if   a   corporation   knowingly   permits   one   of   its   officers,   or   any   other  
agent,  to  act  within  the  scope  of  an  apparent  authority,  it  holds  him  out  to  the  public  as  possessing  the  power  
to  do  those  acts;  and  thus,  the  corporation  will,  as  against  anyone  who  has  in  good  faith  dealt  with  it  through  
such  agent,  be  estopped  from  denying  the  agents  authority.  
 
Facts:  Sometime  in  1977,  petitioner  Elias  Q.  Tan,  then  President  of  the  co-­‐‑petitioner  Lapulapu  Foundation,  
Inc.,  obtained  four  loans  from  the  respondent  Allied  Banking  Corporation  covered  by  four  promissory  notes.  
As   of   January   23,   1979,   the   entire   obligation   become   due   and   despite   demands   made   on   them   by   the  
respondent   Bank,   the   petitioners   failed   to   pay   the   same.   In   its   answer   to   the   complaint,   the   petitioner  
Foundation  denied  incurring  indebtedness  from  the  respondent  Bank  alleging  that  the  loans  were  obtained  
by  petitioner  Tan  in  his  personal  capacity,  for  his  own  use  and  benefit  and  on  the  strength  of  the  personal  
information   he   furnished   the   respondent   Bank.   The   petitioner   Foundation   maintained   that   it   never  
authorized   petitioner   Tan   to   co-­‐‑sign   in   his   capacity   as   its   President   any   promissory   note   and   that   the  
respondent  Bank  fully  knew  that  the  loans  contracted  were  made  in  petitioner  Tans  personal  capacity  and  
for  his  own  use  and  that  the  petitioner  Foundation  never  benefited,  directly  or  indirectly,  therefrom.    
 
For  his  part,  petitioner  Tan  admitted  that  he  contracted  the  loans  from  the  respondent  Bank  in  his  personal  
capacity.  The  parties,  however,  agreed  that  the  loans  were  to  be  paid  from  the  proceeds  of  petitioner  Tans  
shares  of  common  stocks  in  the  Lapulapu  Industries  Corporation,  a  real  estate  firm.  The  loans  were  covered  
by  promissory  notes  which  were  automatically  renewable  (rolled-­‐‑over)  every  year  at  an  amount  including  
unpaid  interests,  until  such  time  as  petitioner  Tan  was  able  to  pay  the  same  from  the  proceeds  of  his  aforesaid  
shares.  
 
According   to   petitioner   Tan,   the   respondent   Banks   employee   required   him   to   affix   two   signatures   on   every  
promissory   note,   assuring   him   that   the   loan   documents   would   be   filled   out   in   accordance   with   their  
agreement.  However,  after  he  signed  and  delivered  the  loan  documents  to  the  respondent  Bank,  these  were  
filled  out  in  a  manner  not  in  accord  with  their  agreement,  such  that  the  petitioner  Foundation  was  included  
as  party  thereto.    
 
The  petitioner  Foundation  asserts  that  it  has  a  personality  separate  and  distinct  from  that  of  its  President,  
petitioner  Tan,  and  that  it  cannot  be  held  solidarily  liable  for  the  loans  of  the  latter.  
 
Issue:  Whether  or  not  the  CA  correctly  held  that  the  petitioners  jointly  and  solidarily  liable  therefor  
 
Ruling:  The  evidence  shows  that  Tan  has  been  representing  himself  as  the  President  of  Lapulapu  Foundation,  
Inc.   He   opened   a   savings   account   and   a   current   account   in   the   names   of   the   corporation,   and   signed   the  
application   form   as   well   as   the   necessary   specimen   signature   cards   twice,   for   himself   and   for   the  

   
3H  A.Y.  2017-­‐2018   124  
 
 

CORPORATION  LAW  CASE  DIGESTS  –  ATTY.  DANTE  DELA  CRUZ  

foundation.  He  submitted  a  notarized  Secretarys  Certificate  from  the  corporation,  attesting  that  he  has  been  
authorized,  inter  alia,  to  sign  for  and  in  behalf  of  the  Lapulapu  Foundation  any  and  all  checks,  drafts  or  other  
orders  with  respect  to  the  bank;  to  transact  business  with  the  Bank,  negotiate  loans,  agreements,  obligations,  
promissory  notes  and  other  commercial  documents;  and  to  initially  obtain  a  loan  for  P100,000.00  from  any  
bank.  Under  these  circumstances,  the  defendant  corporation  is  liable  for  the  transactions  entered  into  by  Tan  
on  its  behalf.    
 
Per  its  Secretarys  Certificate,  the  petitioner  Foundation  had  given  its  President,  petitioner  Tan,  ostensible  
and   apparent   authority   to   inter   alia   deal   with   the   respondent   Bank.   Accordingly,   the   petitioner   Foundation  
is  estopped  from  questioning  petitioner  Tans  authority  to  obtain  the  subject  loans  from  the  respondent  Bank.  
It   is   a   familiar   doctrine   that   if   a   corporation   knowingly   permits   one   of   its   officers,   or   any   other   agent,   to   act  
within  the  scope  of  an  apparent  authority,  it  holds  him  out  to  the  public  as  possessing  the  power  to  do  those  
acts;  and  thus,  the  corporation  will,  as  against  anyone  who  has  in  good  faith  dealt  with  it  through  such  agent,  
be  estopped  from  denying  the  agents  authority.  
 
 
 
94.   THE   BOARD   OF   LIQUIDATORS*   REPRESENTING   THE   GOVERNMENT   OF   THE   REPUBLIC   OF   THE  
PHILIPPINES  VS.  HEIRS  OF  MAXIMO  M.  KALAW,  JUAN  BOCAR,  ESTATE  OF  THE  DECEASED  CASIMIRO  
GARCIA,  AND  LEONOR  MOLL  
G.R.  NO.  L-­‐‑18805,  AUGUST  14,  1967  
SANCHEZ.  J.  
 
*Original  plaintiff,  National  Coconut  Corporation,  was  dissolved  on  November  24,  1950  by  the  President's  
Executive  Order  372,  which  created  the  Board  of  Liquidators.  
 
DOCTRINE:  
Accepted  in  this  jurisdiction  are  three  methods  by  which  a  corporation  may  wind  up  its  affairs:  (1)  under  
Section  3,  Rule  104,  of  the  Rules  of  Court  [which  superseded  Section  66  of  the  Corporation  Law]  whereby,  
upon  voluntary  dissolution  of  a  corporation,  the  court  may  direct  "such  disposition  of  its  assets  as  justice  
requires,  and  may  appoint  a  receiver  to  collect  such  assets  and  pay  the  debts  of  the  corporation;"  (2)  under  
Section  77  of  the  Corporation  Law,  whereby  a  corporation  whose  corporate  existence  is  terminated,  "shall  
nevertheless  be  continued  as  a  body  corporate  for  three  years  after  the  time  when  it  would  have  been  so  
dissolved,  for  the  purpose  of  prosecuting  and  defending  suits  by  or  against  it  and  of  enabling  it  gradually  to  
settle  and  close  its  affairs,  to  dispose  of  and  convey  its  property  and  to  divide  its  capital  stock,  but  not  for  the  
purpose  of  continuing  the  business  for  which  it  was  established;"  and  (3)  under  Section  78  of  the  Corporation  
Law,   by   virtue   of   which   the   corporation,   within   the   three   year   period   just   mentioned,   "is   authorized   and  
empowered  to  convey  all  of  its  property  to  trustees  for  the  benefit  of  members,  stockholders,  creditors,  and  
others  interested.”  
 
FACTS:  
The  National  Coconut  Corporation  (NACOCO)  was  chartered  as  a  non-­‐‑profit  governmental  organization  on  
May   7,   1940   by   Commonwealth   Act   518   avowedly   for   the   protection,   preservation   and   development   of   the  
coconut  industry  in  the  Philippines.  
 
On  August  1,  1946,  NACOCO's  charter  was  amended  [Republic  Act  5]  to  grant  that  corporation  the  express  
power  "to  buy,  sell,  barter,  export,  and  in  any  other  manner  deal  in,  coconut,  copra,  and  dessicated  coconut,  
as  well  as  their  by-­‐‑products,  and  to  act  as  agent,  broker  or  commission  merchant  of  the  producers,  dealers  
or  merchants"  thereof.  
 

   
3H  A.Y.  2017-­‐2018   125  
 
 

CORPORATION  LAW  CASE  DIGESTS  –  ATTY.  DANTE  DELA  CRUZ  

Maximo  M.  Kalaw  -­‐‑  General  manager  and  board  chairman  


Defendants  Juan  Bocar  and  Casimiro  Garcia  -­‐‑  members  of  the  Board  
Defendant  Leonor  Moll  -­‐‑  became  director  only  on  December  22,  1947  
 
NACOCO,  after  the  passage  of  Republic  Act  5,  embarked  on  copra  trading  activities.  Amongst  the  scores  of  
contracts  executed  by  general  manager  Kalaw  are  the  disputed  contracts,  for  the  delivery  of  copra.  
 
An   unhappy   chain   of   events   conspired   to   deter   NACOCO   from   fulfilling   these   contracts.   Nature   supervened.  
Four  devastating  typhoons  visited  the  Philippines:  the  first  in  October,  the  second  and  third  in  November,  
and  the  fourth  in  December,  1947.  Coconut  trees  throughout  the  country  suffered  extensive  damage.  Copra  
production   decreased.   Prices   spiralled.   Warehouses   were   destroyed.   Cash   requirements   doubled.  
Deprivation   of   export   facilities   increased   the   time   necessary   to   accumulate   shiploads   of   copra.   Quick  
turnovers  became  impossible,  financing  a  problem.  
 
When   it   became   clear   that   the   contracts   would   be   unprofitable,   Kalaw   submitted   them   to   the   board   for  
approval.  It  was  not  until  December  22,  1947  when  the  membership  was  completed.  Defendant  Moll  took  
her  oath  on  that  date.  A  meeting  was  then  held.  Kalaw  made  a  full  disclosure  of  the  situation,  apprised  the  
board   of   the   impending   heavy   losses.   No   action   was   taken   on   the   contracts.   Neither   did   the   board   vote  
thereon   at   the   meeting   of   January   7,   1948   following.   Then,   on   January   11,   1948,   President   Roxas   made   a  
statement  that  the  NACOCO  head  did  his  best  to  avert  the  losses,  emphasized  that  government  concerns  faced  
the  same  risks  that  confronted  private  companies,  that  NACOCO  was  recouping  its  losses,  and  that  Kalaw  was  
to  remain  in  his  post.  Not  long  thereafter,  that  is,  on  January  30,  1948,  the  board  met  again  with  Kalaw,  Bocar,  
Garcia  and  Moll  in  attendance.  They  unanimously  approved  the  contracts  hereinbefore  enumerated.  
 
As  expected,  NACOCO  partially  performed  the  contracts.  
 
The  buyers  threatened  damage  suits.  Some  of  the  claims  were  settled,  viz:  Pacific  Vegetable  Oil  Co.,  in  copra  
delivered   by   NACOCO,   P539,000.00;   Franklin   Baker   Corporation,   P78,210.00;   Spencer   Kellog   &   Sons,  
P159,040.00.  
 
But  one  buyer,  Louis  Dreyfus  &  Go.  (Overseas)  Ltd.,  did  in  fact  sue  before  the  Court  of  First  Instance  of  Manila.  
 
These  cases  culminated  in  an  out-­‐‑of-­‐‑court  amicable  settlement  when  the  Kalaw  management  was  already  
out.   The   corporation   thereunder   paid   Dreyfus   P567,024.52   representing   70%   of   the   total   claims.   With  
particular  reference  to  the  Dreyfus  claims,  NACOCO  put  up  the  defenses  that:  (1)  the  contracts  were  void  
because  Louis  Dreyfus  &  Co.  (Overseas)  Ltd.  did  not  have  license  to  do  business  here;  and  (2)  failure  to  deliver  
was  due  to  force  majeure,  the  typhoons.  
 
In   this   suit   started   in   February,   1949,   NACOCO   seeks   to   recover   the   sum   of   P1,343,274.52   from   general  
manager  and  board  chairman  Maximo  M.  Kalaw,  and  directors  Juan  Bocar,  Casimiro  Garcia  and  Leonor  Moll.  
It  charges  Kalaw  with  negligence  under  Article  1902  of  the  old  Civil  Code  (now  Article  2176,  new  Civil  Code);  
and   defendant   board   members,   including   Kalaw,   with   bad   faith   and/or   breach   of   trust   for   having   approved  
the  contracts.  
 
CFI:  dismissed  the  complaint;  NACOCO  was  ordered  to  pay  the  heirs  of  Maximo  Kalaw  the  sum  of  P2,601.94  
for  unpaid  salaries  and  cash  deposit  due  the  deceased  Kalaw  from  NACOCO.  
 
ISSUE:  W/N  the  plaintiff  Board  of  Liquidators  has  lost  its  legal  personality  to  continue  with  this  suit.  -­‐‑  YES  
 
HELD:  

   
3H  A.Y.  2017-­‐2018   126  
 
 

CORPORATION  LAW  CASE  DIGESTS  –  ATTY.  DANTE  DELA  CRUZ  

Accepted  in  this  jurisdiction  are  three  methods  by  which  a  corporation  may  wind  up  its  affairs:  (1)  under  
Section  3,  Rule  104,  of  the  Rules  of  Court  [which  superseded  Section  66  of  the  Corporation  Law]  whereby,  
upon  voluntary  dissolution  of  a  corporation,  the  court  may  direct  "such  disposition  of  its  assets  as  justice  
requires,  and  may  appoint  a  receiver  to  collect  such  assets  and  pay  the  debts  of  the  corporation;"  (2)  under  
Section  77  of  the  Corporation  Law,  whereby  a  corporation  whose  corporate  existence  is  terminated,  "shall  
nevertheless  be  continued  as  a  body  corporate  for  three  years  after  the  time  when  it  would  have  been  so  
dissolved,  for  the  purpose  of  prosecuting  and  defending  suits  by  or  against  it  and  of  enabling  it  gradually  to  
settle  and  close  its  affairs,  to  dispose  of  and  convey  its  property  and  to  divide  its  capital  stock,  but  not  for  the  
purpose  of  continuing  the  business  for  which  it  was  established;"  and  (3)  under  Section  78  of  the  Corporation  
Law,   by   virtue   of   which   the   corporation,   within   the   three   year   period   just   mentioned,   "is   authorized   and  
empowered  to  convey  all  of  its  property  to  trustees  for  the  benefit  of  members,  stockholders,  creditors,  and  
others  interested.”  
 
It  is  defendants'  pose  that  their  case  comes  within  the  coverage  of  the  second  method.  They  reason  out  that  
suit  was  commenced  in  February,  1949;  that  by  Executive  Order  372,  dated  November  24,  1950,  NACOCO,  
together   with   other   government-­‐‑owned   corporations,   was   abolished,   and   the   Board   of   Liquidators   was  
entrusted  with  the  function  of  settling  and  closing  its  affairs;  and  that,  since  the  three  year  period  has  elapsed,  
the   Board   of   Liquidators   may   not   now   continue   with,   and   prosecute,   the   present   case   to   its   conclusion,  
because  Executive  Order  372  provides  in  Section  1  thereof  that  —  
 
Sec.1.   The   National   Abaca   and   Other   Fibers   Corporation,   the   National   Coconut   Corporation,   the   National  
Tobacco   Corporation,   the   National   Food   Producer   Corporation   and   the   former   enemy-­‐‑owned   or   controlled  
corporations   or   associations,   .   .   .   are   hereby   abolished.   The   said   corporations   shall   be   liquidated   in  
accordance  with  law,  the  provisions  of  this  Order,  and/or  in  such  manner  as  the  President  of  the  Philippines  
may  direct;  Provided,  however,  That  each  of  the  said  corporations  shall  nevertheless  be  continued  as  a  body  
corporate  for  a  period  of  three  (3)  years  from  the  effective  date  of  this  Executive  Order  for  the  purpose  of  
prosecuting  and  defending  suits  by  or  against  it  and  of  enabling  the  Board  of  Liquidators  gradually  to  settle  
and  close  its  affairs,  to  dispose  of  and,  convey  its  property  in  the  manner  hereinafter  provided.  
 
The   executive   order   abolishing   NACOCO   and   creating   the   Board   of   Liquidators   should   be   examined   in  
context.  The  proviso  in  Section  1  of  EO  372,  whereby  the  corporate  existence  of  NACOCO  was  continued  for  
a  period  of  three  years  from  the  effectivity  of  the  order  for  "the  purpose  of  prosecuting  and  defending  suits  
by  or  against  it  and  of  enabling  the  Board  of  Liquidators  gradually  to  settle  and  close  its  affairs,  to  dispose  of  
and  convey  its  property  in  the  manner  hereinafter  provided",  is  to  be  read  not  as  an  isolated  provision  but  in  
conjunction  with  the  whole.  So  reading,  it  will  be  readily  observed  that  no  time  limit  has  been  tacked  to  the  
existence  of  the  Board  of  Liquidators  and  its  function  of  closing  the  affairs  of  the  various  government  owned  
corporations,  including  NACOCO.  
 
By  Section  2  of  the  executive  order,  while  the  boards  of  directors  of  the  various  corporations  were  abolished,  
their   powers   and   functions   and   duties   under   existing   laws   were   to   be   assumed   and   exercised   by   the   Board  
of   Liquidators.   The   President   thought   it   best   to   do   away   with   the   boards   of   directors   of   the   defunct  
corporations;  at  the  same  time,  however,  the  President  had  chosen  to  see  to  it  that  the  Board  of  Liquidators  
step  into  the  vacuum.  And  nowhere  in  the  executive  order  was  there  any  mention  of  the  lifespan  of  the  Board  
of  Liquidators.  Thus,  liquidation  by  the  Board  of  Liquidators  may,  under  section  1,  proceed  in  accordance  
with   law,   the   provisions   of   the   executive   order,   "and/or   in   such   manner   as   the   President   of   the   Philippines  
may   direct."   By   Section   4,   when   any   property,   fund,   or   project   is   transferred   to   any   governmental  
instrumentality   "for   administration   or   continuance   of   any   project,"   the   necessary   funds   therefor   shall   be  
taken   from   the   corresponding   special   fund   created   in   Section   5.   Section   5,   in   turn,   talks   of   special   funds  
established  from  the  "net  proceeds  of  the  liquidation"  of  the  various  corporations  abolished.  And  by  Section,  
7,  fifty  per  centum  of  the  fees  collected  from  the  copra  standardization  and  inspection  service  shall  accrue  

   
3H  A.Y.  2017-­‐2018   127  
 
 

CORPORATION  LAW  CASE  DIGESTS  –  ATTY.  DANTE  DELA  CRUZ  

"to   the   special   fund   created   in   section   5   hereof   for   the   rehabilitation   and   development   of   the   coconut  
industry."   Implicit   in   all   these,   is   that   the   term   of   life   of   the   Board   of   Liquidators   is   without   time   limit.  
Contemporary  history  gives  us  the  fact  that  the  Board  of  Liquidators  still  exists  as  an  office  with  officials  and  
numerous  employees  continuing  the  job  of  liquidation  and  prosecution  of  several  court  actions.  
 
By  EO  372,  the  government,  the  sole  stockholder,  abolished  NACOCO,  and  placed  its  assets  in  the  hands  of  
the   Board   of   Liquidators.   The   Board   of   Liquidators   thus   became   the   trustee   on   behalf   of   the   government.   It  
was   an   express   trust.   The   legal   interest   became   vested   in   the   trustee   —   the   Board   of   Liquidators.   The  
beneficial  interest  remained  with  the  sole  stockholder  —  the  government.  The  provisions  of  Section  78  of  
the  Corporation  Law  —  the  third  method  of  winding  up  corporate  affairs  —  find  application.  
 
We,  accordingly,  rule  that  the  Board  of  Liquidators  has  personality  to  proceed  as  party-­‐‑plaintiff  in  this  case.  
 
 
 
95.  ADVANCE  PAPER  CORPORATION  VS.  ARMA  TRADERS  CORPORATION  
G.R.  NO.176897                              DECEMBER  11,  2013  
BRION,  J.  
 
 
DOCTRINE:  
  The  doctrine  of  apparent  authority  provides  that  a  corporation  will  be  estopped  from  denying  the  
agent’s  authority  if  it  knowingly  permits  one  of  its  officers  or  any  other  agent  to  act  within  the  scope  of  an  
apparent  authority,  and  it  holds  him  out  to  the  public  as  possessing  the  power  to  do  those  acts.  The  doctrine  
of  apparent  authority  does  not  apply  if  the  principal  did  not  commit  any  acts  or  conduct  which  a  third  party  
knew  and  relied  upon  in  good  faith  as  a  result  of  the  exercise  of  reasonable  prudence.  Moreover,  the  agent’s  
acts  or  conduct  must  have  produced  a  change  of  position  to  the  third  party’s  detriment.  
 
FACTS:  
  Petitioner   Advance   Paper   is   a   domestic   corporation   engaged   in   the   business   of   producing,   printing,  
manufacturing,   distributing   and   selling   of   various   paper   products.   Petitioner   George   Haw   (Haw)   is   the  
President  while  his  wife,  Connie  Haw,  is  the  General  Manager.  
  Respondent  Arma  Traders  is  also  a  domestic  corporation  engaged  in  the  wholesale  and  distribution  
of   school   and   office   supplies,   and   novelty   products.   Respondent   Antonio   Tan   (Tan)   was   formerly   the  
President   while   respondent   Uy   Seng   Kee   Willy   (Uy)   is   the   Treasurer   of   Arma   Traders.   They   represented  
Arma  Traders  when  dealing  with  its  supplier,  Advance  Paper,  for  about  14  years.    
  On  various  dates  from  September  to  December  1994,  Arma  Traders  purchased  on  credit  notebooks  
and  other  paper  products  amounting  to  ₱7,533,001.49  from  Advance  Paper.  
  Arma  Traders  also  obtained  three  loans  from  Advance  Paper  in  November  1994  in  the  amounts  of  
₱3,380,171.82,  ₱1,000,000.00,  and  ₱3,408,623.94  or  a  total  of  ₱7,788,796.76.  Arma  Traders  needed  the  loan  
to   settle   its   obligations   to   other   suppliers   because   its   own   collectibles   did   not   arrive   on   time.   Because   of   its  
good  business  relations  with  Arma  Traders,  Advance  Paper  extended  the  loans.  
  Arma  Traders  issued  82  postdated  checks14  payable  to  cash  or  to  Advance  Paper.  Tan  and  Uy  were  
Arma   Traders’   authorized   bank   signatories   who   signed   and   issued   these   checks   which   had   the   aggregate  
amount  of  ₱15,130,636.87.  
  Advance   Paper   presented   the   checks   to   the   drawee   bank   but   these   were   dishonored   either   for  
"insufficiency   of   funds"   or   "account   closed."   Despite   repeated   demands,   however,   Arma   Traders   failed   to  
settle  its  account  with  Advance  Paper.  
  The   petitioners   filed   a   complaint17   for   collection   of   sum   of   money   with   application   for   preliminary  
attachment   against   Arma   Traders,   Tan,   Uy,   Ting,   Gui,   and   Ng.   Respondents   Manuel   Ting,   Cheng   Gui   and  

   
3H  A.Y.  2017-­‐2018   128  
 
 

CORPORATION  LAW  CASE  DIGESTS  –  ATTY.  DANTE  DELA  CRUZ  

Benjamin   Ng   worked   for   Arma   Traders   as   Vice-­‐‑President,   General   Manager   and   Corporate   Secretary,  
respectively.  
  The   respondents   claimed   that   the   loan   transactions   were   ultra   vires   because   the   board   of   directors  
of   Arma   Traders   did   not   issue   a   board   resolution   authorizing   Tan   and   Uy   to   obtain   the   loans   from   Advance  
Paper.  They  claimed  that  the  borrowing  of  money  must  be  done  only  with  the  prior  approval  of  the  board  of  
directors  because  without  the  approval,  the  corporate  officers  are  acting  in  excess  of  their  authority  or  ultra  
vires.   When   the   acts   of   the   corporate   officers   are   ultra   vires,   the   corporation   is   not   liable   for   whatever   acts  
that  these  officers  committed  in  excess  of  their  authority.  
 
ISSUE:  
  Whether  Arma  Traders  is  liable  to  pay  the  loans  applying  the  doctrine  of  apparent  authority.  
 
HELD:  
  Apparent  authority  is  derived  not  merely  from  practice.  Its  existence  may  be  ascertained  through  (1)  
the   general   manner   in   which   the   corporation   holds   out   an   officer   or   agent   as   having   the   power   to   act   or,   in  
other  words  the  apparent  authority  to  act  in  general,  with  which  it  clothes  him;  or  (2)  the  acquiescence  in  
his  acts  of  a  particular  nature,  with  actual  or  constructive  knowledge  thereof,  within  or  beyond  the  scope  of  
his  ordinary  powers.  It  requires  presentation  of  evidence  of  similar  act(s)  executed  either  in  its  favor  or  in  
favor   of   other   parties.   It   is   not   the   quantity   of   similar   acts   which   establishes   apparent   authority,   but   the  
vesting  of  a  corporate  officer  with  the  power  to  bind  the  corporation.  
  In  the  present  petition,  we  do  not  agree  with  the  CA’s  findings  that  Arma  Traders  is  not  liable  to  pay  
the  loans  due  to  the  lack  of  board  resolution  authorizing  Tan  and  Uy  to  obtain  the  loans.  To  begin  with,  Arma  
Traders’   Articles   of   Incorporation   provides   that   the   corporation   may   borrow   or   raise   money   to   meet   the  
financial   requirements   of   its   business   by   the   issuance   of   bonds,   promissory   notes   and   other   evidence   of  
indebtedness.  Likewise,  it  states  that  Tan  and  Uy  are  not  just  ordinary  corporate  officers  and  authorized  bank  
signatories   because   they   are   also   Arma   Traders’   incorporators   along   with   respondents   Ng   and   Ting,   and  
Pedro   Chao.   Furthermore,   the   respondents,   through   Ng   who   is   Arma   Traders’   corporate   secretary,  
incorporator,   stockholder   and   director,   testified   that   the   sole   management   of   Arma   Traders   was   left   to   Tan  
and  Uy  and  that  he  and  the  other  officers  never  dealt  with  the  business  and  management  of  Arma  Traders  
for  14  years.  He  also  confirmed  that  since  1984  up  to  the  filing  of  the  complaint  against  Arma  Traders,  its  
stockholders  and  board  of  directors  never  had  its  meeting.  
 
 
 
96.  NO  CASE  
 
 
 
97.  GRACE  CHRISTIAN  HIGH  SCHOOL  VS.  THE  COURT  OF  APPEALS    
G.R.  NO.  108905.  OCTOBER  23,  1997  
MENDOZA,  J.:  
 
FACTS:    
Petitioner   Grace   Christian   High   School   is   an   educational   institution   offering   preparatory,   kindergarten  
and  secondary  courses  at  the  Grace  Village  in  Quezon  City.  Private  respondent  Grace  Village  Association,  Inc.,  
on  the  other  hand,  is  an  organization  of  lot  and/or  building  owners,  lessees  and  residents  at  Grace  Village,  
while  private  respondents  Alejandro  G.  Beltran  and  Ernesto  L.  Go  were  its  president  and  chairman  of  the  
committee  on  election,  respectively,  in  1990,  when  this  suit  was  brought.  
As  adopted  in  1968,  the  by-­‐‑laws  of  the  association  provided  in  Article  IV,  as  follows:  

   
3H  A.Y.  2017-­‐2018   129  
 
 

CORPORATION  LAW  CASE  DIGESTS  –  ATTY.  DANTE  DELA  CRUZ  

The  annual  meeting  of  the  members  of  the  Association  shall  be  held  on  the  first  Sunday  of  January  in  each  
calendar  year  at  the  principal  office  of  the  Association  at  2:00  P.M.  where  they  shall  elect  by  plurality  vote  
and   by   secret   balloting,   the   Board   of   Directors,   composed   of   eleven   (11)   members   to   serve   for   one   (1)   year  
until  their  successors  are  duly  elected  and  have  qualified.  
It   appears,   that   on   December   20,   1975,   a   committee   of   the   board   of   directors   prepared   a   draft   of   an  
amendment  to  the  by-­‐‑laws  
This  draft  was  never  presented  to  the  general  membership  for  approval.  Nevertheless,  from  1975,  after  it  
was   presumably   submitted   to   the   board,   up   to   1990,   petitioner   was   given   a   permanent   seat   in   the   board   of  
directors   of   the   association.   On   February   13,   1990,   the   associations   committee   on   election   in   a   letter  
informed  James  Tan,  principal  of  the  school,  that  it  was  the  sentiment  that  all  directors  should  be  elected  by  
members  of  the  association  because  to  make  a  person  or  entity  a  permanent  Director  would  deprive  the  right  
of  voters  to  vote  for  fifteen  (15)  members  of  the  Board,  and  it  is  undemocratic  for  a  person  or  entity  to  hold  
office  in  perpetuity.  
For  this  reason,  Tan  was  told  that  the  proposal  to  make  the  Grace  Christian  High  School  representative  
as   a   permanent   director   of   the   association,   although   previously   tolerated   in   the   past   elections   should   be  
reexamined.  Following  this  advice,  notices  were  sent  to  the  members  of  the  association  that  the  provision  on  
election  of  directors  of  the  1968  by-­‐‑laws  of  the  association  would  be  observed.  
Petitioner  requested  the  chairman  of  the  election  committee  to  change  the  notice  of  election  by  following  
the  procedure  in  previous  elections,  claiming  that  the  notice  issued  for  the  1990  elections  ran  counter  to  the  
practice   in   previous   years   and   was   in   violation   of   the   by-­‐‑laws   (of   1975)   and   unlawfully   deprive[d]   Grace  
Christian  High  School  of  its  vested  right  [to]  a  permanent  seat  in  the  board.  
As  the  association  denied  its  request,  the  school  brought  suit  for  mandamus  in  the  Home  Insurance  and  
Guaranty  Corporation  to  compel  the  board  of  directors  of  the  association  to  recognize  its  right  to  a  permanent  
seat  in  the  board.  Petitioner  based  its  claim  on  the  following  portion  of  the  proposed  amendment  which,  it  
contended,  had  become  part  of  the  by-­‐‑laws  of  the  association  as  Article  VI,  paragraph  2,  thereof:  
The  Charter  and  Associate  Members  shall  elect  the  Directors  of  the  Association.  The  candidates  receiving  
the   first   fourteen   (14)   highest   number   of   votes   shall   be   declared   and   proclaimed   elected   until   their  
successors   are   elected   and   qualified.   GRACE   CHRISTIAN   HIGH   SCHOOL   representative   is   a   permanent  
Director  of  the  ASSOCIATION.  
It  appears  that  the  opinion  of  the  Securities  and  Exchange  Commission  on  the  validity  of  this  provision  was  
sought  by  the  association  and  that  in  reply  to  the  query,  the  SEC  rendered  an  opinion  to  the  effect  that  the  
practice  of  allowing  unelected  members  in  the  board  was  contrary  to  the  existing  by-­‐‑laws  of  the  association  
and  to  92  of  the  Corporation  Code.  On  June  20,  1990,  the  hearing  officer  of  the  HIGC  rendered  a  decision  
dismissing   petitioners   action.   The   hearing   officer   held   that   the   amended   by-­‐‑laws,   upon   which   petitioner  
based  its  claim,  [was]  merely  a  proposed  by-­‐‑laws  which,  although  implemented  in  the  past,  had  not  yet  been  
ratified  by  the  members  of  the  association  nor  approved  by  competent  authority;  that,  on  the  contrary,  in  the  
meeting   held   on   April   17,   1990,   the   directors   of   the   association   declared   the   proposed   by-­‐‑law   dated  
December  20,  1975  prepared  by  the  committee  on  by-­‐‑laws  .  .  .  null  and  void  and  the  by-­‐‑laws  of  December  17,  
1968   as   the   prevailing   by-­‐‑laws   under   which   the   association   is   to   operate   until   such   time   that   the   proposed  
amendments   to   the   by-­‐‑laws   are   approved   and   ratified   by   a   majority   of   the   members   of   the   association   and  
duly   filed   and   approved   by   the   pertinent   government   agency.   The   hearing   officer   rejected   petitioners  
contention  that  it  had  acquired  a  vested  right  to  a  permanent  seat  in  the  board  of  directors.  He  held  that  past  
practice  in  election  of  directors  could  not  give  rise  to  a  vested  right  and  that  departure  from  such  practice  
was  justified  because  it  deprived  members  of  association  of  their  right  to  elect  or  to  be  voted  in  office,  not  to  
say  that  allowing  the  automatic  inclusion  of  a  member  representative  of  petitioner  as  permanent  director  
[was]  contrary  to  law  and  the  registered  by-­‐‑laws  of  respondent  association.  
 
ISSUE:  
Whether  or  not  petitioner’s  contentions  that  he  has  a  vested  right  to  hold  office  is  correct.    
 

   
3H  A.Y.  2017-­‐2018   130  
 
 

CORPORATION  LAW  CASE  DIGESTS  –  ATTY.  DANTE  DELA  CRUZ  

HELD:  
NO.    The  provisions  of  the  former  and  present  corporation  law  leave  no  room  for  doubt  as  to  their  meaning:  
the  board  of  directors  of  corporations  must  be  elected  from  among  the  stockholders  or  members.  There  may  
be  corporations  in  which  there  are  unelected  members  in  the  board  but  it  is  clear  that  in  the  examples  cited  
by  petitioner  the  unelected  members  sit  as  ex  officio  members,  i.e.,  by  virtue  of  and  for  as  long  as  they  hold  a  
particular  office.  But  in  the  case  of  petitioner,  there  is  no  reason  at  all  for  its  representative  to  be  given  a  seat  
in  the  board.  Nor  does  petitioner  claim  a  right  to  such  seat  by  virtue  of  an  office  held.  In  fact  it  was  not  given  
such   seat   in   the   beginning.   It   was   only   in   1975   that   a   proposed   amendment   to   the   by-­‐‑laws   sought   to   give   it  
one.  
Since  the  provision  in  question  is  contrary  to  law,  the  fact  that  for  fifteen  years  it  has  not  been  questioned  or  
challenged  but,  on  the  contrary,  appears  to  have  been  implemented  by  the  members  of  the  association  cannot  
forestall  a  later  challenge  to  its  validity.  Neither  can  it  attain  validity  through  acquiescence  because,  if  it  is  
contrary  to  law,  it  is  beyond  the  power  of  the  members  of  the  association  to  waive  its  invalidity.  For  that  
matter  the  members  of  the  association  may  have  formally  adopted  the  provision  in  question,  but  their  action  
would  be  of  no  avail  because  no  provision  of  the  by-­‐‑laws  can  be  adopted  if  it  is  contrary  to  law.  
It  is  probable  that,  in  allowing  petitioners  representative  to  sit  on  the  board,  the  members  of  the  association  
were  not  aware  that  this  was  contrary  to  law.  It  should  be  noted  that  they  did  not  actually  implement  the  
provision   in   question   except   perhaps   insofar   as   it   increased   the   number   of   directors   from   11   to   15,   but  
certainly  not  the  allowance  of  petitioners  representative  as  an  unelected  member  of  the  board  of  directors.  It  
is  more  accurate  to  say  that  the  members  merely  tolerated  petitioners  representative  and  tolerance  cannot  
be  considered  ratification.  
Nor  can  petitioner  claim  a  vested  right  to  sit  in  the  board  on  the  basis  of  practice.Practice,  no  matter  how  
long  continued,  cannot  give  rise  to  any  vested  right  if  it  is  contrary  to  law.  Even  less  tenable  is  petitioners  
claim  that  its  right  is  coterminus  with  the  existence  of  the  association.  
 
 
 
98.  ZAMBOANGA  TRANSPORTATION  COMPANY,  INC.  vs.  THE  BACHRACH  MOTOR  CO.,  INC.    
G.R.  No.  L-­‐‑27694,  October  24,  1928  
Villa-­‐‑Real,  J  
 
DOCTRINE:    
When  the  president  of  a  corporation,  who  is  one  of  the  principal  stockholders  and  at  the  same  time  its  general  
manager,   auditor,   attorney   or   legal   adviser,   is   empowered   by   its   by-­‐‑laws   to   enter   into   chattel   mortgage  
contracts,   subject   to   the   approval   of   the   board   of   directors,   and   enters   into   such   contracts   with   the   tacit  
approval   of   two   members   of   the   board   of   directors,   one   of   whom   is   a   principal   shareholder,   both   of   whom,  
together   with   the   president,   form   a   majority,   and   said   corporation   takes   advantage   of   the   benefits   afforded  
by  said  contract,  such  acts  are  equivalent  to  an  implied  ratification  of  said  contract  by  the  board  of  directors  
and  binds  the  corporation  even  if  not  formally  approved  by  said  board  of  directors  as  required  by  the  by-­‐‑
laws  of  the  aforesaid  corporation.  
 
Though  a  chattel  mortgage  contract  entered  into  by  a  public  service  corporation  is  ineffective  without  the  
authorization  and  approval  of  the  Public  Utility  Commission,  it  may  be  valid  if  it  contains  all  the  material  and  
formal   requisites   demanded   by   the   law   for   its   validity,   and   said   Public   Utility   Commission   may   make   it  
retroactive  by  nunc  pro  tunc  authorization  and  approval.  
 
FACTS:    
Zamboanga   Transportation   Co.,   Inc.   (Zamboanga),   is   managed   by   a   board   of   directors   composed   of   five  
stockholders;  Bachrach  Motor  Co.  is  a  corporation  engaged  in  selling  automobiles  and  their  parts.  For  10  
years,  the  two  have  been  dealing  with  each  other.  Zamboanga  buys  trucks,  automobiles,  repair  and  accessory  

   
3H  A.Y.  2017-­‐2018   131  
 
 

CORPORATION  LAW  CASE  DIGESTS  –  ATTY.  DANTE  DELA  CRUZ  

parts  for  use  in  the  business  of  transportation  in  which  it  is  engaged.  Payments  were  made  by  installments,  
and  Zamboanga  executed  several  chattel  mortgages  to  secure  it.  Jose  Erquiaga  (Erquiaga)  was  appointed  as  
general  manager  in  1924,elected  president,  and  acted  as  an  auditor  in  1925.  He  is  also  one  of  the  majority  
stockholders  and  has  been  its  attorney  and  legal  adviser.    
  Zamboanga  lacked  funds  and  contacted  Mons.  Jose  Clos,  Bishop  of  Zamboanga  and  a  principal  stock  
holder   of   the   company,   for   loans   of   money.   Since,   he   was   leaving   for   Rome   in   February   1925   and   could   not  
continue   to   loan   money   to   Zamboanga,   additional   agreements   were   entered   between   Mons.   Clos   and   the  
Bachrach   Motor   Co.,   Inc.   A   new   chattel   mortgage   was   executed   on   by   Zamboanga   represented   by   President  
Erquiaga.  In  this  last  mortgage  the  same  goods  were  pledged  that  had  been  hypothecated  by  the  Zamboanga  
Transporatation  Co.,  Inc.,  to  the  Bachrach  Motor  Co.,  by  virtue  of  instruments  to  Mons.  Jose  Clos  Bishop  of  
Zamboanga,   by   the   virtue   of   the   deed.   President   Erquiaga   submitted   the   mortgage   deed   to   the   Board   of  
directors.  Upon  returning  to  Zamboanga  from  Manila,  He  discussed  the  mortgage  with  two  board  of  directors,  
who  expressed  satisfaction.  Zamboanga  also  partially  complied  with  the  mortgage  contract.  Zamboanga  paid  
Bachrach  two  times.  Bachrach  sent  a  letter  cancelling  the  2  former  chattel  mortgage.  Bachrach  told  Erguiaga  
to  register  the  cancellation.  Erquiaga  replied  by  stating  that  the  last  mortgage  was  not  approved  by  the  Board  
of  Directors.  Jose  Erquiaga  went  to  E.M.  Bachman,  president  of  Bachrach  Motor  co.,  to  secure  his  consent  to  
sell   the   trucks   that   were   mortgaged.   He   said   this   will   be   used   to   pay   the   unpaid   debt.   Bachrach   denied.  
Erquiaga  and  Zamboaga  later  on  discovered  that  the  last  mortgage  was  registered  in  the  register  of  deed.  
Zamboanga,  then  filed  for  annulment  of  the  last  mortgage  because  it  was  registered  without  their  consent.  
Bachrach,  filed  a  complaint  for  Zamboanga  to  obtain  possession  of  all  the  chattels.  Bchrach  won  and  sold  the  
chattel  in  a  public  auction  where  they  were  held  the  highest  bidder.    
 
ISSUE:  
WON  the  chattel  mortgage  executed  by  the  president  and  general  manager  of  the  plaintiff  corporation,  the  
Zamboanga  Transportation  Co.,  Inc.,  is  valid  
 
HELD:    
Yes.  While  it  is  true  that  said  last  chattel  mortgage  contract  was  not  approved  by  the  board  of  directors  of  
the  Zamboanga  Transportation  Co.,  Inc.,  whose  approval  was  necessary  in  order  to  validate  it  according  to  
the  by-­‐‑laws  of  said  corporation,  the  broad  powers  vested  in  Jose  Erquiaga  as  president,  general  manager,  
auditor,   attorney   or   legal   adviser,   and   one   of   the   largest   shareholders;   the   approval   of   his   act   in   connection  
with  said  chattel  mortgage  contract  in  question,  with  which  two  other  directors  expressed  satisfaction,  one  
of   which   is   also   one   of   the   largest   shareholders,   who   together   with   the   president   constitute   a   majority:   The  
payments   made   under   said   contract   with   the   knowledge   of   said   three   directors   are   equivalent   to   a   tacit  
approval   by   the   board   of   directors   of   said   chattel   mortgage   contract   and   binds   the   Zamboanga  
Transportation  Co.,  Inc.  In  truth  and  in  fact  Jose  Erquiaga,  in  his  multiple  capacity,  was  and  is  the  factotum  of  
the  corporation  and  may  be  said  to  be  the  corporation  itself.  
 
"Halley  First  National  Bank  vs.  G.  V.  B.  Min.  Co.":  Where  the  chief  officers  of  a  corporation  are  in  reality  its  
owners,  holding  nearly  all  of  its  stock,  and  are  permitted  to  manage  the  business  by  the  directors,  who  are  
only  interested  nominally  or  to  a  small  extent,  and  are  controlled  entirely  by  the  officers,  the  acts  of  such  
officers  are  binding  on  the  corporation,  which  cannot  escape  liability  as  to  third  persons  dealing  with  it  in  
good  faith  on  the  pretense  that  such  acts  were  ultra  vires.  
When  the  president  of  a  corporation,  who  is  one  of  the  principal  stockholders  and  at  the  same  time  its  general  
manager,   auditor,   attorney   or   legal   adviser,   is   empowered   by   its   by-­‐‑laws   to   enter   into   chattel   mortgage  
contracts,   subject   to   the   approval   of   the   board   of   directors,   and   enters   into   such   contracts   with   the   tacit  
approval  of  two  other  members  of  the  board  of  directors,  one  of  whom  is  also  a  principal  shareholder,  both  
of  whom,  together  with  the  president,  form  a  majority,  and  said  corporation  takes  advantage  of  the  benefits  
afforded  by  said  contract,  such  acts  are  equivalent  to  an  implied  ratification  of  said  contract  by  the  board  of  

   
3H  A.Y.  2017-­‐2018   132  
 
 

CORPORATION  LAW  CASE  DIGESTS  –  ATTY.  DANTE  DELA  CRUZ  

directors  and  binds  the  corporation  even  if  not  formally  approved  by  said  board  of  directors  as  required  by  
the  by-­‐‑laws  of  the  aforesaid  corporation.  
 
 
 
99.   THE   BOARD   OF   DIRECTORS   and   ELECTION   COMMITTEE   OF   THE   SMB   WORKERS   SAVINGS   AND  
LOAN  ASSOCIATION,  INC.,  ET  AL.,  petitioners,  vs.  HON  BIENVENIDO  A.  TAN,  ETC.,  ET  AL.,  respondent.  
G.R.  No.  L-­‐‑12282.  March  31,  1959.  
Padilla,  J.  
 
Doctrine:  When  it  appears  that  a  fair  election  cannot  be  had,  the  court  in  the  exercise  of  its  equity  jurisdiction  
may  appoint  a  committee  with  the  authority  to  call,  conduct  and  supervise  the  election  of  the  directors  or  the  
association.  
   
Facts:  On  January  1957  John  de  Castillo  et  al.,  commenced  a  suit  in  CFI-­‐‑Manila  to  declare  null  and  void  the  
election   of   the   members   of   the   BOD   of   the   SMB   Workers   Savings   and   Loan   Association,   Inc.   (SMB-­‐‑SWSLAI)  
and  of  the  members  of  the  Election  Committee  for  the  year  1957  held  on  January;  to  compel  the  BOD  of  the  
association   to   call   for   and   hold   another   election   in   accordance   with   its   constitution   and   by-­‐‑laws   and   the  
Corporation   Law;   to   restrain   the   defendants   who   had   been   illegally   elected   as   members   of   the   BOD   from  
exercising  the  functions  of  their  office.  On  the  day  set  for  trial  of  the  case,  neither  the  defendants  nor  their  
attorney  appeared.  The  Court  proceeded  to  receive  the  plaintiffs'  evidence.  On  February,  the  Court  rendered  
judgment   declaring  the   election   held   on   January   null  and   void,   ordering   the   defendants   to   call   for   and   hold  
another   election   in   accordance   with   the   constitution   and   by-­‐‑laws   of   the   association   and   the   Corporation  
Law.  On  15  February,  before  the  expiration  of  the  time  to  appeal,  the  plaintiffs  (SMB-­‐‑SWSLAI)  moved  for  
immediate  execution  of  the  judgment.  On  4  March  the  Court  granted  the  plaintiffs  motion  and  issued  the  writ  
of   execution   prayed   for.   On   9   March   the   defendants   moved   for   stay   of   execution   of   the   judgment,   for   which  
they  offered  to  file  a  supersedes  bond  in  the  amount  to  be  fixed  by  the  Court.  On  23  March  the  Court  denied  
the  defendants'  motion.  In  compliance  with  the  judgment  rendered  by  the  Court,  on  26  March  the  election  
committee  composed  of  Tesalona,  Dumaup  and  Santos  set  the  meeting  of  the  members  of  the  association  for  
28   March   at   5:30   o'clock   in   the   afternoon   to   elect   the   new   members   of   the   BOD.   On   27   March   the   plaintiffs  
filed  an  ex-­‐‑parte  motion  alleging  that  the  election  committee  that  had  called  the  meeting  of  members  of  the  
association   is   composed   of   the   same   members   that   had   conducted   and   supervised   the   election   of   the  
members  of  the  BOD  that  was  declared  null  and  void  by  the  Court;  that  in  view  thereof  it  would  be  inequitable  
to  allow  them  to  conduct  and  supervise  again  the  forth-­‐‑coming  election;  that  the  election  to  be  conducted  
and  supervised  by  the  said  committee  would  not  be  held  in  accordance  with  the  constitution  and  by-­‐‑laws  of  
the  association  providing  for  five  days  notice  to  the  members  before  the  election,  since  the  notice  was  posted  
and  sent  out  only  on  26  March,  and  the  election  would  be  held  on  28  March,  or  two  days  after  notice;  that  the  
notice   that   beginning   26   March   any   member   could   secure   his   ballot   and   proxy   from   the   office   of   the  
association  is  in  violation  of  section  5,  article  III  of  the  constitution  and  by-­‐‑laws,  which  prohibits  voting  by  
proxy   in   the   election   of   members   of   the   board   of   directors,   and   that   the   defendants   did   not   show   that  
arrangement   is   being   made   "to   guarantee   that   the   election   will   be   held   in   accordance   with   the   constitution  
and  by-­‐‑laws  and  by  the  law."  They  prayed  that  the  Court  appoint  its  representative  or  representatives,  whose  
compensation  shall  be  paid  out  of  the  funds  of  the  association,  to  supervise  and  conduct  the  election  ordered  
by  it.  
 
On  the  same  day,  27  March,  the  Court  entered  an  order  providing  as  follows:  
 
.   .   .   the   Court   hereby   orders   that   the   election   scheduled   for   March   28,   1957   be,   as   it   hereby   is,  
cancelled,   and   a   committee   of   three   is   hereby   constituted   and   appointed   to   call,   conduct   and  
supervise  the  election  of  the  members  of  the  BOD  for  1957,  said  committee  to  be  composed  of:  Mr.  

   
3H  A.Y.  2017-­‐2018   133  
 
 

CORPORATION  LAW  CASE  DIGESTS  –  ATTY.  DANTE  DELA  CRUZ  

Viernes  as  representative  of  the  Court  and  to  act  as  Chairman;  and  one  representative  each  from  the  
plaintiffs   and   defendants,   as   members,   as   members.   The   committee   is   vested   with   the   sole   and  
exclusive  power  and  authority  to  call  conduct  and  supervise  the  election  of  the  members  of  the  BOD  
for  the  year  1957.  
 
The  chairman  of  the  committee  shall  receive  a  compensation  of  P50.00  per  day  and  the  members  
thereof  P30.00  each  per  day,  said  compensation  to  be  paid  by  the  association.  
 
The   defendants   moved   for   reconsideration   of   the   foregoing   order   but   the   Court   denied   the   motion   for  
reconsideration.  
 
Issues:  Whether  or  not  the  Court,  in  issuing  the  order  to  appoint  the  members  of  the  committee  acted  without  
or  in  excess  of  jurisdiction  or  with  grave  abuse  of  discretion?  
 
Held:  Section  3,  article  III,  of  the  constitution  and  by-­‐‑laws  of  the  association  provides:  
 
Notice  of  the  time  and  place  of  holding  of  any  annual  meeting,  or  any  special  meeting,  of  the  members,  
shall  be  given  either  by  posting  the  same  in  a  postage  prepaid  envelope,  addressed  to  each  member  
on  record  at  the  address  left  by  such  member  with  the  Secretary  of  the  Association,  or  at  his  known  
post-­‐‑office  address,  or  by  delivering  the  same  in  person,  at  least  five  (5)  days  before  the  date  set  for  
such  meeting.  .  .  .  In  lieu  of  addressing  or  serving  personal  notices  to  the  members,  notice  of  a  regular  
annual  meeting  or  of  a  special  meeting  of  the  members  may  be  given  by  posting  copies  of  said  notice  
at  the  different  departments  and  plants  of  the  San  Miguel  Brewery  Inc.,  not  less  than  five  (5)  days  
prior  to  the  date  of  the  meeting.  
 
Notice  of  a  special  meeting  of  members  should  be  given  at  least  five  days  before  the  date  of  the  meeting.  It  
appears  that  the  notice  was  posted  on  26  March  and  the  election  was  set  for  28  March.  Therefore,  the  five  
days  previous  notice  required  would  not  be  complied  with.  
 
As  regards  the  creation  of  a  committee  of  three  vested  with  the  authority  to  call,  conduct  and  supervise  the  
election,  and  the  appointment  thereto  of  Candido  C.  Viernes  as  chairman  and  representative  of  the  court  and  
one  representative  each  from  the  parties,  the  Court  in  the  exercise  of  its  equity  jurisdiction  may  appoint  such  
committee,  it  having  been  shown  that  the  Election  Committee  provided  for  in  section  7  of  the  by-­‐‑laws  of  the  
association   that   conducted   the   election   annulled   by   the   respondent   court   if   allowed   to   act   as   such   may  
jeopardize  the  rights  of  the  respondents.  
 
In  a  proper  proceeding  a  court  of  equity  may  direct  the  holding  of  a  stockholders'  meeting  under  the  control  
of  a  special  master,  and  the  action  taken  at  such  a  meeting  will  not  be  set  aside  because  of  a  wrongful  use  of  
the  court's  interlocutory  decree,  where  not  brought  to  the  attention  of  the  court  prior  to  the  meeting.  
 
A  court  equity  may,  on  showing  of  good  reason,  appoint  a  master  to  conduct  and  supervise  an  election  of  
directors  when  it  appears  that  a  fair  election  cannot  otherwise  be  had.  Such  a  court  cannot  make  directions  
contrary  to  statute  and  public  policy  with  respect  to  the  conduct  of  such  election.  
 
 
 
100.  DOMINGO  PONCE  AND  BUHAY  L.  PONCE  VS.  DEMETRIO  B.  ENCARNACION,  JUDGE  OF  THE  COURT  
OF  FIRST  INSTANCE  OF  MANILA,  BRANCH  I,  AND  POTENCIANO  GAPOL,  RESPONDENTS  
G.R.  NO.  L-­‐‑5883.  NOVEMBER  28,  1953.    
PADILLA,  J.    

   
3H  A.Y.  2017-­‐2018   134  
 
 

CORPORATION  LAW  CASE  DIGESTS  –  ATTY.  DANTE  DELA  CRUZ  

 
 
DOCTRINE:With  respect  to  Sec.  26  of  the  Corporation  Law,  The  requirement  that  "on  the  showing  of  good  
cause  therefor,"  the  court  may  grant  to  a  stockholder  the  authority  to  call  such  meeting  and  to  preside  thereat  
does  not  mean  that  the  petition  must  be  set  for  hearing  with  notice  served  upon  the  board  of  directors  –it  
may  be  granted  ex  parte.    
 
The  alleged  illegality  of  the  election  of  one  member  of  the  board  of  directors  at  the  meeting  called  cannot  
affect   the   validity   and   legality   of   the   order.   If   it   be   true,   the   remedy   of   the   aggrieved   party   would   be   quo  
warranto.      
 
FACTS:  Daguhoy  Enterprises,  Inc.,  is  duly  registered  corporation  in  the  Philippines;  that  on  16  April  1951  at  
a  meeting  duly  called,  the  voluntary  dissolution  of  the  corporation  and  the  appointment  of  Potenciano  Gapol  
as  receiver  were  agreed  upon  and  to  that  end  a  petition  for  voluntary  dissolution  was  drafted  which  was  sent  
to,   and   signed   by,   the   petitioner   Domingo   Ponce.   Instead   of   filing   the   petition   for   voluntary   dissolution,  
respondent  Potenciano  Gapol,  who  is  the  largest  stockholder,  changed  his  mind  and  filed  a  complaint  in  the  
CFI   to   compel   the   petitioners   to   render   an   accounting   of   the   funds   and   assets   of   the   corporation,   among  
others.    
 
On  18  May  1951  the  plaintiff  in  that  case,  the  respondent  Potenciano  Gapol  in  this  case,  filed  a  motion  praying  
that  the  petitioners  be  removed  as  members  of  the  board  of  directors  which  was  denied  by  the  court;  Gapol  
then  filed  a  petition  praying  for  an  order  directing  him  to  call  a  meeting  of  the  stockholders  of  the  corporation  
and   to   preside   at   such   meeting   in   accordance   with   section   26   of   the   Corporation   Law.   2   days   later,   without  
notice  to  the  petitioners  and  to  the  other  members  of  the  board  of  directors,  he  respondent  court  issued  the  
order  as  prayed  for.      
ISSUE:  Whether  or  not  the  respondent  correctly  issued  said  order    
HELD:   YES.   Section   26   of   Act   No.   1459,   known   as   the   Corporation   Law,   the   respondent   court   may   issue   the  
order  complained  of.  Said  section  provides:  —    
Whenever,  from  any  cause,  there  is  no  person  authorized  to  call  a  meeting,  or  when  the  officer  
authorized  to  do  so  refuses,  fails,  or  neglects  to  call  a  meeting,  any  judge  of  a  Court  of  First  Instance,  
on   the   showing   of   good   cause   therefor,   may   issue   an   order   to   any   stockholder   or   member   of   a  
corporation,  directing  him  to  call  a  meeting  of  the  corporation  by  giving  the  proper  notice  required  
by  this  Act  or  the  by-­‐‑laws;  and  if  there  be  no  person  legally  authorized  to  preside  at  such  meeting,  
the  judge  of  the  Court  of  First  Instance  may  direct  the  person  calling  the  meeting  to  preside  at  the  
same  until  a  majority  of  the  members  or  stockholders  representing  a  majority  of  the  stock  present  
and   permitted   by   law   to   be   voted   have   chosen   one   of   their   number   to   act   as   presiding   officer   for  
the  purposes  of  the  meeting.    
On  the  showing  of  good  cause  therefor,  the  court  may  authorize  a  stockholder  to  call  a  meeting  and  to  preside  
thereat   until   the   majority   stockholders   representing   a   majority   of   the   stock   present   and   permitted   to   be  
voted  shall  have  chosen  one  among  them  to  preside  it.  And  this  showing  of  good  cause  therefor  exists  when  
the   court   is   apprised   of   the   fact   that   the   by-­‐‑laws   of   the   corporation   require   the   calling   of   a   general   meeting  
of  the  stockholders  to  elect  the  board  of  directors  but  the  call  for  such  meeting  has  not  been  done.    
The   requirement   that   "on   the   showing   of   good   cause   therefor,"   the   court   may   grant   to   a   stockholder   the  
authority  to  call  such  meeting  and  to  preside  thereat  does  not  mean  that  the  petition  must  be  set  for  hearing  
with  notice  served  upon  the  board  of  directors.  The  respondent  court  was  satisfied  that  there  was  a  showing  
of  good  cause  for  authorizing  the  respondent  Potenciano  Gapol  to  call  a  meeting  of  the  stockholders  for  the  
purpose  of  electing  the  board  of  directors  as  required  and  provided  for  in  the  by-­‐‑laws,  because  the  chairman  
of  the  board  of  directors  called  upon  to  do  so  had  failed,  neglected,  or  refused  to  perform  his  duty.  It  may  be  
likened  to  a  writ  of  preliminary  injunction  or  of  attachment  which  may  be  issued  ex-­‐‑parte  upon  compliance  
with  the  requirements  of  the  rules  and  upon  the  court  being  satisfied  that  the  same  should  issue.    

   
3H  A.Y.  2017-­‐2018   135  
 
 

CORPORATION  LAW  CASE  DIGESTS  –  ATTY.  DANTE  DELA  CRUZ  

The  alleged  illegality  of  the  election  of  one  member  of  the  board  of  directors  at  the  meeting  called  by  the  
respondent  Potenciano  Gapol  as  authorized  by  the  court  being  subsequent  to  the  order  complained  of  cannot  
affect  the  validity  and  legality  of  the  order.  If  it  be  true  that  one  of  the  directors  elected  at  the  meeting  called  
by  the  respondent  Potenciano  Gapol,  as  authorized  by  the  order  of  the  court  complained  of,  was  not  qualified  
in  accordance  with  the  provisions  of  the  by-­‐‑laws,  the  remedy  of  an  aggrieved  party  would  be  quo  warranto.  
Also,  the  alleged  previous  agreement  to  dissolve  the  corporation  does  not  affect  or  render  illegal  the  order  
issued  by  the  respondent  court.    
 
 
 
101.    DETECTIVE  &  PROTECTIVE  BUREAU,  INC  VS.  THE  HONORABLE  GAUDENCIO  CLORIBEL  
G.R.  NO.  L-­‐‑23428.  NOVEMBER  29,  1968.  
ZALDIVAR,  J  
 
DOCTRINES:    
•   Every   director   must   own   in   his   own   right   at   least   one   share   of   the   capital   stock   of   the   stock  
corporation   of   which   he   is   a   director,   which   stock   shall   stand   in   his   name   on   the   books   of   the  
corporation  (Sec.  30,  Corporation  Law).  So  that,  if  the  By-­‐‑Laws  of  the  Corporation  provides  that  "The  
manager  shall  be  elected  by  the  Board  of  Directors  from  among  its  members,"  one  could  not  be  a  
managing  director  of  said  corporation  unless  he  owns  at  least  one  share  of  stock  thereof.    
•   Where   ownership   of   the   controlling   interest   in   the   corporation   is   in   dispute,   the   party   in   control   or  
in   possession   of   the   disputed   interest   is   presumed   to   have   the   better   right   (to   the   position   of  
managing   corporate   director)   until   the   contrary   is   adjudged,   and   hence,   that   party   should   not   be  
deprived  of  the  control  or  possession  until  the  court  is  prepared  to  adjudicate  the  controverted  right  
in  favor  of  the  other  party    
 
FACTS:    
Plaintiff   was   a   corporation   duly   organized   and   existing   under   the   laws   of   the   Philippines.   Defendant   was  
managing  director  of  plaintiff  corporation.  In  June  1963,  defendant  allegedly  illegally  seized  and  took  control  
of  all  the  assets  as  well  as  the  books,  records,  vouchers  and  receipts  of  the  corporation  from  the  accountant-­‐‑
cashier,  concealed  them  illegally  and  refused  to  allow  any  member  of  the  corporation  to  see  and  examine  the  
same.   On   January   14,   1964,   the   stockholders,   in   a   meeting,   removed   defendant   as   managing   director   and  
elected   Jose   de   la   Rosa   in   his   stead.   However,   defendant   not   only   refused   to   vacate   his   office   and   to   deliver  
the  assets  and  books  to  Jose  de  la  Rosa,  but  also  continued  to  allegedly  perform  unauthorized  acts  for  and  in  
behalf   of   plaintiff   corporation.     Plaintiff   then   prayed   that   a   preliminary   injunction   ex-­‐‑parte   be   issued  
restraining  defendant  from  exercising  the  functions  of  managing  director  and  from  disbursing  and  disposing  
of   its   funds;   that   Jose   M.   Barredo   be   appointed   receiver;   that,   after   judgment,   the   injunction   be   made  
permanent   and   defendant   be   ordered   to   render   an   accounting.   Respondent   Judge   granted   the   writ   of  
preliminary  injunction  prayed  for,  conditioned  upon  plaintiff's  filing  a  bond  of  P5,000.00.  Plaintiff  filed  the  
bond,   but   while   the   same   was   pending   approval,   defendant   Fausto   S.   Alberto   filed   a   motion   to   admit   a  
counter-­‐‑bond   for   the   purpose   of   lifting   the   order   granting   the   writ   of   preliminary   injunction.   Alberto  
contended  that  he  really  was  the  owner  of  the  controlling  interest  in  the  business,  having  invested  therein  a  
total  of  P57,727.29  as  against  the  sum  of  P4,000  only  invested  by  one  other  director,  Jose  M.  Barredo.  In  spite  
of  the  opposition  filed  by  plaintiff,  respondent  Judge  issued  an  order  admitting  the  counter-­‐‑bond  and  setting  
aside  the  writ  of  preliminary  injunction.  Plaintiff  then  filed  with  the  Court  the  instant  petition  for  certiorari,  
praying   that   a   writ   of   preliminary   injunction   enjoining   defendant   Fausto   S.   Alberto   from   exercising   the  
functions   of   managing   director   be   issued,   and   that   the   order   dated   August   5,   1964   of   respondent   Judge  
approving  the  counter-­‐‑bond  and  lifting  the  writ  of  preliminary  injunction  he  had  previously  issued  be  set  
aside  and  declared  null  and  void.    
 

   
3H  A.Y.  2017-­‐2018   136  
 
 

CORPORATION  LAW  CASE  DIGESTS  –  ATTY.  DANTE  DELA  CRUZ  

ISSUE:  
Whether  the  order  of  respondent  Judge  admitting  and  approving  the  counter-­‐‑bond  and  setting  aside  the  writ  
of  preliminary  injunction,  was  issued  contrary  to  law  and  with  grave  abuse  of  discretion.    
 
 
HELD:  
NO.  There  is  in  the  record  no  showing  that  Jose  de  la  Rosa  owned  a  share  of  stock  in  the  corporation.  If  he  did  
not   own   any   share   of   stock,   certainly   he   could   not   be   a   director   pursuant   to   the   mandatory   provision   of  
Section  30  of  the  Corporation  Law,  which  in  part  provides:  "Sec.  30.  Every  director  must  own  in  his  own  right  
at  least  one  share  of  the  capital  stock  of  the  stock  corporation  of  which  he  is  a  director,  which  stock  shall  
stand  in  his  name  on  the  books  of  the  corporation  .."  If  he  could  not  be  a  director,  he  could  also  not  be  a  
managing   director   of   the   corporation,   pursuant   to   Article   V,   Section   3   of   the   By-­‐‑Laws   of   the   Corporation  
which  provides  that:.  "The  manager  shall  be  elected  by  the  Board  of  Directors  from  among  its  members  .  .  ."  
If  the  managing  director-­‐‑elect  was  not  qualified  to  become  managing  director,  respondent  Fausto  Alberto  
could  not  be  compelled  to  vacate  his  office  and  cede  the  same  to  the  managing  director-­‐‑elect  because  the  by-­‐‑
laws   of   the   corporation   provides   in   Article   IV,   Section   1   that   "Directors   shall   serve   until   the   election   and  
qualification  of  their  duly  qualified  successor."    
 
Furthermore,  where  ownership  is  in  dispute,  the  party  in  control  or  possession  of  the  disputed  interest  is  
presumed  to  have  the  better  right  until  the  contrary  is  adjudged,  and  hence  that  party  should  not  be  deprived  
of  the  control  or  possession  until  the  court  is  prepared  to  adjudicate  the  controverted  right  in  favor  of  the  
other  party.  Should  it  be  the  truth  that  respondent  Alberto  is  the  controlling  stockholder,  then  the  damages  
said  respondent  would  suffer  would  be  the  same,  if  not  more,  as  the  damages  that  the  corporation  would  
suffer  if  the  injunction  were  maintained.  If  the  bond  of  P5,000  filed  by  petitioner  for  the  injunction  would  be  
sufficient   to   answer   for   the   damages   that   would   be   suffered   by   respondent   Alberto   by   reason   of   the  
injunction,   there   seems   to   be   no   reason   why   the   same   amount   would   not   be   sufficient   to   answer   for   the  
damages  that  might  be  suffered  by  the  petitioning  corporation  by  reason  of  the  lifting  of  the  injunction.    
 
 
 
 
102.  BALDOMERO  ROXAS  V.  HONORABLE  MARIANO  DE  LA  ROSA  
G.R.  NO.  L-­‐‑26555        NOVEMBER  16,  1926  
STREET,  J.;  
 
 
FACTS:  
-­‐‑   Binalbagan  Estate,  Inc.,  is  a  corporation  engaged  in  the  manufacture  of  raw  sugar  from  canes  grown  
upon  farms  accessible  to  its  central.    
-­‐‑   Possessors  of  a  majority  of  the  shares  of  the  Binalbagan  Estate,  Inc.,  formed  a  voting  trust  composed  
of  three  members.  
-­‐‑   Various  substitutions  have  been  made  in  the  personnel  of  the  voting  trust,  and  at  the  present  time  
the  petitioners  Roxas,  Echaus,  and  Lacson  presumably  constitute  its  membership.  
-­‐‑   The  present  officers  of  the  Binalbagan  Estate,  Inc.,  were  elected  by  the  representative  of  the  voting  
trust.  
-­‐‑   The   petitioners   in   their   character   as   members   of   the   voting   trust,   on   August   2,   1926,   caused   the  
secretary   of   the   Binalbagan   Estate,   Inc.,   to   issue   to   the   shareholders   a   notice   calling   for   a   special  
general  meeting  of  shareholders  to  be  held  at  10  a.  m.,  on  August  16,  1926,  "for  the  election  of  the  
board   of   directors,   for   the   amendment   of   the   By-­‐‑Laws,   and   for   any   other   business   that   can   be   dealt  
with  in  said  meeting."  

   
3H  A.Y.  2017-­‐2018   137  
 
 

CORPORATION  LAW  CASE  DIGESTS  –  ATTY.  DANTE  DELA  CRUZ  

-­‐‑  
The  respondents  file  a  preliminary  injunction  against  the  trustees  and  the  Binalbagan  Estate,  Inc.,  for  
the  purpose  of  enjoining  the  meeting.  
-­‐‑   the  respondent  judge  issued  the  restraining  order,  or  preliminary  injunction,  which  gave  rise  to  the  
present  petition  for  certiorari.  
ISSUE:   Whether   the   making   of   the   injunction   order   was   beyond   the   legitimate   powers   of   the   respondent  
judge.  
HELD:  
•   The   contention   is   untenable   and   that   the   respondent   judge   acted   within   his   legitimate   powers   in  
making  the  order  against  which  relief  is  sought.    
•   Under   the   law   the   directors   of   a   corporation   can   only   be   removed   from   office   by   a   vote   of   the  
stockholders  representing  at  least  two-­‐‑thirds  of  the  subscribed  capital  stock  entitled  to  vote  (Act  No.  
1459,  sec.  34);  while  vacancies  in  the  board,  when  they  exist,  can  be  filled  by  mere  majority  vote,  (Act  
No.  1459,  sec.  25).    
•   Moreover,   the   law   requires   that   when   action   is   to   be   taken   at   a   special   meeting   to   remove   the  
directors,  such  purpose  shall  be  indicated  in  the  call  (Act  No.  1459,  sec.  34).  
•   Upon   examining   into   the   number   of   shares   controlled   by   the   voting   trust,   it   will   be   seen   that,   while  
the  trust  controls  a  majority  of  the  stock,  it  does  not  have  a  clear  two-­‐‑thirds  majority.    
•   It  was  therefore  impolitic  for  the  petitioners,  in  forcing  the  call  for  the  meeting  of  August  16.  
•   Instead,  the  call  was  limited  to  the  election  of  the  board  of  directors,  it  being  the  evident  intention  of  
the  voting  trust  to  elect  a  new  board  as  if  the  directorate  had  been  then  vacant.  
 
 
 
103.   HIGINIO   ANGELES,   JOSE   E.   LARA   AND   AGUEDO   BERNABE,    
AS  STOCKHOLDERS  FOR  AN  IN  BEHALF  AND  FOR  THE  BENEFIT  OF  THE  CORPORATION,  PARAÑAQUE  
RICE   MILL,   INC.   AND   THE   OTHER   STOCKHOLDERS   WHO   MAY   DESIRE   TO   JOIN,   PLAINTIFFS-­‐‑
APPELLEES,  VS.  TEODORICO  B.  SANTOS,  ESTANISLAO  MAYUGA,  APOLONIO  PASCUAL,  AND  BASILISA  
RODRIGUEZ,DEFENDANT-­‐‑APPELLANTS.  
G.R.  NO.  L-­‐‑43413              AUGUST  31,  1937  
LAUREL,  J.:  
 
 
DOCTRINE:    
•   It   is   well   settled   in   this   jurisdiction   that   where   corporate   directors   are   guilty   of   a   breach   of   trust  —  
not  of  mere  error  of  judgment  or  abuse  of  discretion  —  and  intracorporate  remedy  is  futile  or  useless,  
a  stockholder  may  institute  a  suit  in  behalf  of  himself  and  other  stockholders  and  for  the  benefit  of  
the  corporation,  to  bring  about  a  redress  of  the  wrong  inflicted  directly  upon  the  corporation  and  
indirectly  upon  the  stockholers.  
•   There   are   abundant   authorities,   however,   which   hold   that   if   the   court   has   acquire   jurisdiction   to  
appoint   a   receiver   because   of   the   mismanagement   of   directors   these   may   thereafter   be   remove   and  
others  appointed  in  their  place  by  the  court  in  the  exercise  of  its  equity  
 
FACTS:  
Plaintiffs-­‐‑appellees,  as  stockholders,  for  and  in  behalf  of  the  corporation,  Parañaque  Rice  Mill,  Inc.,  filed  a  
complaint   against   herein   respondents,   constituting   a   majority   of   the   board   of   directors   of   the   said  
corporation.    They  allege  that  Teodorico  B.  Santos,  president,  had  appropriated  to  his  own  benefit  properties,  
funds,  and  income  of  the  corporatio;  that  Teodorico  in  connivance  with  his  co-­‐‑defendants,  was  disposing  of  
the   properties   and   records   of   the   corporation   without   authority   from   the   board   of   directors   or   the  
stockholders   of   the   corporation   and   without   making   any   report   of   his   acts   to   the   said   board   of   directors   or  

   
3H  A.Y.  2017-­‐2018   138  
 
 

CORPORATION  LAW  CASE  DIGESTS  –  ATTY.  DANTE  DELA  CRUZ  

to   any   other   officer   of   the   corporation.   The   court   issue   an   ex   parte   order   of   receivership   of   the   corporation  
upon  the  filling  of  a  bond  of  P1,000  by  the  plaintiffs-­‐‑appellees.  The  plaintiffs-­‐‑appellees  renewed  their  petition  
for   the   appointment   of   a   receiver   pendente   lite   alleging,   among   other   things,   that   defendant   Teodorico   was  
using  the  funds  of  the  corporation  for  purely  personal  ends;  that  said  defendant  did  not  render  any  account  
of  his  management  or  for  the  condition  of  the  business  of  the  corporation;  that  since  1932  said  defendant  
called   no   meeting   of   the   board   of   directors   or   of   the   stockholders   thus   enabling   him   to   continue   holding,  
without   any   election,   the   position   of   present   and,   finally,   that   of   manager;   and   that,   without   the   knowledge  
and  consent  of  the  stockholders.  The  defendant-­‐‑appellants  objected  to  the  petition  for  the  appointment  of  a  
receiver   on   the   ground,   among   others,   that   the   court   had   no   jurisdiction   over   the   Parañaque   Rice   Mill,   Inc.,  
because   it   had   not   been   include   as   party   defendant   in   this   case   and   that,   therefore   the   court   could   not  
properly  appoint  a  receiver  of  the  corporation  pendente  lite.  The  trial  court  by  order,  appointed  a  receiver  
and  thereafter,  ruled  in  favour  of  the  plaintiffs,  ordering  the  defendant  Teodorico  to  render  account  of  the  
property,   funds   and   income   of   the   corporation   Parañaque   Rice   Mill,   Inc.,   from   1931   to   the   present;   and  
dismissing   the   defendants   of   their   position   as   directors   of   the   corporation   until   the   new   election   by   the  
shareholders  that  will  be  convened  once  this  judgment  is  signed.  
 
ISSUES:  
1.  The  lower  court  erred  in  holding  that  it  has  jurisdiction  to  appoint  a  receiver  of  the  corporation.    
2.  The  lower  court  erred  in  ordering  the  destitution  of  the  defendants  from  their  office  as  members  of  the  
board  of  directors  of  the  corporation,  until  the  new  election  of  the  stockholders  which  shall  be  held  once  the  
decision  has  become  final.  
 
HELD:  
It  is  well  settled  in  this  jurisdiction  that  where  corporate  directors  are  guilty  of  a  breach  of  trust  —  not  of  
mere  error  of  judgment  or  abuse  of  discretion  —  and  intracorporate  remedy  is  futile  or  useless,  a  stockholder  
may  institute  a  suit  in  behalf  of  himself  and  other  stockholders  and  for  the  benefit  of  the  corporation,  to  bring  
about  a  redress  of  the  wrong  inflicted  directly  upon  the  corporation  and  indirectly  upon  the  stockholers.  The  
contention  of  the  defendants  in  the  case  at  bar  that   the  Parañaque  Rice  Mill,  Inc.,  should  have  been  brought  
in  as  necessary  party  and  the  action  maintained  in  its  name  and  in  its  behalf  directly  states  the  general  rule,  
but  not  the  exception  recognize  by  this  court.  The  action  having  been  properly  brought  and  by  the  lower  
court   entertained   it   was   within   its   power,   upon   proper   showing,   to   appoint   a   receiver   of   the  
corporation  pendente  lite.  The  receivership  in  this  case  shall  continue  until  a  new  board  of  directors  shall  
have  been  elected  and  the  corporation.  
 
Our  Corporation  Law  (Act  No.  1459,  as  amended),  does  not  confer  expressly  upon  the  court  the  power  to  
remove  a  director  of  a  corporation.  There  are  abundant  authorities,  however,  which  hold  that  if  the  court  has  
acquire   jurisdiction   to   appoint   a   receiver   because   of   the   mismanagement   of   directors   these   may   thereafter  
be  remove  and  others  appointed  in  their  place  by  the  court  in  the  exercise  of  its  equity.  In  the  present  case,  
however,   the   properties   and   assets   of   the   corporation   being   amply   protected   by   the   appointment   of   a  
receiver  and  view  of  the  statutory  provisions  above  referred  to,  we  are  of  the  opinion  that  the  removal  of  the  
directors  is,  under  the  circumstances,  unnecessary  and  unwarranted.    
 
 
 
104.  JOSE  A.  BERNAS,  CECILE  H.  CHENG,  VICTOR  AFRICA,  JESUS  B.  MARAMARA,  JOSE  T.  FRONDOSO,  
IGNACIO   T.   MACROHON,   JR.,   AND   PAULINO   T.   LIM,   ACTING   IN   THEIR   CAPACITY   ASINDIVIDUAL  
DIRECTORS   OF   MAKATI   SPORTS   CLUB,   INC.,   AND   ON   BEHALF   OF   THE   BOARD   OF   DIRECTORS   OF  
MAKATI   SPORTS   CLU   VS.   JOVENCIO   F.   CINCO,   VICENTE   R.   AYLLON,   RICARDO   G.   LIBREA,   SAMUEL   L.  
ESGUERRA,  ROLANDO  P.  DELA  CUESTA,  RUBEN  L.  TORRES,  ALEX  Y.  PARDO,  MA.  CRISTINA  SIM,  ROGER  
T.  AGUILING,  JOSE  B.  QUIMSON,  CELESTINO  L.  ANG,  ELISEO  V.  VILLAMOR,  FELIPE  L.  GOZON,  CLAUDIO  

   
3H  A.Y.  2017-­‐2018   139  
 
 

CORPORATION  LAW  CASE  DIGESTS  –  ATTY.  DANTE  DELA  CRUZ  

B.  ALTURA,  ROGELIO  G.  VILLAROSA,  MANUEL  R.  SANTIAGO,  BENJAMIN  A.  CARANDANG,  REGINA  DE  
LEON-­‐‑HERLIHY,  CARLOS  Y.  RAMOS,  JR.,  ALEJANDRO  Z.  BARIN,  EFRENILO  M.  CAYANGA  AND  JOHN  DOES  
G.R.  NOS.  163356-­‐‑57.  JULY  1,  2015  
J.  PEREZ      
 
DOCTRINE:  A  corporation's  board  of  directors  is  understood  to  be  that  body  which  (1)  exercises  all  powers  
provided  for  under  the  Corporation  Code;  (2)  conducts  all  business  of  the  corporation;  and  (3)  controls  and  
holds   all   the   property   of   the   corporation.   Its   members   have   been   characterized   as   trustees   or   directors  
clothed  with  fiduciary  character.  It  is  ineluctably  clear  that  the  fiduciary  relation  is  between  the  stockholders  
and  the  board  of  directors  and  who  are  vested  with  the  power  to  manage  the  affairs  of  the  corporation.  The  
ordinary   trust   relationship   of   directors   of   a   corporation   and   stockholders   is   not   a   matter   of   statutory   or  
technical  law.  It  springs  from  the  fact  that  directors  have  the  control  and  guidance  of  corporate  affairs  and  
property  and  hence  of  the  property  interests  of  the  stockholders.  
 
 
FACTS:  Makati  Sports  Club  (MSC)  is  a  domestic  corporation  duly  organized  and  existing  under  Philippine  
laws  for  the  primary  purpose  of  establishing,  maintaining,  and  providing  social,  cultural,  recreational  and  
athletic  activities  among  its  members.  
 
Alarmed   with   the   rumored   anomalies   in   handling   the   corporate   funds,   the   MSC   Oversight   Committee  
(MSCOC),  composed  of  the  past  presidents  of  the  club,  demanded  from  the  Bernas  Group,  who  were  then  
incumbent   officers   of   the   corporation,   to   resign   from   their   respective   positions   to   pave   the   way   for   the  
election  of  new  set  of  officers.  Resonating  this  clamor  were  the  stockholders  of  the  corporation  representing  
at  least  100  shares  who  sought  the  assistance  of  the  MSCOC  to  call  for  a  special  stockholders  meeting  for  the  
purpose  of  removing  the  sitting  officers  and  electing  new  ones.  Pursuant  to  such  request,  the  MSCOC  called  
a  Special  Stockholders'  Meeting  and  sent  out  notices  to  all  stockholders  and  members  stating  therein  the  
time,  place  and  purpose  of  the  meeting.  For  failure  of  the  Bernas  Group  to  secure  an  injunction  before  the  
Securities  Commission  (SEC),  the  meeting  proceeded  wherein  Jose  A.  Bernas,  Cecile  H.  Cheng,  Victor  Africa,  
Jesus  Maramara,  Jose  T.  Frondoso,  Ignacio  T.  Macrohon,  Jr.  and  Paulino  T.  Lim  were  removed  from  office  and,  
in   their   place   and   stead,   Jovencio   F.   Cinco,   Ricardo   G.   Librea,   Alex   Y.   Pardo,   Roger   T.   Aguiling,   Rogelio   G.  
Villarosa,   Armando   David,   Norberto   Maronilla,   Regina   de   Leon-­‐‑Herlihy   and   Claudio   B.   Altura,   were   elected.  
Aggrieved  by  the  turn  of  events,  the  Bernas  Group  initiated  an  action  before  the  Securities  Investigation  and  
Clearing  Department  (SICD)  of  the  SEC  docketed  as  SEC  Case  No.  5840  seeking  for  the  nullification  of  the  
December  1997  Special  Stockholders  Meeting  on  the  ground  that  it  was  
improperly  called.  Citing  Section  28  of  the  Corporation  Code,  the  Bernas  Group  argued  that  the  authority  to  
call   a   meeting   lies   with   the   Corporate   Secretary   and   not   with   the   MSCOC   which   functions   merely   as   an  
oversight  body  and  is  not  vested  with  the  power  to  call   corporate   meetings.   For   their   part,   the   Cinco   Group  
insisted  that  the  17  December  1997  Special  Stockholders'  Meeting  is  sanctioned  by  the  Corporation  Code  
and   the   MSC   by-­‐‑laws.   In   justifying   the   call   effected   by   the   MSCOC,   they   reasoned   that   Section   25   of   the   MSC  
by-­‐‑laws  merely  authorized  the  Corporate  Secretary  to  issue  notices  of  meetings  and  nowhere  does  it  state  
that  such  authority  solely  belongs  to  him.    
 
Prior  to  the  resolution  of  SEC  Case  No.  5840,  an  Annual  Stockholders'  Meeting  was  held  on  20  April  1998  
pursuant  to  Section  8  of  the  MSC  bylaws.  During  the  said  meeting,  which  was  attended  by  1,017  stockholders  
representing   2/3   of   the   outstanding   shares,   the   majority   resolved   to   approve,   confirm   and   ratify,   among  
others,  the  calling  and  holding  of  17  December  1997  Special  Stockholders'  Meeting,  the  acts  and  resolutions  
adopted  therein  including  the  removal  of  Bernas  Group  from  the  Board  and  the  election  of  
their   replacements.   The   conduct   of   the   17   December   1997   Special   Stockholders'   Meeting   was   likewise  
ratified  by  the  stockholders  during  the  2000  Annual  Stockholders'  Meeting  which  was  held  on  17  April  2000.  
 

   
3H  A.Y.  2017-­‐2018   140  
 
 

CORPORATION  LAW  CASE  DIGESTS  –  ATTY.  DANTE  DELA  CRUZ  

On  9  May  2000,  the  SICD  rendered  a  Decision  17  in  SEC  Case  No.  12-­‐‑97-­‐‑5840  finding,  among  others,  that  the  
17  December  1997  Special  Stockholders'  Meeting  and  the  Annual  Stockholders'  Meeting  conducted  on  20  
April   1998   and   19   April   1999   are   invalid.   The   SICD   likewise   nullified   the   expulsion   of   Bernas   from   the  
corporation  and  the  sale  of  his  share  at  the  public  auction:  
(a)   The  supposed  Special  Stockholders'  Meeting  of  December  17,  1997  was  prematurely  or  invalidly  called  
by  the  [the  Cinco  Group].  It  therefore  failed  to  produce  any  legal  effects  
(b)  The  April  20,  1998  meeting  was  not  attended  by  a  sufficient  number  of  valid  proxies.  No  quorum  could  
have   been   present   at   the   said   meeting.   No   corporate   business   could   have   been   validly   completed  
and/or   transacted   during   the   said   meeting.   Further,   it   was   not   called   by   the   validly   elected   Corporate  
Secretary  Victor  Africa  nor  presided  over  by  the  validly  elected  president  Jose  A.  Bernas.  
 
On  appeal,  the  SEC  En  Banc,  in  its  12  December  2000  Decision  reversed  the  findings  of  the  SICD  and  validated  
the   holding   of   the   17   December   1997   Special   Stockholders'   Meeting   as   well   as   the   Annual   Stockholders'  
Meeting  held  on  20  April  1998  and  19  April  1999.    On  28  April  2003,  the  Court  of  Appeals  rendered  a  Decision  
declaring   the   17   December   1997   Special   Stockholders'   Meeting   invalid   for   being   improperly   called   but  
affirmed  the  actions  taken  during  the  Annual  Stockholders'  Meeting  held  on  20  April  1998,  19  April  1999  
and  17  April  2000.  
 
ISSUE:  Was  the  meeting  indeed  invalid?    
 
HELD:    No.    
The   Corporation   Code   laid   down   the   rules   on   the   removal   of   the   Directors   of   the   corporation   by   providing,  
inter  alia,  the  persons  authorized  to  call  the  meeting  and  the  number  of  votes  required  for  the  purpose  of  
removal  in  Sec.  28  of  the  Corporation  Code.  Corollarily,  the  pertinent  provisions  of  MSC  by-­‐‑laws  which  govern  
the   manner   of   calling   and   sending   of   notices   of   the   annual   stockholders'   meeting   and   the   special  
stockholders'  meeting  provide:    
 
SEC.  8.  Annual  Meetings.  —  The  annual  meeting  of  stockholders  shall  be  held  at  the  Clubhouse  on  the  
third  Monday  of  April  of  every  year  unless  such  day  be  a  holiday  in  which  case  the  annual  meeting  
shall  be  held  on  the  next  succeeding  business  day  
 
SEC.  10.  Special  Meetings.  —  Special  meetings  of  stockholders  shall  be  held  at  the  Clubhouse  when  
called   by   the   President   or   by   the   Board   of   Directors   or   upon   written   request   of   the   stockholders  
representing  not  less  than  one  hundred  (100)  shares.  
 
SEC.  25.  Secretary.  —  The  Secretary  shall  keep  the  stock  and  transfer  book  and  the  corporate  seal,  
which  he  shall  stamp  on  all  documents  requiring  such  seal,  fill  and  sign  together  with  the  President,  
all  the  certificates  of  stocks  issued,  give  or  caused  to  be  given  all  notices  required  by  law  of  these  By-­‐‑
laws  as  well  as  notices  of  all  meeting  of  the  Board  and  of  the  stockholders;  shall  certify  as  to  quorum  
at  meetings;  shall  approve  and  sign  all  correspondence  pertaining  to  the  Office  of  the  Secretary.    
Textually,   only   the   President   and   the   Board   of   Directors   are   authorized   by   the   by-­‐‑laws   to   call   a   special  
meeting.  In  cases  where  the  person  authorized  to  call  a  meeting  refuses,  fails  or  neglects  to  call  a  meeting,  
then  the  stockholders  representing  at  least  100  shares,  upon  written  request,  may  file  a  petition  to  call  a  
special   stockholder's   meeting.   In   the   instant   case,   there   is   no   dispute   that   the   17   December   1997   Special  
Stockholders'  Meeting  was  called  neither  by  the  President  nor  by  the  Board  of  Directors  but  by  the  MSCOC.  
While  the  MSCOC,  as  its  name  suggests,  is  created  for  the  purpose  of  overseeing  the  affairs  of  the  corporation,  
nowhere  in  the  by-­‐‑laws  does  it  state  that  it  is  authorized  to  exercise  corporate  powers,  such  as  the  power  to  
call  a  special  meeting,  solely  vested  by  law  and  the  MSC  by-­‐‑laws  on  the  President  or  the  Board  of  Directors.  
A  corporation's  board  of  directors  is  understood  to  be  that  body  which  (1)  exercises  all  powers  provided  for  
under  the  Corporation  Code;  (2)  conducts  all  business  of  the  corporation;  and  (3)  controls  and  holds  all  the  

   
3H  A.Y.  2017-­‐2018   141  
 
 

CORPORATION  LAW  CASE  DIGESTS  –  ATTY.  DANTE  DELA  CRUZ  

property   of   the   corporation.   Its   members   have   been   characterized   as   trustees   or   directors   clothed   with  
fiduciary   character.   It   is   ineluctably   clear   that   the   fiduciary   relation   is   between   the   stockholders   and   the  
board  of  directors  and  who  are  vested  with  the  power  to  manage  the  affairs  of  the  corporation.  The  ordinary  
trust  relationship  of  directors  of  a  corporation  and  stockholders  is  not  a  matter  of  statutory  or  technical  law.  
It  springs  from  the  fact  that  directors  have  the  control  and  guidance  of  corporate  affairs  and  property  and  
hence  of  the  property  interests  of  the  stockholders.  
Relative  to  the  powers  of  the  Board  of  Directors,  nowhere  in  the  Corporation  Code  or  in  the  MSC  by-­‐‑laws  can  
it  be  gathered  that  the  Oversight  Committee  is  authorized  to  step  in  wherever  there  is  breach  of  fiduciary  
duty   and   call   a   special   meeting   for   the   purpose   of   removing   the   existing   officers   and   electing   their  
replacements  even  if  such  call  was  made  upon  the  request  of  shareholders.  Needless  to  say,  the  MSCOC  is  
neither  empowered  by  law  nor  the  MSC  by-­‐‑laws  to  call  a  meeting  and  the  subsequent  ratification  made  by  
the  stockholders  did  not  cure  the  substantive  infirmity,  the  defect  having  set  in  at  the  time  the  void  act  was  
done.  The  defect  goes  into  the  very  authority  of  the  persons  who  made  the  call  for  the  meeting.  It  is  apt  to  
recall  that  illegal  acts  of  a  corporation  which  contemplate  the  doing  of  an  act  which  is  contrary  to  law,  morals  
or   public   order,   or   contravenes   some   rules   of   public   policy   or   public   duty,   are,   like   similar   transactions  
between  individuals,  void.  They  cannot  serve  as  basis  for  a  court  action,  nor  acquire  validity  by  performance,  
ratification  or  estoppel.  
Consequently,   such   Special   Stockholders'   Meeting   called   by   the   Oversight   Committee   cannot   have   any   legal  
effect.  The  removal  of  the  Bernas  Group,  as  well  as  the  election  of  the  Cinco  Group,  effected  by  the  assembly  
in  that  improperly  called  meeting  is  void,  and  since  the  Cinco  Group  has  no  legal  right  to  sit  in  the  board,  their  
subsequent   acts   of   expelling   Bernas   from   the   club   and   the   selling   of   his   shares   at   the   public   auction,   are  
likewise  invalid.  The  Cinco  Group  cannot  invoke  the  application  of  de  facto  officership  doctrine  to  justify  the  
actions  taken  after  the  invalid  election  since  the  operation  of  the  principle  is  limited  to  third  persons  who  
were  originally  not  part  of  the  corporation  but  became  such  by  reason  of  voting  of  government-­‐‑sequestered  
shares.    
The  case  would  have  been  different  if  the  petitioning  stockholders  went  directly  to  the  SEC  and  sought  its  
assistance  to  call  a  special  stockholders'  meeting  citing  the  previous  refusal  of  the  Corporate  Secretary  to  call  
a  meeting.  Where  there  is  an  officer  authorized  to  call  a  meeting  and  that  officer  refuses,  fails,  or  neglects  to  
call  a  meeting,  the  SEC  can  assume  jurisdiction  and  issue  an  order  to  the  petitioning  stockholder  to  call  a  
meeting  pursuant  to  its  regulatory  and  administrative  powers  to  implement  the  Corporation  Code.  Given  the  
broad  administrative  and  regulatory  powers  of  the  SEC  outlined  under  Section  50  of  the  Corporation  Code  
and  Section  6  of  Presidential  Decree  (PD)  No.  902-­‐‑A,  the  Cinco  Group  cannot  claim  that  if  was  left  without  
recourse  after  the  Corporate  Secretary  previously  refused  to  heed  its  demand  to  call  a  special  stockholders'  
meeting.  If  it  be  true  that  the  Corporate  Secretary  refused  to  call  a  meeting  despite  fervent  demand  from  the  
MSCOC,  the  remedy  of  the  stockholders  would  have  been  to  file  a  petition  to  the  SEC  to  direct  him  to  call  a  
meeting  by  giving  proper  notice  required  under  the  Code  
 
 
 
105.  DE  LA  RAMA  VS.  MA-­‐‑AO  SUGAR  CENTRAL  CO.,  INC  
G.R.  NO.  L-­‐‑17504  &  L-­‐‑17506                          FEBRUARY  28,  1969  
CAPISTRANO,  J.:  
 
DOCTRINE:  An  investment  of  corporate  funds  in  another  corporation,  if  done  in  pursuance  of  the  corporate  
purpose,   does   not   need   the   approval   of   the   stockholders.   But   when   the   purchase   of   shares   of   another  
corporation  is  done  solely  for  investment  and  not  to  accomplish  the  purpose  of  its  incorporation,  the  vote  of  
approval  of  the  stockholders  is  necessary.  
 
Further,  when  the  purpose  is  as  stated  in  its  articles  of  incorporation,  the  approval  of  the  stockholders  is  not  
necessary.The  Corporation  Law  allows  a  corporation  to  invest  its  funds  in  any  other  corporation  or  business,  

   
3H  A.Y.  2017-­‐2018   142  
 
 

CORPORATION  LAW  CASE  DIGESTS  –  ATTY.  DANTE  DELA  CRUZ  

or   for   any   purpose   other   than   the   main   purpose   for   which   it   was   organized,   provided   that   its   board   of  
directors   has   been  so   authorized   by  the  affirmative  vote  of  stockholders  holding  shares  entitling  them  to  
exercise  at  lease  2/3  of  the  voting  power  
 
FACTS:  This  was  a  representative  or  derivative  suit  commenced  by  four  minority  stockholders  against  the  
Ma-­‐‑ao  Sugar  Central  Co.,  Inc.  and  J.  Amado  Araneta  and  three  other  directors  of  the  corporation.  
 
The  complaint  alleges  to  wit:  (1)  for  alleged  illegal  and  ultra-­‐‑vires  acts  consisting  of  self-­‐‑dealing  irregular  
loans,   and   unauthorized   investments;   (2)   for   alleged   gross   mismanagement;   (3)   for   alleged   forfeiture   of  
corporate   rights   warranting   dissolution;   (4)   for   alleged   damages   and   attorney's   fees;   and   (5)   for  
receivership.  
 
Defendants   denied   "the   allegations   regarding   the   supposed   gross   mismanagement,   fraudulent   use   and  
diversion   of   corporate   funds,   disregard   of   corporate   requirements,   abuse   of   trust   and   violation   of   fiduciary  
relationship,   etc.,   supposed   to   have   been   discovered   by   plaintiffs,   all   of   which   are   nothing   but   gratuitous,  
unwarranted,   exaggerated   and   distorted   conclusions   not   supported   by   plain   and   specific   facts   and  
transactions  alleged  in  the  complaint."  
 
It  is  alleged  that  Ma-­‐‑ao  Sugar  Central,  through  its  President  (Araneta),  subscribed  for  Php  300K  worth  of  
capital  stock  of  the  Philippine  Fiber  Processing  Co.At  that  at  the  time  the  first  two  payments  were  made,  there  
was   no   board   resolution   authorizing   such   investment.   It   was   only   a   few   months   after   that   Araneta   was  
authorized  by  the  Board  of  Directors.  
 
It   was   also   alleged   that   355,000   shares   of   stock   of   Philippine   Fiber,   owned   by   Luzon   Industrial,   were  
transferred  to  Ma-­‐‑ao  without  prior  board  resolution.  Such  transfer  however  was  subsequently  approved  
 
RTC:  dismisses  the  petition  for  dissolution  but  condemns  J.  Amado  Araneta  to  pay  unto  Ma-­‐‑ao  Sugar  Central  
Co.,  Inc.  the  amount  of  P46,270.00  with  8%  interest  from  the  date  of  the  filing  of  this  complaint,  plus  the  
costs;   the   Court   reiterates   the   preliminary   injunction   restraining   the   Ma-­‐‑ao   Sugar   Central   Co.,   Inc.  
management  to  give  any  loans  or  advances  to  its  officers  and  orders  that  this  injunction  be  as  it  is  hereby  
made,  permanent;  and  orders  it  to  refrain  from  making  investments  in  Acoje  Mining,  Mabuhay  Printing,  and  
any  other  company  whose  purpose  is  not  connected  with  the  Sugar  Central  business;    
 
ISSUE:  1)  Whether  the  investment  of  the  corporate  funds  by  Ma-­‐‑ao  in  Philippine  Fiber  constitutes  a  violation  of  the  
 Corporation  Law.  
2)  Whether  Ma-­‐‑ao  may  make  investments  in  any  other  company  whose  purpose  is  not  connected  with  the  sugar    
central  business.  
 
HELD:  
 
1)NO.  The  SC  agreed  with  the  finding  of  the  lower  court  that  the  investment  in  question  does  not  fall  under  
the   purview   of   the   Section   17   ½   of   the   Corporation   Law.The   SC   quoted   Prof.   Guevara   in   explaining   the   said  
provision.   Such   an   act,   if   done   in   pursuance   of   the   corporate   purpose,   does   not   need   the   approval   of   the  
stockholders.    
 
But  when  the  purchase  of  shares  of  another  corporation  is  done  solely  for  investment  and  not  to  accomplish  
the   purpose   of   its   incorporation,   the   vote   of   approval   of   the   stockholders   is   necessary.   Also,   when   the  
investment  is  necessary  to  accomplish  its  purpose  or  purposes  as  stated  in  the  articles  of  incorporation,  the  
approval  of  stockholders  is  not  necessary.  
 

   
3H  A.Y.  2017-­‐2018   143  
 
 

CORPORATION  LAW  CASE  DIGESTS  –  ATTY.  DANTE  DELA  CRUZ  

2)  YES.  The  SC  reversed  the  order  of  the  lower  court  refraining  Ma-­‐‑ao  from  making  investments  in  other  
company  whose  purpose  not  connected  with  the  sugar  central  business.  It  reasoned  that  the  Corporation  
Law  allows  a  corporation  to  invest  its  funds  in  any  other  corporation  or  business,  
or   for   any   purpose   other   than   the   main   purpose   for   which   it   was   organized,   provided   that   its   board   of  
directors   has  been  so  authorized  by  the  affirmative  vote  of  stockholders  holding  shares  entitling  them  to  
exercise  at  least  2/3  of  the  voting  power.  
 
An   investment   of   corporate   funds   in   another   corporation,   if   done   in   pursuance   of   the   corporate   purpose,  
does   not   need   the   approval   of   the   stockholders.   But   when   the   purchase   of   shares   of   another   corporation   is  
done  solely  for  investment  and  not  to  accomplish  the  purpose  of  its  incorporation,  the  vote  of  approval  of  the  
stockholders  is  necessary.  
 
Further,  when  the  purpose  is  as  stated  in  its  articles  of  incorporation,  the  approval  of  the  stockholders  is  not  
necessary.The  Corporation  Law  allows  a  corporation  to  invest  its  funds  in  any  other  corporation  or  business,  
or   for   any   purpose   other   than   the   main   purpose   for   which   it   was   organized,   provided   that   its   board   of  
directors   has   been  so   authorized   by  the  affirmative  vote  of  stockholders  holding  shares  entitling  them   to  
exercise  at  lease  2/3  of  the  voting  power.  
 
In  the  judgment,  the  lower  court  ordered  the  management  of  the  Ma-­‐‑ao  Sugar  Central  Co.,  Inc.  "to  refrain  
from   making   investments   in   Acoje   Mining,   Mabuhay   Printing   and   any   other   company   whose   purpose   is   not  
connected  with  the  sugar  central  business."  This  portion  of  the  decision  should  be  reversed  because,  Sec.  17-­‐‑
½  of  the  Corporation  Law  allows  a  corporation  to  "invest  its  fund  in  any  other  corporation  or  business,  or  
for  any  purpose  other  than  the  main  purpose  for  which  it  was  organized,"  provided  that  its  board  of  directors  
has  been  so  authorized  by  the  affirmative  vote  of  stockholders  holding  shares  entitling  them  to  exercise  at  
least  two-­‐‑thirds  of  the  voting  power.  
 
 
106.  SAME  WITH  119  
 
 
 
107.  EVERETTE  VS.  ASIA  BANKING  CORP.  
G.R.  NO.  L-­‐‑25241,  NOVEMBER  3,  1926  
OSTRAND,  J.  
 
DOCTRINE:  Shareholders  cannot  ordinarily  sue  in  equity  to  redress  wrongs  done  to  the  corporation,  but  that  
the  action  must  be  brought  by  the  Board  of  Directors,  except  if  a  demand  upon  the  Board  of  Directors  to  
institute  an  action  and  prosecute  the  same  effectively  would  have  been  useless,  and  the  law  does  not  require  
litigants  to  perform  useless  acts.  
 
FACTS:  
 
Plaintiffs   Everett,   Clifford,   Teal   and   Robinson   were   the   principal   stockholders   of   Teal   and   Company   (the  
Company)  a  domestic  corporation  engaged  in  the  business  of  merchandising  of  automobiles,  trucks,  tractors,  
spare  parts  and  accessories  therefor,  and  the  repairing  thereof.  The  defendants  Nicholas  E.  Mullen,  Alfred  F.  
Kelly,  John  W.  Mears,  and  Charles  D.  Macintosh  were  officers,  agents  and  employees  of  the  said  Asia  Banking  
Corporation  (the  Bank).  The  Company  was  indebted  to  the  Bank  in  about  the  sum  of  P750,000,  which  said  
sum  was  secured  by  mortgage  on  its  personal  property  and  the  improvements  upon  the  real  estate  occupied  
by  it  
 

   
3H  A.Y.  2017-­‐2018   144  
 
 

CORPORATION  LAW  CASE  DIGESTS  –  ATTY.  DANTE  DELA  CRUZ  

On  March  1921,  the  Bank  persuaded  the  Company  and  other  creditors  to  enter  into  a  so  called  "creditors  
agreement"  with  itself,  wherein  it  was  mutually  agreed  that  neither  of  the  parties  should  take  action  to  collect  
its  debts  from  the  Company  for  the  term  of  two  years  after  the  date  thereof.  Later  on,  Mullen  (voting  trustee)  
represented   that   in   order   to   protect   the   mutual   interests   of   the   Bank   and   the   Company,   it   was   necessary   to  
carry   into   effect   the   said   proposed   voting   trust   agreement   without   the   knowledge   of   the   creditors   above  
named   and   thereby   place   the   Bank   in   an   advantageous   position   with   regard   to   them.   Thereafter   said  
defendants  conducted  the  business  of  the  Company  without  consulting  the  stockholders  thereof  and  denied  
to  the  stockholders  any  knowledge  or  information  as  to  their  actions,  or  the  business  of  the  Company.    
 
In   1923,   defendants   made,   executed   and   filed   in   the   Bureau   of   Commerce   and   Industry   of   the   Philippine  
Islands,   articles   of   incorporation   of   a   corporation   called   the   "Philippine   Motors   Corporation.”(PMC)     Acting  
in  their  double  capacity  as  directors  of  both  corporations,  defendants  permitted  and  assisted  the  said  PMC  
to  enter  and  possess  itself  of  the  premises  and  good  will  of  the  Company  and  to  continue  and  carry  on  the  
business   for   the   sole   benefit   of   the   new   corporation   and   to   collect   the   debts   owing   to   the   Company   and  
convert  the  advantages,  profits  and  proceeds  thereof  to  itself.  
 
Plaintiffs   filed   a   complaint   in   CFI   Manila   praying   to   cancel   the   said   Voting   trust   and   to   return   to   these  
plaintiffs  their  shares  of  the  stock  of  Teal  and  Company.  And  if    it  be  found  that  the  said  PMC  is  in  fact  the  Asia  
Banking  Corporation  that  a  decree  be  entered  ordering  the  said  Bank  immediately  to  dissolve  the  same  and  
to   account   to   these   plaintiffs   for   all   profits   made   thereby   since   its   organization.   CFI   Manila   granted  
defendants  demurrer.  
 
ISSUE:  Whether  the  plaintiffs  as  stockholders  of  Teal  and  Company  has  a  cause  of  action  against  defendants.  
 
HELD:    
The   court   below   sustained   the   demurrer   on   the     grounds   that   complaint   is   ambiguous,   confusing,  
unintelligible   and   vague;   that   Teal   and   Company   should   have   been   joined   as   a   party   plaintiff;   that,   as   far   as  
the  Philippine  Motors  Corporation  is  concerned,  the  plaintiffs,  not  being  stockholders  in  that  corporation,  
had  no  legal  right  to  proceed  against  it  in  this  case.  
 
Invoking   the   rule   that   shareholders   cannot   ordinarily   sue   in   equity   to   redress   wrongs   done   to   the  
corporation,   but   that   the   action   must   be   brought   by   the   Board   of   Directors,   except   if   a   demand   upon   the  
Board  of  Directors  to  institute  an  action  and  prosecute  the  same  effectively  would  have  been  useless,  and  the  
law  does  not  require  litigants  to  perform  useless  acts.  
 
However,  the  corporation  Teal  and  Company  is  a  necessary  party  plaintiff  and  that  the  plaintiff  stockholders,  
not  having  made  any  demand  on  the  Board  to  bring  the  action,  are  not  the  proper  parties  plaintiff.    
 
Also,  this  court  concludes  that  the  plaintiffs,  not  being  stockholders  in  the  Philippine  Motors  Corporation,  
had  no  legal  right  to  proceed  against  that  corporation  in  the  manner  suggested  in  the  complaint  evidently  
rest  upon  a  misconception  of  the  character  of  the  action.  
 
 
 
108.  NO  CASE  
 
 
109.  ALFREDO  MONTELIBANO.  ET.  AL.,  V.  BACOLOD-­‐‑MURCIA  MILLING  
G.R.  NO.  L-­‐‑15092,  MAY  18,  1962  
REYES,  J.B.L.,  J.  

   
3H  A.Y.  2017-­‐2018   145  
 
 

CORPORATION  LAW  CASE  DIGESTS  –  ATTY.  DANTE  DELA  CRUZ  

 
DOCTRINE:  As  the  resolution  in  question  was  passed  in  good  faith  by  the  board  of  directors,  it  is  valid  and  
binding,   and   whether   or   not   it   will   cause   losses   or   decrease   the   profits   of   the   central,   the   court   has   no  
authority  to  review  them.  
 
FACTS:  Plaintiffs-­‐‑appellants,  Alfredo  Montelibano,  Alejandro  Montelibano,  and  the  Limited  co-­‐‑partnership  
Gonzaga  and  Company,  had  been  and  are  sugar  planters  adhered  to  the  defendant-­‐‑appellee’s  sugar  central  
mill  under  identical  milling  contracts.  Originally  executed  in  1919,  said  contracts  were  stipulated  to  be  in  
force  for  30  years  starting  with  the  1920-­‐‑21  crop,  and  provided  that  the  resulting  product  should  be  divided  
in   the   ratio   of   45%   for   the   mill   and   55%   for   the   planters.   Sometime   in   1936,   it   was   proposed   to   execute  
amended  milling  contracts,  increasing  the  planters’  share  to  60%  of  the  manufactured  sugar  and  resulting  
molasses,  besides  other  concessions,  but  extending  the  operation  of  the  milling  contract  from  the  original  30  
years   to   45   years.   The   Board   of   Directors   of   the   appellee   Bacolod-­‐‑Murcia   Milling   Co.,   Inc.,   adopted   a  
resolution   granting   further   concessions   to   the   planters   over   and   above   those   contained   in   the   printed  
Amended  Milling  Contract.  The  appellants  initiated  the  present  action,  contending  that  three  Negros  sugar  
centrals  with  a  total  annual  production  exceeding  one-­‐‑third  of  the  production  of  all  the  sugar  central  mills  in  
the  province,  had  already  granted  increased  participation  (of  62.5%)to  their  planters,  and  that  under  the  
resolution   the   appellee   had   become   obligated   to   grant   similar   concessions   to   the   plaintiffs.   The   appellee  
Bacolod-­‐‑Murcia  Milling  Co.,  inc.,  resisted  the  claim,  and  defended  by  urging  that  the  stipulations  contained  in  
the   resolution   were   made   without   consideration;   that   the   resolution   in   question   was,   therefore,   null   and  
void  ab  initio,  being  in  effect  a  donation  that  was  ultra  vires  and  beyond  the  powers  of  the  corporate  directors  
to  adopt.  
 
ISSUE:  WON  the  board  resolution  is  an  ultra  vires  act  and  in  effect  a  donation  from  the  board  of  directors?  
 
HELD:  No.  There  can  be  no  doubt  that  the  directors  of  the  appellee  company  had  authority  to  modify  the  
proposed  terms  of  the  Amended  Milling  Contract  for  the  purpose  of  making  its  terms  more  acceptable  to  the  
other  contracting  parties.  As  the  resolution  in  question  was  passed  in  good  faith  by  the  board  of  directors,  it  
is   valid   and   binding,   and   whether   or   not   it   will   cause   losses   or   decrease   the   profits   of   the   central,   the   court  
has  no  authority  to  review  them.  Whether  the  business  of  a  corporation  should  be  operated  at  a  loss  during  
depression,  or  close  down  at  a  smaller  loss,  is  a  purely  business  and  economic  problem  to  be  determined  by  
the  directors  of  the  corporation  and  not  by  the  court.  The  appellee  Bacolod-­‐‑Murcia  Milling  Company  is,  under  
the  terms  of  its  Resolution  of  August  20,  1936,  duty  bound  to  grant  similar  increases  to  plaintiffs-­‐‑appellants  
herein.  
 
 
 
110.  PHILIPPINE  STOCK  EXCHANGE,  INC  VS  CA  
G.R.  NO.  125469.  OCTOBER  27,  1997  
TORRES,  JR.,  J.  
 
DOCTRINE:    
 
A  corporation  is  but  an  association  of  individuals,  allowed  to  transact  under  an  assumed  corporate  name,  
and   with   a   distinct   legal   personality.   In   organizing   itself   as   a   collective   body,   it   waives   no   constitutional  
immunities   and   perquisites   appropriate   to   such   body.   As   to   its   corporate   and   management   decisions,  
therefore,  the  state  will  generally  not  interfere  with  the  same.  Questions  of  policy  and  of  management  are  
left  to  the  honest  decision  of  the  officers  and  directors  of  a  corporation,  and  the  courts  are  without  authority  
to   substitute   their   judgment   for   the   judgment   of   the   board   of   directors.   The   board   is   the   business   manager  
of  the  corporation,  and  so  long  as  it  acts  in  good  faith,  its  orders  are  not  reviewable  by  the  courts.  

   
3H  A.Y.  2017-­‐2018   146  
 
 

CORPORATION  LAW  CASE  DIGESTS  –  ATTY.  DANTE  DELA  CRUZ  

 
 
FACTS:    
 
In  this  Petition  for  Review  of  Certiorari,  petitioner  assails  the  resolution  of  the  respondent  Court  of  Appeals  
which  affirmed  the  decision  of  the  Securities  and  Exchange  Commission  ordering  the  petitioner  Philippine  
Stock  Exchange,  Inc.  to  allow  the  private  respondent  Puerto  Azul  Land,  Inc.  to  be  listed  in  its  stock  market,  
thus  paving  the  way  for  the  public  offering  of  PALIs  shares.  
The  Puerto  Azul  Land,  Inc.  (PALI),  a  domestic  real  estate  corporation,  had  sought  to  offer  its  shares  to  the  
public   in   order   to   raise   funds   allegedly   to   develop   its   properties   and   pay   its   loans   with   several   banking  
institutions.   PALI   was   issued   a   Permit   to   Sell   its   shares   to   the   public   by   the   Securities   and   Exchange  
Commission  (SEC).  To  facilitate  the  trading  of  its  shares  among  investors,  PALI  sought  to  course  the  trading  
of  its  shares  through  the  Philippine  Stock  Exchange,  Inc.  (PSE),  for  which  purpose  it  filed  with  the  said  stock  
exchange  an  application  to  list  its  shares,  with  supporting  documents  attached.  
 
The   Listing   Committee   of   the   PSE,   upon   a   perusal   of   PALIs   application,   recommended   to   the   PSEs   Board   of  
Governors  the  approval  of  PALIs  listing  application.  But  before  it  could  act  upon  PALIs  application,  the  Board  
of  Governors  of  PSE  received  a  letter  from  the  heirs  of  Ferdinand  E.  Marcos,  claiming  that  the  late  President  
Marcos  was  the  legal  and  beneficial  owner  of  certain  properties  forming  part  of  the  Puerto  Azul  Beach  Hotel  
and   Resort   Complex   which   PALI   claims   to   be   among   its   assets   and   that   the   Ternate   Development  
Corporation,  which  is  among  the  stockholders  of  PALI,  likewise  appears  to  have  been  held   and  continue  to  
be  held  in  trust  by  one  Rebecco  Panlilio  for  then  President  Marcos  and  now,  effectively  for  his  estate,  and  
requested  PALIs  application  to  be  deferred.    
 
The  Board  of  Governors  of  the  PSE  reached  its  decision  to  reject  PALIs  application,  citing  the  existence  of  
serious   claims,   issues   and   circumstances   surrounding   PALIs   ownership   over   its   assets   that   adversely   affect  
the  suitability  of  listing  PALIs  shares  in  the  stock  exchange.  
PALI  wrote  a  letter  to  the  SEC  bringing  to  the  SECs  attention  the  action  taken  by  the  PSE  in  the  application  of  
PALI  for  the  listing  of  its  shares  with  the  PSE,  and  requesting  that  the  SEC,  in  the  exercise  of  its  supervisory  
and  regulatory  powers  over  stock  exchanges  under  Section  6(j)  of  P.D.  No.  902-­‐‑A,  review  the  PSEs  action  on  
PALIs  listing  application  and  institute  such  measures  as  are  just  and  proper  and  under  the  circumstances.  
The   SEC   reversed   the   decision   of   the   PSE   to   deny   the   application   for   listing   in   the   stock   exchange   of   the  
private  respondent  PALI.  The  SEC  likewise  ordered  the  PSE  to  cause  the  listing  of  PALI’s  shares  in  the  stock  
exchange.  The  SECs  action  was  affirmed  by  the  Court  of  Appeals.  
 
 
ISSUE:  Whether  or  not  it  is  within  the  power  and  authority  of  the  SEC  to  reverse  actions  done  by  the  PSE.  
 
 
HELD:  
Yes.  The  SEC  has  both  jurisdiction  and  authority  to  look  into  the  decision  of  PSE  pursuant  to  the  Revised  
Securities  Act  and  for  the  purpose  of  ensuring  fair  administration  of  the  exchange.  PSE,  as  a  corporation  itself  
and  as  a  stock  exchange  is  subject  to  SEC’s  jurisdiction,  regulation,  and  control.  In  order  to  ensure  fair  dealing  
of  securities  and  a  fair  administration  of  exchanges  in  the  PSE,  the  SEC  has  the  authority  to  look  into  the  
rulings  issued  by  the  PSE.  The  SEC  is  the  entity  with  the  primary  say  as  to  whether  or  not  securities,  including  
shares  of  stock  of  a  corporation,  may  be  traded  or  not  in  the  stock  exchange.This  is  in  line  with  the  SECs  
mission  to  ensure  proper  compliance  with  the  laws,  such  as  the  Revised  Securities  Act  and  to  regulate  the  
sale   and   disposition   of   securities   in   the   country.   As   the   appellate   court   explains:   Paramount   policy   also  
supports   the   authority   of   the   public   respondent   to   review   petitioners   denial   of   the   listing.   Being   a   stock  
exchange,  the  petitioner  performs  a  function  that  is  vital  to  the  national  economy,  as  the  business  is  affected  

   
3H  A.Y.  2017-­‐2018   147  
 
 

CORPORATION  LAW  CASE  DIGESTS  –  ATTY.  DANTE  DELA  CRUZ  

with  public  interest.  As  a  matter  of  fact,  it  has  often  been  said  that  the  economy  moves  on  the  basis  of  the  rise  
and  fall  of  stocks  being  traded.  By  its  economic  power,  the  petitioner  certainly  can  dictate  which  and  how  
many  users  are  allowed  to  sell  securities  thru  the  facilities  of  a  stock  exchange,  if  allowed  to  interpret  its  own  
rules   liberally   as   it   may   please.   Petitioner   can   either   allow   or   deny   the   entry   to   the   market   of   securities.   To  
repeat,   the   monopoly,   unless   accompanied   by   control,   becomes   subject   to   abuse;   hence,   considering   public  
interest,  then  it  should  be  subject  to  government  regulation.  
 
This  is  not  to  say,  however,  that  the  PSEs  management  prerogatives  are  under  the  absolute  control  of  the  
SEC.   The   PSE   is,   after   all,   a   corporation   authorized   by   its   corporate   franchise   to   engage   in   its   proposed   and  
duly  approved  business.  Thus,  the  Supreme  Court  emphasizes  that  notwithstanding  the  regulatory  power  of  
the   SEC   over   the   PSE,   and   the   resultant   authority   to   reverse   the   PSEs   decision   in   matters   of   application   for  
listing  in  the  market,  the  SEC  may  exercise  such  power  only  if  the  PSEs  judgment  is  attended  by  bad  faith.  In  
this  case,  there  was  no  showing  that  PSE  acted  with  bad  faith  when  it  denied  the  application  of  PALI.  Based  
on   the   multiple   adverse   claims   against   the   assets   of   PALI,   PSE   deemed   that   granting   PALI’s   application   will  
only  be  contrary  to  the  best  interest  of  the  general  public.  The  purpose  of  the  Revised  Securities  Act,  after  all,  
is   to   give   adequate   and   effective   protection   to   the   investing   public   against   fraudulent   representations,   or  
false  promises,  and  the  imposition  of  worthless  ventures.  Thus,  it  was  reasonable  for  the  PSE  to  exercise  its  
judgment  in  the  manner  it  deems  appropriate  for  its  business  identity,  as  long  as  no  rights  are  trampled  upon,  
and  public  welfare  is  safeguarded.    
 
In  resum,  the  Court  finds  that  the  PSE  has  acted  with  justified  circumspection,  discounting,  therefore,  any  
imputation  of  arbitrariness  and  whimsical  animation  on  its  part.  Its  action  in  refusing  to  allow  the  listing  of  
PALI  in  the  stock  exchange  is  justified  by  the  law  and  by  the  circumstances  attendant  to  this  case.  Hence,  the  
decisions   of   the   CA   and   the   SEC   are   hereby   REVERSED   and   SET   ASIDE,   and   a   new   Judgment   is   hereby  
ENTERED,   affirming   the   decision   of   the   PSE   to   deny   the   application   for   listing   of   the   private   respondent  
Puerto  Azul  Land,  Inc.  
 
 
 
111.  C.  H.  STEINBERG  V.  GREGORIO  VELASCO  
G.R.  NO.  30460.  MARCH  12,  1929  
EN  BANC  
 
Doctrine:  Creditors  of  a  corporation  have  the  right  to  assume  that  so  long  as  there  are  outstanding  debts  and  
liabilities,  the  board  of  directors  will  not  use  the  assets  of  the  corporation  to  purchase  its  own  stock,  and  that  
it  will  not  declare  dividends  to  stockholders  when  the  corporation  is  insolvent.  
 
Facts:  
It   is   alleged   that   the   defendants,   Gregorio   Velasco,   as   president,   Felix   del   Castillo,   as   vice-­‐‑president,   Andres  
L.  Navallo,  as  secretary-­‐‑treasurer,  and  Rufino  Manuel,  as  director  of  the  Trading  Company,  at  a  meeting  of  
the  board  of  directors  held  on  July  24,  1922,  approved  and  authorized  various  unlawful  purchases  already  
made  of  a  large  portion  of  the  capital  stock  of  the  company  from  its  various  stockholders,  thereby  diverting  
its   funds   to   the   injury,   damage   and   in   fraud   of   the   creditors   of   the   corporation.   That   pursuant   to   such  
resolution  and  on  March  31,  1922,  the  corporation  purchased  from  the  defendant  S.  R.  Ganzon  100  shares  of  
its   capital   stock   of   the   par   value   of   P10,   and   on   June   29,1922,   it   purchased   from   the   defendant   Felix   D.  
Mendaros  100  shares  of  the  par  value  of  P10,  and  on  July  16,  1922,  it  purchased  from  the  defendant  Felix  D.  
Mendaros  100  shares  of  the  par  value  of  P10,  each,  and  on  April  5,  1922,  it  purchased  from  the  defendant  
Dionisio  Saavedra  10  shares  of  the  same  par  value,  and  on  June  29,  1922,  it  purchased  from  the  defendant  
Valentin  Matias  20  shares  of  like  value.  That  the  total  amount  of  the  capital  stock  unlawfully  purchased  was  
P3,300.   That   at   the   time   of   such   purchase,   the   corporation   had   accounts   payable   amounting   to   P13,807.50,  

   
3H  A.Y.  2017-­‐2018   148  
 
 

CORPORATION  LAW  CASE  DIGESTS  –  ATTY.  DANTE  DELA  CRUZ  

most  of  which  were  unpaid  at  the  time  the  petition  for  the  dissolution  of  the  corporation  was  presented,  and  
that  the  corporation  was  then  in  a  bad  financial  condition,  in  contemplation  of  an  insolvency  and  dissolution.  
 
As  a  second  cause  of  action,  plaintiff  alleges  that  on  July  24,  1922,  the  officers  and  directors  of  the  corporation  
approved  a  resolution  for  the  payment  of  P3,000  as  dividends  to  its  stockholders,  which  was  wrongfully  done  
and  in  bad  faith,  and  to  the  injury  and  fraud  of  its  creditors.  That  at  the  time  the  petition  for  the  dissolution  
of  the  corporation  was  presented  it  had  accounts  payable  in  the  sum  of  P9,241.19,  "and  practically  worthless  
accounts  receivable."  
 
Issue:  Whether  the  directors  acted  in  bad  faith.  (YES)  
 
Ratio:  
From  all  of  which,  it  appears  that  on  June  30,  1922,  the  board  of  directors  of  the  corporation  authorized  the  
purchase  of,  purchased  and  paid  for,  330  shares  of  the  capital  stock  of  the  corporation  at  the  agreed  price  of  
P3,300,  and  that  at  the  time  the  purchase  was  made,  the  corporation  was  indebted  in  the  sum  of  P13,807.50,  
and  that  according  to  its  books,  it  had  accounts  receivable  in  the  sum  of  P19,126.02.  That  on  September  11,  
1923,   when   the   petition   was   filed   for   its   dissolution   upon   the   ground   that   it   was   insolvent,   its   accounts  
payable  amounted  to  P9,241.19,  and  its  accounts  receivable  P12,512.47,  or  an  apparent  asset  of  P3,271.28  
over  and  above  its  liabilities.  
 
But  it  will  be  noted  that  there  is  no  stipulation  or  finding  of  fact  as  to  what  was  the  actual  cash  value  of  its  
accounts  receivable.  Neither  is  there  any  stipulation  that  those  accounts  or  any  part  of  them  ever  have  been  
or  will  be  collected,  and  it  does  appear  that  after  his  appointment  on  February  28,  1924,  the  receiver  made  a  
diligent  effort  to  collect  them,  and  that  he  was  unable  to  do  so,  and  it  also  appears  from  the  minutes  of  the  
board  of  directors  that  the  president  and  manager  "recommended  that  P3,000  —  out  of  the  surplus  account  
to   be   set   aside   for   dividends   payable,   and   that   payments   be   made   in   installments   so   as   not   to   affect   the  
financial  condition  of  the  corporation."  
If  in  truth  and  in  fact  the  corporation  had  an  actual  bona  fide  surplus  of  P3,000  over  and  above  all  of  its  debts  
and   liabilities,   the   payment   of   the   P3,000   in   dividends   would   not   in   the   least   impair   the   financial   condition  
of  the  corporation  or  prejudice  the  interests  of  its  creditors.  
 
It   is   very   apparent   that   on   June   24,   1922,   the   board   of   directors   acted   on   the   assumption   that   because   it  
appeared  from  the  books  of  the  corporation  that  it.  Had  accounts  receivable  of  the  face  value  of  P19,126.02,  
therefore  it  had  a  surplus  over  and  above  its  debts  and  liabilities.  But  as  stated,  there  is  no  stipulation  as  to  
the  actual  cash  value  of  those  accounts,  and  it  does  appear  from  the  stipulation  that  on  February  28,1924,  
P12,512.47  of  those  accounts  had  but  little,  if  any,  value,  and  it  must  be  conceded  that,  in  the  purchase  of  its  
own  stock  to  the  amount  of  P3,300  and  in  declaring  the  dividends  to  the  amount  of  P3,000,  the  real  assets  of  
the   corporation   were   diminished   P6,300.   It   also   appears   from   paragraph   4   of   the   stipulation   that   the  
corporation  had  a  "surplus  profit"  of  P3,314.72  only.  It  is  further  stipulated  that  the  dividends  should  "be  
made   in   installments   so   as   not   to   affect   the   financial   condition   of   the   corporation."   In   other   words.   that   the  
corporation  did  not  then  have  an  actual  bona  fide  surplus  from  which  the  dividends  could  be  paid,  and  that  
the  payment  of  them  in  full  at  that  time  would  "affect  the  financial  condition  of  the  corporation."  
 
It   is,   indeed,   peculiar   that   the   action   of   the   board   in   purchasing   the   stock   from   the   corporation   and   in  
declaring  the  dividends  on  the  stock  was  all  done  at  the  same  meeting  of  the  board  of  directors,  and  it  appears  
in  those  minutes  that  both  Ganzon  and  Mendaros  were  formerly  directors  and  resigned  before  the  board  
approved  the  purchase  and  declared  the  dividends,  and  that  out  of  the  whole  330  shares  purchased,  Ganzon  
sold  100  and  Mendaros  200,  or  a  total  of  300  shares  out  of  the  330,  which  were  purchased  by  the  corporation,  
and   for   which   it   paid   P3,300.   In   other   words,   that   the   directors   were   permitted   to   resign   so   that   they   could  
sell  their  stock  to  the  corporation.  As  stated,  the  authorized  capital  stock  was  P20,000  divided  into  2,000  

   
3H  A.Y.  2017-­‐2018   149  
 
 

CORPORATION  LAW  CASE  DIGESTS  –  ATTY.  DANTE  DELA  CRUZ  

shares  of  the  par  value  of  P10  each,  of  which  only  P10,030  was  subscribed  and  paid.  Deducting  the  P3,300  
paid  for  the  purchase  of  the  stock,  there  would  be  left  P7,000  of  paid  up  stock,  from  which  deduct  P3,000  
paid  in  dividends,  there  would  be  left  P4,000  only.  In  this  situation  and  upon  this  state  of  facts,  it  is  very  
apparent  that  the  directors  did  not  act  in  good  faith  or  that  they  were  grossly  ignorant  of  their  duties.  
 
Creditors  of  a  corporation  have  the  right  to  assume  that  so  long  as  there  are  outstanding  debts  and  liabilities,  
the  board  of  directors  will  not  use  the  assets  of  the  corporation  to  purchase  its  own  stock,  and  that  it  will  not  
declare  dividends  to  stockholders  when  the  corporation  is  insolvent.  
 
The  amount  involved  in  this  case  is  not  large,  but  the  legal  principles  are  important,  and  we  have  given  them  
the  consideration  which  they  deserve.  
 
 
 
112.  PEDRO  R.  PALTING  V.  SAN  JOSE  PETROLEUM  INCORPORATED  
G.R.  NO.  L-­‐‑14441  -­‐‑    DECEMBER  17,  1966  
BARRERA,  J.  
 
DOCTRINE:  
The   privilege   to   utilize,   exploit   and   develop   the   natural   resources   of   the   Philippines   was   granted   by   Article  
XIII  of  the  [1935]  Constitution,  to  Filipino  citizens  or  to  corporations  or  associations  60%  of  the  capital  of  
which   is   owned   by   such   citizens.   With   the   Parity   Amendment   to   the   Constitution,   the   same   right   was  
extended  to  citizens  of  the  United  States  and  business  enterprise  owned  or  controlled,  directly  or  indirectly,  
by  citizens  of  the  United  States.  There  can  be  no  serious  doubt  as  to  the  meaning  of  the  word  "citizens"  used  
in  the  aforementioned  provisions  of  the  Constitution.  The  right  was  granted  to  two  types  of  persons;  natural  
persons  (Filipino  or  American  citizen)  and  juridical  persons  (corporations  60%  of  which  capital  is  owned  by  
Filipinos  and  business  enterprises  owned  or  controlled  directly  or  indirectly  by  citizens  of  the  United  States).  
 
NB:  This  case  was  decided  during  the  effectivity  of  the  1935  Constitution  and  the  Corporation  Law  (Act  
No.  1459).  The  parity  right  is  found  in  the  Bell  Trade  Act;  it  lasted  until  July  3,  1974  as  indicated  in  Article  
VII  of  the  Act.  
 
FACTS:  
SAN  JOSE  OIL,  is  a  domestic  mining  corporation,  90%  of  the  outstanding  capital  stock  of  which  is  owned  by  
respondent  SAN  JOSE  PETROLEUM,  a  foreign  (Panamanian)  corporation,  the  majority  interest  of  which  is  
owned  by  OIL  INVESTMENTS,  INC.,  another  foreign  (Panamanian)  company.  This  latter  corporation  in  turn  
is   wholly   (100%)   owned   by   PANTEPEC   OIL   COMPANY,   C.   A.,   and   PANCOASTAL   PETROLEUM   COMPANY,   C.  
A.,  both  organized  and  existing  under  the  laws  of  Venezuela.  
 
San  Jose  Petroleum  filed  with  the  Philippine  SEC  for  a  registration  and  licensing  for  sale  in  the  Philippines  
Voting  Trust  Certificates  It  was  alleged  that  the  entire  proceeds  of  the  sale  of  said  securities  will  be  devoted  
or   used   exclusively   to   finance   the   operations   of   SAN   JOSE   OIL   which   has   14   petroleum   exploration  
concessions  covering  an  area  of  a  little  less  than  1,000,000  hectares  in  the  Philippines.  While  this  application  
for   registration   was   pending   consideration   by   the   SEC,   SAN   JOSE   PETROLEUM   filed   an   amended   Statement  
for  registration  of  the  sale  in  the  Philippines  of  its  shares  of  capital  stock.  
 
Pedro  R.  Palting  and  others,  allegedly  prospective  investors  in  the  shares  of  SAN  JOSE  PETROLEUM,  filed  
with  the  SEC  an  opposition  to  the  registration  and  licensing  of  the  securities  on  the  grounds,  among  others,  
that   the   tie-­‐‑up   between   the   issuer,   SAN   JOSE   PETROLEUM,   a   Panamanian   corporation,   and   SAN   JOSE   OIL,   a  
domestic  corporation,  violates  the  Constitution  of  the  Philippines,  Sec.  13  of  the  Corporation  Law  and  the  

   
3H  A.Y.  2017-­‐2018   150  
 
 

CORPORATION  LAW  CASE  DIGESTS  –  ATTY.  DANTE  DELA  CRUZ  

Petroleum  Act  of  1949  -­‐‑   which  inhibits  a  mining  corporation  from  acquiring  an  interest  in  another  mining  
corporation.  
 
ISSUE:  
Is  SAN  JOSE  PETROLEUM  an  American  business  enterprise  entitled  to  parity  rights  in  the  Philippines?  
 
HELD:  
NO.  The  privilege  to  utilize,  exploit,  and  develop  the  natural  resources  of  this  country  was  granted,  by  Article  
XIII   of   the   Constitution,   to   Filipino   citizens   or   to   corporations   or   associations   60%   of   the   capital   of   which  is  
owned   by   such   citizens.   With   the   Parity   Amendment   to   the   Constitution,   the   same   right   was   extended   to  
citizens  of  the  United  States  and  business  enterprises  owned  or  controlled  directly  or  indirectly,  by  citizens  
of  the  United  States.  
 
There   could   be   no   serious   doubt   as   to   the   meaning   of   the   word   "citizens"   used   in   the   aforementioned  
provision   of   the   Constitution.   The   right   was   granted   to   2   types   of   persons:   natural   persons   (Filipino   or  
American   citizens)   and   juridical   persons   (corporations   60%   of   which   capital   is   owned   by   Filipinos   and  
business  enterprises  owned  or  controlled  directly  or  indirectly,  by  citizens  of  the  United  States).    
 
San  Jose  Petroleum  Incorporated  is  not  owned  or  controlled  directly  by  citizens  of  the  United  States,  because  
it   is   owned   and   controlled   by   Oil   Investments,   Inc.,   another   foreign   (Panamanian)   corporation.   Neither   is   it  
indirectly   owned   or   controlled   by   American   citizens   through   Oil   Investments,   Inc.,   which   is   owned   and  
controlled,  not  by  citizens  of  the  United  States,  but  by  two  foreign  (Venezuelan)  corporations.  There  is  no  
showing  that  the  stockholders  in  these  two  corporations  are  citizens  of  the  United  States.  But  even  granting  
that  they  are,  it  is  still  necessary  to  establish  that  the  different  states  of  which  they  are  citizens  allow  Filipino  
citizens  or  corporations  or  associations  owned  or  controlled  by  Filipino  citizens  to  engage  in  the  exploitation,  
etc.  of  the  natural  resources  of  these  states  (par.  3,  Art.  VI  of  the  Laurel-­‐‑Langley  Agreement).  
 
And  even  if  these  requirements  are  satisfied,  to  hold  that  the  set-­‐‑up  disclosed  in  the  present  case,  with  a  long  
chain   of   intervening   foreign   corporations,   comes   within   the   purview   of   the   Parity   Amendment   regarding  
business  enterprises  indirectly  owned  or  controlled  by  citizens  of  the  United  States,  is  to  unduly  stretch  and  
strain  the  language  and  intent  of  the  law.  
 
Moreover,  the  shares  of  stock  of  the  PANTEPEC  and  PANCOASTAL  which  are  allegedly  owned  or  controlled  
directly   by   citizens   of   the   United   States,   are   traded   in   the   stock   exchange   in   New   York,   and   you   have   a  
situation  where  it  becomes  a  practical  impossibility  to  determine  at  any  given  time,  the  citizenship  of  the  
controlling  stock  required  by  the  law.  
 
Thus,  the  respondent  SAN  JOSE  PETROLEUM,  as  presently  constituted,  is  not  a  business  enterprise  that  is  
authorized   to   exercise   the   parity   privileges   under   the   Parity   Ordinance,   the   Laurel-­‐‑Langley   Agreement   and  
the  Petroleum  Law.  Its  tie-­‐‑up  with  SAN  JOSE  OIL  is,  consequently,  illegal.  
 
 
 
113.  MAED  V  MCCULLOUGH  
 
 
 
114.  NO  CASE  
 
 

   
3H  A.Y.  2017-­‐2018   151  
 
 

CORPORATION  LAW  CASE  DIGESTS  –  ATTY.  DANTE  DELA  CRUZ  

 
 
 
 
115.  (SAME  WITH  25)    THE  GOVERNMENT  OF  THE  PHILIPPINE  ISLANDS    VS.  EL  HOGAR  FILIPINO  
GR  NO.  L-­‐‑26649/JULY  13,  1927  
STREET,  J.  
 
DOCTRINE:   A   provision   in   the   by-­‐‑laws   allowing   the   BOD,   by   vote   of   absolute   majority,   to   cancel   shares   is   a  
patent  nullity,  being  in  direct  conflict  with  Sec.  187  of  the  Corp.  Law  which  prohibits  forced  surrender  of  
unmatured  stocks  except  in  case  of  dissolution.  
A  provision  in  the  by-­‐‑laws  fixing  the  salary  of  directors  is  valid  since  the  Corporation  Law  does  not  prescribe  
the  rate  of  compensation,  the  power  to  fix  compensation  lies  with  the  corporation.  
A  provision  requiring  persons  elected  to  the  Board  of  Directors  to  own  at  least  P  5,000  shares  is  valid  because  
the  Corporation  Law  gives  the  corporation  the  power  to  provide  qualifications  of  its  directors.  
 
FACTS:   This   is   a   quo   warranto   proceeding   instituted   originally   in   this   court   by   the   Government   of   the  
Philippine  Islands  on  the  relation  of  the  Attorney-­‐‑General  against  the  building  and  loan  association  known  
as  El  Hogar  Filipino,  for  the  purpose  of  depriving  it  of  its  corporate  franchise,  excluding  it  from  all  corporate  
rights  and  privileges,  and  effecting  a  final  dissolution  of  said  corporation.    
March  1906,  Corporation  law  took   effect.  Sec  171  to  190  are  devoted  to  building  and  loan  association.  El  
Hogar  was  the  first  corporation  in  the  Philippines  under  the  said  provisions.  Under  the  law  then,  the  capital  
of   an   association   was   not   permitted   to   exceed   3M   but   then   by   Act   2092   it   was   amended   to   10M.   Soon  
thereafter  the  association  took  advantage  of  this  enactment  by  amending  its  articles  so  as  to  provide  that  the  
capital  should  be  in  an  amount  not  exceeding  the  then  lawful  limit.      
 
 
   
116.  BARRETTO  V.  LA  PREVISORA  FILIPINA  
 
 
 
117.   CENTRAL   COOPERATIVE   EXCHANGE,   INC.,   VS.   CONCORDIO   TIBE,   SR.   AND   THE   HONORABLE  
COURT  OF  APPEALS  
G.R.  NO.  L-­‐‑27972  JUNE  30,  1970.  
REYES,  J.B.L.,  J.:  
 
DOCTRINE:   The   matter   of   providing   for   their   compensations   are   specifically   withheld   from   the   board   of  
directors,  and  reserved  to  the  stockholders.  
 
FACTS:   Petitioner   is   a   national   federation   of   farmers'   cooperative   marketing   associations,   or   FACOMAS,  
scattered   throughout   the   country.   Its   single   majority   stockholder   is   the   former   Agricultural   Credit   and  
Cooperative  Financing  Administration  (ACCFA),  now  Agricultural  Credit  Administration  (ACA).  As  a  member  
of  the  petitioner's  board  of  directors  from  23  May  1958  to  26  May  1960,  respondent  Concordio  Tibe,  Sr.  drew  
and  collected  from  petitioner  CCE  cash  advances  amounting  to  P5,668.00.  Respondent  had  already  liquidated  
P3,317.25,  leaving  the  sum  of  P2,350.75  still  to  be  accounted  for.  Respondent  Tibe  had  also  drawn  several  
sums,  amounting  to  P14,436.95,  representing  commutable  per  diems  for  attending  meetings  of  the  Board  of  
Directors  in  Manila,  per  diems  and  transportation  expenses  for  FACOMA  visitations,  representation  expenses  
and  cummutable  discretionary  funds.  All  these  sums  were  disbursed  with  the  approval  of  general  manager,  
treasurer  and  auditor  of  CCE.  

   
3H  A.Y.  2017-­‐2018   152  
 
 

CORPORATION  LAW  CASE  DIGESTS  –  ATTY.  DANTE  DELA  CRUZ  

 
The   resolutions   of   the   Board   of   Directors   under   which   respondent   Tibe   drew   and   collected   the   sums   of  
money  sought  to  be  recovered,  and  which  petitioner  claims  are  invalid  resolutions,  are  the  following:  
 
(a)   Res.  No.  55,  May  5,  1957,  authorizing  visitation  of  FACOMAS,  in  order  to  be  official,  must  be  with  prior  
sanction  or  authority  of  the  board,  except  when  it  is  urgent,  in  which  case  Board  confirmation  is  needed;  
authorizing  per  diem  of  P10.00  is  authorized  for  visitations  outside  the  place  of  residence  of  the  director  
concerned;  
(b)   Res.  No.  52,  July  8,  1958,  appropriating  P10,000.00  as  discretionary  fund  of  the  board  of  directors,  
disbursement   from   which   will   be   made   upon   authorization   of   the   board   chairman   and   for   which   no  
supporting  receipts  need  be  presented,  
(c)   Res.   No.   49,   July   10,   1958,   granting   monthly   commutable   allowance   of   P200.00   to   each   director  
starting  from  July  1,  1958,  in  lieu  of  the  regular  waiting  time  per  diems  and  transportation  expenses  while  in  
the  City  of  Manila  attending  Board  and  committee  meetings.  
(d)   Res.  No.  57,  July  24,  1958,  amending  Res.  No.  49  by  adding  P20.00  to  the  P200.00  as  commutable  
transportation  allowances  while  attending  meetings  in  Manila.  
(e)   Res.   No.   35,   June   11,   1959,   increasing   the   monthly   commutable   allowance   for   each   director   from  
P300.00  to  P500.00  per  month  effective  June  1,  1959.  
(f)   Res.   No.   87,   October   9,   1959,   appropriating   P10,000.00   as   commutable   discretionary   fund   of   the  
board  of  directors.  
 
The   By-­‐‑Laws   of   petitioner   federation   provides   that   the   compensation,   if   any,   and   the   per   diems   for  
attendance  at  meetings  of  the  members  of  the  Board  of  Directors  shall  be  determined  by  the  members  at  any  
annual   meeting   or   special   meeting   of   the   Exchange   called   for   the   purpose.   In   the   annual   meeting   of   the  
stockholders   it   was   resolved   that   the   members   of   the   Board   of   Directors   attending   the   CCE   board   meetings  
be  entitled  to  actual  transportation  expenses  plus  the  per  diems  of  P30.00  and  actual  expenses  while  waiting.    
 
ISSUE:   Whether   or   not   the   board   of   directors   of   the   CCE   had   the   power   and   authority   to   adopt   various  
resolutions  which  appropriated  the  funds  of  the  corporation  for  the  enumerated  expenses  for  the  members  
of  the  said  board.  
 
 
HELD:   No.   The   questioned   resolutions   are   contrary   to   the   By-­‐‑Laws   of   the   federation   and,   therefore,   are   not  
within  the  power  of  the  board  of  directors  to  enact.  The  By-­‐‑Laws  explicitly  reserved  unto  the  stockholders  
the  power  to  determine  the  compensation  of  members  of  the  board  of  directors,  and  the  stockholders  did  
restrict   such   compensation   to   "actual   transportation   expenses   plus   the   per   diems   of   P30.00   and   actual  
expenses   while   waiting."   Even   without   the   express   reservation   of   said   power,   the   directors   are   not   entitled  
to   compensation,   for   The   law   is   well-­‐‑settled   that   directors   of   corporations   presumptively   serve   without  
compensation  and  in  the  absence  of  an  express  agreement  or  a  resolution  in  relation  thereto,  no  claim  can  
be  asserted  therefor.  Thus,  the  directors,  in  assigning  themselves  additional  duties,  such  as  the  visitation  of  
FACOMAS,  acted  within  their  power,  but,  by  voting  for  themselves  compensation  for  such  additional  duties,  
they  acted  in  excess  of  their  authority,  as  expressed  in  the  By-­‐‑Laws.  
 
Nor  may  the  directors  rely  on  Section  28  of  the  Corporation  Law,  giving  the  exercise  of  corporate  powers  and  
the  control  of  the  corporation's  business  and  property  to  the  board  of  directors,  or  on  Section  1  of  Article  VI  
of   the   By-­‐‑Laws,   empowering   the   board   with   "general   supervision   and   control   of   the   affairs   and   property   of  
the  Exchange,"  as  justifications  for  the  adoption  of  the  questioned  resolutions,  because  these  provisions  of  
the   law   and   the   By-­‐‑Laws   pertain   to   the   board's   general   powers   merely   and   do   not   extend   to   giving   the  
members  of  the  said  board  the  compensations  stated  in  the  resolution,  as  the  matter  of  providing  for  their  
compensations  are  specifically  withheld  from  the  board  of  directors,  and  reserved  to  the  stockholders.  

   
3H  A.Y.  2017-­‐2018   153  
 
 

CORPORATION  LAW  CASE  DIGESTS  –  ATTY.  DANTE  DELA  CRUZ  

 
 
118.  STRONG  V.  GUTTIEREZ  REPIDE  
 
 
119.  GOKONGWEI  JR.  V.  SEC  
G.R.  NO.  L-­‐‑45911  APRIL  11,  1979  
ANTONIO,  J.:  
 
(Note:  Super  haba  nung  case  guys  and  mahaba  yung  digest  kasi  super  daming  important  and  relevant  points.  
Eto  yung  sinasabi  ni  sir  na  case  J  )  
Facts:    
SEC  CASE  NO  1375  
On   October   22,   1976,   petitioner,   as   stockholder   of   respondent   San   Miguel   Corporation,   filed   with   the  
Securities   and   Exchange   Commission   (SEC)   a   petition   for   "declaration   of   nullity   of   amended   by-­‐‑laws,  
cancellation  of  certificate  of  filing  of  amended  by-­‐‑  laws,  injunction  and  damages  with  prayer  for  a  preliminary  
injunction"  against  the  majority  of  the  members  of  the  Board  of  Directors  and  San  Miguel  Corporation  as  an  
unwilling  petitioner.    
As  a  first  cause  of  action,  petitioner  alleged  that  on  September  18,  1976,  individual  respondents  amended  by  
bylaws  of  the  corporation,  basing  their  authority  to  do  so  on  a  resolution  of  the  stockholders  adopted  on  
March  13,  1961,  when  the  outstanding  capital  stock  of  respondent  corporation  was  only  P70,139.740.00,  
divided  into  5,513,974  common  shares  at  P10.00  per  share  and  150,000  preferred  shares  at  P100.00  per  
share.  At  the  time  of  the  amendment,  the  outstanding  and  paid  up  shares  totalled  30,127,047  with  a  total  par  
value  of  P301,270,430.00.  It  was  contended  that  according  to  section  22  of  the  Corporation  Law  and  Article  
VIII   of   the   by-­‐‑laws   of   the   corporation,   the   power   to   amend,   modify,   repeal   or   adopt   new   by-­‐‑laws   may   be  
delegated  to  the  Board  of  Directors  only  by  the  affirmative  vote  of  stockholders  representing  not  less  than  
2/3  of  the  subscribed  and  paid  up  capital  stock  of  the  corporation,  which  2/3  should  have  been  computed  on  
the  basis  of  the  capitalization  at  the  time  of  the  amendment.  Since  the  amendment  was  based  on  the  1961  
authorization,   petitioner   contended   that   the   Board   acted   without   authority   and   in   usurpation   of   the   power  
of  the  stockholders.  
As  a  second  cause  of  action,  it  was  alleged  that  the  authority  granted  in  1961  had  already  been  exercised  in  
1962  and  1963,  after  which  the  authority  of  the  Board  ceased  to  exist.  
As  a  third  cause  of  action,  petitioner  averred  that  the  membership  of  the  Board  of  Directors  had  changed  
since  the  authority  was  given  in  1961,  there  being  six  (6)  new  directors.  
As  a  fourth  cause  of  action,  it  was  claimed  that  prior  to  the  questioned  amendment,  petitioner  had  all  the  
qualifications  to  be  a  director  of  respondent  corporation,  being  a  Substantial  stockholder  thereof;  that  as  a  
stockholder,  petitioner  had  acquired  rights  inherent  in  stock  ownership,  such  as  the  rights  to  vote  and  to  be  
voted   upon   in   the   election   of   directors;   and   that   in   amending   the   by-­‐‑laws,   respondents   purposely   provided  
for  petitioner's  disqualification  and  deprived  him  of  his  vested  right  as  afore-­‐‑mentioned  hence  the  amended  
by-­‐‑laws  are  null  and  void.  1  
It   was,   therefore,   prayed   that   the   amended   by-­‐‑laws   be   declared   null   and   void   and   the   certificate   of   filing  
thereof   be   cancelled,   and   that   individual   respondents   be   made   to   pay   damages,   in   specified   amounts,   to  
petitioner.  
Petitioner  likewise  filed  with  the  Securities  and  Exchange  Commission  an  "Urgent  Motion  for  Production  and  
Inspection   of   Documents",   alleging   that   the   Secretary   of   respondent   corporation   refused   to   allow   him   to  
inspect  its  records  despite  request  made  by  petitioner  for  production  of  certain  documents  enumerated  in  
the   request,   and   that   respondent   corporation   had   been   attempting   to   suppress   information   from   its  
stockholders  despite  a  negative  reply  by  the  SEC  to  its  query  regarding  their  authority  to  do  so.    
While   the   petition   was   yet   to   be   heard,   respondent   corporation   issued   a   notice   of   special   stockholders'  
meeting  for  the  purpose  of  "ratification  and  confirmation  of  the  amendment  to  the  By-­‐‑laws",  setting  such  

   
3H  A.Y.  2017-­‐2018   154  
 
 

CORPORATION  LAW  CASE  DIGESTS  –  ATTY.  DANTE  DELA  CRUZ  

meeting   for   February   10,   1977.   This   prompted   petitioner   to   ask   respondent   Commission   for   a   summary  
judgment   insofar   as   the   first   cause   of   action   is   concerned,   for   the   alleged   reason   that   by   calling   a   special  
stockholders'   meeting   for   the   aforesaid   purpose,   private   respondents   admitted   the   invalidity   of   the  
amendments.  In  view  of  the  fact  that  the  annual  stockholders'  meeting  of  respondent  corporation  had  been  
scheduled  for  May  10,  1977,  petitioner  filed  with  respondent  Commission  a  Manifestation  stating  that  he  
intended   to   run   for   the   position   of   director   of   respondent   corporation.   Thereafter,   respondents   filed   a  
Manifestation  with  respondent  Commission,  submitting  a  Resolution  of  the  Board  of  Directors  of  respondent  
corporation   disqualifying   and   precluding   petitioner   from   being   a   candidate   for   director   unless   he   could  
submit   evidence   on   May   3,   1977   that   he   does   not   come   within   the   disqualifications   specified   in   the  
amendment  to  the  by-­‐‑laws,  subject  matter  of  SEC  Case  No.  1375.  
SEC.  CASE  NO.  1423  
Petitioner  likewise  alleges  that,  having  discovered  that  respondent  corporation  has  been  investing  corporate  
funds   in   other   corporations   and   businesses   outside   of   the   primary   purpose   clause   of   the   corporation,   in  
violation   of   section   17   1/2   of   the   Corporation   Law,   he   filed   with   respondent   Commission,   on   January   20,  
1977,  a  petition  seeking  to  have  private  respondents  Andres  M.  Soriano,  Jr.  and  Jose  M.  Soriano,  as  well  as  
the  respondent  corporation  declared  guilty  of  such  violation,  and  ordered  to  account  for  such  investments  
and  to  answer  for  damages.  
Issues:   (1.)Whether   or   not   the   provisions   of   the   amended   by-­‐‑laws   of   respondent   corporation,   disqualifying  
a  competitor  from  nomination  or  election  to  the  Board  of  Directors  are  valid  and  reasonable;  
(2)  Whether  or  not  respondent  SEC  gravely  abused  its  discretion  in  denying  petitioner's  request  for  
an   examination   of   the   records   of   San   Miguel   International,   Inc.,   a   fully   owned   subsidiary   of   San   Miguel  
Corporation;  and  
(3)  Whether  or  not  respondent  SEC  committed  grave  abuse  of  discretion  in  allowing  discussion  of  
Item   6   of   the   Agenda   of   the   Annual   Stockholders'   Meeting   on   May   10,   1977,   and   the   ratification   of   the  
investment  in  a  foreign  corporation  of  the  corporate  funds,  allegedly  in  violation  of  section  17-­‐‑1/2  of  the  
Corporation  Law.  
Ruling:    
Whether  or  not  the  amended  by-­‐‑laws  of  SMC  of  disqualifying  a  competitor  from  nomination  or  election  to  
the  Board  of  Directors  of  SMC  are  valid  and  reasonable  —  
Petitioner   claims   that   the   amended   by-­‐‑laws   are   invalid   and   unreasonable   because   they   were   tailored   to  
suppress   the   minority   and   prevent   them   from   having   representation   in   the   Board",   at   the   same   time  
depriving  petitioner  of  his  "vested  right"  to  be  voted  for  and  to  vote  for  a  person  of  his  choice  as  director.  
Upon  the  other  hand,  respondents  Andres  M.  Soriano,  Jr.,  Jose  M.  Soriano  and  San  Miguel  Corporation  content  
that  ex.  conclusion  of  a  competitor  from  the  Board  is  legitimate  corporate  purpose,  considering  that  being  a  
competitor,   petitioner   cannot   devote   an   unselfish   and   undivided   Loyalty   to   the   corporation;   that   it   is  
essentially  a  preventive  measure  to  assure  stockholders  of  San  Miguel  Corporation  of  reasonable  protective  
from  the  unrestrained  self-­‐‑interest  of  those  charged  with  the  promotion  of  the  corporate  enterprise;  that  
access   to   confidential   information   by   a   competitor   may   result   either   in   the   promotion   of   the   interest   of   the  
competitor  at  the  expense  of  the  San  Miguel  Corporation,  or  the  promotion  of  both  the  interests  of  petitioner  
and   respondent   San   Miguel   Corporation,   which   may,   therefore,   result   in   a   combination   or   agreement   in  
violation   of   Article   186   of   the   Revised   Penal   Code   by   destroying   free   competition   to   the   detriment   of   the  
consuming  public.  It  is  further  argued  that  there  is  not  vested  right  of  any  stockholder  under  Philippine  Law  
to   be   voted   as   director   of   a   corporation.   It   is   alleged   that   petitioner,   as   of   May   6,   1978,   has   exercised,  
personally  or  thru  two  corporations  owned  or  controlled  by  him,  control  over  the  following  shareholdings  
in  San  Miguel  Corporation,  vis.:  (a)  John  Gokongwei,  Jr.  —  6,325  shares;  (b)  Universal  Robina  Corporation  —  
738,647  shares;  (c)  CFC  Corporation  —  658,313  shares,  or  a  total  of  1,403,285  shares.  Since  the  outstanding  
capital  stock  of  San  Miguel  Corporation,  as  of  the  present  date,  is  represented  by  33,139,749  shares  with  a  
par   value   of   P10.00,   the   total   shares   owned   or   controlled   by   petitioner   represents   4.2344%   of   the   total  
outstanding  capital  stock  of  San  Miguel  Corporation.  It  is  also  contended  that  petitioner  is  the  president  and  
substantial  stockholder  of  Universal  Robina  Corporation  and  CFC  Corporation,  both  of  which  are  allegedly  

   
3H  A.Y.  2017-­‐2018   155  
 
 

CORPORATION  LAW  CASE  DIGESTS  –  ATTY.  DANTE  DELA  CRUZ  

controlled   by   petitioner   and   members   of   his   family.   It   is   also   claimed   that   both   the   Universal   Robina  
Corporation  and  the  CFC  Corporation  are  engaged  in  businesses  directly  and  substantially  competing  with  
the   alleged   businesses   of   San   Miguel   Corporation,   and   of   corporations   in   which   SMC   has   substantial  
investments.  
It  is  further  asserted  that  in  1977,  the  CFC-­‐‑Robina  group  was  in  direct  competition  on  product  lines  of  SMC.    
According   to   private   respondents,   at   the   Annual   Stockholders'   Meeting   of   March   18,   1976,   9,894  
stockholders,   in   person   or   by   proxy,   owning   23,436,754   shares   in   SMC,   or   more   than   90%   of   the   total  
outstanding  shares  of  SMC,  rejected  petitioner's  candidacy  for  the  Board  of  Directors  because  they  "realized  
the  grave  dangers  to  the  corporation  in  the  event  a  competitor  gets  a  board  seat  in  SMC."  On  September  18,  
1978,  the  Board  of  Directors  of  SMC,  by  "virtue  of  powers  delegated  to  it  by  the  stockholders,"  approved  the  
amendment   to   '   he   by-­‐‑laws   in   question.   At   the   meeting   of   February   10,   1977,   these   amendments   were  
confirmed   and   ratified   by   5,716   shareholders   owning   24,283,945   shares,   or   more   than   80%   of   the   total  
outstanding   shares.   Only   12   shareholders,   representing   7,005   shares,   opposed   the   confirmation   and  
ratification.  At  the  Annual  Stockholders'  Meeting  of  May  10,  1977,  11,349  shareholders,  owning  27,257.014  
shares,  or  more  than  90%  of  the  outstanding  shares,  rejected  petitioner's  candidacy,  while  946  stockholders,  
representing   1,648,801   shares   voted   for   him.   On   the   May   9,   1978   Annual   Stockholders'   Meeting,   12,480  
shareholders,  owning  more  than  30  million  shares,  or  more  than  90%  of  the  total  outstanding  shares.  voted  
against  petitioner.  
AUTHORITY  OF  CORPORATION  TO  PRESCRIBE  QUALIFICATIONS  OF  DIRECTORS  EXPRESSLY  CONFERRED  
BY  LAW  
Private  respondents  contend  that  the  disputed  amended  by  laws  were  adopted  by  the  Board  of  Directors  of  
San   Miguel   Corporation   a-­‐‑,   a   measure   of   self-­‐‑defense   to   protect   the   corporation   from   the   clear   and   present  
danger  that  the  election  of  a  business  competitor  to  the  Board  may  cause  upon  the  corporation  and  the  other  
stockholders   inseparable   prejudice.   Submitted   for   resolution,   therefore,   is   the   issue   —   whether   or   not  
respondent   San   Miguel   Corporation   could,   as   a   measure   of   self-­‐‑   protection,   disqualify   a   competitor   from  
nomination  and  election  to  its  Board  of  Directors.  
It  is  recognized  by  an  authorities  that  'every  corporation  has  the  inherent  power  to  adopt  by-­‐‑laws  'for  its  
internal  government,  and  to  regulate  the  conduct  and  prescribe  the  rights  and  duties  of  its  members  towards  
itself  and  among  themselves  in  reference  to  the  management  of  its  affairs.   12  At  common  law,  the  rule  was  
"that  the  power  to  make  and  adopt  by-­‐‑laws  was  inherent  in  every  corporation  as  one  of  its  necessary  and  
inseparable  legal  incidents.  In  this  jurisdiction,  under  section  21  of  the  Corporation  Law,  a  corporation  may  
prescribe  in  its  by-­‐‑laws  "the  qualifications,  duties  and  compensation  of  directors,  officers  and  employees  ...  "  
This  must  necessarily  refer  to  a  qualification  in  addition  to  that  specified  by  section  30  of  the  Corporation  
Law,   which   provides   that   "every   director   must   own   in   his   right   at   least   one   share   of   the   capital   stock   of   the  
stock  corporation  of  which  he  is  a  director    
NO  VESTED  RIGHT  OF  STOCKHOLDER  TO  BE  ELECTED  DIRECTOR  
Any  person  "who  buys  stock  in  a  corporation  does  so  with  the  knowledge  that  its  affairs  are  dominated  by  a  
majorityof  the  stockholders  and  that  he  impliedly  contracts  that  the  will  of  the  majority  shall  govern  in  all  
matters   within   the   limits   of   the   act   of   incorporation   and   lawfully   enacted   by-­‐‑laws   and   not   forbidden   by  
law."  15  To  this  extent,  therefore,  the  stockholder  may  be  considered  to  have  "parted  with  his  personal  right  
or   privilege   to   regulate   the   disposition   of   his   property   which   he   has   invested   in   the   capital   stock   of   the  
corporation,   and   surrendered   it   to   the   will   of   the   majority   of   his   fellow   incorporators.   ...   It   cannot   therefore  
be  justly  said  that  the  contract,  express  or  implied,  between  the  corporation  and  the  stockholders  is  infringed  
...  by  any  act  of  the  former  which  is  authorized  by  a  majority.    
Pursuant  to  section  18  of  the  Corporation  Law,  any  corporation  may  amend  its  articles  of  incorporation  by  a  
vote  or  written  assent  of  the  stockholders  representing  at  least  two-­‐‑thirds  of  the  subscribed  capital  stock  of  
the  corporation  If  the  amendment  changes,  diminishes  or  restricts  the  rights  of  the  existing  shareholders  
then  the  disenting  minority  has  only  one  right,  viz.:  "to  object  thereto  in  writing  and  demand  payment  for  his  
share."  Under  section  22  of  the  same  law,  the  owners  of  the  majority  of  the  subscribed  capital  stock  may  
amend   or   repeal   any   by-­‐‑law   or   adopt   new   by-­‐‑laws.   It   cannot   be   said,   therefore,   that   petitioner   has   a   vested  

   
3H  A.Y.  2017-­‐2018   156  
 
 

CORPORATION  LAW  CASE  DIGESTS  –  ATTY.  DANTE  DELA  CRUZ  

right   to   be   elected   director,   in   the   face   of   the   fact   that   the   law   at   the   time   such   right   as   stockholder   was  
acquired  contained  the  prescription  that  the  corporate  charter  and  the  by-­‐‑law  shall  be  subject  to  amendment,  
alteration  and  modification.      
It  being  settled  that  the  corporation  has  the  power  to  provide  for  the  qualifications  of  its  directors,  the  next  
question  that  must  be  considered  is  whether  the  disqualification  of  a  competitor  from  being  elected  to  the  
Board  of  Directors  is  a  reasonable  exercise  of  corporate  authority.  
A  DIRECTOR  STANDS  IN  A  FIDUCIARY  RELATION  TO  THE  CORPORATION  AND  ITS  SHAREHOLDERS  
Although  in  the  strict  and  technical  sense,  directors  of  a  private  corporation  are  not  regarded  as  trustees,  
there   cannot   be   any   doubt   that   their   character   is   that   of   a   fiduciary   insofar   as   the   corporation   and   the  
stockholders  as  a  body  are  concerned.  As  agents  entrusted  with  the  management  of  the  corporation  for  the  
collective   benefit   of   the   stockholders,   "they   occupy   a   fiduciary   relation,   and   in   this   sense   the   relation   is   one  
of   trust."   18   "The   ordinary   trust   relationship   of   directors   of   a   corporation   and   stockholders",   according  
to   Ashaman   v.   Miller,   19   "is   not   a   matter   of   statutory   or   technical   law.   It   springs   from   the   fact   that   directors  
have  the  control  and  guidance  of  corporate  affairs  and  property  and  hence  of  the  property  interests  of  the  
stockholders.   Equity   recognizes   that   stockholders   are   the   proprietors   of   the   corporate   interests   and   are  
ultimately  the  only  beneficiaries  thereof  
AN   AMENDMENT   TO   THE   CORPORATION   BY-­‐‑LAW   WHICH   RENDERS   A   STOCKHOLDER   INELIGIBLE   TO   BE  
DIRECTOR,  IF  HE  BE  ALSO  DIRECTOR  IN  A  CORPORATION  WHOSE  BUSINESS  IS  IN  COMPETITION  WITH  
THAT  OF  THE  OTHER  CORPORATION,  HAS  BEEN  SUSTAINED  AS  VALID  
(A)n  amendment  which  renders  ineligible,  or  if  elected,  subjects  to  removal,  a  director  if  he  be  also  a  director  
in   a   corporation   whose   business   is   in   competition   with   or   is   antagonistic   to   the   other   corporation   is  
valid."   24This   is   based   upon   the   principle   that   where   the   director   is   so   employed   in   the   service   of   a   rival  
company,   he   cannot   serve   both,   but   must   betray   one   or   the   other.   As   section   21   of   the   Corporation   Law  
expressly  provides  that  a  corporation  may  make  by-­‐‑laws  for  the  qualifications  of  directors.  Thus,  it  has  been  
held   that   an   officer   of   a   corporation   cannot   engage   in   a   business   in   direct   competition   with   that   of   the  
corporation   where   he   is   a   director   by   utilizing   information   he   has   received   as   such   officer,   under   "the  
established   law   that   a   director   or   officer   of   a   corporation   may   not   enter   into   a   competing   enterprise   which  
cripples  or  injures  the  business  of  the  corporation  of  which  he  is  an  officer  or  director.  26  
It   is   also   well   established   that   corporate   officers   "are   not   permitted   to   use   their   position   of   trust   and  
confidence   to   further   their   private   interests."   27   In   a   case   where   directors   of   a   corporation   cancelled   a  
contract   of   the   corporation   for   exclusive   sale   of   a   foreign   firm's   products,   and   after   establishing   a   rival  
business,  the  directors  entered  into  a  new  contract  themselves  with  the  foreign  firm  for  exclusive  sale  of  its  
products,  the  court  held  that  equity  would  regard  the  new  contract  as  an  offshoot  of  the  old  contract  and,  
therefore,  for  the  benefit  of  the  corporation,  as  a  "faultless  fiduciary  may  not  reap  the  fruits  of  his  misconduct  
to  the  exclusion  of  his  principal.  28  
The   doctrine   of   "corporate   opportunity"   29   is   precisely   a   recognition   by   the   courts   that   the   fiduciary  
standards  could  not  be  upheld  where  the  fiduciary  was  acting  for  two  entities  with  competing  interests.  This  
doctrine   rests   fundamentally   on   the   unfairness,   in   particular   circumstances,   of   an   officer   or   director   taking  
advantage  of  an  opportunity  for  his  own  personal  profit  when  the  interest  of  the  corporation  justly  calls  for  
protection.  30  
It  is  not  denied  that  a  member  of  the  Board  of  Directors  of  the  San  Miguel  Corporation  has  access  to  sensitive  
and  highly  confidential  information,  such  as:  (a)  marketing  strategies  and  pricing  structure;  (b)  budget  for  
expansion   and   diversification;   (c)   research   and   development;   and   (d)   sources   of   funding,   availability   of  
personnel,  proposals  of  mergers  or  tie-­‐‑ups  with  other  firms.  
It  is  obviously  to  prevent  the  creation  of  an  opportunity  for  an  officer  or  director  of  San  Miguel  Corporation,  
who  is  also  the  officer  or  owner  of  a  competing  corporation,  from  taking  advantage  of  the  information  which  
he   acquires   as   director   to   promote   his   individual   or   corporate   interests   to   the   prejudice   of   San   Miguel  
Corporation  and  its  stockholders,  that  the  questioned  amendment  of  the  by-­‐‑laws  was  made.  Certainly,  where  
two  corporations  are  competitive  in  a  substantial  sense,  it  would  seem  improbable,  if  not  impossible,  for  the  

   
3H  A.Y.  2017-­‐2018   157  
 
 

CORPORATION  LAW  CASE  DIGESTS  –  ATTY.  DANTE  DELA  CRUZ  

director,  if  he  were  to  discharge  effectively  his  duty,  to  satisfy  his  loyalty  to  both  corporations  and  place  the  
performance  of  his  corporation  duties  above  his  personal  concerns.  
Indeed,   access   by   a   competitor   to   confidential   information   regarding   marketing   strategies   and   pricing  
policies  of  San  Miguel  Corporation  would  subject  the  latter  to  a  competitive  disadvantage  and  unjustly  enrich  
the  competitor,  for  advance  knowledge  by  the  competitor  of  the  strategies  for  the  development  of  existing  
or  new  markets  of  existing  or  new  products  could  enable  said  competitor  to  utilize  such  knowledge  to  his  
advantage.  32  
There   is   another   important   consideration   in   determining   whether   or   not   the   amended   by-­‐‑laws   are  
reasonable.   The   Constitution   and   the   law   prohibit   combinations   in   restraint   of   trade   or   unfair   competition.  
Thus,   section   2   of   Article   XIV   of   the   Constitution   provides:   "The   State   shall   regulate   or   prohibit   private  
monopolies  when  the  public  interest  so  requires.  No  combinations  in  restraint  of  trade  or  unfair  competition  
shall  be  snowed."  
From  the  foregoing  definitions,  it  is  apparent  that  the  contentions  of  petitioner  are  not  in  accord  with  reality.  
The  election  of  petitioner  to  the  Board  of  respondent  Corporation  can  bring  about  an  illegal  situation.  This  is  
because  an  express  agreement  is  not  necessary  for  the  existence  of  a  combination  or  conspiracy  in  restraint  
of  trade.   40  It  is  enough  that  a  concert  of  action  is  contemplated  and  that  the  defendants  conformed  to  the  
arrangements,  41  and  what  is  to  be  considered  is  what  the  parties  actually  did  and  not  the  words  they  used.  
There  is  here  a  statutory  recognition  of  the  anti-­‐‑competitive  dangers  which  may  arise  when  an  individual  
simultaneously  acts  as  a  director  of  two  or  more  competing  corporations.  A  common  director  of  two  or  more  
competing   corporations   would   have   access   to   confidential   sales,   pricing   and   marketing   information   and  
would  be  in  a  position  to  coordinate  policies  or  to  aid  one  corporation  at  the  expense  of  another,  thereby  
stifling  competition.    
Finally,   considering   that   both   Robina   and   SMC   are,   to   a   certain   extent,   engaged   in   agriculture,   then   the  
election   of   petitioner   to   the   Board   of   SMC   may   constitute   a   violation   of   the   prohibition   contained   in   section  
13(5)   of   the   Corporation   Law.   Said   section   provides   in   part   that   "any   stockholder   of   more   than   one  
corporation   organized   for   the   purpose   of   engaging   in   agriculture   may   hold   his   stock   in   such  
corporations  solely  for  investment  and  not  for  the  purpose  of  bringing  about  or  attempting  to  bring  about  a  
combination  to  exercise  control  of  incorporations  ...  ."  
Neither  are  We  persuaded  by  the  claim  that  the  by-­‐‑law  was  Intended  to  prevent  the  candidacy  of  petitioner  
for  election  to  the  Board.  If  the  by-­‐‑law  were  to  be  applied  in  the  case  of  one  stockholder  but  waived  in  the  
case  of  another,  then  it  could  be  reasonably  claimed  that  the  by-­‐‑law  was  being  applied  in  a  discriminatory  
manner.  However,  the  by  law,  by  its  terms,  applies  to  all  stockholders.  Sound  principles  of  public  policy  and  
management,  therefore,  support  the  view  that  a  by-­‐‑law  which  disqualifies  a  competition  from  election  to  the  
Board  of  Directors  of  another  corporation  is  valid  and  reasonable.  
In   the   absence   of   any   legal   prohibition   or   overriding   public   policy,   wide   latitude   may   be   accorded   to   the  
corporation   in   adopting   measures   to   protect   legitimate   corporation   interests.   Thus,   "where   the  
reasonableness  of  a  by-­‐‑law  is  a  mere  matter  of  judgment,  and  upon  which  reasonable  minds  must  necessarily  
differ,  a  court  would  not  be  warranted  in  substituting  its  judgment  instead  of  the  judgment  of  those  who  are  
authorized  to  make  by-­‐‑laws  and  who  have  expressed  their  authority.  45  
Although  it  is  asserted  that  the  amended  by-­‐‑laws  confer  on  the  present  Board  powers  to  perpetua  themselves  
in   power   such   fears   appear   to   be   misplaced.   This   power,   but   is   very   nature,   is   subject   to   certain   well  
established  limitations.  One  of  these  is  inherent  in  the  very  convert  and  definition  of  the  terms  "competition"  
and   "competitor".   "Competition"   implies   a   struggle   for   advantage   between   two   or   more   forces,   each  
possessing,  in  substantially  similar  if  not  Identical  degree,  certain  characteristics  essential  to  the  business  
sought.  It  means  an  independent  endeavor  of  two  or  more  persons  to  obtain  the  business  patronage  of  a  third  
by   offering   more   advantageous   terms   as   an   inducement   to   secure   trade.   46   The   test   must   be   whether   the  
business   does   in   fact   compete,   not   whether   it   is   capable   of   an   indirect   and   highly   unsubstantial   duplication  
of   an   isolated   or   non-­‐‑characteristics   activity.   47   It   is,   therefore,   obvious   that   not   every   person   or   entity  
engaged  in  business  of  the  same  kind  is  a  competitor.  Such  factors  as  quantum  and  place  of  business,  Identity  
of  products  and  area  of  competition  should  be  taken  into  consideration.  It  is,  therefore,  necessary  to  show  

   
3H  A.Y.  2017-­‐2018   158  
 
 

CORPORATION  LAW  CASE  DIGESTS  –  ATTY.  DANTE  DELA  CRUZ  

that  petitioner's  business  covers  a  substantial  portion  of  the  same  markets  for  similar  products  to  the  extent  
of  not  less  than  10%  of  respondent  corporation's  market  for  competing  products.  While  We  here  sustain  the  
validity   of   the   amended   by-­‐‑laws,   it   does   not   follow   as   a   necessary   consequence   that   petitioner   is   ipso  
facto   disqualified.   Consonant   with   the  requirement  of  due  process,  there   must   be   due  hearing  at  which   the  
petitioner  must  be  given  the  fullest  opportunity  to  show  that  he  is  not  covered  by  the  disqualification.  As  
trustees  of  the  corporation  and  of  the  stockholders,  it  is  the  responsibility  of  directors  to  act  with  fairness  to  
the   stockholders.48Pursuant   to   this   obligation   and   to   remove   any   suspicion   that   this   power   may   be   utilized  
by  the  incumbent  members  of  the  Board  to  perpetuate  themselves  in  power,  any  decision  of  the  Board  to  
disqualify   a   candidate   for   the   Board   of   Directors   should   be   reviewed   by   the   Securities   behind   Exchange  
Commission  en  banc  and  its  decision  shall  be  final  unless  reversed  by  this  Court  on  certiorari.  49  Indeed,  it  is  
a  settled  principle  that  where  the  action  of  a  Board  of  Directors  is  an  abuse  of  discretion,  or  forbidden  by  
statute,  or  is  against  public  policy,  or  is  ultra  vires,  or  is  a  fraud  upon  minority  stockholders  or  creditors,  or  
will  result  in  waste,  dissipation  or  misapplication  of  the  corporation   assets,   a   court   of   equity   has   the   power  
to  grant  appropriate  relief.  50  
Whether   or   not   respondent   SEC   gravely   abused   its   discretion   in   denying   petitioner's   request   for   an  
examination   of   the   records   of   San   Miguel   International   Inc.,   a   fully   owned   subsidiary   of   San   Miguel  
Corporation  —  
The  stockholder's  right  of  inspection  of  the  corporation's  books  and  records  is  based  upon  their  ownership  
of   the   assets   and   property   of   the   corporation.   It   is,   therefore,   an   incident   of   ownership   of   the   corporate  
property,  whether  this  ownership  or  interest  be  termed  an  equitable  ownership,  a  beneficial  ownership,  or  
a  ownership.  52  This  right  is  predicated  upon  the  necessity  of  self-­‐‑protection.  It  is  generally  held  by  majority  
of  the  courts  that  where  the  right  is  granted  by  statute  to  the  stockholder,  it  is  given  to  him  as  such  and  must  
be   exercised   by   him   with   respect   to   his   interest   as   a   stockholder   and   for   some   purpose   germane   thereto   or  
in   the   interest   of   the   corporation.   53   In   other   words,   the   inspection   has   to   be   germane   to   the   petitioner's  
interest  as  a  stockholder,  and  has  to  be  proper  and  lawful  in  character  and  not  inimical  to  the  interest  of  the  
corporation.     In   other   words,   the   specific   provisions   take   from   the   stockholder   the   burden   of   showing  
propriety   of   purpose   and   place   upon   the   corporation   the   burden   of   showing   impropriety   of   purpose   or  
motive.   58   It   appears   to   be   the   general   rule   that   stockholders   are   entitled   to   full   information   as   to   the  
management  of  the  corporation  and  the  manner  of  expenditure  of  its  funds,  and  to  inspection  to  obtain  such  
information,  especially  where  it  appears  that  the  company  is  being  mismanaged  or  that  it  is  being  managed  
for  the  personal  benefit  of  officers  or  directors  or  certain  of  the  stockholders  to  the  exclusion  of  others."  
Whether   or   not   respondent   SEC   gravely   abused   its   discretion   in   allowing   the   stockholders   of   respondent  
corporation  to  ratify  the  investment  of  corporate  funds  in  a  foreign  corporation  
Petitioner  reiterates  his  contention  in  SEC  Case  No.  1423  that  respondent  corporation  invested  corporate  
funds  in  SMI  without  prior  authority  of  the  stockholders,  thus  violating  section  17-­‐‑1/2  of  the  Corporation  
Law,  and  alleges  that  respondent  SEC  should  have  investigated  the  charge,  being  a  statutory  offense,  instead  
of  allowing  ratification  of  the  investment  by  the  stockholders.  
Respondent  SEC's  position  is  that  submission  of  the  investment  to  the  stockholders  for  ratification  is  a  sound  
corporate  practice  and  should  not  be  thwarted  but  encouraged.  
Section  17-­‐‑1/2  of  the  Corporation  Law  allows  a  corporation  to  "invest  its  funds  in  any  other  corporation  or  
business  or  for  any  purpose  other  than  the  main  purpose  for  which  it  was  organized"  provided  that  its  Board  
of  Directors  has  been  so  authorized  by  the  affirmative  vote  of  stockholders  holding  shares  entitling  them  to  
exercise   at   least   two-­‐‑thirds   of   the   voting   power.   If   the   investment   is   made   in   pursuance   of   the   corporate  
purpose,  it  does  not  need  the  approval  of  the  stockholders.  It  is  only  when  the  purchase  of  shares  is  done  
solely  for  investment  and  not  to  accomplish  the  purpose  of  its  incorporation  that  the  vote  of  approval  of  the  
stockholders  holding  shares  entitling  them  to  exercise  at  least  two-­‐‑thirds  of  the  voting  power  is  necessary.  69  
As   stated   by   respondent   corporation,   the   purchase   of   beer   manufacturing   facilities   by   SMC   was   an  
investment   in   the   same   business   stated   as   its   main   purpose   in   its   Articles   of   Incorporation,   which   is   to  
manufacture  and  market  beer.  It  appears  that  the  original  investment  was  made  in  1947-­‐‑1948,  when  SMC,  
then  San  Miguel  Brewery,  Inc.,  purchased  a  beer  brewery  in  Hongkong  (Hongkong  Brewery  &  Distillery,  Ltd.)  

   
3H  A.Y.  2017-­‐2018   159  
 
 

CORPORATION  LAW  CASE  DIGESTS  –  ATTY.  DANTE  DELA  CRUZ  

for  the  manufacture  and  marketing  of  San  Miguel  beer  thereat.  Restructuring  of  the  investment  was  made  in  
1970-­‐‑1971  thru  the  organization  of  SMI  in  Bermuda  as  a  tax  free  reorganization.  
Assuming  arguendo  that  the  Board  of  Directors  of  SMC  had  no  authority  to  make  the  assailed  investment,  
there  is  no  question  that  a  corporation,  like  an  individual,  may  ratify  and  thereby  render  binding  upon  it  the  
originally  unauthorized  acts  of  its  officers  or  other  agents.  70  This  is  true  because  the  questioned  investment  
is  neither  contrary  to  law,  morals,  public  order  or  public  policy.  It  is  a  corporate  transaction  or  contract  which  
is  within  the  corporate  powers,  but  which  is  defective  from  a  supported  failure  to  observe  in  its  execution  
the.   requirement   of   the   law   that   the   investment   must   be   authorized   by   the   affirmative   vote   of   the  
stockholders  holding  two-­‐‑thirds  of  the  voting  power.  This  requirement  is  for  the  benefit  of  the  stockholders.  
The   stockholders   for   whose   benefit   the   requirement   was   enacted   may,   therefore,   ratify   the   investment   and  
its  ratification  by  said  stockholders  obliterates  any  defect  which  it  may  have  had  at  the  outset.  
 
 
 
120.    PHILPOTTS  V.  PHILIPPINE  MANUFACTURING  
 
 
 
121.  PARDO  VS.  THE  HERCULES  LUMBER  CO.,  INC.  
G.R.  NO.  L-­‐‑22442                          AUGUST  1,  1924  
STREET,  J.  
 
DOCTRINE:  
  Our   statute   declares   that   the   right   of   inspection   can   be   exercised   "at   reasonable   hours."   This   means  
at  reasonable  hours  on  business  days  throughout  the  year,  and  not  merely  during  some  arbitrary  period  of  a  
few  days  chosen  by  the  directors.  
 
FACTS:  
  Antonio  Pardo,  a  stockholder  in  the  Hercules  Lumber  Company,  Inc.,  one  of  the  respondents  herein,  
seeks   by   this   original   proceeding   in   the   Supreme   Court   to   obtain   a   writ   of   mandamus   to   compel   the  
respondents  to  permit  the  plaintiff  and  his  duly  authorized  agent  and  representative  to  examine  the  records  
and  business  transactions  of  said  company.  
  The  main  ground  upon  which  the  defense  appears  to  be  rested  has  reference  to  the  time,  or  times,  
within  which  the  right  of  inspection  may  be  exercised.  In  this  connection  the  answer  asserts  that  in  article  
10   of   the   By-­‐‑laws   of   the   respondent   corporation   it   is   declared   that   "Every   shareholder   may   examine   the  
books   of   the   company   and   other   documents   pertaining   to   the   same   upon   the   days   which   the   board   of  
directors  shall  annually  fix."  
  The   board   also   resolved   to   call   the   usual   general   (meeting   of   shareholders)   for   March   30   of   the  
present  year,  with  notice  to  the  shareholders  that  the  books  of  the  company  are  at  their  disposition  from  the  
15th  to  25th  of  the  same  month  for  examination,  in  appropriate  hours.  
 
ISSUE:  
  Whether  the  right  of  a  stockholder  to  examine  the  records  and  business  transactions  of  a  corporation  
may  be  deprived.  
 
HELD:  
  We  are  entirely  unable  to  concur  in  this  contention.  The  general  right  given  by  the  statute  may  not  
be  lawfully  abridged  to  the  extent  attempted  in  this  resolution.  It  may  be  admitted  that  the  officials  in  charge  
of  a  corporation  may  deny  inspection  when  sought  at  unusual  hours  or  under  other  improper  conditions;  but  

   
3H  A.Y.  2017-­‐2018   160  
 
 

CORPORATION  LAW  CASE  DIGESTS  –  ATTY.  DANTE  DELA  CRUZ  

neither  the  executive  officers  nor  the  board  of  directors  have  the  power  to  deprive  a  stockholder  of  the  right  
altogether.  A  by-­‐‑law  unduly  restricting  the  right  of  inspection  is  undoubtedly  invalid.  
  It  will  be  noted  that  our  statute  declares  that  the  right  of  inspection  can  be  exercised  "at  reasonable  
hours."  This  means  at  reasonable  hours  on  business  days  throughout  the  year,  and  not  merely  during  some  
arbitrary  period  of  a  few  days  chosen  by  the  directors.  
 
 
 
122.  RAMON  A.  GONZALES      V.  THE  PHILIPPINE  NATIONAL  BANK  
G.R.  NO.  L-­‐‑33320  MAY  30,  1983  
VASQUEZ,  J.  
 
DOCTRINE:  
 
Among   the   changes   introduced   in   the   new   Code   with   respect   to   the   right   of   inspection   granted   to   a  
stockholder   are   the   following   the   records   must   be   kept   at   the   principal   office   of   the   corporation;   the  
inspection  must  be  made  on  business  days;  the  stockholder  may  demand  a  copy  of  the  excerpts  of  the  records  
or  minutes;  and  the  refusal  to  allow  such  inspection  shall  subject  the  erring  officer  or  agent  of  the  corporation  
to  civil  and  criminal  liabilities.  However,  while  seemingly  enlarging  the  right  of  inspection,  the  new  Code  has  
prescribed  limitations  to  the  same.  It  is  now  expressly  required  as  a  condition  for  such  examination  that  the  
one   requesting   it   must   not   have   been   guilty   of   using   improperly   any   information   through   a   prior  
examination,   and   that   the   person   asking   for   such   examination   must   be   "acting   in   good   faith   and   for   a  
legitimate  purpose  in  making  his  demand."    
 
FACTS:    
 
Petitioner  Ramon  A.  Gonzales  instituted  in  the  CFI  of  Manila  a  special  civil  action  for  mandamus  against  the  
herein  respondent  praying  that  the  latter  be  ordered  to  allow  him  to  look  into  the  books  and  records  of  the  
respondent  bank  in  order  to  satisfy  himself  as  to  the  truth  of  the  published  reports  that  the  respondent  has  
guaranteed  the  obligation  of  Southern  Negros  Development  Corporation  in  the  purchase  of  a  US$  23  million  
sugar-­‐‑mill   to   be   financed   by   Japanese   suppliers   and   financiers;   that   the   respondent   is   financing   the  
construction   of   the   P   21   million   Cebu-­‐‑Mactan   Bridge   to   be   constructed   by   V.C.   Ponce,   Inc.,   and   the  
construction  of  Passi  Sugar  Mill  at  Iloilo  by  the  Honiron  Philippines,  Inc.,  as  well  as  to  inquire  into  the  validity  
of  Id  transactions.  The  petitioner  has  alleged  hat  his  written  request  for  such  examination  was  denied  by  the  
respondent.  The  trial  court  having  dismissed  the  petition  for  mandamus,  the  instant  appeal  to  review  the  
said  dismissal  was  filed.    
 
The  court  a  quo  denied  the  prayer  of  the  petitioner  that  he  be  allowed  to  examine  and  inspect  the  books  and  
records   of   the   respondent   bank   regarding   the   transactions   mentioned   on   the   grounds   that   the   right   of   a  
stockholder   to   inspect   the   record   of   the   business   transactions   of   a   corporation   granted   under   Section   51   of  
the  former  Corporation  Law  (Act  No.  1459,  as  amended)  is  not  absolute,  but  is  limited  to  purposes  reasonably  
related   to   the   interest   of   the   stockholder,   must   be   asked   for   in   good   faith   for   a   specific   and   honest   purpose  
and   not   gratify   curiosity   or   for   speculative   or   vicious   purposes;   that   such   examination   would   violate   the  
confidentiality  of  the  records  of  the  respondent  bank  as  provided  in  Section  16  of  its  charter,  Republic  Act  
No.  1300,  as  amended;  and  that  the  petitioner  has  not  exhausted  his  administrative  remedies.  
 
ISSUE:  
 
WON  the  lower  court  erred  in  finding  that  petitioner  had  improper  motive  in  asking  for  an  examination  of  
the  books  and  records  of  the  respondent  bank  thus  disqualifying  him  from  exercising  the  right.  

   
3H  A.Y.  2017-­‐2018   161  
 
 

CORPORATION  LAW  CASE  DIGESTS  –  ATTY.  DANTE  DELA  CRUZ  

 
HELD:  
 
•   Petitioner   may   no   longer   insist   on   his   interpretation   of   Section   51   of   Act   No.   1459.   The   former  
Corporation   Law   (Act   No.   1459,   as   amended)   has   been   replaced   by   Batas   Pambansa   Blg.   68,  
otherwise  known  as  the  "Corporation  Code  of  the  Philippines."    
 
Among   the   changes   introduced   in   the   new   Code   with   respect   to   the   right   of   inspection   granted   to   a  
stockholder  are  the  following  the  records  must  be  kept  at  the  principal  office  of  the  corporation;  the  
inspection  must  be  made  on  business  days;  the  stockholder  may  demand  a  copy  of  the  excerpts  of  
the  records  or  minutes;  and  the  refusal  to  allow  such  inspection  shall  subject  the  erring  officer  or  
agent  of  the  corporation  to  civil  and  criminal  liabilities.  However,  while  seemingly  enlarging  the  right  
of  inspection,  the  new  Code  has  prescribed  limitations  to  the  same.  It  is  now  expressly  required  as  a  
condition   for   such   examination   that   the   one   requesting   it   must   not   have   been   guilty   of   using  
improperly   any   information   through   a   prior   examination,   and   that   the   person   asking   for   such  
examination  must  be  "acting  in  good  faith  and  for  a  legitimate  purpose  in  making  his  demand."    
 
•   Although  the  petitioner  has  claimed  that  he  has  justifiable  motives  in  seeking  the  inspection  of  the  
books  of  the  respondent  bank,  he  has  not  set  forth  the  reasons  and  the  purposes  for  which  he  desires  
such   inspection,   except   to   satisfy   himself   as   to   the   truth   of   published   reports   regarding   certain  
transactions   entered   into   by   the   respondent   bank   and   to   inquire   into   their   validity.   The  
circumstances   under   which   he   acquired   one   share   of   stock   in   the   respondent   bank   purposely   to  
exercise   the   right   of   inspection   do   not   argue   in   favor   of   his   good   faith   and   proper   motivation.  
Admittedly   he   sought   to   be   a   stockholder   in   order   to   pry   into   transactions   entered   into   by   the  
respondent  bank  even  before  he  became  a  stockholder.  His  obvious  purpose  was  to  arm  himself  with  
materials   which   he   can   use   against   the   respondent   bank   for   acts   done   by   the   latter   when   the  
petitioner  was  a  total  stranger  to  the  same.  He  could  have  been  impelled  by  a  laudable  sense  of  civic  
consciousness,  but  it  could  not  be  said  that  his  purpose  is  germane  to  his  interest  as  a  stockholder.    
 
•   The  Philippine  National  Bank  is  not  an  ordinary  corporation.  Having  a  charter  of  its  own,  it  is  not  
governed,  as  a  rule,  by  the  Corporation  Code  of  the  Philippines.  Section  4  of  the  said  Code  provides:    
 
SEC.  4.  Corporations  created  by  special  laws  or  charters.   —  Corporations  created  by  special  laws  or  
charters  shall  be  governed  primarily  by  the  provisions  of  the  special  law  or  charter  creating  them  or  
applicable  to  them.  supplemented  by  the  provisions  of  this  Code,  insofar  as  they  are  applicable.    
 
Section  16  of  its  charter  provides:  
 
Sec.   16.   Confidential   information.   —The   Superintendent   of   Banks   and   the   Auditor   General,   or   other  
officers  designated  by  law  to  inspect  or  investigate  the  condition  of  the  National  Bank,  shall  not  reveal  
to  any  person  other  than  the  President  of  the  Philippines,  the  Secretary  of  Finance,  and  the  Board  of  
Directors  the  details  of  the  inspection  or  investigation,  nor  shall  they  give  any  information  relative  to  
the   funds   in   its   custody,   its   current   accounts   or   deposits   belonging   to   private   individuals,  
corporations,  or  any  other  entity,  except  by  order  of  a  Court  of  competent  jurisdiction,'    
 
The  provision  of  Section  74  of  Batas  Pambansa  Blg.  68  of  the  new  Corporation  Code  with  respect  to  
the  right  of  a  stockholder  to  demand  an  inspection  or  examination  of  the  books  of  the  corporation  
may  not  be  reconciled  with  the  abovequoted  provisions  of  the  charter  of  the  respondent  bank.    
 

   
3H  A.Y.  2017-­‐2018   162  
 
 

CORPORATION  LAW  CASE  DIGESTS  –  ATTY.  DANTE  DELA  CRUZ  

123.  EUGENIO  VERAGUTH,  DIRECTOR  AND  STOCKHOLDER  OF  THE  ISABELA  SUGAR  COMPANY,  INC.  
VS.  ISABELA  SUGAR  COMPANY,  INC.,  GIL  MONTILLA,  ACTING  PRESIDENT,  AND  AGUSTIN  B.  MONTILLA  
G.R.  NO.  L-­‐‑37064                          OCTOBER  4,  1932  
MALCOLM,  J.:  
 
FACTS:  
The  parties  to  this  action  are  Eugenio  Veraguth,  a  director  and  stockholder  of  the  Isabela  Sugar  Company,  
Inc.,  who  is  the  petitioner,  and  the  Isabela  Sugar  Company,  Inc.,  Gil  Montilla,  acting  president  of  the  company,  
and   Agustin   B.   Montilla,   secretary   of   the   company,   who   are   the   respondents.   The   petitioner   prays:   (a)   That  
the  respondents  be  required  within  five  days  from  receipt  of  notice  of  this  petition  to  show  cause  why  they  
refuse  to  notify  the  petitioner,  as  director,  of  the  regular  and  special  meetings  of  the  board  of  directors,  and  
to   place   at   his   disposal   at   reasonable   hours,   the   minutes,   and   documents,   and   books   of   the   aforesaid  
corporation,  for  his  inspection  as  director  and  stockholder,  and  to  issue,  upon  payment  of  the  fees,  certified  
copies  of  any  documentation  in  connection  with  said  minutes,  documents,  and  books  of  the  corporation;  and  
(b)   that,   in   view   of   the   memoranda   and   hearing   of   the   parties,   a   final   and   absolute   writ   of   mandamus   be  
issued  to  each  and  all  of  the  respondents  to  notify  immediately  the  petitioner  within  the  reglamentary  period,  
of   all   regular   and   special   meetings   of   the   board   of   directors   of   the   Isabela   Sugar   Central   Company,   Inc.,   and  
to  place  at  his  disposal  at  reasonable  hours  the  minutes,  documents,  and  books  of  said  corporation  for  his  
inspection  as  director  and  stockholder,  and  to  issue  immediately,  upon  payment  of  the  fees,  certified  copies  
of   any   documentation   in   connection   with   said   minutes,   documents,   and   the   books   of   the   aforesaid  
corporation.   To   the   petition   an   answer   has   been   interposed   by   the   respondent,   too   long   to   be   here  
summarized,  which  raised  questions  of  fact  and  law.  
 
ISSUE:  
1.   Whether  or  not  the  respondents  failed  to  notify  the  petitioner  of  the  regular  and  special  meetings  of  
the  BOD.  
2.   Whether  or  not  petitioner  is  entitled  to  inspect  the  books  of  the  company.  
 
HELD:  
Speaking  to  the  first  point  with  which  the  petition  is  concerned,  relating  to  the  alleged  failure  of  the  secretary  
of  the  company  to  notify  the  petitioner  in  due  time  of  a  special  meeting  of  the  company,  we  find  by-­‐‑laws,  
together   with   a   resolution   of   the   board   of   directors,   providing   for   the   holding   of   ordinary   and   special  
meetings.  Whether  there  was  a  malicious  attempt  to  keep  Director  Veraguth  from  attending  a  special  meeting  
of  the  board  of  the  board  of  directors  at  which  the  compensation  of  the  attorneys  of  the  company  was  fixed,  
or  whether  Director  Veraguth,  in  a  spirit  of  antogonism,  has  made  this  merely  a  pretext  to  cause  trouble,  we  
are  unable  definitely  to  say.  This  much,  however,  can  appropriately  be  stated  and  is  decisive,  and  this  is  that  
the  meeting  in  question  is  in  the  past  and,  therefore,  now  merely  presents  an  academic  question;  that  no  
damage  was  caused  to  Veraguth  by  the  action  taken  at  the  special  meeting  which  he  did  not  attend,  since  his  
interests   were   fully   protected   by   the   Philippine   National   Bank;   and   that   as   to   meetings   in   the   future   it   is   to  
be  presumed  that  the  secretary  of  the  company  will  fulfill  the  requirements  of  the  resolutions  of  the  company  
pertaining   to   regular   and   special   meetings.   It   will,   of   course,   be   incumbent   upon   Veraguth   to   give   formal  
notice   to   the   secretary   of   his   post-­‐‑office   address   if   he   desires   notice   sent   to   a   particular  
residence.  1awphil.net  
On  the  second  question  pertaining  to  the  right  of  inspection  of  the  books  of  the  company,  we  find  Director  
Veraguth  telegraphing  the  secretary  of  the  company,  asking  the  latter  to  forward  in  the  shortest  possible  
time  a  certified  copy  of  the  resolution  of  the  board  of  directors  concerning  the  payment  of  attorney's  fees  in  
the  case  against  the  Isabela  Sugar  Company  and  others.  To  this  the  secretary  made  answer  by  letter  stating  
that,  since  the  minutes  of  the  meeting  in  question  had  not  been  signed  by  the  directors  present,  a  certified  
copy   could   not   be   furnished   and   that   as   to   other   proceedings   of   the   stockholders   a   request   should   be   made  
to   the   president   of   the   Isabela   Sugar   Company,   Inc.   It   further   appears   that   the   board   of   directors   adopted   a  

   
3H  A.Y.  2017-­‐2018   163  
 
 

CORPORATION  LAW  CASE  DIGESTS  –  ATTY.  DANTE  DELA  CRUZ  

resolution  providing  for  inspection  of  the  books  and  the  taking  of  copies  "by  authority  of  the  President  of  the  
corporation  previously  obtained  in  each  case."  
The  Corporation  Law,  section  51,  provides  that:  
All  business  corporations  shall  keep  and  carefully  preserve  a  record  of  all  business  transactions,  and  
a   minute   of   all   meetings   of   directors,   members,   or   stockholders,   in  which   shall   be   set   forth   in   detail  
the  time  and  place  of  holding  the  meeting  was  regular  or  special,  if  special  its  object,  those  present  
and  absent,  and  every  act  done  or  ordered  done  at  the  meeting.  .  .  .  
The  record  of  all  business  transactions  of  the  corporation  and  the  minutes  of  any  meeting  shall  be  
open   to   the   inspection   of   any   director,   member,   or   stockholder   of   the   corporation   at   reasonable  
hours.  
The  above  puts  in  statutory  form  the  general  principles  of  Corporation  Law.  Directors  of  a  corporation  have  
the  unqualified  right  to  inspect  the  books  and  records  of  the  corporation  at  all  reasonable  times.  Pretexts  
may  not  be  put  forward  by  officers  of  corporations  to  keep  a  director  or  shareholder  from  inspecting  the  
books  and  minutes  of  the  corporation,  and  the  right  of  inspection  is  not  to  be  denied  on  the  ground  that  the  
director  or  shareholder  is  on  unfriendly  terms  with  the  officers  of  the  corporation  whose  records  are  sought  
to   be   inspected.   A   director   or   stockholder   can   not   of   course   make   copies,   abstracts,   and   memoranda   of  
documents,  books,  and  papers  as  an  incident  to  the  right  of  inspection,  but  cannot,  without  an  order  of  a  
court,  be  permitted  to  take  books  from  the  office  of  the  corporation.  We  do  not  conceive,  however,  that  a  
director  or  stockholder  has  any  absolute  right  to  secure  certified  copies  of  the  minutes  of  the  corporation  
until  these  minutes  have  been  written  up  and  approved  by  the  directors.  
Combining   the   facts   and   the   law,   we   do   not   think   that   anything   improper   occurred   when   the   secretary  
declined   to   furnish   certified   copies   of   minutes   which   had   not   been   approved   by   the   board   of   directors,   and  
that   while   so   much   of   the   last   resolution   of   the   board   of   directors   as   provides   for   prior   approval   of   the  
president  of  the  corporation  before  the  books  of  the  corporation  can  be  inspected  puts  an  illegal  obstacle  in  
the  way  of  a  stockholder  or  director,  that  resolution,  so  far  as  we  are  aware,  has  not  been  enforced  to  the  
detriment  of  anyone.  
 
 
 
124.  CANDIDO  PASCUAL  VS.  EUGENIO  DEL  SAZ  OROZCO,  ET  AL  
G.R.  NO.  L-­‐‑5174,  MARCH  17,  1911  
TRENT,  J.:  
 
DOCTRINE:    
A  stockholder  in  a  banking  corporation  had  a  right  to  maintain  a  suit  for  and  on  behalf  of  the  corporation,  
but  the  extent  of  such  right  depends  upon  when  and  for  what  purpose  he  acquired  the  shares  of  stock  of  
which   he   is   the   owner.   A   stockholder   in   a   corporation   who   was   not   such   at   the   time   when   alleged  
objectionable   transactions   took   place,   or   whose   shares   of   stock   have   not   since   devolved   upon   him   by  
operation  of  law,  cannot  maintain  suits  of  this  character,  unless  such  transactions  continue  and  are  injurious  
to  such  stockholder  or  affect  him  especially  or  specifically  in  some  other  way.  
 
FACTS:    
This  action  was  brought  by  the  plaintiff  Pascual,  in  his  own  right  as  a  stockholder  of  the  bank,  for  the  benefit  
of   the   bank,   and   all   the   other   stockholders   thereof.   The   Banco   Español-­‐‑Filipino   is   a   banking   corporation,  
constituted  as  such  by  royal  decree  of  the  Crown  of  Spain  in  the  year  1854,  the  original  grant  having  been  
subsequently  extended  and  modified  by  royal  decree  of  July  14,  1897,  and  by  Act  No.  1790  of  the  Philippine  
Commission.  
 

   
3H  A.Y.  2017-­‐2018   164  
 
 

CORPORATION  LAW  CASE  DIGESTS  –  ATTY.  DANTE  DELA  CRUZ  

It   is   alleged   in   the   amended   complaint   that   the   only   compensation   contemplated   or   provided   for   the  
managing  officers  of  the  bank  was  a  certain  per  cent  of  the  net  profits  resulting  from  the  bank's  operations,  
as  set  forth  in  article  30  of  its  reformed  charter  or  statutes.  
 
The  gist  of  the  first  and  second  causes  of  action  is  as  follows:  The  defendants  constitute  a  majority  of  the  
present  board  of  directors  of  the  bank,  who  alone  can  authorize  an  action  against  them  in  the  name  of  the  
corporation.   It   appears   that   during   the   years   1903,   1904,   1905,   and   1907   the   defendants   and   appellees,  
without   the   knowledge,   consent,   or   acquiescence   of   the   stockholders,   deducted   their   respective  
compensation  from  the  gross  income  instead  of  from  the  net  profits  of  the  bank,  thereby  defrauding  the  bank  
and  its  stockholders  of  approximately  P20,000  per  annum.    
The  second  cause  of  action  sets  forth  that  defendants'  and  appellees'  immediate  predecessors  in  office  in  the  
bank   during   the   years   1899,   1900,   1901,   and   1902,   committed   the   same   illegality   as   to   their   compensation  
as  is  charged  against  the  defendants  themselves.  In  the  four  years  immediately  following  the  year  1902,  the  
defendants   and   appellees   were   the   only   officials   or   representatives   of   the   bank   who   could   and   should  
investigate   and   take   action   in   regard   to   the   sums   of   money   thus   fraudulently   appropriated   by   their  
predecessors.  They  were  the  only  persons  interested  in  the  bank  who  knew  of  the  fraudulent  appropriation  
by  their  predecessors.  
 
The  court  below  sustained  the  demurrer  as  to  the  first  and  second  causes  of  action  on  the  ground  that  in  
actions   of   this   character   the   plaintiff   must   aver   in   his   complaint   that   he   was   the   owner   of   stock   in   the  
corporation  at  the  time  of  the  occurrences  complained  of,  or  else  that  the  stock  has  since  devolved  upon  him  
by  operation  of  law.  
 
ISSUE:  
WON  the  petitioner  has  a  cause  of  action  to  file  a  derivative  suit.  
 
HELD:    
Yes.   As   to   the   first   cause   of   action:   In   suits   of   this   character   the   corporation   itself   and   not   the   plaintiff  
stockholder  is  the  real  party  in  interest.  The  rights  of  the  individual  stockholder  are  merged  into  that  of  the  
corporation.  It  is  a  universally  recognized  doctrine  that  a  stockholder  in  a  corporation  has  no  title  legal  or  
equitable   to   the   corporate   property;   that   both   of   these   are   in   the   corporation   itself   for   the   benefit   of   all   the  
stockholders.   So   it   is   clear   that   the   plaintiff,   by   reason   of   the   fact   that   he   is   a   stockholder   in   the   bank  
(corporation)  has  a  right  to  maintain  a  suit  for  and  on  behalf  of  the  bank,  but  the  extent  of  such  a  right  must  
depend  upon  when,  how,  and  for  what  purpose  he  acquired  the  shares  which  he  now  owns.  
  As  to  the  Second  cause  of  action:  It  affirmatively  appears  from  the  complaint  that  the  plaintiff  was  
not  a  stockholder  during  any  of  the  time  in  question  in  this  second  cause  of  action.  Upon  the  question  whether  
or  not  a  stockholder  can  maintain  a  suit  of  this  character  upon  a  cause  of  action  pertaining  to  the  corporation  
when  it  appears  that  he  was  not  a  stockholder  at  the  time  of  the  occurrence  of  the  acts  complained  of  and  
upon  which  the  action  is  based,  the  authorities  do  not  agree.  
 
 
 
125.  ADERITO  Z.  YUJUICO  AND  BONIFACIO  C.  SUMBILLA  VS.  CEZAR  T.  QUIAMBAO  AND  ERIC  C.  PILAPIL  
G.R.  NO.  180416.  JUNE  2,  2014  
PEREZ,  J.  
 
Doctrine:   Section   144   already   purports   to   penalize   "[v]iolations"   of   "any   provision"   of   the   Corporation  
Code  "not  otherwise  specifically  penalized  therein."  Hence,  we  find  inconsequential  the  fact  that  that  Section  
74  expressly  mentions  the  application  of  Section  144  only  to  a  specific  act,  but  not  with  respect  to  the  other  
possible  violations  of  the  former  section;  

   
3H  A.Y.  2017-­‐2018   165  
 
 

CORPORATION  LAW  CASE  DIGESTS  –  ATTY.  DANTE  DELA  CRUZ  

 
A  criminal  action  based  on  the  violation  of  the  stockholder’s  right  to  examine  or  inspect  the  corporate  records  
and  the  stock  and  transfer  book  of  a  corporation  under  the  second  and  fourth  paragraphs  of  Section  74  of  
the  Corporation  Code  can  only  be  maintained  against  corporate  officers  or  any  other  persons  acting  on  behalf  
of  such  corporation.  
 
Facts:   Strategic   Alliance   Development   Corporation   (STRADEC)   is   a   domestic   corporation   operating   as   a  
business  development  and  investment  company.  
 
On   March   2004,   during   the   annual   stockholder's   meeting   (SH’s   Meeting)   of   STRADEC,   petitioner   Aderito  
Z.  Yujuico  (Yujuico)  was  elected  as  president  and  chairman  of  the  company.  Yujuico  replaced  respondent  
Cezar  T.  Quiambao  (Quiambao),  who  had  been  the  president  and  chairman  of  STRADEC  since  1994.  
 
With  Yujuico  at  the  helm,  STRADEC  appointed  petitioner  Bonifacio  C.  Sumbilla  (Sumbilla)  as  treasurer  and  
one   Joselito   John   G.   Blando   (Blando)   as   corporate   secretary.   Blando   replaced   respondent   Eric   C.   Pilapil  
(Pilapil),  the  previous  corporate  secretary  of  STRADEC.  
 
Thereafter,   petitioners   filed   a   criminal   complaint   against   respondents   and   one   Giovanni   T.   Casanova  
(Casanova)  before  the  Office  of  the  City  Prosecutor  (OCP)  of  Pasig  City.  
 
The  complaint  accuses  respondents  and  Casanova  of  violating  Section  74  in  relation  to  Section  144  of  the  
Corporation  Code.  The  petitioners  premise  such  accusation  on  the  following  factual  allegations:  
1.   During   the   SH’s   Meeting,   Yujuico   —   as   newly   elected   president   and   chairman   of   STRADEC   —  
demanded   Quiambao   for   the   turnover   of   the   corporate   records   of   the   company,   particularly   the  
accounting  files,  ledgers,  journals  and  other  records  of  the  corporation's  business.  Quiambao  refused.    
2.   As   it   turns   out,   the   corporate   records   of   STRADEC   were   in   the   possession   of   Casanova   —   the  
accountant  of  STRADEC.  Casanova  was  keeping  custody  of  the  said  records  on  behalf  of  Quiambao,  
who  allegedly  needed  the  same  as  part  of  his  defense  in  a  pending  case  in  court.  
3.  After  the  SH’s  Meeting,  Quiambao  and  Casanova  caused  the  removal  of  the  corporate  records  of  
STRADEC  from  the  company's  offices  in  Pasig  City.  
4.  Upon  his  appointment  as  corporate  secretary  on  21  June  2004,  Blando  likewise  demanded  Pilapil  
for  the  turnover  of  the  stock  and  transfer  book  (SATB)  of  STRADEC.  Pilapil  refused.  
5.  Instead,  on  25  June  2004,  Pilapil  proposed  to  Blando  to  have  the  SATB  deposited  in  a  safety  deposit  
box   with   Equitable-­‐‑PCI   Bank,   Kamias   Road,   Quezon   City.   Blando   acceded   and   the   SATB   was  
deposited.   It   was   agreed   that   the   safety   deposit   box   may   only   be   opened   in   the   presence   of  
both  Quiambao  and  Blando.  
6.  On  30  June  2004,  however,  Quiambao  and  Pilapil  withdrew  the  SATB  from  the  safety  deposit  box  
and   brought   it   to   the   office   of   STRADCOM   in   Quezon   City.   Quiambao   thereafter   asked   Blando   to  
proceed  to  the  STRADCOM  office  in  QC.  Upon  arriving  thereat,  Quiambao  pressured  Blando  to  make  
certain   entries   in   the   SATB.   After   making   such   entries,   Blando   again   demanded   that   he   be   given  
possession  of  the  SATB.  Quiambao  refused.    
7.  On  1  July  2004,  Blando  received  an  order  issued  by  the  RTC-­‐‑Pasig  in  a  Civil  Case,  which  directed  
him  to  cancel  the  entries  he  made.  Hence,  on  even  date,  Blando  wrote  letters  to  Quiambao  and  Pilapil  
once   again   demanding   for   the   turnover   of   the   SATB.   Pilapil   replied   thru   a   letter   where   he   appeared  
to  agree  to  Blando's  demand.  
8.  However,  upon  meeting  with  Pilapil  and  Quiambao,  the  latter  still  refused  to  turnover  the  SATB  to  
Blando.  Instead,  Blando  was  once  again  constrained  to  agree  to  a  proposal  by  Pilapil  to  have  the  SATB  
deposited  with  the  RTC-­‐‑Pasig.  The  said  court,  however,  refused  to  accept  such  deposit  on  the  ground  
that  it  had  no  place  for  safekeeping.  

   
3H  A.Y.  2017-­‐2018   166  
 
 

CORPORATION  LAW  CASE  DIGESTS  –  ATTY.  DANTE  DELA  CRUZ  

9.  Since  Quiambao  and  Pilapil  still  refused  to  turnover  the  SATB,  Blando  again  acceded  to  have  the  
book  deposited  in  a  safety  deposit  box,  this  time,  with  the  Export  and  Industry  Bank  in  San  Miguel  
Avenue,  Pasig  City.  
 
Petitioners   theorize   that   the   refusal   by   the   respondents   and   Casanova   to   turnover   STRADEC's   corporate  
records  and  SATB  violates  their  right,  as  stockholders,  directors  and  officers  of  the  corporation,  to  inspect  
such  records  and  book  under  Section  74  of  the  Corporation  Code.  For  such  violation,  petitioners  conclude,  
respondents  may  be  held  criminally  liable  pursuant  to  Section  144  of  the  Corporation  Code.  
 
Two  informations  were  filed.  The  first  information  is  for  removing  the  SATB  from  its  principal  place  of  office.  
The  second  information  is  for  refusing  access  to,  and  examination  of,  the  corporate  records  and  the  stock  and  
transfer  book  of  STRADEC  at  its  principal  office.  
 
The  first  information  was  dismissed  as  it  was  later  found  by  the  Court  it  did  not  violate  the  Corporation  Code.  
The  second  information  pushed  through,  but  was  later  on  dismissed  because  the  RTC  opined  that  refusing  to  
allow   inspection   of   the   stock   and   transfer   book,   as   opposed   to   refusing   examination   of   other   corporate  
records,  is  not  punishable  as  an  offense  under  the  Corporation  Code.  
 
Issue:  Whether  or  not  Section  74,  by  expressly  mentioning  the  application  of  Section  144  only  in  relation  to  
the  act  of  "refus[ing]  to  allow  any  director,  trustees,  stockholder  or  member  of  the  corporation  to  examine  
and  copy  excerpts  from  [the  corporation's]  records  or  minutes,"  excludes  the  act  of  refusing  access  to  SATB?  
 
Held:  No.  However,  the  dismissal  is  still  proper.  
 
The  RTC  indeed  made  an  inaccurate  pronouncement  when  it  held  that  the  act  of  refusing  to  allow  inspection  
of  the  stock  and  transfer  book  of  a  corporation  is  not  a  punishable  offense  under  the  Corporation  Code.  Such  
refusal,   when   done   in   violation   of   Section   74   (4)   of   the   Corporation   Code,   properly   falls   within   the   purview  
of  Section  144  of  the  same  code  and  thus  may  be  penalized  as  an  offense.  
 
However,  a  criminal  action  based  on  the  violation  of  a  stockholder's  right  to  examine  or  inspect  the  corporate  
records  and  the  stock  and  transfer  book  of  a  corporation  under  the  second  and  fourth  paragraphs  of  Section  
74   —   second   information   —   can   only   be   maintained   against   corporate   officers   or   any   other   persons   acting  
on   behalf   of   such   corporation.   The   submissions   of   the   petitioners   during   the   preliminary   investigation,  
however,  clearly  suggest  that  respondents  are  neither  in  relation  to  STRADEC  because  they  are  no  longer  
corporate  officers.  
 
While   Section   74   expressly   mentions   the   application   of   Section   144   only   in   relation   to   the   act   of   "refus[ing]  
to  allow  any  director,  trustees,  stockholder  or  member  of  the  corporation  to  examine  and  copy  excerpts  from  
[the  corporation's]  records  or  minutes,"  the  same  does  not  mean  that  the  latter  section  no  longer  applies  to  
any  other  possible  violations  of  the  former  section.    
 
It  must  be  emphasized  that  Section  144  already  purports  to  penalize  "[v]iolations"  of  "any  provision"  of  the  
Corporation  Code  "not  otherwise  specifically  penalized  therein."  Hence,  we  find  inconsequential  the  fact  that  
that  Section  74  expressly  mentions  the  application  of  Section  144  only  to  a  specific  act,  but  not  with  respect  
to  the  other  possible  violations  of  the  former  section.  
Indeed,  we  find  no  cogent  reason  why  Section  144  cannot  be  made  to  apply  to  violations  of  the  right  of  a  
stockholder  to  inspect  the  SATB  of  a  corporation  under  Section  74  (4)  given  the  already  unequivocal  intent  
of  the  legislature  to  penalize  violations  of  a  parallel  right,  i.e.,  the  right  of  a  stockholder  or  member  to  examine  
the   other   records   and   minutes   of   a   corporation   under   Section   74   (2).   Certainly,   all   the   rights   guaranteed   to  
corporators  under  Section  74  are  mandatory  for  the  corporation  to  respect.  All  such  rights  are  just  the  same  

   
3H  A.Y.  2017-­‐2018   167  
 
 

CORPORATION  LAW  CASE  DIGESTS  –  ATTY.  DANTE  DELA  CRUZ  

underpinned  by  the  same  policy  consideration  of  keeping  public  confidence  in  the  corporate  vehicle  thru  an  
assurance  of  transparency  in  the  corporation's  operations.  
 
Verily,  we  find  inaccurate  the  pronouncement  of  the  RTC  that  the  act  of  refusing  to  allow  inspection  of  the  
stock  and  transfer  book  is  not  a  punishable  offense  under  the  Corporation  Code.  Such  refusal,  when  done  in  
violation   of   Section   74   (4),   properly   falls   within   the   purview   of   Section   144   of   the   same   code   and   thus   may  
be  penalized  as  an  offense.  
 
 
 
126.  JUAN  D.  EVANGELISTA  ET  AL.  VS.  RAFAEL  SANTOS  
G.R.  NO.  L-­‐‑1721.  MAY  19,  1950  
REYES,  J.  
 
DOCTRINE:  MISMANAGEMENT  BY  ITS  OFFICER;  RIGHT  OF  STOCKHOLDERS  TO  BEING  SUIT.  —  The  plaintiff  
stockholders  have  brought  the  action  not  for  the  benefit  of  the  corporation  but  for  their  own  benefit,  since  
they  ask  that  the  defendant  make  good  the  losses  occasioned  by  his  mismanagement  and  pay  to  them  the  
value  of  their  respective  participation  in  the  corporate  assets  on  the  basis  of  their  respective  holdings.  (dapat  
yung  prayer  nila  is  pay  the  value  to  the  corporation.,  not  to  them)  
 
While   plaintiffs   ask   for   a   remedy   to   which   they   are   not   entitled   unless   the   requirement   of   section   16   of   the  
Corporation  Law  be  first  complied  with,  we  note  that  the  action  stated  in  their  complaint  is  susceptible  of  
being  converted  into  a  derivative  suit  for  the  benefit  of  the  corporation  by  a  mere  change  in  the  prayer.    
 
FACTS:   This   is   an   action   by   the   minority   stockholders   of   a   corporation   against   its   principal   officer   for  
damages  resulting  from  his  mismanagement  of  its  affairs  and  misuse  of  its  assets.    
 
The   complaint   alleges   that   plaintiff's   are   minority   stockholders   of   the   Vitali   Lumber   Company,   Inc.,   a  
Philippine  corporation  organized  for  the  exploitation  of  a  lumber  concession  in  Zamboanga,  Philippines;  that  
defendant  holds  more  than  50  per  cent  of  the  stocks  of  said  corporation  and  also  is  and  always  has  been  the  
president,  manager,  and  treasurer  thereof;  and  that  defendant,  in  such  triple  capacity,  through  fault,  neglect,  
and   abandonment   allowed   its   lumber   concession   to   lapse   and   its   properties   and   assets,   among   them  
machineries,   buildings,   warehouses,   trucks,   etc.,   to   disappear,   thus   causing   the   complete   ruin   of   the  
corporation   and   total   depreciation   of   its   stocks.   The   complaint   therefore   prays   for   judgment   requiring  
defendant:   (1)   to   render   an   account   of   his   administration   of   the   corporate   affairs   and   assets:   (2)   to   pay  
plaintiffs  the  value  of  their  respective  participation  in  said  assets  on  the  basis  of  the  value  of  the  stocks  held  
by  each  of  them;  and  (3)  to  pay  the  costs  of  suit.  Plaintiffs  also  ask  for  such  other  remedy  as  may  be  just  and  
equitable.    
 
ISSUE:  Whether  or  not  the  present  suit  is  a  derivative  suit    
 
HELD:  NO.  The  complaint  shows  that  the  action  is  for  damages  resulting  from  mismanagement  of  the  affairs  
and  assets  of  the  corporation  by  its  principal  offcer,  it  being  alleged  that  defendant's  maladministration  has  
brought   about   the   ruin   of   the   corporation   and   the   consequent   loss   of   value   of   its   stocks.   The   injury  
complained  of  is  thus  primarily  to  the  corporation,  so  that  the  suit  for  the  damages  claimed  should  be  by  the  
corporation   rather   than   by   the   stockholders.   Stockholders   may   not   directly   claim   those   damages   for  
themselves   for   that   would   result   in   the   appropriation   by,   and   the   distribution   among   them   of   part   of   the  
corporate   assets   before   the   dissolution   of   the   corporation   and   the   liquidation   of   its   debts   and   liabilities,  
something  which  cannot  be  legally  done  in  view  of  section  16  of  the  Corporation  Law,    
 

   
3H  A.Y.  2017-­‐2018   168  
 
 

CORPORATION  LAW  CASE  DIGESTS  –  ATTY.  DANTE  DELA  CRUZ  

But  while  it  is  to  the  corporation  that  the  action  should  pertain  in  cases  of  this  nature,  however,  if  the  offcers  
of  the  corporation,  who  are  the  ones  called  upon  to  protect  their  rights,  refuse  to  sue,  or  where  a  demand  
upon  them  to  file  the  necessary  suit  would  be  futile  because  they  are  the  very  ones  to  be  sued  or  because  
they  hold  the  controlling  interest  in  the  corporation,  then  in  that  case  any  one  of  the  stockholders  is  allowed  
to  bring  suit.  But  in  that  case  it  is  the  corporation  itself  and  not  the  plaintiff  stockholder  that  is  the  real  party  
in  interest    
 
In  the  present  case,  the  plaintiff  stockholders  have  brought  the  action  not  for  the  benefit  of  the  corporation  
but   for   their   own   benefit,   since   they   ask   that   the   defendant   make   good   the   losses   occasioned   by   his  
mismanagement  and  pay  to  them  the  value  of  their  respective  participation  in  the  corporate  assets  on  the  
basis  of  their  respective  holdings.  Clearly,  this  cannot  be  done  until  all  corporate  debts,  if  there  be  any,  are  
paid  and  the  existence  of  the  corporation  terminated  by  the  limitation  of  its  charter  or  by  lawful  dissolution  
in  view  of  the  provisions  of  section  16  of  the  Corporation  Law.  It  results  that  plaintiffs'  complaint  shows  no  
cause  of  action  in  their  favor  so  that  the  lower  court  did  not  err  in  dismissing  the  complaint  on  that  ground.    
While   plaintiffs   ask   for   a   remedy   to   which   they   are   not   entitled   unless   the   requirement   of   section   16   of   the  
Corporation  Law  be  first  complied  with,  we  note  that  the  action  stated  in  their  complaint  is  susceptible  of  
being  converted  into  a  derivative  suit  for  the  benefit  of  the  corporation  by  a  mere  change  in  the  prayer.  Such  
amendment,  however,  is  not  possible  now,  since  the  complaint  has  been  filed  in  the  wrong  court,  so  that  the  
same  has  to  be  dismissed.      
 
(NOTE:  isa  pang  issue  sa  case  na  ito  is  wrong  venue,  finile  sa  rizal  eh  hindi  naman  yun  residence  ng  defendant,  
dapat  sa  residence  of  parties  kapag  personal  action  diba,  walang  jurisdiction  ang  court  dito,  kaya  hindi  rin  
niya  pwede  i-­‐‑amend  yung  complaint),    
 
 
 
127.  REPUBLIC  BANK  VS.  MIGUEL  CUADERNO  
G.R.  NO.  L-­‐‑22399.  MARCH  30,  1967  
REYES,  J.B.L.,  J  
 
DOCTRINES:    
•   Whenever  the  officials  of  the  corporation  refuse  to  sue,  or  are  the  ones  to  be  sued  or  hold  the  control  
of  the  corporation,  an  individual  stockholder  is  permitted  to  institute  a  derivative  or  representative  
suit  on  behalf  of  the  corporation  wherein  he  holds  stock,  in  order  to  protect  or  vindicate  corporate  
rights.  In  such  actions,  the  suing  stockholder  is  regarded  as  a  nominal  party,  with  the  corporation  as  
the  real  party  in  interest.    
•   In  corporate  derivative  suits  it  is  not  important  whether  the  corporation  is  made  party  plaintiff  or  
party   defendant   because   the   trial   court   has   the   power   to   direct   amendments   of   the   pleadings,   by  
adding   or   dropping   parties,   as   may   be   required   in   the   interest   of   justice.   It   is   enough   that   the  
corporation   is   made   a   party   to   the   suit   so   that   judgment   will   be   binding   upon   it   to   bar   future  
relitigation  of  issues.    
 
FACTS:    
Damaso  Perez,  a  stockholder  of  the  Republic  Bank,  a  Philippine  banking  corporation  domiciled  in  Manila,  
instituted  a  derivative  suit  for  and  in  behalf  of  said  Bank,  against  Miguel  Cuaderno,  Bienvenido  Dizon,  the  
Board  of  Directors  of  the  Republic  Bank,  and  the  Monetary  Board  of  the  Central  Bank  of  the  Philippines,  for  
the  reason  that  such  formal  demand  to  institute  the  present  complaint  would  be  a  futile  formality  since  the  
members  of  the  board  are  personally  chosen  by  defendant  Pablo  Roman  himself.  Damaso  alleged  that:    1.  
Roman  had  fraudulently  granted  or  caused  to  be  granted  loans  to  fictitious  and  non-­‐‑existing  persons  and  to  
their   close   friends,   relative   and/or   employees,   who   were   in   reality   their   dummies,   on   the   basis   of   fictitious  

   
3H  A.Y.  2017-­‐2018   169  
 
 

CORPORATION  LAW  CASE  DIGESTS  –  ATTY.  DANTE  DELA  CRUZ  

and  in flated  appraised  values  of  real  estate  properties;  2.  Acting  upon  the  complaint,  Miguel  Cuaderno  (then  
Governor  of  the  Central  Bank)  and  the  Monetary  Board  ordered  an  investigation;  3.  No  information  was  filed  
up  to  the  time  of  the  retirement  of  Cuaderno;  4.  To  neutralize  the  impending  action  against  him,  Pablo  Roman  
engaged  Miguel  Cuaderno  as  technical  consultant  at  a  compensation  of  P12,500.00  per  month,  even  if  court  
actions   involving   the   actuations   of   Cuaderno   as   Governor   and   Member   or   Chairman   of   the   Monetary   Board  
are  still  pending  in  court;  4.  Pablo  Roman  selecting  Bienvenido  Dizon  as  chairman  of  the  Board  of  Directors  
of   the   Republic   Bank   after   he   was   forced   to   resign   from   the   presidency   of   the   Philippine   National   Bank   and  
from   membership   of   the   Monetary   Board   and   within   one   year   thereafter   is   in   violation   of   section   3,  
subparagraph   (d)   of   the   Anti-­‐‑Graft   and   Corrupt   Practices   Act;   5.   The   Board   of   Directors   composed   of  
individuals   personally   selected   and   chosen   by   Roman,   connived   and   confederated   in   approving   the  
appointment  and  selection  of  Cuaderno  and  Dizon;  6.    Both  Cuaderno  and  Dizon  were  alter  egos  of  Pablo  
Roman;  and  7.  The  Monetary  Board  was  about  to  approve  the  appointment  of  Cuaderno  and  Dizon  and  would  
do  so  unless  enjoined.    
 
The  complaint,  therefore,  prayed  for  a  writ  of  preliminary  injunction  against  the  Monetary  Board  to  prevent  
its  confirmation  of  the  appointments  of  Dizon  and  Cuaderno;  against  the  Board  of  Directors  of  the  Republic  
Bank  from  recognizing  Cuaderno  as  technical  consultant  and  Dizon  as  Chairman  of  the  Board;  and  against  
Pablo  Roman  from  appointing  or  selecting  officers  or  directors  of  the  Republic  Bank.  The  courtdenied  the  
petition  for  a  writ  of  preliminary  injunction  and  dismissed  the  case.  The  court  in  effect  suggested  that  the  
matter  at  issue  in  the  case  may  be  presented  in  any  of  the  pending  eight  cases  (pending  in  different  branches  
between  practically  the  same  parties)  by  means  of  amended  and  supplemental  pleadings.  Plaintiff  Damaso  
Perez  thereupon  appealed  to  this  Court.    
 
ISSUES:  
1.   Whether  or  not  the  Court  below  erred  in  dismissing  the  complaint.    
2.   Whether  the  corporation  itself  must  be  made  party  defendant.    
 
HELD:  
1.   YES.  An  individual  stockholder  is  permitted  to  institute  a  derivative  or  representative  suit  on  behalf  
of  the  corporation  wherein  he  holds  stock,  in  order  to  protect  or  vindicate  corporate  rights,  whenever  
the  officials  of  the  corporation  refuse  to  sue,  or  are  the  ones  to  be  sued  or  hold  the  control  of  the  
corporation.   In   such   actions,   the   suing   stockholder   is   regarded   as   a   nominal   party,   with   the  
corporation   as   the   real   party   in   interest.   It   was   futile   to   demand   action   by   the   corporation,   since   its  
Directors  were  nominees  and  creatures  of  defendant  Pablo  Roman.  That  no  other  stockholder  has  
chosen   to   make   common   cause   with   plaintiff   Perez   is   irrelevant,   since   the   smallness   of   plaintiff's  
holdings  is  no  ground  for  denying  him  relief.  This  case  cannot  be  dismissed  simply  because  of  the  
possibility   that   the   cause   of   action   here   can   be   incorporated   or   introduced   in   any   of   those   other  
pending  cases.  The  case  is  remanded  to  the  court  of  origin  with  instructions  to  overrule  the  motions  
to  dismiss  and  require  the  defendants  to  answer  the  complaint.  Thereafter,  the  case  shall  be  tried  and  
decided  on  its  merits.    
2.   YES.  What  is  important  is  that  the  corporation  should  be  made  a  party,  in  order  to  make  the  Court's  
judgment  binding  upon  it,  and  thus  bar  future  re-­‐‑litigation  of  the  issues.  On  what  side  the  corporation  
appears  loses  importance  when  it  is  considered  that  it  lay  within  the  power  of  the  trial  court  to  direct  
the  making  of  such  amendments  of  the  pleadings,  by  adding  or  dropping  parties,  as  may  be  required  
in  the  interest  of  justice.  Misjoinder  of  parties  is  not  a  ground  to  dismiss  on  action.    
 
 
 
128.  CATALINA  REYES  v.  HON.  BIENVENIDO  TAN  
G.R.  No.  L-­‐‑16982                          September  30,  1961  

   
3H  A.Y.  2017-­‐2018   170  
 
 

CORPORATION  LAW  CASE  DIGESTS  –  ATTY.  DANTE  DELA  CRUZ  

LABRADOR,  J.;  
 
 
FACTS:  
-­‐‑   Roxas-­‐‑Kalaw  Textile  Mills,  Inc.  made  several  purchases  aggregating  $289,678.86  in  New  York  for  raw  
materials  for  the  textile  mill  and  shipped  to  the  Philippines,  which  shipment  were  found  out  to  consist  
not  of  raw  materials  but  already  finished  products.  
-­‐‑   The   Central   Bank   of   the   Philippines   stopped   all   dollar   allocations   for   raw   materials   for   the  
corporation  resulting  to  the  paralyzation  of  the  operation  of  the  textile  mill  and  its  business.  
-­‐‑   The  supplier  of  the  aforesaid  finished  goods  was  the  United  Commercial  Company  of  New  York  in  
which  defendant  Dalamal  had  interests,  so  as  the  letter  of  credit  for  said  goods.  
-­‐‑   Plaintiff  and  some  members  of  the  board  of  directors  urged  defendants  to  proceed  against  Dalamal,  
exposing  his  offense  to  the  Central  Bank,  and  to  initiate  suit  against  Dalamal  for  his  fraud  against  the  
corporation.  
-­‐‑   Defendants,   petitioner   in   this   certiorari   case,   refused   to   proceed   against   Dalamal   and   instead  
continued   to   deal   with   the   Indian   Commercial   Company   to   the   damage   and   prejudice   of   the  
corporation.    
-­‐‑   The  prayer  asks  for  the  appointment  of  a  receiver  and  a  judgment  marking  defendants  jointly  and  
severally  liable  for  the  damages.  
 
ISSUE:  Whether  the  stockholders  are  justified  in  constituting  the  derivative  suit.  
 
HELD:  
•   It  is  also  not  denied  by  petitioner  that  the  allocation  of  dollars  to  the  corporation  for  the  importation  
of  raw  materials  was  suspended.    
•   The  importation  of  textiles  instead  of  raw  materials,  as  well  as  the  failure  of  the  Board  of  Directors  to  
take  action  against  those  directly  responsible  for  the  misuse  of  dollar  allocations  constitute  fraud,  or  
consent  thereto  on  the  part  of  the  directors.    
•   Therefore,   a   breach   of   trust   was   committed   which   justified   the   derivative   suit   by   a   minority  
stockholder  on  behalf  of  the  corporation.  
•   It   is   well   settled   in   this   jurisdiction   that   where   corporate   directors   are   guilty   of   a   breach   of   trust   —  
not   of   mere   error   of   judgment   or   abuse   of   discretion   —   and   intra-­‐‑corporate   remedy   is   futile   or  
useless,   a   stockholder   may   institute   a   suit   in   behalf   of   himself   and   other   stockholders   and   for   the  
benefit   of   the   corporation,   to   bring   about   a   redress   of   the   wrong   inflicted   directly   upon   the  
corporation  and  indirectly  upon  the  stockholders.  
 
 
 
129.   ELTON   W.   CHASE,   AS   MINORITY   STOCKHOLDER   AND   ON   BEHALF   OF   THE   STOCKHOLDERS  
SIMILARLY   SITUATED   AND   FOR   THE   BENEFIT   OF   AMERICAN   MACHINERY   AND   PARTS  
MANUFACTURING,   INC   VS.THE   COURT   OF   FIRST   INSTANCE   OF   MANILA,   BRANCH   XIV,   DR.   VICTOR  
BUENCAMINO,  SR.,  VICTOR  BUENCAMINO,  JR.,  DOLORES  A.  BUENCAMINO  AND  JULIO  B.  FRANCIA,  JR  
G.R.  NO.  L-­‐‑20457                        OCTOBER  29,  1966  
DIZON,  J.;  
 
DOCTRINE:  The  appointment  of  a  receiver  is  a  matter  addressed  to  the  sound  discretion  of  the  court,  and  it  
has  been  frequently  held  that  such  discretion  to  appoint  a  receiver  who  would  take  over  the  administration  
of  the  corporate  business  should  be  exercised  with  great  caution  and  only  when  the  necessity  therefor  is  
clear.  
 
   
3H  A.Y.  2017-­‐2018   171  
 
 

CORPORATION  LAW  CASE  DIGESTS  –  ATTY.  DANTE  DELA  CRUZ  

FACTS:  
Petitioner,   a   minority   stockholder   of   American   Machinery   &   Parts   Manufacturing,   Inc   (AMPARTS),   filed   a  
derivative   suit   against   herein   private   respondents,   majority   stockholders   and   corporate   directors   of  
AMPARTS   charging   them   with   breach   of   trust.   Attached   to   the   complaint   was   an   application   for   the  
appointment   of   a   receiver   of   AMPARTS.   Respondents   opposed   the   application   for   receivership.   The   trial  
court,   then   presided   by   the   Hon.   Magno   S.   Gatmaitan,   issued   an   order   denying   the   same,   but   requiring  
respondents  to  file  bond  in  the  amount  of  P100,000.00  to  answer  for  whatever  damages  petitioner  might  
suffer  by  reason  of  the  denial.  After  trial  on  the  merits,  the  court  rendered  judgment  finding  Dr.  Buencamino  
guilty  of  mismanagement  and  condemning  him  "to  pay  Amparts  the  sum  of  P1,970,200  with  legal  interest.  
Petitioner  filed  a  motion  for  the  appointment  of  a  receiver  until  the  full  amount  of  the  above  judgment  against  
respondent   Buencamino   is   fully   satisfied   or   until   the   dissolution   or   liquidation   of   said   corporation.   The  
respondent   court,   now   presided   by   the   Hon.   Jesus   De   Veyra,   issued   the   following   order   of:   ‘As   for   the  
appointment  of  a  receiver,  Judge  Gatmaitan  decided  on  the  temporary  measure  of  giving  plaintiff  (petitioner  
herein)   a   veto   right,   appealable   to   this   Court,   on   all   decisions   of   management.   Considering   that   up   to   the  
present,   the   Buencaminos   own   2/3   of   the   stock   of   the   corporation,   the   solution   is   equitable   and   must   be  
allowed  to  continue  subject  to  the  condition  that  once  a  decision  of  management  is  made  known  to  plaintiff,  
he  must  make  known  his  objection  thereto  to  the  Court  within  five  (5)  days  from  receipt  of  said  decision,  
otherwise  he  shall  be  deemed  to  have  waived  any  objection  to  the  decision.  
 
ISSUE:  Whether  or  not  the  respondent  court  committed  a  grave  abuse  of  discretion  in  issuing  the  said  orders.  
 
HELD:  
The  appointment  of  a  receiver  is  a  matter  addressed  to  the  sound  discretion  of  the  court,  and  it  has  been  
frequently   held   that   such   discretion   to   appoint   a   receiver   who   would   take   over   the   administration   of   the  
corporate  business  should  be  exercised  with  great  caution  and  only  when  the  necessity  therefor  is  clear.  The  
facts   of   the   present   case   show   that,   in   connection   with   the   order   which   denied   petitioner's   first   application  
for  the  appointment  of  a  receiver,  the  court  required  respondents  herein  to  file  a  bond.  Again,  perhaps  by  
reason   of   the   judgment   rendered   against   Dr.   Buencamino   finding   him   guilty   of   mismanagement   etc.,   the  
respondent  court,  through  the  Hon.  Jesus  de  Veyra,  issued  the  aforementioned  order.  Upon  the  facts  of  the  
case,   and   considering   the   precautionary   measures   adopted   by   the   respondent   court   for   the   protection   of  
petitioner's  rights  and  interest  in  AMPARTS,  it  cannot  be  said  that  the  court  had  committed  a  grave  abuse  of  
discretion  in  issuing  the  orders  complained  of.  
 
 
 
130.  RICARDO  L.  GAMBOA  VS.  HON.  OSCAR  R.  VICTORIANO    
G.R.  NO.  L-­‐‑40620.  MAY  5,  1979  
J.  CONCEPCION  JR.      
 
DOCTRINE:  An  individual  stockholder  is  permitted  to  institute  a  derivative  suit  on  behalf  of  the  corporation  
wherein   he   holds   stock   in   order   to   protect   and   vindicate   corporate   rights,   whenever   the   officials   of   the  
corporation  refuses  to  sue,  or  are  the  ones  to  be  used  or  hold  the  control  of  the  corporation.  In  such  actions,  
the  suing  stockholders  is  regarded  as  a  nominal  party,  with  the  corporation  as  the  real  party  in  interest.  Thus,  
a  derivative  suit  will  not  lie  where  stockholders  are  vindicating  their  own  individual  interest  or  prejudice,  
and  not  that  of  the  corporation.  
 
FACTS:   Plaintiffs,   with   the   exception   of   Anastacio   Dacles,   who   was   joined   as   a   formal   party,   are   the   owners  
of  1,328  shares  of  stock  of  the  Inocentes  de  la  Rama,  Inc.,  a  domestic  corporation,  with  an  authorized  capital  
stock  of  3,000  shares,  with  a  par  value  of  P100.00  per  share,  2,177  of  which  were  subscribed  and  issued,  thus  
leaving  823  shares  unissued;  that  upon  the  plaintiffs'  acquisition  of  the  shares  of  stock  held  by  

   
3H  A.Y.  2017-­‐2018   172  
 
 

CORPORATION  LAW  CASE  DIGESTS  –  ATTY.  DANTE  DELA  CRUZ  

Rafael  Ledesma  and  Jose  Sicangco,  Jr.,  then  President  and  Vice-­‐‑President  of  the  corporation,  respectively,  the  
defendants  Mercedes  R.  Borromeo,  Honorio  de  la  Rama,  and  Ricardo  Gamboa,  remaining  members  of  the  
board   of   directors   of   the   corporation,   in   order   to   forestall   the   takeover   by   the   plaintiffs   of   the   afore-­‐‑named  
corporation,  surreptitiously  met  and  elected  Ricardo  L.  Gamboa  and  Honorio  de  la  Rama  as  president  and  
vicepresident  of  the  corporation,  respectively,  and  thereafter  passed  a  resolution  authorizing  
the  sale  of  the  823  unissued  shares  of  the  corporation  to  the  defendants,  Ricardo  L.  Gamboa,  Lydia  R.  Gamboa,  
Honorio  de  la  Rama,  Ramon  de  la  Rama,  Paz  R.  Battistuzzi,  Eduardo  de  la  Rama,  and  Mercedes  R.  Borromeo,  
at  par  value,  after  which  the  defendants  Honorio  de  la  Rama,  Lydia  de  la  Rama-­‐‑Gamboa,  and  Enzo  Battistuzzi  
were  elected  to  the  board  of  directors  of  the  corporation;  that  the  sale  of  the  unissued  823  shares  of  stock  of  
the   corporation   was   in   violation   of   the   plaintiffs'   and   pre-­‐‑emptive   rights   and   made   without   the   approval   of  
the  board  of  directors  representing  2/3  of  the  outstanding  capital  stock,  and  is  in  disregard  of  the  strictest  
relation  of  trust  existing  between  the  defendants,  as  stockholders  thereof;  and  that  the  defendants  Lydia  de  
la  Rama-­‐‑Gamboa,  Honorio  de  la  Rama,  and  Enzo  Battistuzzi  were  not  legally  elected  to  the  board  of  directors  
of   the   said   corporation   and   has   unlawfully   usurped   or   intruded   into   said   office   to   the   prejudice   of   the  
plaintiffs.  
 
Acting   upon   the   complaint,   the   respondent   judge,   after   proper   hearing,   directed   the   clerk   of   court   "to   issue  
the  corresponding  writ  of  preliminary  injunction  restraining  the  defendants  and/or  their  representatives,  
agents,   or   persons   acting   in   their   behalf   from   the   commission   or   continuance   of   any   act   tending   in   any   way  
to   prejudice,   diminish   or   otherwise   injure   plaintiffs'   rights   in   the   corporate   properties   and   funds   of   the  
corporation  'Inocentes  de  la  Rama,  Inc.'  and  from  disposing,  transferring,  selling  or  otherwise  impairing  the  
value  of  the  certificates  of  stock  allegedly  issued  illegally  in  their  names  on  February  11,  1972,  or  at  any  date  
thereafter,  and  ordering  them  to  deposit  with  the  Clerk  of  Court  the  corresponding  certificates  of  stock  for  
the  823  shares  issued  to  said  defendants  on  February  11,  1972.    
 
On  October  31,  1972,  the  plaintiffs  therein,  now  private  respondents,  entered  into  a  compromise  agreement  
with   the   defendants   Ramon   de   la   Rama,   Paz   de   la   Rama-­‐‑Battistuzzi,   and   Enzo   Battistuzzi,   whereby   the  
contracting   parties   withdrew   their   respective   claims   against   each   other   and   the   aforenamed   defendants  
waived   and   transferred   their   rights   and   interests   over   the   questioned   823   shares   of   stock   in   favor   of   the  
plaintiffs.  The  compromise  agreement  was  approved  by  the  trial  court  on  December  4,  1972.  As  a  result,  the  
defendants  filed  a  motion  to  dismiss  the  complaint,  on  November  19,  1974  which  was  denied  on  January  2,  
1975.  The  defendants  also  filed  a  motion  to  declare  the  defendants  Ramon  L.  de  la  Rama,  Paz  de  la  Rama-­‐‑
Battistuzzi,  and  Enzo  Battistuzzi  in  contempt  of  court,  for  having  violated  the  writ  of  preliminary  injunction  
when  they  entered  into  the  aforesaid  compromise  agreement  with  the  plaintiffs,  but  the  respondent  judge  
denied  the  said  motion  for  lack  of  merit.    
   
ISSUE:  Does  the  court  have  jurisdiction  to  interfere  with  the  management  of  the  corporation  by  the  board  of  
directors,   and   the   enactment   of   a   resolution   by   the   defendants,   as   members   of   the   board   of   directors   of   the  
corporation,  allowing  the  sale  of  the  823  shares  of  stock?    
 
HELD:    No.  The  petition  is  without  merit.  The  questioned  order  denying  the  petitioners'  motion  to  dismiss  
the   complaint   is   merely   interlocutory   and   cannot   be   the   subject   of   a   petition   for   certiorari.   The   proper  
procedure   to   be   followed   in   such   a   case   is   to   continue   with   the   trial   of   the   case   on   the   merits   and,   if   the  
decision  is  adverse,  to  reiterate  the  issue  on  appeal.  It  would  be  a  breach  of  orderly  procedure  to  allow  a  
party  to  come  before  this  Court  every  time  an  order  is  issued  with  which  he  does  not  agree.  
 
The  well-­‐‑known  rule  is  that  courts  cannot  undertake  to  control  the  discretion  of  the  board  of  directors  about  
administrative  matters  as  to  which  they  have  legitimate  power  of  action,  and  contracts  intra  vires  entered  
into  by  the  board  of  directors  are  binding  upon  the  corporation  and  courts  will  not  interfere  unless  such  
contracts  are  so  unconscionable  and  oppressive  as  to  amount  to  a  wanton  destruction  of  the  rights  of  the  

   
3H  A.Y.  2017-­‐2018   173  
 
 

CORPORATION  LAW  CASE  DIGESTS  –  ATTY.  DANTE  DELA  CRUZ  

minority.   In   the   instant   case,   the   plaintiffs   aver   that   the   defendants   have   concluded   a   transaction   among  
themselves   as   will   result   to   serious   injury   to   the   interests   of   the   plaintiffs,   so   that   the   trial   court   has  
jurisdiction  over  the  case.  
 
The  petitioners  further  contend  that  the  proper  remedy  of  the  plaintiffs  would  be  to  institute  a  derivative  
suit  against  the  petitioners  in  the  name  of  the  corporation  in  order  to  secure  a  binding  relief  after  exhausting  
all  the  possible  remedies  available  within  the  corporation.  An  individual  stockholder  is  permitted  to  institute  
a   derivative   suit   on   behalf   of   the   corporation   wherein   he   holds   stock   in   order   to   protect   or   vindicate  
corporate   rights,   whenever   the   officials   of   the   corporation   refuse   to   sue,   or   are   the   ones   to   be   sued   or   hold  
the  control  of  the  corporation.  In  such  actions,  the  suing  stockholder  is  regarded  as  a  nominal  party,  with  the  
corporation  as  the  real  party  in  interest.  In  the  case  at  bar,  however,  the  plaintiffs  are  alleging  and  vindicating  
their  own  individual  interests  or  prejudice,  and  not  that  of  the  corporation.  At  any  rate,  it  is  yet  too  early  in  
the  proceedings  since  the  issues  have  not  been  joined  
 
 
 
131.    SAN  MIGUEL  V.  KHAN  
 
 
 
132.  ALFREDO  VILLAMOR,  JR.    V.  JOHN  S.  UMALE  
GR  NO.  172843  24  SEPTEMBER  2014  
JUSTICE  LEONEN  
 
DOCTRINE:  
  The  5  requisites  for  a  derivative  suit  to  prosper  must  be  complied  with  and  alleged  by  the  stockholder  
or  member  instituting  the  action.  Failure  to  do  so  will  result  to  the  filing  of  merely  an  individual  suit.  
 
FACTS:  
  MC   Home   Depot   occupied   a   Rockland   area   property   in   Pasig   City   which   was   part   of   the   area   owned  
by  Mid-­‐‑Pasig  Development  Corporation  (Mid-­‐‑Pasig).  On  01  March  2004,  the  Pasig  Printing  Corporation  (PPC)  
obtained  an  option  to  lease  portions  of  Mid-­‐‑Pasig’s  property,  including  the  subject  property.  In  November  
2004,  PPC’s  Board  of  Directors  issued  a  Resolution  waiving  all  its  rights,  interests  and  participation  in  the  
option  to  lease  contract  in  favor  of  the  law  firm  of  Atty.  Alfredo  Villamor,  Jr.  without  any  consideration.    
 
  PPC,   represented   by   Villamor,   entered   into   a   Memorandum   of   Agreement   with   MC   Home   Depot  
allowing  the  latter  to  continue  occupying  the  area  as  PPC’s  sublessee  for  4  years,  renewable  every  4  years  at  
a  monthly  rental  of  P4,500,000.00  plus  goodwill  of  P18,000,000.00.  Thus,  MC  Home  Depot  issued  20  post  
dated  checks  for  the  rental  payments  for  1  year  and  the  goodwill  money  and  giving  them  to  Villamor  who  in  
turn  did  not  turn  over  these  amounts  to  PPC.  
 
  Hernando  Balmores,  a  stockholder  and  director  of  PPC,  wrote  a  letter  to  PPC’s  directors  informing  
them  that  Villamor  should  deliver  and  account  for  MC  Home  Depot’s  payments  to  PPC.  However,  PPC  BOD  
failed  to  act  on  Balmores’  letter  thus  prompting  the  latter  to  file  an  intra-­‐‑corporate  controversy  with  the  RTC,  
alleging   therein   that   PPC’s   assets   were   in   imminent   danger   and   have   actually   been   lost,   wasted   and  
destroyed.  He  futher  prayed  for  the  annulment  of  the  Board  Resolution  waiving  PPC’s  rights  to  Villamor’s  
law  firm.    
 
  The  trial  court  denied  Balmores’  petition  thus  prompting  the  latter  to  appeal  to  the  Court  of  Appeals.  
The  appellate  court  reversed  the  trial  court’s  decision  and  granted  Balmores’  appeal.  

   
3H  A.Y.  2017-­‐2018   174  
 
 

CORPORATION  LAW  CASE  DIGESTS  –  ATTY.  DANTE  DELA  CRUZ  

 
ISSUE:  
  Whether  or  not  the  Court  of  Appeal  correctly  characterized  Balmores’  action  as  a  derivative  suit  
 
RULING:  
  PPC’s  directors  argued  that  the  CA  erred  in  characterizing  Balmores’  suit  as  a  deriviative  suit  because  
of  his  failure  to  implead  PPC  as  party  in  the  case.  Thus,  the  appellate  court  did  not  acquire  jurisdiction  over  
the  corporation.  
 
  No,  Balmores’  actions  cannot  be  characterized  as  a  derivative  suit.  A  derivative  suit  is  an  action  filed  
by   stockholder   to   enforce   a   corporate   action.   It   is   an   exception   to   the   general   rule   that   the   corporation’s  
power  to  sue  is  exercised  only  by  the  BOD/BOT.  
 
  Individual   stockholders   may   be   allowed   to   sue   on   behalf   of   the   corporation   whenever   the   directors  
or  officers  of  the  corporation  refuse  to  sue  to  vindicate  the  rights  of  the  corporation  or  are  the  ones  to  be  
sued  and  are  in  control  of  the  corporation.    
 
  Rule  8,  Section  1  of  the  Interim  Rules  of  Procedure  for  Intra-­‐‑corporate  Controversies  provides  the  5  
requisites  for  filing  derivative  suits:  
SECTION1.   Derivative   Action   –   A   stockholder   or   member   may   bring   an   action   in   the   name   of   the  
corporation  or  association,  as  the  case  may  be,  provided,  that:  
1.   He   was   a   stockholder   or   member   at   the   time   the   acts   or   transactions   subject   of   the   action  
occurred  and  at  the  time  the  action  was  filed;  
2.   He   exerted   all   reasonable   efforts,   and   alleges   the   same   with   particularity   in   the   complaint,   to  
exhaust  all  remedies  available  under  the  AOI,  by-­‐‑laws,  laws  or  rules  governing  the  corporation  
or  partnership  to  obtain  the  relief  he  desires;  
3.   No  appraisal  rights  are  available  for  the  act  or  acts  complained  of;  and    
4.   The  suit  is  not  a  nuisance  or  harassment  suit.  
 
The  fifth  requisite  for  filing  derivative  suits,  while  not  included  in  the  enumeration,  is  implied  in  the  
first  paragraph,  which  states  that  “the  action  must  be  in  the  name  of  the  corporation  or  association.”  
 
  It  has  been  held  that  among  the  basic  requirements  for  a  derivative  suit  to  propser  is  that  the  minority  
shareholder  who  is  suing  for  and  on  behalf  of  the  corporation  must  allege  in  his  complaint  that  he  is  suing  
for  and  on  behalf  of  the  corporation.  It  is  thus  important  that  the  corporation  be  made  a  party  to  the  case.  
 
  In  the  case  at  bar,  Respondent  Balmores  failed  to  exhaust  all  remedies  to  obtain  the  reliefs  he  prayed  
for.  He  also  failed  to  allege  that  appraisal  rights  were  not  available  for  the  acts  complained  of,  as  another  
requisite.  Further,  he  also  failed  to  implead  PPC  as  party  in  the  case  nor  did  he  allege  that  he  was  filing  on  
behalf  of  the  corporation.    
 
  Thus,  in  this  case,  Respondent  Balmores  filed  an  individual  suit.    
 
 
 
 
133.  JUANITO  ANG  FOR  AND  IN  BEHALF  OF  SUNRISE  MARKETING  (BACOLOD),  INC.    VS.  SPS.  ROBERTO  
AND  RACHEL  ANG  
G.R.  NO.  201675,  JUNE  19,  2013  
CARPIO,  J.  

   
3H  A.Y.  2017-­‐2018   175  
 
 

CORPORATION  LAW  CASE  DIGESTS  –  ATTY.  DANTE  DELA  CRUZ  

 
DOCTRINE:  The  directors  or  officers,  as  provided  under  the  by-­‐‑laws,  have  the  right  to  decide  whether  or  not  
a  corporation  should  sue.  Since  these  directors  or  officers  will  never  be  willing  to  sue  themselves,  or  impugn  
their   wrongful   or   fraudulent   decisions,   stockholders   are   permitted   by   law   to   bring   an   action   in   the   name   of  
the  corporation  to  hold  these  directors  and  officers  accountable.  
 
FACTS:   Sunrise   Marketing   (Bacolod),   Inc.   (SMBI)   is   a   duly   registered   corporation   owned   by   the   Ang   family.  
Juanito  Ang    (Juanito)  and  Ang  (Roberto)  are  siblings.    Roberto  was  elected  President  of  SMBI,  while  Juanito  
was  elected  as  its  Vice  President.    
On   31   July   1995,   Nancy   Ang   (Nancy),   the   sister   of   Juanito   and   Roberto,   and   her   husband,   Theodore   Ang  
(Theodore),  agreed  to  extend  a  loan  to  settle  the  obligations  of  SMBI  and  other  corporations  owned  by  the  
Ang   family.   Nancy   and   Theodore   issued   a   check   in   the   amount   of   $1,000,000.00   payable   to   "Juanito   Ang  
and/or   Anecita   Ang   (the   wife)   and/or   Roberto   Ang   and/or   Rachel   Ang   (the   wife)."   Nancy   was   a   former  
stockholder  of  SMBI,  but  she  no  longer  appears  in  SMBI’s  General  Information  Sheets  as  early  as  1996.  Nancy  
and  Theodore  are  now  currently  residing  in  the  United  States.  There  was  no  written  loan  agreement,  in  view  
of  the  close  relationship  between  the  parties.  
On  24  November  2008,  Nancy  and  Theodore,  sent  a  demand  letter  to  "Spouses  Juanito  L.  Ang/Anecita  L.  Ang  
and   Spouses   Roberto   L.   Ang/Rachel   L.   Ang"   for   payment   of   the   loan   obligation.   Roberto   and   Rachel   refused  
to  pay  and  denied  having  personally  contracted  a  loan  from  Nancy  and  Theodore.  
On   8   January   2009,   Juanito   and   Anecita   executed   a   Deed   of   Acknowledgment   and   Settlement   Agreement  
(Settlement   Agreement)   and   an   Extra-­‐‑Judicial   Real   Estate   Mortgage   (Mortgage).   Under   the   foregoing  
instruments,  Juanito  and  Anecita  admitted  that  they,  together  with  Roberto  and  Rachel,  obtained  a  loan  from  
Nancy  and  Theodore  for  $1,000,000.00  on  31  July  1995.  
Thereafter,   Juanito   filed   a   "Stockholder   Derivative   Suit   with   prayer   for   an   ex-­‐‑parte   Writ   of  
Attachment/Receivership"   before   the   RTC   Bacolod   alleging   that   "the   intentional   and   malicious   refusal   of  
defendant   Sps.   Roberto   and   Rachel   Ang   to   settle   their   50%   share   x   x   x   of   the   total   obligation   x   x   x   will  
definitely  affect  the  financial  viability  of  plaintiff  SMBI."  Rachel  countered,  claiming  that  it  was  not  a  bona  
fide  derivative  suit  but  is  actually  a  collection  suit  since  the  real  party  in  interest  is  not  SMBI,  but  Nancy  and  
Theodore  
 
RTC  Bacolod  decided  in  favor  of  Juanito  Ang  and  ruled  that  the  complaint  was  a  derivative  suit.  But  the  CA  
reversed  the  decision.  
 
ISSUE:  Whether  the  suit  between  the  debtors  and  creditors  is  a  derivative  suit.  NO.  
HELD:  The  Complaint  is  not  a  derivative  suit.  A  derivative  suit  is  an  action  brought  by  a  stockholder  on  behalf  
of  the  corporation  to  enforce  corporate  rights  against  the  corporation’s  directors,  officers  or  other  insiders.  
 
The  Complaint  failed  to  show  how  the  acts  of  Rachel  and  Roberto  resulted  in  any  detriment  to  SMBI.  The  CA  
correctly  concluded  that  the  loan  was  not  a  corporate  obligation,  but  a  personal  debt  of  the  Ang  brothers  and  
their  spouses.  The  check  was  issued  to  "Juanito  Ang  and/or  Anecita  Ang  and/or  Roberto  Ang  and/or  Rachel  
Ang"   and   not   SMBI.   The   proceeds   of   the   loan   were   used   for   payment   of   the   obligations   of   the   other  
corporations  owned  by  the  Angs  as  well  as  the  purchase  of  real  properties  for  the  Ang  brothers.  SMBI  was  
never  a  party  to  the  Settlement  Agreement  or  the  Mortgage.  It  was  never  named  as  a  co-­‐‑debtor  or  guarantor  
of  the  loan.  Both  instruments  were  executed  by  Juanito  and  Anecita  in  their  personal  capacity,  and  not  in  
their  capacity  as  directors  or  officers  of  SMBI.  Thus,  SMBI  is  under  no  legal  obligation  to  satisfy  the  obligation.  
 
The  fact  that  Juanito  and  Anecita  attempted  to  constitute  a  mortgage  over  "their"  share  in  a  corporate  asset  
cannot  affect  SMBI.Juanito  and  Anecita,  as  stockholders  of  SMBI,  are  not  co-­‐‑owners  of  SMBI  assets.  They  do  
not   own   pro-­‐‑indiviso   shares,   and   therefore,   cannot   mortgage   the   same   except   in   their   capacity   as   directors  
or  officers  of  SMBI.  

   
3H  A.Y.  2017-­‐2018   176  
 
 

CORPORATION  LAW  CASE  DIGESTS  –  ATTY.  DANTE  DELA  CRUZ  

 
 
134.  TIRSO  GARCIA  VS.  LIM  CHU  SING  
G.R.  NO.  L-­‐‑39427;  FEBRUARY  24,  1934  
VILLA-­‐‑REAL,  J.  
 
Doctrine:   A   share   of   stock   or   the   certificate   thereof   is   not   an   indebtedness   to   the   owner   nor   evidence   of  
indebtedness  and,  therefore,  it  is  not  a  credit.  Stockholders,  as  such,  are  not  creditors  of  the  corporation.  
 
FACTS:  
 
Defendant-­‐‑appellant  Lim  Chu  Sing  executed  and  delivered  to  the  Mercantile  Bank  of  China  promissory  note.  
The  defendant  had  been,  making  several  partial  payments  thereon,  leaving  an  unpaid  balance  of  P9,105.17.  
However,   he   defaulted   in   the   payment   of   several   installments   by   reason   of   which   the   unpaid   balance   of  
P9,105.17  on  the  promissory  note  has  ipso  facto  become  due  and  demandable.  The  debt  which  is  the  subject  
matter  of  the  complaint  was  not  really  an  indebtedness  of  the  defendant  but  of  Lim  Cuan  Sy,  who  had  an  
account  with  the  plaintiff  bank  in  the  form  of  "trust  receipts".  The  defendant  is  the  owner  of  shares  of  stock  
of  the  plaintiff  Mercantile  Bank  of  China  amounting  to  P10,000.  The  plaintiff  bank  is  now  under  liquidation.  
Sing  filed  a  motion  praying  for  the  inclusion  of  the  principal  debtor  Sy  as  party  defendant  so  that  he  could  
avail  himself  of  the  benefit  of  the  exhaustion  of  the  property  of  Sy.  The  proceeds  of  the  sale  of  the  mortgaged  
chattels  together  with  other  payments  made  were  applied  to  the  amount  of  the  promissory  note  in  question,  
leaving  the  balance  which  the  plaintiff  now  seeks  to  collect.  
 
ISSUE:  
Whether  or  not  defendant-­‐‑appellant's  indebtedness  may  be  compensated  with  the  value  representing  the  
value  of  his  shares  of  stock.  
 
RULING:  
 
A  share  of  stock  or  the  certificate  thereof  is  not  an  indebtedness  to  the  owner  nor  evidence  of  indebtedness  
and,  therefore,  it  is  not  a  credit.  Stockholders,  as  such,  are  not  creditors  of  the  corporation.  It  is  the  prevailing  
doctrine   of   the   American   courts,   repeatedly   asserted   in   the   broadest   terms,   that   the   capital   stock   of   a  
corporation  is  a  trust  fund  to  be  used  more  particularly  for  the  security  of  creditors  of  the  corporation,  who  
presumably  deal  with  it  on  the  credit  of  its  capital  stock.  Therefore,  the  defendant-­‐‑appellant  Lim  Chu  Sing  
not   being   a   creditor   of   the   Mercantile   Bank   of   China,   although   the   latter   is   a   creditor   of   the   former,   there   is  
no  sufficient  ground  to  justify  a  compensation.  
The   SC   holds   that   the   shares   of   a   banking   corporation   do   not   constitute   an   indebtedness   of   the   corporation  
to  the  stockholder  and,  therefore,  the  latter  is  not  a  creditor  of  the  former  for  such  shares.  
 
 
 
135.  NO  CASE  
 
 
 
136.  DATU  TAGORANAO  BENITO,  VS.  SECURITIES  AND  EXCHANGE  COMMISSION    
G.R.  NO.  L-­‐‑56655  -­‐‑  JULY  25,  1983  
RELOVA,  J;  
 
 

   
3H  A.Y.  2017-­‐2018   177  
 
 

CORPORATION  LAW  CASE  DIGESTS  –  ATTY.  DANTE  DELA  CRUZ  

DOCTRINE:  Pre-­‐‑emptive  right  is  recognized  only  with  respect  to  new  issue  of  shares,  and  not  with  respect  to  
additional  issues  of  originally  authorized  shares.  This  is  on  the  theory  that  when  a  corporation  at  its  inception  
offers  its  first  shares,  it  is  presumed  to  have  offered  all  of  those  which  it  is  authorized  to  issue.  
 
 
FACTS:    
On   February   6,   1959,   the   Articles   of   Incorporation   of   respondent   Jamiatul   Philippine-­‐‑Al   Islamia,   Inc.  
(originally  Kamilol  Islam  Institute,  Inc.)  were  filed  with  the  Securities  and  Exchange  Commission  (SEC)  and  
were   approved   on   December   14,   1962.   The   corporation   had   an   authorized   capital   stock   of   P200,000.00  
divided  into  20,000  shares  at  a  par  value  of  P10.00  each.  Of  the  authorized  capital  stock,  8,058  shares  worth  
P80,580.00   were   subscribed   and   fully   paid   for.   Herein   petitioner   Datu   Tagoranao   Benito   subscribed   to   460  
shares  worth  P4,600.00.    
 
In  1975,  the  respondent  corporation  filed  a  certificate  of  increase  of  its  capital  stock  from  P200,000.00  to  
P1,000,000.00.  It  was  shown  in  said  certificate  that  P191,560.00  worth  of  shares  were  represented  in  the  
stockholders'  meeting  at  which  time  the  increase  was  approved.  Thus,  P110,980.00  worth  of  shares  were  
subsequently   issued   by   the   corporation   from   the   unissued   portion   of   the   authorized   capital   stock   of  
P200,000.00.  Of  the  increased  capital  stock  of  P1,000,000.00,  P160,000.00  worth  of  shares  were  subscribed.  
 
Petitioner  Datu  Tagoranao  filed  with  SEC  a  petition  alleging  that  the  additional  issue  of  previously  subscribed  
shares  of  the  corporation  was  made  in  violation  of  his  pre-­‐‑emptive  right  to  said  additional  issue  and  that  the  
increase  in  the  authorized  capital  stock  of  the  corporation  from  P200,000.00  to  P1,000,000.00  was  illegal  
considering  that  the  stockholders  of  record  were  not  notified  of  the  meeting  wherein  the  proposed  increase  
was  in  the  agenda.    
 
SEC,  after  due  proceedings,  rendered  a  decision,  which  was  affirmed  by  the  Commission  En  Banc,    
(a)  That  the  issuance  by  the  corporation  of  its  unissued  shares  was  validly  made  and  was  not  subject  to  the  
pre-­‐‑emptive  rights  of  stockholders,  including  the  petitioner,  herein;    
(b)  That  there  is  no  sufficient  legal  basis  to  set  aside  the  certificate  issued  by  this  Commission  authorizing  
the   increase   in   capital   stock   of   respondent   corporation   from   P200,000.00   to   Pl,000,000.00.   Considering,  
however,  that  petitioner  has  not  waived  his  pre-­‐‑emptive  right  to  subscribe  to  the  increased  capitalization,  
respondent   corporation   is   hereby   directed   to   allow   petitioner   to   subscribe   thereto,   at   par   value,  
proportionate   to   his   present   shareholdings,   adding   thereto   the   2,540   shares   transferred   to   him   by   Mr.  
Domocao  Alonto  and  Mrs.  Moki-­‐‑in  Alonto;    
 xxx  
 
ISSUE:    
W/N  petitioner  has  pre-­‐‑emptive  right  over  the  issuance  of  the  unsubscribed  capital  stock.  
W/N  the  increase  in  the  capital  stock  was  illegal  being  made  without  the  consent,  expressed  or  implied,  of  
the  stockholders.  
 
HELD:    
We  are  not  persuaded.  As  aptly  stated  by  the  Securities  and  Exchange  Commission  in  its  decision:    
xxx  xxx  xxx  
...  the  questioned  issuance  of  the  unsubscribed  portion  of  the  capital  stock  worth  P110,980.00  is  '  not  invalid  
even  if  assuming  that  it  was  made  without  notice  to  the  stockholders  as  claimed  by  petitioner.  The  power  to  
issue  shares  of  stocks  in  a  corporation  is  lodged  in  the  board  of  directors  and  no  stockholders'  meeting  is  
necessary   to   consider   it   because   additional   issuance   of   shares   of   stocks   does   not   need   approval   of   the  
stockholders.  The  by-­‐‑laws  of  the  corporation  itself  states  that  'the  Board  of  Trustees  shall,  in  accordance  with  

   
3H  A.Y.  2017-­‐2018   178  
 
 

CORPORATION  LAW  CASE  DIGESTS  –  ATTY.  DANTE  DELA  CRUZ  

law,   provide   for   the   issue   and   transfer   of   shares   of   stock   of   the   Institute   and   shall   prescribe   the   form   of   the  
certificate  of  stock  of  the  Institute.    
 
Petitioner   bewails   the   fact   that   in   view   of   the   lack   of   notice   to   him   of   such   subsequent   issuance,   he   was   not  
able  to  exercise   his   right   of   pre-­‐‑emption  over  the  unissued  shares.  However,  the  general  rule  is  that  pre-­‐‑
emptive  right  is  recognized  only  with  respect  to  new  issue  of  shares,  and  not  with  respect  to  additional  issues  
of   originally   authorized   shares.   This   is   on   the   theory   that   when   a   corporation   at   its   inception   offers   its   first  
shares,  it  is  presumed  to  have  offered  all  of  those  which  it  is  authorized  to  issue.  An  original  subscriber  is  
deemed  to  have  taken  his  shares  knowing  that  they  form  a  definite  proportionate  part  of  the  whole  number  
of   authorized   shares.   When   the   shares   left   unsubscribed   are   later   re-­‐‑offered,   he   cannot   therefore   claim   a  
dilution  of  interest.    
 
With  respect  to  the  claim  that  the  increase  in  the  authorized  capital  stock  was  without  the  consent,  expressed  
or   implied,   of   the   stockholders,   it   was   the   finding   of   the   Securities   and   Exchange   Commission   that   a  
stockholders'   meeting   was   held   on   November   25,1975,   presided   over   by   Mr.   Ahmad   Domocao   Alonto,  
Chairman  of  the  Board  of  Trustees  and,  among  the  many  items  taken  up  then  were  the  change  of  name  of  the  
corporation  from  Kamilol  Islam  Institute  Inc.  to  Jamiatul  Philippine-­‐‑Al  Islamia,  Inc.,  the  increase  of  its  capital  
stock  from  P200,000.00  to  P1,000,000.00,  and  the  increase  of  the  number  of  its  Board  of  Trustees  from  five  
to   nine.   "Despite   the   insistence   of   petitioner,   this   Commission   is   inclined   to   believe   that   there   was   a  
stockholders'   meeting   on   November   25,   1975   which   approved   the   increase.   The   petitioner   had   not  
sufficiently   overcome   the   evidence   of   respondents   that   such   meeting   was   in   fact   held.   What   petitioner  
successfully  proved,  however,  was  the  fact  that  he  was  not  notified  of  said  meeting  and  that  he  never  attended  
the   same   as   he   was   out   of   the   country   at   the   time.   The   documentary   evidence   of   petitioner   conclusively  
proved  that  he  was  attending  the  Mecca  pilgrimage  when  the  meeting  was  held  on  November  25,  1975.  While  
petitioner  doubts  the  authenticity  of  the  alleged  minutes  of  the  proceedings  (Exh.  '4'),  the  Commission  notes  
with  significance  that  said  minutes  contain  numerous  details  of  various  items  taken  up  therein  that  would  
negate   any   claim   that   it   was   not   authentic.   Another   thing   that   petitioner   was   able   to   disprove   was   the  
allegation  in  the  certificate  of  increase  that  all  stockholders  who  did  not  subscribe  to  the  increase  of  capital  
stock  have  waived  their  pre-­‐‑emptive  right  to  do  so.  As  far  as  the  petitioner  is  concerned,  he  had  not  waived  
his  pre-­‐‑emptive  right  to  subscribe  as  he  could  not  have  done  so  for  the  reason  that  he  was  not  present  at  the  
meeting  and  had  not  executed  a  waiver,  thereof.  Not  having  waived  such  right  and  for  reasons  of  equity,  he  
may  still  be  allowed  to  subscribe  to  the  increased  capital  stock  proportionate  to  his  present  shareholdings."    
 
ACCORDINGLY,  this  petition  is  hereby  dismissed  for  lack  of  merit.    
 
 
 
137.  MIGUEL  VELASCO  V.  JEAN  POIZAT  
G.R.  NO.  L-­‐‑11528.  MARCH  15,  1918  
STREET,  J.;  
 
Doctrine:  The  better  doctrine  is  that  when  insolvency  supervenes  all  unpaid  subscription  become  at  once  
due  and  enforceable.  
 
Facts:  
The   defendant   was   a   stockholder   in   the   company   from   the   inception   of   the   enterprise,   and   for   sometime  
acted  as  its  treasurer  and  manager.  While  serving  in  this  capacity  he  called  in  and  collected  all  subscriptions  
to  the  capital  stock  of  the  company,  except  the  aforesaid  15  shares  subscribed  by  himself  and  another  shares  
owned  by  Jose  R.  Infante.  
 

   
3H  A.Y.  2017-­‐2018   179  
 
 

CORPORATION  LAW  CASE  DIGESTS  –  ATTY.  DANTE  DELA  CRUZ  

Upon   July   13,   1914,   a   meeting   of   the   board   of   directors   of   the   company   was   held   at   which   a   majority   of   the  
stock   was   represented.   Upon   this   occasion   two   resolutions,   important   to   be   here   noted,   were   adopted.   The  
first   was   a   proposal   that   the   directors,   or   shareholders,   of   the   company   should   make   good   by   new  
subscription,  in  proportion  to  their  respective  holdings,  15  shares  which  had  been  surrendered  in  Infante.  It  
seems   that   this   shareholder   had   already   paid   25   per   cent   of   his   subscription   upon   20   shares,   leaving   15  
shares  unpaid  for,  and  an  understanding  had  been  reached  by  him  and  the  management  by  which  he  was  to  
be  released  from  the  obligation  of  his  subscription,  it  being  understood  that  what  he  had  already  paid  should  
not   be   refunded.   Accordingly   the   directors   present   at   this   meeting   subscribed   P1,200   toward   taking   up   his  
shares,  leaving  a  deficiency  of  P300  to  be  recovered  by  voluntary  subscription  from  stockholders  not  present  
at  the  meeting.  
 
The  other  proposition  was  to  the  effect  that  Juan  M.  Poizat,  who  was  absent,  should  be  required  to  pay  the  
amount  of  his  subscription  upon  the  15  shares  for  which  he  was  still  indebted  to  the  company.  The  resolution  
further  provided  that,  in  case  he  should  refuse  to  make  such  payment,  the  management  of  the  corporation  
should   be   authorized   to   undertake   judicial   proceedings   against   him.   When   notification   of   this   resolution  
reached   Poizat   through   the   mail   it   evoked   from   him   a   manifestation   of   surprise   and   pain,   which   found  
expression  in  a  letter  written  by  him  a  reply.  In  this  letter  Poizat  states  that  he  had  been  given  to  understand  
by   some   member   of   the   board   of   directors   that   he   was   to   be   relieved   from   his   subscription   upon   the   terms  
conceded  to  Infante.  
 
Issue:  Whether  Poizat  is  liable  for  subscription.  (Yes)  
 
Ratio:  
We   think   that   Poizat   is   liable   upon   this   subscription.   A   stock   subscription   is   a   contract   between   the  
corporation  on  one  side,  and  the  subscriber  on  the  other,  and  courts  will  enforce  it  for  or  against  either.  It  is  
a  rule,  accepted  by  the  Supreme  Court  of  the  United  States,  that  a  subscription  for  shares  of  stock  does  not  
require  an  express  promise  to  pay  the  amount  subscribed,  as  the  law  implies  a  promise  to  pay  on  the  part  of  
the  subscriber.  Section  36  of  the  Corporation  Law  clearly  recognizes  that  a  stock  subscription  is  a  subsisting  
liability  from  the  time  the  subscription  is  made,  since  it  requires  the  subscriber  to  pay  interest  quarterly  from  
that  date  unless  he  is  relieved  from  such  liability  by  the  by-­‐‑laws  of  the  corporation.  The  subscriber  is  a  much  
bound  to  pay  the  amount  of  the  share  subscriber  by  him  as  he  would  be  to  pay  any  other  debt,  and  the  right  
of  the  company  to  demand  payment  is  no  less  incontestable.  
 
It   is   generally   accepted   doctrine   that   the   statutory   right   to   sell   the   subscriber's   stock   is   merely   a   remedy   in  
addition  to  that  which  proceeds  by  action  in  court;  and  it  has  been  held  that  the  ordinary  legal  remedy  by  
action  exists  even  though  no  express  mention  thereof  is  made  in  the  statute.  
 
But   there   is   another   reason   why   the   present   plaintiff   must   prevail   in   this   case.   .   .   That   reason   is   this:   When  
insolvency  supervenes  upon  a  corporation  and  the  court  assumes  jurisdiction  to  wind  it  up,  all  unpaid  stock  
subscriptions  become  payable  on  demand,  and  are  at  once  recoverable  in  an  action  instituted  by  the  assignee  
or  receiver  appointed  by  the  court.  This  rule  apparently  had  its  origin  in  a  recognition  of  the  principle  that  a  
court  of  equity,  having  jurisdiction  of  the  insolvency  proceedings,  could,  if  necessary,  make  the  call  itself,  in  
its   capacity   as   successor   to   the   powers   exercised   by   the   board   of   directors   of   the   defunct   company.   Later   a  
further  rule  gained  recognition  to  the  effect  that  the  receiver  or  assignee,  in  an  action  instituted  by  proper  
authority,  could  himself  proceed  to  collect  the  subscription  without  the  necessity  of  any  prior  call  whether.  
 
It  evidently  cannot  be  permitted  that  a  subscriber  should  escape  from  his  lawful  obligation  by  reason  of  the  
failure  of  the  officers  of  the  corporation  to  perform  their  duty  in  making  a  call;  and  when  the  original  mode  
of   making   the   call   becomes   impracticable,   the   obligation   must   be   treated   as   due   upon   demand.   If   the  
corporation   were   still   an   active   entity,   and   this   action   should   be   dismissed   for   irregularity   in   the   making   of  

   
3H  A.Y.  2017-­‐2018   180  
 
 

CORPORATION  LAW  CASE  DIGESTS  –  ATTY.  DANTE  DELA  CRUZ  

the  call,  other  steps  could  be  taken  by  the  board  to  cure  the  defect  and  another  action  taken  by  the  board  to  
cure  the  defect  and  another  action  could  be  brought;  but  where  the  company  is  being  wound  up,  no  such  
procedure   would   be   practicable.   The   better   doctrine   is   that   when   insolvency   supervenes   all   unpaid  
subscription  become  at  once  due  and  enforceable.  
 
 
 
138.    LINGAYEN  GULF  ELECTRIC  POWER  COMPANY,  INC.  V.  IRINEO  BALTAZAR  
G.R.  NO.  L-­‐‑4824  -­‐‑  JUNE  30,  1953  
MONTEMAYOR,  J.  
 
DOCTRINE:  
In  order  to  effect  the  release  of  a  stockholder  from  his  stock  subscription,  there  must  be  unanimous  consent  
of   the   stockholders   of   the   corporation.   From   this   rule,   however,   there   are   exceptions:   "Where   it   is   given  
pursuant  to  a  bona  fide  compromise,  or  to  set  off  a  debt  due  from  the  corporation,  a  release,  supported  by  
consideration,   will   be   effectual   as   against   dissenting   stockholders   and   subsequent   and   existing   creditors.   A  
release  which  might  originally  have  been  held  invalid  may  be  sustained  after  a  considerable  lapse  of  time."  
 
NB:   This   is   a   1953   case   decided   under   the   effectivity   of   the   Corporation   Law.   The   Corporation   Code   we  
now   use   took   effect   on   May   1,   1980.   The   Corp   Code   now   provides   under   Sec.   61   that   a   subscription   for  
shares   of   stock   of   a   corporation   still   to   be   formed   shall   be   irrevocable   for   a   period   of   at   least   six   (6)  
months  from  the  date  of  subscription,  unless  all  of  the  other  subscribers  consent  to  the  revocation,  or  
unless   the   incorporation   of   said   corporation   fails   to   materialize   within   said   period   or   within   a   longer  
period   as   may   be   stipulated   in   the   contract   of   subscription:   Provided,   That   no   pre-­‐‑incorporation  
subscription  may  be  revoked  after  the  submission  of  the  articles  of  incorporation  to  the  Securities  and  
Exchange  Commission.  
 
FACTS:  
Lingayen   Gulf   Electric   Power   Company   is   a   domestic   corporation   while   the   defendant,   Irineo   Baltazar,  
appears  to  have  subscribed  for  600  shares  on  account  of  which  he  had  paid  upon  the  organization  of  the  
corporation  the  sum  of  P15,000.  After  incorporation,  the  Baltazar  made  further  payments  on  account  of  his  
subscription,  leaving  a  balance  of  P18,500  unpaid  for,  which  amount,  the  plaintiff  now  claims  in  this  action.  
 
On  July  23,  1946,  a  majority  of  the  stockholders  of  the  corporation,  among  them  the  herein  defendant,  held  a  
meeting   and   adopted   stockholders'   resolution   No.   17.   By   said   resolution,   it   was   agreed   upon   by   the  
stockholders  present  to  call  the  balance  of  all  unpaid  subscribed  capital  stock  as  of  July  23,  1946  and  provided  
for   the   time   of   payment   thereof   and   that   all   subscribed   stocks   remaining   unpaid   would   revert   to   the  
corporation.  
 
On  April  17,  1948,  the  Board  of  Directors  of  the  plaintiff  corporation  held  a  meeting,  and  in  the  course  of  the  
said  meeting  they  adopted  Resolution  No.  17.  This  resolution  in  effect  set  aside  the  stockholders  resolution  
approved  on  June  23,  1946,  on  the  ground  that  said  stockholders'  resolution  was  null  and  void,  and  because  
the  plaintiff  corporation  was  not  in  a  financial  position  to  absorb  the  unpaid  balance  of  the  subscribed  capital  
stock.  
 
The  defendant,  in  his  answer,  disclaims  liability  to  the  plaintiff  corporation  on  the  ground  that  the  he  was  
released  from  the  obligation  of  the  balance  of  his  subscription  by  stockholders'  resolution  No.  17.  
 
ISSUE:  
Did  Resolution  No.  17  of  1946  released  Baltazar  from  the  obligation  to  pay  for  his  unpaid  subscription?  

   
3H  A.Y.  2017-­‐2018   181  
 
 

CORPORATION  LAW  CASE  DIGESTS  –  ATTY.  DANTE  DELA  CRUZ  

 
HELD:  
NO.  The  authorities  are  generally  agreed  that  in  order  to  effect  the  release,  there  must  be  unanimous  consent  
of  the  stockholders  of  the  corporation.  The  general  rule  is  that  a  valid  and  binding  subscription  for  stock  of  a  
corporation  cannot  be  cancelled  so  as  to  release  the  subscriber  from  liability  thereon  without  the  consent  of  
all  the  stockholders  or  subscribers.    
 
Furthermore,  a  subscription  cannot  be  cancelled  by  the  company,  even  under  a  secret  or  collateral  agreement  
for   cancellation   made   with   the   subscriber   at   the   time   of   the   subscription,   as   against   persons   who  
subsequently  subscribed  or  purchased  without  notice  of  such  agreement.  
 
In  particular  circumstances,  as  where  it  is  given  pursuant  to  a  bona  fide  compromise,  or  to  set  off  a  debt  due  
from   the   corporation,   a   release,   supported   by   consideration,   will   be   effectual   as   against   dissenting  
stockholders  and  subsequent  and  existing  creditors.  A  release  which  might  originally  have  been  held  invalid  
may  be  sustained  after  a  considerable  lapse  of  time.  
 
In  the  present  case,  the  release  claimed  by  defendant  and  appellant  does  not  fall  under  the  exception  above  
referred  to,  because  it  was  not  given  pursuant  to  a  bona  fide  compromise,  or  to  set  off  a  debt  due  from  the  
corporation,  and  there  was  no  consideration  for  it.  The  release  attempted  in  Resolution  No.  17  of  1946  was  
not  valid  for  lack  of  a  unanimous  vote.  It  found  that  at  least  seven  stockholders  were  absent  from  the  meeting  
when  said  resolution  was  approved.  
 
A  contract  of  subscription  is,  at  least  in  the  sense  which  creates  as  estoppel,  a  contract  among  the  several  
subscribers.   For   this   reason   no   one   of   the   subscribers   can   withdraw   from   the   contract   without   the   consent  
of  all  the  others,  and  thereby  diminish,  without  the  universal  consent,  the  common  fund  in  which  all  have  
acquired  an  interest.  
 
 
 
139.   ROMANA   MIRANDA,   IN   HER   CAPACITY   AS   JUDICIAL   ADMINISTRATRIX   OF   THE   INTESTATE  
ESTATE  OF  HER  DECEASED  FATHER,  ALBERTO  MIRANDA  VS.  THE  TARLAC  RICE  MILL  CO.,  INC.  
G.R.  NO.  L-­‐‑35961                  DECEMBER  2,  1932  
VICKERS,  J.:  
 
DOCTRINE:  Section  38  of  the  Corporation  Law  provides  that  the  board  of  directors  of  every  corporation  may  
at  any  time  declare  due  and  payable  to  the  corporation  unpaid  subscriptions  to  the  capital  stock  and  may  
collect  the  same  with  interest  accrued  thereon  or  such  percentage  of  said  unpaid  subscriptions  as  it  may  
deem  necessary.  No  call  is  necessary  when  a  subscription  is  payable,  not  upon  call  or  demand  by  the  directors  
or   stockholders,   but   immediately,   or   on   specified   day,   or   on   or   before   a   specified   day,   or   when   it   is   payable  
in  installments  at  specified  times.  In  such  cases  it  is  the  duty  of  the  subscriber  to  pay  the  subscription  or  
instalment  thereof  as  soon  as  it  is  due,  without  any  call  or  demand,  and,  if  he  fails  to  do  so,  an  action  may  be  
brought  at  any  time.  
 
FACTS:  Alberto  Miranda  executed  a  written  contract  whereby  he  subscribed  for  100  shares  of  Tarlac  Rice  
Mill  Company,  Inc.,  that  the  par  value  of  each  share  was  P100  and  to  be  paid  in  installments.    
 
On  July  10,  1926  Alberto  Miranda  by  means  of  a  public  document  "assigned"  mortgaged,  or  transferred  in  
lieu  of  cash  for  the  benefit  and  to  the  credit  of  the  Tarlac  Rice  Mill  Company,  Inc.  the  parcel  of  land  described  
in  certificate  No.  751  in  the  land  records  of  the  Province  of  Tarlac.  
 

   
3H  A.Y.  2017-­‐2018   182  
 
 

CORPORATION  LAW  CASE  DIGESTS  –  ATTY.  DANTE  DELA  CRUZ  

On  February  19,  1927  the  president  and  vice-­‐‑president  of  the  Tarlac  Rice  Mill  Company,  Inc.,  and  C.  M.  Dizon,  
acting  on  behalf  of  said  corporation  and  Alberto  Miranda,  borrowed  P10,000  from  Mariano  Tablante,  and  
agreed  to  repay  said  sum  on  or  before  February  19,  1928,  with  interest  at  12  per  cent  per  annum,  and  to  pay  
a   further   sum   of   25   per   cent   of   the   principal   for   attorney's   fees   and   expenses   of   collection   in   case   the  
promissory  note  should  not  be  paid  at  maturity.  Marcos  Puno,  Evaristo  Magbag,  and  Dizon  &  Co.,  Inc.,  jointly  
and  severally  guaranteed  the  payment  of  this  sum;  and  the  president  and  vice-­‐‑president  of  the  Tarlac  Rice  
Mill   Company,   Inc.,   and   C.   M.   Dizon   as   attorneys-­‐‑in-­‐‑fact   of   Alberto   Miranda   mortgaged   to   Mariano   Tablante  
the  aforementioned  parcel  of  land  to  secure  the  payment  of  said  promissory  note.  
 
When   the   promissory   note   became   due,   Alberto   Miranda   arranged   for   an   extension   of   time   in   which   to   pay  
it,  and  on  July  19,  1929  he  sold  the  aforementioned  parcel  of  land  under  pacto  de  retro  to  Vicente  Panlilio.  
 
It   is   agreed   that   the   defendant   corporation   ceased   to   do   business   from   the   year   1928,   and   that   the   other  
stockholders  have  not  paid  for  their  shares  in  accordance  with  their  subscription  agreement,  and  that  no  
action  has  been  taken  by  the  corporation  to  require  them  to  do  so.  
 
The   principal   contention   of   the   appellant   is   that   the   officers   of   the   corporation   violated   the   terms   of   the  
power  of  attorney  in  mortgaging  the  land  on  February  19,  1927  for  P10,000,  because  the  only  sum  then  due  
and  payable  by  Alberto  Miranda  to  the  corporation  was  P3,000,  and  that  when  the  remaining  instalments  of  
the   stock   subscription   became   due,   Alberto   Miranda   was   under   no   obligation   to   pay   them,   because   the  
corporation  had  already  ceased  to  do  business,  and  it  had  taken  no  steps  to  compel  the  other  stockholders  
to  pay  for  the  shares  for  which  they  had  subscribed.  
 
 
ISSUE  Whether  or  not  the  mortgage  and  subsequent  sale  of  the  land  is  valid?  
 
HELD:  Yes.  Section  38  of  the  Corporation  Law  provides  that  the  board  of  directors  of  every  corporation  may  
at  any  time  declare  due  and  payable  to  the  corporation  unpaid  subscriptions  to  the  capital  stock  and  may  
collect  the  same  with  interest  accrued  thereon  or  such  percentage  of  said  unpaid  subscriptions  as  it  may  
deem  necessary.  
 
No  call  is  necessary  when  a  subscription  is  payable,  not  upon  call  or  demand  by  the  directors  or  stockholders,  
but  immediately,  or  on  specified  day,  or  on  or  before  a  specified  day,  or  when  it  is  payable  in  installments  at  
specified  times.  In  such  cases  it  is  the  duty  of  the  subscriber  to  pay  the  subscription  or  instalment  thereof  as  
soon  as  it  is  due,  without  any  call  or  demand,  and,  if  he  fails  to  do  so,  an  action  may  be  brought  at  any  time.  
 
When  this  action  was  filed  on  September  2,  1930,  the  last  of  the  installments  had  already  become  payable  in  
accordance   with   the   subscription   agreement.   it   must   be   borne   in   mind   that   this   is   not   an   action   by   the  
corporation  to  recover  on  a  subscription  agreement,  but  an  action  by  the  administratrix  of  a  stockholder  to  
recover  what  was  paid  in  to  the  corporation  by  the  stockholder.  It  does  not  appear  from  the  evidence  whether  
or  not  the  corporation  has  any  debts.  Neither  the  fact  that  the  corporation  has  ceased  to  do  business  nor  the  
fact   that   the   other   stockholders   have   not   been   required   to   pay   for   their   shares   in   accordance   with   their  
subscription  agreement  justifies  us  in  ordering  the  corporation  to  return  to  the  plaintiff  the  amount  paid  in  
by  Alberto  Miranda.  If  the  directors  have  failed  to  perform  their  duty  with  respect  to  the  other  stockholders,  
the  law  provides  a  remedy  therefor.  
 
 
 
140.    DE  SILVA  V  ABOITIZ  
 

   
3H  A.Y.  2017-­‐2018   183  
 
 

CORPORATION  LAW  CASE  DIGESTS  –  ATTY.  DANTE  DELA  CRUZ  

141.  THE  NATIONAL  EXCHANGE  CO.,  INC  VS.  I.  B.  DEXTER  
GR  NO.  L-­‐‑27872  /  FEBRUARY  25,  1928  
STREET,  J.:  
 
DOCTRINE:  The  prohibition  against  the  issuance  of  shares  by  corporations  except  for  actual  cash  to  the  par  
value  of  the  stock  to  its  full  equivalent  in  property  is  thus  enshrined  in  both  the  organic  and  statutory  law  of  
the  Philippine;  Islands;  A  stipulation  such  as  that  now  under  consideration,  in  a  stock  subcription,  is  illegal,  
for  this  stipulation  obligates  the  subcriber  to  pay  nothing  for  the  shares  except  as  dividends  may  accrue  upon  
the  stock.  In  the  contingency  that  dividends  are  not  paid,  there  is  no  liability  at  all.  This  is  a  discrimination  in  
favor  of  the  particular  subcriber,  and  hence  the  stipulation  is  unlawful.  
 
FACTS:    On  August  10,  1919,  I.  B.  Dexter,  signed  a  written  subscription  to  the  corporate  stock  of  C.  S.  Salmon  
&  Co.  in  the  following  form:  
 
I  hereby  subscribe  for  300  shares  of  the  capital  stock  of  C.  S.  Salmon  and  Company,  payable  from  the  first  
dividends  declared  xxx  until  the  full  amount  of  this  subscription  has  been  paid.  
 
The  sum  of  P15,000  was  paid  in  January,  1920,  from  a  dividend  declared  at  about  that  time  by  the  company.  
Beyond   this   nothing   has   been   paid   on   the   shares   and   no   further   dividend   has   been   declared   by   the  
corporation  leaving  a  balance  of  P15,000.    
 
 
ISSUE:  whether  the  stipulation  contained  in  the  subscription  to  the  effect  that  the  subscription  is  payable  
from   the   first   dividends   declared   on   the   shares   has   the   effect   of   relieving   the   subscriber   from   personal  
liability  in  an  action  to  recover  the  value  of  the  shares.  
 
 
HELD:  YES.  Such  stipulation  is  illegal.  We  find  that  the  Philippine  Commission  inserted  in  the  Corporation  
Law,  enacted  March  1,  1906,  the  following  provision:  ".  .  .  no  corporation  shall  issue  stock  or  bonds  except  in  
exchange  for  actual  cash  paid  to  the  corporation  or  for  property  actually  received  by  it  at  a  fair  valuation  
equal  to  the  par  value  of  the  stock  or  bonds  so  issued."    
 
The  prohibition  against  the  issuance  of  shares  by  corporations  except  for  actual  cash  to  the  par  value  of  the  
stock   to   its   full   equivalent   in   property   is   thus   enshrined   in   both   the   organic   and   statutory   law   of   the  
Philippine;   Islands;   A   stipulation   such   as   that   now   under   consideration,   in   a   stock   subcription,   is   illegal,   for  
this  stipulation  obligates  the  subcriber  to  pay  nothing  for  the  shares  except  as  dividends  may  accrue  upon  
the  stock.  In  the  contingency  that  dividends  are  not  paid,  there  is  no  liability  at  all.  This  is  a  discrimination  in  
favor  of  the  particular  subcriber,  and  hence  the  stipulation  is  unlawful.  
 
 
 
142.    LUMANLAN  V.  CUA  
 
 
 
143.  FUA  CUN  VS.  RICARDO  SUMMERS  AND  THE  CHINA  BANKING  CORPORATION  
G.R.  NO.  L-­‐‑19441  MARCH  27,  1923  
OSTRAND,  J.:  
 
DOCTRINE:   A   corporation   has   no   lien   upon   the   shares   of   stockholders   for   any   indebtedness   to   the  

   
3H  A.Y.  2017-­‐2018   184  
 
 

CORPORATION  LAW  CASE  DIGESTS  –  ATTY.  DANTE  DELA  CRUZ  

corporation.  The  reasons  for  this  doctrine  are  obvious.  If  banking  corporations  were  given  a  lien  on  their  
own  stock  for  the  indebtedness  of  the  stockholders,  the  prohibition  against  granting  loans  or  discounts  upon  
the  security  of  the  stock  would  become  largely  ineffective.  
 
FACTS:   On   August   26,   1920,   one   Chua   Soco   subscribed   for   five   hundred   shares   of   stock   of   the   defendant  
Banking  Corporation  at  a  par  value  of  P100  per  share,  paying  the  sum  of  P25,000,  one-­‐‑half  of  the  subscription  
price.   On   May   18,   1921,   Chua   Soco   executed   a   promissory   note   in   favor   of   the   plaintiff   Fua   Cun   for   the   sum  
of  P25,000,  securing  the  note  with  a  chattel  mortgage  on  the  shares  of  stock  subscribed  for  by  Chua  Soco,  
who  also  endorsed  the  receipt  above  mentioned  and  delivered  it  to  the  mortgagee.    
 
In   the   meantime   Chua   Soco   appears   to   have   become   indebted   to   the   China   Banking   Corporation   in   the   sum  
of   P37,731.68   for   dishonored   acceptances   of   commercial   paper   and   in   an   action   brought   against   him   to  
recover  this  amount,  Chua  Soco's  interest  in  the  five  hundred  shares  subscribed  for  was  attached  and  the  
receipt  seized  by  the  sheriff.  The  attachment  was  levied  after  the  defendant  bank  had  received  notice  of  the  
facts  that  the  receipt  had  been  endorsed  over  to  the  plaintiff.  
 
Fua   Cun   thereupon   brought   an   action   maintaining   that   by   virtue   of   the   payment   of   the   one-­‐‑half   of   the  
subscription  price  of  five  hundred  shares  Chua  Soco  in  effect  became  the  owner  of  two  hundred  and  fifty  
shares  and  praying  that  his,  the  plaintiff's,  lien  on  said  shares,  by  virtue  of  the  chattel  mortgage,  be  declared  
to  hold  priority  over  the  claim  of  the  defendant  Banking  Corporation.    
 
The  trial  court  rendered  judgment  in  favor  of  the  plaintiff  declaring  that  Chua  Soco,  through  the  payment  of  
the  P25,000,  acquired  the  right  to  two  hundred  and  fifty  shares  fully  paid  up,  upon  which  shares  the  plaintiff  
holds  a  lien  superior  to  that  of  the  defendant  Banking  Corporation  and  ordering  that  the  receipt  be  returned  
to  said  plaintiff.    
 
ISSUE:  Whether  or  not  the  plaintiff  holds  a  lien  superior  to  that  of  the  defendant  Banking  Corporation.  
 
HELD:  Yes.    Though  the  court  below  erred  in  holding  that  Chua  Soco,  by  paying  one-­‐‑half  of  the  subscription  
price   of   five   hundred   shares,   in   effect   became   the   owner   of   two   hundred   and   fifty   shares,   the   judgment  
appealed  from  is  in  the  main  correct.  
 
The   claim   of   the   defendant   Banking   Corporation   upon   which   it   brought   the   action   in   which   the   writ   of  
attachment   was   issued,   was   for   the   non-­‐‑payment   of   drafts   accepted   by   Chua   Soco   and   had   no   direct  
connection  with  the  shares  of  stock  in  question.  At  common  law  a  corporation  has  no  lien  upon  the  shares  of  
stockholders   for   any   indebtedness   to   the   corporation.   The   reasons   for   this   doctrine   are   obvious.   If   banking  
corporations   were   given   a   lien   on   their   own   stock   for   the   indebtedness   of   the   stockholders,   the   prohibition  
against  granting  loans  or  discounts  upon  the  security  of  the  stock  would  become  largely  ineffective.  
 
With  regard  to   the  rights  of  the  plaintiff  in  the  stock  in  question,  it  is  argued  that  the  interest  held  by  Chua  
Soco  was  merely  an  equity  which  could  not  be  made  the  subject  of  a  chattel  mortgage.  Though  the  courts  
have  uniformly  held  that  chattel  mortgages  on  shares  of  stock  and  other  choses  in  action  are  valid  as  between  
the  parties,  there  is  still  much  to  be  said  in  favor  of  the  defendants'  contention  that  the  chattel  mortgage  here  
in  question  would  not  prevail  over  liens  of  third  parties  without  notice;  an  equity  in  shares  of  stock  is  of  such  
an   intangible   character   that   it   is   somewhat   difficult   to   see   how   it   can   be   treated   as   a   chattel   and   mortgaged  
in  such  a  manner  that  the  recording  of  the  mortgage  will  furnish  constructive  notice  to  third  parties.      
 
There  can  be  no  doubt  that  an  equity  in  shares  of  stock  may  be  assigned  and  that  the  assignment  is  valid  as  
between   the   parties   and   as   to   persons   to   whom   notice   is   brought   home.   Such   an   assignment   exists   here,  
though  it  was  made  for  the  purpose  of  securing  a  debt.    

   
3H  A.Y.  2017-­‐2018   185  
 
 

CORPORATION  LAW  CASE  DIGESTS  –  ATTY.  DANTE  DELA  CRUZ  

 
As  against  the  rights  of  the  plaintiff  the  defendant  bank  had,  as  we  have  seen,  no  lien  unless  by  virtue  of  the  
attachment.   But   the   attachment   was   levied   after   the   bank   had   received   notice   of   the   assignment   of   Chua  
Soco's  interests  to  the  plaintiff  and  was  therefore  subject  to  the  rights  of  the  latter.  It  follows  that  as  against  
these  rights  the  defendant  bank  holds  no  lien  whatever.  
 
 
 
144.    BALTAZAR  V.  LINGAYEN  GULF  ELECTRIC  POWER  CO.  
 
 
 
145.  NAVA  V.  PEERS  MARKETING  CORPORATION,  RENATO  CUSI  
G.R.  NO.  L-­‐‑28120  NOVEMBER  25,  1976  
AQUINO,  J;  
 
Doctrine:  As  already  stressed,  in  this  case  no  stock  certificate  was  issued  to  Po.  Without  stock  certificate,  
which   is   the   evidence   of   ownership   of   corporate   stock,   the   assignment   of   corporate   shares   is   effective   only  
between  the  parties  to  the  transaction.    
 
The   delivery   of   the   stock   certificate,   which   represents   the   shares   to   be   alienated   ,   is   essential   for   the  
protection  of  both  the  corporation  and  its  stockholders  
 
Facts:  In  this  case,  Teofilo  Po  as  an  incorporator  subscribed  to  eighty  shares  of  Peers  Marketing  Corporation  
at  one  hundred  pesos  a  share  or  a  total  par  value  of  eight  thousand  pesos.  Po  paid  two  thousand  pesos  or  
twenty-­‐‑five  percent  of  the  amount  of  his  subscription.  No  certificate  of  stock  was  issued  to  him  or,  for  that  
matter,  to  any  incorporator,  subscriber  or  stockholder.  
 
On  April  2,  1966  Po  sold  to  Ricardo  A.  Nava  for  two  thousand  pesos  twenty  of  his  eighty  shares.  In  the  deed  
of  sale  Po  represented  that  he  was  "the  absolute  and  registered  owner  of  twenty  shares"  of  Peers  Marketing  
Corporation.  
 
Nava  requested  the  officers  of  the  corporation  to  register  the  sale  in  the  books  of  the  corporation.  The  request  
was  denied  because  Po  has  not  paid  fully  the  amount  of  his  subscription.  Nava  was  informed  that  Po  was  
delinquent  in  the  payment  of  the  balance  due  on  his  subscription  and  that  the  corporation  had  a  claim  on  his  
entire  subscription  of  eighty  shares  which  included  the  twenty  shares  that  had  been  sold  to  Nava.  
 
Nava  filed  this  mandamus  action  in  the  Court  of  First  Instance  of  Negros  Occidental,  Bacolod  City  Branch  to  
compel   the   corporation   and   Renato   R.   Cusi   and   Amparo   Cusi,   its   executive   vice-­‐‑president   and   secretary,  
respectively,  to  register  the  said  twenty  shares  in  Nava's  name  in  the  corporation's  transfer  book.  
 
The  respondents  in  their  answer  pleaded  the  defense  that  no  shares  of  stock  against  which  the  corporation  
holds  an  unpaid  claim  are  transferable  in  the  books  of  the  corporation.  
 
Appellant  Nava  contends  that  the  Fua  Cun  case  was  decided  under  section  36  of  the  Corporation  Law  which  
provides  that  "no  certificate  of  stock  shall  be  issued  to  a  subscriber  as  fully  paid  up  until  the  full  par  value  
thereof  has  been  paid  by  him  to  the  corporation".  Section  36  was  amended  by  Act  No.  3518.  It  is  now  section  
37.  Section  37  provides  that  "no  certificate  of  stock  shall  be  issued  to  a  subscriber  as  fully  paid  up  until  the  full  
par  value  thereof,  or  the  full  subscription  in  case  of  no  par  stock,  has  been  paid  by  him  to  the  corporation".  

   
3H  A.Y.  2017-­‐2018   186  
 
 

CORPORATION  LAW  CASE  DIGESTS  –  ATTY.  DANTE  DELA  CRUZ  

Issue:   Whether   or   not   the   officers   of   Peers   Marketing   Corporation   can   be   compelled   by   mandamus   to   enter  
in   its   stock   and   transfer   book   the   sale   made   by   Po   to   Nava   of   the   twenty   shares   forming   part   of   Po's  
subscription  of  eighty  shares,  with  a  total  par  value  of  P8,000  and  for  which  Po  had  paid  only  P2,000,  it  being  
admitted  that  the  corporation  has  an  unpaid  claim  of  P6,000  as  the  balance  due  on  Po's  subscription  and  that  
the  twenty  shares  are  not  covered  by  any  stock  certificate  
 
Ruling:  Apparently,  no  provision  of  the  by-­‐‑laws  of  the  corporation  covers  that  situation.  The  parties  did  not  
bother  to  submit  in  evidence  the  by-­‐‑laws  nor  invoke  any  of  its  provisions.  The  corporation  can  include  in  its  
by-­‐‑laws  rules,  not  inconsistent  with  law,  governing  the  transfer  of  its  shares  of  stock.    
 
We  hold  that  the  transfer  made  by  Po  to  Nava  is  not  the  "alienation,  sale,  or  transfer  of  stock"  that  is  supposed  
to  be  recorded  in  the  stock  and  transfer  book,  as  contemplated  in  section  52  of  the  Corporation  Law.  
 
A   corporation   cannot   release   an   original   subscriber   from   paying   for   his   shares   without   a   valuable  
consideration.     Under   the   facts   of   this   case,   there   is   no   clear   legal   duty   on   the   part   of   the   officers   of   the  
corporation  to  register  the  twenty  shares  in  Nava's  name,  Hence,  there  is  no  cause  of  action  for  mandamus.  
 
Nava  argues  that  under  section  37  a  certificate  of  stock  may  be  issued  for  shares  the  par  value  of  which  have  
already   been   paid   for   although   the   entire   subscription   has   not   been   fully   paid.   He   contends   that   Peers  
Marketing  Corporation  should  issue  a  certificate  of  stock  for  the  twenty  shares,  notwithstanding  that  Po  had  
not   paid   fully   his   subscription   for   the   eighty   shares,   because   section   37   requires   full   payment   for   the  
subscription,  as  a  condition  precedent  for  the  issuance  of  the  certificate  of  stock,  only  in  the  case  of  no  par  
stock.  
 
Nava  relies  on  Baltazar  v  Lingayen  Gulf  Electric  Power  Co.,  Inc.,  L-­‐‑16236-­‐‑38,  June  30,  1965,  14  SCRA  522,  
where  it  was  held  that  section  37  "requires  as  a  condition  before  a  shareholder  can  vote  his  shares  that  his  full  
subscription  be  paid  in  the  case  of  no  par  value  stock;  and  in  case  of  stock  corporation  with  par  value,  the  
stockholder  can  vote  the  shares  fully  paid  by  him  only,  irrespective  of  the  unpaid  delinquent  shares".  
 
There  is  no  parallelism  between  this  case  and  the  Baltazar  case.  It  is  noteworthy  that  in  the  Baltazar  case  the  
stockholder,  an  incorporator,  was  the  holder  of  a  certificate  of  stock  for  the  shares  the  par  value  of  which  had  
been  paid  by  him.  The  issue  was  whether  the  said  shares  had  voting  rights  although  the  incorporator  had  not  
paid  fully  the  total  amount  of  his  subscription.  That  is  not  the  issue  in  this  case.  
 
In  the  Baltazar  case,  it  was  held  that  where  a  stockholder  subscribed  to  a  certain  number  of  shares  with  par  
value   and   he   made   a   partial   payment   and   was   issued   a   certificate   for   the   shares   covered   by   his   partial  
payment,  he  is  entitled  to  vote  the  said  shares,  although  he  has  not  paid  the  balance  of  his  subscription  and  
a  call  or  demand  had  been  made  for  the  payment  of  the  par  value  of  the  delinquent  shares.  
As  already  stressed,  in  this  case  no  stock  certificate  was  issued  to  Po.  Without  stock  certificate,  which  is  the  
evidence   of   ownership   of   corporate   stock,   the   assignment   of   corporate   shares   is   effective   only   between   the  
parties  to  the  transaction.    
 
The   delivery   of   the   stock   certificate,   which   represents   the   shares   to   be   alienated   ,   is   essential   for   the  
protection  of  both  the  corporation  and  its  stockholders  
 
 
 
146.  NELSON  V.  LEPANTO  CONSOLIDATED  MINING  
 
 

   
3H  A.Y.  2017-­‐2018   187  
 
 

CORPORATION  LAW  CASE  DIGESTS  –  ATTY.  DANTE  DELA  CRUZ  

 
147.  TURNER  VS.  LORENZO  SHIPPING  CORPORATION  
G.R.  NO.  157479                              NOVEMBER  24,  2010  
BERSAMIN,  J.  
 
DOCTRINE:  
  A  corporation  can  purchase  its  own  shares,  provided  payment  is  made  out  of  surplus  profits  and  the  
acquisition  is  for  a  legitimate  corporate  purpose.  In  the  Philippines,  this  new  rule  is  embodied  in  Section  41  
of  the  Corporation  Code.  
 
FACTS:  
  The  petitioners  held  1,010,000  shares  of  stock  of  the  respondent,  a  domestic  corporation  engaged  
primarily   in   cargo   shipping   activities.   In   June   1999,   the   respondent   decided   to   amend   its   articles   of  
incorporation  to  remove  the  stockholders’  pre-­‐‑emptive  rights  to  newly  issued  shares  of  stock.  Feeling  that  
the   corporate   move   would   be   prejudicial   to   their   interest   as   stockholders,   the   petitioners   voted   against   the  
amendment   and   demanded   payment   of   their   shares   at   the   rate   of   ₱2.276/share   based   on   the   book   value   of  
the  shares,  or  a  total  of  ₱2,298,760.00.  
  The  disagreement  on  the  valuation  of  the  shares  led  the  parties  to  constitute  an  appraisal  committee  
pursuant  to  Section  82  of  the  Corporation  Code,  each  of  them  nominating  a  representative,  who  together  
then  nominated  the  third  member  who  would  be  chairman  of  the  appraisal  committee.  
  The   appraisal   committee   reported   its   valuation   of   ₱2.54/share,   for   an   aggregate   value   of  
₱2,565,400.00  for  the  petitioners.  Petitioners  demanded  payment  based  on  the  valuation  of  the  appraisal  
committee.  
  In  its  letter  to  the  petitioners  dated  January  2,  2001,  the  respondent  refused  the  petitioners’  demand,  
explaining   that   pursuant   to   the   Corporation   Code,   the   dissenting   stockholders   exercising   their   appraisal  
rights  could  be  paid  only  when  the  corporation  had  unrestricted  retained  earnings  to  cover  the  fair  value  of  
the  shares,  but  that  it  had  no  retained  earnings  at  the  time  of  the  petitioners’  demand,  as  borne  out  by  its  
Financial  Statements  for  Fiscal  Year  1999  showing  a  deficit  of  ₱72,973,114.00  as  of  December  31,  1999.  
 
ISSUE:  
  WHETHER   AT   THE   TIME   THE   "COMPLAINT"   WAS   FILED,   LSC   HAD   NO   RETAINED   EARNINGS,   AND  
THUS  WAS  COMPLYING  WITH  THE  LAW,  AND  NOT  VIOLATING  ANY  RIGHTS  OF  THE  SPOUSES  TURNER,  
WHEN  IT  REFUSED  TO  PAY  THEM  THE  VALUE  OF  THEIR  LSC  SHARES.  
 
HELD:  
  A   stockholder   who   dissents   from   certain   corporate   actions   has   the   right   to   demand   payment   of   the  
fair  value  of  his  or  her  shares.  This  right,  known  as  the  right  of  appraisal,  is  expressly  recognized  in  Section  
81  of  the  Corporation  Code.  Now,  however,  a  corporation  can  purchase  its  own  shares,  provided  payment  is  
made  out  of  surplus  profits  and  the  acquisition  is  for  a  legitimate  corporate  purpose.  
  That  the  respondent  had  indisputably  no  unrestricted  retained  earnings  in  its  books  at  the  time  the  
petitioners   commenced   Civil   Case   No.   01-­‐‑086   on   January   22,   2001   proved   that   the   respondent’s   legal  
obligation  to  pay  the  value  of  the  petitioners’  shares  did  not  yet  arise.  Thus,  the  CA  did  not  err  in  holding  that  
the  petitioners  had  no  cause  of  action,  and  in  ruling  that  the  RTC  did  not  validly  render  the  partial  summary  
judgment.   Neither   did   the   subsequent   existence   of   unrestricted   retained   earnings   after   the   filing   of   the  
complaint  cure  the  lack  of  cause  of  action  in  Civil  Case  No.  01-­‐‑086.  The  petitioners’  right  of  action  could  only  
spring  from  an  existing  cause  of  action.  Thus,  a  complaint  whose  cause  of  action  has  not  yet  accrued  cannot  
be  cured  by  an  amended  or  supplemental  pleading  alleging  the  existence  or  accrual  of  a  cause  of  action  during  
the  pendency  of  the  action.  
 
 

   
3H  A.Y.  2017-­‐2018   188  
 
 

CORPORATION  LAW  CASE  DIGESTS  –  ATTY.  DANTE  DELA  CRUZ  

 
148.  PHILIPPINE  TRUST  COMPANY  V.  MARCIANO  RIVERA  
G.R.  NO.  L-­‐‑19761  /  JANUARY  29,  1923  
STREET,  J.  
 
DOCTRINE:  
 
A  corporation  has  no  power  to  release  an  original  subscriber  to  its  capital  stock  from  the  obligation  of  paying  
for  his  shares,  without  a  valuable  consideration  for  such  release;  and  as  against  creditors  a  reduction  of  the  
capital   stock   can   take   place   only   in   the   manner   an   under   the   conditions   prescribed   by   the   statute   or   the  
charter   or   the   articles   of   incorporation.   Moreover,   strict   compliance   with   the   statutory   regulations   is  
necessary.  
 
FACTS:    
 
In   1918   the   Cooperativa   Naval   Filipina   was   duly   incorporated   under   the   laws   of   the   Philippine   Islands.  
Among  the  incorporators  of  this  company  was  the  defendant  Mariano  Rivera,  who  subscribed  for  450  shares  
representing  a  value  of  P45,000.    
 
In   the   course   of   time   the   company   became   insolvent   and   went   into   the   hands   of   the   Philippine   Trust  
Company,   as   assignee   in   bankruptcy;   and   by   it   this   action   was   instituted   to   recover   one-­‐‑half   of   the   stock  
subscription  of  the  defendant,  which  admittedly  has  never  been  paid.    
 
The   reason   given   for   the   failure   of   the   defendant   to   pay   the   entire   subscription   is,   that   not   long   after  
the   Cooperativa   Naval   Filipina   had   been   incorporated,   a   meeting   of   its   stockholders   occurred,   at   which   a  
resolution  was  adopted  to  the  effect  that  the  capital  should  be  reduced  by  50  per  centum  and  the  subscribers  
released  from  the  obligation  to  pay  any  unpaid  balance  of  their  subscription  in  excess  of  50  per  centum  of  
the   same.   As   a   result   of   this   resolution   it   seems   to   have   been   supposed   that   the   subscription   of   the   various  
shareholders   had   been   cancelled   to   the   extent   stated;   and   fully   paid   certificate   were   issued   to   each  
shareholders  for  one-­‐‑half  of  his  subscription.  It  does  not  appear  that  the  formalities  prescribed  in  section  17  
of  the  Corporation  Law  (Act  No.  1459),  as  amended,  relative  to  the  reduction  of  capital  stock  in  corporations  
were  observed,  and  in  particular  it  does  not  appear  that  any  certificate  was  at  any  time  filed  in  the  Bureau  of  
Commerce  and  Industry,  showing  such  reduction.    
 
The   trial   judge   having   given   judgment   in   favor   of   the   plaintiff   for   the   amount   sued   for,   the   defendant  
appealed.    
 
ISSUE:  
WON  the  defendant  was  still  liable  for  the  unpaid  balance  of  his  subscription.    
 
HELD:  
 
•   It  is  established  doctrine  that  subscription  to  the  capital  of  a  corporation  constitute  a  find  to  which  
creditors   have   a   right   to   look   for   satisfaction   of   their   claims   and   that   the   assignee   in   insolvency   can  
maintain  an  action  upon  any  unpaid  stock  subscription  in  order  to  realize  assets  for  the  payment  of  
its   debts.   (Velasco   vs.   Poizat,   37   Phil.,   802.)   A   corporation   has   no   power   to   release   an   original  
subscriber   to   its   capital   stock   from   the   obligation   of   paying   for   his   shares,   without   a   valuable  
consideration  for  such  release;  and  as  against  creditors  a  reduction  of  the  capital  stock  can  take  place  
only  in  the  manner  an  under  the  conditions  prescribed  by  the  statute  or  the  charter  or  the  articles  of  

   
3H  A.Y.  2017-­‐2018   189  
 
 

CORPORATION  LAW  CASE  DIGESTS  –  ATTY.  DANTE  DELA  CRUZ  

incorporation.  Moreover,  strict  compliance  with  the  statutory  regulations  is  necessary  (14  C.  J.,  498,  
620).    
 
In  the  case  before  us  the  resolution  releasing  the  shareholders  from  their  obligation  to  pay  50  per  
centum  of  their  respective  subscriptions  was  an  attempted  withdrawal  of  so  much  capital  from  the  
fund  upon  which  the  company's  creditors  were  entitled  ultimately  to  rely  and,  having  been  effected  
without  compliance  with  the  statutory  requirements,  was  wholly  ineffectual.    
   
 
 
149.    NO  CASE  
 
 
 
150.  ANTONIO  ESCAÑO  VS.  FILIPINAS  MINING  CORPORATION,  ET  AL.  
G.R.  NO.  L-­‐‑49003  JULY  28,  1944  
OZAETA,  J.:  
 
DOCTRINE:    
The  registration  of  transfers  of  shares  of  stock  upon  the  books  of  the  corporation  is  required  as  a  condition  
precedent  to  their  validity  against  the  corporation  and  third  parties,  is  also  applicable  to  unissued  shares  
held  by  the  corporation  in  escrow  
 
FACTS:    
On   March   8,   1937,   the   plaintiff-­‐‑appellee   obtained   judgment   against   Silverio   Salvosa   whereby   the   latter   was  
ordered  to  transfer  and  deliver  to  the  former  116  active  shares  and  an  undetermined  number  of  shares  in  
escrow  of  the  Filipinas  Mining  Corporation  and  to  pay  the  sum  of  P500  as  damages,  with  the  proviso  that  the  
escrow  shares  shall  be  transferred  and  delivered  to  the  plaintiff  only  after  they  shall  have  been  released  by  
the   company.   A   writ   of   garnishment   was   served   by   the   sheriff   of   Manila   upon   the   Filipinas   Mining  
Corporation  to  satisfy  the  said  judgment;  and  Filipinas  Mining  Corporation  advised  the  sheriff  of  Manila  that  
according  to  its  books  the  judgment  debtor  Silverio  Salvosa  was  the  registered  owner  of  1,000  active  shares  
and   about   21,339   unissued   shares   held   in   escrow   by   the   said   corporation.  The   sheriff   sold   the   1,000   active  
shares  at  public  auction.  
 
It  appears  that  Silverio  Salvosa  sold  to  Jose  P.  Bengzon  all  his  right,  title,  and  interest  in  and  to  18,580  shares  
of  stock  of  the  Filipinas  Mining  Corporation  held  in  escrow  which  the  said  Salvosa  was  entitled  to  receive,  
and   which   Bengzon   in   turn   subsequently   sold   and   transferred   to   Standard   Investment   of   the   Philippines.  
Neither   Salvosa's   sale   to   Bengzon   nor   Bengzon's   sale   to   the   Standard   Investment   of   the   Philippines   was  
notified  to  and  recorded  in  the  books  of  the  Filipinas  Mining  Corporation  more  than  three  years  after  the  
escrow   shares   in   question   were   attached   by   garnishment   served   on   the   Filipinas   Mining   Corporation   as  
hereinbefore  set  forth.  
 
On   January   24,   1941,   the   defendant   Filipinas   Mining   Corporation   issued   in   favor   of   the   defendant   Standard  
Investment  of  the  Philippines  certificate  of  stock  for  the  18,580  shares  formerly  held  in  escrow  by  Silverio  
Salvosa  and  which  had  been  adversely  by  the  present  plaintiff-­‐‑appellee  on  the  one  hand  and  the  Standard  
Investment  of  the  Philippines  on  the  other,  the  first  by  virtue  of  garnishment  proceedings  and  the  second  by  
virtue  of  the  sale  made  to  it  by  Jose  P.  Bengzon  as  aforesaid.  
 
ISSUE:  

   
3H  A.Y.  2017-­‐2018   190  
 
 

CORPORATION  LAW  CASE  DIGESTS  –  ATTY.  DANTE  DELA  CRUZ  

WON  the  issuance  by  the  Filipinas  Mining  Corporation  of  the  said  18,580  shares  of  its  stock  to  the  Standard  
Investment  of  the  Philippines  was  valid  as  against  the  attaching  judgment  creditor  of  the  original  owner.  
 
HELD:    
No.    The  transfer  of  duly  issued  shares  of  stock  is  not  valid  as  against  third  parties  and  the  corporation  until  
it   is   noted   upon   the   books   of   the   corporation.   The   reasons   for   the   registration   are   (1)   to   enable   the  
corporation  to  know  at  all  times  who  its  actual  stockholders  are,  because  mutual  rights  and  obligations  exist  
between  the  corporation  and  its  stockholders;  (2)  to  afford  to  the  corporation  an  opportunity  to  object  or  
refuse  its  consent  to  the  transfer  in  case  it  has  any  claim  against  the  stock  sought  to  be  transferred,  or  for  any  
other  valid  reason;  and  (3)  to  avoid  fictitious  or  fraudulent  transfers.  
 
Moreover,   it   seems   illogical   and   unreasonable   to   hold   that   inactive   or   unissued   shares   still   held   by   the  
corporation   in   escrow   pending   receipt   of   authorization   from   the   Government   to   issue   them,   may   be  
negotiated   or   transferred   unrestrictedly   and   more   freely   than   active   or   issued   shares   evidenced   by  
certificates  of  stock.  
 
We   are,   therefore,   of   the   opinion   and   so   hold   that   section   35   of   the   Corporation   Law,   which   requires   the  
registration  of  transfers  of  shares  stock  upon  the  books  of  the  corporation  as  a  condition  precedent  to  their  
validity   against   the   corporation   and   third   parties,   is   also   applicable   to   unissued   shares   held   by   the  
corporation  in  escrow.  
 
 
 
151.   AQUILINO   RIVERA,   ISAMU   AKASAKO   AND   FUJIYAMA   HOTEL   &   RESTAURANT,  
INC.,   PETITIONERS,   VS.   THE   HON.   ALFREDO   C.   FLORENDO,   AS   JUDGE   OF   THE   COURT   OF   FIRST  
INSTANCE   OF   MANILA   (BRANCH   XXXVI),   LOURDES   JUREIDINI   AND   MILAGROS  
TSUCHIYA,  RESPONDENTS.  
G.R.  NO.  L-­‐‑57586.  OCTOBER  8,  1986.  
PARAS,  J.  
 
Doctrine:   Intracorporate   controversy   would   call   for   the   jurisdiction   of   the   SEC.   An   intra-­‐‑corporate  
controversy  has  been  defined  as  "one  which  arises  between  a  stockholder  and  the  corporate.  There  is  no  
distinction,   qualification,   nor   any   exemption   whatsoever."   This   Court   has   also   ruled   that   cases   of   private  
respondents   who   are   not   shareholders   of   the   corporation,   cannot   be   a   "controversy   arising   out   of  
intracorporate  or  partnership  relations  between  and  among  stockholders,  members  or  associates;  between  
any  or  all  of  them  and  the  corporation,  partnership  or  association,  of  which  they  are  stockholders,  members  
or  associates,  respectively."  
 
Facts:  Petitioner  corporation  is  a  domestic  corporation  with  a  capital  stock  of  P1,000,000.00  divided  into  
10,000  shares  of  P100.00  par  value  each  by  the  herein  petitioner  Rivera  and  4  other  incorporators.  Sometime  
thereafter  Rivera  increased  his  subscription  from  the  original  1,250  to  a  total  of  4899  shares.  
 
Subsequently,  Isamu  Akasako,  a  Japanese  national  and  co-­‐‑petitioner  who  is  allegedly  the  real  owner  of  the  
shares  of  stock  in  the  name  of  petitioner  Rivera,  sold  2550  shares  of  the  same  to  private  respondent  Milagros  
Tsuchiya  for  P440k  with  the  assurance  that  Tsuchiya  will  be  made  the  President  and  Lourdes  Jureidini  a  
director  after  the  purchase.  Rivera  who  was  in  Japan  also  assured  private  respondents  by  overseas  call  that  
he   will   sign   the   stock   certificates   because   Akasako   is   the   real   owner.   However,   after   the   sale   was  
consummated,  Rivera  refused  to  make  the  indorsement  unless  he  is  also  paid.  
 

   
3H  A.Y.  2017-­‐2018   191  
 
 

CORPORATION  LAW  CASE  DIGESTS  –  ATTY.  DANTE  DELA  CRUZ  

It  also  appears  that  the  other  incorporators  sold  their  shares  to  both  respondent  Jureidini  and  Tsuchiya  such  
that  both  respondents  became  the  owners  of  a  total  of  3300  shares  or  the  majority  out  of  5,649  outstanding  
subscribed  shares,  and  that  there  was  no  dispute  as  to  the  legality  of  the  transfer  of  the  stock  certificate  to  
Jureidini,  all  of  which  bear  the  signatures  of  the  president  and  the  secretary  as  required  by  the  Corporation  
Law   with   the   proper   indorsements   of   the   respective   owners   appearing   thereon.   Some   are   specifically  
indorsed  to  her  while  others  are  indorsed  in  blank.  Rivera  admitted  the  genuineness  of  all  the  signatures  of  
the  officers  of  the  corporation  and  of  all  the  indorsee  therein.  
 
Private  respondents  attempted  several  times  to  register  their  stock  certificates  with  the  corporation  but  the  
latter   refused   to   register   the   same.   Thus,   private   respondents   filed   a   special   civil   action   for   mandamus   and  
damages  with  preliminary  mandatory  injunction  and/or  receivership  against  petitioners.  A  hearing  was  held  
on  the  application  for  preliminary  mandatory  injunction  and/or  receivership,  after  which  respondent  Judge  
issued   an   order   for   a   writ   of   preliminary   mandatory   injunction   authorizing   respondent   Jureidini   and  
Tsuchiya  to  manage  the  corporation's  hotel  and  restaurant,  upon  the  filing  of  a  bond  in  the  amount  of  30k.  
Then   through   another   counsel   Atty.   Ignacio   in   collaboration   with   their   counsel   of   record,   Atty.   Bueno,  
petitioners  filed  a  motion  to  dismiss  the  petition  on  the  ground  that  respondent  Judge  has  no  jurisdiction  to  
entertain   the   case,   while   through   Atty.   Bueno,   they   filed   a   motion   for   reconsideration   of   the   Order   granting  
the  issuance  of  a  writ  of  mandatory  preliminary  injunction.  Private  respondents  filed  their  opposition  to  both  
motions  and  respondent  Judge  issued  an  Order  denying  both  the  MR  and  the  motion  to  dismiss  the  petition  
but  increased  the  amount  of  the  bond  to  120k  to  sufficiently  protect  the  interests  of  herein  petitioners.  
 
Issues:  1.  Whether  or  not  the  ordinary  court  has  jurisdiction?  
2.  Whether  or  not  the  certificate  of  stock  was  properly  transferred?  
3.  Whether  of  not  the  mandamus  is  a  proper  course  of  action?  
 
Held:   1.   No.   It   has   already   been   settled   that   an   intracorporate   controversy   would   call   for   the   jurisdiction   of  
the  SEC.  An  intra-­‐‑corporate  controversy  has  been  defined  as  "one  which  arises  between  a  stockholder  and  
the   corporate.   There   is   no   distinction,   qualification,   nor   any   exemption   whatsoever."   This   Court   has   also  
ruled   that   cases   of   private   respondents   who   are   not   shareholders   of   the   corporation,   cannot   be   a  
"controversy   arising   out   of   intracorporate   or   partnership   relations   between   and   among   stockholders,  
members  or  associates;  between  any  or  all  of  them  and  the  corporation,  partnership  or  association,  of  which  
they  are  stockholders,  members  or  associates,  respectively."  
 
The  present  controversy  is  not  an  intracorporate  controversy;  private  respondents  are  not  yet  stockholders;  
they  are  only  seeking  to  be  registered  as  stockholders  because  of  an  alleged  sale  of  shares  of  stock  to  them.  
Therefore,   as   the   petition   is   filed   by   outsiders   not   yet   members   of   the   corporation,   jurisdiction   properly  
belongs  to  the  regular  courts.  
 
2.  No.  Under  "The  Corporation  Code  of  the  Philippines,"  shares  of  stock  are  transferred  as  follow:  
"SEC.  63.  Certificate  of  stock  and  transfer  of  shares.  –  x  x  x  Shares  of  stock  so  issued  are  personal  
property  and  may  be  transferred  by  delivery  of  the  certificate  or  certificates  indorsed  by  the  owner  
or  his  attorney-­‐‑in-­‐‑fact  or  other  person  legally  authorized  to  make  the  transfer.  No  transfer,  however,  
shall   be   valid,   except   as   between   the   parties,   until   the   transfer   is   recorded   in   the   book   of   the  
corporation  showing  the  names  of  the  parties  to  the  transaction,  the  date  of  the  transfer,  the  number  
of  the  certificate  or  certificates  and  the  number  of  shares  transferred.  
 
Shares  of  stock  may  be  transferred  by  delivery  to  the  transferee  of  the  certificate  properly  indorsed.  `Title  
may   be   vested   in   the   transferee   by   delivery   of   the   certificate   with   a   written   assignment   or   indorsement  
thereof.  There  should  be  compliance  with  the  mode  of  transfer  prescribed  by  law.  
 

   
3H  A.Y.  2017-­‐2018   192  
 
 

CORPORATION  LAW  CASE  DIGESTS  –  ATTY.  DANTE  DELA  CRUZ  

3.  No.  Mandamus  will  not  lie  in  the  instant  case  where  the  shares  of  stock  in  question  are  not  even  indorsed  
by   the   registered   owner   Rivera   who   is   specifically   resisting   the   registration   thereof   in   the   books   of   the  
corporation.  Under  the  above  ruling,  even  the  shares  of  stock  which  were  purchased  by  private  respondents  
from  the  other  incorporators  cannot  also  be  the  subject  of  mandamus  on  the  strength  of  mere  indorsement  
of  the  supposed  owners  of  said  shares  in  the  absence  of  express  instructions  from  them.  The  rights  of  the  
parties  will  have  to  be  threshed  out  in  an  ordinary  action.  
 
 
 
152.    NAUTICA  CANNING  CORPORATION,  FIRST  DOMINION  PRIME  HOLDINGS,  INC.  AND  FERNANDO  R.  
ARGUELLES,  JR.  VS.  ROBERTO  C.  YUMUL  
G.R.  NO.  164588.  OCTOBER  19,  2005.  
YNARES-­‐‑SANTIAGO,  J  
 
DOCTRINE:  As  between  the  corporation  on  the  one  hand,  and  its  shareholders  and  third  persons  on  the  other,  
the  corporation  looks  only  to  its  books  for  the  purpose  of  determining  who  its  shareholders  are.    
 
FACTS:  Nautica  Canning  Corporation  (Nautica)  was  organized  and  incorporated  on  May  11,  1994  with  an  
authorized   capital   stock   of   P40,000,000   divided   into   400,000   shares   with   a   par   value   of   P100.00   per   share.  
It  had  a  subscribed  capital  stock  of  P10,000,000.  One  of  it’s  stockholders  is  Alvin  Dee,  having  89,991  number  
of  shares,  making  him  own  the  majority  of  the  outstanding  capital  shares.    
Respondent  Roberto  C.  Yumul  was  appointed  Chief  Operating  Officer/General  Manager  of  Nautica.  On  the  
same  date,  first  Dominion  Prime  Holdings,  Inc.,  Nautica's  parent  company,  through  its  Chairman  Alvin  Y.  Dee,  
granted  Yumul  an  Option  to  Purchase  up  to  15%  of  the  total  stocks  it  subscribed  from  Nautica.    
A   Deed   of   Trust   and   Assignment   was   executed   between   First   Dominion   Prime   Holdings,   Inc.   and   Yumul  
whereby  the  former  assigned  14,999  of  its  subscribed  shares  in  Nautica  to  the  latter.  The  deed  stated  that  
the   14,999   "shares   were   acquired   and   paid   for   in   the   name   of   the   ASSIGNOR   only   for   convenience,   but  
actually  executed  in  behalf  of  and  in  trust  for  the  ASSIGNEE."    
Nautica  declared  a  P35,000,000  cash  dividend,  P8,250,000  of  which  was  paid  to  Yumul  representing  his  15%  
share.    
After  Yumul's  resignation  from  Nautica  ,  he  wrote  a  letter  to  Dee  requesting  the  latter  to  formalize  his  offer  
to  buy  Yumul's  15%  share  in  Nautica  and  demanding  the  issuance  of  the  corresponding  certificate  of  shares  
in   his   name   should   Dee   refuse   to   buy   the   same.   Dee   denied   the   request   claiming   that   Yumul   was   not   a  
stockholder  of  Nautica.    
Yumul   requested   that   the   Deed   of   Trust   and   Assignment   be   recorded   in   the   Stock   and   Transfer   Book   of  
Nautica,  and  that  he,  as  a  stockholder,  be  allowed  to  inspect  its  books  and  records    
Yumul's  requests  were  denied  allegedly  because  he  neither  exercised  the  option  to  purchase  the  shares  nor  
paid  for  the  acquisition  price  of  the  14,999  shares.  Atty.  Arguelles  maintained  that  the  cash  dividend  received  
by  Yumul  is  held  by  him  only  in  trust  for  First  Dominion  Prime  Holdings,  Inc.    
Yumul  filed  before  the  SEC  a  petition  for  mandamus  with  damages,  with  prayer  that  the  Deed  of  Trust  and  
Assignment   be   recorded   in   the   Stock   and   Transfer   Book   of   Nautica   and   that   the   certificate   of   stocks  
corresponding  thereto  be  issued  in  his  name.    
SEC  en  banc  ruled  n  favor  of  Yumul.    
 
ISSUE:  Whether  or  not  the  Yumul  may  inspect  the  stock  and  transfer  book  
 
HELD:   YES.   Yumul   is   found   to   be   a   stockholder   of   Nautica,   of   one   share   of   stock   recorded   in   Yumul's   name,  
although   allegedly   held   in   trust   for   Dee.   Nautica's   Articles   of   Incorporation   and   By-­‐‑laws,   as   well   as   the  
General  Information  Sheet  filled  with  the  SEC  indicated  that  Yumul  was  an  incorporator  and  subscriber  of  
one  share.  Even  granting  that  there  was  an  agreement  between  Yumul  and  Dee  whereby  the  former  is  holding  

   
3H  A.Y.  2017-­‐2018   193  
 
 

CORPORATION  LAW  CASE  DIGESTS  –  ATTY.  DANTE  DELA  CRUZ  

the  share  in  trust  for  Dee,  the  same  is  binding  only  as  between  them.  From  the  corporation's  vantage  point,  
Yumul   is   its   stockholder   with   one   share,   considering   that   there   is   no   showing   that   Yumul   transferred   his  
subscription  to  Dee,  the  alleged  real  owner  of  the  share,  after  Nautica's  incorporation.      
Indeed,  it  is  possible  for  a  business  to  be  wholly  owned  by  one  individual.  The  validity  of  its  incorporation  is  
not  affected  when  such  individual  gives  nominal  ownership  of  only  one  share  of  stock  to  each  of  the  other  
four  incorporators.  This  is  not  necessarily  illegal.  But,  this  is  valid  only  between  or  among  the  incorporators  
privy  to  the  agreement.  It  does  bind  the  corporation  which,  at  the  time  the  agreement   is  made,  was  non-­‐‑
existent.   Thus,   incorporators   continue   to   be   stockholders   of   a   corporation   unless,   subsequent   to   the  
incorporation,  they  have  validly  transferred  their  subscriptions  to  the  real  parties  in  interest.  As  between  the  
corporation  on  the  one  hand,  and  its  shareholders  and  third  persons  on  the  other,  the  corporation  looks  only  
to  its  books  for  the  purpose  of  determining  who  its  shareholders  are    
The   conduct   of   the   parties   also   constitute   sufficient   proof   of   Yumul's   status   as   a   stockholder.   Yumul   was  
elected   during   the   regular   annual   stockholders'   meeting   as   a   Director   of   Nautica's   Board   of   Directors.  
Thereafter,   he   was   elected   as   president   of   Nautica.Thus,   Nautica   and   its   stockholders   knowingly   held  
respondent  out  to  the  public  as  an  officer  and  a  stockholder  of  the  corporation.    Section  23  of    the  Corporation  
Code  requires  that  every  director  must  own  at  least  one  share  of  the  capital  stock  of  the  corporation  of  which  
he  is  a  director.  Before  one  may  be  elected  president  of  the  corporation,  he  must  be  a  director.  Since  Yumul  
was  elected  as  Nautica's  Director  and  as  President  thereof,  it  follows  that  he  must  have  owned  at  least  one  
share  of  the  corporation's  capital  stock.    
 
NOTE:   There   was   another   issue   regarding   whether   or   not   Yumil   is   the   beneficial   owner   of   14,999   shares   of  
Nautica.  (WON  Yumul  may  compel  the  corporate  secretary  to  record  said  deed)  
 Petitioners  allege  that  Yumul  was  given  the  option  to  purchase  shares  of  stocks  in  Nautica  under  the  Option  
to  Purchase.  However,  he  failed  to  exercise  the  option,  thus  there  was  no  cause  or  consideration  for  the  Deed  
of  Trust  and  Assignment,  which  makes  it  void  for  being  simulated  or  fictitious.    
The  Court  ruled  that  neither  the  SEC  nor  CA  ruled  upon  this  issue.  SC  said  that  when  the  controversy  involves  
matters  purely  civil  in  character,  (in  this  case,  the  alleged  simulated  contract)  it  is  beyond  the  ambit  of  the  
limited  jurisdiction  of  the  SEC.  the  better  policy  in  determining  which  body  has  jurisdiction  over  a  case  would  
be  to  consider  not  only  the  status  or  relationship  of  the  parties,  but  also  the  nature  of  the  question  that  is  the    
subject  of  their  controversy.  This,  however,  is  now  moot  and  academic  due  to  the  passage  of  RA  8799  or  The  
Securities   Regulation   Code   which   took   effect   on   August   8,   2000.   The   Act   transferred   from   the   SEC   to   the  
regional  trial  court  jurisdiction  over  cases  involving  intra-­‐‑corporate  disputes.    
Considering   that   the   issue   of   the   validity   of   the   Deed   of   Trust   and   Assignment   is   civil   in   nature,   thus,   under  
the   competence   of   the   regular   courts,   and   the   failure   of   the   SEC   and   the   Court   of   Appeals   to   make   a  
determinative  finding  as  to  its  validity,  we  are  constrained  to  refrain  from  ruling  on  whether  or  not  Yumul  
can   compel   the   corporate   secretary   to   register   said   deed.   It   is   only   after   an   appropriate   case   is   filed   and  
decision  rendered  thereon  by  the  proper  forum  can  the  issue  be  resolved.  (In  short,  hindi  nag  rule  ang  SC  
with  regard  this  issue  cos  ang  dapat  mag  file  dapat  sa  trial  court  ng  appropriate  action)    
 
 
 
153.  ENRIQUE  RAZON  VS.  IAC  AND  VICENTE  B.  CHUIDIAN  
G.R.  NO.  74306.  MARCH  16,  1992.  
GUTIERREZ,  JR.,  J.;  
 
DOCTRINE:    
For  an  effective  transfer  of  shares  of  stock  the  mode  and  manner  of  transfer  as  prescribed  by  law  must  be  
followed.  Shares  of  stock  may  be  transferred  by  delivery  to  the  transferee  of  the  certificate  properly  indorsed.  
Title  may  be  vested  in  the  transferee  by  the  delivery  of  the  duly  indorsed  certificate  of  stock.  Indorsement  of  
the  certificate  of  stock  is  a  mandatory  requirement  of  law  for  an  effective  transfer  of  a  certificate  of  stock.    

   
3H  A.Y.  2017-­‐2018   194  
 
 

CORPORATION  LAW  CASE  DIGESTS  –  ATTY.  DANTE  DELA  CRUZ  

 
FACTS:    
E.   Razon,   Inc.   was   organized   in   1962   by   petitioner   Enrique   Razon   for   the   purpose   of   participating   in   the  
bidding  for  the  arrastre  services  in  South  Harbor,  Manila.  The  business,  however,  did  not  start  operations  
until  1966.  According  to  the  petitioner,  some  of  the  incorporators  withdrew  from  the  said  corporation.  The  
petitioner   then   distributed   the   stocks   previously   placed   in   the   names   of   the   withdrawing   nominal  
incorporators  to  some  friends,  among  them  the  late  Juan  T.  Chuidian  to  whom  he  gave  1,500  shares  of  stock.  
The   shares   of   stock   were   registered   in   the   name   of   Chuidian   only   as   nominal   stockholder   and   with   the  
agreement   that   the   said   shares   of   stock   were   owned   and   held   by   the   petitioner   but   Chuidian   was   given   the  
option   to   buy   the   same.   In   view   of   this   arrangement,   Chuidian   in   1966   delivered   to   the   petitioner   the   stock  
certificate  covering  the  1,500  shares  of  stock  of  E.  Razon,  Inc.  Since  then,  the  petitioner  had  in  his  possession  
the  certificate  of  stock  until  the  time  he  delivered  it  for  deposit  with  the  Philippine  Bank  of  Commerce  under  
the  parties'  joint  custody  pursuant  to  their  agreement.  
 
In   his   complaint   filed   on   June   29,   1971,   and   amended   on   November   16,   1971,   Vicente   B.   Chuidian,   in   his  
capacity   as   Administrator   of   the   Estate   of   the   Deceased   Juan   T.   Chuidian,   prayed   for   the   delivery   of  
certificates  of  stocks  representing  the  share  holdings  of  the  deceased  Juan  T.  Chuidian  in  the  E.  Razon,  Inc.  
 
ISSUE:  
Whether  petitioner’s  oral  testimony  as  regards  the  true  nature  of  his  agreement  with  the  late  Juan  Chuidian  
on  the  1,500  shares  of  stock  of  E.  Razon,  Inc.  is  sufficient  to  prove  his  ownership  over  the  said  1,500  shares  
of  stock.    
 
HELD:  
NO.  There  is  no  dispute  the  questioned  1,5000  shares  of  stock  of  E.  Razon,  Inc.  are  in  the  name  of  the  late  
Juan  Chuidian  in  the  books  of  the  corporation.  Moreover,  the  records  show  that  during  his  lifetime  Chuidian  
was   elected   member   of   the   Board   of   Directors   of   the   corporation   which   clearly   shows   that   he   was   a  
stockholder  of  the  corporation.  From  the  point  of  view  of  the  corporation,  therefore,  Chuidian  was  the  owner  
of  the  1,500  shares  of  stock.  In  such  a  case,  the  petitioner  who  claims  ownership  over  the  questioned  shares  
of   stock   must   show   that   the   same   were   transferred   to   him   by   proving   that   all   the   requirements   for   the  
effective  transfer  of  shares  of  stock  in  accordance  with  the  corporation's  by  laws,  if  any,  were  followed  or  in  
accordance  with  the  provisions  of  law.  The  petitioner  failed  in  both  instances.  The  petitioner  did  not  present  
any   by-­‐‑laws   which   could   show   that   the   1,500   shares   of   stock   were   effectively   transferred   to   him.   In   the  
absence   of   the   corporation's   by-­‐‑laws   or   rules   governing   effective   transfer   of   shares   of   stock,   the   provisions  
of   the   Corporation   Law   are   made   applicable   to   the   instant   case.   The   law   is   clear   that   in   order   for   a   transfer  
of  stock  certificate  to  be  effective,  the  certificate  must  be  properly  indorsed  and  that  title  to  such  certificate  
of  stock  is  vested  in  the  transferee  by  the  delivery  of  the  duly  indorsed  certificate  of  stock.  Since  the  certificate  
of  stock  covering  the  questioned  1,5000  shares  of  stock  registered  in  the  name  of  the  late  Juan  Chuidian  was  
never  indorsed  to  the  petitioner,  the  inevitable  conclusion  is  that  the  questioned  shares  of  stock  belong  to  
Chuidian.   To   reiterate,   indorsement   of   the   certificate   of   stock   is   a   mandatory   requirement   of   law   for   an  
effective  transfer  of  a  certificate  of  stock.    
LLjur    
Moreover,  the  preponderance  of  evidence  supports  the  appellate  court's  factual  findings  that  the  shares  of  
stock  were  given  to  Juan  T.  Chuidian  for  value.  Juan  T.  Chuidian  was  the  legal  counsel  who  handled  the  legal  
affairs  of  the  corporation.  We  give  credence  to  the  testimony  of  the  private  respondent  that  the  shares  of  
stock  were  given  to  Juan  T.  Chuidian  in  payment  of  his  legal  services  to  the  corporation.  Petitioner  Razon  
failed  to  overcome  this  testimony.    
 
 
 

   
3H  A.Y.  2017-­‐2018   195  
 
 

CORPORATION  LAW  CASE  DIGESTS  –  ATTY.  DANTE  DELA  CRUZ  

154.  RURAL  BANK  OF  SALINAS  V.  CA  


G.R.  NO.  96674   JUNE  26,  1992  
PARAS,  J.;  
 
FACTS:  
-­‐‑   Clemente   G.   Guerrero,   President   of   the   Rural   Bank   of   Salinas,   Inc.,   executed   a   Special   Power   of  
Attorney  in  favor  of  his  wife,  private  respondent  Melania  Guerrero,  giving  and  granting  the  latter  full  
power  and  authority  to  sell  or  otherwise  dispose  of  and/or  mortgage  473  shares  of  stock  of  the  Bank  
registered  in  his  name.  
-­‐‑   Melania   Guerrero   presented   to   petitioner   Rural   Bank   of   Salinas   the   2   Deeds   of   Assignment   for  
registration   with   a   request   for   the   transfer   in   the   Bank's   stock   and   transfer   book   shares   of   stock   so  
assigned,  the  cancellation  of  stock  certificates  in  the  name  of  Clemente  G.  Guerrero,  and  the  issuance  
of  new  stock  certificates  covering  the  transfer  in  the  name  of  the  new  owners  thereof.    
-­‐‑   However,  petitioner  Bank  denied  the  request  of  respondent  Melania  Guerrero.  
-­‐‑   Guerrero  filed  with  the  Securities  and  Exchange  Commission"  (SEC)  an  action  for  mandamus  against  
petitioners  Rural  Bank  of  Salinas,  its  President  and  Corporate  Secretary.  
-­‐‑   SEC  En  Banc  affirmed  the  decision  of  the  Hearing  Officer.    
-­‐‑   Petitioner  filed  a  petition  for  review  with  the  Court  of  Appeals  but  said  Court  likewise  affirmed  the  
decision  of  the  SEC.  
 
ISSUE:  Whether  SEC  has  jurisdiction  over  the  instant  case  and  whether  the  corporation  is  obliged  to  register  
the  transfer.  
 
HELD:  
•   We  rule  in  favor  of  the  respondents.  
•   Section  5  (b)  of  P.D.  No.  902-­‐‑A  grants  to  the  SEC  the  original  and  exclusive  jurisdiction  to  hear  and  
decide  cases  involving  intra-­‐‑corporate  controversies.    
•   An  intra-­‐‑corporate  controversy  has  been  defined  as  one  that  arises  between  a  stockholder  and  the  
corporation.    
•   The   case   at   bar   involves   shares   of   stock,   their   registration,   cancellation   and   issuances   thereof   by  
petitioner  Rural  Bank  of  Salinas.    
•   It  is  therefore  within  the  power  of  respondent  SEC  to  adjudicate.  
•   Sec.   63   of   the   Corporation   Code   contemplates   no   restriction   as   to   whom   the   stocks   may   be  
transferred.  
•   The  owner  of  shares,  as  owner  of  personal  property,  is  at  liberty,  under  said  section  to  dispose  them  
in  favor  of  whomever  he  pleases,  without  limitation  in  this  respect,  than  the  general  provisions  of  
law.  
•   Corporation's  obligation  to  register  is  ministerial.  
•   In  transferring  stock,  the  secretary  of  a  corporation  acts  in  purely  ministerial  capacity,  and  does  not  
try  to  decide  the  question  of  ownership.  
 
 
 
155.   JOSEFA   SANTAMARIA,   ASSISTED   BY   HER   HUSBAND,   FRANCISCO   SANTAMARIA,   JR.     VS.   THE  
HONGKONG  AND  SHANGHAI  BANKING  CORPORATION  AND  R.  W.  TAPLIN  
G.R.  NO.  L-­‐‑2808                          AUGUST  31,  1951  
BAUTISTA  ANGELO,  J.;  
   
DOCTRINE:  

   
3H  A.Y.  2017-­‐2018   196  
 
 

CORPORATION  LAW  CASE  DIGESTS  –  ATTY.  DANTE  DELA  CRUZ  

It   is   a   well-­‐‑known   rule   that   a   bona   fide   pledgee   or   transferee   of   a   stock   from   the   apparent   owner   is   not  
chargeable   with   knowledge   of   the   limitations   placed   on   it   by   the   real   owner,   or   of   any   secret   agreement  
relating  to  the  use  which  might  be  made  of  the  stock  by  the  holder  
It  is  a  well-­‐‑known  practice  that  a  certificate  of  stock,  indorsed  in  blank,  deemed  quasi  negotiable,  and  as  such  
the  transferee  thereof  is  justified  in  believing  that  it  belongs  to  the  holder  and  transferor.  
FACTS:  
Mrs.   Josefa   T.   Santamaria   bought   10,000   shares   of   the   Batangas   Minerals,   Inc.,   through,   Woo,   Uy-­‐‑Tioco   &  
Naftaly  ,  a  stock  brokerage  firm  which  issued  the  stock  certificate  (Cert  no.  517)  in  its  name  and  indorsed  in  
blank  by  this  firm.  Mrs.  Santamaria  purchased  10,000  shares  of  the  Crown  Mines,  Inc.  with  R.J.  Campos  &  Co.,  
a  brokerage  firm,  and  delivered  Cert  No.  517  to  the  latter  as  security  therefor  with  the  understanding  that  
said   certificate   would   be   returned   to   her   upon   payment   of   the   shares.   When   the   certificate   of   stock   was  
delivered  to  R.J.  Campos  &  Co.,  Inc.,  the  manager  thereof,  Cosculluela,  wrote  in  pencil  on  the  right  margin  the  
name  of  Josefa  T.  Santamaria.  Cert  517  came  into  possession  of  the  HSBC  because  R.J.  Campos  &  Co.,  Inc.  had  
opened  an  overdraft  account  with  this  bank  and  to  this  effect  it  had  executed  a  document  of  hypothecation,  
by  the  term  of  which  R.J.  Campos  &  Co.,  Inc.  pledged  its  stocks  to  said  bank.  Upon  request  by  HSBC,  Batangas  
Minerals,  Inc.  issued  a  new  certificate  (Cert  no.  715)  in  the  name  of  R.W.  Taplin  as  trustee  and  nominee  of  the  
banking  corporation  thereby  cancelling  Cert  517.  According  to  Mrs.  Santamaria,  she  made  the  claim  to  the  
bank   for   her   certificate,   through   Taplin,   the   bank's   representative.   When   R.J   Campos   &   Co.   was   declared  
insolvent,  the  10,000  shares  of  Batangas  Minerals,  Inc.,  was  sold  to  the  same  bank.  R.J.  Campos,  the  president  
of  R.J.  Campos  &  Co.,  Inc.,  was  convicted  with  estafa  and  was  ordered  to  pay  Mrs.  Josefa  Santamaria,  in  the  
amount   of   P8,041.20   representing   the   value   of   the   10,000   shares   of   Batangas   Minerals,   Inc.   When   Mrs.  
Santamaria  failed  in  her  efforts  to  force  the  civil  judgment  rendered  in  her  favor  in  the  criminal  case  because  
the  accused  became  insolvent,  she  filed  her  complaint  in  this  case.    
   
ISSUE:  
1.  Whether  or  not  the  plaintiff-­‐‑appellee  was  chargeable  with  negligence  in  the  transaction  which  gave  rise  to  
this  case.  
2.  Whether  or  not  it  was  the  obligation  of  the  bank  to  have  inquired  into  the  ownership  of  the  certificate  
when  it  received  it  from  R.J.  Campos  &  Company  and  in  concluding  that  the  bank  was  negligent  for  not  having  
done  so.  
   
HELD:  
1.  Certificate  of  stock  No.  517  was  made  out  in  the  name  of  Wo,  Uy-­‐‑Tioco  &  Naftaly,  brokers,  and  was  duly  
indorsed  in  blank  by  said  brokers.  Plaintiff  did  not  take  any  precaution  to  protect  herself  against  the  possible  
misuse  of  the  shares  represented  by  the  certificate  of  stock.  There  is  no  question  that,  in  this  case,  plaintiff  
made  the  negotiation  of  the  certificate  of  stock  to  other  parties  possible  and  the  confidence  she  placed  in  R.J.  
Campos   &   Co.,   Inc.   made   the   wrong   done   possible.   This   was   the   proximate   cause   of   the   damage   suffered   by  
her.  She  is,  therefore,  estopped  from  claiming  further  title  to  or  interest  therein  as  against  a  bona  fide  pledge  
or  transferee  thereof,  for  it  is  a  well-­‐‑known  rule  that  a  bona  fide  pledgee  or  transferee  of  a  stock  from  the  
apparent  owner  is  not  chargeable  with  knowledge  of  the  limitations  placed  on  it  by  the  real  owner,  or  of  any  
secret  agreement  relating  to  the  use  which  might  be  made  of  the  stock  by  the  holder.  The  rule  is  "where  one  
of  two  innocent  parties  must  suffer  by  reason  of  a  wrongful  or  unauthorized  act,  the  loss  must  fall  on  the  one  
who  first  trusted  the  wrong  doer  and  put  in  his  hands  the  means  of  inflicting  such  loss".  It  is  therefore  clear  
that  plaintiff,  in  failing  to  take  the  necessary  precautions  upon  delivering  the  certificate  of  stock  to  her  broker,  
was  chargeable  with  negligence  in  the  transaction  which  resulted  to  her  own  prejudice,  and  as  such,  she  is  
estopped  from  asserting  title  to  it  as  against  the  defendant  Bank.  
   
2.  The  certificate  was  duly  indorsed  in  blank  which  is  known  as  street  certificate.  Upon  its  face,  the  holder  
was  entitled  to  demand  its  transfer  into  his  name  from  the  issuing  corporation.  The  Bank  was  not  obligated  
to   look   beyond   the   certificate   to   ascertain   the   ownership   of   the   stock   at   the   time   it   received   the   same   from  

   
3H  A.Y.  2017-­‐2018   197  
 
 

CORPORATION  LAW  CASE  DIGESTS  –  ATTY.  DANTE  DELA  CRUZ  

R.J.  Campos  &  Co.,  Inc.  Moreover,  it  is  a  well-­‐‑known  practice  that  a  certificate  of  stock,  indorsed  in  blank,  
deemed  quasi  negotiable,  and  as  such  the  transferee  thereof  is  justified  in  believing  that  it  belongs  to  the  
holder  and  transferor.  Even  assuming  that  what  plaintiff  has  stated  to  Robert  Taplin  is  true,  such  an  incident  
would  merely  show  that  plaintiff  has  an  adverse  claim  to  the  ownership  of  said  certificate  of  stock.  A  mere  
claim  and  of  ownership  does  not  establish  the  fact  of  ownership.  The  right  of  the  plaintiff  in  such  a  case  would  
be  against  the  transferor.  The  fact  that  on  the  right  margin  of  the  said  certificate  the  name  of  the  plaintiff  
appeared  written,  granting  it  to  be  true,  can  not  be  considered  sufficient  reason  to  indicate  that  its  owner  
was  the  plaintiff.  Said  indicium  could  at  best  give  the  impression  that  the  plaintiff  was  the  original  holder  of  
the  certificate.    
 
 
 
156.   APOLINARIO   G.   DE   LOS   SANTOS   AND   ISABELO   ASTRAQUILLO   VS.   J.   HOWARD   MCGRATH  
ATTORNEY   GENERAL   OF   THE   UNITED   STATES,   SUCCESSOR   TO   THE   PHILIPPINE   ALIEN   PROPERTY  
ADMINISTRATION  OF  THE  UNITED  STATES,  REPUBLIC  OF  THE  PHILIPPINES  
G.R.  NO.  L-­‐‑4818                          FEBRUARY  28,  1955  
J.  CONCEPCION    
 
DOCTRINE:   a   share   of   stock   may   be   transferred   by   endorsement   of   the   corresponding   stock   certificate,  
coupled  with  its  delivery.  However,  the  transfer  shall  "not  be  valid,  except  as  between  the  parties,"  until  it  is  
"entered  and  noted  upon  the  books  of  the  corporation."  no  such  entry  in  the  name  of  the  plaintiffs  herein  
having  been  made  
 
FACTS:    This  action  involves  the  title  to  1,600,000  shares  of  stock  of  the  Lepanto  Consolidated  Mining  Co.,  
Inc.,  a  corporation  duly  organized  and  existing  under  the  laws  of  the  Philippines.  Originally,  one-­‐‑half  of  said  
shares   of   stock   were   claimed   by   plaintiff,   Apolinario   de   los   Santos,   and   the   other   half,   by   his   co-­‐‑plaintiff  
Isabelo   Astraquillo.   During   the   pendency   of   this   case,   the   latter   has   allegedly   conveyed   and   assigned   his  
interest   in   and   to   said   half   claimed   by   him   to   the   former.   The   shares   of   stock   in   question   are   covered   by  
several  stock  certificates  issued  in  favor  of  Vicente  Madrigal,  who  is  registered  in  the  books  of  the  Lepanto  
as  owner  of  said  stocks  and  whose  indorsement  in  blank  appears  on  the  back  of  said  certificates,  all  of  which,  
except  certificates  No.  2279  —  marked  Exhibit  2  —  covering  55,000  shares,  are  in  plaintiffs'  possession.  
 
Plaintiffs  contend  that  De  los  Santos  bought  55,000  shares  from  Juan  Campos,  in  Manila,  early  in  December,  
1942;  that  he  bought  300,000  shares  from  Carl  Hess,  in  the  same  city,  several  days  later;  and  that,  before  
Christmas   of   1942,   be   bought   800,000   shares   from   Carl   Hess,   this   time   for   the   account   and   benefit   of  
Astraquillo.   By   virtue   of   vesting   P-­‐‑12,   dated   February   18,   1945,   title   to   the   1,600,000   shares   of   stock   in  
dispute   was,   however,   vested   in   the   Alien   Property   Custodian   of   the   U.   S.   as   Japanese   property.   Hence,  
plaintiffs  filed  their  respective  claims  with  the  Property  Custodian.  In  due  course,  the  Vested  Property  Claims  
Committee  of  the  Philippine  Alien  Property  Administration  made  a  "determination,"  dated  March  9,  1948,  
allowing  said  claims,  which  were  considered  and  heard  jointly  as  Claim  No.  535,  but,  upon  personal  review,  
the   Philippine   Alien   Property   Administration   made   by   said   Committee   and   decreed   that   "title   to   the   shares  
in  question  shall  remain  in  the  name  of  the  Philippine  Alien  Property  Administrator."  
 
The   defendant   herein   is   the   Attorney   General   of   the   U.   S.,   successor   to   the   "Administrator".   He   contends,  
substantially,  that,  prior  to  the  outbreak  of  the  war  in  the  Pacific,  said  shares  of  stock  were  bought  by  Vicente  
Madrigal,  in  trust  for,  and  for  the  benefit  of,  the  Mitsui  Bussan  Kaisha,  a  corporation  organized  in  accordance  
with  the  laws  of  Japan,  the  true  owner  thereof,  with  branch  office  in  the  Philippines;  that  on  or  before  March,  
1942,  Madrigal  delivered  the  corresponding  stock  certificates,  with  his  blank  indorsement  thereon,  to  the  
Mitsuis,  which  kept  said  certificates,  in  the  files  of  its  office  in  Manila,  until  the  liberation  of  the  latter  by  the  
American  forces  early  in  1945;  that  the  Mitsuis  had  never  sold,  or  otherwise  disposed  of,  said  shares  of  stock;  

   
3H  A.Y.  2017-­‐2018   198  
 
 

CORPORATION  LAW  CASE  DIGESTS  –  ATTY.  DANTE  DELA  CRUZ  

and   that   the   stock   certificates   aforementioned   must   have   been   stolen   or   looted,   therefore,   during   the  
emergency  resulting  from  said  liberation.  
 
After  due  hearing,  the  Court  of  First  Instance  of  Manila,  presided  over  by  Honorable  Higinio  B.  Macadaeg,  
Judge,  rendered  a  decision  the  dispositive  part  of  which  reads,  as  follows:  
 
In  view  of  the  foregoing  consideration,  judgment  is  hereby  rendered  in  favor  of  the  plaintiffs  and  against  the  
defendant,   declaring   the   former   the   absolute   owners   of   the   shares   of   stock   of   the   Lepanto   consolidated  
Mining  Company,  covered  by  the  certificates  of  stock,  respectively,  in  their  (plaintiffs')  possession.  
 
 
 
ISSUE:  whether  or  not  plaintiffs  had  purchased  the  shares  of  stock  in  question  
 
 
HELD:    Even,  however,  if  Juan  Campos  and  Carl  Hess  had  sold  the  shares  of  stock  in  question,  as  testified  to  
by  De  los  Santos,  the  result,  insofar  as  plaintiffs  are  concerned,  would  be  the  same.  It  is  not  disputed  that  said  
shares  of  stock  were  registered,  in  the  records  of  the  Lepanto,  in  the  name  of  Vicente  Madrigal.  Neither  it  is  
denied   that   the   latter   was,   as   regards   said   shares   of   stock,   a   mere   trustee   for   the   benefit   of   the   Mitsuis.   The  
record  shows  that  Madrigal  had  never  disposed  of  said  shares  of  stock  in  any  manner  whatsoever,  except  by  
turning  over  the  corresponding  stock  certificates,  late  in  1941,  to  the  Mitsuis,  the  beneficial  and  true  owners  
thereof.   It   has,   moreover,   been   established   by   the   uncontradicted   testimony   of   Kitajima   and   Miwa,   the  
managers   of   the   Mitsuis   in   the   Philippines,   from   1941   to   1945,   that   the   Mitsuis   had   neither   sold,   conveyed,  
or  alienated  said  shares  of  stock,  nor  delivered  the  aforementioned  stock  certificates,  to  anybody  during  said  
period.  Section  35  of  the  Corporation  Law  reads:  
 
The   capital   stock   corporations   shall   be   divided   into   shares   for   which   certificates   signed   by   the   president   or  
the  vice-­‐‑president,  countersigned  by  the  secretary  or  clerk  and  sealed  with  the  seal  of  the  corporation,  shall  
be   issued   in   accordance   with   the   by-­‐‑laws.   Shares   of   stock   so   issued   are   personal   property   and   may   be  
transferred  by  delivery  of  the  certificate  endorsed  by  the  owner  or  his  attorney  in  fact  or  other  person  legally  
authorized  to  make  the  transfer.  No  transfer,  however,  shall  be  valid,  except  as  between  the  parties,  until  the  
transfer  is  entered  and  noted  upon  the  books  of  the  corporation  so  as  to  show  the  names  of  the  parties  to  the  
transaction,  the  date  of  the  transfer,  the  number  of  the  certificate,  and  the  number  of  shares  transferred.  
 
No  shares  of  stock  against  which  the  corporation  holds  any  unpaid  claim  shall  be  transferable  on  the  books  
of  the  corporation.  (Emphasis  supplied.)  
 
Pursuant   to   this   provision,   a   share   of   stock   may   be   transferred   by   endorsement   of   the   corresponding   stock  
certificate,  coupled  with  its  delivery.  However,  the  transfer  shall  "not  be  valid,  except  as  between  the  parties,"  
until  it  is  "entered  and  noted  upon  the  books  of  the  corporation."  no  such  entry  in  the  name  of  the  plaintiffs  
herein  having  been  made,  it  follows  that  the  transfer  allegedly  effected  by  Juan  Campos  and  Carl  Hess  in  their  
favor   is   "not   valid,   except   as   between"   themselves.   It   does   not   bind   either   Madrigal   or   the   Mitsuis,   who   are  
not  parties  to  said  alleged  transaction.  What  is  more,  the  same  is  "not  valid,"  or,  in  the  words  of  the  Supreme  
Court  of  Wisconsin  "absolutely  void"  and,  hence,  as  good  as  non-­‐‑existent,  insofar  as  Madrigal  and  the  Mitsuis  
are  concerned.  For  this  reason,  although  a  stock  certificate  is  sometimes  regarded  as  quasi-­‐‑negotiable,  in  the  
sense  that  it  may  be  transferred  by  endorsement,  coupled  with  delivery,  it  is  well  settled  that  the  instrument  
is   non-­‐‑negotiable,   because   the   holder   thereof   takes   it   without   prejudice   to   such   rights   or   defenses   as   the  
registered   owner   or   creditor   may   have   under  the   law,   except   insofar   as   such   rights   or   defenses   are   subject  
to  the  limitations  imposed  by  the  principles  governing  estoppel.  
 

   
3H  A.Y.  2017-­‐2018   199  
 
 

CORPORATION  LAW  CASE  DIGESTS  –  ATTY.  DANTE  DELA  CRUZ  

In  the  case  at  bar,  neither  Madrigal  nor  the  Mitsuis  had  alienated  shares  of  stock  in  question.  It  is  not  even  
claimed  that  either  had,  through  negligence,  given  —  occasion  for  an  improper  or  irregular  disposition  of  the  
corresponding  stock  certificates.  Plaintiffs  merely  argue  without  any  evidence  whatsoever  thereon  —  that  
Kitajima  might  have,  or  must  have,  assigned  the  certificates  on  or  before  December  1942,  this  is,  not  only,  
improbable,   under   the   conditions,   then   obtaining,   but,   also.,   impossible,   considering   that,   in   April   1943,  
Kitajima  delivered  the  instruments  to  Miwa,  who  kept  them  in  its  possession  until  1945.  At  any  rate,  such  
assignment  by  Miwa  —  granting  for  the  sake  of  argument  the  accuracy  of  the  surmise  of  plaintiffs  herein  —  
was  unauthorized  by  the  mitsuis,  who,  in  the  light  of  the  precedents  cited  above,  are  not  chargeable  with  
negligence.  In  other  words,  assuming  that  Kitajima  had  been  guilty  of  embezzlement,  by  negotiating  the  stock  
certificates   in   question   for   his   personal   benefit,   as   claimed   by   the   plaintiffs,   the   title   of   his   assignees   and  
successors   in   interest   would   still   be   subject   to   the   rights   of   the   registered   owner,   namely,   Madrigal,   and  
consequently,   of   the   party   for   whose   benefit   and   account   the   latter   held   the   corresponding   shares   of   stock,  
that  is  to  say,  the  Mitsuis.  
At  any  rate,  at  the  time  of  the  alleged  sales  in  their  favor,  plaintiffs  were  aware  of  sufficient  facts  to  put  them  
on  notice  of  the  need  of  inquiring  into  the  regularity  of  the  transactions  and  the  title  of  the  supposed  vendors.  
Indeed,   the  certificates   of  stock  in   question  were  in   the  name   of  madrigal.  Obviously,   therefore,  the  alleged  
sellers  (Campos  and  Hess)  were  not  registered  owners  of  the  corresponding  shares  of  stock.  Being  presumed  
to   know   the   law   —   particularly   the   provisions   of   section   35   of   Act   No.   1459   —   and,   as   experienced   traders  
in  shares  of  stock,  plaintiffs  must  have,  accordingly,  been  conscious  of  the  consequent  infirmities  in  the  title  
of   the   supposed   vendors,   or   of   the   handicaps   thereof.   Moreover,   the   aforementioned   sales   were   admittedly  
hostile  to  the  Japanese,  who  had  prohibited  it  and  plaintiffs  had  actual  knowledge  of  these  facts  and  of  the  
risks  attendant  to  the  alleged  transaction.  In  other  words,  plaintiffs  advisedly  assumed  those  risks  and,  hence,  
they  can  not  validly  claim,  against  the  registered  stockholder,  the  status  of  purchasers  in  good  faith.  
 
 
 
157.  CHUA  GUAN  VS.  SAMAHANG  MAGSASAKA,  INC.  
G.R.  NO.  L-­‐‑42091                          NOVEMBER  2,  1935  
BUTTE,  J.:  
   
DOCTRINE:  It  is  to  be  noted,  however,  that  section  35  of  the  Corporation  Law  (Act  No.  1459)  enacts  that  
shares  of  stock  "may  be  transferred  by  delivery  of  the  certificate  endorsed  by  the  owner  or  his  attorney  in  
fact  or  other  person  legally  authorized  to  make  the  transfer."  The  use  of  the  verb  "may"  does  not  exclude  the  
possibility  that  a  transfer  may  be  made  in  a  different  manner,  thus  leaving  the  creditor  in  an  insecure  position  
even  though  he  has  the  certificate  in  his  possession.    
   
Moreover,  the  shares  still  standing  in  the  name  of  the  debtor  on  the  books  of  the  corporation  will  be  liable  to  
seizure   by   attachment   or   levy   on   execution   at   the   instance   of   other   creditors.   The   transfer   by   endorsement  
and  delivery  of  a  certificate  with  intention  to  pledge  the  shares  covered  thereby  should  be  sufficient  to  give  
legal  effect  to  that  intention  and  to  consummate  the  juristic  act  without  necessity  for  registration.lawphil  
   
   
FACTS:   Defendant   Samahang   Magsasaka,   Inc.,   is   a   corporation   duly   organized   under   the   laws   of   the  
Philippine  Islands  with  principal  office  in  Cabanatuan,  Nueva  Ecija,  and  that  the  individual  defendants  are  
the  president,  secretary  and  treasurer  respectively  of  the  same  
   
Gonzalo  H.  Co  Toco  was  the  owner  of  5,894  shares  of  the  capital  stock  of  the  said  corporation  represented  by  
nine  certificates  having  a  par  value  of  P5  per  share;  that  on  said  date  Gonzalo  H.  Co  Toco,  a  resident  of  Manila,  
mortgaged  said  5,894  shares  to  Chua  Chiu  to  guarantee  the  payment  of  a  debt  of  P20,000.    

   
3H  A.Y.  2017-­‐2018   200  
 
 

CORPORATION  LAW  CASE  DIGESTS  –  ATTY.  DANTE  DELA  CRUZ  

The   said   certificates   of   stock   were   delivered   with   the   mortgage   to   the   mortgagee,   Chua   Chiu.   The   said  
mortgage   was   duly   registered   in   the   office   of   the   register   of   deeds   of   Manila   and   in   the   office   of   the   said  
corporation    
   
Chua  Chiu  assigned  all  his  right  and  interest  in  the  said  mortgage  to  the  plaintiff  and  the  assignment  was  
registered  in  the  office  of  the  register  of  deeds  in  the  City  of  Manila  and  in  the  office  of  the  said  corporation.  
   
The   debtor,   Gonzalo   H.   Co   Toco,   having   defaulted   in   the   payment   of   said   debt   at   maturity,   the   plaintiff  
foreclosed  said  mortgage  and  delivered  the  certificates  of  stock  and  copies  of  the  mortgage  and  assignment  
to  the  sheriff.  The  sheriff  auctioned  said  5,894  shares  of  stock  on  and  the  plaintiff  having  been  the  highest  
bidder,  executed  in  his  favor  a  certificate  of  sale  of  said  shares.  
   
The   plaintiff   tendered   the   certificates   of   stock   standing   in   the   name   of   Gonzalo   H.   Co   Toco   to   the   proper  
officers  of  the  corporation  for  cancellation  and  demanded  that  they  issue  new  certificates  in  the  name  of  the  
plaintiff.  The  said  officers  (the  individual  defendants)  refused  and  still  refuse  to  issue  said  new  shares  in  the  
name  of  the  plaintiff.  
   
Defendant  contends  that  they  refuse  to  cancel  the  said  certificates  standing  in  the  name  of  Gonzalo  H.  Co  Toco  
on  the  books  of  the  corporation  and  to  issue  new  ones  in  the  name  of  the  plaintiff  because  prior  to  the  date  
when  the  plaintiff  made  his  demand,  to  wit,  February  4,  1933,  nine  attachments  had  been  issued  and  served  
and  noted  on  the  books  of  the  corporation  against  the  shares  of  Gonzalo  H.  Co  Toco  and  the  plaintiff  objected  
to  having  these  attachments  noted  on  the  new  certificates  which  he  demanded.  These  attachments  noted  on  
the  books  of  the  corporation  against  the  shares  of  Gonzalo  H.  Co  Toco.    
   
It  will  be  noted  that  the  first  eight  of  the  said  writs  of  attachment  were  served  on  the  corporation  and  noted  
on  its  records  before  the  corporation  received  notice  from  the  mortgagee  Chua  Chiu  of  the  mortgage  of  said  
shares    
   
ISSUE:  Did  the  registration  of  said  chattel  mortgage  in  the  registry  of  chattel  mortgages  in  the  office  of  the  
register   of   deeds   of   Manila,   under   date   of   July   23,   1931,   give   constructive   notice   to   the   said   attaching  
creditors?  
HELD:    
In  passing,  let  it  be  noted  that  the  registration  of  the  said  chattel  mortgage  in  the  office  of  the  corporation  
was  not  necessary  and  had  no  legal  effect.    
   
Section   4   of   Act   No.   1508   provides   two   ways   for   executing   a   valid   chattel   mortgage   which   shall   be  effective  
against  third  persons.  First,  the  possession  of  the  property  mortgage  must  be  delivered  to  and  retained  by  
the  mortgagee;  and,  second,  without  such  delivery  the  mortgage  must  be  recorded  in  the  proper  office  or  
offices  of  the  register  or  registers  of  deeds.    
   
If   a   chattel   mortgage   of   shares   of   stock   of   a   corporation   may   validly   be   made   without   the   delivery   of  
possession   of   the   property   to   the   mortgagee   and   the   mere   registration   of   the   mortgage   is   sufficient   to  
constructive  notice  to  third  parties,  we  are  confronted  with  the  question  as  to  the  proper  place  of  registration  
of  such  a  mortgage.    
   
If  with  respect  to  a  chattel  mortgage  of  shares  of  stock  of  a  corporation,  registration  in  the  province  of  the  
owner's   domicile   should   be   sufficient,   those   who   lend   on   such   security   would   be   confronted   with   the  
practical   difficulty   of   being   compelled   not   only   to   search   the   records   of   every   province   in   which   the  
mortgagor  might  have  been  domiciled  but  also  every  province  in  which  a  chattel  mortgage  by  any  former  
owner  of  such  shares  might  be  registered.    

   
3H  A.Y.  2017-­‐2018   201  
 
 

CORPORATION  LAW  CASE  DIGESTS  –  ATTY.  DANTE  DELA  CRUZ  

   
It   is   a   general   rule   that   for   purposes   of   execution,   attachment   and   garnishment,   it   is   not   the   domicile   of   the  
owner  of  a  certificate  but  the  domicile  of  the  corporation  which  is  decisive.    
   
By  analogy  with  the  foregoing  and  considering  the  ownership  of  shares  in  a  corporation  as  property  distinct  
from   the   certificates   which   are   merely   the   evidence   of   such   ownership,   it   seems   to   us   a   reasonable  
construction  of  section  4  of  Act  No.  1508  to  hold  that  the  property  in  the  shares  may  be  deemed  to  be  situated  
in  the  province  in  which  the  corporation  has  its  principal  office  or  place  of  business.    
   
In  this  sense  the  property  mortgaged  is  not  the  certificate  but  the  participation  and  share  of  the  owner  in  the  
assets  of  the  corporation.  
   
It  is  to  be  noted,  however,  that  section  35  of  the  Corporation  Law  (Act  No.  1459)  enacts  that  shares  of  stock  
"may   be   transferred   by   delivery   of   the   certificate   endorsed   by   the   owner   or   his   attorney   in   fact   or   other  
person   legally   authorized   to   make   the   transfer."   The   use   of   the   verb   "may"   does   not   exclude   the   possibility  
that  a  transfer  may  be  made  in  a  different  manner,  thus  leaving  the  creditor  in  an  insecure  position  even  
though  he  has  the  certificate  in  his  possession.    
   
Moreover,  the  shares  still  standing  in  the  name  of  the  debtor  on  the  books  of  the  corporation  will  be  liable  to  
seizure   by   attachment   or   levy   on   execution   at   the   instance   of   other   creditors.   The   transfer   by   endorsement  
and  delivery  of  a  certificate  with  intention  to  pledge  the  shares  covered  thereby  should  be  sufficient  to  give  
legal  effect  to  that  intention  and  to  consummate  the  juristic  act  without  necessity  for  registration.lawphil.net  
   
In   view   of   the   premises,   the   attaching   creditors   are   entitled   to   priority   over   the   defectively   registered  
mortgage  of  the  appellant.  
   
 
 
158.  MAKATI  SPORTS  CLUB,  INC.  V.  CECILE  H.  CHENG,  ET.  AL  
GR  NO.  178523  16  JUNE  2010  
JUSTICE  NACHURA  
 
DOCTRINE:  
  Fraud  committed  by  a  corporation’s  officer  is  a  question  of  fact  that  must  be  alleged  and  proved.  It  
cannot   be   presumed   and   must   be   established   by   clear   and   convincing   evidence.   The   party   alleging   the  
existence  of  fraud  has  the  burden  of  proof.  
 
FACTS:  
  On   20   October   1994,   Makati   Sports   Club   Inc   (MSCI)’s   Board   of   Directors   adopted   a   Resolution  
authorizing  the  sale  of  19  unissued  shares  at  a  floor  price  of  P400,000  and  P450,000  per  share  for  Class  A  
and  B,  respectively.  Defendant  Cheng  was  the  Treasurer  of  MSCI  in  1985.    
 
In  July  1995,  Hodreal  wrote  a  letter  to  MSCI  expressing  his  interest  to  buy  a  share  and  to  include  him  in  the  
waiting  list  of  buyers.  During  the  same  year,  McFoods  also  expressed  its  interest  in  buying  a  share  from  MSCI  
and  was  able  to  acquire  one  upon  payment.  Subsequently,  a  deed  of  sale  and  a  certificate  of  stock  was  issued  
in  the  name  of  McFoods.  However,  it  appeared  that  while  the  sale  between  MSCI  and  McFoods  were  on-­‐‑going,  
there  were  negotiations  between  McFoods  and  Hodreal  as  well  for  the  purchase  by  the  latter  of  a  share  of  
MSCI.  Hodreal  paid  McFoods  the  purchase  price  for  such  share.  MSCI  was  advised  of  the  sale  of  the  stock  thus  
issued  a  new  certificate  of  stock  in  the  name  of  Hodreal.    
 

   
3H  A.Y.  2017-­‐2018   202  
 
 

CORPORATION  LAW  CASE  DIGESTS  –  ATTY.  DANTE  DELA  CRUZ  

In  an  investigation  conducted  in1997,  it  appeared  that  Defendant  Cheng  profited  from  the  transaction  becaue  
of   her   knowledge.   Thus,   MSCI   sought   judgment   that   P1,000,000.00   be   paid   to   them   by   the   respondents  
representing  the  amount  allegedly  defrauded  them.  However,  the  RTC  dismissed  their  complaint.  On  appeal,  
the  CA  affirmed  the  RTC’s  Decision.  
 
ISSUE:  
  Whether  or  not  the  respondents  indeed  defrauded  the  petitioner  
 
RULING:  
  No,  the  Court  is  not  convinced  with  MSCI’s  defenses.    
 
The   Court   takes   note   that   Hodreal   already   expressed   its   intent   to   purchase   one   Class   A   stock   to   MSCI’s  
Membership  Committee  which  failed  to  act  on  his  letter.  Charged  with  ascertaining  the  compliance  of  all  the  
requirements   for   the   purchase   of   MSCI’s   shares   of   stock,   the   Membership   Committee   failed   to   question   the  
alleged  irregularities  attending  McFoods  purchase  of  one  Class  A  share.    
 
MSCI’s  position  that  Cheng  was  in  collaboration  with  McFoods  in  depriving  it  of  selling  an  original,  unissued  
Class  A  share  of  stock  is  not  supported  by  evidence  on  record.  Fraud  is  a  question  of  fact  that  must  be  alleged  
and   proved.   It   cannot   be   presumed   and   must   be   established   by   clear   and   convincing   evidence.   The   party  
alleging  the  existence  of  fraud  has  the  burden  of  proof.  
 
 
 
159.  NORA  BITONG  VS.  CA,  EUGENIA  APOSTOL  
G.R.  NO.  123553.  JULY  13,  1998  
BELLOSILLO,  J.  
 
DOCTRINE:  The  basis  of  a  stockholder’s  suit  is  always  one  in  equity.  However,  it  cannot  prosper  without  first  
complying  with  the  legal  requisites  for  its  institution.  The  most  important  of  these  is  the  bona  fide  ownership  
by  a  stockholder  of  a  stock  in  his  own  right  at  the  time  of  the  transaction  complained  of  which  invests  him  
with  standing  to  institute  a  derivative  action  for  the  benefit  of  the  corporation.  
 
FACTS:   Nora   Bitong   claimed   before   the   SEC   that   she   had   been   the   Treasurer   and   a   Member   of   the   Board   of  
Directors  of  private  respondent  Mr.  &  Ms.  Publishing  Co.,  Inc.  (Mr.  &  Ms.)  from  the  time  it  was  incorporated  
on  29  October  1976  to  11  April  1989,  and  was  the  registered  owner  of  1,000  shares  of  stock  out  of  the  4,088  
total  outstanding  shares.  
 
Allegedly   acting   for   the   benefit   of   Mr.   &   Ms.   Co.,   Bitong   filed   a   derivative   suit   before   the   SEC   against  
respondent  spouses  Eugenia  D.  Apostol  and  Jose  A.  Apostol,  who  were  officers  in  said  corporation,  to  hold  
them  liable  for  fraud  and  mismanagement  in  directing  its  affairs.    
 
Respondent   spouses   moved   to   dismiss   on   the   ground   that   petitioner   had   no   legal   standing   to   bring   the   suit  
as  she  was  merely  a  holder-­‐‑in-­‐‑trust  of  shares  of  JAKA  Investments  which  continued  to  be  the  true  stockholder  
of  Mr.  &  Ms.  
 
Petitioner  contends  that  she  was  a  holder  of  proper  stock  certificates  and  that  the  transfer  was  recorded.  She  
further   contends   that   even   in   the   absence   of   the   actual   certificate,   mere   recording   will   suffice   for   her   to  
exercise  all  stockholder  rights,  including  the  right  to  file  a  derivative  suit  in  the  name  of  the  corporation.  The  
SEC  Hearing  Panel  dismissed  the  suit.  On  appeal,  the  SEC  En  Banc  found  for  petitioner.  CA  reversed  the  SEC  
En  Banc  decision.  

   
3H  A.Y.  2017-­‐2018   203  
 
 

CORPORATION  LAW  CASE  DIGESTS  –  ATTY.  DANTE  DELA  CRUZ  

ISSUE:  Whether  or  not  petitioner  is  the  true  holder  of  stock  certificates  to  be  able  institute  a  derivative  suit.    
 
HELD:  
NO.  Sec  63  of  the  Corporation  Code  envisions  a  formal  certificate  of  stock  which  can  be  issued  only  upon  
compliance  with  certain  requisites.  
 
First,  the  certificates  must  be  signed  by  the  president  or  vice-­‐‑president,  countersigned  by  the  secretary  or  
assistant   secretary,   and   sealed   with   the   seal   of   the   corporation.   A   mere   typewritten   statement   advising   a  
stockholder   of   the   extent   of   his   ownership   in   a   corporation   without   qualification   and/or   authentication  
cannot  be  considered  as  a  formal  certificate  of  stock.  Second,  delivery  of  the  certificate  is  an  essential  element  
of  its  issuance.  Hence,  there  is  no  issuance  of  a  stock  certificate  where  it  is  never  detached  from  the  stock  
books  although  blanks  therein  are  properly  filled  up  if  the  person  whose  name  is  inserted  therein  has  no  
control  over  the  books  of  the  company.  Third,  the  par  value,  as  to  par  value  shares,  or  the  full  subscription  as  
to  no  par  value  shares,  must  first  be  fully  paid.  Fourth,  the  original  certificate  must  be  surrendered  where  the  
person  requesting  the  issuance  of  a  certificate  is  a  transferee  from  a  stockholder.  
 
The   certificate   of   stock   itself   once   issued   is   a   continuing   affirmation   or   representation   that   the   stock  
described  therein  is  valid  and  genuine  and  is  at  least  prima  facie  evidence  that  it  was  legally  issued  in  the  
absence  of  evidence  to  the  contrary.  However,  this  presumption  may  be  rebutted.  Aside  from  petitioner’s  
own  admissions,  several  corporate  documents  disclose  that  the  true  party-­‐‑in-­‐‑interest  is  not  petitioner  but  
JAKA.  It  should  be  emphasized  that  JAKA  executed,  a  deed  of  sale  over  1,000  Mr.  &  Ms.  shares  in  favor  of  
respondent  Eugenio  D.  Apostol.  On  the  same  day,  respondent  Apostol  signed  a  declaration  of  trust  stating  
that   she   was   the   registered   owner   of   1,000   Mr.   &   Ms.   shares   covered   by   a   Certificate   of   Stock.   And,   there   is  
nothing  in  the  records  which  shows  that  JAKA  had  revoked  the  trust  it  reposed  on  respondent  Eugenia  D.  
Apostol.   Neither   was   there   any   evidence   that   the   principal   had   requested   her   to   assign   and   transfer   the  
shares  of  stock  to  petitioner.  In  fine,  the  records  are  unclear  on  how  petitioner  allegedly  acquired  the  shares  
of  stock  of  JAKA.  
Thus,  for  a  valid  transfer  of  stocks,  the  requirements   are  as  follows:  (a)  There  must  be  delivery  of  the  stock  
certificate;  (b)  The  certificate  must  be  endorsed  by  the  owner  or  his  attorney-­‐‑in-­‐‑fact  or  other  persons  legally  
authorized  to  make  the  transfer;  and,  (c)  to  be  valid  against  third  parties,  the  transfer  must  be  recorded  in  
the  books  of  the  corporation.    
At  most,  in  the  instant  case,  petitioner  has  satisfied  only  the  third  requirement.  Compliance  with  the  first  two  
requisites  has  not  been  clearly  and  sufficiently  shown.  
 
 
 
160.  MANUEL  A.  TORRES,  JR.  V  COURT  OF  APPEALS  
G.R.  NO.  120138  SEPTEMBER  5,  1997  
KAPUNAN,  J.:  
 
Doctrine:  It  is  the  corporate  secretary's  duty  and  obligation  to  register  valid  transfers  of  stocks.  Stock  and  
Transfer  books  must  be  kept  at  the  principal  office  of  the  corporation.  All  corporations,  big  or  small,  must  
abide  by  the  provisions  of  the  code  including  the  requirements  in  Section  74.  
 
Facts:  
Judge   Manuel   Torres   was   the   majority   stockholder   of   Tormil   Realty   &   Development   Corporation   (TRDC)  
while   private   respondents   who   are   the   children   of   Judge   Torres'   deceased   brother   Antonio   A.   Torres,  
constituted   the   minority   stockholders.   TRDC   is   a   small   family   owned   corporation   and   other   stockholders  
thereof  include  Judge  Torres’  nieces  and  nephews.    Before  the  regular  election  of  TRDC  officers,  Judge  Torres  
assigned  one  share  each  to  his  assignees  for  the  purpose  of  qualifying  them  to  be  elected  as  directors  in  the  

   
3H  A.Y.  2017-­‐2018   204  
 
 

CORPORATION  LAW  CASE  DIGESTS  –  ATTY.  DANTE  DELA  CRUZ  

board   and   thereby   strengthen   Judge   Torres’   power   over   other   family   members.   However,   the   said  
assignment  of  shares  were  not  recorded  by  the  corporate  secretary,  in  the  stock  and  transfer  book  of  TRDC.  
When  the  validity  of  said  assignments  were  questioned,  Judge  Torres  ratiocinated  that  it  is  impractical  for  
him  to  order  Carlos  to  make  the  entries  because  Carlos  is  one  of  his  opposition.  So  what  Judge  Torres  did  was  
to  make  the  entries  himself  because  he  was  keeping  the  stock  and  transfer  book.  He  further  ratiocinated  that  
he  can  do  what  a  mere  secretary  can  do  because  in  the  first  place,  he  is  the  president.  
Judge  Torres  and  his  assignees  then  decided  to  conduct  the  election  among  themselves  considering  that  the  
6  of  them  constitute  a  quorum.  
 
Issue:  
Whether  or  not  the  recording  made  in  the  books  by  Judge  Torres,  and  his  custody  of  the  books  complied  with  
procedural  requirements  in  Section  74  of  the  Corporation  Code.  
 
Ruling:  
No.  The  assignment  of  the  shares  of  stocks  did  not  comply  with  procedural  requirements.  It  did  not  comply  
with  the  by  laws  of  TRDC  nor  did  it  comply  with  Section  74  of  the  Corporation  Code.  Section  74  provides  that  
the  stock  and  transfer  book  should  be  kept  at  the  principal  office  of  the  corporation.  In  the  case  at  bar,  the  
stock  and  transfer  book  was  not  kept  at  the  principal  office  of  the  corporation  either  but  at  the  place  of  Torres.  
These  being  the  obtaining  circumstances,  any  entries  made  in  the  stock  and  transfer  book  by  respondent  
cannot  therefore  be  given  any  valid  effect.    
Here,  it  was  Judge  Torres  who  was  keeping  it  and  was  bringing  it  with  him.  Petitioners  cannot  use  the  flimsy  
excuse  that  it  would  have  been  a  vain  attempt  to  force  the  incumbent  corporate  secretary  to  register  the  
aforestated   assignments   in   the   stock   and   transfer   book   because   the   latter   belonged   to   the   opposite   faction.  
It  is  the  corporate  secretary's  duty  and  obligation  to  register  valid  transfers  of  stocks  and  if  said  corporate  
officer   refuses   to   comply,   the   transferor-­‐‑stockholder   may   rightfully   bring   suit   to   compel   performance.   In  
other   words,   there   are   remedies   within   the   law   that   petitioners   could   have   availed   of,   instead   of   taking   the  
law  in  their  own  hands.    
The   Supreme   Court   also   emphasized:   all   corporations,   big   or   small,   must   abide   by   the   provisions   of   the  
Corporation  Code.  Being  a  simple  family  corporation  is  not  an  exemption.  Such  corporations  cannot  have  
rules  and  practices  other  than  those  established  by  law.  
 
 
 
161.  FINANCING  CORPORATION  AND  J.  AMADO  ARANETA  V.  HON.  JOSE  TEODORO  
G.R.  NO.  L-­‐‑4900,  AUGUST  31,  1953  
MONTEMAYOR,  J.  
 
DOCTRINE:  Although  as  a  rule  minority  stockholders  of  a  corporation  may  not  ask  for  its  dissolution  in  a  
private  suit  and  such  action  should  be  brought  by  the  Government  through  its  legal  officer  in  a  quo  warranto  
case  at  their  instance  and  request,  there  might  be  exceptional  cases  wherein  the  intervention  of  the  State,  for  
one   reason   or   another,   cannot   be   obtained,   as   when   the   State   is   not   interested   because   the   complaint   is  
strictly   a   matter   between   the   stockholders   and   does   not   involve,   in   the   opinion   of   the   legal   officer   of   the  
Government,   any   of   the   acts   or   omissions   warranting   quo   warranto   proceedings   in   which   minority  
stockholders  are  entitled  to  have  such  dissolution.  
 
 
FACTS:  Asuncion  Lopez  Vda.  de  Lizares,  Encarnacion  Lizares  Vda.  de  Panlilio  and  Efigenia  Vda.  de  Paredes  
(respondents),   in   their   own   behalf   and   in   behalf   of   the   other   minority   stockholders   of   the   Financing  
Corporation  of  the  Philippines  (Financing  Corp.),  filed  a  complaint  against  the  said  corporation  and  J.  Amado  
Araneta   (Araneta),   president   and   general   manager,   claiming   among   other   things,   alleged   gross  

   
3H  A.Y.  2017-­‐2018   205  
 
 

CORPORATION  LAW  CASE  DIGESTS  –  ATTY.  DANTE  DELA  CRUZ  

mismanagement  and  fraudulent  conduct  of  the  corporate  affairs  of  the  corporation  by  Araneta,  and  asking  
that   the   corporation   be   dissolved   xxx   and   that   pending   trial   and   disposition   of   the   case   on   its   merits,   a  
receiver  be  appointed  to  take  possession  of  the  books,  records  and  assets  of  Financing  Corp.  preparatory  to  
its  dissolution  and  liquidation  and  distribution  of  the  assets.  
 
The   main   contention   of   the   petitioners   in   opposing   the   appointment   of   a   receiver   in   this   case   is   said  
appointment   is   merely   an   auxiliary   remedy;   that   the   principal   remedy   sought   by   the   respondents   in   the  
action   was   the   dissolution   of   Financing   Corp.;   that   according   to   the   law,   a   suit   for   the   dissolution   of   a  
corporation  can  be  brought  and  maintained  only  by  the  State  through  its  legal  counsel,  and  that  respondents,  
much   less   the   minority   stockholders   of   Financing   Corp.,   have   no   right   or   personality   to   maintain   the   action  
for   dissolution,   and   that   inasmuch   as   said   action   cannot   be   maintained   legally   by   the   respondents,   then   the  
auxiliary  remedy  for  the  appointment  of  a  receiver  has  no  basis.  
 
ISSUES:  
1.  Whether  or  not  minority  stockholders  of  a  corporation  may  ask  for  its  dissolution  in  a  private  suit.  
2.  Whether  or  not  the  appointment  of  a  receiver  pendente  lite  is  within  the  power  of  the  trial  court.  
 
HELD:  
1.  Yes,  minority  stockholders  of  a  corporation  may  ask  for  its  dissolution  in  a  private  suit.  Although  as  a  rule  
minority  stockholders  of  a  corporation  may  not  ask  for  its  dissolution  in  a  private  suit  and  such  action  should  
be  brought  by  the  Government  through  its  legal  officer  in  a  quo  warranto  case  at  their  instance  and  request,  
there  might  be  exceptional  cases  wherein  the  intervention  of  the  State,  for  one  reason  or  another,  cannot  be  
obtained,   as   when   the   State   is   not   interested   because   the   complaint   is   strictly   a   matter   between   the  
stockholders  and  does  not  involve,  in  the  opinion  of  the  legal  officer  of  the  Government,  any  of  the  acts  or  
omissions  warranting  quo  warranto  proceedings  in  which  minority  stockholders  are  entitled  to  have  such  
dissolution.  When  such  action  or  private  suit  is  brought  by  them,  the  trial  court  has  jurisdiction  and  may  or  
may  not  grant  the  prayer,  depending  upon  the  facts  and  circumstances  attending  it.  
 
2.  Yes,  it  is  within  the  power  of  the  trial  court  to  appoint  a  receiver  pendente  lite.  Although  the  appointment  
of  a  receiver  upon  application  of  the  minority  stockholders  is  a  power  to  be  exercised  with  great  caution,  
nevertheless  it  should  be  exercised  when  necessary  in  order  not  to  entirely  ignore  and  disregard  the  rights  
of   said   minority   stockholders,   especially   when   said   minority   stockholders   are   unable   to   obtain   redress   and  
protection  of  their  rights  within  the  corporation  itself.  
 
 
 
162.  REPUBLIC  OF  THE  PHILIPPINES  VS  BISAYA  LAND  TRANSPORTATION  CO.,  INC.  
G.R.  NO.  L-­‐‑31490  JANUARY  6,  1978  
CASTRO,  C.  J.  
 
DOCTRINE:    
And,   as   a   rule,   the   attorney-­‐‑general   has   power,   both   under   the   common   law   and   by   statute,   to   make   any  
disposition   of   the   state's   litigation   that   the   deems   for   its   best   interest;   for   instance,   he   may   abandon,  
discontinue,  dismiss,  or  compromise  it.  But  he  cannot  enter  into  any  agreement  with  respect  to  the  conduct  
of  litigation  which  will  bind  his  successor  in  office,  nor  can  he  empower  any  other  person  to  do  so.  ...  The  
attorney-­‐‑general   may   dismiss   any   suit   or   proceeding,   prosecuted   solely   in   the   public   interest,   regardless   of  
the  relator's  wishes.  ...  Where  the  attorney-­‐‑general  is  empowered,  either  generally  or  specifically,  to  conduct  
a  criminal  prosecution,  he  may  do  any  act  which  the  prosecuting  attorney  might  do  in  the  premises;  that  is,  
he   can   do   each   and   every   thing   essential   to   prosecute   in   accordance   e   with   the   law   of   the   land,   and   this  

   
3H  A.Y.  2017-­‐2018   206  
 
 

CORPORATION  LAW  CASE  DIGESTS  –  ATTY.  DANTE  DELA  CRUZ  

includes   appearing   in   proceedings   before   the   grand   jury.   So   an   attorney-­‐‑general,   even   at   common   law.   had  
the  right  to  enter  a  nolle  prosequi;  although  he  could  not  do  so  during  the  trial  without  leave  of  court.    
 
 
FACTS:    
The  Bisaya  Land  Transportation  Company  is  a  corporation  organized  under  Act  No.  1459,  otherwise  known  
as   the   Corporation   Law,   for   the   principal   purpose   of   engaging   in   the   business   of   land   and   water  
transportation,  having  its  domicile  and  principal  place  of  business  in  Cebu  City.    
The  instant  case  came  into  being  when  the  Republic  of  the  Philippines,  through  Solicitor  General  Edilberto  
Barot,  filed  a  petition  for  quo  warranto  in  the  CFI  Manila  for  the  dissolution  of  the  Bisaya  Land  Transportation  
Company.    
The  petition  alleges  that  respondent  corporation,  through  its  co-­‐‑respondents  named  therein,  acting  in  their  
offended  as  officers  and  controlling  stockholder  of  the  corporation,  by  conspiring  and  confabulating  together  
and  with  the  aid  offended  their  associates,  agents  and  confederates,  had  violated  and  continues  to  violate,  
offended  and  continues  to  offend  the  proceeding  of  the  Corporation  Law  and  other  statutes  of  the  Philippines  
by  having  committed  and  continuing  to  commit  acts  amounting  to  a  forfeiture  of  the  present  corporation's  
franchise,  rights  and  private  and,  through  venous  means,  misused  and  continues  to  and  continues  to  abuse,  
the  terms  of  its  franchise,  palpably  in  contravention  of  the  law  and  public  policy.  
The   acts   allegedly   committed   by   the   corporation,   through   as   corespondent,   are   embodied   in   nine   causes   of  
action  which,  in  substance,  are  as  follows:    
FIRST  CAUSE  OF  ACTION    
To  conceal  its  illegal  transaction,  respondent  corporation  falsely  reconstituted  its  articles  of  incorporation  in  
July  1948  by  adding  new  cattle  ranch,  agriculture,  and  general  merchandise;    
SECOND  CAUSE  OF  ACTION    
On  May  25,  1948,  respondent  corporation  through  its  Board  of  Directors,  adopted  a  resolution  authorizing  it  
to  acquire  1,024  hectares  of  public  land  in  Zamboanga  and  10,000  hectares  of  timber  concession  in  Mindanao  
in  violation  of  Section  6,  Act  No.  143);    
THIRD  CAUSE  OF  ACTION    
In  May,  1949,  respondent  office  constituting  themselves  as  Board  of  Directors  of  respondent  corporation,  
passed  a  resolution  authorizing  the  corporation  to  lease  a  pasture  land  of  2,000  hectares  of  cattle  ranch  on  a  
public  land  in  Bayawan,  Negros  Occidental;    
FOURTH  CAUSE  OF  ACTION    
From   August   1946   to   the   end   of   1952,   respondent   corporation   operated   a   general   merchandise   store,   a  
business  which  is  neither  for,  nor  incidental  to,  the  accomplishment  of  its  principal  business  for  which  it  was  
organized,  i.e.,  the  operation  of  land  and  water  transportation;    
FIFTH  CAUSE  OF  ACTION    
Respondent  corporation  snowed  Mariano  Cuenco  and  Manuel  Cuenco  to  act  as  president  in  1945  to  1948  
and  1953  to  1954,  respectively,  when  at  that  time,  neither  of  them  owned  a  single  stock;    
SIXTH  CAUSE  OF  ACTION    
In  violation  of  its  charter  and  articles  of  incorporation,  as  well  as  applicable  statutes  concerning  its  operation,  
it  engaged  in  mining  by  organizing  the  Jose  P.  Velez  Coal  Mines,  and  allowing  said  corporation  to  use  the  
facilities  and  assets  of  respondent  corporation;  
SEVENTH  CAUSE  OF  ACTION    
It   imported   and   sold   at   black   market   prices   to   third   persons   truck   spare   Parts,   the   of   which   were  
appropriated  by  respondent  directors;    
EIGHTH  CAUSE  OF  ACTION    
It  paid  its  laborers  and  employees  wages  below  the  minimum  wage  law  to  the  great  prejudice  of  its  labor  
force,  and  in  violation  of  the  laws  of  the  state,  manipulating  its  books  and  records  so  as  to  make  it  appear  that  
its   laborers   and   employees   were   and   have   been   paid   their   salaries   and   wages   in   accordance   with   the  
minimum  wage  law;    

   
3H  A.Y.  2017-­‐2018   207  
 
 

CORPORATION  LAW  CASE  DIGESTS  –  ATTY.  DANTE  DELA  CRUZ  

NINTH  CAUSE  OF  ACTION    


It  deliberately  failed  to  maintain  accurate  and  faithful  stock  and  transfer  books  since  1945  up  to  the  filing  of  
the   petition,   enabling   it   to   defraud   the   state,   mislead   the   general   public,   its   creditors,   investors   and   its  
stockholders  by  not  accurately  and  faithfully  making    
a.  an  adequate,  accurate  and  complete  record  of  dividend  distribution,  and    
b.  an  adequate,  accurate  and  complete  record  of  transfers  of  its  stocks.    
Respondent   corporation   filed   a   motion   for   judgment   on   consent,   manifesting   its   consent   to   and   moving   for  
judgment   to   be   rendered   ordering   the   dissolution   of   respondent   Bisaya   Land   transportation   Company,   Inc.  
and,   in   furtherance   of   that   dissolution,   ordering   its   board   of   directors   to   proceed   to   the   liquidation   of   its  
assets  in  accordance  with  the  provisions  of  the  corporation  law.  In  said  motion  for  judgment  on  consent,  
respondent   corporation   did   not   admit   having   committed   any   act   requiring   its   forcible   dissolution,   but  
alleged,   as   reason   for   the   filing   of   said   motion,   that   the   pendency   of   the   petition   of   quo   warranto   had  
prejudiced   the   corporation   its   business.   as   well   as   its   innocent   stockholders,   and   that   its   business   interests  
that  late  relief  be  given  to  the  corporation  and  to  its  thousands  of  stockholders;  and  that  the  majority  of  the  
board  of  directors  and  stockholders  representing  more  than  two-­‐‑thirds  of  its  capital  stock  had  indicated  their  
election  to  voluntarily  dissolve  the  corporation  as  the  most  feasible  remedy  to  the  corporation's  problems  
brought  about  by  the  respondent  Miguel  Cuenco.    
After  some  more  motions  filed,  Solicitor  General  eventually  filed  a  “Motion  for  Dismissal  of  the  Quo  Warranto  
Proceedings”  alleging  that:  the  only  purpose  of  his  motion  for  the  dismissal  of  this  quo  warranto  is  to  take  
the   State   out   of   an   unnecessary   court   litigation,   so   that   the   dismissal   of   the   case   would   result   in   the  
disposition   solely   of   the   quo   warranto   by   and   between   petitioner   Republic   of   the   Philippines   and   the  
respondents  named  therein.    
Appellant-­‐‑   defendant   Miguel   Cuenco   maintains   the   negative   of   the   above   proposition,   giving   three   main  
reasons  therefor,  namely:    
1.  The  evidence  so  far  adduced  was  in  fact  sufficient  to  dissolve  the  respondent  corporation;    
2.   There   was   a   pending   motion   of   respondents   for   judgment   on   consent,   by   virtue   of   which   instead   of  
dismissal  of  the  petition,  the  corporation  should  be  considered  dissolved;    
3.  A  cross-­‐‑claim  has  been  interposed  by  Miguel  Cuenco  which  precluded  the  dismissal  of  the  petition  for  quo  
warranto.  
 
ISSUE:    
The  pivotal  question  in  this  case  is  case  is  whether  or  not  the  lower  court  erred  in  holding  that  the  Solicitor  
General  was  vested  with  full  power  to  manage  and  control  the  State's  litigation,  which  includes  the  power  to  
continue  such  litigation,  if  and  when  in  his  opinion  this  should  be  done.    
 
HELD:  
Meeting   squarely   the   issue   of   whether   or   not   the   Solicitor   General   is   vested   with   absolute   and   unlimited  
power   to   discontinue   the   State's   litigation   and,   accordingly,   to   have   the   quo   warranto   petition   dismissed,   if  
and   when   in   his   opinion   this   should   be   done,   the   general   rule   seems   to   be   that   the   plaintiff   may   do   so   with  
the  approval  of  the  court,  subject  to  be  defined  exceptions  (such  as,  for  example,  where  the  answer  sets  up  a  
counterclaim  which  cannot  stand  independently  of  the  main  action).    
The  right  of  the  plaintiff  to  dismiss  an  action  with  the  consent  of  the  court  is  universally  recognized  with  
certain   well-­‐‑defined   exceptions.   If   the   plaintiff   discovers   that   the   action   which   he   commenced   was   brought  
for   purposes   of   enforcing   a   right   or   a   benefit,   the  advisability   or   necessity  of   which  he   later  discovers  no  
longer   exists,   or   that   the   result   of   the   action   would   be   different   from   what   he   had   intended,   then   he   should  
be  permitted  to  withdraw  his  action,  subject  to  the  approval  of  the  court.  The  plaintiff  should  not  be  required  
to   continue   the   action,   subject   to   some   well-­‐‑defined   exceptions,   when   it   is   not   to   his   advantage   so   to   do.  
Litigation  should  be  discouraged  and  not  encouraged.  Courts  should  not  with  require  parties  to  litigate  when  
they  no  longer  desire  so  to  do.  Courts,  in  granting  permission  to  dismiss  an  action,  of  course,  should  always  
take  into  consideration  the  effect  which  said  dismissal  would  have  upon  the  rights  of  the  defendant.    

   
3H  A.Y.  2017-­‐2018   208  
 
 

CORPORATION  LAW  CASE  DIGESTS  –  ATTY.  DANTE  DELA  CRUZ  

...   in   the   case   of   a   municipality,   where   the   agents   of   the   public   are   spending   public   money,   we   are   of   the  
opinion   that   such   agent   should   not   be   required   to   continue   an   action   when   (a)   it   clearly   appears   that   there  
is  no  longer  a  necessity  therefor,  or  (b)  when  it  clearly  appears  that  to  continue  the  action,  the  result  would  
be  prejudicial  to  the  interests  of  the  public.  We  think  that  this  conclusion  is  more  in  harmony  with  the  rational  
conduct  of  public  affairs  than  the  opposite  rule.  (City  of  Manila  vs.  Ruymann,  37  Phil.  421,  424-­‐‑425,  427,  cited  
in  Metropolitan  Water  District  vs.  De  los  Angeles,  55  Phil.  776,  790.)    
American   authorities   likewise   uphold   the   power   and   authority   of   the   state   attorney   to   control   and   manage  
all   litigation   in   behalf   of   the   State,   which   power   involves   the   power   to   discontinue   the   same   if   and   when,   in  
his  opinion,  this  should  be  done.  (7  Am.  Jur.  2d  18-­‐‑  19).    
In   view   of   our   conclusion   that   the   court   a   quo   committed   no   error   in   dismissing   the   quo   warranto  
proceedings,  it  also  stands  to  reason  that  it  acted  correctly  in  dismissing  appellant  Miguel  Cuenco's  cross-­‐‑  
claim.  A  cross-­‐‑claim  is  proper  only  where  the  cross-­‐‑claimant  stands  to  be  prejudiced  by  the  filing  of  an  action  
against  him.  Hence,  where  such  action  has  been  dismissed,  his  cross-­‐‑claim  would  have  no  leg  to  stand  on.  
 
 
 
163.    GONZALES  V.  SUGAR  REGULATORY  ADMINISTRATION  
G.R.  NO.  84606.  JUNE  28,  1989  
FELICIANO,  J.  
 
Doctrine:  The  termination  of  the  life  of  a  juridical  entity  does  not  by  itself  imply  the  diminution  or  extinction  
of  rights  demandable  against  such  juridical  entity.  
 
Facts:  
On  23  December  1987,  petitioner  spouses,  Ramon  A.  Gonzales  and  Lilia  Y.  Gonzales,  filed  a  complaint  seeking  
cancellation  of  a  mortgage  and  recovery  of  a  sum  of  money  against  the  Republic  Planters  Bank  (RPBank),  
Philippine   Sugar   Commission   (Philsucom)   and   the   SRA.   The   complaint   alleged   that   on   13   May   1980,  
petitioners  obtained  a  loan  from  the  RPBank  in  the  amount  of  P176,000.00  secured  by  a  real  estate  mortgage.  
The  proceeds  of  the  loan  were  released  on  a  staggered  basis  and  the  loan  was  payable  from  [the]  1980-­‐‑1981  
sugar  crop,    the  amortization  payments  to  be  remitted  by  the  Philsucom  to  the  RPBank.    
 
The   RPBank   is   owned   and   controlled   by   the   Philsucom.   On   24   September   1987,   petitioners   received   a  
statement  of  account  from  the  RPBank  setting  forth  that  petitioners  had  an  outstanding  loan  balance  due  to  
the  bank  of  P652,446.38.  Petitioners  then  requested  copies  of  the  promissory  notes  executed  by  them  as  well  
as  the  breakdown  of  re-­‐‑payments  they  had  made  on  their  loan.  On  the  basis  of  the  promissory  notes  and  the  
list  of  re-­‐‑payments  made,  the  complaint  continued,  it  appeared  that  petitioners  had  received  the  total  amount  
of  P1,041,610.55  in  loan  funds  from  the  RPBank  and  that  petitioners  had  re-­‐‑paid  thereon  the  total  amount  
of  P1,051,296.77;  in  other  words,  petitioners  had  already  more  than  fully  repaid  their  loan.  The  complaint  
further  averred  that  Philsucom  had  deducted  from  the  export  sugar  proceeds  of  petitioners  the  amount  of  
P421,517.32   without   the   authority   and   consent   of   petitioners   with   the   result   that   petitioners   had   overpaid  
the   RPBank   by   P289,260.88.   Petitioners   prayed   that   the   real   estate   mortgage   be   cancelled,   and   that  
Philsucom  and  SRA  be  required  jointly  and  severally  to  reimburse  the  petitioners  the  amount  of  P289,260.88  
as  well  as  moral  damages  of  P50,000.00  and  attorney's  fees  of  another  P50,000.00.  
 
Petitioners  filed,  on  17  March  1988,  an  amended  complaint  which  assailed  the  constitutionality  of  Executive  
Order   No.   18.   Petitioners   urged   that   the   abolition   of   the   Philsucom   by   Executive   Order   No.   18   in   effect  
destroyed   the   petitioners'   right   to   recover   from   Philsucom   what   petitioners   claim   in   their   complaint   is   due  
to  them.  Petitioners  hence  assert  that  they  had  been  deprived  of  property  without  due  process  of  law  and  
that   the   abolition   of   Philsucom   and   the   transfer   of   assets   from   Philsucom   to   respondent   SRA,   are  
unconstitutional   and   ineffective.   In   a   separate   pleading,   petitioners   also   opposed   the   motions   to   dismiss  

   
3H  A.Y.  2017-­‐2018   209  
 
 

CORPORATION  LAW  CASE  DIGESTS  –  ATTY.  DANTE  DELA  CRUZ  

arguing,  once  more,  that  Executive  Order  No.  18,  to  the  extent  it  abolished  the  Philsucom  and  transferred  its  
assets  to  respondent  SRA,  deprived  petitioners  of  a  property  right  without  due  process  of  law.    
 
Issue:  Whether  the  liability  extinguished.  (No)  
 
Ratio:  
Petitioners'  argument  on  unconstitutionality  is  too  impressionistic  and  needs  to  be  more  sharply  focused.  
One  who  asserts  a  claim  against  a  juridical  entity  has  no  constitutional  right  to  the  perpetual  existence  of  
such  entity.  Juridical  persons,  whether  incorporated  or  not,  whether  owned  by  the  government  or  the  private  
sector,   may   come   to   an   end   at   one   time   or   another   for   a   variety   of   reasons,   e.g.,   the   fulfillment   or   the  
abandonment   of   the   business   purposes   for   which   a   corporation   was   set   up.   Thus,   the   Corporation   Code  
provides  for  termination  of  corporate  life,  the  dissolution  of  the  corporation,  the  winding  up  of  its  operations,  
the   liquidation   of   its   assets,   the   payment   of   its   obligations   and   distribution   of   any   residual   assets   to   its  
stockholders.   The   termination   of   the   life   of   a   juridical   entity   does   not   by   itself   imply   the   diminution   or  
extinction  of  rights  demandable  against  such  juridical  entity.  
 
 
 
164.   PEPSI-­‐‑COLA   PRODUCTS   PHILIPPINES,   INC.   V   .THE   COURT   OF   APPEALS,   AND   PEPSI-­‐‑COLA  
PRODUCTS   PHILIPPINES,   INC.   EMPLOYEES   &   WORKERS   UNION   (UOEF   NO.   70)   REPRESENTED   BY   ITS  
INCUMBENT  PRESIDENT,  ISIDRO  REALISTA  
G.R.  NO.  145855  -­‐‑  NOVEMBER  24,  2004  
CALLEJO,  SR.,  J.  
 
DOCTRINE:  
A  corporation  whose  corporate  existence  is  terminated  in  any  manner  continues  to  be  a  body  corporate  for  
three  (3)  years  after  its  dissolution  for  purposes  of  prosecuting  and  defending  suits  by  and  against  it  and  to  
enable  it  to  settle  and  close  its  affairs,  culminating  in  the  disposition  and  distribution  of  its  remaining  assets.  
It  may,  during  the  three-­‐‑year  term,  appoint  a  trustee  or  a  receiver  who  may  act  beyond  that  period.  
 
FACTS:  
Pepsi-­‐‑Cola   Products   Philippines,   Inc.   Employees   and   Workers   Union   (PCEWU)   is   a   duly-­‐‑registered   labor  
union  of  the  employees  of  the  Pepsi-­‐‑Cola  Distributors  of  the  Philippines  (PCDP).  PCEWU  filed  a  Complaint  
against   PCDP   with   the   DOLE   for   payment   of   overtime   services   rendered   by   53   of   its   members,   on   the   eight  
(8)  days  duly-­‐‑designated  as  Muslim  holidays  for  calendar  year  1985,  
 
The  Executive  Labor  Arbiter  (ELA)  rendered  a  Decision  in  favor  of  PCEWU,  ordering  PCDP  to  pay  the  claims  
of  its  workers.  The  respondent  appealed  the  decision  to  the  NLRC  which  affirmed  the  decision  of  the  ELA.  
The  PCDP  filed  its  motion  for  partial  reconsideration  of  the  NLRC  decision.  The  employees  also  filed  a  motion  
for  reconsideration.  Pending  resolution  of  the  said  motions,  ownership  of  various  Pepsi-­‐‑Cola  bottling  plants  
was  transferred  to  petitioner  Pepsi-­‐‑Cola  Products  Philippines,  Inc.  (PCPPI).  The  NLRC  directed  the  parties  to  
file  their  respective  pleadings  concerning  the  respondent's  existence  as  a  corporate  entity.  The  PCDP  alleged  
that  it  had  ceased  to  exist  as  a  corporation  on  July  24,  1989  and  that  it  has  winded  up  its  corporate  affairs  in  
accordance  with  law.  It  also  averred  that  it  was  now  owned  by  PCPPI.  
 
The  NLRC  issued  a  Resolution  dismissing  the  complaint  of  the  PCEWU  for  the  reason  that,  with  the  cessation  
and   dissolution   of   the   corporate   existence   of   the   PCDP,   rendering   any   judgment   against   it   is   incapable   of  
execution  and  satisfaction.  On  appeal,  the  CA  annulled  the  Resolution  of  the  NLRC.  It  ruled  that  PCDP  was  
still   in   existence   when   the   complaint   was   filed,   and   that   the   supervening   dissolution   of   the   corporation   did  
not  warrant  the  dismissal  of  the  complaint  against  it.  

   
3H  A.Y.  2017-­‐2018   210  
 
 

CORPORATION  LAW  CASE  DIGESTS  –  ATTY.  DANTE  DELA  CRUZ  

 
ISSUE:  
Did  PDCP  lost  its  corporate  personality  upon  the  PCPPI’s  acquisition  of  the  latter?  
 
HELD:  
NO.  Under  Section  122  of  the  Corporation  Code,  a  corporation  whose  corporate  existence  is  terminated  in  
any   manner   continues   to   be   a   body   corporate   for   three   (3)   years   after   its   dissolution   for   purposes   of  
prosecuting   and   defending   suits   by   and   against   it   and   to   enable   it   to   settle   and   close   its   affairs,   culminating  
in   the   disposition   and   distribution   of   its   remaining   assets.   It   may,   during   the   three-­‐‑year   term,   appoint   a  
trustee  or  a  receiver  who  may  act  beyond  that  period.  
 
The   termination   of   the   life   of   a   corporate   entity   does   not   by   itself   cause   the   extinction   or   diminution   of   the  
rights   and   liabilities   of   such   entity.   If   the   three-­‐‑year   extended   life   has   expired   without   a   trustee   or   receiver  
having  been  expressly  designated  by  the  corporation,  within  that  period,  the  board  of  directors  (or  trustees)  
itself,   may   be   permitted   to   so   continue   as   "trustees"   by   legal   implication   to   complete   the   corporate  
liquidation.  
 
 
 
165.  NATIONAL  ABACA  AND  OTHER  FIBERS  CORPORATION  VS.  APOLONIA  PORE  
G.R.  NO.  L-­‐‑16779                          AUGUST  16,  1961  
CONCEPCION,  J.  
 
DOCTRINE:  It  is  to  be  noted  that  the  time  during  which  the  corporation,  through  its  own  officers,  may  conduct  
the  liquidation  of  its  assets  and  sue  and  be  sued  as  a  corporation  is  limited  to  three  years  from  the  time  the  
period   of   dissolution   commences;   but   that   there   is   no   time   limited   within   the   trustees   must   complete   a  
liquidation  placed  in  their  hands.  It  is  provided  only  that  the  conveyance  to  the  trustees  must  be  made  within  
the  three-­‐‑year  period.  It  may  be  found  impossible  to  complete  the  work  of  liquidation  within  the  three-­‐‑year  
period  or  to  reduce  disputed  claims  to  judgment.  The  authorities  are  to  the  effect  that  suits  by  or  against  a  
corporation  abate  when  it  ceased  to  be  an  entity  capable  of  suing  or  being  sued  but  trustees  to  whom  the  
corporate  assets  have  been  conveyed  pursuant  to  the  authority  of  section  78  may  used  and  be  sued  as  such  
in  all  matters  connected  with  the  liquidation.  By  the  terms  of  the  statute  the  effect  of  the  conveyance  is  to  
make   the   trustees   the   legal   owners   of   the   property   conveyed,   subject   to   the   beneficial   interest   therein   of  
creditors  and  stockholders.    
 
FACTS:   On   November   14,   1953,   plaintiff   filed   with   the   Municipal   Court   of   Tacloban,   Leyte,   a   complaint,  
against   defendant   Apolonia   Pore,   for   the   recovery   of   P1,213.34,   allegedly   advanced   to   her   for   the   purchase  
of   hemp   for   the   account   of   the   former   and   for   which   she   had   allegedly   failed   to   account.   In   her   answer,  
defendant   alleged   that   she   had   accounted   for   all   cash   advances   received   by   her   for   the   aforementioned  
purpose  from  the  plaintiff.  In  due  course,  said  court  rendering  judgment  finding  that  the  defendant  had  not  
accounted  for  cash  advances  which  she  was,  accordingly,  sentenced  to  pay  to  the  plaintiff.  
 
Said   court   having   subsequently   denied   a   reconsideration   of   this   decision,   as   well   a   new   trial   prayed   for   the  
plaintiff,  the  latter  appealed  to  the  Court  of  First  Instance  of  Leyte,  in  which  defendant  moved  to  dismiss  the  
complaint  upon  the  ground  that  plaintiff  has  no  legal  capacity  to  sue,  it  having  abolished  by  Executive  Order  
No.   372   of   the   President   of   the   Philippines,   dated   November   24,1950.   Plaintiff   objected   thereto   upon   the  
ground  that  pursuant  to  said  executive  order,  plaintiff  "shall  nevertheless  be  continued  as  a  body  corporate  
for  a  period  of  three  (3)  years  from  the  effective  date"  of  said  executive  order,  which  was  November  30,  1950,  
"for  the  purpose  of  prosecuting  and  defending  suits  by  or  against  it  and  of  enabling  the  Board  of  Liquidators"  

   
3H  A.Y.  2017-­‐2018   211  
 
 

CORPORATION  LAW  CASE  DIGESTS  –  ATTY.  DANTE  DELA  CRUZ  

—  thereby  created  —  "gradually  to  settle  and  close  its  affairs",  .  .  .  and  that  this  case  was  begun  on  November  
14,  1953,  or  before  the  expiration  of  the  period  aforementioned.  
 
After  due  hearing,  the  court  of  first  instance  issued  an  order  directing  plaintiff  to  amend  the  complaint,  within  
ten  (10)  days  from  notice,  by  including  the  Board  of  Liquidators  as  co-­‐‑party  plaintiff,  with  the  admonition  
that  otherwise  the  case  would  be  dismissed.  Hence,  an  appeal  by  plaintiff  National  Abaca  and  other  Fibers  
Corporation,  from  two  (2)  orders  of  the  Court  of  First  Instance  of  Leyte.  
 
 
ISSUE:   Whether   or   not   an   action,   commenced   within   three   (3)   years   after   the   abolition   of   plaintiff,   as   a  
corporation,  may  be  continued  by  the  same  after  the  expiration  of  said  period?  
 
HELD:   No.   The   rule   appears   to   be   well   settled   that,   in   the   absence   of   statutory   provision   to   the   contrary,  
pending  actions  by  or  against  a  corporation  are  abated  upon  expiration  of  the  period  allowed  by  law  for  the  
liquidation  of  its  affairs.  
 
Our   Corporation   Law   contains   no   provision   authorizing   a   corporation,   after   three   (3)   years   from   the  
expiration  of  its  lifetime,  to  continue  in  its  corporate  name  actions  instituted  by  it  within  said  period  of  three  
(3)  years.  in  fact,  section  77  of  said  law  provides  that  the  corporation  shall  "be  continued  as  a  body  corporate  
for  three  (3)  years  after  the  time  when  it  would  have  been  .  .  .  dissolved,  for  the  purposed  of  prosecuting  and  
defending   suits   by   or   against   it   .   .   .",   so   that,   thereafter,   it   shall   no   longer   enjoy   corporate   existence   for   such  
purpose.  For  this  reason,  section  78  of  the  same  law  authorizes  the  corporation,  "at  any  time  during  said  
three  years  .  .  .  to  convey  all  of  its  property  to  trustees  for  the  benefit  of  members,  stockholders,  creditors  
and  other  interested",  evidently  for  the  purpose,  among  others,  of  enabling  said  trustees  to  prosecute  and  
defend  suits  by  or  against  the  corporation  begun  before  the  expiration  of  said  period.  
 
Obviously,   the   complete   loss   of   plaintiff's   corporate   existence   after   the   expiration   of   the   period   of   three   (3)  
years   for   the   settlement   of   its   affairs   is   what   impelled   the   President   to   create   a   Board   of   Liquidators,   to  
continue  the  management  of  such  matters  as  may  then  be  pending.  The  first  question  must,  therefore,  be  
answered  in  the  negative.  
 
 
 
166.  CHINA  BANKING  V.  MICHELLIN  
 
 
 
167.  REPUBLIC  OF  THE  PHILIPPINES  VS.  MARSMAN  DEVELOPMENT  COMPANY    
GR  NO.  18956/  APRIL  27,  1972  
BARREDO,  J.;  
 
DOCTRINE:  While  section  77  of  the  Corporation  Law  provides  for  a  three-­‐‑year  period  for  the  continuation  of  
the   corporate   existence   of   the   corporation   for   purposes   of   liquidation,   there   is   nothing   in   said   provision  
which  bars  an  action  for  the  recovery  of  the  debts  of  the  corporation  against  the  liquidator  thereof,  after  the  
lapse  of  the  said  three-­‐‑year  period.  
 
FACTS:  Defendant  corporation  was  a  timber  licensee  with  concessions  in  Camarines  Norte.  An  investigation  
was   conducted   on   the   business   operation   and   activities   of   the   corporation   leading   to   the   discovery   that  
certain   taxes   were   due   (from)   it   on   logs   produced   from   its   concession.   The   three   assessments   totalling  
P59,133.78  are  the  subject  matter  of  the  instant  case  for  collection.  

   
3H  A.Y.  2017-­‐2018   212  
 
 

CORPORATION  LAW  CASE  DIGESTS  –  ATTY.  DANTE  DELA  CRUZ  

 
Atty.  Moya,  in  behalf  of  the  corporation,  received  the  first  2  assessments.  He  requested  for  reinvestigation.  
As  a  result,  corporation  failed  to  pay  within  the  prescribed  period.  Numerous  BIR  warnings  were  given.  After  
3  years  of  futile  notifications,  BIR  sued  the  corporation.  
 
The  defendants  contend  that  the  present  action  is  already  barred  under  section  77  of  the  Corporation  Law  
which   allows   the   corporate   existence   of   a   corporation   to   continue   only   for   three   years   after   its   dissolution,  
for  the  purpose  of  presenting  or  defending  suits  by  or  against  it,  and  to  settle  and  close  its  affairs.  They  point  
out   that   inasmuch   as   the   Marsman   Development   Co.   was   extra-­‐‑judicially   dissolved   on   April   23,   1954,   a   fact  
admitted   in   the   amended   complaint,   the   filing   of   both   the   original   complaint   on   September   8,   1958   and   the  
amended  complaint  on  August  26,  1956  was  beyond  the  aforesaid  three-­‐‑year  period.  
 
ISSUE:   WON   present   action   is   barred   by   prescription,   in   light   of   the   fact   that   the   corporation   law   allows  
corporations  to  continue  only  for  3  years  after  its  dissolution,  for  the  purpose  of  presenting  or  defending  
suits  by  or  against  it,  and  to  settle  its  affairs.  
 
 
HELD:  NO.  Although  it  is  an  admitted  fact  that  the  defendant  corporation  was  extrajudicially  dissolved  on  
April  23,  1954,  there  is  no  claim  that  the  affairs  of  said  corporation  had  already  been  finally  liquidated  or  
settled.   Evidently,   Mr.   F.H.   Burgess   is   still   continuing   in   his   aforesaid   capacity   as   liquidator   of   the   Marsman  
Development   Co.   While   section   77   of   the   Corporation   Law   provides   for   a   three-­‐‑year   period   for   the  
continuation  of  the  corporate  existence  of  the  corporation  for  purposes  of  liquidation,  there  is  nothing  in  said  
provision  which  bars  an  action  for  the  recovery  of  the  debts  of  the  corporation  against  the  liquidator  thereof,  
after  the  lapse  of  the  said  three-­‐‑year  period.  
 
Section  78,  adds  for  clarification:  
 
At   any   time   during   said   three   years   said   corporation   is   authorized   and   empowered   to   convey   all   of   its  
property  to  trustees  for  the  benefit  of  members,  stock-­‐‑holders,  creditors,  and  others  interested.  From  and  
after   any   such   conveyance   by   the   corporation   of   its   property   in   trust   for   the   benefit   of   its   members,  
stockholders,   creditors,   and   others   in   interest,   all   interest   which   the   corporation   had   in   the   property  
terminates,  the  legal  interest  vests  in  the  trustee,  and  the  beneficial  interest  in  the  members,  stockholders,  
creditors,  or  other  persons  in  interest.  
 
It   is   to   be   recalled   that   the   assessments   against   appellant   corporation   for   deficiency   taxes   due   for   its  
operations  since  1947  were  made  by  the  Bureau  of  Internal  Revenue  on  October  15,  1953,  September  13,  
1954  and  November  8,  1954,  such  that  the  first  was  before  its  dissolution  and  the  last  two  not  later  than  six  
months  after  such  dissolution.  Thus,  in  whatever  way  the  matter  may  be  viewed,  the  Government  became  
the   creditor   of   the   corporation   before   the   completion   of   its   dissolution   by   the   liquidation   of   its   assets.  
Appellant  F.H.  Burgess,  whom  it  chose  as  liquidator,  became  in  law  the  trustee  of  all  its  assets  for  the  benefit  
of   all   persons   enumerated   in   Section   78,   including   its   creditors,   among   whom   is   the   Government,   for   the  
taxes   herein   involved.   To   assume   otherwise   would   render   the   extra-­‐‑judicial   dissolution   illegal   and   void,  
since,   according   to   Section   62   of   the   Corporation   Law,   such   kind   of   dissolution   is   permitted   only   when   it  
"does  not  affect  the  rights  of  any  creditor  having  a  claim  against  the  corporation."  It  is  immaterial  that  the  
present   action   was   filed   after   the   expiration   of   three   years   after   April   23,   1954,   for   at   the   very   least,   and  
assuming   that   judicial   enforcement   of   taxes   may   not   be   initiated   after   said   three   years   despite   the   fact   that  
the  actual  liquidation  has  not  been  terminated  and  the  one  in  charge  thereof  is  still  holding  the  assets  of  the  
corporation,   obviously   for   the   benefit   of   all   the   creditors   thereof,   the   assessment   aforementioned,   made  
within  the  three  years,  definitely  established  the  Government  as  a  creditor  of  the  corporation  for  whom  the  
liquidator  is  supposed  to  hold  assets  of  the  corporation.  And  since  the  suit  at  bar  is  only  for  the  collection  of  

   
3H  A.Y.  2017-­‐2018   213  
 
 

CORPORATION  LAW  CASE  DIGESTS  –  ATTY.  DANTE  DELA  CRUZ  

taxes   finally   assessed   against   the   corporation   within   the   three   years   invoked   by   appellants,   their   fourth  
assignment  of  error  cannot  be  sustained.    
 
 
 
168.    
 
 
 
169.   VITALIANO   N.   AGUIRRES   II   AND   FIDEL   N.   AGUIRRE   VS.   FQB+7,   INC.,   NATHANIEL   D.   BOCOBO,  
PRISCILA  BOCOBO  AND  ANTONIO  DE  VILLA  
G.R.  NO.  170770  JANUARY  9,  2013  
DEL  CASTILLO,  J.:  
 
DOCTRINE:  Pursuant  to  Section  145  of  the  Corporation  Code,  an  existing  intra-­‐‑corporate  dispute,  which  does  
not   constitute   a   continuation   of   corporate   business,   is   not   affected   by   the   subsequent   dissolution   of   the  
corporation.  The  dissolution  of  the  corporation  simply  prohibits  it  from  continuing  its  business.  However,  
despite  such  dissolution,  the  parties  involved  in  the  litigation  are  still  corporate  actors.  The  dissolution  does  
not   automatically   convert   the   parties   into   total   strangers   or   change   their   intra-­‐‑corporate   relationships.  
Neither   does   it   change   or   terminate   existing   causes   of   action,   which   arose   because   of   the   corporate   ties  
between   the   parties.   Thus,   a   cause   of   action   involving   an   intra-­‐‑corporate   controversy   remains   and   must   be  
filed  as  an  intra-­‐‑corporate  dispute  despite  the  subsequent  dissolution  of  the  corporation.  
 
FACTS:  On  October  5,  2004,  Vitaliano  filed,  in  his  individual  capacity  and  on  behalf  of  FQB+7,  Inc.  (FQB+7),  a  
Complaint  for  intra-­‐‑corporate  dispute,  injunction,  inspection  of  corporate  books  and  records,  and  damages,  
against  respondents  Nathaniel  D.  Bocobo,  Priscila  D.  Bocobo,  and  Antonio  De  Villa.  The  Complaint  alleged  
that  there  were  substantive  changes  found  in  the  General  Information  Sheet  of  FQB+7  in  the  SEC  records  
respecting  the  directors  and  subscribers  of  FQB+7  sometime  in  year  2002,  prompted  Vitaliano  to  write  to  
the  "real"  Board  of  Directors,  represented  by  Fidel  N.  Aguirre.  Then,  Vitaliano  questioned  the  validity  and  
truthfulness  of  the  alleged  stockholders  meeting  held  on  September  3,  2002.  He  asked  the  "real"  Board  to  
rectify  what  he  perceived  as  erroneous  entries  in  the  GIS,  and  to  allow  him  to  inspect  the  corporate  books  
and  records.  The  "real"  Board  allegedly  ignored  Vitaliano’s  request.  
 
On  September  27,  2004,  Nathaniel,  in  the  exercise  of  his  power  as  FQB+7’s  president,  appointed  Antonio  as  
the   corporation’s   attorney-­‐‑in-­‐‑fact,   with   power   of   administration   over   the   corporation’s   farm   in   Quezon  
Province.   Antonio   attempted   to   take   over   the   farm,   but   was   allegedly   prevented   by   Fidel   and   his   men.  
Characterizing   Nathaniel’s,   Priscila’s,   and   Antonio’s   continuous   representation   of   the   corporation   as   a  
usurpation  of  the  management  powers  and  prerogatives  of  the  "real"  Board  of  Directors,  the  Complaint  asked  
for  an  injunction  against  them  and  for  the  nullification  of  all  their  previous  actions  as  purported  directors,  
including  the  GIS  they  had  filed  with  the  SEC.  The  Complaint  also  sought  damages  for  the  plaintiffs  and  a  
declaration  of  Vitaliano’s  right  to  inspect  the  corporate  records.  The  trial  court  issued  the  writ  of  preliminary  
injunction  after  Vitaliano  filed  an  injunction  bond.  
 
The   respondents   filed   a   Petition   for   Certiorari   and   Prohibition   before   the   CA.     The   respondents   sought,   the  
annulment  of  all  the  proceedings  and  issuances  in  a  SEC  case  on  the  ground  that  Branch  24  of  the  Manila  RTC  
has  no  jurisdiction  over  the  subject  matter,  which  they  defined  as  being  an  agrarian  dispute.  They  theorized  
that  Vitaliano’s  real  goal  in  filing  the  Complaint  was  to  maintain  custody  of  the  corporate  farm  in  Quezon  
Province.  Since  this  land  is  agricultural  in  nature,  they  claimed  that  jurisdiction  belongs  to  the  Department  
of  Agrarian  Reform  (DAR),  not  to  the  Manila  RTC.  Respondents  also  raised  their  defenses  to  Vitaliano’s  suit,  
particularly  the  alleged  disloyalty  and  fraud  committed  by  the  "real"  Board  of  Directors,    and  respondents’  

   
3H  A.Y.  2017-­‐2018   214  
 
 

CORPORATION  LAW  CASE  DIGESTS  –  ATTY.  DANTE  DELA  CRUZ  

"preferential  right  to  possess  the  corporate  property"  as  the  heirs  of  the  majority  stockholder  Francisco  Q.  
Bocobo.  
 
The   respondents   further   informed   the   CA   that   the   SEC   had   already   revoked   FQB+7’s   Certificate   of  
Registration   on   September   29,   2003   for   its   failure   to   comply   with   the   SEC   reportorial   requirements.30   The  
CA   determined   that   the   corporation’s   dissolution   was   a   conclusive   fact   after   petitioners   Vitaliano   and   Fidel  
failed  to  dispute  this  factual  assertion  
 
ISSUE:  Whether  the  RTC  has  jurisdiction  over  an  intra-­‐‑corporate  dispute  involving  a  dissolved  corporation.  
 
HELD:  Yes.  Jurisdiction  over  the  subject  matter  is  conferred  by  law.  R.A.  No.  879945  conferred  jurisdiction  
over  intra-­‐‑corporate  controversies  on  courts  of  general  jurisdiction  or  RTCs,  to  be  designated  by  the  Supreme  
Court.   Thus,   as   long   as   the   nature   of   the   controversy   is   intra-­‐‑corporate,   the   designated   RTCs   have   the  
authority  to  exercise  jurisdiction  over  such  cases.  
 
To  be  considered  as  an  intra-­‐‑corporate  dispute,  the  case:  (a)  must  arise  out  of  intra-­‐‑corporate  or  partnership  
relations,  and  (b)  the  nature  of  the  question  subject  of  the  controversy  must  be  such  that  it  is  intrinsically  
connected  with  the  regulation  of  the  corporation  or  the  enforcement  of  the  parties’  rights  and  obligations  
under  the  Corporation  Code  and  the  internal  regulatory  rules  of  the  corporation.  So  long  as  these  two  criteria  
are  satisfied,  the  dispute  is  intra-­‐‑corporate  and  the  RTC,  acting  as  a  special  commercial  court,  has  jurisdiction  
over  it.  In  the  case  at  bar,  the  Court  finds  and  so  holds  that  the  case  is  essentially  an  intra-­‐‑corporate  dispute.  
It  obviously  arose  from  the  intra-­‐‑corporate  relations  between  the  parties,  and  the  questions  involved  pertain  
to   their   rights   and   obligations   under   the   Corporation   Code   and   matters   relating   to   the   regulation   of   the  
corporation.  The  Court  futher  hold  that  the  nature  of  the  case  as  an  intra-­‐‑corporate  dispute  was  not  affected  
by  the  subsequent  dissolution  of  the  corporation.  
 
It  bears  reiterating  that  Section  145  of  the  Corporation  Code  protects,  among  others,  the  rights  and  remedies  
of   corporate   actors   against   other   corporate   actors.   The   statutory   provision   assures   an   aggrieved   party   that  
the   corporation’s   dissolution   will   not   impair,   much   less   remove,   his/her   rights   or   remedies   against   the  
corporation,  its  stockholders,  directors  or  officers.  It  also  states  that  corporate  dissolution  will  not  extinguish  
any  liability  already  incurred  by  the  corporation,  its  stockholders,  directors,  or  officers.  In  short,  Section  145  
preserves  a  corporate  actor’s  cause  of  action  and  remedy  against  another  corporate  actor.  In  so  doing,  Section  
145  also  preserves  the  nature  of  the  controversy  between  the  parties  as  an  intra-­‐‑corporate  dispute.  
 
The  dissolution  of  the  corporation  simply  prohibits  it  from  continuing  its  business.  However,  despite  such  
dissolution,   the   parties   involved   in   the   litigation   are   still   corporate   actors.   The   dissolution   does   not  
automatically  convert  the  parties  into  total  strangers  or  change  their  intra-­‐‑corporate  relationships.  Neither  
does  it  change  or  terminate  existing  causes  of  action,  which  arose  because  of  the  corporate  ties  between  the  
parties.  Thus,  a  cause  of  action  involving  an  intra-­‐‑corporate  controversy  remains  and  must  be  filed  as  an  
intra-­‐‑corporate  dispute  despite  the  subsequent  dissolution  of  the  corporation.  
 
 
 
170.    REYES  V.  BLOUSE  
 
 
171.    THE  EDWARD  NELL  CO.  V.  PACIFIC  FARMS  INC.  
G.R.  NO.  L-­‐‑20850;  NOVEMBER  29,  1965  
CONCEPCION,  J;  

   
3H  A.Y.  2017-­‐2018   215  
 
 

CORPORATION  LAW  CASE  DIGESTS  –  ATTY.  DANTE  DELA  CRUZ  

Doctrine:  The  rule  is  set  forth  in  Fletcher  Cyclopedia  Corporations,  “Generally  where  one  corporation  sells  
or   otherwise   transfers   all   of   its   assets   to   another   corporation,   the   latter   is   not   liable   for   the   debts   and  
liabilities  of  the  transferor,  except:  (1)  where  the  purchaser  expressly  or  impliedly  agrees  to  assume  such  
debts;  (2)  where  the  transaction  amounts  to  a  consolidation  or  merger  of  the  corporations;  (3)  where  the  
purchasing  corporation  is  merely  a  continuation  of  the  selling  corporation;  and  (4)  where  the  transaction  is  
entered  into  fraudulently  in  order  to  escape  liability  for  such  debts.  “  
 
Facts:  On  October  9,  1958,  appellant  secured  in  Civil  Case  No.  58579  of  the  Municipal  Court  of  Manila  against  
Insular   Farms,   Inc.   —   hereinafter   referred   to   as   Insular   Farms   a   judgment   for   the   sum   of   P1,853.80   —  
representing  the  unpaid  balance  of  the  price  of  a  pump  sold  by  appellant  to  Insular  Farms  —  with  interest  
on  said  sum  plus  other  costs.  A  writ  of  execution,  issued  after  the  judgment  had  become  final,  was,  on  August  
14,  1959,  returned  unsatisfied,  stating  that  Insular  Farms  had  no  leviable  property.  Soon  thereafter,  appellant  
filed  with  said  court  the  present  action  against  Pacific  Farms,  Inc.  —  hereinafter  referred  to  as  appellee  —  
for  the  collection  of  the  judgment  aforementioned,  upon  the  theory  that  appellee  is  the  alter  ego  of  Insular  
Farms,   which   appellee   has   denied.   In   due   course,   the   municipal   court   rendered   judgment   dismissing  
appellant's   complaint.   Appellant   appealed,   with   the   same   result,   to   the   court   of   first   instance   and,  
subsequently,  to  the  Court  of  Appeals  
 
Issue:   Whether   or   not   CA   erred   in   not   holding   liable   the   appellee   liable   for   the   said   unpaid   obligation   of  
Insular  Farms  
 
Ruling:  It  should  be  noted  that  appellant's  complaint  in  the  municipal  court  was  anchored  upon  the  theory  
that  appellee  is  an  alter  ego  of  Insular  Farms,  because  the  former  had  purchased  all  or  substantially  all  of  the  
shares  of  stock,  as  well  as  the  real  and  personal  properties  of  the  latter,  including  the  pumping  equipment  
sold  by  appellant  to  Insular  Farms.  The  record  shows  that,  on  March  21,  1958,  appellee  purchased  1,000  
shares  of  stock  of  Insular  Farms  for  P285,126.99;  that,  thereupon,  appellee  sold  said  shares  of  stock  to  certain  
individuals,   who   forthwith   reorganized   said   corporation;   and   that   the   board   of   directors   thereof,   as  
reorganized,   then   caused   its   assets,   including   its   leasehold   rights   over   a   public   land   in   Bolinao,   Pangasinan,  
to  be  sold  to  herein  appellee  for  P10,000.00.  We  agree  with  the  Court  of  Appeals  that  these  facts  do  not  prove  
that  the  appellee  is  an  alter  ego  of  Insular  Farms,  or  is  liable  for  its  debts.    
 
In  the  case  at  bar,  there  is  neither  proof  nor  allegation  that  appellee  had  expressly  or  impliedly  agreed  to  
assume  the  debt  of  Insular  Farms  in  favor  of  appellant  herein,  or  that  the  appellee  is  a  continuation  of  Insular  
Farms,  or  that  the  sale  of  either  the  shares  of  stock  or  the  assets  of  Insular  Farms  to  the  appellee  has  been  
entered   into   fraudulently,   in   order   to   escape   liability   for   the   debt   of   the   Insular   Farms   in   favor   of   appellant  
herein.  In  fact,  these  sales  took  place  (March,  1958)  not  only  over  six  (6)  months  before  the  rendition  of  the  
judgment   (October   9,  1958)   sought   to   be   collected   in  the   present   action,   but,   also,   over   a   month   before   the  
filing  of  the  case  (May  29,  1958)  in  which  said  judgment  was  rendered.  Moreover,  appellee  purchased  the  
shares  of  stock  of  Insular  Farms  as  the  highest  bidder  at  an  auction  sale  held  at  the  instance  of  a  bank  to  
which  said  shares  had  been  pledged  as  security  for  an  obligation  of  Insular  Farms  in  favor  of  said  bank.  It  
has,  also,  been  established  that  the  appellee  had  paid  P285,126.99  for  said  shares  of  stock,  apart  from  the  
sum  of  P10,000.00  it,  likewise,  paid  for  the  other  assets  of  Insular  Farms.  
 
Neither  is  it  claimed  that  these  transactions  have  resulted  in  the  consolidation  or  merger  of  the  Insular  Farms  
and  appellee  herein.  On  the  contrary,  appellant's  theory  to  the  effect  that  appellee  is  an  alter  ego  of  the  Insular  
Farms  negates  such  consolidation  or  merger,  for  a  corporation  cannot  be  its  own  alter  ego.  
 
It   is   urged,   however,   that   said   P10,000.00   paid   by   appellee   for   other   assets   of   Insular   Farms   is   a   grossly  
inadequate   price,   because,   appellant   now   claims,   said   assets   were   worth   around   P285,126.99,   and   that,  
consequently,  the  sale  must  be  considered  fraudulent.  However,  the  sale  was  submitted  to  and  approved  by  

   
3H  A.Y.  2017-­‐2018   216  
 
 

CORPORATION  LAW  CASE  DIGESTS  –  ATTY.  DANTE  DELA  CRUZ  

the  Securities  and  Exchange  Commission.  It  must  be  presumed,  therefore,  that  the  price  paid  was  fair  and  
reasonable.  Moreover,  the  only  issue  raised  in  the  court  of  origin  was  whether  or  not  appellee  is  an  alter  ego  
of  Insular  Farms.  The  question  of  whether  the  aforementioned  sale  of  assets  for  P10,000.00  was  fraudulent  
or  not,  had  not  been  put  in  issue  in  said  court.  Hence,  it  may,  not  be  raised  on  appeal.  
 
 
 
172.  BANK  OF  COMMERCE  V.  RADIO  PHILIPPINES  
 
 
 
173.     MARSHALL-­‐‑WELLS  COMPANY  VS.  HENRY  W.  ELSER  &  CO.,  INC.  
G.R.  NO.  22015                      SEPTEMBER  1,  1924  
MALCOLM,  J.  
 
DOCTRINE:  
  It  was  never  the  purpose  of  the  Legislature  to  exclude  a  foreign  corporation  which  happens  to  obtain  
an  isolated  order  for  business  from  the  Philippines,  from  securing  redress  in  the  Philippine  courts,  and  thus,  
in  effect,  to  permit  persons  to  avoid  their  contracts  made  with  such  foreign  corporations.  
 
FACTS:  
  Marshall-­‐‑Wells   Company,   an   Oregon   corporation,   sued   Henry   W.   Elser   &   Co.,   Inc.,   a   domestic  
corporation,  in  the  Court  of  First  Instance  of  Manila,  for  the  unpaid  balance  of  a  bill  of  goods  amounting  to  
P2,660.74,   sold   by   plaintiff   to   defendant   and   for   which   plaintiff   holds   accepted   drafts.   Defendant   demurred  
to   the   complaint   on   the   statutory   ground   that   the   plaintiff   has   not   legal   capacity   to   sue.   In   the   demurrer,  
counsel  stated  that  "The  said  complaint  does  not  show  that  the  plaintiff  has  complied  with  the  laws  of  the  
Philippine  Islands  in  that  which  is  required  of  foreign  corporations  desiring  to  do  business  in  the  Philippine  
Islands,  neither  does  it  show  that  it  was  authorized  to  do  business  in  the  Philippine  Islands."  The  demurrer  
was   sustained   by   the   trial   judge.   Inasmuch   as   the   plaintiff   could   not   allege   compliance   with   the   statute,   the  
order  was  allowed  to  become  final  and  an  appeal  was  perfected.  
 
ISSUE:  
  Whether   the   obtaining   of   the   license   a   condition   precedent   to   the   maintaining   of   any   kind   of   action  
in  the  courts  of  the  Philippine  Islands  by  a  foreign  corporation?  
 
HELD:  
  Section  69  of  the  Corporation  Law,  its  literal  terminology  is  as  follows:  
 
No  foreign  corporation  or  corporation  formed,  organized,  or  existing  under  any  laws  other  that  those  of  the  
Philippine  Islands  shall  be  permitted  to  transact  business  in  the  Philippine  Islands  or  maintain  by  itself  or  
assignee  any  suit  for  the  recovery  of  any  debt,  claim,  or  demand  whatever,  unless  it  shall  have  the  license  
prescribed  in  the  section  immediately  preceding.  Any  officer,  director,  or  agent  of  the  corporation  not  having  
the  license  prescribed  shall  be  punished  by  imprisonment  for  not  less  than  six  months  nor  more  than  two  
years  or  by  a  fine  of  not  less  than  two  hundred  pesos  nor  more  than  one  thousand  pesos,  or  by  both  such  
imprisonment  and  fine,  in  the  discretion  of  the  court.  
   
  Defendant  isolates  a  portion  of  one  sentence  of  section  69  of  the  Corporation  Law  and  asks  the  court  
to  give  it  a  literal  meaning.  Counsel  would  have  the  law  read  thus:  "No  foreign  corporation  shall  be  permitted  
to   maintain   by   itself   or   assignee   any   suit   for   the   recovery   of   any   debt,   claim,   or   demand   whatever,   unless   it  
shall   have   the   license   prescribed   in   section   68   of   the   law."   Plaintiff,   on   the   contrary,   desires   for   the   court   to  

   
3H  A.Y.  2017-­‐2018   217  
 
 

CORPORATION  LAW  CASE  DIGESTS  –  ATTY.  DANTE  DELA  CRUZ  

consider  the  particular  point  under  discussion  with  reference  to  all  the  law,  and  thereafter  to  give  the  law  a  
common  sense  interpretation.  
  The   object   of   the   statute   was   to   subject   the   foreign   corporation   doing   business   in   the   Philippines   to  
the   jurisdiction   of   its   courts.   The   object   of   the   statute   was   not   to   prevent   the   foreign   corporation   from  
performing  single  acts,  but  to  prevent  it  from  acquiring  a  domicile  for  the  purpose  of  business  without  taking  
the  steps  necessary  to  render  it  amenable  to  suit  in  the  local  courts.  The  implication  of  the  law  is  that  it  was  
never  the  purpose  of  the  Legislature  to  exclude  a  foreign  corporation  which  happens  to  obtain  an  isolated  
order  for  business  from  the  Philippines,  from  securing  redress  in  the  Philippine  courts,  and  thus,  in  effect,  to  
permit   persons   to   avoid   their   contracts   made   with   such   foreign   corporations.   The   effect   of   the   statute  
preventing  foreign  corporations  from  doing  business  and  from  bringing  actions  in  the  local  courts,  except  on  
compliance  with  elaborate  requirements,  must  not  be  unduly  extended  or  improperly  applied.  It  should  not  
be  construed  to  extend  beyond  the  plain  meaning  of  its  terms,  considered  in  connection  with  its  object,   and  
in  connection  with  the  spirit  of  the  entire  law.  
 
   
 
174.  COLUMBIA  PICTURES,  INC.  V.  COURT  OF  APPEALS  
G.R.  NO.  110318  /  AUGUST  28,  1996  
REGALADO,  J.  
 
DOCTRINE:  
 
See  Underscored  Portions  
 
FACTS:    
 
Complainants  lodged  a  complaint  with  the  NBI  for  violation  of  PD  No.  49  and  sought  its  assistance  in  their  
anti-­‐‑film   piracy   drive.   Agents   of   the   NBI   and   private   researchers   made   discreet   surveillance   on   video  
establishments   in   Metro   Manila   including   Sunshine   Home   Video   Inc.   owned   and   operated   by   Danilo   A.  
Pelindario  with  address  at  No.  6  Mayfair  Center,  Magallanes,  Makati,  Metro  Manila.  
 
NBI  Senior  Agent  Lauro  C.  Reyes  applied  for  a  search  warrant  with  the  court  a  quo  against  Sunshine  seeking  
the  seizure,  among  others,  of  pirated  video  tapes  of  copyrighted  films  all  of  which  were  enumerated  in  a  list  
attached  to  the  application;  and,  television  sets,  video  cassettes  and/or  laser  disc  recordings  equipment  and  
other   machines   and   paraphernalia   used   or   intended   to   be   used   in   the   unlawful   exhibition,   showing,  
reproduction,  sale,  lease  or  disposition  of  videograms  tapes  in  the  premises  above  described.  Search  Warrant  
No.  87-­‐‑053  for  violation  of  Section  56  of  PD  No.  49,  as  amended,  was  issued  by  the  court  a  quo.  
 
In  the  course  of  the  search  of  the  premises  indicated  in  the  search  warrant,  the  NBI  Agents  found  and  seized  
various  video  tapes  of  duly  copyrighted  motion  pictures/films  owned  or  exclusively  distributed  by  private  
complainants,  and  machines,  equipment,  television  sets,  paraphernalia,  materials,  accessories  all  of  which  
were  included  in  the  receipt  for  properties  accomplished  by  the  raiding  team.  
 
On  December  16,  1987,  a  "Return  of  Search  Warrant"  was  filed  with  the  Court.  
 
A  "Motion  To  Lift  the  Order  of  Search  Warrant"  was  filed  but  was  later  denied  for  lack  of  merit.  
 
A  Motion  for  reconsideration  of  the  Order  of  denial  was  filed.  The  court  a  quo  granted  the  said  motion  for  
reconsideration.  
 

   
3H  A.Y.  2017-­‐2018   218  
 
 

CORPORATION  LAW  CASE  DIGESTS  –  ATTY.  DANTE  DELA  CRUZ  

Petitioners   thereafter   appealed   the   order   of   the   trial   court   granting   private   respondents'   motion   for  
reconsideration,  thus  lifting  the  search  warrant  which  it  had  theretofore  issued,  to  the  Court  of  Appeals.  Said  
appeal  was  dismissed  and  the  motion  for  reconsideration  thereof  was  denied.  
 
ISSUE:  
WON  petitioner  has  legal  standing  to  maintain  the  present  action.  
 
HELD:  
 
The  Corporation  Code  provides:  
 
Sec.  133.  Doing  business  without  a  license.  —  No  foreign  corporation  transacting  business  in  the  Philippines  
without  a  license,  or  its  successors  or  assigns,  shall  be  permitted  to  maintain  or  intervene  in  any  action,  suit  
or  proceeding  in  any  court  or  administrative  agency  of  the  Philippines;  but  such  corporation  may  be  sued  or  
proceeded   against   before   Philippine   courts   or   administrative   tribunals   on   any   valid   cause   of   action  
recognized  under  Philippine  laws.  
 
The  obtainment  of  a  license  prescribed  by  Section  125  of  the  Corporation  Code  is  not  a  condition  precedent  
to  the  maintenance  of  any  kind  of  action  in  Philippine  courts  by  a  foreign  corporation.  However,  under  the  
aforequoted   provision,   no   foreign   corporation   shall   be   permitted   to   transact   business   in   the   Philippines,   as  
this   phrase   is   understood   under   the   Corporation   Code,   unless   it   shall   have   the   license   required   by   law,   and  
until  it  complies  with  the  law  in  transacting  business  here,  it  shall  not  be  permitted  to  maintain  any  suit  in  
local  courts.  As  thus  interpreted,  any  foreign  corporation  not  doing  business  in  the  Philippines  may  maintain  
an  action  in  our  courts  upon  any  cause  of  action,  provided  that  the  subject  matter  and  the  defendant  are  
within  the  jurisdiction  of  the  court.  It  is  not  the  absence  of  the  prescribed  license  but  "doing  business"  in  the  
Philippines  without  such  license  which  debars  the  foreign  corporation  from  access  to  our  courts.  In  other  
words,   although   a   foreign   corporation   is   without   license   to   transact   business   in   the   Philippines,   it   does   not  
follow  that  it  has  no  capacity  to  bring  an  action.  Such  license  is  not  necessary  if  it  is  not  engaged  in  business  
in  the  Philippines.  
 
No  general  rule  or  governing  principles  can  be  laid  down  as  to  what  constitutes  "doing"  or  "engaging  in"   or  
"transacting"   business.   Each   case   must   be   judged   in   the   light   of   its   own   peculiar   environmental  
circumstances.   The   true   tests,   however,   seem   to   be   whether   the   foreign   corporation   is   continuing   the   body  
or  substance  of  the  business  or  enterprise  for  which  it  was  organized  or  whether  it  has  substantially  retired  
from  it  and  turned  it  over  to  another.    
 
The  Corporation  Code  does  not  itself  define  or  categorize  what  acts  constitute  doing  or  transacting  business  
in  the  Philippines.  Jurisprudence  has,  however,  held  that  the  term  implies  a  continuity  of  commercial  dealings  
and  arrangements,  and  contemplates,  to  that  extent,  the  performance  of  acts  or  works  or  the  exercise  of  some  
of   the   functions   normally   incident   to   or   in   progressive   prosecution   of   the   purpose   and   subject   of   its  
organization.  
   
Accordingly,   the   certification   issued   by   the   Securities   and   Exchange   Commission   stating   that   its   records   do  
not  show  the  registration  of  petitioner  film  companies  either  as  corporations  or  partnerships  or  that  they  
have  been  licensed  to  transact  business  in  the  Philippines,  while  undeniably  true,  is  of  no  consequence  to  
petitioners'   right   to   bring   action   in   the   Philippines.   Verily,   no   record   of   such   registration   by   petitioners   can  
be   expected   to   be   found   for,   as   aforestated,   said   foreign   film   corporations   do   not   transact   or   do   business   in  
the  Philippines  and,  therefore,  do  not  need  to  be  licensed  in  order  to  take  recourse  to  our  courts.  
 

   
3H  A.Y.  2017-­‐2018   219  
 
 

CORPORATION  LAW  CASE  DIGESTS  –  ATTY.  DANTE  DELA  CRUZ  

The  fact  that  petitioners  are  admittedly  copyright  owners  or  owners  of  exclusive  distribution  rights  in  the  
Philippines  of  motion  pictures  or  films  does  not  convert  such  ownership  into  an  indicium  of  doing  business  
which  would  require  them  to  obtain  a  license  before  they  can  sue  upon  a  cause  of  action  in  local  courts.  
 
As  a  general  rule,  a  foreign  corporation  will  not  be  regarded  as  doing  business  in  the  State  simply  because  it  
enters  into  contracts  with  residents  of  the  State,  where  such  contracts  are  consummated  outside  the  State.  
In   fact,   a   view   is   taken   that   a   foreign   corporation   is   not   doing   business   in   the   State   merely   because   sales   of  
its  product  are  made  there  or  other  business  furthering  its  interests  is  transacted  there  by  an  alleged  agent,  
whether   a   corporation   or   a   natural   person,   where   such   activities   are   not   under   the   direction   and   control   of  
the  foreign  corporation  but  are  engaged  in  by  the  alleged  agent  as  an  independent  business.  
 
 
 
 
175.   GENERAL   GARMENTS   CORPORATION   VS.   THE   DIRECTOR   OF   PATENTS   AND   PURITAN  
SPORTSWEAR  CORPORATION  
G.R.  NO.  L-­‐‑24295  SEPTEMBER  30,  1971  
MAKALINTAL,  J.:  
 
DOCTRINE:  
Petitioner  argues  that  Section  21-­‐‑A  militates  against  respondent's  capacity  to  maintain  a  suit  for  cancellation,  
since  it  requires,  before  a  foreign  corporation  may  bring  an  action,  that  its  trademark  or  tradename  has  been  
registered   under   the   Trademark   Law.   The   argument   misses   the   essential   point   in   the   said   provision,   which  
is   that   the   foreign   corporation   is   allowed   there   under   to   sue   "whether   or   not   it   has   been   licensed   to   do  
business  in  the  Philippines"  pursuant  to  the  Corporation  Law.  
 
FACTS:  
The  General  Garments  Corporation,  organized  and  existing  under  the  laws  of  the  Philippines,  is  the  owner  of  
the   trademark   "Puritan,"   under   Registration   No.   10059   issued   on   November   15,   1962   by   the   Philippine  
Patent  Office,  for  assorted  men's  wear,  such  as  sweaters,  shirts,  jackets,  undershirts  and  briefs.  
On   March   9,   1964   the   Puritan   Sportswear   Corporation,   organized   and   existing   in   and   under   the   laws   of   the  
state   of   Pennsylvania,   U.S.A.,   filed   a   petition   with   the   Philippine   Patent   Office   for   the   cancellation   of   the  
trademark  "Puritan"  registered  in  the  name  of  General  Garments  Corporation,  alleging  ownership  and  prior  
use  in  the  Philippines  of  the  said  trademark  on  the  same  kinds  of  goods,  which  use  it  had  not  abandoned;  and  
alleging   further   that   the   registration   thereof   by   General   Garments   Corporation   had   been   obtained  
fraudulently  and  in  violation  of  Section  17(c)  of  Republic  Act  No.  166,  as  amended,  in  relation  to  Section  4(d)  
thereof.  
On  March  30,  1964  General  Garments  Corporation  moved  to  dismiss  the  petition  on  several  grounds,  all  of  
which  may  be  synthesized  in  one  single  issue:  whether  or  not  Puritan  Sportswear  Corporation,  which  is  a  
foreign  corporation  not  licensed  to  do  business  and  not  doing  business  in  the  Philippines,  has  legal  capacity  
to   maintain   a   suit   in   the   Philippine   Patent   Office   for   cancellation   of   a   trademark   registered   therein.   The  
Director   of   Patents   denied   the   motion   to   dismiss   on   August   6,   1964,   and   denied   likewise   the   motion   for  
reconsideration   on   March   5,   1965,   whereupon   General   Garments   Corporation,   hereinafter   referred   to   as  
petitioner,  filed  the  instant  petition  for  review.  
Petitioner  contends  that  Puritan  Sportswear  Corporation  (hereinafter  referred  to  as  respondent),  being  a  
foreign  corporation  which  is  not  licensed  to  do  and  is  not  doing  business  in  the  Philippines,  is  not  considered  
as  a  person  under  Philippine  laws  and  consequently  is  not  comprehended  within  the  term  "any  person"  who  
may  apply  for  cancellation  of  a  mark  or  trade-­‐‑name  under  Section  17(c)  of  the  Trademark  Law  
 
ISSUE:  

   
3H  A.Y.  2017-­‐2018   220  
 
 

CORPORATION  LAW  CASE  DIGESTS  –  ATTY.  DANTE  DELA  CRUZ  

 
Whether  or  not  the  petitioner  is  entitled  to  use  the  trademark  “Puritan.”  
 
HELD:  
Petitioner  argues  that  the  ruling  in  Western  Equipment  has  been  superseded  by  the  later  decision  of  this  
Court   in   Mentholatum   Co.,   Inc.   v.   Mangaliman   (1941),   72   Phil.   524,   where   it   was   held   that   inasmuch   as  
Mentholatum   Co.,   Inc.   was   a   foreign   corporation   doing   business   in   the   Philippines   without   the   license  
required   by   Section   68   of   the   Corporation   Law   it   could   not   prosecute   an   action   for   infringement   of   its  
trademark   which   was   the   subject   of   local   registration.   The   court   itself,   however,   recognized   a   distinction  
between   the   two   cases,   in   that   in   Western   Equipment   the   foreign   corporation   was   not   engaged   in   business  
in  the  Philippines,  and  observed  that  if  it  had  been  so  engaged  without  first  obtaining  a  license  "another  and  
a  very  different  question  would  be  presented."  
Parenthetically,  it  may  be  stated  that  the  ruling  in  the  Mentholatum  case  was  subsequently  derogated  when  
Congress,  purposely  to  "counteract  the  effects"  of  said  case,  enacted  Republic  Act  No.  638,  inserting  Section  
21-­‐‑A   in   the   Trademark   Law,   which   allows   a   foreign   corporation   or   juristic   person   to   bring   an   action   in  
Philippine  courts  for  infringement  of  a  mark  or  trade-­‐‑name,  for  unfair  competition,  or  false  designation  of  
origin  and  false  description,  "whether  or  not  it  has  been  licensed  to  do  business  in  the  Philippines  under  Act  
Numbered  Fourteen  hundred  and  fifty-­‐‑nine,  as  amended,  otherwise  known  as  the  Corporation  Law,  at  the  
time  it  brings  complaint."  
Petitioner  argues  that  Section  21-­‐‑A  militates  against  respondent's  capacity  to  maintain  a  suit  for  cancellation,  
since  it  requires,  before  a  foreign  corporation  may  bring  an  action,  that  its  trademark  or  tradename  has  been  
registered   under   the   Trademark   Law.   The   argument   misses   the   essential   point   in   the   said   provision,   which  
is   that   the   foreign   corporation   is   allowed   there   under   to   sue   "whether   or   not   it   has   been   licensed   to   do  
business   in   the   Philippines"   pursuant   to   the   Corporation   Law   (precisely   to   counteract   the   effects   of   the  
decision  in  the  Mentholatum  case).  
In  any  event,  respondent  in  the  present  case  is  not  suing  for  infringement  or  unfair  competition  under  Section  
21-­‐‑A,  but  for  cancellation  under  Section  17,  on  one  of  the  grounds  enumerated  in  Section  4.  The  first  kind  of  
action,   it   maybe   stated,   is   cognizable   by   the   Courts   of   First   Instance   (Sec.   27);   the   second   partakes   of   an  
administrative   proceeding   before   the   Patent   Office   (Sec.   18,   in   relation   to   Sec.   8).   And   while   a   suit   under  
Section  21-­‐‑A  requires  that  the  mark  or  tradename  alleged  to  have  been  infringed  has  been  "registered  or  
assigned"  to  the  suing  foreign  corporation,  a  suit  for  cancellation  of  the  registration  of  a  mark  or  tradename  
under  Section  17  has  no  such  requirement.  For  such  mark  or  tradename  should  not  have  been  registered  in  
the   first   place   (and   consequently   may   be   cancelled   if   so   registered)   if   it   "consists   of   or   comprises   a   mark   or  
tradename  which  so  resembles  a  mark  or  tradename  ...  previously  used  in  the  Philippines  by  another  and  not  
abandoned,  as  to  be  likely,  when  applied  to  or  used  in  connection  with  goods,  business  or  services  of  the  
applicant,  to  cause  confusion  or  mistake  or  to  deceive  purchasers;  ..."  
Petitioner's  last  argument  is  that  under  Section  37  of  the  Trademark  Law  respondent  is  not  entitled  to  the  
benefits   of   said   law   because   the   Philippines   is   not   a   signatory   to   any   international   treaty   or   convention  
relating  to  marks  or  tradenames  or  to  the  repression  of  unfair  competition.  
As  correctly  pointed  out  by  respondents,  this  provision  was  incorporated  in  the  law  in  anticipation  of  the  
eventual   adherence   of   the   Philippines   to   any   international   convention   or   treaty   for   the   protection   of  
industrial  property.  It  speaks  of  persons  who  are  nationals  of  domiciled  in,  or  have  a  bona  fide  or  effective  
business   or   commercial   establishment   in   any   foreign   country,   which   is   a   party   to   any   international  
convention  or  treaty  relating  to  industrial  property  to  which  the  Philippines  may  be  a  party.  In  other  words,  
the  provision  will  be  operative  only  when  the  Philippines  becomes  a  party  to  such  a  convention  or  treaty.  
.That   this   was   the   intention   of   Congress   is   clear   from   the   explanatory   note   to   House   Bill   No.   1157   (now  
Republic  Act  166)  
 
 
 

   
3H  A.Y.  2017-­‐2018   221  
 
 

CORPORATION  LAW  CASE  DIGESTS  –  ATTY.  DANTE  DELA  CRUZ  

176.  LA  CHEMISE  LACOSTE,  S.  A.  VS.  HON.  OSCAR  C.  FERNANDEZ    
G.R.  NO.  L-­‐‑63796-­‐‑97  MAY  2,  1984  
 
GOBINDRAM  HEMANDAS  SUJANANI  VS.  HON.  ROBERTO  V.  ONGPIN  
G.R.  NO.  L-­‐‑65659  MAY  2L,  1984  
GUTIERREZ,  JR.,  J.:  
 
DOCTRINE:    
The  registration  of  transfers  of  shares  of  stock  upon  the  books  of  the  corporation  is  required  as  a  condition  
precedent  to  their  validity  against  the  corporation  and  third  parties,  is  also  applicable  to  unissued  shares  
held  by  the  corporation  in  escrow  
 
FACTS:    
The   petitioner   is   a   foreign   corporation,   organized   and   existing   under   the   laws   of   France   and   not   doing  
business  in  the  Philippines.  It  is  the  actual  owner  of  the  abovementioned  trademarks  used  on  clothings  and  
other  goods  specifically  sporting  apparels  sold  in  many  parts  of  the  world  and  which  have  been  marketed  in  
the   Philippines   since   1964.   The   main   basis   of   the   private   respondent's   case   is   its   claim   of   alleged   prior  
registration.  In  1975,  Hemandas  &  Co.,  a  duly  licensed  domestic  firm  applied  for  and  was  issued  Reg.  No.  SR-­‐‑
2225  (SR  stands  for  Supplemental  Register)  for  the  trademark  "CHEMISE  LACOSTE  &  CROCODILE  DEVICE"  
by  the  Philippine  Patent  Office  for  use  on  T-­‐‑shirts,  sportswear  and  other  garment  products  of  the  company.  
Two  years  later,  it  applied  for  the  registration  of  the  same  trademark  under  the  Principal  Register.  The  Patent  
Office   eventually   issued   an   order   which   granted   the   application."Thereafter,   Hemandas   &   Co.   assigned   to  
respondent   Gobindram   Hemandas   all   rights,   title,   and   interest   in   the   trademark   "CHEMISE   LACOSTE   &  
DEVICE".  
 
The  petitioner  filed  its  application  for  registration  of  the  trademark  "Crocodile  Device"  and  "Lacoste".  The  
former   was   approved   for   publication   while   the   latter   was   opposed   by   Games   and   Garments.   The   petitioner  
filed  with  the  National  Bureau  of  Investigation  (NBI)  a  letter-­‐‑complaint  alleging  therein  the  acts  of  unfair  
competition   being   committed   by   Hemandas   and   requesting   their   assistance   in   his   apprehension   and  
prosecution.  
 
ISSUE:  
WON  the  petitioner  has  the  capacity  to  sue.  
 
HELD:    
YES.   The   petitioner   is   a   foreign   corporation   not   doing   business   in   the   Philippines.   The   marketing   of   its  
products  in  the  Philippines  is  done  through  an  exclusive  distributor,  Rustan  Commercial  Corporation.  The  
latter   is   an   independent   entity   which   buys   and   then   markets   not   only   products   of   the   petitioner   but   also  
many   other   products   bearing   equally   well-­‐‑known   and   established   trademarks   and   tradenames.   In   other  
words,  Rustan  is  not  a  mere  agent  or  conduit  of  the  petitioner.  
 
The  court  finds  and  concludes  that  the  petitioner  is  not  doing  business  in  the  Philippines.  Rustan  is  actually  
a  middleman  acting  and  transacting  business  in  its  own  name  and  or  its  own  account  and  not  in  the  name  or  
for  the  account  of  the  petitioner.  More  important  is  the  nature  of  the  case  which  led  to  this  petition.  What  
preceded  this  petition  for  certiorari  was  a  letter-­‐‑complaint  filed  before  the  NBI  charging  Hemandas  with  a  
criminal   offense,   i.e.,   violation   of   Article   189   of   the   Revised   Penal   Code.   If   prosecution   follows   after   the  
completion  of  the  preliminary  investigation  being  conducted  by  the  Special  Prosecutor  the  information  shall  
be  in  the  name  of  the  People  of  the  Philippines  and  no  longer  the  petitioner  which  is  only  an  aggrieved  party  
since  a  criminal  offense  is  essentially  an  act  against  the  State.  
 

   
3H  A.Y.  2017-­‐2018   222  
 
 

CORPORATION  LAW  CASE  DIGESTS  –  ATTY.  DANTE  DELA  CRUZ  

It  is  the  latter  which  is  principally  the  injured  party  although  there  is  a  private  right  violated.  Petitioner's  
capacity   to   sue   would   become,   therefore,   of   not   much   significance   in   the   main   case.   We   cannot   allow   a  
possible   violator   of   our   criminal   statutes   to   escape   prosecution   upon   a   far-­‐‑fetched   contention   that   the  
aggrieved   party   or   victim   of   a   crime   has   no   standing   to   sue.   In   upholding   the   right   of   the   petitioner   to  
maintain  the  present  suit  before  our  courts  for  unfair  competition  or  infringement  of  trademarks  of  a  foreign  
corporation,   we   are   moreover   recognizing   our   duties   and   the   rights   of   foreign   states   under   the   Paris  
Convention  for  the  Protection  of  Industrial  Property  to  which  the  Philippines  and  France  are  parties.  
 
 
 
177.  LITTON  MILLS,  INC.  VS.  COURT  OF  APPEALS  AND  GELHAAR  UNIFORM  COMPANY,  INC.,    
G.R.  NO.  94980.  MAY  15,  1996.  
MENDOZA,  J.  
 
Doctrine:   In   accordance   with   Rule   14,   Sec.   14,   service   upon   Gelhaar   could   be   made   in   three   ways:   (1)   by  
serving  upon  the  agent  designated  in  accordance  with  law  to  accept  service  of  summons;  (2)  if  there  is  no  
resident  agent,  by  service  on  the  government  official  designated  by  law  to  that  effect;  and  (3)  by  serving  on  
any  officer  or  agent  of  said  corporation  within  the  Philippines.  
 
Facts:   Petitioner   Litton   Mills,   Inc.   (Litton)   entered   into   an   agreement   with   Empire   Sales   Philippines  
Corporation   (Empire),   as   local   agent   of   private   respondent   Gelhaar   Uniform   Company   (Gelhaar),   a  
corporation  organized  under  the  laws  of  the  United  States,  whereby  Litton  agreed  to  supply  Gelhaar  7,770  
dozens  of  soccer  jerseys.  The  agreement  stipulated  that  before  it  could  collect  from  the  bank  on  the  letter  of  
credit,   Litton   must   present   an   inspection   certificate   issued   by   Gelhaar's   agent   in   the   Philippines,   Empire  
Sales,  that  the  goods  were  in  satisfactory  condition.  
Litton  sent  4  shipments  totalling  4,770  dozens  of  the  soccer  jerseys  between  December  2  and  December  30,  
1983.  A  5th    shipment,  consisting  of  2,110  dozens  of  the  jerseys,  was  inspected  by  Empire  from  January  9  to  
January  19,  1984,  but  Empire  refused  to  issue  the  required  certificate  of  inspection.  
 
Alleging  that  Empire's  refusal  to  issue  a  certificate  was  without  valid  reason,  Litton  filed  a  complaint  with  the  
RTC   Pasig,   for   specific   performance.   Litton   alleged   that   under   the   terms   of   the   letter   of   credit,   the   goods  
should   be   shipped   not   later   than   January   30,   1984;   that   the   vessel   stipulated   to   carry   the   shipment   was  
scheduled  to  receive  the  cargo  only  on  January  27,  1984;  and  that  the  letter  of  credit  itself  was  due  to  expire  
on  February  14,  1984.  Litton  sought  the  issuance  of  a  writ  of  preliminary  mandatory  injunction  to  compel  
Empire  to  issue  the  inspection  certificate  and  the  recovery  of  compensatory  and  exemplary  damages,  costs,  
attorney's  fees  and  other  just  and  equitable  relief.  
 
The   trial   court   issued   the   writ.   The   next   day,   Empire   issued   the   inspection   certificate,   so   that   the   cargo   was  
shipped  on  time.  
 
On  February  8,  1984,  Atty.  Remie  Noval  filed  in  behalf  of  the  defendants,  Gelhaar,  several  motions.  
 
On  January  1985,  the  law  firm  of  Sycip,  Salazar,  Feliciano  and  Hernandez  entered  a  special  appearance  for  
the   purpose   of   objecting   to   the   jurisdiction   of   the   court   over   Gelhaar.   On   February   4,   1985,   it   moved   to  
dismiss  the  case  and  to  quash  the  summons  on  the  ground  that  Gelhaar  was  a  foreign  corporation  not  doing  
business  in  the  Philippines,  and  as  such,  was  beyond  the  reach  of  the  local  courts.  
 
It  contended  that  Litton  failed  to  allege  and  prove  that  Gelhaar  was  doing  business  in  the  Philippines,  which  
they  argued  was  required  by  the  ruling  in  Pacific  Micronisian  Lines,  Inc.  v.  Del  Rosario,  before  summons  could  
be  served  under  Rule  14,  Sec.  14.  

   
3H  A.Y.  2017-­‐2018   223  
 
 

CORPORATION  LAW  CASE  DIGESTS  –  ATTY.  DANTE  DELA  CRUZ  

 
It  likewise  denied  the  authority  of  Atty.  Noval  to  appear  for  Gelhaar  and  contended  that  the  answer  filed  by  
Atty.  Noval  could  not  bind  Gelhaar  and  its  filing  did  not  amount  to  Gelhaar's  submission  to  the  jurisdiction  of  
the  court.  
 
Litton   opposed   the   motion.   On   the   other   hand,   Empire   moved   to   dismiss   on   the   ground   of   failure   of   the  
complaint   to   state   a   cause   of   action   since   the   complaint   alleged   that   Empire   only   acted   as   agent   of   Gelhaar;  
that   it   was   made   party-­‐‑defendant   only   for   the   purpose   of   securing   the   issuance   of   an   inspection   certificate;  
and  that  it  had  already  issued  such  certificate  and  the  shipment  had  already  been  shipped  on  time.  
 
For  his  part,  Atty.  Noval  claimed  that  he  had  been  authorized  by  Gelhaar  to  appear  for  it  in  the  case;  that  he  
had  in  fact  given  legal  advice  to  Empire  and  his  advice  had  been  transmitted  to  Gelhaar;  that  Gelhaar  had  
been  furnished  a  copy  of  the  answer;  that  Gelhaar  denied  his  authority  only  on  December  of  1984;  and  that  
the   belated   repudiation   of   his   authority   could   be   only   an   afterthought   because   of   problems   which   had  
developed  between  Gelhaar  and  Empire.  (Gelhaar  refused  to  pay  Empire  for  its  services  as  agent).  Atty.  Noval  
withdrew  his  appearance.  
 
On  September  24,  1986,  the  trial  court  issued  an  order  denying  for  lack  of  merit  Gelhaar's  motion  to  dismiss  
and  to  quash  the  summons.  It  held  that  Gelhaar  was  doing  business  in  the  Philippines,  and  that  the  service  of  
summons  on  Gelhaar  was  therefore  valid.  Gelhaar  filed  a  motion  for  reconsideration,  but  it  was  denied.  
 
Gelhaar  then  filed  a  special  civil  action  of  certiorari  with  the  CA,  which  on  August  20,  1990,  set  aside  the  
orders  of  the  RTC.  CA  held  that  proof  that  Gelhaar  was  doing  business  in  the  Philippines  should  have  been  
presented  because,  under  the  doctrine  of  Pacific  Micronisian,  this  is  a  condition  sine  qua  nonfor  the  service  
of  summons  under  Rule  14,  Sec.  14  of  the  Rules  of  Court,  and  that  it  was  error  for  the  trial  court  to  rely  on  
the  mere  allegations  of  the  complaint.  
 
The  appellate  court  held  that  neither  did  the  trial  court  acquire  jurisdiction  over  Gelhaar  through  voluntary  
submission  because  the  authority  of  Atty.  Noval  to  represent  Gelhaar  had  been  questioned.  Pursuant  to  Rule  
138,  Sec.  21,  the  trial  court  should  have  required  Atty.  Noval  to  prove  his  authority.  
 
Consequently,  the  appellate  court  ordered  the  trial  court  to  issue  anew  summons  to  be  served  on  Empire,  
after  the  allegation  in  the  complaint  that  Gelhaar  was  doing  business  in  the  Philippines  had  been  established.  
Hence  this  petition.  
 
Litton   contends   that   jurisdiction   over   Gelhaar   was   acquired   by   the   trial   court   by   the   service   of   summons  
through  Gelhaar's  agent  and,  at  any  rate,  by  the  voluntary  appearance  of  Atty.  Noval  as  counsel  of  Gelhaar.  
 
Issues:  W/N  the  Gelhaar  is  doing  business  in  the  Philippines  and  may  therefore  be  under  the  jurisdiction  of  
the  RTC?  
 
Held:  Yes.  We  sustain  petitioner's  contention  based  on  the  first  ground,  namely,  that  the  trial  court  acquired  
jurisdiction  over  Gelhaar  by  service  of  summons  upon  its  agent  pursuant  to  Rule  14,  Sec.  14.  
 
First.  The  appellate  court  invoked  the  ruling  in  Pacific  Micronisian,  in  which  it  was  stated  that  the  fact  of  
doing  business  must  first  be  established  before  summons  can  be  served  in  accordance  with  Rule  14,  Sec.  14.  
The  Court  of  Appeals  quoted  the  following  portion  of  the  opinion  in  that  case:  
Rule  14,  Section  14  provides  for  3  modes  of  effecting  service  upon  a  private  corporation.  But,  it  should  be  
noted,  in  order  that  service  may  be  effected  in  the  manner  above  stated,  said  section  also  requires  that  the  

   
3H  A.Y.  2017-­‐2018   224  
 
 

CORPORATION  LAW  CASE  DIGESTS  –  ATTY.  DANTE  DELA  CRUZ  

foreign  corporation  be  one  which  is  doing  business  in  the  Philippines.  This  is  a  sine  qua  non  requirement.  
This  fact  must  first  be  established  in  order  that  summons  can  be  made  and  jurisdiction  acquired.  
 
In   the   later   case   of   Signetics   Corporation   v.   Court   of   Appeals,   however,   we   clarified   the   holding   in   Pacific  
Micronisian,  thus:  
 
The   petitioner   opines   that   the   phrase,   "(the)   fact   (of   doing   business   in   the   Philippines)   must   first   be  
established  in  order  that  summons  be  made  and  jurisdiction  acquired,"  used  in  the  above  pronouncement,  
would   indicate   that   a   mere   allegation   to   that   effect   in   the   complaint   is   not   enough   —   there   must   instead   be  
proof  of  doing  business.  In  any  case,  the  petitioner  points  out,  the  allegations  themselves  did  not  sufficiently  
show  the  fact  of  its  doing  business  in  the  Philippines.  
 
It  should  be  recalled  that  jurisdiction  and  venue  of  actions  are,  as  they  should  so  be,  initially  determined  by  
the  allegations  of  the  complaint.  Jurisdiction  cannot  be  made  to  depend  on  independent  pleas  set  up  in  a  
mere   motion   to   dismiss,   otherwise   jurisdiction   would   become   dependent   almost   entirely   upon   the  
defendant.  The  fact  of  doing  business  must  then,  in  the  first  place,  be  established  by  appropriate  allegations  
in  the  complaint.  This  is  what  the  Court  should  be  seen  to  have  meant  in  the  Pacific  Micronisian  case.  The  
complaint,   it   is   true,   may   have   been   vaguely   structured   but,   taken   correlatively,   not   disjunctively   as   the  
petitioner  would  rather  suggest,  it  is  not  really  so  weak  as  to  be  fatally  deficient  in  the  above  requirement.  .  .  
.  
 
Hence,  a  court  need  not  go  beyond  the  allegations  in  the  complaint  to  determine  whether  or  not  a  defendant  
foreign  corporation  is  doing  business  for  the  purpose  of  Rule  14,  Sec.  14.  In  the  case  at  bar,  the  allegation  that  
Empire,  for  and  in  behalf  of  Gelhaar,  ordered  7,770  dozens  of  soccer  jerseys  from  Litton  and  for  this  purpose  
Gelhaar  caused  the  opening  of  an  irrevocable  letter  of  credit  in  favor  of  Litton  is  a  sufficient  allegation  that  
Gelhaar  was  doing  business  in  the  Philippines.  
 
Second.   Gelhaar   contends   that   the   contract   with   Litton   was   a   single,   isolated   transaction   and   that   it   did   not  
constitute   "doing   business."   Reference   is   made   to   Pacific   Micronisian   in   which   the   only   act   done   by   the  
foreign   company   was   to   employ   a   Filipino   as   a   member   of   the   crew   on   one   of   its   ships.   This   court   held   that  
the   act   was   an   isolated,   incidental   or   casual   transaction,   not   sufficient   to   indicate   a   purpose   to   engage   in  
business.  
 
It  is  not  really  the  fact  that  there  is  only  a  single  act  done  that  is  material.  The  other  circumstances  of  the  case  
must  be  considered.  Where  a  single  act  or  transaction  of  a  foreign  corporation  is  not  merely  incidental  or  
casual  but  is  of  such  character  as  distinctly  to  indicate  a  purpose  on  the  part  of  the  foreign  corporation  to  do  
other  business  in  the  state,  such  act  will  be  considered  as  constituting  doing  business.  This  Court  referred  to  
acts  which  were  in  the  ordinary  course  of  business  of  the  foreign  corporation.  
 
In   the   case   at   bar,   the   trial   court   was   certainly   correct   in   holding   that   Gelhaar's   act   in   purchasing   soccer  
jerseys   to   be   within   the   ordinary   course   of   business   of   the   company   considering   that   it   was   engaged   in   the  
manufacture  of  uniforms.  The  acts  noted  above  are  of  such  a  character  as  to  indicate  a  purpose  to  do  business.  
 
In  accordance  with  Rule  14,  Sec.  14,  service  upon  Gelhaar  could  be  made  in  three  ways:  (1)  by  serving  upon  
the  agent  designated  in  accordance  with  law  to  accept  service  of  summons;  (2)  if  there  is  no  resident  agent,  
by  service  on  the  government  official  designated  by  law  to  that  effect;  and  (3)  by  serving  on  any  officer  or  
agent  of  said  corporation  within  the  Philippines.  Here,  service  was  made  through  Gelhaar's  agent,  the  Empire  
Sales   Philippines   Corp.   There   was,   therefore,   a   valid   service   of   summons   on   Gelhaar,   sufficient   to   confer   on  
the  trial  court  jurisdiction  over  the  person  of  Gelhaar.  
 

   
3H  A.Y.  2017-­‐2018   225  
 
 

CORPORATION  LAW  CASE  DIGESTS  –  ATTY.  DANTE  DELA  CRUZ  

 
 
178.  THE  MENTHOLATUM  CO.,  INC.,  ET  AL.VS.  ANACLETO  MANGALIMAN,  ET  AL.,    
G.R.  NO.  47701.  JUNE  27,  1941    
LAUREL,  J  
 
DOCTRINE:  A  foreign  corporation  doing  business  in  the  Philippines  without  the  license  required  by  section  
68  of  the  Corporation  Law,  may  not  prosecute  an  action  for  violation  of  trade  mark  and  unfair  competition.  
Neither  its  agent  in  the  Philippines  may  maintain  the  action  for  the  reason  that  the  distinguishing  features  of  
the  agent  being  his  representative  character  and  derivative  authority,  it  cannot  now,  to  the  advantage  of  its  
principal,  claim  an  independent  standing  in  court.    
 
FACTS   The   Mentholatum   Co.,   Inc.,   is   a   foreign   corporation   which   manufactures   "Mentholatum,"   a  
medicament  and  salve  adapted  for  the  treatment  of  colds,  nasal  irritations,  chapped  skin,  insect  bites,  rectal  
irritation  and  other  external  ailments  of  the  body.  The  Philippine-­‐‑American  Drug  Co.,  Inc.,  is  its  exclusive  
distributing  agent  in  the  Philippines  authorized  by  it  to  look  after  and  protect  its  interests.  The  Mentholatum  
Co.,  Inc.,  registered  with  the  Bureau  of  Commerce  and  Industry  the  word,  "Mentholatum",  as  trade  mark  for  
its  products.  The  Mangaliman  brothers  prepared  a  medicament  and  salve  named  "Mentholiman"  which  they  
sold  to  the  public  packed  in  a  container  of  the  same  size,  color  and  shape  as  "Mentholatum."  As  a  consequence  
of  these  acts  of  the  Mangalimans,  Mentholatum,  etc.  suffered  damages  from  the  diminution  of  their  sales  and  
the   loss   of   goodwill   and   reputation   of   their   product   in   the   market.   The   Mentholatum   Co.,   Inc.,   and   the  
Philippine-­‐‑American  Drug,  Co.,  Inc.  instituted  an  action  in  the  CFI  of  Manila  for  infringement  of  trade  mark  
and  unfair  competition  against  Anacleto  Mangaliman,  Florencio  Mangaliman  and  the  Director  of  the  Bureau  
of  Commerce.    
Mentholatum,  etc.  prayed  for  the  issuance  of  an  order  restraining  Anacleto  and  Florencio  Mangaliman  from  
selling  their  product  "Mentholiman,"  and  directing  them  to  render  an  accounting  of  their  sales  and  profits  
and  to  pay  damages.    The  CFI  rendered  judgment  in  favor  of  Mentholatum,  etc.  The  Court  of  Appeals  reversed  
the   CFI’s   decision   holding   that   the   activities   of   the   Mentholatum   Co.,   Inc.,   were   business   transactions   in   the  
Philippines.  CA  further  ruled  that  by  virtue  of  Sec.  69  of  the  Corporation  Law,  Mentholatum  may  not  maintain  
the  suit.    Mentholatum,  etc.  filed  the  petition  for  certiorari.    
   
ISSUE:  Whether  or  not  Mentholatum  may  maintain  the  instant  action  withoung  having  secured  the  license  
required  in  Sec.  69  of  the  Corporation  Law.    
 
HELD:  NO.  In  the  present  case,  no  dispute  exists  as  to  facts:  (1)  that  the  plaintiff,  the  Mentholatum  Co.,  Inc.,  is  
a  foreign  corporation:  and  (2)  that  it  is  not  licensed  to  do  business  in  the  Philippines.  The  controversy,  in  
reality,   hinges   on   the   question   of   whether   the   said   corporation   is   or   is   not   transacting   business   in   the  
Philippines.    
No  general  rule  or  governing  principle  can  be  laid  down  as  to  what  constitutes  "doing"  or  "engaging  in"  or  
"transacting"   business.   Indeed,   each   case   must   be   judged   in   the   light   of   its   peculiar   environmental  
circumstances.   The   true   test,   however,   seems   to   be   whether   the   foreign   corporation   is   continuing   the   body  
or  substance  of  the  business  or  enterprise  for  which  it  was  organized  or  whether  it  has  substantially  retired  
from  it  and  turned  it  over  to  another.    
Herein,  Mentholatum  Co.,  through  its  agent,  the  Philippine-­‐‑American  Drug  Co.,  Inc.,  has  been  doing  business  
in   the   Philippines   by   selling   its   products   here   since   the   year   1929,   at   least.   Whatever   transactions   the  
Philippine-­‐‑American  Drug  Co.,  Inc.,  had  executed  in  view  of  the  law,  the  Mentholatum  Co.,  Inc.,  being  a  foreign  
corporation  doing  business  in  the  Philippines  without  the  license  required  by  section  68  of  the  Corporation  
Law,  it  may  not  prosecute  this  action  for  violation  of  trade  mark  and  unfair  competition.  Neither  may  the  
Philippine-­‐‑American  Drug  Co.,  Inc.,  maintain  the  action  here  for  the  reason  that  the  distinguishing  features  
of   the   agent   being   his   representative   character   and   derivative   authority,   it   cannot   now,   to   the   advantage   of  

   
3H  A.Y.  2017-­‐2018   226  
 
 

CORPORATION  LAW  CASE  DIGESTS  –  ATTY.  DANTE  DELA  CRUZ  

its  principal,  claim  an  independent  standing  in  court.  Further,  the  recognition  of  the  legal  status  of  a  foreign  
corporation  is  a  matter  affecting  the  policy  of  the  forum,  and  the  distinction  drawn  in  Philippine  Corporation  
Law  is  an  expression  of  the  policy.  The  general  statement  made  in  Western  Equipment  and  Supply  Co.  vs.  
Reyes  regarding  the  character  of  the  right  involved  should  not  be  construed  in  the  derogation  of  the  policy-­‐‑
determining   authority   of   the   State.   The   right   of   Mentholatum   conditioned   upon   compliance   with   the  
requirement  of  section  69  of  the  Corporation  Law  to  protect  its  rights,  is  reserved.  
 
DISSENTING  OPINION:  Suits  regarding  trademark  may  be  instituted  and  maintained  by  a  foreign  corporation  
doing  business  in  the  Philippines  even  without  the  required  license  under  Sec.  69  og  the  Corporation  Law.    
Moran,  J.    
Section  69  of  the  Corporation  Law  provides  that,  without  license  no  foreign  corporation  may  maintain  by  
itself  or  assignee  any  suit  in  the  Philippine  courts  for  the  recovery  of  any  debt,  claim  or  demand  whatever.  
But  this  provision,  as  we  have  held  in  Western  Equipment  &  Supply  Company  vs.  Reyes  (51  Phil.,  115),  does  
not  apply  to  suits  for  infringement  of  trade  marks  and  unfair  competition,  the  theory  being  that  "the  right  to  
the  use  of  the  corporate  and  trade  name  of  a  foreign  corporation  is  a  property  right,  a  right  in  rem,  which  it  
may   assert   and   protect   in   any   of   the   courts   of   the   world   even   in   countries   where   it   does   not   personally  
transact  any  business,"  and  that  "trade  mark  does  not  acknowledge  any  territorial  boundaries  but  extends  
to  every  mark  where  the  traders'  goods  have  become  known  and  identified  by  the  use  of  the  mar    
 
 
 
179.   AGILENT   TECHNOLOGIES   SINGAPORE   (PTE)   LTD   VS.   INTEGRATED   SILICON   TECHNOLOGY  
PHILIPPINES  CORPORATION  
G.R.  NO.  154618.  APRIL  14,  2004  
YNARES-­‐‑SANTIAGO,  J  
 
DOCTRINE:    
There   is   no   definitive   rule   on   what   constitutes   “doing”,   “engaging   in”,   or   “transacting”   business   in   the  
Philippines.  Jurisprudence  has  it,  however,  that  the  term  “implies  a  continuity  of  commercial  dealings  and  
arrangements,  and  contemplates,  to  that  extent,  the  performance  of  acts  or  works  or  the  exercise  of  some  of  
the   functions   normally   incident   to   or   in   progressive   prosecution   of   the   purpose   and   subject   of   its  
organization.”  By  and  large,  to  constitute  “doing  business”,  the  activity  to  be  undertaken  in  the  Philippines  is  
one  that  is  for  profit-­‐‑making.    
 
FACTS:    
Petitioner  Agilent  Technologies  Singapore  (Pte.),  Ltd.  ("Agilent")  is  a  foreign  corporation,  which,  by  its  own  
admission,   is   not   licensed   to   do   business   in   the   Philippines.   Respondent   Integrated   Silicon   Technology  
Philippines  Corporation  (“Integrated  Silicon”)  is  a  private  domestic  corporation,  100%  foreign  owned,  which  
is   engaged   in   the   business   of   manufacturing   and   assembling  electronics   components.   The  juridical  relation  
among  the  various  parties  in  this  case  can  be  traced  to  a  5-­‐‑year  Value  Added  Assembly  Services  Agreement  
(“VAASA”),   entered   between   Integrated   Silicon   and   the   Hewlett-­‐‑Packard   Singapore.   Under   the   terms   of   the  
VAASA,  Integrated  Silicon  was  to  locally  manufacture  and  assemble  fiber  optics  for  export  to  HP-­‐‑Singapore.  
HP-­‐‑Singapore,  for  its  part,  was  to  consign  raw  materials  to  Integrated  Silicon;  transport  machinery  to  the  
plant  of  Integrated  Silicon;  and  pay  Integrated  Silicon  the  purchase  price  of  the  finished  products.  The  VAASA  
had  a  five-­‐‑year  term,  with  a  provision  for  annual  renewal  by  mutual  written  consent.  With  the  consent  of  
Integrated   Silicon,   HP-­‐‑Singapore   assigned   all   its   rights   and   obligations   in   the   VAASA   to   Agilent.   On   May   25,  
2001,  Integrated  Silicon  filed  a  complaint  for  “Specific  Performance  and  Damages”  against  Agilent  and  its  
officers.  It  alleged  that  Agilent  breached  the  parties’  oral  agreement  to  extend  the  VAASA.  Integrated  Silicon  
thus  prayed  that  defendant  be  ordered  to  execute  a  written  extension  of  the  VAASA  for  a  period  of  five  years  
as  earlier  assured  and  promised;  to  comply  with  the  extended  VAASA.    

   
3H  A.Y.  2017-­‐2018   227  
 
 

CORPORATION  LAW  CASE  DIGESTS  –  ATTY.  DANTE  DELA  CRUZ  

 
Agilent   filed   a   separate   complaint   against   Integrated   Silicon   for   “Specific   Performance,   Recovery   of  
Possession,   and   Sum   of   Money   with   Replevin,   Preliminary   Mandatory   Injunction,   and   Damages”,   before   the  
Regional   Trial   Court.   Agilent   prayed   that   a   writ   of   replevin   or,   in   the   alternative,   a   writ   of   preliminary  
mandatory  injunction,  be  issued  ordering  Integrated  Silicon  to  immediately  return  and  deliver  to  Agilent  its  
equipment,  machineries  and  the  materials  to  be  used  for  fiber-­‐‑optic  components  which  were  left  in  the  plant  
of  Integrated  Silicon.  Respondents  filed  a  Motion  to  Dismiss,  on  the  ground  of  lack  of  Agilent’s  legal  capacity  
to   sue,   among   others.   The   trial   court   denied   the   Motion   to   Dismiss   and   granted   petitioner   Agilent’s  
application  for  a  writ  of  replevin.  Without  filing  a  motion  for  reconsideration,  respondents  filed  a  petition  for  
certiorari  with  the  Court  of  Appeals.  The  Court  of  Appeals  granted  respondents’  petition  for  certiorari,  set  
aside   the   assailed   Order   of   the   trial   court,   and   ordered   the   dismissal   of   the   case.   Hence,   the   instant   petition  
where   respondents   argue   that   since   Agilent   is   an   unlicensed   foreign   corporation   doing   business   in   the  
Philippines,  it  lacks  the  legal  capacity  to  file  suit.  The  assailed  acts  of  petitioner  Agilent,  purportedly  in  the  
nature  of  “doing  business”  in  the  Philippines,  are  the  following:  (1)  mere  entering  into  the  VAASA,  which  is  a  
“service   contract”;   (2)   appointment   of   a   full-­‐‑   time   representative   in   Integrated   Silicon,   to   “oversee   and  
supervise   the   production”   of   Agilent’s   products;     (3)   the   appointment   by   Agilent   of   six   full-­‐‑time   staff  
members,  who  were  permanently  stationed  at  Integrated  Silicon’s  facilities  in  order   to   inspect   the   finished  
goods   for   Agilent;   and   (4)   Agilent’s   participation   in   the   management,   supervision   and   control   of   Integrated  
Silicon,   including   instructing   Integrated   Silicon   to   hire   more   employees   to   meet   Agilent’s   increasing  
production   needs,   regularly   performing   quality   audit,   evaluation   and   supervision   of   Integrated   Silicon’s  
employees,  regularly  performing  inventory  audit  of  raw  materials  to  be  used  by  Integrated  Silicon,  which  
was  also  required  to  provide  weekly  inventory  updates  to  Agilent,    and  providing  and  dictating  Integrated  
Silicon  on  the  daily  production  schedule,  volume  and  models  of  the  products  to  manufacture  and  ship  for  
Agilent.    
 
ISSUE:  
Whether  Agilent  lacks  the  legal  capacity  to  file  suit,  being  an  unlicensed  foreign  corporation  doing  business  
in  the  Philippines  
 
HELD:  
NO.  The  principles  regarding  the  right  of  a  foreign  corporation  to  bring  suit  in  Philippine  courts  may  thus  be  
condensed  in  four  statements:  (1)  if  a  foreign  corporation  does  business  in  the  Philippines  without  a  license,  
it  cannot  sue  before  the  Philippine  courts;  (2)  if  a  foreign  corporation  is  not  doing  business  in  the  Philippines,  
it  needs  no  license  to  sue  before  Philippine  courts  on  an  isolated  transaction  or  on  a  cause  of  action  entirely  
independent   of   any   business   transaction   ;   (3)   if   a   foreign   corporation   does   business   in   the   Philippines  
without  a  license,  a  Philippine  citizen  or  entity  which  has  contracted  with  said  corporation  may  be  estopped  
from  challenging  the  foreign  corporation’s  corporate  personality  in  a  suit  brought  before  Philippine  courts;  
and  (4)  if  a  foreign  corporation  does  business  in  the  Philippines  with  the  required  license,  it  can  sue  before  
Philippine  courts  on  any  transaction.    
 
The   challenge   to   Agilent’s   legal   capacity   to   file   suit   hinges   on   whether   or   not   it   is   doing   business   in   the  
Philippines.  However,  there  is  no  definitive  rule  on  what  constitutes  “doing”,  “engaging  in”,  or  “transacting”  
business  in  the  Philippines.  Jurisprudence  has  it,  however,  that  the  term  “implies  a  continuity  of  commercial  
dealings   and   arrangements,   and   contemplates,   to   that   extent,   the   performance   of   acts   or   works   or   the  
exercise   of   some   of   the   functions   normally   incident   to   or   in   progressive   prosecution   of   the   purpose   and  
subject  of  its  organization.”.  In  the  case  at  bar,  the  acts  enumerated  in  the  VAASA  do  not  constitute  “doing  
business”  in  the  Philippines.  By  and  large,  to  constitute  “doing  business”,  the  activity  to  be  undertaken  in  the  
Philippines   is   one   that   is   for   profit-­‐‑making.   By   the   clear   terms   of   the   VAASA,   Agilent’s   activities   in   the  
Philippines  were  confined  to  (1)  maintaining  a  stock  of  goods  in  the  Philippines  solely  for  the  purpose  of  
having  the  same  processed  by  Integrated  Silicon;  and  (2)  consignment  of  equipment  with  Integrated  Silicon  

   
3H  A.Y.  2017-­‐2018   228  
 
 

CORPORATION  LAW  CASE  DIGESTS  –  ATTY.  DANTE  DELA  CRUZ  

to   be   used   in   the   processing   of   products   for   export.   As   such,   we   hold   that,   based   on   the   evidence   presented  
thus  far,  Agilent  cannot  be  deemed  to  be  “doing  business”  in  the  Philippines.  Respondents’  contention  that  
Agilent  lacks  the  legal  capacity  to  file  suit  is  therefore  devoid  of  merit.  As  a  foreign  corporation  not  doing  
business  in  the  Philippines,  it  needed  no  license  before  it  can  sue  before  our  courts.    
 
 
 
180.    MERRILL  LYNCH  FUTURES,  INC.  v.  CA  
G.R.  No.  97816   July  24,  1992  
NARVASA,  C.  J.;  
 
FACTS:  
Merrill  Lynch  Futures,  Inc.  filed  a  complaint  with  the  Regional  Trial  Court  at  Quezon  City  against  the  Spouses  
Pedro  M.  Lara  and  Elisa  G.  Lara  for  the  recovery  of  a  debt  and  interest  thereon,  damages,  and  attorney's  fees.  
Due  to  the  loss  amounting  to  US$160,749.69  incurred  in  respect  of  3  transactions  involving  "index  futures,"  
and  after  setting  this  off  against  an  amount  of  US$75,913.42  then  owing  by  ML  FUTURES  to  the  Lara  Spouses,  
said  spouses  became  indebted  to  ML  FUTURES  for  the  ensuing  balance  of  US$84,836.27,  which  the  latter  
asked  them  to  pay;  
Lara  Spouses  however  refused  to  pay  this  balance,  alleging  that  the  transactions  were  null  and  void  because  
Merrill   Lynch   Philippines,   Inc.,   the   Philippine   company   servicing   accounts   of   plaintiff,   had   no   license   to  
operate  as  a  commodity  and/or  financial  futures  broker.  
 
ISSUE:  Whether  a  foreign  corporation  has  a  capacity  to  maintain  an  action  against  a  Philippine  corporation.  
 
HELD:  
The  rule  is  that  a  party  is  estopped  to  challenge  the  personality  of  a  corporation  after  having  acknowledged  
the  same  by  entering  into  a  contract  with  it.  
And   the   "doctrine   of   estoppel   to   deny   corporate   existence   applies   to   foreign   as   well   as   to   domestic  
corporations;"  "one  who  has  dealt  with  a  corporation  of  foreign  origin  as  a  corporate  entity  is  estopped  to  
deny  its  corporate  existence  and  capacity."  
The  principle  "will  be  applied  to  prevent  a  person  contracting  with  a  foreign  corporation  from  later  taking  
advantage   of   its   noncompliance   with   the   statutes,   chiefly   in   cases   where   such   person   has   received   the  
benefits   of   the   contract   where   such   person   has   acted   as   agent   for   the   corporation   and   has   violated   his  
fiduciary   obligations   as   such,   and   where   the   statute   does   not   provide   that   the   contract   shall   be   void,   but  
merely  fixes  a  special  penalty  for  violation  of  the  statute.  
The   general   rule   that   in   the   absence   of   fraud   of   person   who   has   contracted   or   otherwise   dealt   with   an  
association  in  such  a  way  as  to  recognize  and  in  effect  admit  its  legal  existence  as  a  corporate  body  is  thereby  
estopped  to  deny  its  corporate  existence  in  any  action  leading  out  of  or  involving  such  contract  or  dealing,  
unless  its  existence  is  attacked  for  causes  which  have  arisen  since  making  the  contract  or  other  dealing  relied  
on  as  an  estoppel  and  this  applies  to  foreign  as  well  as  domestic  corporations.  
 
 
 
181.  TOP-­‐‑WELD  MANUFACTURING,  INC.  VS.  ECED,  S.A.,  IRTI,  S.A.,  EUTECTIC  CORPORATION,  VICTOR  
C.  GAERLAN,  AND  THE  HON.  COURT  OF  APPEALS,    
G.R.  NO.  L-­‐‑44944  AUGUST  9,  1985  
GUTTIERREZ  JR.,  J.;  
 

   
3H  A.Y.  2017-­‐2018   229  
 
 

CORPORATION  LAW  CASE  DIGESTS  –  ATTY.  DANTE  DELA  CRUZ  

DOCTRINE:  The  term  ‘doing  business’  implies  a  continuity  of  commercial  dealings  and  arrangements,  and  
contemplates,   to   that   extent,   the   performance   of   acts   or   works   or   the   exercise   of   some   of   the   functions  
normally  incident  to,  and  in  progressive  prosecution  of,  the  purpose  and  object  of  its  organization.  
 
FACTS:  
Petitioner  Top-­‐‑weld  Manufacturing,  Inc.  (Top-­‐‑weld)  is  a  Philippine  corporation  engaged  in  the  business  of  
manufacturing  and  selling  welding  supplies  and  equipment.  the  petitioner  entered  into  separate  contracts  
with  two  different  foreign  entities-­‐‑   IRTI  and  ECED  for  license  and  technical  assistance  and  distributorship  
agreement,  respectively.  Upon  learning  that  the  two  foreign  entities  were  negotiating  with  another  group  to  
replace   the   petitioner,   the   latter   instituted   Civil   Case   against   respondents.   the   lower   court   issued   a  
restraining   order   against   the   corporation.   IRTI   and   ECED   wrote   Top-­‐‑weld   separate   notices   about   the  
termination   of   their   respective   contracts.   Top-­‐‑weld   filed   an   amended   complaint   together   with   a  
supplemental  complaint  which  embodied  a  new  application  for  a  preliminary  mandatory  injunction.  Among  
others,  the  petitioner  invoked  the  provisions  of  No.  9.  Section  4  of  Republic  Act  5455  on  alien  firms  doing  
business  in  the  Philippines.  The  corporations  filed  their  answers  setting  up  as  affirmative  defenses  violations  
of   the   contracts   allegedly   committed   by   the   petitioner.   The   respondent   corporation   further   alleged   that  
Section  4  (9)  of  R.A.  No.  5455  cannot  possibly  apply  to  the  instant  case  because  of  said  violations,  they  are  
justified  in  terminating  plaintiff  without  obligation  to  reimburse  the  plaintiff  and  in  fact,  the  defendants  have  
sent   written   notices   of   the   termination   and   Since   no   written   certificate   was   applied   for   nor   obtained   by  
defendant  entities  from  the  Board  of  Investments,  the  latter  cannot  legally  require  of  them  compliance  with  
No.   9,   Section   4,   R.A.   No,   5455.   the   trial   court   issued   an   order   granting   the   petitioner's   application   for  
preliminary  injunction.  The  case  was  elevated  to  the  Court  of  Appeals  which  ruled  in  favour  of  respondents.  
Hence  this  petition.  
 
ISSUE:  Whether  or  not  respondent  corporations  can  be  considered  as  "doing  business"  in  the  Philippines  
and,  therefore,  subject  to  the  provisions  of  R.A.  No.  5455.    
 
HELD:  
There  is  no  dispute  that  respondents  are  foreign  corporations  not  licensed  to  do  business  in  the  Philippines.  
There   is   no   general   rule   or   governing   principle   laid   down   as   to   what   constitutes   "doing"   or   engaging   in"   or  
"transacting"  business  in  the  Philippines.  Each  case  must  be  judged  in  the  light  of  its  peculiar  circumstances.  
The  term  implies  a  continuity  of  commercial  dealings  and  arrangements,  and  contemplates,  to  that  extent,  
the   performance   of   acts   or   works   or   the   exercise   of   some   of   the   functions   normally   incident   to,   and   in  
progressive  prosecution  of,  the  purpose  and  object  of  its  organization.  Judged  by  the  foregoing  standards,  
the   Court   agrees   with   the   Court   of   Appeals   in   considering   the   respondents   as   "doing   business"   in   the  
Philippines.   When   the   respondents   entered   into   the   disputed   contracts   with   the   petitioner,   they   were  
carrying  out  the  purposes  for  which  they  were  created,  i.e.  to  manufacture  and  market  welding  products  and  
equipment.  The  terms  and  conditions  of  the  contracts  as  well  as  the  respondents'  conduct  indicate  that  they  
established  within  our  country  a  continuous  business,  and  not  merely  one  of  a  temporary  character.  This  
necessarily  brings  them  under  the  provisions  of  R.A.  No.  5455.  
 
On  the  basis  of  the  foregoing,  "IRTI  AND  ECED  for  doing  business  and  engaging  in  economic  activity  in  the  
Philippines,  as  a  prerequisite  to  which  they  should  have  first  secured  a  written  certificate  from  the  Board  of  
Investments."   The   respondent   court,   however,   erred   in   holding   that   "IRTI   and   ECED   have   not   secured   such  
written  certificate  in  consequence  of  which  there  is  no  occasion  for  the  Board  of  Investments  to  impose  the  
requirements  prescribed  in  the  aforequoted  provisions  of  Sec.  4,  R.A.  No.  5455  ...  ."  To  accept  this  view  would  
open  the  way  for  an  interpretation  that  by  doing  business  in  the  country  without  first  securing  the  required  
written   certificate   from   the   Board   of   Investments,   a   foreign   corporation   may   violate   or   disregard   the  
safeguards  which  the  law,  by  its  provisions,  seeks  to  establish.  
 

   
3H  A.Y.  2017-­‐2018   230  
 
 

CORPORATION  LAW  CASE  DIGESTS  –  ATTY.  DANTE  DELA  CRUZ  

However,  TOP-­‐‑WELD  cannot  invoke  the  same  against  the  former.  As  between  the  parties  themselves,  R.A.  
No.  5455  does  not  declare  as  void  or  invalid  the  contracts  entered  into  without  first  securing  a  license  or  
certificate   to   do   business   in   the   Philippines.   Neither   does   it   appear   to   intend   to   prevent   the   courts   from  
enforcing  contracts  made  in  contravention  of  its  licensing  provisions.  There  is  no  denying,  though,  that  an  
"illegal   situation,"   as   the   appellate   court   has   put   it,   was   created   when   the   parties   voluntarily   contracted  
without  such  license.  a  person  is  presumed  to  be  more  knowledgeable  about  his  own  state  law  than  his  alien  
or  foreign  contemporary.  The  very  purpose  of  the  law  was  circumvented  and  evaded  when  the  petitioner  
entered  into  said  agreements  despite  the  prohibition  of  R.A.  No.  5455.  The  parties  in  this  case  being  equally  
guilty   of   violating   R.A,   No.   5455,   they   are   in   pari   delicto,   in   which   case   it   follows   as   a   consequence   that  
petitioner  is  not  entitled  to  the  relief  prayed  for  in  this  case.  
 
 
 
182.   ANTAM   CONSOLIDATED,   INC.,   TAMBUNTING   TRADING   CORPORATION   AND   AURORA  
CONSOLIDATED   SECURITIES   AND   INVESTMENT   CORPORATION   VS.   THE   COURT   OF   APPEALS,   THE  
HONORABLE   MAXIMIANO   C.   ASUNCION   (COURT   OF   FIRST   INSTANCE   OF   LAGUNA,   BRANCH   II   [STA.  
CRUZ])  AND  STOKELY  VAN  CAMP,  INC.  
G.R.  NO.  L-­‐‑61523   JULY  31,  1986  
J.  GUTIERREZ,  JR  
 
DOCTRINE:.  Where  a  single  act  or  transaction,  however,  is  not  merely  incidental  or  casual  but  indicates  the  
foreign   corporation's   intention   to   do   other   business   in   the   Philippines,   said   single   act   or   transaction  
constitutes  'doing'  or  'engaging  in'  or  'transacting'  business  in  the  Philippines.    
 
FACTS:     On   April   9,   1981,   respondent   Stokely   Van   Camp.   Inc.   filed   a   complaint   against   Banahaw   Milling  
Corporation,   Antam   Consolidated,   Inc.,   Tambunting   Trading   Corporation,   Aurora   Consolidated   Securities  
and   Investment   Corporation,   and   United   Coconut   Oil   Mills,   Inc.   for   collection   of   sum   of   money.     In   its  
complaint,  Stokely  alleged  that  Stokely  and  Capital  City  were  not  engaged  in  business  in  the  Philippines  prior  
to   the   commencement   of   the   suit   so   that   Stokely   is   not   licensed   to   do   business   in   this   country   and   is   not  
required  to  secure  such  license;  that  on  August  21,  1978,  Capital  City  and  Coconut  Oil  Manufacturing  (Phil.)  
Inc.   (Comphil)   with   the   latter   acting   through   its   broker   Roths   child   Brokerage   Company,   entered   into   a  
contract   (No.   RBS   3655)   wherein   Comphil   undertook   to   sell   and   deliver   and   Capital   City   agreed   to   buy   500  
long  tons  of  crude  coconut  oil  to  be  delivered  in  October/November  1978  at  the  c.i.f.  price  of  US$0.30/1b.  
but  Comphil  failed  to  deliver  the  coconut  oil  so  that  Capital  City  covered  its  coconut  oil  needs  in  the  open  
market  at  a  price  substantially  in  excess  of  the  contract  and  sustained  a  loss  of  US$103,600;  that  to  settle  
Capital  City's  loss  under  the  contract,  the  parties  entered  into  a  second  contract  (No.  RBS  3738)  on  November  
3,  1978  wherein  Comphil  undertook  to  buy  and  Capital  City  agreed  to  sell  500  long  tons  of  coconut  crude  oil  
under  the  same  terms  and  conditions  but  at  an  increased  c.i.f.  price  of  US$0.3925/lb.;  that  the  second  contract  
states  that  "it  is  a  wash  out  against  RBS  3655"  so  that  Comphil  was  supposed  to  repurchase  the  undelivered  
coconut  oil  at  US$0.3925  from  Capital  City  by  paying  the  latter  the  sum  of  US$103,600.00  which  is  the  same  
amount   of   loss   that   Capital   City   sustained   under   the   first   contract;   that   Comphil   again   failed   to   pay   said  
amount,   so   to   settle   Capital   City's   loss,   it   entered   into   a   third   contract   with  Comphil   on   January   24,   1979  
wherein  the  latter  undertook  to  sell  and  deliver  and  Capital  City  agreed  to  buy  the  same  quantity  of  crude  
coconut  oil  to  be  delivered  in  April/May  1979  at  the  c.i.f.  price  of  US$0.3425/lb.;  that  the  latter  price  was  9.25  
cents/lb.   or   US$103,600   for   500   long   tons   below   the   then   current   market   price   of   43.2   cents/lb.   and   by  
delivering  said  quantity  of  coconut  oil  to  Capital  City  at  the  discounted  price,  Comphil  was  to  have  settled  its  
US$103,600  liability  to  Capital  City;  that  Comphil  failed  to  deliver  the  coconut  oil  so  Capital  City  notified  the  
former  that  it  was  in  default;  that  Capital  City  sustained  damages  in  the  amount  of  US$175,000;  and  (8)  that  
after  repeated  demands  from  Comphil  to  pay  the  said  amount,  the  latter  still  refuses  to  pay  the  same.  
 

   
3H  A.Y.  2017-­‐2018   231  
 
 

CORPORATION  LAW  CASE  DIGESTS  –  ATTY.  DANTE  DELA  CRUZ  

On  June  11,  1981,  the  petitioners  filed  a  motion  to  dismiss  the  complaint  on  the  ground  that  the  respondent,  
being  a  foreign  corporation  not  licensed  to  do  business  in  the  Philippines,  has  no  personality  to  maintain  the  
instant  suit.  On  June  11,  1981,  the  petitioners  filed  a  motion  to  dismiss  the  complaint  on  the  ground  that  the  
respondent,  being  a  foreign  corporation  not  licensed  to  do  business  in  the  Philippines,  has  no  personality  to  
maintain  the  instant  suit.  
 
On  June  14,  1982,  the  appellate  court  dismissed  the  petition  stating  that  the  respondent  judge  did  not  commit  
any  grave  abuse  of  discretion  in  deferring  the  petitioners'  motion  to  dismiss  because  the  said  judge  is  not  yet  
satisfied  that  he  has  the  necessary  facts  which  would  permit  him  to  make  a  judicious  resolution.  
 
ISSUE:  whether  or  not  respondent  has  no  personality  to  sue  as  they  were  not  registered  in  the  Philippines?    
 
HELD:   No.   In   the   case   at   bar,   the   transactions   entered   into   by   the   respondent   with   the   petitioners   are   not   a  
series  of  commercial  dealings  which  signify  an  intent  on  the  part  of  the  respondent  to  do  business  in  the  
Philippines  but  constitute  an  isolated  one  which  does  not  fall  under  the  category  of  "doing  business."  The  
records  show  that  the  only  reason  why  the  respondent  entered  into  the  second  and  third  transactions  with  
the  petitioners  was  because  it  wanted  to  recover  the  loss  it  sustained  from  the  failure  of  the  petitioners  to  
deliver  the  crude  coconut  oil  under  the  first  transaction  and  in  order  to  give  the  latter  a  chance  to  make  good  
on  their  obligation.  Instead  of  making  an  outright  demand  on  the  petitioners,  the  respondent  opted  to  try  to  
push  through  with  the  transaction  to  recover  the  amount  of  US$103,600.00  it  lost.  
 
From   these   facts   alone,   it   can   be   deduced   that   in   reality,   there   was   only   one   agreement   between   the  
petitioners  and  the  respondent  and  that  was  the  delivery  by  the  former  of  500  long  tons  of  crude  coconut  oil  
to   the   latter,   who   in   turn,   must   pay   the   corresponding   price   for   the   same.   The   three   seemingly   different  
transactions  were  entered  into  by  the  parties  only  in  an  effort  to  fulfill  the  basic  agreement  and  in  no  way  
indicate  an  intent  on  the  part  of  the  respondent  to  engage  in  a  continuity  of  transactions  with  petitioners  
which  will  categorize  it  as  a  foreign  corporation  doing  business  in  the  Philippines.  Thus,  the  trial  court,  and  
the  appellate  court  did  not  err  in  denying  the  petitioners'  motion  to  dismiss  not  only  because  the  ground  
thereof  does  not  appear  to  be  indubitable  but  because  the  respondent,  being  a  foreign  corporation  not  doing  
business  in  the  Philippines,  does  not  need  to  obtain  a  license  to  do  business  in  order  to  have  the  capacity  to  
sue.  
 
Also,  in  the  case  of  Top-­‐‑Weld  Manufacturing,  Inc.  v.  ECED,  S.A.,  the  Court  stated:  
 
There  is  no  general  rule  or  governing  principle  laid  down  as  to  what  constitutes  'doing'  or  'engaging  in'  or  
'transacting   business   in   the   Philippines.   Each   case   must   be   judged   in   the   Light   of   its   peculiar   circumstance.  
Thus,   a   foreign   corporation   with   a   settling   agent   in   the   Philippines   which   issues   twelve   marine   policies  
covering   different   shipments   to   the   Philippines   and   a   foreign   corporation   which   had   been   collecting  
premiums   on   outstanding   policies   were   regarded   as   doing   business   here.   The   acts   of   these   corporations  
should  be  distinguished  from  a  single  or  isolated  business  transaction  or  occasional,  incidental  and  casual  
transactions   which   do   not   come   within   the   meaning   of   the   law.   Where   a   single   act   or   transaction,   however,  
is  not  merely  incidental  or  casual  but  indicates  the  foreign  corporation's  intention  to  do  other  business  in  the  
Philippines,  said  single  act  or  transaction  constitutes  'doing'  or  'engaging  in'  or  'transacting'  business  in  the  
Philippines  
 
 
 
183.    CARGILL,  INC.,  VS.  INTRA  STRATA  ASSURANCE  CORPORATION,  
G.R.  NO.  168266   MARCH  15,  2010  
CARPIO,  J.:  

   
3H  A.Y.  2017-­‐2018   232  
 
 

CORPORATION  LAW  CASE  DIGESTS  –  ATTY.  DANTE  DELA  CRUZ  

 
DOCTRINE:  Under  Article  123  of  the  Corporation  Code,  a  foreign  corporation  must  first  obtain  a  license  and  
a  certificate  from  the  appropriate  government  agency  before  it  can  transact  business  in  the  Philippines  
 
To  be  doing  or  transacting  business  in  the  Philippines  for  purposes  of  Section  133  of  the  Corporation  Code,  
the  foreign  corporation  must  actually  transact  business  in  the  Philippines,  that  is,  perform  specific  business  
transactions  within  the  Philippine  territory  on  a  continuing  basis  in  its  own  name  and  for  its  own  account.  
Actual   transaction   of   business   within   the   Philippine   territory   is   an   essential   requisite   for   the   Philippines   to  
to   acquire   jurisdiction   over   a   foreign   corporation   and   thus   require   the   foreign   corporation   to   secure   a  
Philippine  business  license.  If  a  foreign  corporation  does  not  transact  such  kind  of  business  in  the  Philippines,  
even  if  it  exports  its  products  to  the  Philippines,  the  Philippines  has  no  jurisdiction  to  require  such  foreign  
corporation  to  secure  a  Philippine  business  license  
 
FACTS:  Petitioner  Cargill,  Inc.  (petitioner)  is  a  corporation  organized  and  existing  under  the  laws  of  the  State  
of  Delaware,  USA.  
 
 Petitioner   and   Northern   Mindanao   Corporation   (NMC)   executed   a   contract   dated     whereby   NMC   agreed   to  
sell   to   petitioner   metric   tons   of   molasses,   to   be   delivered   from   1   January   to   30   June   1990     The   contract  
provides  that  petitioner  would  open  a  Letter  of  Credit  with  BPI.    
   
The  contract  was  amended  three  times.  In  compliance  with  the  terms  of  the  third  amendment  of  the  contract,  
respondent   issued   performance   bond   to   guarantee   NMCs   delivery   of   the   10,500   tons   of   molasses,   and   a  
surety  bond  to  guarantee  the  repayment  of  down  payment  as  provided  in  the  contract.  
   
NMC   was   only   able   to   deliver   219.551   metric   tons   of   molasses   out   of   the   agreed   10,500   metric   tons.   ;Thus,  
petitioner  sent  demand  letters  to  respondent  claiming  payment  under  the  performance  and  surety  bonds.    
 
When  respondent  refused  to  pay,  petitioner  filed  a  complaint  for  sum  of  money  against  NMC  and  respondent.  
   
Petitioner,  NMC,  and  respondent  entered  into  a  compromise  agreement.  However,  NMC  still  failed  to  comply  
with  its  obligation  under  the  compromise  agreement.  Hence,  trial  proceeded  against  respondent.  
 
RTC:  judgment  is  rendered  in  favor  of  plaintiff  
CA:   reversed   the   trial   courts   decision.   That   petitioner   does   not   have   the   capacity   to   file   this   suit   since   it   is   a  
foreign   corporation   doing   business   in   the   Philippines   without   the   requisite   license.   ;   That   petitioners  
purchases   of   molasses   were   in   pursuance   of   its   basic   business   and   not   just   mere   isolated   and   incidental  
transactions.  
 
ISSUE:   whether   petitioner,   an   unlicensed   foreign   corporation,   has   legal   capacity   to   sue   before   Philippine  
courts  
 
HELD:   Under  Article  123  of  the  Corporation  Code,  a  foreign  corporation   must   first   obtain   a   license   and   a  
certificate  from  the  appropriate  government  agency  before  it  can  transact  business  in  the  Philippines.  Where  
a  foreign  corporation  does  not  do  business  in  the  Philippines  without  the  proper  license,  it  cannot  maintain  
any  action  or  proceeding  before  Philippine  courts  as  provided  under  Section  133  of  the  Corporation  Code.    
 
Thus,  the  threshold  question  in  this  case  is  whether  petitioner  was  doing  business  in  the  Philippines.  The  
Corporation   Code   provides   no   definition   for   the   phrase   doing   business.   Nevertheless,   Section   1   of   RA   5455  
provides  that:  
   

   
3H  A.Y.  2017-­‐2018   233  
 
 

CORPORATION  LAW  CASE  DIGESTS  –  ATTY.  DANTE  DELA  CRUZ  

The   phrase   doing   business   shall   include   soliciting   orders,   purchases,   service   contracts,   opening   offices,  
whether   called   liaison   offices   or   branches;   appointing   representatives   or   distributors   who   are   domiciled   in  
the   Philippines   or   who   in   any   calendar   year   stay   in   the   Philippines   for   a   period   or   periods   totalling   one  
hundred   eighty   days   or   more;   participating   in   the   management,   supervision   or   control   of   any   domestic  
business  firm,  entity  or  corporation  in  the  Philippines;  and  any  other  act  or  acts  that  imply  a  continuity  of  
commercial   dealings   or   arrangements,   and   contemplate   to   that   extent   the   performance   of   acts   or   works,   or  
the  exercise  of  some  of  the  functions  normally  incident  to,  and  in  progressive  prosecution  of,  commercial  
gain  or  of  the  purpose  and  object  of  the  business  organization.      
 
RA  7042,  otherwise  known  as  the  Foreign  Investments  Act  of  199.  enumerated  not  only  the  acts  or  activities  
which  constitute  doing  business  but  also  those  activities  which  are  not  deemed  doing  business.  
1.     Mere   investment   as   a   shareholder   by   a   foreign   entity   in   domestic   corporations   duly   registered   to   do  
business,  and/or  the  exercise  of  rights  as  such  investor;  
2.    Having  a  nominee  director  or  officer  to  represent  its  interests  in  such  corporation;  
3.    Appointing  a  representative  or  distributor  domiciled  in  the  Philippines  which  transacts  business  in  the  
representative's  or  distributor's  own  name  and  account;  
4.    The  publication  of  a  general  advertisement  through  any  print  or  broadcast  media;  
5.    Maintaining  a  stock  of  goods  in  the  Philippines  solely  for  the  purpose  of  having  the  same  processed  by  
another  entity  in  the  Philippines;  
6.     Consignment   by   a   foreign   entity   of   equipment   with   a   local   company   to   be   used   in   the   processing   of  
products  for  export;  
7.    Collecting  information  in  the  Philippines;  and  
8.    Performing  services  auxiliary  to  an  existing  isolated  contract  of  sale  which  are  not  on  a  continuing  basis,  
such  as  installing  in  the  Philippines  machinery  it  has  manufactured  or  exported  to  the  Philippines,  servicing  
the  same,  training  domestic  workers  to  operate  it,  and  similar  incidental  services.  
 
In  this  case,  we  find  that  respondent  failed  to  prove  that  petitioners  activities  in  the  Philippines  constitute  
doing  business  as  would  prevent  it  from  bringing  an  action.  
 
In  this  case,  the  contract  between  petitioner  and  NMC  involved  the  purchase  of  molasses  by  petitioner  from  
NMC.  It  was  NMC,  the  domestic  corporation,  which  derived  income  from  the  transaction  and  not  petitioner.  
To  constitute  doing  business,  the  activity  undertaken  in  the  Philippines  should  involve  profit-­‐‑making.    
   
Other   factors   which   support   the   finding   that   petitioner   is   not   doing   business   in   the   Philippines   are:   (1)  
petitioner  does  not  have  an  office  in  the  Philippines;  (2)  petitioner  imports  products  from  the  Philippines  
through   its   non-­‐‑exclusive   local   broker,   whose   authority   to   act   on   behalf   of   petitioner   is   limited   to   soliciting  
purchases  of  products  from  suppliers  engaged  in  the  sugar  trade  in  the  Philippines;  and  (3)  the  local  broker  
is  an  independent  contractor  and  not  an  agent  of  petitioner  
 
As  explained  by  the  Court  in  B.  Van  Zuiden  Bros.,  Ltd.  v.  GTVL  Marketing  Industries,  Inc.:  
 
   
To  be  doing  or  transacting  business  in  the  Philippines  for  purposes  of  Section  133  of  the  Corporation  Code,  
the  foreign  corporation  must  actually  transact  business  in  the  Philippines,  that  is,  perform  specific  business  
transactions  within  the  Philippine  territory  on  a  continuing  basis  in  its  own  name  and  for  its  own  account.  
Actual   transaction   of   business   within   the   Philippine   territory   is   an   essential   requisite   for   the   Philippines   to  
to   acquire   jurisdiction   over   a   foreign   corporation   and   thus   require   the   foreign   corporation   to   secure   a  
Philippine  business  license.  If  a  foreign  corporation  does  not  transact  such  kind  of  business  in  the  Philippines,  
even  if  it  exports  its  products  to  the  Philippines,  the  Philippines  has  no  jurisdiction  to  require  such  foreign  
corporation  to  secure  a  Philippine  business  license.[23]  (Emphasis  supplied)  

   
3H  A.Y.  2017-­‐2018   234  
 
 

CORPORATION  LAW  CASE  DIGESTS  –  ATTY.  DANTE  DELA  CRUZ  

   
In  the  present  case,  petitioner  is  a  foreign  company  merely  importing  molasses  from  a  Philipine  exporter.  A  
foreign   company   that   merely   imports   goods   from   a   Philippine   exporter,   without   opening   an   office   or  
appointing  an  agent  in  the  Philippines,  is  not  doing  business  in  the  Philippines.  
 
   
 
 
184.  REV.  JORGE  BARLIN  V.  P.  VICENTE  RAMIREZ  
GR  NO.  L-­‐‑2832  24  NOVEMBER  1906  
JUSTICE  WILLARD  
 
DOCTRINE:  
  The  Roman  Catholic  Church  is  a  juridical  entity  capable  of  owning  property  in  the  Philippines  its  own  
name.  The  Church,  like  any  other  juridical  entity,  depends  its  existence  and  the  exercise  of  its  rights  under  
the  laws  of  the  Roman  Empire.    
 
FACTS:  
  Defendant   Ramirez,   having   been   appointed   by   the   parish   priest   of   Lagonoy,   Camarines,   took  
possession   of   the   church   in   1901.   The   parish   priest’s   successor,   Rev.   Barlin   later   demanded   Ramirez   to  
deliver   to   him   church,   convent   and   cemetery   and   the   sacred   ornaments,   books,   jewels,   money   and   other  
property  of  the  church.  However,  Ramirez  wrote  to  him  and  refused  to  make  such  delivery  stating  therein  
that   the   town   of   Lagonoy   has   decided   to   sever   connection   with   the   Pope   of   Rome   and   instead   opted   to   join  
the  Filipino  church,  with  the  consent  of  the  parish  priest  and  the  resolution  of  the  people.  
 
  Thus,   in   January   1904,   Rev.   Barlin   brought   this   action   against   Ramirez   alleging   that   the   Roman  
Catholic   Church   Is   the   owner   of   the   church   building,   convent,   cemetery,   books,   etc.   and   asking   that   it   be  
restored  to  the  possession  thereof.  The  latter  however  alleged  that  the  town  of  Lagonoy  was  the  owner  of  
the  property.    
 
ISSUE:  
  Whether  or  not  the  church  and  all  its  appurtenances  are  property  of  the  Roman  Catholic  Church  
 
RULING:  
  Yes,   the   church   was   the   property   of   the   Roman   Catholic   Church.   A   former   law   stated   that   all   church  
buildings  were  made  by  the  Spanish  government  and  representatives  in  the  Philippines  using  government  
and   private   local   funds.   Hence,   its   properties   were   beyond   the   commerce   of   man.   Priests   merely   held   them  
as  guardians  or  stewards.  Furthermore,  the  municipality  cannot  show  evidence  of  title,  right  of  ownership  
or  possession.    
 
  It  was  further  alleged  that  the  Roman  Catholic  Church  had  no  legal  personality  in  the  Philippines.  
However,  the  Preamble  to  Ecclesiastical  obligations,  presented  by  Montero  Rios  said,  that  the  Church  was  
persecuted   as   an   unlawful   association   until   the   existence   of   Galieno   who   admitted   that   the   Church   was  
among  the  juridical  entities  protected  by  the  laws  of  the  Roman  Empire.  Later,  the  church  was  entered  upon  
the   exercise   of   such   rights   as   were   required   for   the   acquisition,   preservation,   and   transmission   of   property  
the  same  as  any  other  juridical  entity  under  the  laws  of  the  Empire.    
 
 
 

   
3H  A.Y.  2017-­‐2018   235  
 
 

CORPORATION  LAW  CASE  DIGESTS  –  ATTY.  DANTE  DELA  CRUZ  

185.  GERTUBOS  VS  DE  LEON,  PNRC  


G.R.  NO.  199440  
PERALTA,  J.  
 
DOCTRINE:  The  PNRC,  as  a  National  Society  of  the  International  Red  Cross  and  Red  Crescent  Movement,  can  
neither  "be  classified  as  an  instrumentality  of  the  State,  so  as  not  to  lose  its  character  of  neutrality"  as  well  as  
its  independence,  nor  strictly  as  a  private  corporation  since  it  is  regulated  by  international  humanitarian  law  
and  is  treated  as  an  auxiliary  of  the  State.  
FACTS:  
Respondent   Corazon   Alma   G.   De   Leon,   Secretary   General   of   the   Philippine   National   Red   Cross,   charged  
petitioner  Mary  Lou  Geturbos  Torres,  the  Chapter  Administrator  of  the  Philippine  National  Red  Cross  (PNRC)  
of  General  Santos  City,  with  Grave  Misconduct  for  violating  Philippine  National  Red  Cross  (PNRC)  Financial  
Policies   on   Oversubscription,   Remittances   and   Disbursement   of   Funds.   Petitioner   was   subjected   to   one  
month  suspension  effective  July  1-­‐‑31,  2007  and  transferred  to  the  National  Headquarters  effective  August  1,  
2007.  
Petitioner  appealed  to  Board  of  Governors  of  the  PNRC  through  respondent  and  furnished  a  copy  thereof  to  
the  Civil  Service  Commission  (CSC).  The  CSC,  on  April  21,  2008,  dismissed  petitioner's  appeal  and  imposing  
upon  her  the  penalty  of  dismissal  from  service.  Petitioner  filed  a  motion  for  reconsideration  with  the  CSC,  
but  the  same  was  denied.  
Petitioner  questions  the  jurisdiction  of  CSC  in  dismissing  the  case.  According  to  petitioner,  this  Court  has  
decided  that  PNRC  is  not  a  government-­‐‑owned  and  controlled  corporation  (GOCC),  hence,  the  CSC  has  no  
jurisdiction   or   authority   to   review   the   appeal   that   she   herself   filed.   As   such,   she   insists   that   the   CSC  
committed  grave  abuse  of  discretion  in  modifying  the  decision  of  respondent  De  Leon.  
 
ISSUE:    Whether  or  not  the  CSC  has  appellate  jurisdiction  over  the  case  
 
HELD:  YES.  
The  sui  generis  character  of  PNRC  requires  us  to  approach  controversies  involving  the  PNRC  on  a  case-­‐‑to-­‐‑
case  basis.  The  PNRC,  as  a  National  Society  of  the  International  Red  Cross  and  Red  Crescent  Movement,  can  
neither  "be  classified  as  an  instrumentality  of  the  State,  so  as  not  to  lose  its  character  of  neutrality"  as  well  as  
its  independence,  nor  strictly  as  a  private  corporation  since  it  is  regulated  by  international  humanitarian  law  
and  is  treated  as  an  auxiliary  of  the  State.  
In  this  particular  case,  the  CA  did  not  err  in  ruling  that  the  CSC  has  jurisdiction  over  the  PNRC  because  the  
issue  at  hand  is  the  enforcement  of  labor  laws  and  penal  statutes,  thus,  in  this  particular  matter,  the  PNRC  
can  be  treated  as  a  GOCC,  and  as  such,  it  is  within  the  ambit  of  Rule  I,  Section  1  of  the  Implementing  Rules  of  
Republic  Act  67135,  stating  that:  
Section   1.   These   Rules   shall   cover   all   officials   and   employees   in   the   government,   elective   and   appointive,  
permanent   or   temporary,   whether   in   the   career   or   non-­‐‑career   service,   including   military   and   police  
personnel,  whether  or  not  they  receive  compensation,  regardless  of  amount.  
Thus,  having  jurisdiction  over  the  PNRC,  the  CSC  had  authority  to  modify  the  penalty  and  order  the  dismissal  
of   petitioner  from   the   service.   Under  the  Administrative   Code  of   1987,6   as  well  as  decisions7  of   this   Court,  
the  CSC  has  appellate  jurisdiction  on  administrative  disciplinary  cases  involving  the  imposition  of  a  penalty  
of  suspension  for  more  than  thirty  (30)  days,  or  fine  in  an  amount  exceeding  thirty  (30)  days  salary.    
 
 

   
3H  A.Y.  2017-­‐2018   236  
 

You might also like